You are on page 1of 614

Pour débuter en géométrie : chasse aux angles et

éléments de géométrie du triangle


Cécile Gachet

Ce document, rédigé à partir de cours donnés lors de stages olympiques ou


d’autres événements mathématiques destinés à des collégiens et lycéens, a pour
but de vous familiariser avec les résultats de base en géométrie.

Table des matières


1 Quelques conseils pour rédiger un problème de géométrie 2

2 Droites remarquables dans un triangle 3


2.1 Médiatrices . . . . . . . . . . . . . . . . . . . . . . . . . . . . . . 3
2.2 Hauteurs . . . . . . . . . . . . . . . . . . . . . . . . . . . . . . . 3
2.3 Médianes . . . . . . . . . . . . . . . . . . . . . . . . . . . . . . . 3
2.4 Bissectrices . . . . . . . . . . . . . . . . . . . . . . . . . . . . . . 3

3 Éléments de  chasse aux angles” 4


3.1 Notions d’angle géométrique et d’angle orienté . . . . . . . . . . 4
3.2 Premières figures de style . . . . . . . . . . . . . . . . . . . . . . 5
3.3 Théorèmes de l’angle au centre et de l’angle inscrit . . . . . . . . 5

4 Points remarquables dans un triangle 10


4.1 Centre du cercle circonscrit . . . . . . . . . . . . . . . . . . . . . 10
4.2 Orthocentre . . . . . . . . . . . . . . . . . . . . . . . . . . . . . . 10
4.3 Centre de gravité . . . . . . . . . . . . . . . . . . . . . . . . . . . 11
4.4 Centre du cercle inscrit . . . . . . . . . . . . . . . . . . . . . . . 13

5 Triangles semblables 14

6 Solutions des exercices 16

1
1 Quelques conseils pour rédiger un problème
de géométrie
Résoudre un problème de géométrie est une vaste entreprise : il y a tant
de départs possibles ! Voici quelques conseils de méthode pour bien débuter, et
tirer le meilleur profit de ce que vous aurez appris dans la pratique :
• commencez par lire l’énoncé en faisant une grande figure à main levée,
dans le cas général : s’il est question d’un triangle ABC quelconque, il
faut montrer le résultat quel que soit le triangle. Il ne faut donc pas le
tracer isocèle, ni rectangle...
• rajoutez sur votre figure les informations qui vous semblent importantes
(si on mentionne le centre du cercle circonscrit, tracez ce cercle : peut-
être voyez-vous des points cocycliques ? ; s’il est question du cercle inscrit,
pensez aux bissectrices du triangle, rajoutez ses points de tangence aux
côtés du triangle ;...)
• si nécessaire, n’hésitez pas à faire une figure propre pour voir ce qui se
passe : si vous faites des conjectures intéressantes, vous aurez ensuite une
meilleure idée des étapes intermédiaires de votre démonstration.
• au moment de rédiger, pensez à votre lecteur : faites une figure lisible,
et reportez-y les points et les angles (en couleur si possible) que vous
avez introduit (donnez leur des noms !). Ne recopiez pas l’énoncé, mais
définissez bien en une phrase les objets que vous introduisez ( soit C
le cercle de diamètre [AB]” ;  soit M le projeté orthogonal de P sur la
bissectrice de A”...).
b
• annoncez les étapes de votre raisonnement si elles ne s’enchaı̂nent pas
immédiatement ( on commence par une chasse aux angles” ;  montrons
maintenant que les triangles ABC et M OH sont semblables”...)
• enfin, n’oubliez pas que c’est à force d’entraı̂nement et d’exercices qu’on
apprend à résoudre des problèmes de plus en plus complexes !

2
2 Droites remarquables dans un triangle
2.1 Médiatrices
Définition 2.1 La médiatrice d’un segment [AB] est la perpendiculaire à (AB)
passant par le milieu de [AB].

Proposition 2.2 Un point appartient à la médiatrice d’un segment si et seule-


ment si il est équidistant des deux extrémités de ce segment.

En d’autres termes, si un point appartient à la médiatrice d’un segment, alors


il est équidistant des deux extrémités de ce segment, et réciproquement ( dans
l’autre sens”), si un point est équidistant des deux extrémités d’un segment,
alors il appartient la médiatrice de ce segment.

2.2 Hauteurs
Définition 2.3 Le projeté orthogonal d’un point A sur une droite (d) est le
point d’intersection de d et de la perpendiculaire à (d) passant par A.

Définition 2.4 Dans un triangle ABC, la hauteur issue de A est la perpendi-


culaire à (BC) passant par A.

2.3 Médianes
Définition 2.5 Dans un triangle ABC, la médiane issue de A est la droite qui
relie A et le milieu de [BC].

2.4 Bissectrices
Définition 2.6 La bissectrice d’un angle est la demi-droite qui le partage en
deux angles adjacents de même mesure.

Définition 2.7 La distance d’un point A à une droite (d) est la plus petite
distance entre A et un point D de la droite (d).

Proposition 2.8 Soit A un point, (d) une droite, H le projeté orthogonal de


A sur (d). La distance de A à (d) vaut AH.

Exercice 2.9 Montrer la proposition 2.8.

Proposition 2.10 Un point est sur la bissectrice d’un angle si et seulement si


il est équidistant des deux côtés de l’angle.

3
3 Éléments de  chasse aux angles”
3.1 Notions d’angle géométrique et d’angle orienté
Ce paragraphe donne quelques définitions et notations utilisées par la suite :
vous pouvez le négliger en première lecture en restant dans une approche intui-
tive, quitte à y revenir dès que vous en avez besoin.
Définition 3.1 Un angle est une portion de plan délimitée par deux demi-
droites issues du même point.
Ce point est le sommet de l’angle, et les demi-droites en sont les côtés.
Deux demi-droites [AB) et [AC) déterminent ainsi deux angles. Celui qui est
entièrement contenu dans un demi-plan est appelé angle saillant (i), et l’autre
est appelé angle rentrant (ii).

(ii)

(i)

En général, l’angle délimité par [AB) et [AC) est noté BAC,


d mais on peut
rencontrer la notation A, s’il n’y a pas d’ambiguı̈té.
b
Il arrive de donner des noms aux angles qui reviennent souvent dans une
figure. On utilise alors des lettres latines comme x, y, z, mais aussi des lettres
grecques comme α (qui se lit  alpha”), β (qui se lit  bêta”), γ (qui se lit
 gamma”), ϕ (qui se lit  phi”).

Enfin, on pourra rencontrer des angles orientés de droites. Cette notation


n’est pas obligatoire, mais vivement conseillée car elle allège souvent la rédaction.
L’angle orienté entre deux droites est une notion qui généralise l’angle géométrique
(avec un chapeau). L’angle orienté entre d1 et d2 se note (d1 , d2 ) et correspond
à la mesure de l’angle dont il faut tourner la droite d1 pour la rendre parallèle
(éventuellement confondue) à la droite d2 , à 180˚ près, et en tournant dans le
sens inverse des aiguilles d’une montre.
Il faut connaı̂tre quelques règles de calculs sur ces angles : tout d’abord, la
convention suivante : si le sens inverse des aiguilles d’une montre est  posi-
tif” (c’est ce qu’on a dit au paragraphe précédent), le sens des aiguilles d’une
montre est, quant à lui, négatif. Autrement dit, pour toutes droites d1 , d2 , on
a : (d1 , d2 ) = −(d2 , d1 ).
Il faut aussi mentionner une propriété très importante, bien qu’assez in-
tuitive : la relation de Chasles : pour toutes droites d, d1 , d2 , on a (d1 , d2 ) =
(d1 , d) + (d, d2 ). Cela nous permet de composer/décomposer des angles comme
bon nous semble, ce qui est très utile !

4
3.2 Premières figures de style
Il est nécessaire, dans l’apprentissage de la géométrie, de se rendre fami-
lier avec un certain nombre de configurations revenant sans cesse dans les
problèmes ou exercices. Pour acquérir ces automatismes, il est conseillé au lec-
teur de regarder régulièrement (du moins au début) les figures suivantes, qui
illustrent quelques résultats de géométrie appris au collège absolument cruciaux,
exprimés sous une forme peut-être inhabituelle, qu’il faut toutefois apprendre à
reconnaı̂tre ainsi.

ϕ
ϕ

ϕ
ϕ

“Chaque fois que je vois des droites parallèles dans un problème d’angles, je
pense aux angles correspondants et alternes-internes sur toutes les sécantes !”

α+β
α

“Chaque fois que je veux calculer l’angle entre deux droites, je pense à la
relation de Chasles !”
Remarque. Une autre manière de voir les choses est de dire : “chaque fois que
je vois un triangle, je pense que la somme de ses angles vaut 180˚!”

le cercle de centre M le triangle ABC


AM = BM = CM passe par A, B, C est rectangle en C
C C C

A M B A M B A M B

“Chaque fois que je vois des égalités de longueurs qui font penser au diamètre
d’un cercle, je pense à un triangle rectangle !”

3.3 Théorèmes de l’angle au centre et de l’angle inscrit


Soit C un cercle de centre O, et A, B, M, N, M 0 quatre points de ce cercle,
avec M, N, O du même côté de (AB), et M 0 de l’autre côté de (AB).

5
M0

B
A

C
O

Définition 3.2 Les angles AM


dB et AN dB sont appelés angles inscrits dans C.
L’angle AOB est appelé angle au centre dans C.
d
Les angles AM
dB, ANdB et AOBd interceptent le même arc du cercle C.

Théorème de l’angle au centre. On a :

AOB
d = 2AM
dB

Démonstration. On peut décomposer l’angle :

2AM
dB = 2AM
dO + 2OM
dB.

Or les triangles AM O et OM B sont isocèles en O. Donc :

2AM
dO + 2OM
dB = AM
dO + Md
AO + OM
dB + OBM
d .

Comme la somme des angles dans un triangle vaut 180˚, on en déduit que :

AM
dO + Md
AO + OM
dB + OBM
d = 180˚− AOM
d + 180˚− BOM
d = AOB
d

(attention au passage entre les angles saillants et rentrants).


En fin de compte, on a donc bien : 2AM dB = AOB. d 2
On en déduit que :

dB = 1 AOB
AN d = AM
dB,
2
et que :
AMd0 B = 1 (360˚− AOB)d = 180˚− AM dB
2
(car un angle rentrant vaut 360˚moins l’angle saillant correspondant). Ceci nous
amène à énoncer le théorème suivant :

Théorème de l’angle inscrit. Soit Z un point du plan quelconque.


Le point Z est sur le cercle C si et seulement si l’une de ces conditions est
satisfaite :
(i) Z et M sont du même côté de (AB) et AZB
d = AM dB ;

6
(ii) Z et M sont de part et d’autre de (AB) et AZB
d = 180˚− AM
dB.

Définition 3.3 Quatre points A, B, M, N appartenant à un même cercle sont


dits cocycliques.

Ainsi, le théorème de l’angle inscrit permet non seulement de montrer que


quatre points sont cocycliques en connaissant des égalités d’angles, mais aussi
de montrer que des angles sont égaux (ou supplémentaires, ou...) en sachant que
des points sont cocycliques.
En montrant que des angles sont égaux ou supplémentaires, on peut donc
trouver des points cocycliques, et déduire du théorème de l’angle inscrit d’autres
relations entre les angles de ces points, qui nous permettent de trouver encore
plus de points cocycliques, donc encore plus d’angles...
De cette manière, on comprend de mieux en mieux ce qui se passe dans la
figure, et on avance dans la résolution du problème... C’est ce qu’on appelle la
“ chasse aux angles”, et qui est une méthode très utile en géométrie.

Proposition 3.4 Un cas particulier très fréquent du théorème de l’angle inscrit


est le suivant : soit A, B, M trois points du plan. Le point M est sur le cercle
de diamètre [AB] si et seulement si l’angle AM
dB est droit.
En particulier, si A, B, M, N sont quatre points du plan, tels que AM dB est
un angle droit, alors ces quatre points sont cocycliques si et seulement si ANdB
est un angle droit, et ce indépendamment de la position relative de M, N et
(AB).

N
M

A O B

M0

Définition 3.5 La tangente (d) à un cercle C de centre O en un point P est la


droite perpendiculaire à (OP ) passant par P .
On dit alors que la droite (d) et le cercle C sont tangents.

Proposition 3.6 Un autre cas particulier très utile : soit C un cercle, A, B, M, N


quatre points de ce cercle tels que M et N sont du même côté de (AB). On a
AM
dB = AN dB. Rapprochant maintenant le point M de A, en restant sur le
cercle. La droite (AM ) se rapproche de la tangente (t) à C en A, et la droite
(M B) se rapproche de la droite (AB), tandis que le théorème de l’angle inscrit

7
reste toujours vrai, autrement dit l’angle entre les droites (AM ) et (M B) reste
égal à AN
dB.
Intuitivement, on peut donc comprendre que le théorème de l’angle inscrit
reste vrai dans le cas limite où (AM ) = (t) et (M B) = (AB). Donc l’angle
entre (t) et (AB) est égal à AN
dB.

Voici une figure pour récapituler ce que nous dit ce magnifique théorème :

C M0

180˚− ϕ B
ϕ
A

ϕ
N
O

Exercice 3.7 (Droites parallèles et antiparallèles.) Soient C1 , C2 deux cercles


ayant deux points d’intersection A et B. Soient dA une droite passant par A et
dB une droite passant par B. On note C et E les points d’intersection de dA
avec C1 et C2 respectivement, et on définit de même D et F comme les points
d’intersection de dB avec C1 et C2 respectivement.
Montrer que les droites CD et EF sont parallèles.

Exercice 3.8 (Premier théorème de Miquel.) Soient C1 , C2 deux cercles de


centres respectifs O1 et O2 s’intersectant en X et Y . Soit A un point de C1 et
B l’intersection de AY et C2 .
Montrer que XO1 O2 et XAB sont semblables.

Exercice 3.9 (Second théorème de Miquel.) Soit ABC un triangle, P un


point de BC, Q un point de CA, R un point de AB. Les cercles circonscrits à
AQR et à BRP ont pour second point d’intersection X.
Montrer que X est aussi sur le cercle circonscrit à CP Q.

8
Exercice 3.10 (Droite de Simpson.) Soient ABC un triangle, P un point et
A0 , B 0 , C 0 ses projetés orthogonaux sur les côtés (BC), (CA), (AB) du triangle.
Montrer que A0 , B 0 , C 0 sont alignés si et seulement si P est sur le cercle
circonscrit à ABC.

Exercice 3.11 Soit C un cercle et BC une corde de ce cercle. Soit A le milieu


de l’arc BC. On considère deux cordes de C passant par A, notons-les AD et
AE, et F et G les points d’intersection respectifs de ces cordes avec BC.
Montrer que les points D, E, F, G sont cocycliques.

Exercice 3.12 Soient C1 , C2 deux cercles ayant deux points d’intersection P et


Q. Soit d une droite coupant Γ1 en A et C et C2 en B et D, les points étant
disposés dans l’ordre A, B, C, D sur la droite.
Montrer que AP dB = CQD.
d

Exercice 3.13 (Point de Miquel.) Soient A, A0 , B, B 0 quatre points, C le


point d’intersection de (AA0 ) et (BB 0 ) et D le point d’intersection de (AB) et
(A0 B 0 ).
Montrer que les cercles circonscrits à CA0 B 0 , CAB, DAA0 , DBB 0 sont concou-
rants.

L’exercice suivant est très important : en effet, de nombreux exercices de


chasse aux angles s’avèrent en être des conséquences directes.

Exercice 3.14 (Théorème du cube.) Soient A, B, C, D, A0 , B 0 , C 0 , D0 des


points tels que A, B, C, D cocycliques, A, A0 , B, B 0 cocycliques, B, B 0 , C, C 0 co-
cycliques, C, C 0 , D, D0 cocycliques, D, D0 , A, A0 cocycliques.
Montrer que A0 , B 0 , C 0 , D0 sont cocycliques.

Exercice 3.15 Soient A, B, C, D quatre points sur un cercle C. On note A0 et


C 0 les projetés orthogonaux respectifs de A et de C sur BD, et B 0 et D0 les
projetés orthogonaux respectifs de B et de D sur AC.
Montrer que les points A0 , B 0 , C 0 , D0 sont cocycliques.

9
4 Points remarquables dans un triangle
4.1 Centre du cercle circonscrit
Théorème 4.1 Dans un triangle, les médiatrices des trois côtés sont concou-
rantes en un point équidistant des trois sommets du triangle.
Démonstration. Soit ABC un triangle. Les médiatrices de [AB] et [BC] ne sont pas
parallèles (sinon (AB) et (BC) seraient perpendiculaires à une même droite, or (AB)
et (BC) ne sont pas parallèles).
Ces deux médiatrices ont donc un point d’intersection O. D’après la proposition 2.2,
on a donc OA = OB = OC. Donc O est équidistant de A et C, donc, réciproquement,
d’après la proposition 2.2 O appartient aussi à la médiatrice de [AC].
Donc les médiatrices de ABC sont concourantes en O, et OA = OB = OC. 2
Remarque. Comme O est équidistant des trois sommets du triangles, il existe
un cercle de centre O qui passe par ces trois sommets.

Définition 4.2 Le cercle passant par les sommets d’un triangle est appelé le
cercle circonscrit au triangle. Son centre, souvent noté O, est le point de concours
des médiatrices des côtés du triangle.

B
C

4.2 Orthocentre
Théorème 4.3 Dans un triangle, les trois hauteurs sont concourantes.
Démonstration. Soit ABC un triangle. Soit DEF le triangle obtenu en prenant les
parallèles à (BC), à (CA) et à (AB) passant par A, B, C respectivement.

10
F A
E

B
C

Par construction, ACBF est un parallélogramme. Donc AC = BF . De même, on


a AC = BD (en regardant le parallélogramme ABCD. Donc BF = BD, donc B est
le milieu de [DF ]. De même, A et C sont les milieux respectifs de [F E] et [ED].
Donc les hauteurs dans ABC sont les médiatrices dans DEF . Or, d’après le
théorème 4.1, les médiatrices dans DEF sont concourantes.
Donc les hauteurs de ABC sont concourantes. 2

Définition 4.4 Dans un triangle, le point de concours des hauteurs, souvent


noté H, est appelé l’orthocentre du triangle.

Exercice 4.5 Soit ABC un triangle, A0 , B 0 , C 0 les pieds des hauteurs issues de
A, B, C respectivement, H l’orthocentre.
Tracer tous les cercles passant par plus de quatre points parmi ceux nommés
ci-dessus.

Exercice 4.6 Soient ABC un triangle dont les trois angles sont aigus, H son
orthocentre et A0 , B 0 , C 0 les pieds des hauteurs issues de A, B et C.
Calculer les angles de A0 B 0 C 0 en fonction de ceux de ABC (qu’on notera α,
β et γ).
Montrer que H est le centre du cercle inscrit à A0 B 0 C 0 (la définition du
centre du cercle inscrit se trouve quelques pages plus loin).

Exercice 4.7 Soient ABC un triangle, O le centre de son cercle circonscrit et


H son orthocentre.
Montrer que BAO
d = CAH.
d

Exercice 4.8 (Symétriques de l’orthocentre.) Soit ABC un triangle d’or-


thocentre H.
Montrer que les symétriques de H par rapport à (AB), (BC), (CA) sont sur
le cercle circonscrit à ABC.

Exercice 4.9 Soient ABC un triangle, O le centre de son cercle circonscrit,


H son orthocentre. Dans ce triangle, on note D le pied de la hauteur issue de
A et E le pied de la hauteur issue de C.
Montrer que O est sur la bissectrice intérieure commune aux angles DHC
d
et AHE.
d

11
4.3 Centre de gravité
Théorème 4.10 Soit un triangle ABC, A0 , B 0 et C 0 les milieux respectifs de
[BC], [CA], [AB]. Les propriétés suivantes sont vérifiées :
(i) les médianes sont concourantes en un point G ;
(ii) ce point partage les médianes aux deux tiers : AG = 32 AD ;
(iii) la figure formée par les trois médianes partage le triangle en six petits
triangles d’aires égales.
Démonstration. Soit G le point d’intersection de (BB 0 ) et (CC 0 ).

C0
B0
G

N
M
B
A0 C

Soit M et N les milieux de [BG] et [CG].


D’après le théorème de la droite des milieux dans ABC, le segment [B 0 C 0 ] est
parallèle à [BC] et moitié moins grand. On montre la même propriété pour le segment
[M N ] en se plaçant dans BCG.
Donc M N B 0 C 0 est un parallélogramme. Ainsi GM = GB 0 , d’où BG = 23 BB 0 . On
établit de même que CG = 23 CC 0 .
Par symétrie des rôles de A, B, C, on montre de même que le point G0 d’intersection
de (AA0 ) et (CC 0 ) est aux deux-tiers des deux médianes, d’où G = G0 .
Donc les trois médianes se coupent toutes trois aux deux-tiers de leur longueur en
G.

F
E
G

B
D C

Enfin, comme GA0 = 13 AA0 , d’après le théorème de Thalès, la longueur de la


hauteur de GBC issue de G est égale au tiers de la longueur de la hauteur de ABC
issue de A (puisqu’elles sont toutes deux perpendiculaires à (BC), elles sont parallèles
entre elles).

12
Ainsi, comme BA0 = 12 BC, compte tenu de la formule :

1
aire = base × hauteur,
2
on obtient que l’aire de GBD vaut 61 de l’aire de ABC.
Le raisonnement est le même pour les cinq petits triangles restants. 2

Définition 4.11 Le point de concours des médianes d’un triangle, souvent noté
G, est appelé le centre de gravité du triangle.

4.4 Centre du cercle inscrit


Théorème 4.12 Dans un triangle ABC, les bissectrices des trois angles A, b B
b
et C sont concourantes en un point I. Ce point I est équidistant de (AB), (BC)
b
et (CA).
Démonstration. Soit ABC un triangle. Soit I le point d’intersection des bissectrices
des angles B b et C.
b
D’après la proposition 2.10, I est équidistant de (BC) et de (BA) d’une part, de
(CB) et de (CA) d’autre part. Donc I est aussi équidistant de (AC) et (AB). Donc,
d’après la proposition 2.10, I appartient à la bissectrice de l’angle A.
b
Les trois bissectrices sont donc concourantes en I. 2
Remarque. Comme I est équidistant de (AB), (BC) et (CA), il existe un cercle
de centre I passant par les projetés orthogonaux de I sur (AB), (BC) et (CA).
Ce cercle est donc tangent au trois côtés du triangle.

Définition 4.13 L’unique cercle tangent aux trois côtés d’un triangle est ap-
pelé le cercle inscrit de ce triangle. Son centre, souvent noté I, est le point de
concours des bissectrices du triangle.

Proposition 4.14 Soit un cercle C de centre O, A et B deux points sur ce


cercle tels que les tangentes à C en A et en B se coupent en Z. Alors ZA = ZB.
A

B0

C0

A0
C

13
5 Triangles semblables
Dans le langage courant, on dit que deux figures sont semblables lorsqu’elles
ont la même “ forme”. Autrement dit, deux figures semblables sont identiques,
à leur position sur la feuille et à leur taille près.
Par exemple, les trois bonshommes ci-dessous sont semblables :

Pour des triangles, le fait d’être semblable est une propriété très intéressante,
parce qu’elle relie des égalités d’angles et des égalités de rapports de longueurs !
En effet, dire que deux triangles ont la même “ forme”, c’est dire qu’ils ont les
mêmes angles, mais c’est aussi dire que les longueurs de leurs côtés respectifs
sont proportionnels !
Ainsi, on donne une proposition rassemblant les définitions équivalentes de
triangles semblables :

Proposition 5.1 Les quatre conditions suivantes sont deux à deux équivalentes :
• les triangles ABC et A0 B 0 C 0 sont semblables (ce qui est noté ABC ∼
A0 B 0 C 0 ),
• Ab=A c0 , B b=B c0 , et Cb=C c0 ,
A0 B 0 B0 C 0
• AB
AC = A0 C 0 et BC
BA = B 0 A0 ,
0 AB A0 B 0
• A = A et AC = A0 C 0 .
b c

Remarque. Attention, les longueurs dont on prend les rapports dans la troisième
condition doivent être celles des segments adjacents aux angles égaux choisis.

A
A00
A0
C 0 B 00

C
B C 00

B0
Exemple 5.2
Dans cet exemple, ABC, A0 B 0 C 0 et A00 B 00 C 00 sont semblables.
Attention, ce n’est pas pour autant que ABC et A0 C 0 B 0 sont semblables ! En
d = A0d
effet, d’après la définition, cela impliquerait que ABC C 0 B 0 . Or, s’il semble
0 0 0
bien que ABC = A B C , on n’a pas pour autant l’autre égalité : il faudrait pour
d d
cela que A0dB 0 C 0 = A0d C 0 B 0 ...

14
Moralité : il faut toujours faire attention à l’ordre des lettres lorsqu’on écrit
que deux triangles sont semblables !

Exercice 5.3 Quels sont les triangles ABC tels que ABC et ACB sont sem-
blables ? Quels sont les triangles ABC tels que ABC et BCA sont semblables ?

Exercice 5.4 (Puissance d’un point par rapport à un cercle.) Soit C


un cercle, P un point à l’extérieur de ce cercle. Soient (d), (d0 ) deux droites
passant par P et coupant C en A, A0 (dans cet ordre) et B, B 0 (dans cet ordre)
respectivement.
Montrer que P A · P A0 = P B · P B 0 .

Exercice 5.5 (Lemme d’Euclide.) Soit ABC un triangle rectangle en C et


H le pied de la hauteur issue de C.
Montrer que AH × AB = AC 2 .
Montrer que BH × BA = BC 2 .
Montrer que AH × BH = CH 2 .

Exercice 5.6 (Théorème du pôle sud.) Soit ABC un triangle, C son cercle
circonscrit. Soit I le centre du cercle inscrit, IA le pied de la bissectrice issue
de A et S le point d’intersection de cette bissectrice et de C.
Montrer que S est sur la bissectrice de [BC]. Ce point S est appelé le pôle
sud de ABC (par rapport à A).
Montrer que BS = CS = IS. Le cercle de centre S passant par B, C, I est
appelé le cercle antarctique de ABC (par rapport à A).
Montrer que les triangles ABS et BIA S sont semblables.

15
6 Solutions des exercices
Solution de l’exercice 2.9.

Pour tout point D de (d), le triangle AHD est rectangle en H, donc d’hy-
poténuse [AD]. Or l’hypoténuse d’un triangle rectangle est son plus long côté.
Donc AD ≥ AH pour tout D, autrement dit AH est minimale (et H appar-
d
tient bien à (d)). Donc la distance de A à (d) vaut bien AH.

Solution de l’exercice 3.7.

C2
C1 E

A
C ϕ

ϕ
D B
F
ϕ

Posons ϕ := CDB.
d Comme les points A, B, C, D sont cocycliques, on obtient
que BAC = 180˚− ϕ. De plus, les points C, A, E sont alignés dans cet ordre.
d
Donc BAE
d = ϕ.
En outre, les points A, B, F, E sont cocycliques. Donc BF
dE = 180˚− ϕ.
Donc les droites CD et EF sont parallèles.

Solution de l’exercice 3.8.

16
X
90˚− α 90˚− β

B
O1
α l β O2 β

α Y

On effectue une chasse aux angles : en posant XAYd = α, on trouve XO d 1Y =


2α grâce au théorème de l’angle au centre. Or, la figure étant symétrique par
rapport à l’axe (O1 O2 ), on en déduit que XO
d 1 O2 = α.
De même, on montre que XBY = XO2 O1 , ce qui conclut.
d d
En outre, on remarque au passage que, pour deux cercles de centres O1 et
O2 s’intersectant en deux points X et Y , les droites O1 O2 et XY sont perpen-
diculaires.

Solution de l’exercice 3.9.

180˚
−α − β P

R α+β
X 180˚
−β
180˚− α

A α Q β B

On pose α := BAC
d et β := CBA.d Comme les points A, Q, R, X sont cocy-
cliques, on a QXR = 180˚− α. De même, on a aussi Pd
d XQ = 180˚− β. On en
déduit que RXP
d = α + β = 180˚− Pd XC, car Pd XC = 180˚− BACd − CBA
d =
180˚− α − β. Donc les points C, P, R, X sont également cocycliques.

17
Solution de l’exercice 3.10.

C0

B A0 C
B0

On ne change pas une méthode qui marche : procédons à une chasse aux
angles. Grâce aux angles droits, on a P, A0 , B, C 0 et P, A0 , B 0 , C cocycliques. On
en déduit :

(P B, P C) − (AB, AC) = (P B, P A0 ) + (P A0 , P C) + (AC, AB)


= (C 0 B, C 0 A0 ) + (B 0 A0 , B 0 C) + (B 0 C, C 0 B)
= (A0 B 0 , A0 C 0 )
Ainsi, P est sur le cercle circonscrit à ABC si et seulement si l’angle que
nous venons de calculer est nul modulo 180˚, ce qui équivaut bien à A0 , B 0 , C 0
alignés.

Solution de l’exercice 3.11. En posant ϕ := DEA d et µ := CEA, d on peut


rédiger une chasse aux angles usuelle, en passant par les étapes indiquées sur la
figure.
On peut aussi rédiger cette chasse aux angles à l’aide d’angles orientés.
A

B F G µ C

µ
ϕ E

18
L’angle bleu foncé est de mesure 180˚− ϕ, et l’angle vert est de mesure
180˚− µ − ϕ.

Pour montrer que les points D, E, F, G sont cocycliques, il suffit de montrer


que (ED, EG) = (F D, F G). Or :

(ED, EG) = (ED, EA), car les points E, A, G sont alignés,


= (ED, EC) + (EC, EA), d’après la relation de Chasles,
= (AD, AC) + (EC, EA), car les points A, C, D, E sont cocycliques,
= (AD, AC) + (AC, BC), car les arcs AB et AC sont de même longueur,
= (AD, BC), d’après la relation de Chasles,
= (F D, BC), car les points A, D, F sont alignés,
= (F D, F G), car les points B, C, F, G sont alignés.

Donc les points D, E, F, G sont cocycliques.

Solution de l’exercice 3.12.

C2

C1 P

C
B
A

Les angles bleus sont de mesure ϕ, les angles rouges de mesure µ, et les angles
violets de mesure ϕ + µ.

En chasse aux angles usuelle : on pose ϕ := AP dB et µ := BAP d . Comme


la somme des angles dans un triangle vaut 180˚, P BA = 180˚− ϕ − µ, d’où
d
PdBD = ϕ + µ.
De plus, comme les points A, C, P, Q sont cocycliques, on a CQP
d = CAP d =
BAP = µ.
d
Enfin, comme les points B, D, P, Q sont cocycliques, on a DQP
d = Pd BD =
ϕ + µ.
Ainsi, DQC
d = DQP d − CQP
d = ϕ + µ − µ = ϕ = AP dB, et c’est ce qu’il fallait
prouver.
Autre solution :

19
En angles orientés :

(AP, BP ) = (AP, AB) + (AB, BP )


= (AP, AC) + (BD, BP ), car les points A, B, C, D sont alignés,
= (QP, QC) + (BD, BP ), car les points A, C, P, Q sont cocycliques,
= (QP, QC) + (QD, QP ), car les points B, D, P, Q sont cocycliques,
= (QD, QC).

Autre solution :
Une autre approche de cet exercice consiste à rajouter les droites AP et
DQ. Ces dernières sont en effet assez sympathiques comme on sait qu’elles sont
parallèles : c’était l’exercice 3.7.

E
C2

C1 P

ϕ D
ϕ

C
B
A
ϕ
ϕ

F
Q

Posons maintenant ϕ := AP dB. On a alors BP


dE = 180˚− ϕ, et donc, comme
les points B, D, E, P sont cocycliques, BDE
d = ϕ. Comme les droites DE et
AF sont parallèles, on a donc Fd AC = ϕ. Comme les points A, C, F, Q sont
cocycliques, FdQC = 180˚− ϕ, d’où l’on déduit que DQC
d = ϕ.
Autre solution :
En angles orientés, cela donne :

(AP, BP ) = (P E, BP ), car les points A, E, P sont alignés,


= (DE, BD), car les points B, D, E, P sont cocycliques,
= (AF, BD), car les droites AF et DE sont parallèles,
= (AF, AC), car les points A, B, C, D sont alignés,
= (QF, QC), car les points A, C, F, Q sont cocycliques,
= (QD, QC), car les points D, F, Q sont alignés.

Solution de l’exercice 3.13.

20
B

B0 A0
D

On définit P comme l’intersection des cercles circonscrits à CA0 B 0 et à


DBB 0 . Montrons que P, D, A, A0 sont cocycliques et alors, par symétrie, on
aura gagné.
Or, par chasse aux angles :
(A0 P, P D) = (A0 P, P B 0 ) + (P B 0 , P D)
= (A0 C, CB 0 ) + (BB 0 , BD)
= (AA0 , BB 0 ) + (BB 0 , AD)
= (AA0 , AD),
ce qui conclut.

Solution de l’exercice 3.14. Plutôt que d’utiliser une figure, on représente


souvent la configuration du théorème du cube par un diagramme donnant les
relations entre les points impliqués. Le théorème dit alors que, si pour cinq faces
du “ cube”, les quatre sommets de la face sont cocycliques, alors c’est aussi vrai
pour la sixième face.
D C
γ
δ

D0 C0

A0 B0

β
α
A B

21
Par cocyclicité, la somme de deux angles de la même couleur vaut toujours
180˚. Comme A, B, C, D sont cocycliques, α + β + γ + δ = 180˚. Donc la somme
des quatre angles intérieurs marqués vaut 180˚−α+180˚−β+180˚−γ+180˚−δ =
540˚.
De plus, la somme des angles autour d’un point vaut 360˚ : donc on a deux
angles opposés dans A0 B 0 C 0 D0 dont la somme vaut 720˚− 540˚= 180˚.
Donc A0 , B 0 , C 0 , D0 sont cocycliques.

Solution de l’exercice 3.15. Dans cet exercice, les projetés orthogonaux


nous donnent beaucoup d’angles droits. Ceci nous permet de trouver beaucoup
de points cocycliques : par exemple, AA d 0 B = 90˚ et AB
d0 B = 90˚. Ainsi, les
points A, A0 , B, B 0 sont cocycliques. On obtient de même que B, B 0 , C, C 0 et
C, C 0 , D, D0 et D, D0 A, A0 sont cocycliques, ce qui nous permet de tracer les
quatre cercles verts de la figure ci-dessous.

C
A

D0 B
C0

A0

B0
D

Les angles bleus clair sont de mesure 90˚.

Dès lors, d’après le théorème du cube, comme A, B, C, D sont cocycliques,


A0 , B, C 0 , D0 le sont aussi.

Solution de l’exercice 4.5. D’après le théorème de l’angle inscrit dans le cas


particulier d’un angle droit, on peut tracer les six cercles suivants :

22
B

C0

A0
H

A B0 C

Solution de l’exercice 4.6. Réutilisons la figure de l’exercice précédent :


comme les points A, C, A0 et C 0 sont cocycliques (sur un cercle bleu), on a
BAd 0 C 0 = 180
d˚− CA d 0 C 0 = α. De même A, B, A0 et B 0 sont cocycliques donc

CAd 0 B 0 = 180˚− BA d0 B 0 = α, donc B 0d


A0 C 0 = 180˚− 2α.
Passons à la deuxième question : on veut montrer que H est sur la bissectrice
de B 0dA0 C 0 (et des trois angles analogues), donc il suffit de vérifier HAd 0B0 =
0 0
HA C . Or, H, A , B et C sont cocycliques (sur un cercle rouge) donc HA0 B 0 =
d 0 0 d
d 0 = Cd
HCB 0 CA = 90˚− α (car ACC 0 est rectangle en C 0 ). De même en utilisant

le cercle rouge passant par H, A0 , C 0 et B, on vérifie que HA d 0 C 0 = 90˚− α,

donc H est bien sur les bissectrices des angles de A0 B 0 C 0 .

Solution de l’exercice 4.7. Cet exercice se résout très bien au moyen d’une
chasse aux angles rapide, faisant intervenir les propriétés angulaires de O et de
H – propriétés qu’il faut au demeurant maı̂triser–.

23
A

90˚
γ
γ O

B 90˚ γ C

Les angles verts sont de mesure 90˚− γ.

Ainsi, si l’on pose BCA


d = γ, comme O est le centre du cercle circonscrit au
triangle ABC, BOA = 2γ. De plus, par définition de O, on a aussi OA = OB,
d
et donc le triangle BOA est isocèle en O. Donc BAO
d = 90˚− γ.
D’autre part, comme H est l’orthocentre du triangle ABC, on sait que le pied
de la hauteur issue de A dans le triangle ABC, noté A0 , est le point d’intersection
des droites AH et BC. Dans le triangle AA0 C, on connaı̂t deux angles : AA d 0C =
0
90˚ et A CA = γ. Comme la somme des angles dans un triangle vaut 180˚, on
d
en déduit que CAAd 0 = 90˚− γ. Donc CAH d = 90˚− γ.
Donc BAO = CAH.
d d

Solution de l’exercice 4.8. Il suffit de noter HA le symétrique de H par


rapport à (BC) et de montrer que A, B, C, HA sont cocycliques : le fait que HB
et HC , définis de façon analogue, soient sur le même cercle, suivra alors par
symétrie de l’énoncé.
Or BH d A C = BHC par symétrie, et on calcule grâce aux angles droits
d
BHC = 180˚− HBC − HCB
d d d = ABC d + ACB d = 180˚− BAC d
Donc A, B, C, HA sont cocycliques, d’où le résultat.

Solution de l’exercice 4.9.

24
A

O
B D C

L’angle bleu est de mesure 60˚, les angles orange sont de mesure 120˚, les
angles rouges de mesure 30˚.

Comme BDH d = 180˚− BEH d = 90˚, les points B, D, E, H sont cocycliques.


Ainsi, DHE = 180˚− 60˚= 120˚. Donc AHC
d d = 120˚.
De plus, comme ABC = 60˚, d’après le théorème de l’angle au centre,
d
d = 2 × 60˚= 120˚.
AOC
Donc les points A, C, H, O sont cocycliques.
De plus, comme le triangle AOC est isocèle en O, on a OAC
d = 30˚. Donc, par
cocyclicité, OHC = 30˚. Comme CHD = 180˚− DHE = 60˚, on a également
d d d
OHD
d = 60˚− 30˚= 30˚.
Comme la bissectrice commune des angles AHE d et CHDd dédouble chacun
de ces angles de 60˚en deux angles de 30˚, tout comme le fait la droite OH, on
en déduit que le point O est sur cette bissectrice.

Solution de l’exercice 5.3. Les triangles ABC et ACB sont semblables si et


seulement si A
b=A b et B
b=C b (pas besoin de préciser la troisième égalité d’angle,
puisque dans chaque triangle, la somme des angles vaut 180˚). Autrement dit,
ABC et ACB sont semblables si et seulement si le triangle ABC est isocèle en
A.
Les triangles ABC et BCA sont semblables si et seulement si A b = B b et
Bb = C,
b ce qui équivaut à ABC équilatéral.

Solution de l’exercice 5.4.

25
P

µ
B
ϕ
180˚− ϕ

A B0

ϕ
A0

Il semble que P AB et P B 0 A0 sont semblables. De plus, cela suffirait pour


0
conclure : on aurait alors PP B
A
= PP B
A0 , et c’est ce qu’on veut.
Montrons-le. Tout d’abord, l’angle en P est le même dans les deux triangles ;
il reste donc à montrer que Pd BA = P d A0 B 0 . Posons Pd BA = ϕ. On a alors
0
ABB = 180˚− ϕ, d’où, par cocyclicité, AA
d d 0 B 0 = ϕ, ce qui conclut.

Solution de l’exercice 5.5. Du fait que la somme des angles dans un triangle
vaut 180˚, on obtient aisément les égalités d’angles indiquées en couleur sur la
figure ci-dessous.

A H B

On en déduit que les triangles ABC, ACH et CBH sont semblables.


Donc AB AC
AC = AH , d’où AB × AH = AC .
2
BA BC
Donc BC = BH , d’où BA × BH = BC 2 .
AH CH
Donc CH = BH , d’où AH × BH = CH 2 .

26
Solution de l’exercice 5.6. Posons BACd = α, ACB d = γ et CBA d = β. On
a alors α + β + γ = 180˚. On fait une chasse aux angles utilisant le fait que
A, B, C, S cocycliques.

B IA C

Les angles roses sont de mesure α2 , les angles verts sont de mesure γ et les
angles violets sont de mesure β2 .

Ainsi, le triangle BCS est isocèle en S, donc S est sur la médiatrice de [BC].
De plus, comme la somme des angles dans un triangle fait 180˚, on a BIS d =
180˚− γ − α2 − β2 = α2 + β2 . Donc BIS isocèle en S, d’où CS = BS = IS.
Enfin, comme BSA d = IAd SB et BAS
d = IAd BS, les triangles ABS et BIA S
sont bien semblables.

27
Lemmes utiles en géométrie

Thomas Budzinski

Avant-propos
La géométrie est un domaine où une bonne culture peut s’avérer très utile pour résoudre des
exercices. Ce document est une liste (non exhaustive !) de lemmes qui servent fréquemment,
accompagnés de rapides éléments de preuve. Certaines démonstrations ne sont pas évidentes,
les compléter peut être un entraı̂nement intéressant mais il n’est pas nécessaire de les connaı̂tre.
Les lemmes soulignés nous paraissent particulièrement importants.

Dans toute la suite, sauf indication contraire, ABC désigne un triangle. On note a, b et c les
longueurs BC, CA et AB, et α, β et γ les angles CAB, [ ABC [ et BCA.[ On notera aussi C son
cercle circonscrit et O le centre de C et R son rayon, H son orthocentre, G son centre de gravité,
I le centre de son cercle inscrit et r le rayon du cercle inscrit. On notera enfin AABC l’aire du
triangle.

Trigonométrie
Lemme 1 (Loi des sinus).
a b c abc
= = = 2R =
sin α sin β sin γ 2AABC

Idée de la démonstration. Pour montrer sinaα = 2R, introduire B 0 diamétralement opposé à B sur
C et considérer le triangle BB 0 C. Pour la dernière partie, exprimer l’aire en fonction de b, c et
α. 

Lemme 2 (Formule d’Al-Kashi).

a2 = b2 + c2 − 2bc cos α

Idée de la démonstration. Si vous connaissez le produit scalaire, utilisez-le ! Sinon, introduire D,


pied de la hauteur issue de B et calculer AD puis BD et CD puis a en fonction de b, c et α. 

Lemme 3 (Formules d’addition). Pour tous angles α et β :


— cos(α + β) = cos α cos β − sin α sin β
— cos(α − β) = cos α cos β + sin α sin β
— sin(α + β) = sin α cos β + sin β cos α
— sin(α + β) = sin α cos β − sin β cos α

Idée de la démonstration. Utiliser le produit scalaire pour démontrer la seconde formule, puis en
déduire les autres. 

1
Géométrie du triangle
Lemme 4 (Pôle Sud). Soit S le point où la bissectrice (AI) de CAB
[ recoupe le cercle circonscrit
à ABC. Alors S est le milieu de l’arc BC qui ne contient pas A. De plus, S est le centre du cercle
circonscrit à ABI. S est généralement appelé pôle Sud du ABC.

Idée de la démonstration. Chasse aux angles : on vérifie que SBC et SBI sont isocèles en S. 

Lemme 5. Les symétriques de H par rapport à (AB), (BC) et (CA) sont sur C .

Idée de la démonstration. Chasse aux angles. 


BD AB
Lemme 6. Soit D le point où la bissectrice (AI) recoupe [BC]. On a CD
= AC
.

Idée de la démonstration. I est équidistant de (AB) et (AC) donc :

AB AABD BD
= =
AC AACD CD


Lemme 7 (Droite d’Euler). Les points O, G, et H sont alignés dans cet ordre et GH = 2 · OG. La
droite qui contient ces trois points est appelée droite d’Euler de ABC.

Idée de la démonstration. On note A 0 , B 0 et C 0 les milieux de [BC], [CA] et [AB]. L’homothétie h de


centre G et de rapport − 12 envoie ABC sur A 0 B 0 C 0 . On vérifie que O est l’orthocentre de A 0 B 0 C 0 ,
donc h(H) = O. 

Lemme 8 (Cercle d’Euler). On note A 0 , B 0 et C 0 les milieux des côtés de ABC, HA , HB et HC les
pieds de ses hauteurs et MA , MB et MC les milieux de [AH], [BH] et [CH]. Alors les neuf points
A 0 , B 0 , C 0 , HA , HB , HC , MA , MB et MC sont cocycliques sur un cercle appelé cercle d’Euler de
ABC. De plus, le centre du cercle d’Euler est le milieu de [OH] et son rayon vaut R2 .

Idée de la démonstration. L’homothétie h de la preuve précédente envoie O sur le milieu Ω de


[OH], donc le cercle circonscrit à A 0 B 0 C 0 , noté C 0 , est le cercle de centre Ω et de rayon R2 . Une
chasse aux angles montre que HA , HB et HC sont sur ce cercle. Enfin, Ω et MA sont les milieux
de [HO] et [HA] donc ΩMA = OA 2
= R2 donc MA est aussi sur ce cercle, de même que MB et
MC . 

Lemme 9 (Droite de Simson/Steiner). — Soit P un point du plan et PA , PB , PC ses projetés


orthogonaux sur (BC), (CA) et (AB). Alors PA , PB et PC sont alignés si et seulement si
P ∈ C . La droite passant par ces trois points est alors appelée droite de Simson de P.
— Soient SA , SB et SC les symétriques de P par rapport à (BC), (CA) et (AB). Alors SA , SB
et SC sont alignés si et seulement si P ∈ C . La droite passant par ces trois points est alors
appelée droite de Steiner de P.

Idée de la démonstration.
— Chasse aux angles (comme il y a beaucoup de positions possibles des points les uns par
rapport aux autres, il est recommandé d’utiliser des angles orientés).
— SA , SB et SC sont les images de PA , PB et PC par l’homothétie de centre P et de rapport 2,
donc les trois premiers ont alignés ssi les trois derniers le sont.


2
Lemme 10 (Points de contact du cercle inscrit). Soient D, E et F les points où le cercle inscrit
touche [BC], [CA] et [AB]. On a :
— AE = AF = b+c−a 2
— BF = BD = c+a−b 2
— CD = CE = a+b−c 2

Idée de la démonstration. On note x = AE = AF, y = BF = BD et z = CD = CE : on a x + y = c,


y + z = a et z + x = b, et on résout le système... 
Définition. La bissectrice extérieure de CAB
[ est la perpendiculaire à la bissectrice de CAB
[ passant
par A. C’est en quelque sorte la ”deuxième bissectrice” formée par les droites (AB) et (AC). On
définit de même les bissectrices extérieures de ABC
[ et BCA.
[
Lemme 11 (Cercle exinscrit). — Il existe un unique cercle tangent au segment [BC], à la
demi-droite [AB) au-delà de B et à la demi-droite [AC) au-delà de C. Ce cercle est appelé
cercle A-exinscrit à ABC.
— Le centre IA du cercle A-exinscrit est l’intersection de la bissectrice intérieure de CAB
[ et
des bissectrices extérieures de ABC
[ et BCA.
[
Idée de la démonstration. Similaire à la preuve correspondante pour le cercle inscrit : les trois
bissectrices sont concourrantes en un point équidistant de (AB), (BC) et (CA). 
Lemme 12 (Cercle exinscrit, suite). — IA est sur le cercle centré au pôle Sud S passant par
B, C et I, et S est le milieu de [IA I].
— Soit D 0 , E 0 et F 0 les point où le cercle A-exinscrit touche respectivement [BC], [AC) et
[AB). On a AE 0 = AF 0 = a+b+c 2
, BD 0 = BF 0 = a+b−c
2
= CD et CD 0 = CE 0 = a+c−b
2
= BE.
Idée de la démonstration.
— Les triangles IA IB et IA IC sont rectangles en B et C.
— Similaire à la preuve pour les points de contact du cercle inscrit.

Notation. AB désignera la longueur algébrique entre A et B, c’est-à-dire que AB vaudra AB ou
AB
−AB selon l’ordre dans lequel A et B apparaissent sur la droite (AB). Par exemple, AC vaudra
AB
− AC si A est entre B et C car AB et AC sont dirigés dans des sens différents. Par contre, il
AB
vaudra + AC si B est entre A et C car alors les deux seront dirigés dans le même sens.
Lemme 13 (Théorème de Ménélaüs). Soient A 0 ∈ (BC), B 0 ∈ (CA) et C 0 ∈ (AB). Alors A 0 , B 0 et
C 0 sont alignés si et seulement si :
BA 0 CB 0 AC 0
· · = +1
CA 0 AB 0 BC 0
Idée de la démonstration. Si A 0 , B 0 et C 0 sont alignés sur (d), on introduit la parallèle à (BC)
passant par A : elle coupe (d) en X, puis on applique deux fois le théorème de Thalès. Pour la
réciproque, étant donnés A 0 et B 0 , on sait qu’un seul point C 0 ∈ (AB) est sur (A 0 B 0 ), et que ce
0 0 0
point permet vérifie AC BC 0
= CA
BA 0
· AB
CB 0
. Or, un rapide calcul montre qu’un seul point C 0 vérifie
cette formule. 
Lemme 14 (Théorème de Céva). Soient A 0 ∈ (BC), B 0 ∈ (CA) et C 0 ∈ (AB). Alors (AA 0 ), (BB 0 )
et (CC 0 ) sont concourantes si et seulement si :
BA 0 CB 0 AC 0
· · = −1
CA 0 AB 0 BC 0

3
AB 0AABX
Idée de la démonstration. Si les trois droites sont concourantes en X, montrer que A 0C = A
ACX
et deux relations similaires, puis faire le produit. Pour la réciproque, ça marche comme pour
Ménélaüs. 
Lemme 15 (Théorème de Céva trigonométrique). Soient (Ax), (By) et (Cz) trois droites passant
par A, B et C. Ces trois droites sont concourantes si et seulement si :
sin BAx
d sin CBy
d sin ACz
d
· · =1
sin CAx [ sin BCz
[ sin ABy d
Idée de la démonstration. Si les trois droites sont concourantes en un point X, appliquer la loi
des sinus dans les triangles ABX, BCX et CAX. Pour la réciproque, c’est similaire aux deux
théorèmes précédents. On a besoin de montrer que sinsin x
(α−x)
est strictement croissante. 
Remarque. Un sens reste vrai avec plus de trois points : si on a un polygone à n sommets
avec une droite partant de chaque sommet et si les n droites sont concourantes, alors on a une
formule du même type. En revanche, la formule ne suffit pas à assurer que les n droites sont
concourantes.
Lemme 16 (Conjugué isogonal). Soit X un point du plan. On note (dA ) le symétrique de (AX)
par rapport à la bissectrice de CAB,
[ et on définit de même les droites (dB ) et (dC ). Alors (dA ),
(dB ) et (dC ) sont concourantes en un point appelé conjugué isogonal de X.
Idée de la démonstration. Utiliser le théorème de Céva trigonométrique. 
Remarque. On pourra vérifier que O est le conjugué isogonal de H, et I est son propre conjugué
isogonal.

Autres
Lemme 17 (Quadrilatère circonscriptible). Soit ABCD un quadrilatère. ABCD est circonscrip-
tible (c’est-à-dire qu’il existe un cercle tangent à ses quatre côtés) si et seulement si AB + CD =
AD + BC.
Idée de la démonstration. Si ABCD est circonscriptible, on fait une ”chasse aux tangentes” : on
décompose chaque côté en deux en coupant au point de contact du cercle, et on utilise le fait
que A est équidistant des points de tangence à [AB] et [AD]...
Pour la réciproque, supposons quitte à changer les noms des points que [AD) et [BC) se re-
coupent en X, et soit ω le cercle inscrit à ABX : il touche [AB] en U, [AX] en V et [BX] en W.
La condition AB + CD = AD + BC donne CD = DV + CW. On a ainsi XV = XV+VW 2
=
XD+XC+VD+CW XD+XC+CD
2
= 2
, donc ω est le cercle X-exinscrit à XCD, donc il est tangent à
[CD], et on savait déjà qu’il était tangent aux trois autres côtés. 
Lemme 18 (Diagonales perpendiculaires). Soit ABCD un quadrilatère. Alors (AC) et (BD) sont
perpendiculaires si et seulement si :
AB2 + CD2 = AD2 + BC2
Idée de la démonstration. Si les diagonales sont perpendiculaires, on obtient la formule en ap-
pliquant plusieurs fois le théorème de Pythagore. Pour la réciproque, on put utiliser la trigo-
nométrie mais le plus simple est d’utiliser le produit scalaire pour montrer :
−→ −→
AB2 + CD2 − AD2 − BC2 = 2BD · CA


4
Lemme 19 (Puissance d’un point). Soit Γ un cercle et P un point. On considère une droite (d)
passant par P qui recoupe Γ en A et B. Alors PA · PB ne dépend pas de la droite (d), et est appelé
puissance de P par rapport à Γ .
De plus, si on note O le centre de Γ et r son rayon, la puissance de P par rapport à Γ vaut
OP2 − r2 .

Idée de la démonstration. Si deux droites (d) et (d 0 ) coupent Γ en A, B et A 0 , B 0 , montrer que PAA 0


et PB 0 B sont semblables. 

Lemme 20 (Axe radical). Soient Γ1 et Γ2 deux cercles de centres O1 et O2 différents. L’ensemble


des points qui ont la même puissance par rapport à Γ1 et Γ2 est une droite perpendiculaire à
(O1 O2 ), appelée axe radical de Γ1 et Γ2 . De plus, si Γ1 et Γ2 se coupent en A et B, il s’agit de la
droite (AB).

Idée de la démonstration. On note r1 et r2 les rayons des deux cercles. Soit X un point qui a la
même puissance par rapport à Γ1 et Γ2 (on admet qu’il existe) : on a O1 X2 − r21 = O2 X2 − r22 . Si Y
est un point quelconque, les deux puissances de Y sont les mêmes ssi O1 Y 2 − O2 Y 2 = r21 − r22 =
O1 X2 − O2 X2 ssi (O1 O2 ) et (XY) sont perpendiculaires d’après le lemme précédent, donc l’axe
radical est la perpendiculaire à (O1 O2 ) passant par X.
Si Γ1 et Γ2 se coupent en A et B, A et B ont une puissance nulle par rapport aux deux cercles,
donc son sur l’axe radical. 

Lemme 21 (Axes radicaux, suite). Soient Γ1 , Γ2 et Γ3 trois cercles de centres deux à deux distincts.
On note (d1 ) l’axe radical de Γ2 et Γ3 , (d2 ) celui de Γ3 et Γ1 et (d1 ) celui de Γ1 et Γ2 . Alors (d1 ), (d2 )
et (d3 ) sont parallèles ou concourrantes.

Idée de la démonstration. Le point d’intersection de (d1 ) et (d2 ) a même puissance par rapport
aux trois cercles donc est sur (d3 ). 

Lemme 22 (Théorème de Miquel). Soient quatre droites (d1 ), (d2 ), (d3 ) et (d4 ). Ces quatre
droites définissent quatre triangles, un pour chaque choix de 3 droites parmi les 4 (une telle
figure est appelée quadrilatère complet). Alors les quatre cercles circonscrits aux quatre triangles
ont un point commun, appelé point de Miquel du quadrilatère complet.

Idée de la démonstration. Donner des noms à tous les points d’intersection et faire une chasse aux
angles. 

Remarque. Le point de Miquel est également le centre de nombreuses similitudes directes im-
pliquant les points d’intersection des quatre droites (cf. cours sur les transformations).

5

 

   "!$#&%('*)+('-,

.*/1032 54678 / 9:7


; <>= ?A@CBEDGFIHKJLBEM*DN@POQJRFI?&SITVUWQDGF X
Y[Z\Y ]I^`_bac^[dGefg^h_cikjldZmZ1Z1ZmZ1ZmZ1Z1ZmZnZ1ZmZ1ZmZ1Z1ZmZ1ZmZnZ1ZmZ1Z1ZmZ1ZmZ1Z1ZmZ o
Y[Zpo ]I^Aeq r&st_cacikuwvtuxdy_bac^[fku Z1ZmZ1Z1ZmZnZ1ZmZ1ZmZ1Z1ZmZ1ZmZnZ1ZmZ1Z1ZmZ1ZmZ1Z1ZmZ o
Y[Z{z ]I^Aacjh_|^h_cikjld ZnZmZ1Z1ZmZ1ZmZ1Z1ZmZnZ1ZmZ1ZmZ1Z1ZmZ1ZmZnZ1ZmZ1Z1ZmZ1ZmZ1Z1ZmZ z
Y[Z~} ]I^Aeq r&st_cacikun^€Nik^[fkuZ1ZmZ1ZmZ1Z1ZmZnZ1ZmZ1ZmZ1Z1ZmZ1ZmZnZ1ZmZ1Z1ZmZ1ZmZ1Z1ZmZ z
Y[Zp‚ ]>ƒ…„Gjlr&j[_c„Gst_ciku ZmZ1Z1ZmZ1ZmZ1Z1ZmZnZ1ZmZ1ZmZ1Z1ZmZ1ZmZnZ1ZmZ1Z1ZmZ1ZmZ1Z1ZmZ z
Y[Z{† ]I^Aeci\r‡i\fkiˆ_b‰GŠGuŠGi\acuxv_bu Z1ZmZ1Z1ZmZnZ1ZmZ1ZmZ1Z1ZmZ1ZmZnZ1ZmZ1Z1ZmZ1ZmZ1Z1ZmZ z
Y[Zp‹ Œ>ajlGf\Žxr&uxeZ1ZnZmZ1Z1ZmZ1ZmZ1Z1ZmZnZ1ZmZ1ZmZ1Z1ZmZ1ZmZnZ1ZmZ1Z1ZmZ1ZmZ1Z1ZmZ }
X <>D‘R’“JRM*’y@PBEO”=SIT–•l=“Bh•yW”= —
o Z\Y Œ>ajlGf\Žxr&uxeZ1ZnZmZ1Z1ZmZ1ZmZ1Z1ZmZnZ1ZmZ1ZmZ1Z1ZmZ1ZmZnZ1ZmZ1Z1ZmZ1ZmZ1Z1ZmZ ˜
™ <>D‘R’“JRM*’y@PBEO”=SITV@PBEOQDGFIRW”= ;“š
z›Z\Y Œ>ajlGf\Žxr&uxeZ1ZnZmZ1Z1ZmZ1ZmZ1Z1ZmZnZ1ZmZ1ZmZ1Z1ZmZ1ZmZnZ1ZmZ1Z1ZmZ1ZmZ1Z1ZmZœYC†
 žŸJRW T@€O”JRFVS9=›?A=y¡=“BE•yO •l= ? ;N¢

Y
]9^ [sxjlr&st_caciku1uxe _w‰Gdh^[e _bu e‰ut_ ‰Nij vxvt‰  u‰GdGu
Gfk^[vxumikr  j[a_|^hd“_cunŠ ^[dGe f\uxe u›uxa
vxikvtuxeGacj jlesxem^[‰› f\q r Gig^hŠGuxe dy_butacd ^h_cikjldG^[fkuteecj[‰lutd“_ ŠGux‰›‘u ›uxacvtikvtuxe e‰Gameiˆ ZI]9^
lsxjlr&st_caci\u_cuxfkf\u  ‰Nu dGjl‰Ge^[fkf\jldGe>f ƒ…st_c‰GŠGikuta i\vxiR^Gac^h_bi ‰Nuxr&uxdy_ŠGi\e ^hac‰ŠGuteGajlab^hr‡r&uxe
ŠGu`f\q vxst
u Rjl‰Ga fk^[ikeecuxa Gfg^[vtu‰GdGu[sxjlr&st_caciku Gfk‰New^[d ^[fˆq“_bi!“‰Gu" fk^acstecjlf\‰N_bi\jld(ŠGutewu›uxa
vxikvtux#e  ^heceb^hd“_1ejl‰ lutdy_ G^[a f ƒ…ux r Gf\jliŠNuxewdGjlrGacutewvxjl r Gfku ›uxex%Z $ ^[dGendGj[_cacu vtjl‰Gae dGjl‰Ne
d ƒ{^h jlaŠGuxajldG&e  ^hevxut__b#u  ^ha_bi\u “‰G'i ›Gikut(d “‰ ƒ~‰›_bikf\u ut_7*u )&vC^[vtu  +Q^[iˆ_ecj[‰ lutd“,_ Gf\‰Ge7^  Rux%f 
ŠGu7fg^ _butv|„GdG!i “‰GuvC^hfkvx‰Nfg^h_cjlikau “‰ -ƒ n‰Gd‡ab^[i\ecjldGdNuxr&uxdy._ lsxj[r‡s_baci “‰Gu[0Z /jl‰Ge>dGjl‰G1e Gai lutacjldNe
ŠGjldGvnŠGunf ƒ…‰Gec^ luwŠ ƒ…‰G2d  ‰Nuxfkvtjl d “‰Gu1ac3u  Žtacu[Z
]9^`ŠG4i )‡vt‰Gf\_cswŠGuwfg
^ [sxjlr&st_caciku ^AŠNux‰›3j[ac!i [ikdGute 5GŠ ƒ…‰GdGu  ^[a _7fg^ ŠG4i )‡vt‰Gf\_cs “‰ ƒ…jl6d Rux‰›_
7^ [jlik8a  j[‰Ga 9:;=<>vxu  ‰NiŸecu  ^[eceu ut_  ^[>a +”j[ike @r ?tr‡Au  jl‰Na _bac^[vxuta ‰GdNu BCl‰Gau vxj[acacutvt_cu Nu_fku
_bacŽte>fg^[>a lu3s [uxdy_|^[i\f Š ƒ…jl‰N_bi\fke u_ ŠNu7_c„Gsxj[acŽxr&uxe>r&ik.e wdGj[_cacu7ŠGi\e Rjleciˆ_bikj[d ZlŒIjl‰Ga>acstecjl‰GŠNacu‰Gd
GacjlNfkŽxr&uwŠGu lstjlr‡s_baiku Njld ŠNjldGdGutab^‡ŠNux‰› vtjldGecutikf\exCZ DIjl‰N_Š ƒ~^[ j[acŠ ikf ute_ ik r  jla _|^[dy_ŠNu
_bab^hvxuxa>‰GdNu RjldGdGu BCl‰Nacu ERjl‰Ga>vtuxfg^njl d  ut‰N_ Š ƒ{^h jlaŠ‡f\ikauf ƒ…sxdGj[dGvxsux d +Q^[ikec^[dy_>‰GdGu BCl‰Gau
r-^hikd&fk3u [sxu Rjl‰G1a ljli\a f\ux8e  jli\dy_be i\dy_bacj ŠG‰Giˆ_be ut8_ ljli\a vxj[r‡r&uxdy_f\uxe ŠGik>e  j[ecux3a NutdGec‰Giˆ_bu jld
_bab^hvxu‰GdGu lac^[dGŠG&u BC[‰Gacu Gasxvxi\ec,u wfg^ aŽ lf\uut_ ^[‰&vxjl r  ^[,e F”ec‰Ga>‰GdNuŠNuxr&4i GG^ luHlj[ikacu7‰GdNu
 ^ lu utdy_bikŽtacJu I%Z $mƒ{^[‰›_bacu  ^[a_ ikfIuxe_  jld ŠNu ac3u  stacuxawe‰Ga fk@^ BCl‰Gau  ut_nJ^ h^[dy_wŠGu v|„Nuxacv|„Nuxa
fk,u GacjlNfkŽxr&u Nacj Gauxr&uxdy_ŠNi\J_ 0“‰Gut!f “‰Guxe3s y^[f\i\_bste ŠŸƒ{^[d lfkute  ŠGu fkj[ d l‰Gut‰Gac8e  ‰Ni jlaikutd“_cuxajldy_
fkuwab^hikecj[dGdGuxr&uxdy_xZ
/jl‰Ge^[fkf\jldGKe ljli\ae‰Gvxvtikdy_butr‡utdy__bajlike^[e Ruxv_beŠGu fkA^ lstjlr‡s_baiku>ŠG
‰ Gfk^[d 5P_cjl‰N_ŠŸƒ{^[RjlaŠ
dGjl‰GLe lutacacj[dGe f\uxe3ŠG4i MŸstacuxdy_cuxe3_bab^hdG>e +”jlar-^E_bikj[dGe6ŠGN‰ Gfg^[Od Rjl‰ h^[dy6_ ?t_bau ‰N_bi\fkuxLe Rjl‰Ga3fg^
acsxejlfk‰›_bikj[d Š ƒ…u›uxavxikvtux3e &G‰Gike-fg^ lstjlr&st_baiku(ŠN‰–vxuxavxf\u 7^ [uxv ŠGut‰›–jl‰›_bikf\e-i\@r Rjla_b^[dy_be 
ebJ^ ljli\a3fgQ^ PSRTUSU7VWTX"YZT[]\^!VSU-ut_ fk_^ N‰Gikeeb^[dGvtu Š ƒ~‰N`d RjlikdyJ_  ut_3uxad B dfk_^ [sxjlr&st_cacikuŠG‰
_bacik^[ d lf\u7j b6dGjl‰Ge ab^  Ruxf\fkutacjldGe ‰Gd6vtuxa_b^[ikd-dGjlr Gau ŠGAu +”jlacr ‰Nfkuxe ‰N_bi\fkute u_ŠGuxce Gacj Naciks_bste
ŠGu  jli\dy_beacutr-^ha “‰ ^[Gf\uxetZ
d e 7  / f(g 1 / 9 f  43 h 2t/ f
i j[r‡r&ux d jtjldG
e G^[as_b‰GŠGi\uxamfkuxem_bab^hd +”jlacr‡^h_cikjldGe “‰G8i Gacstecuxka lutd“_&f\uxe ŠGi\e_b^[dGvxute ut_fkute
^[ d lf\uxexZ
l1mkl
n oqp]r o.st]u ovpw x.s
y jli\_{ z ‰Gd6[uxvt_cux‰Ga ŠG‰2Gfg^[d|Gfg^ _bac^[dGefg^h_cikjld~} ŠNu[uxv_bux‰Na2{ z ^hececj vxi\uA_cjl‰N_, j[ikdy_A
ŠG‰€Gfk^[d fku2 jli\dy_~O _butf ‰Nu†‚ƒ‚„‚ N… #‡ { z Z ]9^ vtjlr@Rjlei\_bi\jld ŠGu ŠGux‰› _cab^[dGefg^h_cikjldNe&ŠNu
luxv_but‰Gace { z ut_
ˆ z uxe _ fk^n_cab^[dGefg^h_cikjldAŠGulutvt_cux‰Ga {,z ‰ ˆ z yutfkf\udNu ŠGs3 utdGŠ~ ^he ŠGuf ƒ…jlaŠGacu7Š ^[dNe
fku“ ‰Gutf>jld u*ŸM utvt_c‰Gu-f\uxenŠGux‰  _bab^hdGecfk^h_bi\jldGexZ‹Š dGuA_bab^hdGecfk^h_bi\jld uxdElj[iku&‰GdGu&ŠGacjliˆ_buAec‰Nam‰GdNu
ŠGacjliˆ_bu# ^hab^[f\fkŽxf\u[Z
l1m'Πn
ot0Ž‘|p]r w ’†“|’Ks1paro.uk’
9] ^6eq r&st_caciku`vtuxdy_bac^[fku`ute_1‰GdGu&^[‰›_bacu _cab^[dNe>+”jlar-^h_cikjldecikr Gf\u[Z ]I^6e q›r&st_caci\u`vxuxdy_cab^[f\u
ŠGuvxutdy_bacuL” uxdEljli\uL ec‰Ga`f\uRjlikdy_  _butfc“‰Gu•”
‚‚ …  ‡ ‚ ”
‚3‚>… Z– f exƒ~^liˆ_ sl^[fkutr‡utd“_
Š ƒ~‰NdGunacj[_b^h_bi\jld6Š ƒ~^[d lf\u— Z
]9^&vxjl r  j[ecsxumŠ ƒ…‰GdGu_cab^[dGefg^h_cikjld3u_ Š ƒ…‰GdGu e q r‡s_baci\umvtuxdy_bac^[fkumuxe _ ‰GdGueq r‡s_baiku1vxuxd]
_bab^hfku[Z y jliˆL_ ˜‘‰NdGu eq r&st_baiku(ŠNu vxuxdy_cacWu ” ut_6˜  ‰GdNu eq r‡s_baiku ŠGu vtuxdy_bau™”  ^[fkjlaefg^
o
_bab^hd+”jlacr‡^h_cikjldL˜,˜1ute_ ‰GdGuw_bac^[dGecfk^h_bi\jldŠGu#[uxvt_cux‰Gaa”‚ƒ‚ ”… ˆZ
l1m n
o‘r xvpaovpawkx.s
]9^nacj[_b^h_bi\jld`ŠGuvxutd“_cacuA” ut_ Š ƒ~^[d lf\u ^[ecej›vtiku,n_cjl‰N_1Rjlikdy_8 ŠG‰Gfk^[dA‰Gd  jli\dy_8 
_buxf| ‰Nu ” ‡Q”
  u _ ` ”
  ‡
 FQ^[d lf\uwjlaci\uxdy_bsJIZ __butd“_cikjld|Geci j[d auxr Gfg^hvxu   ^[a
‚f ƒ…ikŠGutjld“d‘ ^ Gfki  ‰Nu fg^aj[_|^h_cikjldikdElutacecuAŠGuAfk^-acj[_b^h_bi\jld lj[‰Gfk‰GuhZ y i€‡ fg^‡acj[_b^h_bi\jld uxe _
_ci\_cs[Z y i  ‡N— Gvhƒ~ute_ ‰GdGuneq r&st_baikuwvtuxdy_bac^[fkuhZ
l1m n
ot0Ž‘|p]r w ’†ow o.uk’
ŒIjl‰Ga&dGu2 ^[e&fg^ vxjlda”+ jldGŠGau^Jlutv fk^‘e q r‡s_baci\u3vxuxdy_cab^[f\u j[dVf ƒ~^ Ruxf\fkus3y^[f\uxr&uxdy_&acs*
 u›ikj[d Z y jliˆ_ ‰GdNu‡ŠGajli\_cu fk^6acs  u›ikjld‘Š ƒ~^E›u ^[eecj vxiku 6_bjl‰›_ Rjli\d“_  ŠG‰Nfg^[d fku
 jli\dy_  _butfv“ ‰Gu † ‚ƒ‚C‚  …  ejli\_#R uxa> utdGŠGi\vx‰Gfk^[ikau
œut_“‰Gu`f\u r&ikfki\ux‰ ŠGu `  ^ ^[ak
_bikutdGdGutd“_A` fg^ ŠNacjliˆ_bu Z
]9^wvxj[r@R jlecstuŠGuŠGut‰›Aacs  u›i\jldGe uxe _ ‰GdGu_bac^[dGecfk^h_bi\jldei fkute ^E›uxe>ecj[d“_1 ^hab^[f\fkŽxf\uxe3h‰GdNu
acj[_b^h_bi\jld3ecikdNjld Z y i uxe _ f ƒ~^[d [fkuwuxdy_cacu1fkuxe ^E›uxe  u_  ŠNuxe as  u ›ikjldGe˜ ut_˜  R ^[f\jlace7fg^
vxjlr  j[ecsxu˜  ˜ uxe _ ‰GdGu acj[_b^h_bi\jldmŠŸƒ{^[dlfku!" u_ŠGjldy_fku vtuxdy_bauuxe _fku8 j[ikdy_ŠŸƒ~i\d“_cuxaecuxv_bi\jld
ŠGuxe7ŠGajli\_cuxe# ut_  Z
l1m%$ n&'x1‘xvp('A|pawk’
] ƒ…„Gjlr&j[_b„Nst_bi\uwuxe _ ‰NdGu1_cab^[dNe>+”jlar-^h_cikjld(“‰GidNumvtjldGecuta>[u ^[e7f\uxe7ŠGike _|^[dNvxuxetZR]>ƒ…„Gjlr&j"
_b„Gs_biku ŠGuvxutd“_cacu”u_ ŠGuac^ Rjla_*)&^[ecej›vtiku& _cjl‰N_ Rjli\d“_  ŠG‰ Gfg^[dfkuRjlikdy_ N“_buxfa“‰Gu
”
‚‚ …  ‡) ”‚ ‚>$ … Z y +i ) ‡ ‚-, jld aut_cacjl‰[uAfg^-e q›r&st_caci\u vxutd“_cab^[f\u[ZKŠdGu`„Gjlr&j[_c„Gst_cikuutdEljli\u
‰GdGu ŠGacjliˆ_bu6e‰Ga‡‰NdGu3ŠGacjliˆ_buL“‰Gi7fk‰Gi uxe_@ ^[ac^[fkf\Žxf\u[Z]ute‡eux‰Gf\uxeAŠGacj[i\_buteAikdEh^[acik^[dy_buxeAejldy_
vxuxf\fkute “‰GKi  ^[eecuxdy,_  ^[af\u vxutdy_bacunŠGuwf ƒ~„Njlr‡jh_b„Gs_bikuhZ ŠdGun„Gjlr&j[_b„Nst_bi\u vtjldGecut>a [umf\uxe7^[ d [fkuxe
ut_`r ‰Nf\_bi Gfki\uAfkuxefkjl d [‰Gux‰Gae  ^h/a .0).pZ y i 1‚… 2 uxe_A‰Nd luxv_but‰GaAŠG_‰ Gfg^[|d ejld ikr‡^ lu61‚‚ 2… 
lsxai B u 1 ‚E‚H 2…  ‡3) 1‚E… 2 „Z ŠdGun„Gjlr&j[_b„Nst_bi\u vxj[dGecut>a lumŠGjldGvnf\uxeab"^   j[a_be7ŠGunf\jl d l‰Gut‰GacetZ
]9^vtjl r  jlesxu ŠNu ŠGux‰ 6„Gjlr&j[_c„Gst_cikuxe uxe _ecjliˆ_‰GdNu „Gjlr&j[_b„Nst_bi\u ŠNjldy_fku ab^  Rjla _uxe _fku
Gacj ŠG‰Giˆ_ŠGuxeab"^   j[a_b3e Gecj[i\_7‰GdGuw_cab^[dGefg^h_cikjld6eifk#u Gaj›ŠN‰Gi\_ŠNuxeab^  Rjla _be h^[‰N_ , Z
]ute „Gj[r‡j[_c„Gst_cikuteecjldy_7_baŽxe‰N_bi\fkuxef\jlaceŠNu1fk^`acsxejlfk‰›_bikj[d ŠŸƒ~u Nutacvtikvxute Š ^hdGe7fku _baig^[d lfku
F lj[ikavxutacvtfkunŠ 5ƒ 4>‰Nfkuxƒa IZ
l1m6 n
ot]w>‘w u w p87 9 ’:9AwGr ’K“ pa’
]9^&ecikr&ikf\i\_c‰GŠGu ŠNikacutvt_cumŠGu1vxutdy_bacu
”@ ŠGu1ab^ jla _ ) u_ ŠŸƒ{^[dlfku uxe_ fg^Avxj[r@RjlecstumŠNu
f ƒ…„Gjlr&j[_b„Gs_bi\u ŠGu vxuxdy_cacu$ ” u_7ŠGu ac^ R jla_;) ut_ŠGu fg^macj[_b^h_bi\jld-ŠGuwvtuxdy_bau”$u_Š ƒ~^[d lf\u< Z
i ut_c_cuw_bab^hdGe>”+ jlar-^E_bikj[d6vxjldGeuxakl u1fkuxe^[dlfkuteut_7r ‰Gfˆ_biGfki\u fkutefkjldl‰Gux‰Nace& ^[a=. ).{Z
 d  ut‰N_vtjldGeci\ŠGsxauxaA_bjl‰›_buxe‡fkute&_cab^[dGek”+ jlacr‡^h_cikjldGeNacsxvtsxŠGutdy_buxeL‘f ƒ…u›vxu3N_bi\jldVŠNufg^
eq r‡s_baiku vtuxdy_bac^[fku vxjlr&r‡uŠGuteeci\r‡i\fki\_c‰GŠGute,5 ‰GdGu _bab^hdGecfk^h_bi\jld-ute_‰GdNu eci\r‡i\fki\_c‰GŠGuŠGu ab^a
 jla _ , ŠGj[d“_ f\uvtuxdy_bau ute_ eci\_c‰Gs,1 f ƒ~i\dB dGi [ ‰GdNu aj[_|^h_cikjldAute_ ‰GdGuecikr&ikf\i\_c‰GŠGu ŠNu ac^ R jla_ , 
ut_7‰GdGun„Gj[r‡j[_c„Gst_ciku ute_‰GdGuneikr&ikfkiˆ_b‰GŠNu Š ƒ~^[d [fkuwd“‰Gf Z( fkj[acefg^ vtjlr  jlesxunŠGuwŠGut‰› eci\r‡i\fki
_b‰GŠGute7ŠGikauxvt_cuxeuxe _ ‰NdGunecikr&ikf\i\_c‰GŠGu[Z
] ƒ…ikr‡^luwŠ ƒ…‰Gd_cacik^[d lf\u ^[a‰GdGumecikr&ikf\i\_c‰GŠGu ute_ ‰Gd _cacig^hd lfkuwŠNjldy_ f\uxe ^[d lf\uxe7ecjldy_ fkute
r@t? r‡uteu_Š ^[dGe7fkunr x? r&uwjlacŠNacu1^[‰N_cjl‰Ga7ŠG‰ _cacik^[d lf\u[Z f q ^ _cacjli\evtaciˆ_bŽxauxeR jl‰N_ŠGikau#“ ‰Gu
z
ŠGux‰› _bacik^[d lf\uxe ecjldy_9ecutr Gfk^[Gfkutev5Cejli\_r&jldy_bauxa%“‰Gu>f\ux‰Gae9^[d lf\uxe ejldy_s3y^[‰› Cejli\_ r‡jldy_cacuxa
 ‰Nu f\u ac^ Rjla_IŠGutef\jld l‰Gut‰GaceIŠGutevh_bsxeIuxe _f\u r ?xr&uPejli\_9_cacjl‰[uxaI‰GdGu ecikr&ikf\i\_c‰GŠGuŠNikacutvt_cu
 ‰NiuxdEljli\unfkuxe7ejlr‡r&ut_ceŠGunf ƒ…‰Gd3ec‰Ga7f\uxeecj[r‡r&ut_ceŠGunf ƒ~^[‰N_bau[Z
l1m  r xu‹‘’Kt
¡ =“Bh•yOQ•l= ;

y jlikutd“_ V‰Nd vtuxacvtfkuAut_  ‰GdGu&ŠGacj[i\_bu`ŠNjldGdGste9vxjldNe_ba‰Gikau‡‰NdGuAŠGacjliˆ_bu ^hab^[f\fkŽxf\u


vxjl‰ G^[dy_ f\uwvxutacvxf\u uxd ŠGut‰›2 j[ikdy_beei\_b‰Nsxe,`‰GdGu1ŠGike _|^[dGvtu-ŠGjldNdGsxuhZ
¡=“Bh•yOQ•l= X

4_b^[dy_ŠGj[dGdGs&‰G d Rjlf\]q lj[dGu  v h_bsxe1jl™d  ut‰N_vtjldGeikŠGstacuxanf\uxe1r‡i\fkikut‰›• 


ŠGuxe7v h_bsxet Z  Ed lutacecutr‡utdy_ eifkuxce  jli\dy_b,e S`ecj[d“_ŠGj[dGdGsx3e Gu›i\e_bu K_k ikf ‰GdL j[f\q]ljldGu
ŠGjldy_ f\uxe Rjli\d“_c#e ecjldy_ f\uxe r&ikf\ikut‰›6ŠGutewv [_csxe F”st_c‰GŠGikuta f\uxe vx^[e W‡!-ut"_ W‡!#  Rjl‰Ga
fkuwvC^h,e [sxdGstab^[f GŠGike _bi\ d l‰Gutauxdy_ba u   ^hikau_ i\@r G^[ikƒa I%$
¡=“Bh•yOQ•l= ™

 d3vxj[dGeci\ŠGŽxau _bacj[ike ŠNacjliˆ_buxce  ^[ab^hfkfkŽtfkut!e &' )( )*CZ i jldGe_cac‰Gi\acu ‰Nd3_cacik^[ d lf\u s “‰Gi\fg^h_c*s
ab^[f 1  1 ( 1 *9_buxf  ‰Nu fkuxKe Rjli\d“_ce 1  1 ( 1 *I^   ^[a _bi\uxdGdGutdy_9acux>e  utvt_ci lutr‡utdy_^h‰›wŠGajli\_cuxe & )( +*xZ
¡ =“Bh•yOQ•l= ,

y jlikutd“_- u_.  ŠGux‰  vxuxavxf\uxeRr&jldy_bacuta ‰Ÿƒ~i\fu›ike _bu1ŠGux‰› „Gj[r‡j[_c„Gst_cikute_bab^hdGe>+”jlar-^hd“_


uxd  Z / ‰GunŠGi\acuwecif\uxe7ab^Cq[jldGe7ecjldy_sl^[‰›0$
¡=“Bh•yOQ•l= —

y jli\_ 1 221 ‰Gd _cacig^h d lfk"u  vtjldGe_cac‰Gi\ac™ u  fk^ acŽ [fku ut_^[‰ vxjlr  ^[e3‰Gd vC^[aacs ŠGjldy_ ‰Gd
ecjlr&r‡u_^  ^[a_cikutd“_^[‰6v [_cs 12 G‰Gd6ecj[r‡r&ut_^[‰6v [_cs 11 ut_ŠGut‰›3ejlr‡r&ut_ce^[ŠHc^[vxutd“_ce
^   ^[a _bi\uxdGdGutdy_ ^h‰ v [_bs 221 Z
¡=“Bh•yOQ•l4 = 3

y jli\_ 1221 ‰Gd_baig^[d lfkuikej›vtŽxfkuuxd 1 Nf\uxce Rjli\d“_c&e  ut6_ 5 ecj[d“_ Gaci\e ec‰Nafkutev [_csxe 1 2
ut_ 1.1 aux>e  utvt_b4i lutr‡utd“_xZ ]uxe ŠGacjliˆ_but"e 7 2 598 u_ 7 1 :8>ecu vtjl ‰ Ruxdy_ux<d ;&>Z = jldy_bauxa “‰Gu fkute
ŠGacjliˆ_bute 7'?5487u _ 7 221 8ecjldy,_  ^[ac^[fkf\Žxfkuteeciu_7ecut‰Gfkutr‡utd“_eiIj[d ^ 1  ; ‡ 1 ;5 Z
¡=“Bh•yOQ•l= @ ¢

y jli\_ 1  1 ( 1 *‡‰Gd _baig^[d lfkuuA_ ;CB&‰Gd Rjli\d“_AŠN‰ Gfk^[d .Z  d ŠG3s BGdGi\_ 1.D ‡ 1DFE *@Rjl‰Ga
˜GH#G0Z  d-vtjldGe _bac‰Ni\_>‰GdGuec‰Ni\_bIu ;J;C(K;C*lŠGu_butfkf\uecjla _b,u  ‰N.u ;,ecjliˆ_ f ƒ~i\r-"^ lu ŠG-u ;C E 
 ^[a`fg^ acj[_b^h_bi\jld ŠGu3vxutdy_bacu 1 JLMmut_AŠ ƒ~^[ d lf\u (* N OZ = jldy_bacuta “‰Gu3ec0i ;QPFRFSL‡T;CBJ^[fkjlae fku
_bacik^[ d lf\u 1  1 ( 1 * uxe _s “‰Gikfk^h_bstab^[f Z
¡=“Bh•yOQ•l4 = U

4_b^[dy_ ŠGjldNdGsxe _cacjli\e vtuxacvtfkute ŠGut‰› &ŠGux‰›ŠGi\e jli\d“_cexZ Dac^[vxuta f\uxe _cacjli\A
e Rjli\d“_ce Š ƒ…ikdy_cuxka
ecuxv_bi\jld ŠNuxe7_|^[ d [uxdy_bute u _csxaci\ux‰Gauxevxjlr&r ‰GdNux,e &v|„G^ “‰Gu  ^hikacunŠGunvtuxacvtfkutexMZ = jldy_baux,a “‰Gu
vxuxe_cacjli\&e  jli\dy_beecj[d“_ ^[f\!i ldNsxexZ
¡=“Bh•yOQ•l4 = V

}
y lj i\_ 1221  u_ 1  2  1   ŠGut‰› vC^[a _buxe vC^[aacsxute ŠGumfg^&r@?tr‡u1acs3likjld3_bac^[vxstuxe#&ŠGiM s*
acuxdy_cuxestv|„Guxf\fkutemu_Rjlecstuxe1f ƒ…‰GdGu&ec‰Ga1f ƒ~^[‰N_cacuhZ wd‘ec‰  j[ecu“‰Gu‡fg^(Gf\‰Ge Rut_ci\_cu&ŠNuxe1ŠGux‰›
uxe_uxdy_bi\Žxacutr‡utdy_f ƒ…ikdy_csxaci\ux‰Ga ŠGu fk^lac^[dGŠGuhZ = jldy_cacuxac ‰Ÿƒ~i\f u Ni\e_cu ‰Gd6‰GdGi“‰GuRjlikdy_ŠNjldy_
fkuxe ac3u Gacstecuxdy_b^h_bi\jldGewec‰Na fkute ŠGut‰› vx^[a_cuxe vtj  dGvtikŠGutdy_CZ 4 d ŠGj[dGdGuxa ‰GdGu vtjldGe_cac‰Gv_bi\jl•d fg^
acŽ [fkuwut_ ^[‰3vtjl@r G^[exZ
¡=“Bh•yOQ•l= ;  š

y jli\_ 12 1 ‰Nd‡_baig^[d lfkui\ecj vxŽxf\uuxd 1 u6_ (ejld-vtuxacvtfkuvxi\acvxj[dGecvtaci\_x]Z  d-dGj[_cu 3f\u7vxuxavxf\u


_|^[ d [uxdy_1^[‰›(ŠGacjliˆ_bute 12 ut_ 1.1 ut_w_b^[ d lutdy_  ikdy_csxaci\ux‰Gauxr&uxdy_CKZ wd dGjh_b)u ;0+
fkute
 jli\dy_beŠGu vxj[d“_b^[vt_ ŠGu ‘J^ lutv 12  11 J acute Ruxvt_ci [uxr&uxdy_CZ 4 d BG%d   ute_ fkuvtuxdy_bau ŠG
u  
ute_ fkuwr&ikfki\ux‰‡ŠGu  ;
  u_ fkuwr&ikf\ikut‰-ŠGu  221  Z
‰Ne_b4i B uxa6f ƒ~3s y^[f\i\_b
s   ‡   ;Z 4 d ŠGsxŠN‰Gika•u  ‰Nu  uxe_3f\u(vxutdy_bacu ŠG‰ vtuxacvtfku i\dGecvtaci\_
 1 221 Z
 e /  m t 4     2 
i j[r‡r&ux d jtjldGe  ^[a‰NdGu Gacj Naciks_bswi\@r Rjla_b^[dy_bu ŠNuxevxuxavxf\uxexZ
 ’“JRB  M*=*;
!"$#&%('*)+-,.'0/1#324!"5%*'$6 1
7287  # ' 5:9<; #6=)+-,.'0/1#324!"5%*'$6>6?!%('A@B!@DC(@E5"$).+#&6F6D".#'6?#G+HE$#&IJ#&%('
6K"!%L, 7' 1   2 81‡ 7Q5 1 K5 2 8?9
i tu _c_cu•GacjGaci\st_bs ecu(_cab^[ŠG‰Ni\_6^hikdGei utd _cuxar‡uteŠ ƒ~^[d lf\uxeŠGuŠGuxr&i4 KŠGacjliˆ_bute65 eci ut_ 5
ecjldy_mŠG‰ r ?xr&u&v[_bs&ŠGu&fg^3ŠGacj[i\_bu7 12 8 ^hfkjlae1f\uxe“‰ ^h_bauRjlikdy_ce 1  2   ut_ 5 ejldy_
vxj vtq vxf\i!“‰Guxeeiut_7eux‰Gf\uxr&uxdy_7ecijld3^ 1   2 ‡ 1 5 2 Neci  ut_I5 ecj[d“_7ŠNu1v[_bste7j Rjlecste
 ^[aab^  Rjla _ -fk^‡ŠGajli\_c< u 7 12 8 R^[fkjlae 1  2 % u_"5 ejldy_ vtj›vq›vtfki ‰Nuxe eiu_ ecut‰Gfkutr‡utdy_
eci jld ^ 1   2 ‡ — ‚ 1 5 2 Z $ ~u Gf\‰Gemutd ^ Ruxfk^[dy_ f\u‡vtuxdy_bauŠG‰ vtuxavxfku‡vxi\acvxj[dGecvtaci\_
 1 2 ?5WRj[d^ 7 ” 1 S” 2 8v‡3>7' 1   2 8.‡3%7Q5 1 J5 2 8 Z
i u_c_c,u Nacj Gaiks_bseik r Gfk8u  utacr&ut_>ŠGGs  ŠGu&+Q^[ikau vxu“‰ ƒ…jld&^ Ruxf\fku fk^ v|„ ^[eecu ^[‰›A^hd lfkute
-ebJ^ lj[ikawv|„Gutacv|„Guta1e‰Ga ‰GdN
u BCl‰Ga
u “‰Guxf\ewecjldy_wf\uxew^[d lfkute s y^h‰› Ÿec‰  Gfkstr‡utdy_|^[i\acux3e  vxj[r
Gfkstr‡utd“_b^[ikauxetZ\ZˆCZ / j[‰Ge7f ƒ…ikfkf\‰Ge_cacutacjldGe J^ luxv1‰GdGu Gaj Gaikst_cswŠGuxe7_baig^[d lfkutexZ
A
B

D B P

M
M NO=PRQRSUTBVXWGYZNO\[]\^_N&P`[acbdOfegVhPiSjVlkN&QfOmN&noN&VlPpOfN&nqXYUTBqXYZNO Théor̀eme d
LestrianglesM AB etM B C

‚
sont inversemen
t semblables.

A
 ’“JRB M*=(X O TI= W
!" ' 1 221 +H%_'$/K" ,h% XE # #' ; 7  7  '0/1!" 6
24!" %('0686K" 'K+
&6 6B+H/8E$#6 @ <'
6 221 7 11o7 1 2
/?#6 2=# @'$"#&IJ#&%(' 9 E !/K6 E #64@B#&/?@E #64@"5/?@ !%6?@&/K" '06 ,.+8'$/K" ,h% XE #6 1   72 ;
 7 1
 ; 23,h6K61#%('
2 ,h/o+H% 2=!G" %('`@B!I8I +H% 9

A
B

Q
R

D B C
P

 ƒ< V*X]9V 5
M

 y jliˆ_~}œf ƒ~i\d“_cuxaecuxv_bi\jld–ŠGuxe3vtuxacvtfkute vtikavxjldGevxai\_be3^h‰› _cacig^hd lfkute ut_ ZŒ^ha


Théor̀eme de Miquel
riangles
M AB etM B C 1   2 ;

inversemen
t semblables.
vxjlf\ikdGsx^[aciˆ_bs ŠNuxe, jli\dy_bejld ^
}  1 ‡N— ‚ 1  # } }   2 ‡N— ‚ 1   }  }- ;
A 1 ‡N— ‚ 2  ;#}
A Œ^[a7vxj vtq vtfkikvti\_cs jld ^ B

} ‡O— ‚ 1  } }   2 N
‡ — ‚ 2  ;#}
O
 1 D

ŠGstŠG‰Giˆ_ }. ; 1 ‡ — ‚ 1  } – ^[a`vxj[dGecs3 ‰Nuxdy_f\uxe“‰ ^h_cacu(Rjlikdy_be 1 ; –}4œejldy_


P

wd
C
r

vxj vtq vxf\!i “‰Guxe3 ŠGjldGv#} ^  ^[a _bi\uxdy_^[‰ vxuxavxf\u1vtikavxjldGevxai\_^[‰6_baig^[dlfku 1 ;&Z 
 f uxe _ i\@r Rjla_b^[dy_1fkjlae“‰ ƒ…jld*vx^[fkvt‰Gfku&ŠGuxe^[d lf\uxemŠGu@+Q^[i\acu&^h__buxdy_cikjld*^[‰› ecildGuxemut_

E
C

f ƒ…jlaci\uxdy_|^h_cikjld ŠGuxe1^[d lfkutew"^ B d ŠGuAdGu  ^hewsxvxaikau`ŠGuxenv|„Gjleuxe  ab^[i\uxewec‰Nawvxuxa _|^[i\dGuxe#BC[‰Gacute


r-^[i\e +Q^[‰Nececute‡e‰Ga`Š ƒ~^[‰N_cacux6e F=ljli\a u›uxavxi\vxu Y€‹ I Z 4 d [sxdGstab^[f jld sxvtaciˆ_Afkuxeacuxfk^h_bi\jldGe
“‰Gi
B D Puissancede P parrapportà :
P A.PB = P C .PD = P E 2 = O P 2 − r2
ecjldy,_ ›ac^[ikute7ec‰Gafg
^ BCl‰Nacu “‰ ƒ~j[d(^AŠGuxeecikdNsxu Rut_jl2d lstaci BGu “‰ ƒ…uxf\fkute auxe_cuxdy,_ ›ac^[ikute e‰Ga7fkute
^[‰N_baux&e BCl‰Nacux3e  jl‰ ^[‰3r‡jli\dG&e “‰GunŠGuxeacutfg^h_cikjldNeeci\r‡i\fg^[i\acuxe acute_cuxdyA_  ac^[ikutexZ
Théor̀eme de Ptoĺ
emée

Šd^h‰N_bau1asxe‰Gf\_b^h_7ik r  j[a_|^hd“_ ec‰Ga7f\uxe7vxutacvxf\uxe7ut_f\ux&e “‰ ^[ŠGaikfk^h_bŽtacuxeute_ fkuwe‰Gi h^[dy_CZ


  ’“JLB ›M =  ™ n@PJRW”’›M ’ = 
!" ' 1221  +h% ).+X,X/K" E ,.'&/?# 9! % , E#" "5%$
%,lE " '&
p6B+H"?,h%('R#('
11  2  *) 1 2 1  ‰ 221 5 1 

,G#B@(
,hE5" '
6K"=#'`6?#+hE$#I #%(' 6K"E #6\2=!"5%('06 1  2  1   6?!%('@ ! @mC(@E " ).+#6+,h%6 @ #' !G/,</1#
- @B#o).+H" 6D." X%"./*# ).+# E$#&60X/?!" 'f#6 7 11 8 #' 7 2  8 ?6 #
@B!+B2=#&%*'21 E," " %('&
/K" #+H/3(+ @ #/1@E$#4 9
†
D
M
Théor̀eme de Mique
LestrianglesM AB etM B C
sont inversemen
t semblables.

A B
O

P
C
r

E
E C

B D Puissancede P parra

 ƒ< V*X]9V 5
P A.PB = P C .PD =

 y jli\_–f ƒ…‰GdGi“‰Gu. jli\dy_9ŠG‰Gfk^[dw_butf]“‰Gu fkute_bacik^[d lf\uxe 1 2 –ut_ ejlikutd“_IŠGi\acutvt_butr‡utdy_


Théor̀eme de Ptoĺ emée
1  1
ecuxrGfg^[Nfkuxe&FQv[ƒ…uxe _> G ŠGi\acu&“‰ ƒ…ikf›u›ike _bu7‰GdGu7eci\r‡i\fkiˆ_b‰GŠGu>ŠGi\acutvt_buutdEljPqy^[dy_ 12 e‰Ga  1  1 IZ
wd ^ 2 1  ‡ 12 1 Š ƒ…jb 2 ‡ 1 2  1   1 Z $mƒ{^[‰›_bacuL ^ha_Ajld ^ 11 ‡
2 1  u_ 1  12 ‡ 1.1 1 6 ŠGj[dGv fkute‡_baig^[dlfkute 11 u_ 1  2 ecjldy_3ecutr Gfk^[Gfkute 
Š ƒ~jb 1 ‡ 1.1  2   1 Za$ƒ~^GacŽtef ƒ…ikdGs3y^[f\i\_bs_cacig^hd l‰Gfk^[ikauŠG^[dGefku_baig^[ d [fku 2 1 j[d3^
1 ) 12 ‰ 2 "^Jluxv1sl^[fkiˆ_bs eiut_ecut‰Gfkutr‡utd“_eifkuxe&Rjli\d“_ce 1  2  ecj[d“_ ^[f\!i ldGsteŠ ^[dNe
vxut_ jlacŠGau[Z 4 d acuxr Gfk^jC^[dy_ 2 8u_ 1  ^[a fkuteAh^[fkut‰Gace  jlN_cuxd“‰Gux3e  jld _cacjla‰ lu f ƒ~i\dGs y^hfki\_cs
ŠGumf ƒ~stdGjldGvts[Z ] ƒ~s3y^[fkiˆ_bsn^&fkikut‰ ei9ut_ eux‰Gf\uxr&uxdy_ eciIjld(^ 1221 ‡`— ‚ 12 ‡`— ‚ 1  1 
ŠGjldGvneciu_ecux‰Nfkuxr&uxdy_ecif\uxe&Rjlikdy_ce 1  2  1   ejldy_vxj vtq vxf\!i “‰GuxeŠ ^[dNevtut_jlaŠGacuhZ 
ŠdGu`^[‰›_bacu GacjGaci\st_bsmikr  j[a_|^hd“_cu “‰GiIŠGstvxjl‰Gf\u ŠGu fk@^ Gauxr&ikŽtacu1uxe_ vx
u “‰ ƒ…jld ^  Ruxf\fku
fg^
G‰Gi\ecec^[dGvxu1Š ƒ…‰G2d  jli\dy&_  ^[a7ab"^   j[a,_ `‰Gd vxuxavxf\u[Z
 ’“JLlB  M*=  mTIO ?C?tD F•l= S ˆTIF U JRO F @&UIDGBB€DGUIU JLB€ A
@
TF–•l=“BE•yW”= 
B

!" ' +H% @B#/1@E # #G' +H% 24!"5%*' ; 9 !" ' +H%*# </1!" 'R#2 ,h6D6,h%('i2 ,h/ ; #' @ !+g2 ,h%(' E$#@B#/1@E #
#% 1 #' 2 -
G#%('K+<#E E #I #%('4@B!% !G%$(+H64 9  E !/K6=E #F2A/1! (R +h" ' ; 1  ; *% Q# D
R2=#&%$ ).+# # ;
A
2
#' # 7 2 ,h6 # E , X/?!" 'f# 9
T

 ƒ< V*X]9V 5 D estl’axe radical de etG .

 y jli\_‰GdGum^[‰N_cacu1ŠGacj[i\_bu ^heceb^hd“_A ^[a-; tu _ vxj[‰  ^[dy_ fku1vxuxavxf\umutd 1 ut_  F=ljli\a&BCl‰Gau7IZ


1 2
C

wd ^ B C

;  11 ‡ 2  11 ‡ 2  1 ‡
D P

‚ ;- 2 
9
] t
u 
e b
_ 
a g
i [
^ [fkuxe ; 11 u_ ; 2 ecj[d“_7ŠNjldGv euxr Gfk^[Gf\uxeF”Théor̀
d M
ut_Š ƒ…jleme
aikuxdyde_b^hMiquel
_bi\jldGe j  j[ecsxuteSI GŠ ƒ…jb
; 1 ;- M‡ AB
Lestriangles  ; 2 B GCecjliˆ_I; 1  ; 2 ‡ ; 1  ;Z
; 1etM 
]u(G aj›ŠN‰Gi\_2; 1  ; 2 uxe _&^ Ruxf\s
sont inversemen t semblables.

X% a;!USUT[|PSVVŠGu ;  ^[a ab^ jla _ ^[‰


vxuxavxf\u: Z y i” uxe _ f\u vtuxdy_bau ŠG‰
A

vxuxavxf\uut_'& ejldAab^Cqljld|yjld  ut‰N_>v|„Gjli


ecika 7 ” ; 8vtjlr&r‡u>ŠGajli\_cu u_u aGaci\r‡uta
A B

fg^
G ‰Gi\ecec^[dGvxu1ŠGu; vtjlr‡r&u
O

P D
C
7 ” ; ‰ & 87 ” ; ‚ & 8 ‡Q ” ; ( ‚ & (  r

E
E C
‹
&S ] ^ b a
Puissancede P parrapportà :
(
  "
( !$# (
B D
P A.PB = P C .PD = P E 2 = O P 2 − r2
Théor̀eme de Ptoĺ
emée
M
Théor̀eme de Miquel
LestrianglesM AB etM B C
sont inversemen
t semblables.

4
A
>d lstdGsxac^[fGjld`‰›_bikf\ikeu ŠGute f\jld l‰Nux‰Gae B

^[!f lstGaci“‰Guxe3 vtu• ‰Niw^Rjl‰GavtjldGecs


O

 ‰NuxdGvtu“‰Gufk^•G‰Gikeeb^[dNvxuute_@ j[eci4
P D
C

_bi [u ei ; uxe&_ f ƒ…u _bstaci\ux‰Ga ŠG"u ut_dGsy^E_bi[u ei ; uxe_&f ƒ…ikdy_bstaci\ux‰GatZ›]u vC^[ef\ikr&i\_cu 1 ‡ 2
r

vxjlaacux>e  j[dGŠ(^[‰3vx^[e j b fk^AŠNacjliˆ_bu 7 ; 1 8 ute__|^[dluxdy_bum^[‰ vxutacvxf\uC ^haf\u _b„GstjlacŽtr‡u1ŠGu1ŒqE


E
E
_b„ ^ [jlac"u  jld aut_cacjl‰ [umŠNikacutvt_cuxr&uxdy_ fk
^ [^hfkux‰NaŠGumfk^ G‰Gikeeb^[dNvxuutd2+”j[dGvt_cikjld ŠGu” ; u_ ŠNu
C

&›Z
B D Puissancede P parrapportà :

]92^ G‰Gikeeb^[dNvxu-ŠŸƒ~‰G™d  j[ikdy_  ^[a1ab^  Rjla _  ‰Gd*vtuxavxfku-^ ‰GdNu‡asxvxi Gac]j “‰Gu&‰N_bi\fkuL59eci>fkute


P A.PB = P C .PD = P E 2 = O P 2 − r2

ŠGacjliˆ_bute 7 12 8>ut_ 7 1  8 ecu vxjl ‰ Ruxdy_utd‰G(d Rjlikdy_ Œ ut_ “‰ ƒ…jld3 ^ ; 1  ; 2 ‡?; 1  ;- F ^Jlutv
Théor̀eme de Ptoĺ
emée

ŠGuxe7fkj[ d l‰Gut‰Gace^[f lsxGa!i “‰GutSe I*G^[fkjlae 1  2  1 ut_  ecjldy_vxj vtq vtfk!i “‰GutexZ
y j[ikuxdy_ ŠGut‰› vxutacvtfkuxe  ut_ (
ŠNu vtuxdy_bauxe-aux>e  utvt_b4i +”e ”  ut@_ ”(-ŠNu
ac^CqljldGeIacute Ruxv_bi +”e &  ut_ &(JP‰GdG1u “‰Gux e
_cikjld8d ^h_c‰Gacutfkfku uxe _–ŠGu eu ŠGuxr‡^[ad
ŠNuxa  ‰Nuxf>uxe _ f ƒ…uxdNecuxrGfkuAŠGute Rjlikdy_be
^Cql^[dy_ r ?xr&&u G‰Gi\ecec^[dGvx,u  ^[a ac^  Rjla_
 vtuxe7ŠGut‰› vxutacvxf\uxet„Z  d ec^[i\_ [a [vtu 
dNj[_bau&dNj[_bau +”j[acr ‰Gf\u`ua Gf\ikvxiˆ_b
u “‰ ƒ…ikf

etƒ{"^ li\_&ŠGu f ƒ…uxdG( euxr Gf\( u ŠNux~e Rjl( ikdy_c&e ; (


[sxai BL^[dy4 _ ( ; ”  ‚ ( &  ‡ ( ;” ( ( ‚ & ( 
ejli\._ ;”  ;” ( ‡ &  & ( ZŸŒ^[a fku
D
_c„Gsxj[acŽxr&uw‚ ŠGu1Œq“_b„ ^ [jlacun‚ jld3r&jldy_bacu
a NORP=Y T XNpQmT -S 1TBY XNO XN N&Q &YZNO “‰Gu*f ƒ…uxdGeuxr Gf\u*ŠGute(_butfk e Rjli\d“_c9e ;
    

ute_ ‰GdGu ŠGajli\_cNu Rux>a  utdGŠGi\vx‰Gfk^[ikaOu 
f ƒ{^€N.u 7 ”  ”('8^  utfks TJYaV<>T ;'PT^CŠGute
D estl’axe
radical
de 1 etG 2.

ŠGux‰›vxutacvtfkuxe ut"_ (PZ y iIŠGux‰›vxutacvtfkuxe ecu vtjl ‰ Ruxdy_wuxdŠGux‰›  j[ikdy_be 1 u_ 2  ^[fkjlae f\ux‰Ga


^E›u-ab^hŠGikvx^[f uxe _ fg^3ŠGajli\_c9u 7 12 8 Z y i ŠGut‰›*vxuxavxf\uxeejldy__b^[ d lutd“_ceutd ‰Gd Rjli\d“_ 1 I^[f\jlace


fkux‰Na7^E›u1ac^[ŠGi\vC^[f ute_fg^_b^[ d lutdy_bunvxjlr&r ‰GdNu “‰Gif\uxees  ^[au[Z
 d ^`fku _b„NsxjlaŽxr&uwec‰G4i [^hd“_e‰Ga7fkuxe7^€Nute ac^[ŠGi\vC^[‰› Z
  ’“JLB  M*= —  ’ JLB  M*= SI= ?ADN¡= ?‡B€D SIO •lDGT9¡ 
!" '  7  7 * '0/?!G" 6
@ #/1@E$#&6 9  $E !G/K6 E #+H/D6 '$/?!"56 ,  #6 /,<" @?,.+   7 ( #G' * 6?!%('\61!" '
@B!% !G%$(+H6 7 6?!" '`@B!%*@ !+H/&,h%('06 7 61!" '323,h/,hE5E&E$#6 9
 

 ƒ< V*X]9V 5
 Š dN j[ikdy_3^ G^[a_cuxd ^[dy_2 ŠGut‰› ^E›uxe3ac^[ŠGikvx^[‰›–^[‰ r‡jli\dGe6^ r ?xr&u G‰Gi\ecec^[dGvxuW ^[a
ab^ Rjla _1^[‰› _cacjli\ewvxuxavxf\uxeŠNjldGvAikf ^"  ^[a _bikutdy_1^[‰‘_cacjli\eci\Žxr&u`^E›u[Z‹$ j[dGv‡ejli\_  ut_ (
ecjldy_1vxjlad +”jldGŠG‰Genu_nfku`ecj[d“_1ŠGj[dGvA^Jluxv *7 ecj[i\_wi\fke jldy_1‰Gd ecut‰Gf Rjlikdy_nŠ ƒ…ikdy_butaceuxvt_cikjld u_
 

ikfkevtjl ‰ Ruxdy_ ŠNjldGv * uxdvtut_ ‰GdGi ‰NuRjlikdy_J ejli\_i\fkeecj[d“_ ^[ac^[fkf\Žxfkuteut_ * fkut‰Ga7uxe _ ŠGjldNv


 ^[ab^hfkfkŽtfkuhZ 
 

Šd acstec‰Gfˆ_|^h_ik r Rjla_b^[dy_ Rjl‰GaIfg^acstecjlf\‰N_bi\jldmŠŸƒ~u Nutacvtikvxute uxe _fku ec‰Ni h^[dy_ 5Eecjli\uxdy_ 1221 
ut_ 1 
ŠGux‰›6 “‰ ^[ŠNacikfk^h_bŽtacute i\dGecvtaci\_ce7Š ^[dGe ŠGux‰›vxutacvtfkux3e  ei9fkuxe ŠGacj[i\_but e 7 12 8* 7 1  8*
7 8 ecj[d“_vtjldGvxj[‰Gab^[dy_cuxe ^[fkjlaefkut& e  ‰G^h_bau Rjli\d“_ce 1  2  u_ ecjldy_vxj vtq vtfk!i “‰GutexZ

 


Œ&mkl  r xu‹‘’Kt
¡ =“Bh•yOQ•l= ;R;

y jli\_ (fkuvxutacvtfku vtikavxjldGevxai\_ ^[‰‡_cacik^[d lf\us3“‰Gikfk^h_bstab^[f 1 221 Z y j[i\_ ‰GdRjli\d“_ ŠGu f ƒ~^[acv
Š ƒ~u  _bacstr‡iˆ_bste 2 u_ 1 dNu1vtjldy_butd ^[dy_, ^[e 1 Z = j[d“_cacuta,“‰ ƒ…jld ^ 1  ‡ 2  ‰ 1 Z
¡=“Bh•yOQ•l= ;›X

y jli\_ 1 u_ 2 f\uxe i\d“_cuxaecuxv_bi\jldGe>ŠGuŠGut‰›&vxutacvxf\uxe ut_ (CZ y jliˆ_ 1  ‰NdGu7vxjlaŠGu7ŠGuI u_


ut_ f\uxe ecutvxjldNŠGuxe ikdy_butaceuxvt_cikjldNe acux>e  utvt_cilute ŠNuxe>ŠGacj[i\_bute 11 u_ 2 "^JluxvI (PZ = j[d“_cacuta
 ‰Nu1f\uxe7ŠGajli\_cuxe 7 1  8 u2_ 7 8 ecjldy,_  ^[ac^[fkf\Žxf\uxexZ



¡=“Bh•yOQ•l= ;›™ 6B€J O @P= SI= žOQM*U?xJRF 

y jli\_ (‰Gd‡vxutacvxf\u u_ 1  2  1 _bacj[ik.e  j[ikdy_be>ŠG-u Z y jli\_ Œ ‰G d Rjlikdy_ ŠG‰ Nfg^[%d  ; J; J;


ecux&e Nacj utvt_bi\jldGee‰GafkuteŠGacj[i\_bute 7 2 1 8' 7 1 1 8' 7 1 2 8%Z = jldy_baux&a “‰Guwfkut&e  j[ikdy_b0e ; J; J;
 

ecjldy_ ^[fki ldGsteeciu_ eux‰Gf\uxr&uxdy_eci ; "^   ^[a _bikutdy,_  Z


 

¡=“Bh•yOQ•l= ;“

y jli\_ wu _ (1ŠGut‰› vxuxavxf\uxe1ecu&vxjl ‰ G^[dy_^h‰›W   j[ikdy_be 1 ut_ 2 Z y j[i\_ ‰GdGuA_|^h d luxdy_cu
vxjlr&r ‰GdG~u  mu+_ (7 1 u_ 5fkut
e  j[ikdy_be ŠNu6vxjldy_|^hvt_beAŠGu J^ lutv mut)_ (PZ y jliˆ _ 


f ƒ…ikdy_butacecutvt_cikjld6ŠGute ŠNacjliˆ_buxe 7 12 8u _ 7 1  8* r&jldy_baux&a “‰ ƒ…jld ^  1 ‡Q


Z


¡=“Bh•yOQ•l= ;›—

y jli\_ 1221 œ‰Gd “‰ ^hŠGaci\fg^h_cŽxacu1vxj[0d [u›uAikdGevxai\_ ŠG^[dGe ‰Nd vxuxavxf\u ŠGuvtuxdy_bau ”‡Z y jliˆ_ ;
fku Rjli\d“_ Š ƒ…ikdy_butacecutvt_cikjld6ŠG)u 7 11 8 u_ 7 2 <8Z›]9utevxutacvxf\uxevtikacvtjldGevxaciˆ_be ^[‰›‡_bacik^[ d lf\uxe 12 ;
ut_ 1 <; ecuwauxvxj[ ‰  utdy_7uxJd AZ y |i ”AJ;0 ecj[d“_ŠGike _bikdNvt_b3e Gajl‰ [uxa “‰Gu 7 ” 8 uxe &_ Ruxa Rux]d
ŠGikvt‰Gfg^[i\acu 47Q;  8Z
¡=“Bh•yOQ•l= ; 3

y jli\_ 1 B 2 B 1 B ‰Gd_baig^[ d [fkuut_ ; ‰G~ d Rjlikdy_x Z  d6ŠG*s B dGi\_ 1 * 2 * 1  vxjlr&r‡u fkutce Gaj uxv
_bikj[dGe j[a_b„Nj ljld ^hfkuxe ŠGu Œ e‰Ga f\uxe ŠGacjliˆ_but"e 7 2 B 1 B'8*@7 1 B 1 B8*@7 1 B 2 B8 Z  dŠN3s B dGiˆ_ŠGu @r ?tr‡u
fkuxe_cacig^h d lfkute 1 ( 2 ( 1 ( ut_ 1 * 2 * 1 *PZ = jldy_cacux,a “‰Guf\uxe_baig^[d lfkute 1 B 2 B 1 But_ 1 * 2 * 1 * ejldy_
ecuxrGfg^[NfkuxetZ
¡=“Bh•yOQ•l= ;GM ¢

y jli\-_ * (M *7 “‰ ^h_cacumvxuxavxf\uxex|Z  de ‰  Rjlec


u “‰Gu ut_ ( ecumvxjl‰  utd“_ uxd ;u_
) ‹“‰GA u (ut _ *eu‡vtjl ‰ Ruxdy_muxd ;,(mu_ (7K“‰GAu *mut _ eu‡vtjl ‰ Ruxdy_utd ;,*ut_ *u_


 ‰N)u nut_ ecu`vtjl ‰ Ruxdy_wutd ; nut_  €Z = jldy_bauxa “‰Gu`eifkute Rjlikdy_c"e ;O @;C(J@;C*J@; nejldy_


vxj vtq vxf\!i “‰Gux3e L^[fkj[acefkut&e  j[ikdy_be 2*(7 *J  fkuwecj[d“_3s y^[fkutr‡utdy_CZ
   



¡=“Bh•yOQ•l= ; U

y jli\_ 1 221 ‰Gd _cacig^h d lfku “‰Guxf\vxjl d “‰Gu&ut_ 5ejld‘jla_c„Gj›vtuxdy_bau[Z y jlikutd“_nŠNux‰› Rjli\d“_ce 
ut_ 5 Gaike aux>e  utvt_b4i lutr‡utd“_1e‰Gawfkutewv [_csxe 12 u_ 11 Z]uxewvtuxacvtfkutewŠGu`ŠGig^hr‡Ž_bacute 2 55u_

1  ecunvxj[ ‰  utdy_ ut4d ; u_ & Z = jldy_bacutAa “‰Gunf\uxe  j[ikdy_be ;0+ œecj[d“_ ^[f\!i ldNsxexZ

¡=“Bh•yOQ•l= ; V  ; V VL— 

y jli\_ 1  2  1    ‰G^h_bau Rjlikdy_ce ŠGi\e_cikdGv_be Gfg^hvxsxeŠ ^[dNe vtut_ jlacŠGaume‰Ga‰GdGuŠNacjliˆ_bu[ZL]ute




vxuxavxf\uxeŠGu ŠNig^[r&Žt_cacuxe 11 u_ 2  euwvxjl ‰ Ruxdy_7utd ut_ -ZN]9^ ŠGacjliˆ_b2u 7 8 vxjl‰  +u 7 221 8
uxd 1Z y jli\_ ; ‰Nd Rjlikdy_1ŠGike _bi\dGvt_nŠGu
ec‰Ga1fg^3ŠGacj[i\_bu 7 28 ZI]I^3ŠGajli\_cu 7 1 ; 8 vxj[ ‰  u&fku
  
  

˜
vxuxavxf\uŠNuŠNig^[r&Žt_cacu 11 utd 1 u_c “ut_ fg^wŠGajli\_cu7 2 ; 8vxjl‰ ufkuvtuxacvtfku7ŠGuŠGig^hr‡Ž_bacu 2 
uxd 2 u-_ 5 ZLŒ acjla‰ luxa,“‰Gu1fkuxeŠNacjliˆ_buxe 1   <5™ ecjldy_vxjldNvxjl‰Gac^[dy_butexZ  

¡=“Bh•yOQ•l=XL š  ’“JLlB  M*=SI=  D ?C•lD W

y jli\_ 1 221  ‰Gd „GuN^ lj[dGuAikdGevxaciˆ_wŠ ^[dNew‰Gd vxuxavxf\u[Z y jli\uxdy_     f\uxewikdy_cuxaceuxv


_bikj[dGe auxe Ruxv_b4i luxeŠNuxeŠGajli\_cux"e 7 12 8 ut_ 7 28  7 221 8>ut_7 8*@7 1  8 u_7 1 8 Z = j[d“_cacuta


 ‰Nu&f\ux#e Rjlikdy_ce     ejldy_m^[fki ldGsxetZ FQŒjl‰Ganf\ux#e Gf\‰Gewvxjl‰Nab^lut‰› r‡j[d“_cacutawfku`r ?xr&u`acs*


 

ec‰Gfˆ_|^h_ Rjl‰Ga 1  2  1  4  ec‰Ga7‰GdGunvtjldG!i “‰Gu “‰Gutfkvxj[ d “‰Gu-I





 e /  m t 4   / fJ12 


i j[r‡r&ux d jtjldG#e  ^ha “‰Guxf  ‰Nuxenab^  utfkex%Z  d‘fk^[ikeecutab^-fk^ŠGsxr&jldGe _bab^E_bikj[d(ŠGux#e Nacuxr&ikŽtacute
+”jlacr ‰Nfkuxe jl‰3ŠGunf ƒ…u›ike _buxdNvxunŠGuxe Rjlikdy_cevxi\_csxeutd u NutacvtikvxuhZ  d i\d“_cacj ŠG‰Gi\ab^u_7‰N_bi\fkikeuxac^fkute
dGj[_|^E_bikj[dGe‰Ge‰Guxf\fkuxetZ
$ ^[dGe ‰Nd _baig^[d lfku 1 221 Ljld dGj[_cu   wfkute7fkjl d [‰Gux‰Gae ŠNuxe v [_bste j   j[ecsxe ^[‰› ejlr
r‡u_ben„Gjlr&jldyq r‡ute1u_   OK f\ux#e h^[fkut‰GacenŠGute^h d lfkutexZ  d*dNj[_bu  fku`vtuxdy_bau-ŠG u labJ^  i\_cs 
 jli\dy_>ŠGu vtjldGvtjl‰Gace ŠGute r&sxŠGik^[dGute yfku Rjlikdy_  ŠG4i  ikecu7v|„ "^  ‰Nu r&sxŠGik^[dGuuxd‡ŠGux‰ &ecu [r‡utd“_ce
ŠGjldy_>fkuac^  Rjla_>ŠNuxe fkj[ d l‰Gut‰Gacve h^[‰N_  0Z  d&dNj[_bu ”f\uvxutdy_bacu7ŠG‰&vxutacvtfkuvxi\acvtjldGecvtaciˆJ_ Rjli\d“_
ŠGu vxj[dGvxjl‰Nace ŠGutewr‡stŠGig^E_baci\vxute  f ƒ…jla _b„Gj vxutd“_cac"u |Rjlikdy_ ŠGu vxj[dGvxjl‰NacewŠGute „ ^[‰N_cux‰Gae  u_ 
fku1vxuxdy_cacu ŠG‰vxuxavxf\uikdNecvxai\_  Rjli\d“_ ŠGumvxjldGvtjl‰Gae ŠGuxe Gikeecutvt_baikvtuxe i\d“_csxaikux‰Nacuxet%Z  d(dNj[_b u

fku7ŠGuxr&4i  staci\r‡Ž_bac"u [ejli\._ 7  ‰  ‰ 8  nu_ y f ƒ~^[ikauŠG‰‡_cacik^[ d lf\u[0Z  d&dGj[_bu u_ &wf\uxe>ab^Cqlj[dGe
acux>e  utvt_ci +”e7ŠGuxe7vtuxacvtfkute vtikacvtjldGevxaciˆ_beu_ i\dGecvtaciˆ_bexZ
 d ^`fkuxeauxfk^h_bi\jldGee‰Gi h^[dy_bute 5
]9j[i ŠGuxeeci\d“‰Ge   

‡ ‡ ‡ H †‡
         
 [j acr ‰Gf\u ŠGu stacjld
™‡
&‡
,7
 87
87
8
‚ ‚ ‚
D„GsxjlaŽxr&unŠ ƒ5 f> n^[ec„Gi
 (
‡? ( ‰  (
‚   "! 
 j[acr ‰Gf\u Š 5ƒ 4 ‰Gfkuta
”  (
‡  ( ‚ h&
$ acjliˆ_buwŠ 5ƒ 4 ‰Gfkuta
”‚'‚ … ‡ ”‚0… 

Y$#
A

I G O
H

`VcPRQRSUTBUnVXWGYZtriangle
N]\^ob ABTkCN avecses
OfNO quatre
TBPRQfN.poin
egSjVhtscen
PfO N&Vltraux
PRQmT principaux
XQRSjV &ST 

 jPq[jldGer-^[i\dy_buxdG^[dy_ ‰Nuxf!“‰GuteANacjGaiks_bsxeŠNuxe, jli\dy_beacutr-^ha“‰ ^[Gf\uxetZ


A
QB
A

 ’“JRB  M*9 = 3


!" '  E$# 2A" #  .#cE H, h"56K61#B@'$/K" @B#c"5%('
/D"$#G+H/?# " 6D6g+# # 1  ,l%6 E # $' /K" ,h% XEH# 1221 - @ " #&6g'

1
<"5/?# E#" "5%('f#/D6?# @'0" !% # E , h"56K6?# @'$/K"$@ # ,G#B@ E , </1!" 'R# C 7 221 P8 4J#' 16 !" ' E$# 2*"$#  # E ,
B B

h" 6D6?#B@G'0/D"$@B#o#3'&
/D"$#+h/?# " 6D6g+# # 1
7 !G% , ,hE !/K6pE#"
,hE5" '
p6g+H" ?,h%('f# '
A

B
QC HC
 2 2 1 2
‡ ‡
G O

H  1 1 1 H C1
. H

 ƒ< V*X]9V 5
PB
B

 4›utacvxi\vxuhZ
HA

B PC
C HA
QA A

Le triangle
orthique:
lestrianglesAH B H C ,H A B H C ,H A H B C
sont semblables
au triangle
AB C ,
entreeuxetinversemen t semblables

lespointsQ A ,Q B ,Q C sont lessymétriques


YlY Le cercle
d’Euler
ou cercle
desneufpoints
de H parrapportaux côtés.
A

 ’“JRB M*= ¢


!" ' 1221 +H% '0/K" ,h% XE #p"5%6?@&/K" '  ,h%.6 E$# @ #/?@&E$# # @B#&%('0/?# ” 9 !" '  E$# @ #%('0/1# (+ @B#/1@E #
" %61@/K" ' 7  E , 6?# @B!%$.# " %('R#/K61#B@G'0"$!G% .# E , l" 6K61#B@G'0/K" @B# " %('&
/K" #+H/1# " 6K6B+# # 1 , G#B@ 9
 E$!G/K6
H9 E$#&6 </1!" 'R#6 7  ” 8 #' 7 221 8 6?!G%*'24#/ 2=#&%$<" @+HE ,h" /1#6 - !%*@  #&6g'3E$#4I "5E " #+ # E," ,h/?@
 ; 47
 9 E$# @ #/?@&E$# #o@B#&%('0/?#  2 ,l6K6,l%*'F2 ,h/ 2 #' 1 23,h6K61#+
,hE$#&IJ#&%(' 2 ,h/ E$#`24!"5%*'  9
 ƒ< V*X]9V 5
 4›utacvxi\vxu 
I G O
H

 ’ JLB M*= U  BEO”D FIRW”= JLBP@  O ŸTI=


!" '     E$#&6\2A" # <6 #6 ,.+'R#+H/D6 " 6D6g+#&6 /?#&6 24#B@G'0"G#I #%('3#&6 6?!GI I #'$6 1  2  1
 " +h%'0/D" ,h% XE$# ,<@G+' ,l% XE$# - !%(' 'R!B+H6E$#&6 ,h% XE$#&661!%('\,l". +H64 7 61!" ' E," !/g'*! @ #%('0/1# #@ #
'0/K" ,h % XE # 7 !% ,8,lE$!/D6
H9 E$#6 '$/K" ,l % XE$#&6 1   7 2  7  1 6?!%('(<" /1#B@G'f#I #%(' 6?#&I hE , hE$#&6 #%('0/1#
#+  #G'=" $% <"5/?# @'f#&IJ#&%('46?#I hE , lE$#6 , + '$/K" ,h% X E # 1221 7
 9 E$#&6  ,.+'R#+H/D6 (+o'0/K" ,h % XE # 1 221 6?!G%*'E #6 l" 6K61#B@G'0/K" @B#&6F"5%('
/D"$#G+H/?#&6 (+o'0/K" ,h% XE #   7
,B224#E
_cacik^[d lf\u jla_c„Gi!“‰Gu 7

<9 E #6 6fC.I
'$/K"$).+#&6 # 2 ,h/8/,B224!/B' , +  '0/1!" 6 @ <'
6 (+ '0/K" ,h % XE # 1221 ,B2.2 ,h/

'0"$#&%%*#&%*' ,.+ @ #/?@&E$#o@"5/?@ !%6?@&/K" ' 1 1 221 9 Un triangle AB C avecsesquatrepointscentrauxprincipaux

A
A
QB

HB PA HB
C

QC HC
G O
H HC H

PB
B HA B PC
C HA
QA A

 ƒ< V*X]9V 5
Le triangle
orthique:

 4›utacvxi\vxu
lestrianglesAH B H C ,H A B H C ,H A H B C
sont semblables
au triangle
AB C ,
entreeuxetinversemen t semblables

lespointsQ A ,Q B ,Q C sont lessymétriques Le cercle
d’Euler
ou cercl
de H parrapportaux côtés.

Y€o
H

B
  ’“JLB›M = V  m=“BE•yWQ= S' TW”=“B 
!" ' 1   2   1  E$#6 I8" E5"$#+  /1#6 2=# @'0" 6 .#6 @ '&
6 221  11  12 7     E$#&6 2*"$# X6
#6 ,.+'f#G+H/K6\" 6K6B+#63# 1  2  1 #' ; K; J; E$#&6\I8" E " #+c/?#6 2=# @'$" 63.# 1  2  1 9
 E$!G/K6cE$#&6c%*#+ p24!" %('06 1   2   1       J; K; J; ,B2.2 ,h/g'$"$#&%%*#%(' 1 +H%I&IJ#
@B#/1@E # , 22=#&E
vtuxacvtfkunŠ ƒ54 ‰Gfkuta 7 #o@B#&%('0/?# I8" E5"$#G+ (+ 6?#XIJ#&%('  ”  #' # /,?C.!%   .9
Un triangle
AB C avecsesquatrepointscentrauxprincipaux

A
A
QB

HB PA HB
C

B
G O
HC H

PB
B PC
C HA

thique:
Hs B H C ,H A B H C ,H A H B C
les
entreeuxetinversemen t semblables
C,
 ƒ< V*X]9V 5
 y jliˆ_ f\u r&ikfki\ux‰‡ŠGu  ”  % f\uwvxutacvxf\uwŠGuwvxutdy_bacu  ut_7ŠGuwac^Pq[jld   ›Z y jli\_3f ƒ…„Gjlr&j"
,Q B ,Q C sont lessymétriques Le cercle
d’Euler
ou cercle
desneufpoints

_b„Gs_bikuŠGu vxuxdy_cacu  ut_ ŠGu ac^ Rjla_ ‚-,  ›Z 4 fkfkuuxdE[jliku”utd  ›ŠNjldGv f\u vtuxavxfku vxi\acvxj[dGecvtaci\_
portaux côtés. 

 1221 e ‰Ga Z 4 fkfku uxdEljli\u 1 uxd 1   2 utd 2   1 utd 1  GŠGjldGv 1   2   1  ^  ^[a _bi\uxdGdGutdy_
 Z y j[i\_  f ƒ…„Gjlr&j[_b„Nst_bi\u ŠGuAvxutdy_bacu ut_1ŠGuAab^"  j[a_  ] uxf\fku utd0[jliku@” uxd ŠGjldNv
fku&vxuxavxf\u‡vtikacvtjldGevxaciˆ_
 1221 ec‰Na2 Z4>f\fkuAuxdE[jliku 1 uxd ;,  2 uxd ;  1 uxd ; –IŠGjldNv
 

^   ^[a _bi\uxdGdGutdy_  Z y j[i\_  fg^ŠNacjliˆ_bu


 ^hab^[f\fkŽxf\u
7 1 8 ^heceb^hd“_  ^[a Z
 

4>f\fku3uxe 
; J; J;
_ Rux>a  utdGŠGi\vx‰Gfk^[ika6u  7 221 8*>ŠGj[dGv6G^[ab^[f\fkŽtfkuL 7 1  ”28Z i jlr&r&u uxe_-fku3r&ikfki\ux‰




ŠGu  ”    fk^-ŠGacj[i\_bu  ute_nfk^r‡stŠGig^h_caci\vxuŠN/u  1    ŠGjldNv`jld‘^ ‡ 1  ‡   a


ŠGjldGv ^  G^[a_cikuxdy,_ + Z  d r&jldy_bauwŠGun@r ?tr‡#u “‰Gu u_ ^  G^[a_cikuxdNdGuxdy_A+ >Z0
 
 

]ute ŠGut‰› _b„GstjlacŽtr‡ute ec‰Ni h^[dy_be ecjldy_ _cacŽte ‰N_cikf\ux,e  jl‰Na r&jldy_bauxa “‰Gu_bajlike Rjlikdy_ce ejldy_
^[fki ldGsxe jlL‰  ‰Nu1_cacjli\eŠGacjliˆ_buteecjldy_vxjldNvxjl‰Gac^[dy_bute jlL‰  ^hab^[f\fkŽxf\uxetZ
 ’ JLB  M*= ;“š  ’ F9’ WQD I? 

!" ' 1 221 +H% '0/K" ,h% XE #4#' ;0+ $' /?!"56(2=!"5%('06(2A/K"56>/?#6 2=# @'$"#&IJ#&%('(6B+H/ E #6 </1!" 'R#6 7 221 8 7
7 1 1 8 7 7 12 8 9 E !/K6 E #6i24!" %('$6 ; &  6?!G%*' ,hE5".X%
6 6K"#G'46?#G+HE$#&IJ#&%*'=6D"=!G% ,
2 ; 1  1 
; 1  1  2
‡ ‚-,
YPz
!cE$#6 E !%  +#G+H/K6 61!%('\@B!%6D"
&/,
#6 ,hE.
h/K" ).+#I # %(' 9

B
P

A
Q C

Le théor̀eme de Meneläus
 ƒ< V*X]9V 5
 y jli\_  f ƒ…ikdy_butaceuxvt_cikjld ŠGute‡ŠNacjliˆ_buxe  tu _47 1 2 8*v} f ƒ…ikdy_butaceuxvt_cikjld ŠGu3fk^ G ^[ab^[f\fkŽtfku
^[eeb^[dy_A ^[a 1 u _7ŠGuA7 12 8ZLŒ^haf\u _b„GstjlacŽtr‡uwŠNu D„ ^hfkŽxe3Gjld ^
7Q; 8
7Q;  8c
2 ;
‡ 2–}  
1  ‡ }  
; 1  1  1
Š ƒ~jb 2 ;  1 2 
B
}  1  ‡
1
; 1  1  2
‡ –}  1  2 ‚,
Œ^[avtjldGecs3“‰Guxdy_ fkuxe&Rjli\d“_ce0;0+ ecjldy_ ^[fkildGste eciŸut_7ecut‰Gfkutr‡utdy_ ei jld ^  ‡   Gejli\_
†‡ Z   
R

’“JLB›M = ;L; m=GD 


P
  

'0/K" ,h% XE #4#' ;0+ '$/?!"56(2=!"5%('06(2A/K"56>/?#6 2=# @'$"#&IJ#&%('(6B+H/ E #6 </1!" 'R#6 7 221 8 7
T
!" ' 1 221 +H%
7 1 1 8 7 7 12 8 9  E$!/D6pE$#&6 </?!G" 'f#&6 7 1 2 8 7 7 2  8 7 7 1 8 61!%('`@B!%*@ !+H/&,h%('f#66K"F#'46?#G+HE$#&IJ#&%('
6K"!%L, '
YC}
A
Q C
A
Q C

 ‡2 ; 1 1 
  ; 1
Le théor̀1 2 Menelä
eme de , us
!cE$#6 E !%  +#G+H/K6 61!%('\@B!%6D"
&/,
# 6 h, E.
h/K" .) +#I #%(' 9

R
P

A
Q C

 ƒ< V*X]9V 5
 y jli\_.}–f\u,Rjli\d“_ Š ƒ…ikdy_cuxaceuxv_bikj[d-ŠGute GŠ ajli\_cuxe-7 1 ; 8ut_7 2  8  ejli\_ * f\u,Rjli\d“_ Š ƒ…ikdy_cuxaceuxv
Le théor̀eme de Ceva.

_bikj[d6ŠGuxe7ŠGajli\_cuxe7 1 }.8 ut_ 7 2 1 8 Z ]u1ac^ Rjla_ŠGuxe7fkj[d l‰Gut‰Gace ute_sl^[f% ^[aeikr&ikf\i\_b‰NŠGu 

ŠGu&_cacik^[ d lf\uxew^[‰ ab^  jla _nŠGuxe1ŠGi\e _|^hdGvxutemŠGu 2 u_ 1  fk^3ŠNacjliˆ_bu7 1 ; 8 fku“‰Gutf uxe _ ^[f\jlace 

s y^[f ^[‰ ac^  Rjla_ŠGuxeA^hikacute   F”vxute _bacik^[d lf\uxe1jldy_


Rjl‰Gam ^[ecu 1 }8ut_
Rjl‰Gam„ ^[‰N_cux‰Gae


fkuxeŠGi\e_|^hdGvxute ŠGu 2 ut_ 1 -fk^&ŠNacjliˆ_b&u 7 1 ; 8I Z 4>d‰N_bi\fkikec^[dy_fku1@r ?tr‡um^[a[‰Gr‡utdy_Rjl‰Gafkute




^[‰N_bauxe7ab"^   j[aJ_ Njld3jlN_bi\uxdy_


2 ; 1  1 
‡ 1 } 2  2 } 1  1 } 1 
‡
; 1  1  2 1 } 1  1 } 2  2 } 1  ,


w d ^Gfki ‰Nu6^[fkj[ace f\ur@?tr‡u^[al‰Nr‡utd“_ “‰GuLRjl‰Ga fk^(ŠGsxr&jldGe _bac^h_bi\jld ŠG‰ _b„GstjlacŽtr‡u6ŠNu


= uxdGutfg^ NexZ ]uxe‡_cacjli\eAŠGacjliˆ_bute47 1 ; 8 7 2  8 7 1 8 vxj[dGvxjl‰Nacuxdy_-eci7ut_‡ecut‰Gfkutr‡utd“_-eci7jldV^
†‡ Gejli\_  ‡    ŠGjldGvneciu_ecux‰Nfkuxr&uxdy_ecij[d ^
 
2 ; 1  1 
; 1  1  2
‡ ,

 f[u›ike _bu>‰GdGuvh^[aig^[dy_bu_baciljldGj[r‡s_baci“‰GuŠG‰w_c„GsxjlaŽxr&u ŠGu i 3u h^7ŠGjldy_9fg^ŠNsxr&jldGe_cab^h_cikjld
uxe_eikr&ikfg^hikacuhZ
Y€‚
 ’“JRB M*=*;›X m=GD @PBEOQLJRFIJRM*’y@PBEO T9= 


!" ' 1 221 +H% '0/K" ,h% XE #4#' ;0+ '$/?!"56(2=!"5%('06(2A/K"56>/?#6 2=# @'$"#&IJ#&%('(6B+H/ E #6 </1!" 'R#6 7 221 8 7
7 1 1 8 7 7 12 8 9  E$!/D6pE$#&6 </?!G" 'f#&6 7 1 2 8 7 7 2  8 7 7 1 8 61!%('`@B!%*@ !+H/&,h%('f#66K"F#'46?#G+HE$#&IJ#&%('
6K"!%L,
  
11 ;   1 2    22 1 
 ‡
    11

  ; 1 2
    221 ,

,mkl
  r xu‹‘’Kt
¡=“Bh•yOQ•l=X9;


$ sxr&jldy_bacuta7fkuxe_cacjli\e_b„GstjlacŽtr‡ute7dGjld ŠGsxr&jldy_basxe7ŠG‰ vxjl‰GaexZ
¡=“Bh•yOQ•l=XRX

y jlikutd“A_ ut&_ (&ŠGut‰› vxutacvxf\uxe&ŠGu ab^CqljldNe‡auxeRuxv_bi4+”e & ut_ &(‡ecu vxjl‰ ^[dy_&uxd 1 u_
2  ‰NdGu ŠGajli\_cu “‰Gutfkvtjl d “‰G
u  ^heceb^hd“_  ^[a 1 Z y jlikutdy_ 1 ut_  fkuteARjli\d“_ce ŠŸƒ~i\d“_cuxaecuxv_bi\jld
acuxe> utvt_ci +”eŠGu 7^ [uxv  ut._ (CZ i ^[fkvt‰Gfkutafkunab"^   j[a_ 221"2  utd6+”jldGv_bi\jld3‰GdGi!“‰Gutr‡utdy_


ŠGu & u_ &(PZ




¡=“Bh•yOQ•l=XR™

y jli\_ 1221 ‰Gd _bacik^[ d lf\unut"_ ;0+$_cacjli\Ae Rjli\d“_cAe Nacikeacute Ruxv_bi [uxr&uxdy_we‰Ga f\uxe ŠNacjliˆ_buxe
27 21 8*,7 11 8*C7 1 2 8Z  de ‰  Rjlecu “‰Guf\uxe_bajlikeŠNacjliˆ_buxe 1 ;  2   1 ecu1vxjl ‰ Ruxdy_ ux6d }
r‡jldy_cacux&a “‰ ƒ…jld ^A^[fkjlae }-; }  } 
1 ; ‰ 2  ‰ 1  ‡ ,


¡=“Bh•yOQ•l=X  m=“Bh•yW”= S  U JRW W”JRFOQTI?

lj i\_ 1 u_ 2 ŠGux‰  Rjlikdy_ce u_ ‰NdacstuxfIŠGiM sxauxdy_ ŠGu , ZM= j[d“_cacuta“‰Guf ƒ~utdGecutr Gf\umŠGute
 

y
 jli\dy_be ŠN‰™Gfk^[d _butfke“‰Gu‡f\u`ab^ jla _ 1  "2  ecj[i\_nsl^[f. ute_m‰Gd vtuxavxfku&ŠGjldy_1fku


vxuxdy_cacum^  ^[a _bi\uxdy_,`fg^ ŠGajli\_cu 7 12 8Z




¡=“Bh•yOQ•l=XR—  JROQF @€?`S9= DNR= W =y@‡S9= 6=“B€LJ FIF9= 

y jli\_ 1 221 ‰Gd6_baig^[d lfk"u Necj[ikuxdyI_ ;0+ fkuxe&Rjli\d“_ceŠGunvxjldy_b^[vt_7ŠG‰ vxutacvtfkunikdGevxai\_7^Jlutv


 

fkuxev [_bste 221  11  12 NecjliˆI_ ;  f\#u  jli\dy_7ŠGunvxjldy_|^hvt_ŠG‰3vxutacvxf\unu›ikdGevxai\_7Š ^[dNe f ƒ{^[dlfku 1
^[‰_baig^[d lfkumJ^ luxv`f\u v h_bs 221 FQf\u vxutacvtfku u Ni\dGecvtaciˆ_ Š ^[dNe f ƒ~^[ d lf\u 1 uxe_nf\uvtuxacvtfku_b^[d lutdy_
^[‰› _bacj[ikev [_bste`ŠG‰ _baig^[ d [fku 9r‡^[ikemecu6ei\_b‰G^[dy_ ŠGu6f ƒ{^[‰›_bacu‡v [_cs6ŠGufk^ŠGacj[i\_bu 7 221 8  ^[a
ab^  Rjla _^[‰6_bacik^[ d lf\7u I CZ  d3ŠG3s B dNi\_ŠGun@r ?tr‡u fkuxe Rjlikdy_ce   ut_   Z
;  = j[d“_cacut#a  ‰Nu`fkuxe ŠGacjliˆ_but+e 7 1 ; 8' 7 2  8 7 1 8ecj[d“_nvtjldGvtjl‰Gab^hd“_cux@e F Rjli\d“_ ŠGu wuxka
ljldGdGJu IZ

$X  = jldy_bauxa “‰Gu1fkuxe7ŠGajli\_cuxe 7 1 ;  8 7 2   8' 7 1   8ecj[d“_vtjldGvxj[‰Gab^[dy_cuxe F  j[ikdy_7ŠGu / ^


luxf GdGj[_cs 5 IZ
$™  = jldy_bacuta “‰Gu f\uxAe  jli\dy_be    J5 ecjldy_ ^[fki ldGsxeŠ ^[dNe vxu_wjlaŠGacumut_ “‰ ƒ…jld A^ 5  ‡
   Z
!  y jlikutd“_ 1   2   1 lf\uxer&ikf\ikux‰ nŠGuxe v [_bste ŠGu 1 221 Z = jldy_bauxa “‰Guf\uvxutdy_bacuŠN‰`vxuxavxf\u
ikdGevxaciˆ_Š ^[dNe 1  2  1  uxe _fkuwr&ikfki\ux‰ŠN‰ eu lr&uxdy_   5  Z
YP†
¡=“Bh•yOQ•l=X 3

y lj i\_  i $ ‰GdO ^[ac^[fkf\sxf\jlab^hr‡r&u-_butf“‰Gu f ƒ~^[d [fku 1 uxe _ ^[i!l‰ŸZ ]u vxutacvtfku(ŠGu ŠNig^
r‡Ž_bacu 11  auxdGvtjldy_baufkute ŠNacjliˆ_buxe 7 2 1 8ut_)7 1  8uxd "ut_ acuxe> utvt_cilutr‡utdy_CZ ]9^A_|^[da
luxdy_bu ^[‰‡vxuxavxf\u uxd 1 vtjl‰ Ru fk^1ŠNacjliˆ_bu 7 2 <8>utd<;AZ = jldy_bauxac“‰Gu fkute8Rjlikdy_ce ;0 0 ejldy_


^[fki ldGsxetZ


¡=“Bh•yOQ•l=XŸM ¢

y jli\_  ‰GdGu3ŠGacj[i\_bu  ^[ecec^[dy_ G^[a`f ƒ…jla_c„Gj vxuxdy_cacu6ŠG‰ _baig^[d lfku 1 221 ZO= jldy_bauxa “‰Gu3fkute
eq r‡s_ba!i “‰Guxe-ŠGu  ^[a-ac^  Rjla_^[‰›Vv [_csxe6ŠG‰ _bacik^[ d lf\6u G^[eceuxdy_L ^[a‰Nd r@?tr‡u6 j[ikdy_J
r‡jldy_cacux&a “‰Gunvtu Rjlikdy_7"^   ^[a _bikutdy_^[‰ vtuxavxfkunvtikacvtjldGevxaciˆ,_  1221 Z
¡=“Bh•yOQ•l=X U

y jli\_ 1 221 ‰Nd3_cacik^[ d lf\u ^[vt‰N_|^[d lfk"u ›ejlikutd“_  ut_ 5œf\uxei\d“_cuxaecuxv_bi\jldGeŠGuwfk^Gikeecutvt_baikvtu


ikdy_butacdGu ŠNu`f ƒ~^[ d lf\u 1 J^ [uxv 7 2 1 8 u_1J^ luxv`f\u vxutacvtfku vtikvxj[dGecvtaci\_  1221 Z y jlikutd“_  u_ 
fkuxe Gacj utvt_cikjldGewŠN u  e‰Gawfkute v [_csxe  12  ut_  11  Z = jldy_bacut#a “‰Gu`f ƒ~^[ikauŠG•‰ “‰ ^hŠGaci\fg^h_cŽxacu
1  5  uxe _ 3s y^[f\ u Avtuxf\fkuwŠG‰3_baig^[ d [fku 1 221 Z
¡=“Bh•yOQ•l=X V

= jldy_bacuta “‰Gu7fkuxeeq r‡s_ba!i “‰Guxe ŠNuv|„  ^ “‰Gu7ecjlr&r&ut_ Š ƒ…‰Gd`_baig^[d lfk8u  ^[aac^  Rjla_>^h‰`v [_cs
j Rjlecs7ecj[d“_ ^[f\!i ldGsteeciLu_ eux‰Gf\uxr&uxdy_ eci fg^wŠGi\e_b^[dGvxu7ŠGuf ƒ…jla _b„Gj vxutd“_cacu7^[‰&vxutdy_bacu ŠG‰&vxuxavxf\u
vxikavxjldNecvxai\_7ute_s y^hfku `ecjld ŠGik^[r‡Ž_bau[Z
¡=“Bh•yOQ•l=™L š

y jli\_ 1221 ‰Gd_baig^[ d [fku “ejli\_ 1  2  1  ‰Gd_baig^[d lfkuŠGikauxv_buxr&uxdy_euxr Nfg^[Gf\u  1 221 ŠNu
_buxf\fku ejla_cu  ‰Nu 1 ^  ^[a_cikutdGdGun^[‰3v h_bs 2  1   2 ^[‰3v h_bs 1  1  ut_ 1 ^h‰ v [_bs 1  2  Z y jli\_
” f\umvtuxdy_bau ŠG‰ vxutacvtfkuvtikavxjldGevxai\_  1221  ecj[d jla_c„Gj›vtuxdy_bauu_  vxuxf\‰GiIŠGu 1  2  1  Z
= jldy_baux a “‰ ƒ~j[d^ ” ‡N” 6kZ



 2 9 f 4678  -  B  7
¡=“Bh•yOQ•l= ;

y jli\_ { z fk.u lutvt_but‰Gva Rjla_ccs  ^ha €ŠGu fkjld l‰Gux‰Na  FQu_ŠGu ecutdGe ^[aGi\_cab^[i\ac7u I Z y jliˆ_  f ƒ…ikr‡^ lu
ŠGu ™ ^hafg^_cab^[dGefg^h_cikjld‡ŠGu lutvt_cux‰G(a { z CZ  d ^  Ruxfkf\u 1 u_ 2 f\uxei\dy_buxaecutvt_bi\jldGeŠGu utI_  Z
]I^wŠGajli\_cu  ^hab^[f\fkŽxf\u   ^[eeb^[dy1_  ^[a 1 ac3s  j[dGŠ wf ƒ~stdGjldGvts[Z 4 dA3u M ut_ fg^ _cab^[dGefg^h_cikjld`ŠNu
luxv_but‰Ga ‚ { z uxEd ljli\uwfku Rjli\d“_ 1 ec‰Ga‰GL d Rjlikdy_ 1 “‰G'i  G^[avxj[dGe_cac‰Gv_bikj[d ŠNu 1  ^  ^[a_cikutd“_


 ut_7uxe _ _cux%f “‰Gu 11 ‡  Z


¡=“Bh•yOQ•l= X

 dy_cacj ŠG‰GikejldG e ˜-fk^eq r&st_caciku-vxuxdy_cab^[f\u6ŠGu6vxutd“_cac2 u hZ y i f ƒ…jld ec‰   j[ecLu “‰ ƒ~i\f u Ni\e_cu
3u M uxvt_ci [uxr&uxdy_&‰G_ d  jlfˆ]q ldGu M (_but&f “‰G2u 6ecj[i\_`f\u6r‡i\fkikut‰*ŠGu 1  1 JLM~FQj b_ ^[a
1 1
vxjlEd lutdy_bikj[d 1  LM ‡ 1 >I L^[fkj[ac,e ˜h^AutEd ljPqluta fk#u Rjli\d“A_ wec‰Ga7fk#u Rjli\d“A_ JLM Z
ikdGe] i Kh^ ?t_bau ‰Nd RjlikdyK_ BN›u ŠNu fk^7vtjl r  jlesx8u ˜‡†˜8M ˜ |Z i ^[f\vx‰Gf\jldGe9ŠGjldGv vxu_c_cu
vxjl r  j[ecsxuhZ ŠdGu e q›r&st_caci\u vxutd“_cab^[f\u ute_‰GdNu acj[_b^h_bi\jld-ŠŸƒ{^[d lfk,u — ›fk^maŽ lf\u ŠGu vtjl r  jlei\_cikjld
ŠGuxe1acj[_b^h_bi\jldGewdGj[‰GemŠNi\_nŠGjldN@v “‰Gu‡eci  uxe_  ^[i\a ˜ h2^ ?_bau‡‰GdNu&_cab^[dGefg^h_cikjl|d  ut_1ecOi  uxe _
ik r  ^[ika ˜1euxab^A‰GdGune q›r&st_caci\u1vtuxdy_bac^[fkuhZ
ikdGeiec, i ˜3uxe _Aik r  ^[ik3a i\8f h^ Gikutd u›ike _buxa`‰Nd Rjlikdy
_ B›NLs  ^h a ˜Euxd f ƒ…j›vtvx‰GaacutdGvxu3fku
vxuxdy_cacuŠNumfk^Ae q r‡s_baci\unvxuxdy_cab^[f\u[Z  dŠG*s B dGi\_r‡^[ikdy_cuxd ^[dy_ 1 vxjlr&r&uwst_|^hd“_ vxu RjlikdyJ_  1 (
Y€‹
vxjlr&r‡u1st_b^[dy_wf ƒ…ikr‡^luwŠNu 1 , ^ha#˜   1 *wf ƒ…ikr‡^lunŠGu 1 (# ^ha#˜(7 u_w^[i\dGeciIŠGu e‰Gi\_cu[Z fg^
B %d Nj[d6acut_cjlr Ru Gi\uxd6ec‰Na 1  LM ‡ 1  ut_f\u j[f\q] ldGu 1 M 1 uxe__cuxf  ‰Ÿƒ~jld3f\uljl‰Nfg^[iˆ_CZ
Œ^[aAvxjldy_cacu eci  uxe@_  ^[i\ai\fd ƒ…u›ike _bu2 ^[e +”jlacvtsxr&uxdy_&‰Gd _buxfRjlikdy_BN›uhZ Œ f\‰Ge Gacs
vxikesxr&uxdy_ e&i ˜-ute_`fk^ _bac^[dGefg^h_cikjld‘ŠGu [uxvt_cux‰Gad ‰Nf'9_bjl‰GemfkuteRjlikdy_beljldy_`vtjldElutdGika
Rjl‰Ga
1 Gjld vxjldNe_ba‰Gi\_utdGec‰Giˆ_bunfkute^[‰N_baux e Rjlikdy_be7ŠGunr ?xr&u#“‰Gu NacsxvtsxŠGutr‡r&uxdy_xZ $ ^[dGefkuwvC^he
vxjldy_bac^[ikau Gi\f d ƒ q3^[‰Gac^  ^[e7ŠGunecjlf\‰N_bi\jldGe^h6‰ GajlGf\Žxr&u[Z
¡=“Bh•yOQ•l= ™

 iˆ›jldGe7‰Gd Rjli\d“_ 1 ŠG- u &Z /j[_bj[dGe &`fk^


acj[_b^h_bi\jldŠGuvxuxdy_cacu 1 ut_wŠ ƒ~^[ d [fku ‰ N* ZŸŒIjl‰Ga
 ‰Nu 1  1 ( 1 *ejli\_ ‰Gd-_cacik^[ d lf\u3s “‰Gikfk^h_bstab^[fLŠGi
acuxv_Iikf[e‰ )A_  ‰Nu 1 *‡&%7 1 (87Z waj[d [ux‰N_ “‰Gu
A1

fkux8e  j[ikdy_be 1 (ut_ 1 *7^  ^[a_cikutdGdGuxdy_aux>e  utvt_bi


D1
A2 B

luxr&uxdy–_ #+(ut_ )*JPŠGjldNv 1 *ŠGj[i\_I^  G^[a_cuxdGi\a


D2

 f ƒ…ikdy_cuxaceuxv_bikj[dmŠGuteŠGajli\_cuxe &%7+(J89u_ )*CZ i uxe


D3 A 3 = r(A 2)

ŠGacjliˆ_butewecu&vxjl ‰ Ruxdy_1uxd ‰Gd‘‰GdG!i “‰Gu Rjlikdy_  jld


^(ŠGjldGv 1 *Aut_Ajld _cacjl‰ [u 1 (Auxd ^ Gfki  ‰G^[dy_
 1 * fk^ acj[_b^h_bi\jld ‚ &›Z r(D 1)
r(D 3)
r(D 2)
E

¡ =“Bh•yOQ•l= ,

 d eb^hi\_“‰ ƒ…‰GdGuA„Gj[r‡j[_c„Gst_cikumŠGu`ab^ jla _) r ‰Nf\_biGfki\unfkuxe fkjld [‰Gux‰Gae ^[a .0).pZ| vti' jld
lux‰›_`uxdEljPq[uxa`‰Gd vtuxavxfku‡ŠGu-ac^Cqljld & wec‰Gam‰Gd vxutacvtfku-ŠNuab^Cqlj[d&(~5Ifku&ab^ jla _=) ŠGu*›ac^
ŠGjldG#v lstac4i B ux=a .0)."‡  Gvhƒ~ute>_ G ŠNikacu )(‡
   jl/‰ )~‡ ‚    Z
4 d jl‰N_cac u  vtut__bu1„Gjlr&j[_b„Gs_bi\u ŠG3u  ac^‡uxEd [j€q[uxa fkuwvxutdy_bacu1ŠG6‰ Gacutr‡i\uxavtuxacvtfk"u RŠNikecj[dGe” 
ec‰Ga7fkuwvtuxdy_baunŠG‰ euxvtjldG%Š  ”(C Z ikdGeijld3ŠGjliˆ_7J^ ljli\afg^`acutfg^h_cikjld ‚3‚ƒ”… (c‡3) ‚3‚ƒ” …  CZ  f dGunauxe_cu
 
Gfk‰G&e “‰ ƒ AvxjldGe _ba‰Gikau1f\uxeŠGut‰›2  Rjlikdy_b&e “‰Gu1f ƒ…jld6jl›_bikutdy_^[ikdGei Z
$ ^[dGe`fku‡vC^[e`j b & 
‡ &(7‰GdGu6ŠGute ŠGux‰  „Gjlr&j[_b„Nst_bi\uxe GacstvxsxŠNuxdy_bute‡ute_AŠGuac^  Rjla_
ut_fku-vtuxdy_bau6vxjlaacute RjldGŠ ^[dy_`ute_`a u u_b(s f ƒ~i\d B dGi Z i ut_c_cu„Gjlr&j[_c„Gst_ciku&uxe_ uxd +Q^[iˆ_‰GdNu
_b, ab^hdGecfk^h_bi\jld ZˆZ\Z
‰ r‡jli\dGeŠ ^hdGe3fku(vx^[e3 j b–fkute6ŠGux‰›–vxuxavxf\uxe3dGu exƒ…ikdy_butaceuxvt_cuxdy6_  ^[e  jld acuxr‡^[>a  ‰Nu
 ‰Nu fkute vtuxdy_bauxeŠGute„Gjlr&j[_c„Gst_cikuxe vxj[acacute RjldGŠGutdy_7^[‰›@ Rjlikdy_be ŠGu vxjldGvtjl‰GaeŠGuxe _|^[d luxdy_bute
vxjlr&r ‰GdGute7^[‰›3ŠNux‰› vtuxacvtfkutexZ
¡=“Bh•yOQ•l= —

 d‘vxjldNe_ba‰Gi\_1Š ƒ~^[RjlacŠ‘‰Gd vC^[aacs 221 


A

u _bstacikut‰Gacutr‡utdy_V^h‰ v [_bs 2 1 ŠG‰"_bacik^[ d lf\u[Z B C

y jlikutdy<_   u_  fkuxe ikdy_butacecutvt_cikjldGe ŠGu 1 œu_


J^ lutv f\u6v [_cs 2 1 Z y j[i\_ f ƒ~„Njlr‡jh_b„Gs_biku
A1
1 D1

ŠGu vxutdy_bacu 1 utEd ljPqy^[dy_  uxd @!ŸutfkfkumuxEd lj[iku


A2  B C
E D

ŠGu- r ?xr&u uxd  Z y jli\_ 2  1     f ƒ…ikr‡^ lu


D2
A 3 = r(A 2)

ŠGu 221  G^[a ZNŒ^[avtjldGe_cac‰Gv_bi\jld6ikfRexƒ~^ liˆ_


D3

Š ƒ~‰Nd vC^[aacs[|Z  a f\uxe Rjli\d“_ce 1  2  2  ecjldy_w^hfk4i




ldGsx3e G_bjl‰›_7vxjlr&r‡u fkuxe Rjlikdy_ce 1  1  1 !GŠNjldGv


fkuwvC^hacacs 2  1  @
ute_fkuwvx^[acas [jl‰Gf\‰ Z
r(D 1)
r(D 3)
r(D 2)

Y
E D


¡=“Bh•yOQ•l4= 3

] u ecuxdNeŠGikauxv_ ute_c+Q^hvxikf\u5 ei f\uxe ŠGacj[i\_bute7 ?598 ut_7 221 8


ecjldy_# ^hab^[f\fkŽxf\uxe3 ^hfkjlae fk^@BCl‰Gau uxe _wikdEh^[aig^[dy_bu  ^[a ab^ Rjla _
fg^acs  u›i\jld Š ƒ~^E›u47 8* st_b^[dy_nfg^„ ^[‰›_bux‰Nawikece‰Gu`ŠGu 1 Z
wd uxd ŠGsxŠN‰Gi\,_ “‰Gu1fk#u Rjlikdy_I;
^ G^[a_cikuxdy_AAvxut_ ^E›u1ut_,“‰Gu
 

fkuxe7^h d lfkute 1  ;  u#_ 1 ;5 ejldy_sy^[‰ ŸZ 

ŒIjl‰Ga fg^nacsxvt!i Gaaj “‰G"u ›dNjl‰Ge^hfkfkj[dGe>ikdy_baj›ŠG‰Nikacu7fk^1as  u Ni\jld


vxjldGeikŠGstacst•u GacstvxsxŠNuxr&r‡utd“_  vtuxfkf\uŠ ƒ~^E›u 7 8 Z  utfkjldG e fk^
˜yZ 4 f\fku ut0d [jliku 2 ec‰Ga 1 u_ ;8ec‰Gan‰GW d  j[ikdy
_ &Z9]ute^[d lfkute



? ; 2 ut_ 54; 1 ejldy_3s y^[‰›&vx^[a  j  jlesx8e  ^[a fg^necjlr&r‡u_ yikf  


uxdAacstec‰Gfˆ_b&u “‰Gu7fkute> ^[ d lf\uxe 1  ; 2 ut_ 1 ; 1 fkuejldy_ s y^[f\uxr&uxdy_CZ
 ikd ^hfkuxr&uxdy_ ~jld^ 1 ; 2 ‡ 1  2 ‡  Z
= ^[ike1fg^3ecjlr&r&u&ŠNuxem_cacjli\e^h d lfkutemŠŸƒ~‰Gd‘_baig^[ d [fku +Q^[i\#_ —Z
 

wd3utd ŠGstŠG‰GiˆA_ “‰Gu 5


 


1 2 ; ‰ 2 1 ; ‡ 1 2  ‰   1 ; ‡N— ‚ 
2 ;
= ^[iker‡^[dGi4+”uxe_cuxr&uxdy_J›Š ^[dGe7f\uxevxj[dGŠGi\_cikjldNeŠGunfg^BC[‰Gacu"Gjld ^`fkuxei\dGsl^[fkiˆ_bsxe 1
1 2  ut_ 2- 1 ;   1 ;AZN]9^auxfk^h_bi\jld@N acsxvtsxŠGutdy_bu#G ajl‰l u ‰Nu vxuteikdGs3y^[fkiˆ_bste ejldy_utd~+Q^[i\_

ŠGuxe7sl^[fkiˆ_bsxe3Gut_^hikdGeiK ‰Nu ;†‡ &Z i utfg^Ruxacr&ut_7ŠNu1vtjldGvtfk‰Gau[Z


YP˜
¡=“Bh•yOQ•l= @¢

9] ^ vtjlr@Rjlesxu ŠGuaj[_|^E_bikj[dG(e N Š ƒ~^[d [fku (* N ute_7‰GdNuwacj[_b^h_bi\jldŠ ƒ~^[d lf\u F* N GŠGjlFdGN v fg^ vxj[r


 jlesxu ŠNu+-acj[_b^h_bi\(jlN dGe Š ƒ{^hd lfku * uxe _wfg^‡vxjlr  j[ecsxuŠ ƒ~‰NdGu acjh_|^h_cikjld Š ƒ{^hd lfku * ut_ Š ƒ~‰NdGu 

acj[_b^h_bi\jldŠ ƒ~^[d lf\u * NŠGjldGvnv[ƒ…uxe _‰GdGu _bac^[dGefg^h_cikjld Z84 d6G^[a_cikvx‰Nfkikuta fk^vxjlr RjlecstuwŠGuxe_bajlike
acj[_b^h_bi\jldGee‰Gvxvtuxecei [uxe ŠNsxvxai\_cuxe ^[‰N_cjl‰Ga ŠGute1Rjli\d“_ce 1 B  1 ' 1 ( ute_ ‰GdGu_cab^[dGefg^h_cikjldŸZ y jliˆ_{ z
fk~u lutvt_b^[‰GaAŠGu3vxu_c_bu6_cab^[dGefg^h_cikjl|d jld ^ ^[fkj[ace{ z ‡ ;,‚a‚B';C… *(‡ ;,‚]‚E*J;C… SL‡ 8‡ ;O‚‚CQPF‚CRF‚„*';O‚C‚7QPF… RFSPZ
$ jldGv ecIi ;CB2‡ ;OQPFRFS7>^hfkjlae { z uxe_‡fk~ u [uxv_bux‰Nad“‰G'f  u_‡ŠNjldGv f ƒ…ikr‡^ luŠGu3_bj[‰N _  jli\dy_AŠG‰
Gfg^[d^ GaŽxe f\uxe7_bacj[ike Nacuxr&ikŽtacute aj[_|^h_cikjldNe ute_wvtu r ?xr&u Rjlikdy_CZ 4 d  ^ha_bi\vx‰Gf\ikut,a Rjl‰Ga 1 BJ
^ GacŽte-fk™ ^ Gauxr&ikŽxau aj[_|^E_bikj[d i\fute_-uxEd ljPqls utd 1 BJ ^ GaŽxe‡fg^ euxvtjldGŠGu(i\f7uxe_-uxEd ljPq[s‘utd
‰Gd–vxuta_|^hikZd Rjlikdy_ 2  u_6^(N NacŽxe6fk^‘_cacjli\ecikŽtr‡u3aj[_|^h_cikjld ikfacu_bjl‰GadGu(utd 1 BCZ i jlr‡r&u jld
^ 1  B 1  2 ‡ 2  1 ( 1 B2‡ * ut_ 1 B 1 ‡ 1  2  2 1 (2‡ 1 ( 1 BJ>^hfkjlae`(N fkuxe “‰ ^E_bac2u Rjlikdy_be
1 B 1  2  1 (+”jlar‡utd“_ ‰Gd(f\jleb^[ d luŠGj[d“_ f ƒ~^[ d lf\u1utd 1 u_ 1 (h^[‰›_ *  ŠGjldGvmf ƒ~^[ d lf\umutd
1 B[^h‰N_ *  u_7fku _baig^[ d lfku B  1 ( ute_s “‰Gi\fg^h_csxac^[f Z
N
1 1

¡=“Bh•yOQ•l4 = U

A Ruxfkj[dG& e  ^[au Nut@r Nfku   (7utI_ * f\uxe _cacjli\evxutacvxf\ux&e “‰Gunf ƒ…jld ^ `vxjldNecikŠNsxacuta7ut_ &  
&(ut_ &*fkut‰Ga ac^Cqljld6auxe Ruxv_b4i + Z   d h^e ‰  Rjlecutauxdjl‰›_bacAu “‰Guwvxute _bajlike ab^Cqlj[dGeecj[d“_ŠGux‰›
ŠGux‰ 6ŠGi\e_bi\dGvt_ce"^ BG( d  ‰Nuwfkuxe _|^[d luxdy_buteŠGjldy_jld-ŠGjli\c_ GacutdGŠGauwf ƒ…ikdy_butacecutvt_cikjld-^[ikutd“_Gikutd
‰G6d Rjli\d“_vxjlr&r ‰Gd Z
A Ruxfkj[dGewr-^hikdy_butd ^[dy_ 1 Q(1f\
u Rjlikdy_wŠ ƒ…ikdy_cuxaceuxv_bikj[d ŠGuxew_|^h d luxdy_cuxewu _csxaci\ux‰Gauxe  
utI_ (CZ 4_7ŠG3s BGdGikeecikj[dGeŠGun@r ?tr‡u 1 (F*ut_ 1 * CZ wd
^  %‰  Š ^[dNef ƒ…u›uxavxikvtun } “‰Gu 1 Q(st_b^[i\_
fku vtuxdy_baumŠGunf ƒ~‰GdN!i “‰Guw„Gjlr&j[_c„Gst_ciku ŠGuwac^  Rjla_   “‰Gi _bac^[dG>e +”j[acr‡^[i\6_  uxd (C Z A  utfkjldNe
 @Q(vxu_c_cum„Gj[r‡j[_c„Gst_ciku u_ŠG3s B dNikecejldGe7ŠG u +Q^ jxj[d ^hd ^[fkj l‰Gu %(F* u_ %* Z
 

i Au “‰Gu f ƒ~jld‡^mŠNi\8_ GacstvxsxŠNuxr&r‡utd“c_ Gacj[‰ lu ŠGikauxvt_cuxr&uxdy_ “‰Gu %(F* @Q(‡ *E   0Z  fLexƒ~^ li\_
ŠGjldGv ut~d +Q^[i\_ ŠGu Nacjl‰ [uxa‰G~d +Q^hi\_ _bacŽte lstdGsxac^[–f 5 fg^vtjl r  jlesxu ŠNuxe„Gj[r‡j[_c„Gst_cikute ŠGu vtuxdy_bau
  

1 u_ 2 “‰Gi uxe _utdGvxjlau‰GdGu „Gjlr&j[_b„Nst_bi\u ^ ecj[d vxutd“_cacu ec‰Gafg^7ŠGacjliˆ_b. u 7 1 2 8Z =^[i\e Rjl‰GaIvxutfg^ 
jld au y^hacŠGunf ƒ…ikr‡^ lu Š ƒ~‰Nd RjlikdyA_  ŠNumfk^AŠNacjliˆ_bu 7 1 2 8LikfŸuxe_ uxEd [j€q[sx
u G^[a7fg^`vxjl r Rjlecstu
ec‰Ga ‰Gd Rjlikdy_   "^   ^[a _buxdG^[dy_uxdNvxjlau <7 12 8 8Z ikdGeiLfg^nŠGacjliˆ_bu 7 1 2 8>auxe _bu [fkjl ^hfkuxr&uxdy_
ikEd h^[acik^[dy_bu  ^ha f ƒ~„Njlr‡jh_b„Gs_biku vxjl r Rjlecstu yvxutfg#^ Gacj[‰ l,u “‰Gu7fkuvxutdy_bacuŠGu7vxu_c_cu „Gj[r‡j[_c„Gst_ciku
uxe_Gi\uxd3eciˆ_b‰Gsne‰Ga7vxut__bunŠGajli\_cu[Z
¡=“Bh•yOQ•l4 = V

]uteŠGut‰› vC^[a _buxe ecj[d“_ecutr Gfg^hGfkute ŠGjldNv i\fwu›ike _bu*‰NdGu eikr&ikf\i\_b‰NŠG•u ˜*ut0d [jPqy^[dy_ fg^
vC^[a _bu 12 1  ec‰Na6fg^ vC^ha_bu 1  2  1  @gZ y jli\~_ ”5fku(vtuxdy_bau‘ŠNu vxut__bu eci\r‡i\fkiˆ_b‰GŠGuh1Z ” uxe _
fku eux‰GKf Rjlikdy_i\Ed [^hacig^hd“c_  ^[,a ˜Gi\fŸuxe _7ŠGjldGvnf\u ecut‰GKf  j[ikdy_vxj[r‡r ‰Nd6uxdy_bau1f\uxeŠGut‰›3vx^[a_cuxexZ
Œjl‰Ganf\u vxjldNe_ba‰Gikau jld auxr‡^[a “‰Gu “‰GuAf ƒ~^[ d lf\u ŠGuAfg^eikr&ikfkiˆ_b‰GŠNumute_1f ƒ~^[ d lf\u uxdy_cacuAfkute
ŠGacjliˆ_bute 7 1 2 8mu_ 7 1  2  8ŠGjldNLv ” ^  G^[a_cikuxdy_ ^[‰ vxutacvxf\~u  ^[eeb^[dy_  ^ha fkut
e  j[ikdy_be 1  1 
ut_  ^[af ƒ~i\dy_buxaecutvt_bi\jldŠGute7ŠGacjliˆ_bute 7 12 8 ut _ 7 1  2  8FQvhƒ~ute>_ G ŠNikacuwf\u vxutacvtfkuwac3u y^[aŠ ^[dy_ 1 1 
ecjl‰Ge f ƒ~^[ d lf\u –I Z 4 d +Q^hikeb^hd“_ ŠGu r ?xr&u J^ [uxvwfkutce Rjlikdy_ce 2 u_ 2   1 ut_ 1   ut#_ 8ut;_    jld
acut_cacjla‰ lu ”&f ƒ~i\dy_buxaecutvt_bi\jld6ŠGuw_bj[‰Ge7fkuxevtuxavxfkutexZ
¡=“Bh•yOQ•l= ;  š

 d auxr‡^[a “‰Gu ŠŸƒ{^[RjlaŠ “‰ ƒ~i\fq^-RuC^[‰Gvtjl ‰  Š ƒ~^[ d [fkuxe


ŠGacjliˆ_be ec‰Gafk^ BCl‰Nacu[Z  f q6^mfkuxe^[ d lf\uxe 1   ;  1   , ;c  1
ut_ 2  1 Z =^[i\e‡i\fqV^ ^[‰Geecif ƒ~^[ d lf\u  2-1  fku6_baig^[ d [fku
o#   

 
 

  
 2 1 st_b^[dy_i\dGecvtaciˆ_Š ^[dGe9‰GdnŠGutr‡i vtuxacvtfkuhZH d1‰›_bikf\ikeu^[f\jlace
ŠGux‰›+”jlikef\u_b„GstjlacŽtr‡u ŠGu.D„ ^hfkŽxeK“‰GiE+”jl‰NacdGiˆ_fkutesy^[f\i\_csxev5
1 ;
‡ 1  ‡ 1 
12 1  1 
wd3utd GŠ stŠG‰Giˆ_ ŠNikacutvt_cuxr&uxdy_7f ƒ…sy^hfki\_cs ŠGu1f ƒ…sxdNjldGvxshZ
ŒIjl‰Gafg^`ecutvxjldNŠGu “‰Gute_cikjld%Ljld ikdy_cacj ŠG‰Gi\_7f ƒ~„Njlr‡jh_b„Gs_biku ŠGu1vxuxdy_cacu 1 “‰Gi9utd0[jliku 
ec‰Ga  Z  d6dGj[_b#u  u_ 5œfkute i\r-^[uxe auxe> utvt_bi4luteŠNu 2 ut_ŠNu 1  ^[a Z ]9^ŠGacj[i\_bu27'?548


uxe_G^[ab^[f\fkŽtfku  7 221 8 u_wŠGjldGvRuxa>E   utdGŠGi\vx‰Gfk^[ikau 7 1 8Z i\dGecifkuvtuxavxfku  uxe _nfkumvxuxavxf\u




ikdGevxaciˆ_ ŠN‰_baig^[ d [fku 1 ?5‘Z Œ^ha vxu vtuxavxfkuecu_cab^[dGke +”jlacr&u1uxd ‰Gd vtuxacvtfkumŠGu vxutdy_bacu 
st_|^hd“_ŠGj[dGdGsnf ƒ…s y^hfki\_cs ŠGute ac^  Rjla_ceŠGsxr&jldy_basxu GasxvxstŠGuxr&r&uxdyJ_ „“‰Giuxe _ s* ikŠGutr‡r&uxdy_7fku


vxuxavxf\u1i\dGecvtaciˆ_ŠG‰3_bacik^[ d lf\u 12 1 Z


¡=“Bh•yOQ•l= ;R;

]utWe “‰ ^EN _bac_u Rjlikdy_ce 2  1  1 S s_|^[dy_‘vxj vtq vtfk!i “‰Gute wjld^ A

2  1 ‡ Z y jli\6_   f ƒ…ikr‡^ lu ŠGu   ^[a3fg^ acjh_|^h_cikjld & ŠGu


vxuxdy_cacu 2 ut* _ ŠŸƒ{^[d lfku N*  vtut_c_cu–acj[_b^h_bi\jld utEd ljliˆ_  ec‰Ga ‰Gd C

 jli\dy_wŠGu 1  9ŠGj[dGvAjld‘^ 1  ‡ 1 O ‰ `N”Z9]u _bacik^[ d lf\u


M

2 †  ute_63s “‰Gikfk^h_bstab^[f ŠGjldGv jld–^ 2  ‡ 2   ‡   $Z


]I^ acjh_|^h_cikjld & uxEd lj[iku 1 utd 1  Š ƒ…j b  1 ‡   1 >ut_‡ŠGjldGv
B

1  ‡ 1   ‰ †  ‡ 1  ‰ 2 $Z
C

¡=“Bh•y OQ•l= ;›X



 d ^ 1 ‡— ‚ 12 ‡ 12  ‡ — ‚  1.1 2ŠGjldGvAf\uxenŠGacj[i\_buteA7 1  8 ut_&7
ecjldy_, ^[ac^[fkf\ŽxfkutexZ
   8

E
A
C F
M

B
C
B
M D

¡ =“Bh•yOQ•l= ;›™

 d_cab^[iˆ_buxac^fku vC^[ejb6fkuxe8 j[ikdy_beejldy_7Š ^[dGe fg^vtjldBCl‰Nab^h_cikjldŠNuwfg^B l‰Gacu"Gfkute^[‰N_cacute


ecuw_bac^[i\_cuxdy_7ŠGu#+Q^jxj[d eikr&ikfk^[ikau[Z

o›Y
A
2

1 A
PB
B

PA C
B
PC D

C M
P

]ute jli\dy_be2;0J;  2 J; VŠ ƒ…‰GdGu( ^[a _J ;  1 J; J; ŠŸƒ{^[ ‰›_bacu ^[a_ecjldy_Avtj›vq›vtfki ‰Nuxe
  
Š ƒ~jb ;  ; @; ‡ ;  2 ; ‡
; 221 u_ ; ; ; ‡ ;  1 ; ‡ ; 11 Z% a f\uxeRjlikdy_ce;0+
   
ecjldy_ ^[fki ldGsxe eiLut_  ecux‰Nfkuxr&uxdy_ eci ;  ; @; ‡ yecjliˆ_ ;  ; @; ‡ ;  ; ; v“ŠGj[dGv eiLut_ ecut‰Gfku
    
r‡utd“_eci ; 221 ‡ ; 1.1 hŠGjldGveiNut_>eux‰Gf\uxr&uxdy_ eiGfkutev j[ikdy_be 1  2  1 J; ecj[d“_>vtj›vq vxfki“‰GuxetZ
¡=“Bh•yOQ•l= ;“

]92^ G‰Gikeeb^[dNvxu&ŠG(u  ( ^[a1ab^  Rjla _  wetƒ~stvxaciˆ_  1 ( ( ‡  1 ( - 2 vxuxf\fkuAŠGu  ^[a


ab^  Rjla _  ( exƒ…sxvtaciˆ1_ 
 ‡† 1   2 lŠGjldGv jldA#^  1 ‡Q
 [ŠNjldGv  ute_ f\u r&ikfki\ux‰
ŠGu  1   Z
A
2

1 A
PB
B

PA C
B
PC D

C M
P

¡=“Bh•yOQ•l= ;›—

 d1^ 1  ‡ 1 ; ‰ ;
- ‡ 1  2 ; ‰ 1.1  ‡


 2 ;†‡ 1 ”-ZGŒ^[avtjldGecs3“‰Guxdy_ fkuxec jli\dy_be 1 C”@


 1
  ecjldy_(vtj›vq vxfki“‰Guxe3ŠŸƒ~jb ” ; ‡ ”   1 ‰
1  ; ‡  ”- 1 ‰ 12  ‡  ”- 1 ‰ ( 1 ”  ‡ (
 N

vC^[a3fku(_cacik^[ d lf\u 1 ”  ute_ ikecj vxŽtfku utd`” Š ƒ~jb fk^




vxjldGvtfk‰GeikjldŸZ  

¡=“Bh•yOQ•l= ; 3

olo
ŒIjl‰GAa  ‰Numf\uxe ^[ d lf\uxedGumŠNs RuxdGŠGutdy_# ^[eŠGufk^ jlei\_cikjld3ŠG‰ Rjli\d“_-;  jldvtjldGeikŠGŽtacuxac^
ŠGuxe^[d [fkuxej[acikutdy_bsxe7ŠGunŠGajli\_cuxetZ
]u  jli\dy._ ;
s_|^[dyA_ Gacj u_bsmjla_c„GjljldG^[fkutr‡utd“_e‰Ga f\uxev[_csxe ŠN‰b_ aig^[d [fku 1 B 2 B 1 BJLfkute
 jli\dy_be 1 B 2 * ;  1 ejldy_vxj vtq vxf\i!“‰GuxetZ  d^`ŠGjldNv 5
7 1 B 2 ' 1 B 1  8 ‡ 7 1 B 2  1 BJ; 8 ‰ 7 1 BJ;0 1 B 1  8.‡ 7 1  2 ' 1 ; 8 ‰ 7 2  ;0 2  1  8
Œ^[a7vxj vtq vtfkikvti\_cs ŠGuxe,Rjlikdy_be 1 (J 2   1 ( ; ut_7ŠGuteARjli\d“_ce 1 (7 1 S 2 (7 ; Gjld ^L5
7 1  2 ' 1  ; 8 ‰ 7 2  ;0 2  1  8.‡ 7 2 ( 1 (  2 (K; 8 ‰ 7 1 (J;0 1 ( 1 (8

Œ^[a7vxj vtq vtfkikvti\_cs ŠGuxe,R jlikdy_be 1 *J 1 (J 2 * ; ut_7ŠGuteARjli\d“_ce 1 *7 2 (7 1 *7 ; Gjld ^L5
7 2 ( 1 (  2 (J; 8 ‰ 7 1 (J;0 1 ( 1 (81‡ 7 1 * 1 * 1 *K; 8 ‰ 7 1 *K;  1 * 2 *8 ‡ 7 1 * 1 * 1 * 2 *'8

4 dL+Q^[ikec^[dy_7ŠGunr@t? r‡u ^7[  uxv1fkute ^hd lfkute7 2 B 1  2 B 1  8ut_ 7 1 B 1 ' 1 B 2  8*Gjld uxd3ŠGstŠG‰Giˆ_
 ‰Nu f\uxe ^[d lf\uxe6ŠGute _cacik^[d lf\uxe 1 B 2 B 1 B u_ 1 * 2 * 1 * ecj[d“_ sl^[‰› ZŒ^[a3vxj[dGecs3 ‰Nuxdy_Jfkute
_bacik^[d lf\uxeejldy_ecuxrGfg^[NfkuxetZ


 

     

  
 





¡ =“Bh•yOQ•l= ;G¢M

i j[r‡r&uxd jtjldGe ^[a+Q^hikacu ‰Gd ŠNuxeceikd™5


 

 
 
   
 







o[z
Š7_bi\fkikejldGeŠNuxe^[ d lf\uxejlaikutd“_csxe7ŠGunŠGajli\_cuxe  j[‰G&a “‰GundGjle7acutfg^h_cikjldGedNu1ŠNsRuxdGŠGutdy_ ^he
ŠGunf ƒ…jlacŠGauwŠGuxeRjlikdy_ceec‰Ga7fkute7vxuxavxf\uxe ŠNjldGvnŠGunfg^
BCl‰Nacu[Z
Œ^[a7vxj vtq vxf\ikvti\_b"s Njld (^ 5
$ f  m
7 ) (  2
8 ‡ 07 2D() ; 8 ‰ 7$) ;'2m
81‡ 7 ;,(1(K;C(J; 8 ‰ 7 ; ';OJJ;1
8
‡ 7 ;,(( K;C(J;,*'8 ‰ 7Q;C(';C*J;C(J; 8 ‰ 7Q;;'J;J;C*'8 ‰ 7 ;;,*J; 1 8
waeifkute,Rjli\d“_ce  ;,(J ;,*7 ; ecjldy_vxj vtq vtfki!“‰GuteLjld ^(5
;O

7 ;C(K;C* J;,(J;O 8 ‰ 7 ; ';OJ; ';C*J81‡ 

Š ƒ~jbW5
0 D  D
7 2 ( )
8 ‡ 1
7Q;C( (J;C(K;C*8 ‰ 7Q;';,*J; 1 8
‡ 7$*1(  *J;C*'8 ‰ 7$*';C*&*1
8 ‡ 07 *(  * 8

Œ^[a7vxj[dGecs3 ‰Nuxdy_JRfkute& ‰G^h_bauRjli\d“_ce )* (J *7


 ejldy_ vtj›vq›vtfki ‰NuxexZ

oh}
¡=“Bh•yOQ•l= ; U

lj ikutd“_ (ut_ fkuxe8GacjHut_bsteacuteRuxv


y 
_bi+”e ŠGu 5e‰Ga fkute vh_bsxe  11  u_  12  Ÿu_
 

 –ut_  ecute`eq r‡s_bai!“‰Guxe`auxe> utvt_bi4+”e


 ^[a

 
ab^ Rjla _ -vtu r  ?xr&umv [_bshKZ  d ^ 2  1 ‡  
221 1 ŠGjldNv   ^  ^[a_cikutd“_


2 1 ‡ —
‚
^[‰ vxuxavxf\u vtikavxjldGevxai\W_  1221 F”uxd$vxfg^hika

 
ec‰Ga6fk™^ BCl‰Ga7u I‡_cjl‰N_3vxj[r‡r&u  Z $ƒ{^h‰N_bau



 ^[a_ lf ƒ~^[ d [fku   1 st_|^hd“_>ŠNacjliˆJ_  3^ a 

 ^[a_cikutd“_ ^[‰ vxutacvtfku`ŠGu ŠNig^[r&Žt_cacu  1 KZ $ u


  

@r ?tr‡u Vuxe _ e‰Gaf\u&vtuxavxfku&ŠGu‡ŠGig^[r&Žt_cacu


2 5 Z
 

i ^hfkvx‰NfkjldGefg^ G‰Nikecec^[dGvtuŠGu  ^[aab^ a


 jla _ ^[‰›_bacj[ike vxutacvtfkuxe>ŠGu7fk^ BCl‰Gau[]Z  d-^ 5
 

   7 8 ‡ 1

 2‡
 2

 
 
7 8 ‡ 1


   2 7 8 ‡ 2



 &a jld ^  ‡  ut_  ‡  v Š ƒ~jb  


 27 98~‡ (  A7 8(‡
   27 8|Z
       


Œ^[avxj[dGecs3 ‰Nuxdy_ ^"  ^[a _bikutdy_`f ƒ~^E›u ab^[ŠNikvC^hfŸŠGutevtuxavxfkutevxi\acvxj[dGecvtaci\_ce& 1  u_


 ŠGjldGv Afg^ ŠNacjliˆ_bu 7 ;
 8 Z

2 5 

¡=“Bh•yOQ•l= ; V

4 d vC^hfkvx‰Nfg^[dy_ fk6 ^ G‰Nikecec^[dGvtu6ŠGu ; G^[a ab^"  j[a_A^[‰  ŠGut‰› vtuxavxfkute`jld ^ ; 2  ;5 ‡


 ; 4 ;  ‡ ;  ; $ZIŒ^[amvxjldGes “‰Gutd“_Af\@u “‰ ^hŠGaci\fg^h_cŽxacu@ 2 1 5 ute_`i\dGecvtaciN_bi\Gfku" Š ƒ…jb
1
?  5 2 ‡   1-2 Z7]ute _cacig^h d lfkutLe  1.1 u_ 2 5/ ecjldy_ ŠGacj[i\_b3e jld ^ ŠNjldGv  11 ‡
  1 1 ‡ — ‚ ?  5 (ZŒ^[avtjldGes “‰Guxdy_Af\u “‰ ^[ŠGaikfk^h_bŽtacu 1 ?5 5uxe_ikdGevxa!i N_cikGf\u[Z
N
‚
]9ute7ŠGacjliˆ_bute 7 8 7 1 :8' 7 &598ecjldy_7f\uxe7^E›uxe7ac^[ŠGikvx^[‰›ŠGuxe7vxutacvtfkuxe7ŠGunŠGik^[r&Žt_bauxe 1.1 
(

2 Gu_ ŠN‰ vtuxacvtfkunvxi\acvtjldGecvtaciˆ, _  1 ?5/6RŠNjldGvnuxf\fkuxeejldy_vxjldGvtjl‰Gaab^[dy_butexZ




¡=“Bh•yOQ•l=XL š

y jlikutd“4 _ 6f\u‘vtuxavxfku vxi\acvxj[dGke


vxaciˆ_  f ƒ…„Gu G"^ ljldGu u_ ( f\u*_ba4i 

^[ d lf\u vxikavxjldNecvxai\_ ^[‰ _baig^[ d [fku 




1  ZV]9^ ŠNacjliˆ_bu 7 1 8"st_|^hd“_ 


fg^VvtjlacŠGu*vxjlr&r ‰GdGu  vxuxe ŠNux‰›
 


vxuxavxf\uxe yŠŸƒ{^ NacŽxe f ƒ…u›utacvxi\vxunY€ao [fkuxe 

ŠGacjliˆ_but<e 7R  8wut_ 7 2 28wecj[d“_  ^


ab^[f\fkŽxf\ux3e  ^[ikdGe_i  ‰Nu fkute ŠGajli\_cuxe
7f  87ut+ _ 7 2( 8ut_ “‰Gu fkute ŠGajli\_cuxe
7   8u+ _ 7  28 ZŸŒ^[a vtjldGes “‰Guxdy_  
fkuxe7_baig^[d lfkute 2 "ut_   5ecj[d“_ 
ecuxrGfg^[Nfkux@e 5i\f u›i\e_bu-‰GdGu6eci\r‡i
fki\_c‰GŠGuuxEd ljPql^[dy_ f ƒ~‰Nd6ec‰Gaf ƒ{^h‰N_bau[Z 
ol‚
9] u vxutdy_bacu ŠGu vtut_c_cu eikr&ikfkiˆ_b‰GŠNu ^a
 ^[a_cikutd“_ ^[‰› ŠGut‰› ŠGacj[i\_bute27 2  8
ut_&7  8ZŒ^[anvxjldGes“‰Gutd“_i\fexƒ~^liˆ_nŠG‰W jli\dy_ “‰Gi>^"  ^[a _bikutdy_nŠGjldGvAsl^[fkutr‡utd“_3fg^
ŠGacjliˆ_bu 7   8 Z

¡=“Bh•yOQ•l=X9;

RS:<VEZ
]#u Rjlikdy#_  ^  ^[a_cuxd ^hd“A_  fk^ Gikeecuxv_baikvxunŠGuwf ƒ{^hd lfku 1 Gi\fŸux
e _ s“‰GikŠNike_b^[dy_7ŠGuxe7v[_bste
12 u_ 1.1 9ŠGj[dGv-f\u&ac ^  Rjla_mŠGute^[i\acutemŠGut em_cacig^h d lfkute h^[‰N_   ‡ 9Š ƒ~^[‰N_cacu@ ^[a_
 

fku-ac^  Rjla_ ŠGuxeA^[i\acute ŠGuxe _cacik^[ d lf\uxe h^[‰›_ N‰Gik>e  ‰Nu6fg^„ ^h‰N_but‰Ga`ikeec‰Gu6ŠGu 1 fkux‰Na uxe _
  


vxjlr&r ‰GdGuh„Z  d r&jldy_bauwf ƒ…s y^hfki\_cs J^ lutvmf\uwab^  jla _ ŠGunfg^ r ?xr&uwr‡^[dGikŽtacuhZ



RS:<V CZ
i j[r‡r&ujld&^ 2  1  ‡   11 [fkuxe>^hacvxe 2  ut8_  1 ecj[d“_>3s y^[‰›`ŠGjldGAv  ute_ fku
r‡i\fkikut‰6ŠGu1f ƒ~^[acv 221 ZRŒ^[avxjldNecs “‰Gutdy_wfg^AŠGacj[i\_bAu 7 ”
 8uxe_ fg^Ar&sxŠGik^h_baikvxuwŠN‰v h_bs 221 
ŠGjldGvnuxf\fku f\‰Gi ute,_  uta RuxdGŠNikvx‰Nfg^[i\acu[Z
y jli\_ fkc^ h^[fkut‰GaŠGu f ƒ~^[ d [fku 1 u_ &vxutfkf\u ŠGu>f ƒ~^[ d lf\u 2 Cj[d^ 2    ‡ 2  1  ‰ 1 2  ‡
 ( ‰ ( Z $ƒ{^h‰N_ba u G^[a_j[d ^   2  ‡  2 1 ‰ 1- 2  ‡  221 ‰ 1  1  ‡ ( ‰( Z $ jldGvnfku
_bacik^[ d lf\u 2   uxe _ i\ecj vxŽtfkuux~d  ut8_  ^[a>vtjldGec3s “‰Guxdy_j[d^   ‡ 2  4 d +Q^[ikec^[dy_ ŠNu
@r ?tr‡uJ^ [uxvfku_baig^[d lfku 1   j[d‡utd&ŠNsxŠG‰Giˆ1_  ‰Nuf\uxe_cacjli\ve  j[ikdy_be 2  1   ^   ^[a _bi\uxdGdGutdy_
A‰Nd r ?xr&uwvxutacvxf\unŠGunvxuxdy_cacu  Z
RS:<VlZ
]ute^[ d [fkuxe 2  1 ut_ 2  1 s _|^[dy_ŠGacjliˆ_beIfkut–e Rjlikdy_be 2   – 1 ecjldy_vxjPq vxfki “‰Gux3e 
   
Š ƒ~j b 1  ‡ — ‚ 2 ‡ 112 NŠNu @r ?tr‡u jldr&jldy_bau 1  ‡ 1221 Nu&_  ^[a
vxjldGes “‰Gutd“_&f\uxem_cacik^[ d lf\uxe 2 11 jldy_`fkut‰Gace1_bajlike^[ d lf\uxe3s y^[‰›*ut_ ecjldy_`ŠGj[dGv-i\dGŠGikauxv_b*u
     

r‡utd“_7euxr Nfg^[Gf\uxe F=ljli\ca BCl‰NacuwŠ ^[dNef\u vxj[‰Gacƒe I„Z  d3r‡j[d“_cacuwfk^eci\r‡i\fki\_c‰GŠGu J^ luxvnf\uxe7^[‰N_cacute


_bacik^[ d lf\uxeŠNu1 r ?xr&u[Z
i j[r‡r&ujld ^ 2  ‡ 2  11 ‡ 1   fk^*ŠGajli\_cu 7 2 1 8‡ute_3fk^*Gikeecutvt_baikvtu
 
u _bstacikut‰Gacu ŠGu f ƒ{^[d lfku  u_ŠGjldNv fg^1ŠGacj[i\_bu 7 1 8  ‰NiŸfk‰NiRute&_ Rux>a  utdGŠGi\vx‰Gfk^[ikauec^
Gikeecuxv_baikvxuni\d“_csxaikux‰Nacu[Z
 

y jli\_  h l ‡fkutee q r‡s_baci “‰GuxeŠGu  ^[aIab^  jla _^h‰›n_bajlikeIv [_csxeŠG‰m_cacig^h d lfkuhHZ  d
^ 2   1 ‡
2 1 ‡  2 ‰  2 2‡ 1  2 ‰  2 1 ‡N— ‚ 2  1.1 7 ^[a9vtjldGec3s “‰Guxdy_
 

fku Rjli\d“_  ^   ^ha_bi\uxdy_w^h‰ vxuxavxf\u vxi\acvxj[dGecvtaci\_ ^[‰(_baig^[d lfku 1221 KZ  d r&jldy_baufkum@r ?tr‡u
   

acsxe‰Gf\_b^h_ Rjl‰Gafkute  jli\dy_be  ut_  Z


 

¡=“Bh•yOQ•l=XRX

 d`^ 2 1 ‡3 &     2  1.1 ut_ 2  ‡


 &(   2 1  Z  ajld`^ 2 11 ‰ 2-1  ‡
2
A

— ŸŠ ƒ…j b   2 1.1 ‡   2-1 œut_ ŠGjldGv 


 ‡  Z
  A D
C R

2r2

2r1

B
1
M
B

o[†
¡=“Bh•yOQ•l=XR™

]ute _bacik^[d lf\uxe 221 }$ut_ 2 11 ^Cql^[dy_n‰GdGu` ^[eu`vxjlr&r ‰GdNuŸf\u ab^"  j[a_wŠGu`f\ux‰Gaew^[i\acuxe
uxe_,Gaj Rjla_cikjldGdNuxf^[‰3ac^ Rjla_7ŠGunf\ux‰Gae„ ^[‰N_cux‰GaexZ  d^`ŠGjldNv
2 1 } ‡ }-;   } ; 2 }.;
1 ; 2 ‡ 1 ;
 -

2 1 1 
2 1 ;  
>d_+Q^[i\eb^[dy_AŠGu3r ?xr&u6^Jluxv fkute _bacj[ike _baig^[dlfkute
4 221 }  11 }  12 } ŠGjldy_&fg^ ejlr‡r&uŠGute
^[ikauxeh^[‰N_7vtuxfkf\uwŠGu 1221 j[d j[N_bi\uxdy_5
}-; }  }  ‡  221 }   11  } 1 2  } ‡
1 ; ‰ 2  ‰ 1  22 1 1  ‰  11 2 
‰  1221  ,


2
A

A D
C R
2r2

Q
2r1

B T
1
M
B
P
C

¡ =“Bh•yOQ•l=X 

y ‰Ga6fg^*ŠNacjliˆ_bu 7 12 8-i\fq ^ ŠGux‰ O j[ikdy_beec^h_bi\e>+Q^[i\eb^[dy_-fg^*vtjldGŠGiˆ_bikj[d Z 4 d u3M ut_3f\ux‰Ga


vxj jlacŠGj[dGdGsxute ^[a q›vtuxdy_bai!“‰Guteejldy_7acuteRuxvt_ci[uxdy_  7 1   2  , 8 ut_7 1   ‚  2  , 8 Z y jliˆ_; u_
vxute  jli\dy_bexZ y jliˆ_  ‰Gd Rjlikdy_eb^h_cik>e +Q^hikeb^hd“_ ‡ vtjlr&r‡u ‡ ut_
 ‰Nu 
 

^  ^[a _bi\uxdy_ ^h‰*ec3u lr&uxdy_  12  jld utd ŠGstŠG‰Gi\_ “‰G u(  uxe _ fg^ Gikeec utvt_baikvtu- ikdy_csxaci\ux‰Gau‡ŠNu


 


f ƒ~^[ d lf\=u 1   2 Z $ uw r ?xr&u j[d6ŠGsxŠG‰Ni\_ “‰Gu  uxe_fg^Gi\eceuxvt_caci\vxu u ›_csxaikux‰NacuwŠGuwvtu @r ?tr‡u


^[ d lf\u[ZRŒ^[a vxjldGes “‰Gutd“_1f\uxe ŠNacjliˆ_bux2e 7'
<8 ut_ 7  28ecjldy#_ Rux>a  utdGŠGikvt‰Gfg^hikacute u_wŠGjldNv 
^   ^[a _bi\uxdy_^[‰3vxutacvxf\unŠGunŠGig^[r&Žt_cac u  &Z
¡=“Bh•yOQ•l=XR—

i j[r‡r&ux d jtjldGe  ^[a +Q^hikacu ‰GdGu BCl‰Ga u 5

ol‹
A

x DA
P
Q

R N z
I
G

A C
B y P P

PC

;  Œ jlejldGe2‡ 1  ‡ 1 2
‡ 2 `‡ 2 ;0 ‡ 1 ; ‡ 1 &Za d^m^[fkj[ace     . ‡  


‡ ZLŒ^[a7fku _c„GsxjlaŽxr&uwŠGu i u3h^Ajld utd ŠNsxŠG‰Giˆ_7f ƒ…u›ike _butdGvxu1ŠG‰L j[ikdy_7ŠGu nutalj[dGdGu[Z
 


 i ^hfkvx, ‰NfkjldGe M ‰NiGecjldy_>ŠGute fkjldl‰Gux‰Nace ikr Rjla_b^[dy_buxe  j[‰Gafg^ ec‰Giˆ_buŠGuf ƒ~u ›uxacvtikvtulu_

uxd lsxdGstab^[fNŠ ^[L dGe>RE uC^[‰Gvtjl ‰ &Š ƒ…u›uxavxikvtuxetZ  d&^ 12 ‰ 22L 1 E ‡  ‰ ‰ ‰ E  utL _ 221L ‡  ‰ 
Š ƒ~j bL‡ ( CZ $ u1 r ?xr&uwjld vC^[f\vx‰Gf\u ‡ ( u_
‡ ( Z
           

$X  = jldy_bajldGe “‰ ƒ…jld6^ 1 ;#a‡ 2 ; “vxu “‰GiRec‰a)‡ac^Rjl‰Ga ‰›_bikf\ikeuxa fku7_b„NsxjlaŽxr&u ŠGu i u3h^
ŠGuwfg^ r ?xr&u r‡^[dGikŽtacu “‰Gu GasxvtsxŠGutr‡r&uxdy_CZ y j[i\_ ; J; fkute& j[ikdy_beŠNu1vtjldy_|^[v_ŠG‰3vxuxavxf\u
u›ikdGevxai\_ ŠG^[dGe`f ƒ~^[ d [fku 1 J^ luxv6f\uxe v [_bstLe F Nacjlf\jl d lstSe I 1 2  1.1 vZ wd ^ 2 ; Q‡ 2 ;  u_
 

‡ 1 ;  >Š ƒ~j b 2 ; ‰ 1 ; L ‡ 22L 1 1Z  a‡jld ^ E 1 ; q ‡ 1 ; ut_ 1 ;



‡
j b 1 ;  ‡ ( ‚ 12 ‡ L ( L ‡ 2 ;&Z ‰
1 ; 1 ;
12 ‰ 221 ‰ 1 1  Š ƒ…
      
       

$™  y jliˆ_ ‡f ƒ…„Gjlr&j[_b„Nst_bi\uŠGuvxutd“_cacu  ut_ ŠGuab^  jla _ ‚  Z›Œ^[a %yfkuxe>r&ikf\ikux‰  1   2   1 


ŠGuxev [_bsteŠG‰6_bacik^[ d lf\u ejldy_7uxEd ljPq[sxeec‰Gaf\uxeejlr‡r&ut_ce 1  2  1 Z y jliˆ;_  fk^ŠGacj[i\_bAu  ^[ac^[4f
 

fkŽxf\u <7 221 8_|^h d luxdy_cu ^[‰‡vxutacvxf\u i\dGecvtaciˆ_ ut<d ;   Z ] ƒ…„Gjlr&j[_b„Gs_bi\uŠNu vtuxdy_bau 1 uxEd ljPqy^hd“;_ 
ec‰Ga 7 221 83utEd ljli\u*fku vtuxacvtfku ikdGevxaciˆ_ ec‰Ga fku‘vxuxavxf\u u Ni\dGecvtaciˆJ_ 7ŠGjldGv ;   e‰G9a ;  7ut_ fkute
 jli\dy_be 1 J;   J;  ejldy_ ^[f\!i ldGstexZ  a jld ^ ;;   ‡   ;  u_ ;;  ‡   ; 1  7ŠGjldGv‘fkute
ŠGacjliˆ_but4e 7  1  8`ut9_ 7 ;   ;  8Aejldy_ G^[ab^[f\fkŽtfkuxetZ ] ƒ~i\r-"^ lu( ^[a VŠGu fg^ ŠGacj[i\_bu 7  1  8 uxe _‰GdNu
ŠGacjliˆ_bu  ^[ac^[fkf\Žxf\#u  ^[ecec^[dy_  ^ha 1 Lvhƒ~ute_wŠGj[dGvmfk^&ŠNacjliˆ_b&u 7 1 ;# 8 ZR]>ƒ…ikr‡^ lunŠGu1f ƒ…ikdy_cuxaceuxv_bikj[d


ŠGuxeŠNacjliˆ_buxe 7  1  8 7  2  8' 7  1  89uxe_ŠGjldGv f ƒ…ikdy_butacecutvt_cikjldmŠGuxeŠNacjliˆ_buxe 7 1 ; 8 7 2 # 8 7 1  8*
ŠGjldGvnv[ƒ…uxe _fk#u Rjlikdy_ 5‘ZLŒ^[a7vxj[dGec3s  ‰Nuxdy_ f\ux&e Rjlikdy_be    J5œejldy_ ^hfk!i [dGsxeŠG^[dGe7vxu_ j[acŠGau
ut_jld +^ 5  ‡3   Z
!  ] ƒ…„Gjlr&j[_b„Nst_bi\uŠGuvxuxdy_cacu  u_wŠGu ac^  Rjla_ ‚-,  ‡uxEd [jliku  ec‰Na fku r&ikf\ikut‰ ŠN‰ ecu 
r‡utd“!_   5  hjlaIuxfkf\u utEd ljli\u 12 1 e‰Ga 1  2  1  ŠNjldGv utfkf\u ut0d [jliku fku vxuxavxf\u ikdNecvxai\_IŠGuf ƒ~‰Ndec‰Na
vxuxf\‰Gi ŠGunf ƒ{^[‰›_bac"u GŠGjldNv1f\uwvxutd“_cacu1ŠG‰ vxuxavxf\u1i\dGecvtaciˆ_Š ^[dGe 1  2  1  uxe _ f\uwr‡i\fki\ux‰ŠGu   5  Z
¡=“Bh•yOQ•l=X 3

$mƒ{^ NacŽxe fkuw_b„Nsxjl>a ?xr&umŠGu = stdGsxfk ^ Ge "^  Gfki “‰Gs1^[‰ _cacik^[ d lf\u 221  fkut,e Rjli\d“_c.e ; @
ecjldy_ ^[fki ldGste eci ut_ eux‰Gf\uxr&uxdy_ eci jld‡^
 ‡ , Z y jli\*_ (fk#^ h^[f\ux‰Ga ŠNuf ƒ{^h d lfku 22 11 u_

 

(fg ^ h^[fkut‰Ga>ŠGu7f ƒ~^[ d lf\u 1  1 Z y jli\uxdy_  ut_ f\ux.e Gaj ut_csx(e ŠNux1e  jli\dy_be  2 u_ "ec‰Na 1.1 jld
 

^`^[fkjlae
‡   ‡   CZ $mƒ{^[‰›_bacu  ^ha_jld ^  ‡ (    ‡   Z i jlr&r‡u 1221 

  

uxe_ ‰G_d  ^[ab^hfkfkstfkj [ab^[r&r‡ujld ^  2-1 ‡  1  ^[amvxjldNecs “‰Gutdy_-f\uxe_cacig^h d lfkute 1 2 u_


ejldy_ ecutr Gfk^[Gfkute7Š ƒ~j b  ‡ ZR]9u Gacj ŠG‰Giˆ_ŠGumvtuxe_bacj[ike7ab^  jla _be h^[‰N_ ŠGjldGv ,

 
1 
ut_fkut&e  jli\dy_be ;  @ ejldy_ ^[fki ldGsxetZ
 


o 
¡=“Bh•yOQ•l=XŸ¢M

i j[r‡r&uxd jtjldGe ^[a+Q^hikacu ‰GdGuBCl‰Gau 5


lB
P
M
l

P B
lA

C
R

R Q

lj i\_2; &  f\uxe


Rjli\d“_ce Š ƒ…ikdy_butaceuxvt_cikjld ŠNufg^ŠNacjliˆ_bu ^Jlutv6fkuxev[_csxe 12  221  11
Q
y
ŠG‰ _cacik^[d lf\u ut_;      fkute e q r‡s_baci“‰Guxe ŠGu u_      f\uxe3eq r&st_caci!“‰GuteŠGu‘fg^
ŠGacjliˆ_bu A ^[aab^ Rjla _(vxute`r@?tr‡u-v[_csxexZ y jli\_ f ƒ…ikdy_butacecutvt_cikjld ŠNuxe`ŠGajli\_cuxe  ut_  Z
 

]9u6_cacig^h d lfk<u ;      ute_‡f ƒ…ikr‡^ lu‡ŠG‰ _cacik^[ d lf\ujla_c„Gi!“‰Gu6ŠNu 1 221  ^[a`f ƒ~„Gj[r‡j[_c„Gst_cikuŠNu




vxuxdy_cacu u_7ŠGunab^  jla _ ›Z  d ^&ŠGj[dGv    ;    ‡ — ‚ 1 Z„$ƒ~^[‰N_cacu ^[a _7jld ^

      ‡
—      
‚ ‚ 
‚    
f ƒ~^[d lf\u      st_b^[dy_7ikvxif ƒ{^hd lfku jlN_c‰% jld ^&utd vtjldGeikŠGstab^[dy_f ƒ~^[d lf\uw^[il‰
      ‡              ‡      7 —      8'
‚ ‚ ‚ ‚
wa&G^[avxj vtq vxf\ikvti\_bswj[d ^
     ‡O— ‚ 1  Š ƒ~jb
      ‡ — ‰   7 — 1 8.‡O— 1 ‡    ;    
‚ ‚ ‚
Œ^[a vxjldGes“‰Gutd“_1f\u Rjlikdy_ ^  ^ha_bi\uxdy_ ^[‰(vxutacvtfku vtikacvtjldGevxaciˆ_& ;       i\f9exƒ~^li\_ ŠGjldNv
ŠGu fk^‘euxvtjldGŠGu(i\dy_buxaecutvt_bi\jldVŠNu  ^Jl utv(vxu vxutacvtfku[Zc dVr&jldy_bau6“ ‰ ƒ…ikfutd uxe _ŠGu r@?tr‡u
^7[uxvmf ƒ…ikdy_butacecutvt_cikjld ŠGuxe ŠGacjliˆ_bute  u_ vL ŠGj[dGvvtuxe _cacjli\e7ŠGacjliˆ_bute ejldy_ vxj[dGvxjl‰Nab^[dy_butexZ%wa
fku1vxuxavxf\u vxi\acvxj[dGecvtaci\_<;# #Z  ute_wf\uvtuxacvtfku vtikavxjldGevxai\_ 1221 Ÿ ŠNjldGvf\uxe_bacj[ike ŠNacjliˆ_buxe
       ecunvxj[‰  utdy_ e‰Gafku vtuxacvtfkunvxi\acvtjldGecvtaciˆ_, 1 221 Z
¡=“Bh•yOQ•l=X U

y jli\_  ut_  fkuxeGajuxv_bi\jldGeŠGu5 e‰Ga 1 2 u_ 1.1 acuteRuxvt_ci[uxr&uxdy_CZ i jlr&r‡u  u_


5 ecj[d“_ ec‰Ga fg^nGikeecuxv_baikvxu ŠGu1 ›  jld^  ‡ 1 u_   5 ‡?5  Z a jld6^ 2 5 ‡ 1 5
ut_    2 5 ‡    1 5W ŠGj[dGvfkutem_cacig^hd lfkute 2 5   u_ 1 5  ecj[d“_ ec‰ Ruxa> jlec^[Gf\uxeŠ ƒ…jb
2   ‡ 1  Z
y jli\_ f ƒ{^[dlfku r&jliˆ_biks ŠGunf ƒ~^[d lf\u1 N jld^
 1221  ‡ 1 2       ‰ 11  1    ‡ 7 12 ‰ 11 8      

o[˜
waj[d^ 12 ‰ 11 ‡ 1  ‰ 1 OŸŠ ƒ…jb
 1 221  ‡ 71  ‰ 1   8   8  ‰      ‰  8 `   
  W‡
 1

Š ƒ~jb2G^[a ^[ab^hfkfkstfki\ecr&u ŠGuA7  8u_27  548u_ŠGuA7 1 :8ut_ 7 5N 8


 12 1  ‡  1 1  ‰  O5  ‰  1  ?5  ‡  1  5   

¡=“Bh•yOQ•l=X V




 
 





lj i\_ œf ƒ…jla_c„Gj›vtuxdy_bau ŠG‰6_cacig^hd lfku 12 1   ecj[d6vxuxdy_cacunŠGulac^J›iˆ_bs u_”$f\u vxutdy_bacuwŠNu




y
ecjld vxuxavxf\uvxi\acvtjldGecvtaciˆ_CZ y jliˆ_ 1  2  1  f ƒ…ikr‡^luwŠGu 1221  ^ha f ƒ~„Gj[r‡j[_c„Gst_ciku ŠGumvxuxdy_cacu 

ut_wŠGuab^ Rjla _#  ecjli\uxdy_  ut_”  fkute ikr‡^lute ŠGu ut_”  ^[a Z y jli\_  . *fkute


Gacj utvt_cikjldGeŠNu ec‰Naf\uxeŠGajli\_cuxe7 2  1  8*7 1  1  8*,7 1  2  8Z


  

]9^wŠGi\e_|^hdGvxu7ŠGu  wfg^wŠGajli\_cu7 2  1  8uxe _ ##+”jli\efg^wŠGi\e_|^hdGvxuŠGu  wfg^ ŠNacjliˆ_bu7 2 1 8u_


fg^nŠGike _|^[dGvtu ŠNu 1 <7 221 8 uxe 0_ +”jlike>vxu_c_cu ŠGi\e_|^hdGvxu"]G^[a vtjldGes“‰Guxdy_fk^1ŠNike_b^[dGvtu utd“_cacu 1
ut-_ 7 2 1 8ute_ s l^[fkcu nfk^ ŠGi\e_|^hdGvxuutd“_cacu7fkuteŠGajli\_cuxeI7 2 1 8u_-7 2  1  8*[ŠNjldGvfkuc jli\dy_ uxe _
fkueq r&st_cac!i “‰Gu7ŠG ‰ Rjlikdy_ 1  ^[a>ac^  Rjla1_  221 EZ  f uxd&uxe _ ŠNu r ?xr&&u Rjl‰Ga fkut.e  jli\dy_be u_

ZNŒ^[a vtjldGec3s “‰Guxdy_7fkutce Rjlikdy_ce  ut_ ecjldy_^hfk!i [dGsxe eciRut_ ecux‰Nfkuxr&uxdy_ei f\uARjli\d“_


5^   ^ha_bi\uxdy_w^h‰ vxuxavxf\u vxi\acvxj[dGecvtac i\_ ^[‰_cacig^h d lfku 1  2  1   ŠGjldGv eiu_wecut‰Gfkutr‡utd“_ ecij[d ^
  

”  ‡ +7 1  2  1  8‡:#X+7 1221 87utd(dGj[_b^[dy_ +7 1221 87f\umac^Cqljld(ŠN‰vxutacvtfkuvtikavxjldGevxai\_ 


1221 Z

] ƒ…ikr‡^ luwŠGu1fg^AŠGajli\_cu1ŠŸƒ 4 ‰Gf\ux,a  ^[a uxe_ uxf\fku r ?xr&uwvC^[a  Rvtuxdy_bauŠGu %Rute_ ŠGuxeec‰GetZ
wd ^*ŠGjldGv ”‡  ‰  ”#‡  ‰ #E”  ‡ ”  ‰ #E”  ‡”  ‡ ” *Z
 

Œ^[a3vxj[dGec3s  ‰Nuxdy_ uxe _ ec‰Ga3fku vtuxavxfku vtikacvtjldGevxaciˆ2_  1  2  1  eiwut_ ecut‰Gfkutr‡utdy_ eiwjld ^


” ‡3H+7 1221 8 ut_fk


^ Gacut‰ lumuxe_ B dNiku[Z

¡=“Bh•yOQ•l=™L š

y jli\_ ”  fku`vtuxdy_bau‡ŠG‰ vtuxacvtfkuAvxi\acvtjldGecvtaciˆ_  1  2  1  KZ Š dGu BC[‰GacuAec‰ lŽxauAŠGuAr‡jldy_cacuxa


 ‰Nu`fkuxe  jli\dy_be 5u_ ”  ecjldy_nvxj[d +”jldGŠG‰N e FQGi\uxd “‰ ƒ…jld dGu`eb^hv|„Gu  ^[ewutdGvxjlau ~“‰Gjlivtuxfk^

z#
 tu ‰N_ euxa> i\aSIZ i jlr&r‡umjld ^ 2 1 ‡ — ‚ 2  11 ‡ — ‚ 2 1  1  f\uxeA jli\dy_be 2   1  1 
ecjldy_ vtj›vq vxfki“‰GuxetZ|$ uGfk‰Ge fku1ab^Cqlj[d(ŠG‰(vtuxacvtfkumvxikavxjldNecvxai\_-vxuteRjlikdy_beuxe _ s3y^[f ‡vxutfk‰Gi

ŠG‰vxutacvtfku vtikavxjldGevxai\_c 1221 G‰Gi\e“‰ ƒ…ikfLexƒ~^liˆ_ ŠN‰vxutacvxf\u eq r&st_caci!“‰GuA ^[a ab^ Rjla _ 221 Z
$ u r ?xr&unfkux,e Rjlikdy_be 1   2  1  ejldy_ vtj›vq›vtfki ‰Nuxe u_ f\umac^Cqljld ŠG‰(vxutacvtfku ^[ecec^[dy_# ^[a
vxux.e Rjlikdy_ce uxe_ 3s y^[Cf nvxutfk‰GiGŠN‰‡vtuxacvtfku& ^heceb^hd“_8 ^[a 2   1  1  ZE$ƒ~^GacŽte f ƒ~u ›uxacvtikvtu o[oalfg^

vxjlaŠGu 1  1  s_|^[dy_7vtjlr‡r‰GdGuw^[‰ 6ŠGut‰›6vtuxacvtfkute Gjld3^ 1 ‡ 1  ›ŠGuwr ?xr&u jld6r&jldy_bacu


2 Lut_ŠGjldNv ute_fkuwvtuxdy_baumŠG‰ vxutacvtfkunvxi\acvxj[dGecvtaci\_, 1  2  1   ŠŸƒ~jb ‡Q”gZ



1 a‡












= jldy_bacut2a ” ‡ ”  au3 ikutdy_ r&jldy_bacuta(“‰Gu f\u(_baig^[d [fku ”  ute_ ikej›vtŽxfku utd
”‡Z y lj iˆ_A;œfg^ Gacjutvt_cikjld ŠGu ” e‰Ga  >vxutfg^ acu*›i\uxdy_utdGvxj[acu6 r&jldy_bacuta@“‰Gu; uxe _

fku r‡i\fki\ux‰ ŠGu  Z]uxe3_bacik^[d lf\uxe 1 221 ut_ 1  2  1  s_|^[dy_ ecuxrGfg^[Nfkuxe3ikfwu Ni\e_cu ‰GdNu


ecikr&ikf\i\_c‰GŠGu˜&uxdE[j€ql^[dy_ 1 221 ec‰Ga 1  2  1  Z y jli\_ fku`ac^ Rjla_ ŠGu@˜‡ut_- ecj[d*^[dlfkuhZK d


^ 9; ‡ ” "!  ut_  ‡  ”  ‡  ”&Z„ dv|„Gutacv|„GuŠGjldGv ‡r&jldy_bauxa7fg^`acutfg^h_cikjld




@‡   "!  Z


ŒIjl‰Ga&r‡j[d“_cacuta`vxut__bu acutfg^h_cikjld “‰GidGu ŠGs3 utdGŠ€“‰Gu ŠGu3fg^ vxj[dGe_cac‰Gv_bikj[d i\dGi\_cig^[f\u‡ŠGute




_bacik^[ d lf\ux3e Gjl6d h^Avxjl r Gf\Žt_butr‡utdy_ j[‰GGfki\uxa7fkute jli\dy_be „”


ut_  Rjl‰Ga7au3lutdGika ^[‰› eux‰Gf\e
v [_csxeŠGuxe7_baig^[ d [fkuxetZ y j[i\_ 1   u_ 1   fkut,e Nacj u_bsxeŠGu 1 ut_ 1 ec‰Ga 1  1  ZL] ƒ{^[d lfkuwutd“_cacu1fkute


ŠGacjliˆ_butIe 7 11 8ut._ 7 1  1  8 h^[fk^[dy*_ 8ljld‡^ 1   1   ‡ 11  !  0Z  a jld‡^ 1  1 ‡ M 11 yŠGjldNv
ikfŸe‰ )A_ ŠNu1r&jldy_bauxafk^ acuxfk^h_bi\jld  1   1  ‡ 1  1 gZ


y jli\_ 2   fg^ euxvxj[dGŠGuikdy_butacecutvt_cikjldAŠG‰Avxutacvtfkuvxi\acvtjldGecvtaciˆ1_  1221 J^ [uxvfg^ ŠGajli\_cu 7 1  1  8Z


Œ^[aIvxj vtq vxf\ikvti\_bs>ŠGut–e Rjlikdy_ce 1  2  1   2   jld ^ F [jlika9ec‰Nafk^ BC[‰Gac8u  jl‰NaIfk,^ Rjlei\_bi\jldnacutfg^h_ci lu
ŠGuxe Rjlikdy_bƒe I fg^ auxfk^h_bi\jld 1 2   1  ‡ 112 ‡ 1 1  2   GŠGjldGvwf\u _baig^[ d [fku 1 2   1  uxe _7ikej›vtŽxfku
uxd 1 ZLŒ^[avxj[dGec3s  ‰Nuxdy_wjld^ 1   1 „‡ 1   2   ut_ jld r‡j[d“_cacunŠGu1@r ?tr‡u 1   1 „‡ 1   2  gZR]9^
acuxfk^h_bi\jldauxv|„Gutacv|„Gstu exƒ…jlN_cikuxdy,_  ^[aejlr&r‡uwŠGunvtuxe7ŠGux‰  ŠGuxadGikŽtacuxe73s y^[fkiˆ_bstexZ

zNY
Transformations géométriques

Thomas Budzinski

Table des matières

1 Symétries centrales et axiales, translations 2

2 Homothéties 4
2.1 Définitions et propriétés de base : . . . . . . . . . . . . . . . . . . . . . . . . . . . 4
2.2 Quelques applications classiques : . . . . . . . . . . . . . . . . . . . . . . . . . . 6
2.3 Homothéties et cercles : . . . . . . . . . . . . . . . . . . . . . . . . . . . . . . . . 8
2.4 Chasse aux tangentes . . . . . . . . . . . . . . . . . . . . . . . . . . . . . . . . . . 10
2.5 Conseils pour les exercices . . . . . . . . . . . . . . . . . . . . . . . . . . . . . . . 12
2.6 Exercices . . . . . . . . . . . . . . . . . . . . . . . . . . . . . . . . . . . . . . . . . 12

3 Rotations 14
3.1 Cours . . . . . . . . . . . . . . . . . . . . . . . . . . . . . . . . . . . . . . . . . . . 14
3.2 Conseils pour les exercices . . . . . . . . . . . . . . . . . . . . . . . . . . . . . . . 16
3.3 Exercices . . . . . . . . . . . . . . . . . . . . . . . . . . . . . . . . . . . . . . . . . 16

4 Similitudes directes 17
4.1 Définitions et propriétés de base . . . . . . . . . . . . . . . . . . . . . . . . . . . 17
4.2 Centre d’une similitude . . . . . . . . . . . . . . . . . . . . . . . . . . . . . . . . 19
4.3 Deux similitudes pour le prix d’une . . . . . . . . . . . . . . . . . . . . . . . . . 21
4.4 Similitudes indirectes . . . . . . . . . . . . . . . . . . . . . . . . . . . . . . . . . . 22
4.5 Conseils pour les exercices : . . . . . . . . . . . . . . . . . . . . . . . . . . . . . . 22
4.6 Exercices . . . . . . . . . . . . . . . . . . . . . . . . . . . . . . . . . . . . . . . . . 23

5 Indications pour les exercices 25

6 Solutions des exercices 26

Introduction
Une transformation géométrique est une bijection du plan dans lui-même, c’est-à-dire une
manière d’associer à chaque point un autre point, de telle manière que tout point soit l’image

1
d’un autre point, et que deux points différents aient des images différentes. Les transforma-
tions auxquelles on va s’intéresser conservent la plupart des propriétés géométriques inté-
ressantes. Par exemple, toutes envoient une droite sur une droite, un cercle sur un cercle, un
angle de 42◦ sur un angle de 42◦ , un carré sur un carré et ainsi de suite...
Le fait de s’intéresser aux transformations qui préservent certaines propriétés est un élé-
ment essentiel de la géométrie "moderne" (c’est-à-dire celle pratiquée depuis le XIX e siècle)
que vous rencontrerez si vous continuez des études en mathématiques. Cependant, les trans-
formations du plan sont aussi très utiles dans la résolution de problèmes de géométrie plus
élémentaires. Le but de ce document est de vous présenter les transformations usuelles du
plan et la manière de les utiliser pour résoudre des problèmes de géométrie de type olym-
pique. Il est possible de sauter les preuves en première lecture, la plupart des résultats étant
de toute façon assez intuitifs.
Les exercices de ce cours sont de difficultés variables mais rarement très faciles. Si vous
bloquez sur un exercice, vous pouvez d’abord consulter les indications qui se trouvent avant
les solutions. Il est conseillé d’avoir déjà lu un cours de géométrie de type olympique de
niveau débutant avant d’aborder celui-ci.
Ce polycopié se suffit en principe à lui-même, mais le lecteur intéressé pourra également
lire les livres "Geometric Transformations" de Yaglom (en anglais... ou en russe !). Les deux
premiers tomes couvrent le contenu de ce cours et le troisième aborde un sujet plus avancé :
les transformations projectives.

1 Symétries centrales et axiales, translations


Vous connaissez déjà certaines transformations du plan : la symétrie centrale, la symétrie
axiale et, si vous êtes au moins en Seconde, la translation. Rappelons tout de même leurs
définitions :
Définition 1.1. Soit (d) une droite. La symétrie axiale d’axe (d) est la transformation qui à tout
point M associe le point M 0 tel que (d) soit la médiatrice de [M M 0 ].
Définition 1.2. Soit O un point. La symétrie centrale de centre O est la transformation qui à tout
point M associe le point M 0 tel que O soit le milieu de [M M 0 ].
Définition 1.3. Un vecteur, noté →−
v , est un objet géométrique caractérisé par une direction, un
sens et une longueur. On dessine un vecteur avec une flèche. Le vecteur partant du point A et
−→
allant jusqu’au point B est noté AB.
−→ −→
Exemple 1.4. Sur la figure 1, on a AB = → −v car AB et →−
v ont la même direction (20◦ avec
l’horizontale), le même sens (vers la droite) et la même longueur.
Définition 1.5. Soit →

v un vecteur. La translation de vecteur →

v est la transformation qui à tout
0
−−−→0 →

point M associe le point M tel que M M = v .
Etant donnée une transformation géométrique, il est toujours intéressant de savoir quelles
propriétés elle conserve :
Proposition 1.6. a) Les symétries centrales et axiales et les translations conservent les
droites, les cercles, les angles, les longueurs.

2
B


v
−→
AB

F IGURE 1 – Deux vecteurs égaux.


→ (d)
v

F IGURE 2 – Une figure et ses images par la symétrie centrale (en bleu), la symétrie axiale (en
vert), et une translation (en rouge). Notons que l’image par la symétrie axiale lève le bras
droit, car la symétrie axiale est la seule transformation qui inverse les angles orientés.

3
b) Les symétries centrales et les translations conservent les angles orientés : si A, B et C
−−→ −−→ −→ −→
ont pour image A0 , B 0 et C 0 , alors (A0 B 0 , A0 C 0 ) = (AB, AC).
c) Les symétries axiales inversent les angles orientés : si A, B et C ont pour image A0 , B 0
−−→ −−→ −→ −→
et C 0 , alors (A0 B 0 , A0 C 0 ) = −(AB, AC).
Bien que très simples, ces transformations permettent déjà de résoudre certains exercices,
notamment des problèmes de construction :
Exercice 1 On considère une rivière droite de largeur L et deux points A et B de part et d’autre
de cette rivière. On veut construire un pont perpendiculaire à la rivière. Où le construire pour
que le trajet de A à B soit le moins long possible ?
Exercice 2 Etant donnés deux points A et C et un cercle Γ, construire deux points B et D sur
Γ tels que ABCD soit un parallélogramme.
Exercice 3 On se donne un cercle Γ de rayon r, une droite (d) et une longueur a ≤ 2r.
Construire une droite parallèle à (d) qui coupe Γ en X et Y telle que XY = a.

2 Homothéties
2.1 Définitions et propriétés de base :
Nous allons maintenant étudier une classe de transformations plus large et plus utile que
celles que nous connaissons déjà : les homothéties. Les homothéties sont des "agrandisse-
ments" ou des "réductions". Avant de les définir, commençons par un complément sur les
vecteurs :
Définition 2.1. Soient k un nombre réel et →−
v un vecteur :
— Si k ≥ 0, on note k v le vecteur de même direction et de même sens que →

− −
v et dont la
longueur est k fois celle de →

v.
— Si k ≤ 0, on note k →−v le vecteur de même direction que →

v , de sens opposé à celui de →

v


et dont la longueur est −k fois celle de v .



u



v = −2−

u

F IGURE 3 – Exemple de multiplication d’un vecteur par −2.

Définition 2.2. Soient O un point du plan et k un nombre réel. L’homothétie de centre O et de


−−→ −−→
rapport k est la transformation qui à tout point M associe le point M 0 tel que OM 0 = k OM .
Exemple 2.3. Les symétries centrales sont des homothéties de rapport −1.
Les homothéties conservent aussi de nombreuses propriétés :

4
O

F IGURE 4 – Une figure et son image par les homothéties de centre O et de rapport 2 (en bleu)
et − 21 (en rouge).

Proposition 2.4. a) Les homothéties conservent les droites, les cercles, les angles orientés.
b) L’image d’une droite (d) par une homothétie est parallèle à (d).
c) Une homothétie de rapport k multiplie toutes les longueurs par |k|. Elle conserve donc
les rapports de longueurs.
0 0
Démonstration. b) Soient A et B deux points et A0 et B 0 leurs images : on a OA
OA
= |k| = OB
OB
donc d’après le théorème de Thalès (A0 B 0 )//(AB).
0B0 0
c) D’après b) et le théorème de Thalès, on a AAB = OA
OA
= |k|.
a) Soit C un troisième point et C 0 son image : (A0 B 0 )//(AB) et (B 0 C 0 )//(BC) donc on a
A\0 B 0 C 0 = ABC
[ donc l’homothétie conserve les angles, donc elle conserve toutes les
propriétés qui peuvent s’exprimer avec des angles, comme l’alignement ou la cocycli-
cité.

Un intérêt important des transformations géométriques est la possibilité de les composer :


Définition 2.5. Soient t1 et t2 deux transformations : on note t2 ◦ t1 la transformation qui à tout
point M associe t2 (t1 (M )). Cela revient à appliquer t1 puis t2 .
Théorème 2.6. Soient h1 et h2 deux homothéties de centres O1 et O2 et de rapports k1 et k2 :
a) Si k1 k2 6= 1, alors h2 ◦h1 est une homothétie de rapport k1 k2 dont le centre est sur (O1 O2 ).

5
b) Si k1 k2 = 1, alors h2 ◦ h1 est une translation de vecteur parallèle à (O1 O2 ).
Démonstration. a) On admet que la composée est une homothétie, cela peut se prouver
soit avec du calcul vectoriel, soit comme cas particulier des résultats sur la composée de
deux similitudes qu’on verra plus loin dans le cours. Comme h1 multiplie les longueurs
(algébriques) par k1 et h2 par k2 , la composée les multiplie par k1 k2 donc son rapport est
k1 k2 . De plus, le centre de h2 ◦ h1 est aligné avec O1 et h2 ◦ h1 (O1 ) = h2 (O1 ). Ce dernier
point doit être sur (O1 O2 ) donc le centre de h2 ◦ h1 aussi.
b) On admet que la composée est une translation et on note O3 = h2 ◦ h1 (O1 ) = h2 (O1 ) :
−−−→
O3 est aligné avec O1 et O2 donc O1 O3 est parallèle à (O1 O2 ), mais c’est justement le
vecteur de notre translation.

Il est également utile de savoir quand on peut introduire une homothétie qui envoie un
certain objet sur un autre :
Proposition 2.7. a) Soient A, B, A0 et B 0 quatre points : il existe une homothétie ou trans-
lation qui envoie A sur A0 et B sur B 0 si et seulement si (AB)//(A0 B 0 ). De plus, dans ce
cas, une telle transformation est unique.
b) Soient O, A et A0 alignés : il existe une unique homothétie de centre O qui envoie A sur
A0 .
Démonstration. a) Si (AB) et (A0 B 0 ) ne sont pas parallèles, une telle homothétie ne peut
pas exister.
Si elles le sont, le centre doit être sur (AA0 ) et sur (BB 0 ) : soit ces droites sont parallèles,
et alors AA0 B 0 B est un parallélogramme et notre transformation est la translation de
−−→ −−→ 0 0
vecteur AA0 = BB 0 , soit elles se coupent en X avec XA XA
= XB
XB
par Thalès, donc notre
XA0
homothétie est celle de centre X et de rapport XA .
0
b) Il s’agit de l’homothétie de centre O et de rapport OA OA
(en longueurs algébriques).

2.2 Quelques applications classiques :


Commençons par ce résultat que vous connaissez certainement déjà :
Théorème 2.8. Les trois médianes d’un triangle sont concourantes. De plus, le point d’inter-
section se trouve aux deux tiers des médianes.

B0 C0

C B

Démonstration.

6
On note A0 , B 0 et C 0 les milieux de [BC], [CA] et [AB]. On sait que (B 0 C 0 )//(BC), donc il
existe une homothétie h qui envoie B sur B 0 et C sur C 0 . De plus, le centre de h est (BB 0 ) ∩
(CC 0 ). On le note G. Ce point G est sur le segment [BB 0 ] donc h est de rapport négatif, et
B 0 C 0 = 21 BC donc h est de rapport − 12 , donc GB 0 = 21 GB donc BG = 23 BB 0 , et de même
CG = 32 CC 0 .
Le point aux deux tiers de [BB 0 ] est donc le même que le point aux deux tiers de [CC 0 ]. En
faisant le même raisonnement sur A et B, on montre que ce point est aussi aux deux tiers de
[AA0 ].
Remarquons que l’homothétie de centre G et de rapport − 12 envoie donc chaque sommet
de ABC sur le mileu du côté opposé.
Passons à la droite et au cercle d’Euler, deux autres applications classiques des homothé-
ties :

Théorème 2.9. (Droite et cercle d’Euler)


Soient ABC un triangle, O le centre de son cercle circonscrit, G son centre de gravité (i.e le
point d’intersection de ses médianes) et H son orthocentre (i.e le point d’intersection de ses
hauteurs). On note A0 , B 0 et C 0 les milieux de [BC], [CA] et [AB] et D, E, F les pieds des
hauteurs issues de A, B et C. Alors :
a) O, G et H sont alignés.
b) Les points A0 , B 0 , C 0 , D, E et F sont cocycliques.

7
A
E

F
H
B0
Ω C0
G

O
C

A0 D
B

Démonstration. a) Soit h l’homothétie de centre G et de rapport − 12 : on a h(ABC) =


A0 B 0 C 0 , donc h envoie H sur l’orthocentre de A0 B 0 C 0 . Or, O est l’orthocentre de A0 B 0 C 0 .
En effet, (OA0 ) est perpendiculaire à (BC), donc à (B 0 C 0 ) par le théorème de la droite
des milieux, donc c’est la hauteur issue de A0 dans A0 B 0 C 0 .
−→ −−→
O, G et H sont donc alignés et, plus précisément, GO = − 12 GH.
b) Soit Ω le centre du cercle circonscrit à A0 B 0 C 0 : on sait que Ω = h(O) donc GΩ = 12 GO
donc OΩ = 23 OG = 21 OH, donc Ω est donc le milieu de [OH].
Le point Ω est donc équidistant des droites (AD) et (OA0 ), donc ΩA0 = ΩD. Pour mon-
trer cela proprement, on peut par exemple introduire X le projeté orthogonal de Ω sur
(BC) : par Thalès, X est le milieu de [A0 D] donc (ΩX) est la médiatrice de [A0 D].
On en déduit que D est sur le cercle circonscrit à A0 B 0 C 0 , et de même pour E et F .

2.3 Homothéties et cercles :


Les homothéties font également très bon ménage avec les cercles, ce qui peut être utile dès
que de nombreuses tangentes apparaissent dans un problème.

Proposition 2.10. (i) Soient C1 et C2 deux cercles. Il existe exactement deux homothéties

8
O1
O2
X C1 Y

C2

F IGURE 5 – Deux cercles C1 et C2 . X est le centre de l’homothétie positive qui envoie C1 sur C2
et Y le centre de l’homothétie négative.

qui envoie C1 sur C2 : une de rapport positif et une de rapport négatif. Si les deux
cercles sont de même rayon, la première est une translation.
(ii) Si aucun des deux cercles n’est à l’intérieur de l’autre, le centre de l’homothétie positive
est le point d’intersection des tangentes communes extérieures à C1 et C2 .
(iii) Si les deux cercles ne s’intersectent pas, le centre de l’homothétie positive est le point
d’intersection des tangentes communes extérieures à C1 et C2 .
Remarque 2.11. Les points (ii) et (iii) ont des cas dégénérés intéressants : si les deux cercles
sont tangents extérieurement, alors le point de tangence est le centre de l’homothétie négative
qui envoie l’un sur l’autre. Si ils sont tangents intérieurement, le point de tangence est le centre
de l’homothétie positive.
Démonstration. (i) Admis
(ii) Soit h l’homothétie positive qui envoie C1 sur C2 et (t) une des deux tangentes com-
munes : h(t) est parallèle à t et tangente à C2 . Comme on veut une homothétie positive,
elle doit de plus être du même côté de C2 que (t) de C1 , donc h(t) = (t) donc le centre
de h doit être sur t. Il en est de même pour l’autre tangente commune extérieure, donc
le centre est l’intersection des tangentes communes extérieures.
(iii) Similaire à (ii).

En combinant cette proposition avec le fait que le centre d’une composée d’homothéties
est aligné avec les centres des deux homothéties (voir théorème 2.6), on obtient :
Théorème 2.12. (Théorème de Monge) Soient C1 , C2 et C3 trois cercles tels qu’aucun ne soit en-
tièrement à l’intérieur d’un autre. On note X le point d’intersection des tangentes communes

9
C1

C2
C3
Y

F IGURE 6 – Le théorème de Monge.

extérieures à C1 et C2 , Y celui de C2 et C3 ainsi que Z celui de C3 et C1 .


Alors X, Y et Z sont alignés.

Démonstration. X est le centre de l’homothétie positive h1 qui envoie C1 sur C2 et Y celui de


l’homothétie positive h2 qui envoie C2 sur C3 . De même Z est le centre de l’homothétie posi-
tive h3 qui envoie C1 sur C3 . Mais d’après la proposition précédente, il n’existe qu’une telle
homothétie donc h3 = h2 ◦ h1 , donc les centres des trois homothéties sont alignés.

Remarque 2.13. Ce théorème reste vrai si on remplace exactement deux fois les tangentes
communes extérieures par les tangentes communes intérieures. La preuve est à peu près la
même mais avec cette fois une homothétie négative qui est la composée d’une négative et
d’une positive.

2.4 Chasse aux tangentes


Nous terminons cette section par une technique qui n’est pas directement reliée aux homo-
théties mais qui intervient souvent dans le même type de problèmes : la chasse aux tangentes,
qui "remplace" la chasse aux angles dans certains problèmes.
La chasse aux angles, que vous connaissez déjà bien, consiste à exploiter le fait que des
points soient sur des cercles pour obtenir des égalités d’angles et trouver d’autres points co-
cycliques. La chasse aux tangentes consiste à exploiter le fait que des droites soient tangentes à
des cercles pour obtenir des égalités de longueur et trouver d’autres droites tangentes. L’idée
est d’utiliser de manière répétée le résultat facile suivant :

Lemme 2.14. Soit C un cercle et A un point extérieur à C. Les tangentes à C passant par A
touchent C en X et Y .
Alors AX = AY .

Un premier résultat de chasse aux tangentes est le suivant, que vous connaissez peut-être
déjà.

10
Proposition 2.15. Soit ABC un triangle. On pose a = BC, b = CA et c = AB. Le cercle inscrit
à ABC touche [BC] en X. Le cercle A-exinscrit à ABC touche [BC] en T (rappelons que le
cercle A-exinscrit est le cercle tangent au côté [BC] et aux demi-droites [AB) et [AC) au-delà
de B et C).
Alors BX = CT = a+c−b 2
et CX = BT = a+b−c2
.

x x

Z
Y

z y

C X y
B
z T

Démonstration.
Notons Y et Z les points où le cercle inscrit touche [CA] et [AB] : on pose x = AY = AZ,
y = BZ = BX et z = CX = CY . On a alors y + z = a, z + x = b et x + y = c d’où
a+c−b
2
= y+z+x+y−z−x
2
= y et, de même, a+b−c
2
= z.
La preuve pour le cercle exinscrit est similaire (il faut introduire les points où il touche
[AB) et [AC)) et est laissée en exercice.
Un autre résultat classique issu de la chasse aux tangentes est le suivant.
Proposition 2.16. Soit ABCD un quadrilatère convexe. Alors ABCD admet un cercle inscrit
si et seulement si
AB + CD = BC + AD.

D
R
C

A B
P

Démonstration.

11
Commençons par le sens direct : si ABCD admet un cercle inscrit, on note P , Q, R et S les
points de tangence comme sur la figure et on effectue une chasse aux tangentes :

AB + CD = AP + BP + CR + DR = AS + BQ + CQ + DS = AD + BC

Le sens inverse est moins évident : si AB + CD = BC + AD, soit ω un cercle tangent à


[AB], [BC] et [AD] (son centre sera l’intersection des bissectrices de ABC \ Soit D0
[ et BAD).
l’intersection de (AD) avec la deuxième tangente à ω passant par C : le quadrilatère ABCD0
est convexe et admet un cercle inscrit donc AB + CD0 = AD0 + BD donc AD0 − CD0 =
AB − BC = AD − CD donc AD0 − AD = CD0 − CD. Si par exemple AD0 ≥ AD alors comme
A, D et D0 sont alignés, on alors DD0 = AD0 − AD = CD0 − CD donc CD0 = CD + DD0 donc
C, D et D0 sont alignés, donc D0 est sur (AD) et (CD) donc D0 = D. Le cas AD0 ≤ AD se traite
similairement.

Remarque 2.17. La ressemblance est frappante entre ce théorème et la caractérisation "ABCD


admet un cercle circonscrit si et seulement si A
b+C
b=B b + D".
b

2.5 Conseils pour les exercices


— Si il y a deux droites parallèles, il peut y avoir une homothétie intéressante qui envoie
l’une sur l’autre.
— Si vous repérez plusieurs homothéties intéressantes, essayez de les composer !
— Dès que vous repérez deux tangentes communes à deux cercles, intéressez-vous à une
homothétie qui envoie un cercle sur l’autre : vous connaissez son centre !
— Si il y a de nombreuses tangentes communes sur la figure, utilisez le théorème de
Monge ! N’hésitez pas à introduire des cercles supplémentaires (cercles inscrits par
exemple) pour pouvoir l’utiliser.
— Si il y a beaucoup de tangentes, pensez à la chasse aux tangentes !
— Si vous avez bien assimilé le cours, il peut être utile de retenir les exercices 7, 9 et 10 qui
peuvent se retrouver dans des problèmes plus compliqués.

2.6 Exercices
Attention : les derniers exercices sont très difficiles !
Exercice 4 Soient ABC et A0 B 0 C 0 deux triangles tels que (AB) soit parallèle à (A0 B 0 ), (BC) à
(B 0 C 0 ) et (CA) à (C 0 A0 ).
Montrer que les droites (AA0 ), (BB 0 ) et (CC 0 ) sont parallèles ou concourantes.
Exercice 5 Soit ABC un triangle avec trois angles aigus.
Construire un carré qui a deux sommets sur [BC], un sur [AB] et un sur [AC].
Exercice 6 Soient ABCD un trapèze avec (AB) parallèle à (CD), M le milieu de [AB] et P un
point de (BC). On pose X = (P D) ∩ (AB), Q = (P M ) ∩ (AC) et Y = (P Q) ∩ (AB).
Montrer que M est le milieu de [XY ].
Exercice 7 Soit ABCD un quadrilatère convexe tel que les cercles inscrits à ABC et ADC
soient tangents.
Montrer que ABCD admet un cercle inscrit.

12
Exercice 8 Deux cercles Γ1 et Γ2 sont tangents extérieurement en A, et une tangente commune
extérieure est tangente à Γ1 en B et à Γ2 en C. Soit D le point diamétralement opposé à B dans
Γ1 .
Montrer que A, C et D sont alignés.
Exercice 9 Soit ABC un triangle. On note Γ son cercle inscrit et ΓA son cercle A-exinscrit. Le
cercle Γ touche [BC] en I, et ΓA touche [BC] en J. Soit K le point de Γ diamétralement opposé
à I.
Montrer que A, J et K sont alignés.
Exercice 10 Soient ω1 et ω2 deux cercles tangents en un point A avec ω2 à l’intérieur de ω1 . Soit
B ∈ ω2 différent de A. La tangente à ω2 en B recoupe ω1 en X et Y .
Montrer que BAX
\ = BAY [.
Exercice 11 Soient ABC un triangle et Γ son cercle circonscrit. Soit ΓA un cercle tangent à
(AB) et (AC), et tangent intérieurement à Γ en un point A0 . On définit de manière similaire
ΓB , B 0 , ΓC et C 0 .
Montrer que les droites (AA0 ), (BB 0 ) et (CC 0 ) sont concourantes.
Exercice 12 Soient ABC un triangle, I le centre de son cercle inscrit, D le point de contact du
cercle inscrit avec [BC], J le centre du cercle A-exinscrit, E le pied de la hauteur issue de A et
K le projeté orthogonal de I sur cette hauteur.
Montrer que (DK) et (EJ) sont parallèles.
Exercice 13 Soit ABC un triangle, D le point de contact du cercle inscrit avec [BC], I le centre
du cercle A-exinscrit et M le milieu de la hauteur issue de A.
Montrer que M , D et I sont alignés.
Exercice 14 Soit ABCD un quadrilatère convexe. On suppose que les cercles inscrits aux tri-
angles ABC, BCD, CDA et DAB ont un point commun.
Montrer que ABCD est un losange.
Exercice 15 Soient Γ un cercle et Γ1 , Γ2 deux cercles tangents intérieurement à Γ en A et B. On
note (d1 ) et (d2 ) les tangentes à Γ2 passant par A, et (d3 ) et (d4 ) les tangentes à Γ1 passant par
B.
Montrer que ces quatre doites forment un quadrilatère circonscriptible.
Exercice 16 Soit ABCD un quadrilatère circonscriptible et ω son cercle inscrit, de centre O.
On note X l’intersection de (AD) et (BC). Le cercle ω1 est tangent aux prolongements de [AD]
et [BC] et au côté [AB] en K. Le cercle ω2 est tangent aux prolongements de [AD] et [BC] et au
côté [CD] en L. On suppose que X, K et L sont alignés.
Montrer que O, le milieu de [AB] et le milieu de [CD] sont alignés.
Exercice 17 (Shortlist 2007) Soit ABCD un quadrilatère convexe et P sur [AB]. On note ω, ω1
et ω2 les cercles inscrits à P CD, P AD et P BC. On note I le centre de ω. On suppose que ω1 et
ω2 sont tangents à ω en K et L. On pose enfin E = (AC) ∩ (BD) et F = (AK) ∩ (BL).
Montrer que E, I et F sont alignés.
Exercice 18 (IMO 2008) Soit ABCD un quadrilatère convexe avec BA 6= BC. On note ω1 et ω2
les cercles inscrits à ABC et ADC. On suppose qu’il existe Ω tangent à (BA) au-delà de A, à
(BC) au-delà de C, à (AD) et à (CD).
Montrer que les tangentes communes extérieures à ω1 et ω2 se coupent sur Ω.

13
O

F IGURE 7 – Une figure et son image par la rotation de centre O et d’angle 75◦ .

3 Rotations
3.1 Cours
Définition 3.1. Soient O un point du plan et θ un angle. La rotation de centre O et d’angle
(orienté) θ est la transformation qui à tout point M associe le point M 0 tel que OM 0 = OM et
−−→ −−→
(OM , OM 0 ) = θ.

Exemple 3.2. Les symétries centrales sont des rotations d’angle π.

Les rotations conservent toutes les propriétés géométriques intéressantes, y compris les
longueurs :

Proposition 3.3. a) Les rotations conservent les longueurs.


b) Les rotations conservent les droites, les cercles, les angles.
c) L’image d’une droite (d) par une rotation d’angle θ fait un angle θ avec (d).

Démonstration.
a) Si une rotation de centre O envoie A sur A0 et B sur B 0 , alors OA = OA0 , OB = OB 0 et
−→ −−→ −−→ −−→
(OA, OA0 ) = (OB, OB 0 ) donc les triangles OAB et OA0 B 0 sont isométriques et A0 B 0 = AB.
b) Comme une rotation conserve les longueurs, un triangle et son image sont isométriques
donc ont les mêmes angles, et la rotation conserve les angles. Elle conserve donc toutes les
propriétés pouvant s’exprimer avec des angles, comme l’alignement et la cocyclicité.

14
c) Soient A0 et B 0 les images de A et B par une rotation de centre O. On utilise la relation de
Chasles pour les angles orientés :
−→ −−→ −→ −→ −→ −−→ −−→ −−→
(AB, A0 B 0 ) = (AB, AO) + (AO, A0 O) − (A0 B 0 , A0 O)

Comme les triangles OAB et OA0 B 0 sont directement isométriques, le premier et le dernier
terme se simplifient tandis que le second vaut θ.
De même que les homothéties, il est intéressant de composer les rotations :

Proposition 3.4. Soient r1 et r2 deux rotations d’angles θ1 et θ2 :


a) Si θ1 + θ2 est un multiple de 360◦ , alors r2 ◦ r1 est une translation.
b) Sinon, r2 ◦ r1 est une rotation d’angle θ1 + θ2 .

Remarque 3.5. Il est parfois possible dans les exercices d’identifier le centre d’une composée
de rotation, mais on n’a pas de résultat général analogue au théorème de Monge pour les
rotations.

Démonstration. On admet que la composée est une rotation ou une translation. Pour l’angle
de la composée, r1 fait tourner les droites d’un angle θ1 et r2 de θ2 , donc la composée les fait
tourner de θ1 + θ2 .
Notons également que les rotations apparaissent comme les composées de symétries axiales :

Proposition 3.6. Soient (d) et (d0 ) deux droites qui se coupent en un point O et y font un angle
θ. Soient s la symétrie axiale d’axe (d) et s0 celle d’axe s0 . Alors s0 ◦ s est la rotation de centre O
et d’angle 2θ.

D
M

M0
O
θ

D0
M 00

Démonstration.
Soit M un point du plan, M 0 = s(M ) et M 00 = s0 (M 0 ) : la droite (d) est la médiatrice de
[M M 0 ] donc OM 0 = OM et (d0 ) celle de [M 0 M 00 ] donc OM 00 = OM 0 = OM . De plus, en prenant
D sur (d) et D0 sur (d0 ) comme sur la figure, on a (il faudrait en toute rigueur utiliser des angles
orientés)

OM 00 = M
M\ \ \0 + M
OD + DOM \ 0 OD 0 + D\
0 OM 00 = 2DOM
\ 0 + 2M
\ 0 OD 0 = 2DOD
\0 = 2θ.

Comme pour les homothéties, il peut être utile d’introduire une rotation qui envoie un
objet sur un autre :

15
Proposition 3.7. a) Soient A, B, A0 et B 0 quatre points : il existe une homothétie ou trans-
lation qui envoie A sur A0 et B sur B 0 si et seulement si AB = A0 B 0 . De plus, dans ce
cas, une telle transformation est unique.
b) Soient O, A et A0 avec OA = OA0 : il existe une unique rotation de centre O qui envoie
A sur A0 .

Démonstration. a) Si ABB 0 A0 est un parallélogramme, la transformation doit être une trans-


lation, qui fonctionne bien. Sinon, (AA0 ) et (BB 0 ) ne sont pas parallèles donc leurs mé-
diatrices non plus, donc il existe O tel que OA = OA0 et OB = OB 0 . Les triangles OAB
et OA0 B 0 sont alors isométriques donc AOB [ =A \0 OB 0 , donc AOA
\0 = BOB\0 = θ, et la
rotation de centre O et d’angle θ fonctionne.
\0 .
b) Le centre est déjà fixé, et l’angle de la rotation doit être AOA

3.2 Conseils pour les exercices


— Les triangles équilatéraux ou isocèles rectangles dissimulent souvent (assez mal) des
rotations.
— Comme pour les homothéties, si vous repérez plusieurs rotations intéressantes, essayez
de les composer !

3.3 Exercices
Exercice 19 Soient (∆1 ), (∆2 ) et (∆3 ) trois droites parallèles.
Construire un triangle équilatéral ayant un sommet sur chacune des trois droites.
Exercice 20 Soit ABC un triangle équilatéral et M sur son cercle circonscrit, sur l’arc entre B
et C ne contenant pas A.
Montrer que M B + M C = M A.
Exercice 21 Soient ABCD un quadrilatère convexe et E, F , G et H tels que ABE, BCF , CDG
et DAH soient équilatéraux avec ABE et CDG dirigés vers l’extérieur de ABCD et BCF et
DAH dirigés vers l’intérieur. Montrer que EF GH est un parallélogramme.
Exercice 22 Soit A1 A2 A3 un triangle. On note Ai = Ai−3 pour tout i ≥ 4, de sorte que A1 =
A4 = A7 = . . . , A2 = A5 = A8 = . . . et A3 = A6 = A9 = . . . . Soit (Pi )i≥0 une suite de points
telle que pour tout i, il existe un point Qi tel que le triangle Qi Pi Pi+1 est équilatéral direct et a
pour centre Ai .
On suppose que P2014 = P1 . Montrer que A1 A2 A3 est équilatéral.
Exercice 23 (Point de Fermat) Soit ABC un triangle dont tous les angles sont inférieurs à 120◦ .
a) Montrer qu’il existe un unique point F à l’intérieur de ABC tel que AF [ B = BF
\ C =

CF
[ A = 120 .
b) Montrer que pour tout X à l’intérieur de ABC, on a AX + BX + CX ≥ AF + BF + CF .
Exercice 24 (Théorème de Napoléon) Soient ABC un triangle et A0 , B 0 et C 0 tels que A0 BC,
AB 0 C et ABC 0 soient équilatéraux, extérieurs à ABC. Soient A1 , B1 et C1 les centres de ces
triangles équilatéraux.
Montrer que A1 B1 C1 est équilatéral.

16
Exercice 25 Soient ABC et AM N deux triangles directement semblables, tous deux isocèles
en A. On note O le centre du cercle circonscrit à ABM .
Montrer que O, C, N et A sont cocycliques si et seulement si ABC est équilatéral.

4 Similitudes directes
On va maintenant s’intéresser à une classe de transformations plus large, qui contient les
précédentes. Cette section contient en particulier les preuves de certains résultats admis dans
les parties précédentes.
Il est possible de sauter toutes les preuves dans un premier temps si vous n’êtes pas très
à l’aise. Dans ce cas, vous pouvez définir directement une similitude directe comme la com-
posée d’une rotation et d’une homothétie de même centre comme dans 4.10, et admettre les
théorèmes importants 4.4, 4.8 (dont la preuve reste simple), 4.12 et 4.13.
L’essentiel de cette section est issue du cours donné par Jean-François Martin au stage
olympique de Montpellier 2013. Le cours original se trouve dans le polycopié du stage, aux
pages 87 à 101, accessible ici :
http://www.animath.fr/IMG/pdf/poly_2013-2.pdf

4.1 Définitions et propriétés de base


Définition 4.1. On appelle similitude directe toute transformation (i.e. fonction bijective dans
lui-même) du plan qui conserve les angles orientés, c’est-à-dire telle que pour tous points A,
−−→ −−→ −→ −→
B et C, si on note A0 , B 0 et C 0 leurs images, on a (A0 B 0 , A0 C 0 ) = (AB, AC). De manière équiva-
lente, une similitude directe envoie les triangles sur des triangles directement semblables.

Exemple 4.2. Les translations, rotations et homothéties sont des similitudes directes. De plus,
il est évident que la composée de deux similitudes est une similitude.

Remarques 4.3. — L’équivalence entre conservation des angles orientés et des triangles
directement semblables n’est pas immédiatement évidente pour les triangles plats.
Toutefois, étant donné trois points A, B et C alignés, pour montrer que le rapport
AB/AC est conservé, il suffit de considérer un point P en dehors de la droite et de
considérer les triangles ABP et ACP , prouvant que les rapports AB/AP et AP/AC
seront conservés. Une astuce du même genre permettra de toujours mettre de côté ce
genre de cas résiduels. C’est un bon exercice pour le lecteur pointilleux de remplir les
trous...
— Dans la suite, s’il n’y a pas d’ambiguïté, on dira "similitude" pour "similitude directe".
Les similitudes indirectes sont évoquées à la fin du cours.

Théorème 4.4. Étant donnés quatre points A 6= B et A0 6= B 0 , il existe une unique similitude s
telle que :

s : A −7 → A0
B 7−→ B 0 .

17
Démonstration. On pourrait raisonner directement dans l’esprit du théorème 4.11, mais la
preuve naturelle a son intérêt.
Unicité :
Montrons que pour tout point C du plan, cette condition fixe l’image de C par s. On sup-
pose C ∈/ (AB). Par conservation des angles orientés, l’image de C doit être sur la droite
−→ −−→ −−→
formant en B 0 un angle (BA, BC) avec le vecteur B 0 A0 . De même, elle doit être sur la droite
−→ −→ −−→
formant en A0 un angle (AB, AC) avec A0 B 0 . L’intersection de ces deux droites, uniquement
déterminée, est donc le point recherché.

C
A

s(C)

B A0

B0

Existence :
On va exhiber une similitude avec cette propriété en composant différentes similitudes
classiques (translations, rotations, homothéties...) de manière à s’approcher de plus en plus
du but souhaité.
Dans un premier temps, afin d’envoyer déjà A sur A0 , on utilise la translation t de vecteur
−−→0
AA :

t : 7 → A0
A −
B −7 → B 00 .

On utilise ensuite la rotation ρ de centre A0 et d’angle B\


00 A0 B 0 :

ρ : A0 − 7 → A0
B 00 7−→ B 000 .

Comme par construction B 000 ∈ (A0 B 0 ), on peut finalement utiliser l’homothétie h de centre
A et de rapport A0 B 0 /A0 B 000 :
0

18
h : A0 − 7 → A0
B 000 7−→ B 0 .

Finalement, s = h ◦ ρ ◦ t convient par construction.

A0
t
A

B 00
t ρ
B B 000

B0

Remarquons que les homothéties ou les rotations peuvent également être définies comme
les transformations conservant certaines grandeurs ou propriétés. Cependant, la définition
usuelle avec le centre est plus explicite et plus utilisable dans les exercices. On va donc s’inté-
resser à la notion de centre chez les similitudes.

4.2 Centre d’une similitude


Définition 4.5. On appelle centre d’une similitude un point fixe de cette similitude.
Exemple 4.6. Une translation de vecteur non nul n’a aucun centre, une rotation ou une homo-
thétie en a un et la transformation identité admet tous les points du plan comme centre.
Remarques 4.7. — L’unicité du théorème précédent appliquée au cas A = A0 et B = B 0
montre que, hormis l’identité, toute similitude a au plus un point fixe.
— Il n’est pas évident a priori que toute similitude a un centre. Cela découle du théorème
suivant.
Le théorème suivant, essentiel, montre que l’on connaît bien le centre d’une similitude :
Théorème 4.8. Soient A, B, A0 et B 0 supposés en position générale (un couple ne peut être
obtenu à partir de l’autre par translation ou homothétie). Soit P le point d’intersection de
(AA0 ) et (BB 0 ). Soit C1 et C2 les cercles circonscrits à P AB et P A0 B 0 et Q leur deuxième point
d’intersection.
Alors, Q est le centre de la similitude envoyant A et B sur A0 et B 0 .

19
Démonstration. Soit s l’unique similitude envoyant A et B sur A0 et B 0 . On cherche à montrer
que s envoie Q sur lui-même. D’après la démonstration de l’unicité de cette similitude, il
suffit de vérifier que ABQ et A0 B 0 Q sont directement semblables. On procède par chasse aux
angles. Il suffit de montrer que ABQ
[ =A \ 0 B 0 Q, l’égalité BAQ
[ =B\0 A0 Q se montrant de manière

similaire :
[ = 180◦ − AP
ABQ [ Q=A \0P Q = A\ 0 B 0 Q.

B0
A

A0

Remarques 4.9. — Si les points ne sont pas en position générale, la construction marche
toujours en considérant que deux droites parallèles se coupent "à l’infini" et que le
cercle passant par A, B et l’infini n’est autre que la droite (AB). On pourra vérifier que
Q est à l’infini ssi ABB 0 A0 est un parallélogramme, ce qui correspond au cas où s est
une translation.
— Bien sûr, il faudrait écrire cette démonstration en termes d’angles orientés. Ce que le
lecteur pointilleux est invité à faire. Il est d’ailleurs important de se familiariser avec
les deux configurations :

A
A0
P

B0
Q

20
On remarque de plus en utilisant les triangles semblables de la démonstration précé-
\0 = BQB
dente que la composée d’une rotation d’angle AQA \0 et d’une homothétie de rapport
QA /QA = QB /QB, toutes deux de centre Q, envoie A sur A0 et B sur B 0 .
0 0

Définition 4.10. Une similitude directe est dite centrée si elle est la composée d’une rotation et
d’une homothétie de même centre.

Soit donc s une similitude directe et Q son centre : on a obtenu une similitude centrée s0
qui, comme s, envoie A sur A0 et B sur B 0 . Grâce à l’unicité dans le théorème 4.4 on a donc
s = s0 d’où le théorème important suivant :

Théorème 4.11. Toute similitude directe soit est une translation soit est centrée.
Ainsi, les similitudes qui ne sont pas des translations sont décrites par un centre, un rap-
port et un angle.

A ce stade, on peut se demander pourquoi on n’a pas commencé le cours en donnant la


définition 4.10, ce qui nous aurait épargné le besoin de prouver le théorème 4.11. Un avantage
de la définition 4.1 est qu’elle rend trivial le fait que la composée de deux similitudes directes
est une similitude directe, ce qui n’est pas évident avec 4.10. On obtient ainsi le théorème
suivant :

Théorème 4.12. Soient s1 et s2 deux similitudes directes de rapports k1 et k2 , et d’angles θ1 et


θ2 . Alors s2 ◦ s1 est une similitude directe de rapport k1 k2 et d’angle θ1 + θ2 .

Remarquons que les propositions 2.6 et 3.4 en sont des cas particuliers (le cas θ1 = θ2 = 0
pour la première, le cas k1 = k2 = 1 pour la seconde). On a donc finalement prouvé les parties
de ces deux théorèmes qu’on avait admis.

4.3 Deux similitudes pour le prix d’une


Reprenons la figure du théorème 4.8 : grâce aux mêmes triangles semblables on obtient que
la similitude directe de centre Q, d’angle AQB [ = A \ 0 QB 0 et de rapport QB/QA = QB 0 /QA0

envoie A sur B et A0 sur B 0 . En laissant au lecteur le cas particulier des homothéties, on obtient
le théorème important suivant.

Théorème 4.13. Si une similitude directe envoie A sur A0 et B sur B 0 , alors la similitude en-
voyant A sur B et A0 sur B 0 a le même centre.

En appliquant le théorème 4.8 aux deux similitudes précédentes, on obtient le résultat


bien connu suivant, qui (pour l’intersection des cercles) se démontre également par chasse
aux angles (exercice !) :

Théorème 4.14. Soit A, B, A0 et B 0 en position générale. Soit P1 = (AA0 ) ∩ (BB 0 ) et P2 =


(AB) ∩ (A0 B 0 ). Alors les cercles circonscrits à P1 AB, P1 A0 B 0 , P2 AA0 et P2 BB 0 sont concourants
en un point Q, appelé point de Miquel du quadrilatère complet AA0 BB 0 .
Il est le centre de la similitude directe envoyant A sur A0 et B sur B 0 ainsi que de celle
envoyant A sur B et A0 sur B 0 .

21
P2

A B0
P1

B A0
Q

Remarque 4.15. On considérera essentiellement ce point comme centre des similitudes di-
rectes précédentes. Mais il est également d’une importance primordiale en géométrie projec-
tive unidimensionnelle complexe en tant que centre de l’involution échangeant les 6 sommets
du quadrilatère complet (pour comprendre la phrase précédente ... allez au club de mathéma-
tiques discrètes de Lyon !).
Remarque 4.16. Il peut souvent être intéressant de regarder les similitudes d’un point de vue
complexe : il s’agit en fait juste des fonctions affines.

4.4 Similitudes indirectes


Nous terminons ce cours par quelques mots sur les similitudes indirectes.
Définition 4.17. Une similitude indirecte est une transformation du plan qui envoie chaque
angle orienté sur son opposé.
Exemple 4.18. Les symétries axiales sont des similitudes indirectes. La composée d’une sy-
métrie axiale avec une similitude directe est une similitude indirecte.
Les similitudes indirectes conservent de nombreuses propriétés : rapports de longueur,
angles non orientés, alignement, cocyclicité, parallélisme, orthogonalité... Elles sont cepen-
dant moins intéressantes que les similitudes directes car beaucoup n’admettent pas de centre
(considérer par exemple la composée d’une symétrie axiale et d’une translation parallèle à
son axe). Les puissants théorèmes 4.8 et 4.13 n’ont donc pas d’analogue pour les similitudes
indirectes.

4.5 Conseils pour les exercices :


Voici quelques conseils généraux qui peuvent servir dans les exercices qui suivent et dans
bien d’autres :

22
— Les exercices où un point varie et où on cherche à déterminer le lieu d’un autre point ou
bien à montrer qu’un autre point reste fixe font souvent appel à des transformations.
— N’oubliez pas la chasse aux angles, qui permet souvent de "démarrer" un exercice, par
exemple en trouvant deux triangles semblables, ou de le conclure une fois qu’on a
trouvé de nombreux cercles.
— Quand vous trouvez deux triangles semblables (éventuellement plats), intéressez-vous
à la similitude qui envoie l’un sur l’autre : existe-t-il d’autres points dont on connaît
bien l’image ? Si la similitude est directe, peut-on trouver son centre ? Si le centre est
bien connu, le théorème 4.8 donne des points cocycliques !
— L’apparition des figures du théorème 4.8 doit donner envie d’introduire des simili-
tudes !
— Quand cette figure apparaît, n’oubliez pas que vous avez en fait trouvé deux simili-
tudes directes intéressantes de même centre : celle qui envoie A sur A0 et B sur B 0 , et
celle qui envoie A sur B et A0 sur B 0 . Essayez les deux et utilisez celle qui paraît la plus
utile.

4.6 Exercices
Exercice 26 Soit ABCD un quadrilatère, E et F sur [AD] et [BC] respectivement tels que
AE/ED = BF/F C. Soit S = (EF ) ∩ (AB) et T = (EF ) ∩ (CD).
Montrer que les cercles circonscrits aux triangles SAE, SBF , T CF et T DE sont concou-
rants.
Exercice 27 Soient Γ1 et Γ2 deux cercles qui se coupent en deux points A et D. La tangente
à Γ1 en A recoupe Γ2 en B, et la tangente à Γ2 en A recoupe Γ1 en C. Soit E ∈ [AB) tel que
BE = AB, et F la deuxième intersection de [AC) avec le cercle circonscrit Ω à ADE. Montrer
que AC = AF .
Exercice 28 Soit ABCDE un pentagone convexe vérifiant les relations BAC
[ = CAD
\ = DAE
\
et CBA
[ = DCA\ = EDA. \ Soit P = (BD) ∩ (CE).
Montrer que la droite (AP ) coupe le segment [CD] en son milieu.
Exercice 29 Soit ABCD un quadrilatère avec AD = BC et P l’intersection de ses diagonales.
Soit F et E des points variables sur les segments [AD] et [BC] respectivement de manière
à avoir BE = DF . On note R et Q les points d’intersections de (EF ) avec (AC) et (BD)
respectivement.
Montrer que le cercle circonscrit à P QR a un deuxième point fixe quand E et F varient.
Exercice 30 Soit ABC un triangle inscrit dans un cercle Γ, P un point variable sur le l’arc AB
qui ne contient pas C. Soient I et J les centres des cercles inscrits des triangles ACP et BCP
respectivement. On considère Q le point d’intersection de Γ et du cercle circonscrit au triangle
P IJ.
Montrer que Q reste fixe quand P varie.
Exercice 31 Soit Γ1 et Γ2 deux cercles s’intersectant en P et Q. Soit A1 et B1 deux points va-
riables sur Γ1 et A2 et B2 les deuxièmes points d’intersection de Γ2 avec (A1 P ) et (B1 P ) res-
pectivement. Soit C = (A1 B1 ) ∩ (A2 B2 ).
Montrer que le centre O du cercle circonscrit au triangle CA1 A2 reste sur un cercle fixe
quand A1 et A2 varient.

23
L’exercice suivant est important per se et souvent utilisé (il faut par contre penser à le
redémontrer ... ou citer ce poly !).
Exercice 32 Soit ABCD un quadrilatère convexe inscrit dans un cercle de centre O, P le point
d’intersection des diagonales et Q le deuxième point d’intersection des cercles circonscrits aux
triangles AP D et BP C.
[ = 90◦ .
Montrer que OQP
Exercice 33
a) Soit T une similitude directe. Montrer qu’il existe une similitude directe de même
centre envoyant tout point M sur le milieu de M et T (M ).
b) Soit ABCD un quadrilatère, M et N les milieux de ses diagonales et P leur intersection.
Soit O1 et O2 les centres des cercles circonscrits de ABP et CDP . Montrer que le milieu
du segment [O1 O2 ] est le centre du cercle circonscrit de P M N .
Exercice 34 Soit Γ1 , Γ2 et Γ3 trois cercles avec {A, B} = Γ1 ∩ Γ2 , {C, D} = Γ2 ∩ Γ3 et {E, F } =
Γ3 ∩ Γ1 . On considère P1 sur Γ1 et on note P2 le deuxième point d’intersection de (P1 A) et Γ2 ,
P3 le deuxième d’intersection de (P3 C) et Γ3 , P4 le deuxième point d’intersection de (P3 E) et
Γ1 , P5 le deuxième point d’intersection de (P4 B) et Γ2 , P6 le deuxième point d’intersection de
(P5 D) et Γ3 et enfin P7 le deuxième point d’intersection de (P6 F ) et Γ1 .
Montrer que P7 = P1 .
Exercice 35 Soient Γ1 et Γ2 deux cercles se coupant en deux points A et B. Les tangentes à Γ1
en A et B se coupent en K. Soit M un point variable sur Γ1 , distinct de A et B. On note P le
second point d’intersection de (M A) et Γ2 , C le second point d’intersection de (M K) et Γ1 et
Q le second point d’intersection de (AC) avec Γ2 .
a) Montrer que (P Q) passe par un point fixe quand M varie.
b) Montrer que le milieu de [P Q] est sur la droite (M K).
Exercice 36 Soit ABCDEF un hexagone inscriptible vérifiant AB = CD = EF . Soit Z =
(AC) ∩ (BD), X = (CE) ∩ (DF ) et Y = (EA) ∩ (F B).
Montrer que XY Z et BDF sont semblables.
Exercice 37 Soit ABC un triangle, E et D des points sur les côtés [AB] et [AC] de manière à
avoir BE = CD. Soit P l’intersection des diagonales du quadrilatère BEDC et Q le deuxième
point d’intersection des cercles circonscrits à EP B et DP C. Soit K et L les milieux respectifs
de [BE] et [CD] et R le point d’intersection de la perpendiculaire à (QK) passant par K et de
la perpendiculaire à (QL) passant par L.
Montrer que :
a) Q est sur la bissectrice de l’angle BAC.
[
b) R est sur le cercle circonscrit au triangle ABC.
Exercice 38 (BMO 2009) Soit ABC un triangle. Une droite parallèle à (BC) coupe [AB] en M
et [AC] en N . Soit P le point d’intersection de (BN ) et (CM ). Les cercles circonscrits à BM P
et CN P se recoupent en Q.
Montrer que P[ AB = QAC.
[
Exercice 39 Soit ABC un triangle et Γ son cercle circonscrit. On considère trois points A1 , B1
et C1 sur les côtés [BC], [CA] et [AB] respectivement. On note A3 , B3 et C3 les symétriques
de A1 , B1 et C1 par rapport aux milieux de leurs côtés respectifs. On note A2 , B2 et C2 les

24
deuxièmes points d’intersection de Γ avec les cercles circonscrits à AB1 C1 , BC1 A1 et CA1 B1
respectivement.
Montrer que les triangles A2 B2 C2 et A3 B3 C3 sont semblables.

5 Indications pour les exercices


Exercice 1: Quelle est la transformation qui permet d’"écraser" la rivière ?
Exercice 2: Si B décrit Γ et D est le point tel que ABCD soit un parallélogramme, quel en-
semble décrit D ?
Exercice 3: Si X décrit le cercle et Y est tel que XY = a et (XY ) est parallèle à (d), quel
ensemble décrit Y ?
Exercice 4: Trouver une homothétie qui envoie ABC sur A0 B 0 C 0 .
Exercice 5: Coller un carré au triangle par le côté [BC]. Quel est son lien avec le carré qui nous
intéresse ?
Exercice 6: Trouver deux homothéties intéressantes sur la figure et les composer.
Exercice 7: Chasse aux tangentes.
Exercice 8: Une homothétie h de centre A envoie Γ1 sur Γ2 . Il suffit de montrer h(D) = C...
Exercice 9: Une homothétie de centre A envoie ΓA sur Γ...
Exercice 10: L’homothétie à utiliser est évidente. Quelle est l’image de B ?
Exercice 11: Trouver deux homothéties de centres A et A0 et appliquer le théorème de Monge.

Exercice 12: L’homothétie à utiliser est évidente. Introduire les images de E et J par cette
homothétie.
Exercice 13: Utiliser l’homothétie "évidente", puis trouver une homothétie de centre D qui
envoie M sur I.
Exercice 14: Identifier le point d’intersection des quatre cercles. Une chasse aux tangentes per-
met de faire apparaître un nouveau cercle...
Exercice 15: Introduire un cercle tangent à trois droites sur les quatre, et essayer d’appliquer
Monge.
Exercice 16: L’exercice 9 donne deux nouveaux points sur (KL).
Exercice 17: Utiliser Monge pour exprimer E et F comme centres de deux homothéties. On
aura besoin de cercles supplémentaires...
Exercice 18: Introduire P et Q, points où ω1 et ω2 touchent [AC]. Que dire de ces deux points ?

Exercice 19: Fixer A1 . Quand A2 décrit (∆2 ), quel ensemble décrit le troisième sommet du
triangle équilatéral ?
Exercice 20: Pour faire appraître BM + CM , placer N sur (BM ) tel que M N = CM . Quelle
rotation apparaît ?

25
Exercice 21: Trouver des rotations de 60◦ sur la figure...
Exercice 22: Quelle transformation permet de passer de Pi à Pi+1 ? Et de P1 à P2014 ?
Exercice 23: A l’aide d’une rotation bien choisie, "reporter" les distances BF et CF sur la
droite (AF ) au-delà de F . Que donne la même construction avec un X quelconque ?
Exercice 24: Composer des rotations de 120◦ .
Exercice 25: La rotation à utiliser est évidente. Introduire l’image de O.
Exercice 26: Les triangles plats peuvent aussi être semblables...
Exercice 27: La construction du centre d’une similitude marche toujours dans le cas limite où
P est confondu avec un des quatre points...
Exercice 28: Des triangles semblables sautent aux yeux. Terminer la construction du centre...
Exercice 29: Une similitude fixe envoie E sur F . Le point fixe qu’on cherche est son centre !
Exercice 30: Introduire les points nécessaires pour que la construction de Q soit celle du centre
d’une similitude. Que dire de son angle et de son rapport ?
Exercice 31: Commencer par éliminer B1 et B2 . Tester des cas particuliers pour A1 permet
d’identifier le cercle.
Exercice 32: Introduire des milieux pour faire apparaître des angles droits.
Exercice 33:
a) Deux similitudes pour le prix d’une !
b) K l’intersection des cercles circonscrits des triangles ABP et P CD. Chercher à appli-
quer a)...
Exercice 34: Quelle est la transformation qui permet de passer de Pi à Pi+1 ? Et de P1 à P7 ?
Exercice 35:
a) (P Q) est l’image d’une droite bien connue par une similitude de centre B.
b) Introduire les centres des cercles. Les angles droits donnent des points cocycliques.
Exercice 36: Cette fois, la similitude intéressante est une rotation !
Exercice 37:
a) Une similitude saute aux yeux. Quel est son rapport ?
b) Montrer que A, K, L, Q et R sont sur un même cercle. Que dire de l’intersection de ce
cercle et du cercle circonscrit à ABC ?
Exercice 38: La droite (AP ) est connue. S’intéresser aux distances de Q à [AB] et [AC]...
Exercice 39: Montrer que C2 AB et CB3 A3 sont semblables.

6 Solutions des exercices


Solution de l’exercice 1
Soit →
−v le vecteur perpendiculaire à la rivière dirigé vers A et de longueur L, et B 0 l’image
de B par la translation de vecteur →−v : si le pont va de P à Q, la longueur du trajet vaut
AP + P Q + QB = AP + P B + L. On veut donc rendre AP + P B 0 le plus petit possible, donc
0

26
A

B0
Q

P doit être aligné avec A et B 0 . Il faut donc que P soit l’intersection de (AB 0 ) et de la rive du
côté de A.
Solution de l’exercice 2

A
B

D
C

Soit M le milieu de [AC] : ABCD est un parallélogramme ssi M est le milieu de [BD] ssi
B est le symétrique de D par rapport à M . POur que D soit sur Γ, il faut donc donc que B
soit sur le symétrique Γ0 de Γ par rapport à M , ce qui permet de le tracer. D est alors l’autre
intersection des deux cercles.
Solution de l’exercice 3

27


v

Γ0
Γ

X Y

(d)

Soit →

v un vecteur parallèle à (d) et de longueur a : si on a la construction qu’on veut, alors
−−→ −−→
XY = v ou Y X = →

− −
v . Quitte à échanger X et Y , on suppose qu’on est dans la première
situation. Alors Y ∈ Γ, et X ∈ Γ donc Y est sur l’image de Γ par la translation de vecteur → −
v,
0
qu’on note Γ .
On trace donc Γ0 , puis on prend Y à l’intersection de Γ et Γ0 . On peut ensuite tracer la droite
parallèle à (d) passant par Y et placer X.
Solution de l’exercice 4
B0 C0

A0

C B

On note X le point d’intersection de (AA0 ) et (BB 0 ) s’il existe. D’après le théorème de


0 0
Thalès, on peut poser k = XA XA
= XB
XB
. L’homothétie h de centre X et de rapport k envoie alors
A sur A et B sur B . La droite h((AC)) passe par A0 et est parallèle à (AC) donc à (A0 C 0 ), d’où
0 0

h((AC)) = (A0 C 0 ) et, de même h((BC)) = (B 0 C 0 ). On a donc h(C) = C 0 , donc X, C et C 0 sont


alignés, d’où le résultat.
Si (AA0 ) et (BB 0 ) sont parallèles, ABB 0 A0 est un parallélogramme donc il existe une trans-
lation t qui envoie A sur A0 et B sur B 0 et, comme ci-dessus, on montre qu’elle envoie C sur
C 0 , donc (CC 0 ) est parallèle à (AA0 ) et (BB 0 ).

28
Remarque 6.1. On a trouvé une condition nécessaire et suffisante sur deux triangles pour qu’il
existe une homothétie envoyant l’un sur l’autre.

Solution de l’exercice 5

H G

C E F B

P Q

Supposons construit notre carré EF GH, et plaçons sur la figure P et Q tels que BCP Q oit
un carré, situé de l’autre côté de [BC] par rapport à A. Comme (GH) est parallèle à (BC), il
existe une homothétie h de centre A qui envoie G sur B et H sur C. h envoie donc EF GH
sur P QBC, et en particulier P sur E. E est donc sur (AP ) donc E est l’intersection de [BC] et
(AP ), et de même F est celle de [BC] et (AQ). Une fois qu’on a E et F , il est facile de construire
G et H.
Solution de l’exercice 6

A X M Y B

C
D

Les droites parallèles donnent envie de chercher des homothéties qui les envoient l’une
sur l’autre. Deux sont intéressantes : celle de centre Q qui envoie A sur C et Y sur D, qu’on
note hQ et celle de centre P qui envoie C sur B et D sur X, qu’on note hP .

29
hP ◦ hQ est alors une homothétie qui envoie A sur B et Y sur X. Son centre est sur (AB) et sur
(P Q) (car c’est le centre d’une composée d’homothéties de centres P et Q), donc c’est M et,
comme M est le milieu de [AB], hP ◦ hQ est la symétrie centrale de centre M , donc M X = M Y .

Remarque 6.2. Il est aussi possible de résoudre l’exercice en appliquant le théorème de Mé-
nélaüs dans ABC.

Solution de l’exercice 7

D
S
C

R Q

A B
P

Les deux cercles inscrits sont situés de part et d’autre de [AC] donc il sont tangents en
un point de [AC], qui est forcément le point où ils touchent [AC]. On donne des noms aux
différents points de tangence comme sur la figure et on effectue une chasse aux tangentes :

AB + CD = AP + BP + CS + DS = AR + BQ + CR + DT = AT + BQ + CQ + DT = AD + BC

donc ABCD admet un cercle inscrit.


Solution de l’exercice 8

O1 O2
A

30
A est le centre d’une homothétie négative h qui envoie Γ2 sur Γ1 . De plus, D est diamétra-
lement opposé à B donc la tangente à Γ1 en D est parallèle à (BC). On a donc h(C) = D, donc
A, C et D sont alignés.
Solution de l’exercice 9
A

C I B
J

A est l’intersection des tangentes communes extérieures à Γ et ΓA , donc c’est le centre de


l’homothétie positive h qui envoie ΓA sur Γ. De plus, K est diamétralement opposé à I sur Γ
donc la tangente à Γ en K est parallèle en I à (BC). Or, la tangente à ΓA en J est (BC), donc
son image par h est tangente à Γ, différente de (BC) et parallèle à (BC). Il s’agit donc de la
tangente Γ en K, donc K = h(J) et A, J et K sont alignés.
Solution de l’exercice 10

ω1 ω2

X B Y

S C T

On veut montrer que (AB) est la bissectrice de XAY \ . Soit C la deuxième intersection de
(AB) avec ω1 : il suffit de montrer que C est le milieu de l’arc XY . Soit h l’homothétie de
centre A qui envoie ω2 sur ω1 : on a h(B) = C donc la tangente à ω1 en C est parallèle à (XY ).
En notant (ST ) la tangente comme sur la figure, on peut écrire, en utilisant le cas tangent du
théorème de l’angle inscrit puis le fait que (XY ) et (ST ) soient parallèles :

CXY
\ = Y[
CT = CY
\ X

31
donc C est bien le milieu de l’arc XY de ω1 , d’où le résultat.
Solution de l’exercice 11

A
Γ

ΓA

C B

A0

Soient ω le cercle inscrit à (ABC) et X le centre de l’homothétie positive qui envoie Γ


sur ω : le point A0 est le centre de l’homothétie positive qui envoie Γ sur ΓA et A est celui
de l’homothétie positive qui envoie ΓA sur ω, donc X ∈ (AA0 ) et, de même, X ∈ (BB 0 ) et
X ∈ (CC 0 ) d’où le résultat.
Solution de l’exercice 12

F G

K
I

C D B
E

Soit h l’homothétie de centre A qui envoie le cercle A-exinscrit sur le cercle inscrit ω : on
s’intéresse à l’image de (EJ). On a h(J) = I. De plus h(BC) est parallèle à (BC), différente de

32
(BC) et tangente à ω. Si on note F le point de ω diamétralement opposé à D, h(BC) est donc
la tangente à ω en F , donc h(E) est l’intersection de cette tangente avec (AE), qu’on note G.
On a donc (EJ)//(GI).
Il suffit donc de montrer que (DK)//(GI), donc de montrer que DIGK est un parallélo-
gramme, donc il suffit de montrer DI = GK car (DI) et (GK) sont toutes deux perpendicu-
laires à (BC). Or GK = F I car F GKI est un rectangle, et F I = DI est le rayon de ω, d’où le
résultat.
Solution de l’exercice 13

C E D H B

Soit Z le point "en bas" du cercle exinscrit : l’homothétie de centre A qui envoie le cercle
inscrit sur le cercle exinscrit envoie D sur Z, donc A, D et Z sont alignés, donc il existe h
de centre D qui envoie A sur Z. Elle envoie (BC) sur elle-même, donc la hauteur issue de
A sur la droite (IZ), donc le pied H de la hauteur est envoyé sur le point de contact E du
cercle exinscrit avec [BC], donc le milieu de la hauteur M est envoyé sur le milieu de [EZ],
c’est-à-dire I. D étant le centre de cette homothétie, on en déduit que D, M et I sont alignés.
Solution de l’exercice 14

33
D C

ID IC

X
IB
IA

A B

On note ωA le cercle inscrit à DAB et ainsi de suite. Les cercles ωB et ωD sont situés de
part et d’autre de (AC), donc ils ne peuvent s’intersecter que sur (AC). De même, ωA et ωC ne
peuvent s’intersecter que sur (BD), donc l’intersection des 4 cercles ne peut être que l’inter-
section des diagonales, qu’on note X.
De plus, notons R et S les points de tangence de ωB avec [AB] et [BC], et T et U les points
de contact de ωD avec [CD] et [DA]. On a :

AB + CD = AR + BR + CT + DT = AX + BS + CX + DU = AU + BS + CS + DU = AD + BC

donc ABCD est circonscriptible : il existe un cercle à l’intérieur de ABCD tangent à tous les
côtés, qu’on note ω.
Le centre de l’homothétie négative qui envoie ωA sur ωC est X, donc est sur (AC). De plus,
A est le centre de l’homothétie positive qui envoie ωA sur ω, et C est le centre de l’homothétie
positive qui envoie ω sur ωC , donc le centre de l’homothétie positive qui envoie ωA sur ωC
est aussi sur (AC), donc les centres de ωA et ωC sont sur (AC), donc (AC) est la bissectrice
de BCD
\ et DAB,\ donc le quadrilatère est symétrique par rapport à (AC) d’où AB = AD et
CB = CD. On a de même BA = BC et DA = DC, donc ABCD est un losange.
Solution de l’exercice 15

34
Γ

Γ1
ω

Γ2

Pour commencer, d’après le théorème de Monge, le centre X de l’homothétie de rapport


positif qui envoie Γ1 sur Γ2 est sur (AB). On introduit le cercle ω qui est tangent à (d1 ) et (d2 )
entre A et Γ2 , et à (d3 ) entre B et Γ1 . On veut montrer que (d4 ) est aussi tangente à ω.
Soit Z le centre de l’homothétie positive qui envoie Γ1 sur ω : comme X est le centre de
l’homothétie positive qui envoie Γ1 sur Γ2 et A est le centre de l’homothétie positive qui envoie
Γ2 sur ω, on a Z ∈ (AX), soit Z ∈ (AB). D’autre part (d3 ) est une tangente commune extérieure
à Γ1 et ω donc Z ∈ (d3 ). Z est sur deux droites passant par B donc Z = B, donc (d4 ) est
tangente à ω (car B est le centre de l’homothétie positive envoyant Γ1 sur ω).

B
Q
P

O N
M L

K Z
Y
X
A D

35
Solution de l’exercice 16 Sans perte de généralité, on suppose que A ∈ [DX] et B ∈ [CX]. On
note P et Q les points de tangence de ω avec [AB] et [CD]. D’après l’exercice 9 appliqué au
triangle XCD, la droite (XL) passe par le point de ω diamétralement opposé à Q, qu’on note
Y . Dans l’exercice 9, on peut intervertir les rôles du cercle inscrit et du cercle exinscrit (la
preuve est exactement la même). Appliqué au triangle XAB, cela montre que (XK) passe par
le point de ω diamétralement opposé à P . On note ce point Z : les points X, K, Y , Z et L
sont alignés dans cet ordre. De plus, P QZY est un rectangle donc (P Q) est parallèle à (KL).
Comme KP \ [ (cas tangent du théorème de l’angle inscrit), P QLK est donc un trapèze
Q = LQP
isocèle donc P K = QL.
De plus, notons M et N les milieux de [AB] et de [CD] : une rapide chasse aux tangentes
montre que BP = AK = AB+AX−BX 2
et CQ = DL = CD=CX−DX 2
(résultats connus sur les
points de contact du cercle exinscrit) donc M et N sont aussi les milieux de [P K] et [QL]. Or,
on sait que P Y Z est rectangle en Y donc P Y K est rectangle en Y donc M est le centre du
cercle circonscrit à P Y K. En particulier, M est sur la médiatrice de [P Y ] donc (OM ) est la mé-
diatrice de [P Y ]. De même, (ON ) est la médiatrice de [QZ]. Comme P QZY est un rectangle,
ces médiatrices sont confondues, d’où le résultat.
Solution de l’exercice 17

Ω C

ω
D
E ω2
I F

R L
ω1 K

P
A S

Soit X = (AD) ∩ (BC) et Ω le cercle inscrit à ABX : A est le centre de l’homothétie positive
qui envoie Ω sur ω1 et K le centre de l’homothétie négative qui envoie ω1 sur ω, donc le centre
de l’homothétie négative qui envoie Ω sur ω est sur (AK). De même, il est sur (BL) donc il

36
s’agit de F . Pour que E, I et F soient alignés, il suffit donc de montrer que E est le centre de
l’homothétie positive qui envoie Ω sur ω.
D’autre part, d’après l’exercice 7, comme ω et ω1 sont tangents, le quadrilatère AP CD
est circonscriptible : on note Γ1 son cercle inscrit (qui n’est pas sur la figure par souci de
lisibilité). De même, BP DC est circonscriptible, on note Γ2 son cercle inscrit. A est le centre de
l’homothétie positive qui envoie Ω sur Γ1 et C le centre de celle qui envoie Γ1 sur ω, donc le
centre de l’homothétie positive qui envoie Ω sur ω est sur (AC) et par le même raisonnement
avec Γ2 , il est sur (BD) donc il s’agit de E, d’où le résultat.
Solution de l’exercice 18

ω1

A
P
Q
ω2
Γ C
D

X

Une chasse aux tangentes exploitant l’existence de Ω donne AB + AD = CB + CD (la


preuve est la même que celle du théorème 2.16, mais le résultat légèrement différent car le
cercle n’est pas à l’intérieur de ABCD). Si P et Q sont les points de tangence de ω1 et ω2 avec
[AC], on trouve donc :
AB + AC − BC CD + AC − AD
AP = = = CQ
2 2
Cela donne envie d’introduire Γ, cercle B-exinscrit à ABC : il est tangent à [AC] en Q
d’après les résultats du théorème 2.15 sur le cercle exinscrit.
On note X le point d’intersection des tangentes communes extérieures : d’après le théo-
rème de Monge avec ω1 , ω2 et Γ, il est sur (BQ). De même, en introduisant Γ0 , cercle D-exinscrit
à ACD, on obtient X = (BQ) ∩ (DP ).

37
Il faut maintenant identifier le point X : une bonne figure suggère que la tangente à Ω en
X est parallèle à (AC). Soit donc Y le point de Ω le plus proche de (AC) en lequel la tangente
à Ω est parallèle à (AC) : en utilisant l’homothétie de centre B qui envoie Γ sur Ω, on voit que
B, Q est Y sont alignés. De même, en utilisant l’homothétie de centre D qui envoie Γ0 sur Ω,
on voit que D, P est Y sont alignés, d’où X = Y et X ∈ Ω.
Solution de l’exercice 19
r(∆2 )

A1
(∆1 )

A3
(∆3 )

(∆2 ) A2

On choisit A1 arbitrairement sur (∆1 ). Pour tout point A2 , le point A3 tel que A1 A2 A3 soit
équilatéral est l’image de A2 par la rotation r de centre A1 et d’angle 60◦ . A3 doit donc appar-
tenir à (∆3 ) et à r(∆2 ) (qui s’intersectent car ∆2 et ∆3 sont parallèles et r tourne ∆2 de 60◦ ), ce
qui permet de placer A3 et donc de tracer le triangle.
Solution de l’exercice 20
A

C
B
M

En géométrie, il est plus facile de manipuler des sommes de longueurs quand les points
considérés sont alignés. Soit donc N le point de (M B) sur la demi-droite issue de M ne conte-
nant pas B, tel que M N = M C : par chasse aux angles CM\ N = 60◦ , donc CM N est équilaté-
ral. On veut maintenant montrer AM = BN . Or, soit r la rotation de centre C et d’angle 60◦ :
elle envoie A sur B et M sur N , d’où AM = BN = BM + M N = BM + CM .
Solution de l’exercice 21

38
H

E
B

D A

On suppose ABCD direct (c’est-à-dire que l’ordre alphabétique des sommets suit le sens
trigonométrique) : soient r1 la rotation de centre A et d’angle 60◦ et r2 la rotation de centre C
et d’angle −60◦ . On a :
r2 ◦ r1 (E) = r2 (B) = F
et
r2 ◦ r1 (H) = r2 (D) = G
donc EH=FG et on montre de même EF=GH.
Solution de l’exercice 22 Pour tout i, Ai est le centre du triangle équilatéral Qi Pi Pi+1 donc Ai Pi P i + 1
est isocèle avec un angle de 120◦ en Ai , donc Pi+1 est l’image de Pi par la rotation de centre Ai
et d’angle 120◦ .
Si on note ri la rotation de centre Ai et d’angle 120◦ , on a donc

P2014 = r2013 ◦ r2012 ◦ · · · ◦ r1 (P1 ) = (r3 ◦ r2 ◦ r1 )671 (P1 ),

car r3i = r3 , r3i+1 = r1 et r3i+2 = r2 pour tout i.


Or, nos trois rotations ont pour angle 120◦ , donc la somme des angles vaut 360◦ donc la
−−−−→
composée est une translation. On note → −v son vecteur : on a P1 P2014 = 671→ −
v car P2014 est
l’image de P1 par la translation de vecteur v , appliquée 671 fois. Si P2014 = P1 alors →

− −v doit
être le vecteur nul (c’est-à-dire le vecteur de longueur 0) donc r3 ◦ r2 ◦ r1 doit être l’identité,
donc r3 ◦ r2 ◦ r1 (A1 ) = A1 , soit r3 ◦ r2 (A1 ) = A1 . Notons B = r2 (A1 ) : alors A2 A1 B et A3 BA1 sont
isocèles avec un angle de 120◦ et partagent le côté A1 B donc sont isométriques, donc A2 A1 BA3
est un losange avec deux angles de 60◦ et deux de 120◦ , et A1 A2 A3 est équilatéral.
Solution de l’exercice 23

39
A

B
C

F0

a) D’après le théorème de l’angle inscrit, l’ensemble des points P qui sont du même côté
de (AB) que C et tels que AP [ B = 120◦ est un arc de cercle passant par A et B, et de
même l’ensemble des points du même côté de (AC) que B et tels que AP [ C = 120◦ est
un arc de cercle passant par A et C. La condition "tous les angles sont inférieurs à 120◦ "
garantit que les arcs de cercles se recoupent dans le triangle en un point F . On a alors
BF
\ C = 360◦ − AF[ B − AF[ C = 120◦ .
b) Soit X un point dans ABC : l’idée est de faire apparaître les trois distances AX, BX
et CX "à la suite", c’est-à-dire de trouver deux points Y et Z tels que BX = XY et
CX = Y Z, de manière à majorer la somme par AZ. Mais l’énoncé nous dit que quand le
minimum est atteint, on doit avoir AXB\ = 120◦ donc BXY\ = 60◦ , d’où l’idée d’utiliser
une rotation pour "reporter" les longueurs.
Soit donc r la rotation de centre B et d’angle 60◦ . On note Z = r(C), F 0 = r(F ) et
Y = r(X) : on a XY = XB (car BXY est équilatéral) et Y Z = XC (car r conserve les
longueurs). On a donc AX + BX + CX ≥ AZ. D’autre part, AF [ B = 120◦ et BF
\ F 0 = 60◦
donc A, F et F 0 sont alignés et BF
\ 0 Z = BF
\ C = 120◦ (car r conserve les angles) donc F ,
0
F et Z sont alignés d’où :

AF + BF + CF = AF + F F 0 + F 0 Z = AZ ≤ AX + BX + CX

Solution de l’exercice 24

40
B0

A0
B1 C
A1

A B

C1

C0

On suppose ABC direct : la figure présentant des triangles équilatéraux, il est naturel de
considérer des rotations de 60◦ ou 120◦ . On veut les composer pour aboutir à quelque chose
de simple, comme une translation. Soient donc rA , rB et rC les rotations de centres A1 , B1 et
C1 et d’angle 120◦ : leur composée est une translation car 3 × 120◦ = 360◦ . De plus, on a

rA ◦ rB ◦ rC (B) = rA ◦ rB (A) = rA (C) = B,

d’où rA ◦ rB ◦ rC = Id. En particulier, rA ◦ rB (C1 ) = C1 . Soit donc D = rB (C1 ) : on a rA (D) = C1 .



On a donc B1 D = B1 C1 et A1 C1 = A1 D. De plus, C\ 1 B1 D = C1 A1 D = 120 , donc B1 C1 D et
\
A1 C1 D sont isométriques et isocèles avec un angle de 120◦ . On en déduit que A1 DB1 C1 est un
losange avec un angle en C1 de 60◦ , donc A1 B1 C1 est bien équilatéral.

Remarque 6.3. Notons que la fin de la preuve est la même que pour l’exercice 22).

Solution de l’exercice 25

41
A

O0
C O

On note α = BAC [ = M \AN et r la rotation de centre A et d’angle α : on a r(B) = C et


r(M ) = N . De plus, r(O) = O0 , le centre du cercle circonscrit à ACN . On a donc O, C, N
et A cocycliques ssi O est sur le cercle de centre O0 qui passe par A, C et N ssi O0 O = O0 A.
Or, O0 = r(O) donc O0 OA est isocèle en A avec OAO \0 = α d’où : O0 O = O0 A ssi O0 OA est
équilatéral ssi α = 60◦ ssi ABC est équilatéral.
Solution de l’exercice 26

A
B
S

F T
Q

E C

D’après l’égalité sur les rapports de longueur, la similitude directe envoyant A sur B et
D sur C envoie également E sur F . En utilisant le théorème 4.14 pour les couples de points
(E, D) 7→ (F, C), on obtient que son centre est sur les cercles circonscrits à T CF et T DE. En
l’utilisant sur les couples de points (A, E) 7→ (B, F ), ce centre est également sur les cercles
circonscrits à SAE et SBF . D’où la conclusion.
Solution de l’exercice 27

42
F

Γ1
A

Γ2

D’après la construction du centre d’une similitude appliquée aux cercles Γ1 et Γ2 , D est le


centre de la similitude directe s1 qui envoie C sur A et A sur B. En regardant les cercles Γ1 et
Ω, D est le centre de la similitude directe qui envoie C sur F et A sur E, donc le centre de la
similitude directe s2 qui envoie C sur A et F sur E.
Or, comme il existe une unique similitude directe de centre D qui envoie C sur A (car le
DA
rapport est forcément DC et l’angle forcément ADC),
\ on a s1 = s2 , donc on a une similitude
qui envoie C sur A, A sur B et F sur E. Comme B est le milieu de [AE], on en déduit que A
est le milieu de [CF ].
Solution de l’exercice 28

P E

C
M
D

On se rend immédiatement compte qu’il existe une similitude de centre A qui envoie B
sur C, C sur D et D sur E.
Donc, d’après le théorème 4.14 appliqué au couple de point (B, D) 7→ (C, E), A est sur le
cercle circonscrit Γ1 à P BC et Γ2 à P DE. Ici, l’exercice commence à avoir bien la tête d’un

43
exercice utilisant la puissance d’un point. On essaye donc de montrer que Γ1 est tangent à
(CD). Or c’est vrai d’après la réciproque du théorème de l’angle inscrit comme ABC
[ = DCA.
\
De même, comme DEA \ = 180◦ − EAD \ − EDA \ = 180◦ − CAD \ − ACD \ = CDA,\ le cercle Γ2
est également tangent à (CD).
Finalement, en notant M = (AP ) ∩ (CD), M est sur l’axe radical de Γ1 et Γ2 et on peut
donc écrire M C 2 = PΓ1 (M ) = PΓ2 (M ) = M D2 et la conclusion.
Solution de l’exercice 29

P
E
R
Q
F

D
C

Il existe clairement en utilisant les égalités de longueur une similitude (et même une rota-
tion !) envoyant les points A, F et D sur les points C, E et B respectivement.
Il est naturel d’introduire son centre O et quelques dessins peuvent nous convaincre que
c’est vraisemblablement le point recherché. En utilisant successivement le théorème princi-
pal pour les couples (A, F ) 7→ (C, E), (F, D) 7→ (E, B) et (D, A) 7→ (B, C), on sait que O
est sur le cercle circonscrit aux triangles ARF , ERC, F QD, BQE, AP D et BP C. En particu-
lier (en utilisant les deux derniers triangles), il est fixe. Il est donc suffisant (et probablement
raisonnablement aisé au vu de tous les autres cercles...) de démontrer que O, P Q et R sont
cocycliques.
Or, le théorème de Miquel appliqué au quadrilatère AF P Q prouve qu’il suffit de démon-
trer que O est sur le cercle circonscrit à ARF , DP A et DF Q. D’où la conclusion.
Solution de l’exercice 30 On rappelle le théorème du pôle Sud, visiblement pertinent dans cet
exercice et démontrable grâce à une chasse aux angles élementaire (exercice !).

44
X

Si XY Z est un triangle inscrit dans un cercle C, I son centre du cercle inscrit et S le


deuxième point d’intersection de (XI) avec C. Alors, S est le milieu de l’arc Y Z et, plus préci-
sément, SY = SI = SZ.

A
L

I
Q

C
P
J

On est dans la situation classique avec deux cercles qui s’intersectent, on connaît bien
un des points d’intersection et c’est l’autre qui nous intésse. On cherche donc à compléter le
quadrilatère. De manière naturelle, on introduit donc les points fixes L et M , milieux respectifs
des petits arcs AC et BC. D’après le théorème du pôle Sud, P , I et L ainsi que P , J et M sont
alignés.
D’après le théorème 4.14, Q est le centre de la similitude S envoyant I sur J et L sur M .
(Comme toujours se pose la question de quelle similitude choisir : pourquoi pas celle envoyant

45
I sur L et J sur M ? Et comme souvent la réponse sera qu’on connaît mieux la première
similitude parce que l’on maîtrise bien les longueurs impliquées.) Cette similitude envoyant
le point fixe L sur le point fixe M , pour prouver qu’elle est fixe (et donc Q également), il suffit
de montrer que son angle de rotation et son rapport de dilatation sont fixes (un petit dessin
convaincra le lecteur sceptique...). Or, l’angle vaut LQM
\ qui est fixe d’après le théorème de
l’angle inscrit et le rapport de dilatation vaut JM/IL qui vaut CM/CL d’après le théorème
du pôle Sud, d’où la conclusion.
Solution de l’exercice 31

A1

B2
P

A2
B1
Q

On voit qu’on est naturellement dans une situation du type théorème de Miquel dans le
quadrilatère A1 B1 A2 B2 . En particulier, le cercle circonscrit à CA1 A2 passe par Q.
Cette remarque est positive pour de nombreuses raisons : on se rend compte que les points
B1 et B2 sont inutiles (O peut être défini comme le centre du cercle circonscrit à A1 QA2 ), ce
qui permet de simplifier la figure et de perdre un degré de liberté.

46
O

A1

P
O2
M1 A2

M2
O1

La question naturelle est maintenant : quel va être ce cercle que parcourra O ? Le plus
simple est de considérer les cas limites : quand A1 tend vers Q, A2 et donc O également.
Quand A1 tend vers P , A2 tend vers un point de Γ2 et O devient donc le centre O2 du cercle
Γ2 . De même, quand A2 tend vers P , O tend vers le centre O1 de Γ1 .
On cherche donc à montrer que O, O1 , O2 et Q sont cocycliques. Il faut naturellement
travailler avec des angles orientés, mais on s’en passera (exercice...).
Notons M1 et M2 les milieux respectifs de [A1 Q] et [A2 Q]. En utilisant les angles droits dus

aux médiatrices, M1 , O, M2 et Q sont cocycliques, d’où O\ 1 OO2 = 180 − A1 QA2 .
\
Or, la similitude de centre Q qui envoie A1 sur A2 envoie O1 sur O2 (d’après par exemple
le théorème 4.14 appliqué à (A1 , B1 ) 7→ (A2 , B2 )). D’où A\ 1 QA2 = O1 QO2 , ce dont on déduit
\

O\1 OO2 = 180 − O1 QO2 , et la conclusion par le théorème de l’angle inscrit.
\
Solution de l’exercice 32 Il est naturel pour obtenir des angles droits de considérer les milieux
M et N de [AC] et [BD].

47
A

P
Q
N
D M

On considère la similitude de centre Q qui envoie A sur B et C sur D. Elle envoie le seg-
ment [AC] sur le segment [BD] et en particulier M sur N . En utilisant le théorème 4.14 avec
les couples (A, M ) 7→ (B, N ), M , N , P et Q sont cocycliques. Or, en utilisant l’angle droit des
médiatrices, il et clair que M , N , P et O sont cocycliques.
D’où finalement M , N , P , Q et O cocycliques et OQP [ = 90◦ par le théorème de l’angle
inscrit.
Solution de l’exercice 33
a)
N

M0

M00

N0

T (M0 )

T (N )

Soit O le centre de la similitude t. Pour un point M , notons M 0 le milieu de M et de


T (M ). Fixons un point M0 (différent de O) et considérons la similitude directe S de

48
centre O qui envoie M0 sur M00 . Soit N un autre point et montrons qu’elle envoie N sur
N 0 . Les triangles OM0 t(M0 ) et ON t(N ) étant semblables, les triangles OM0 M00 et ON N 0
sont semblables. Il existe donc une similitude directe de centre O notée s2 envoyant M0
sur M00 et N sur N 0 . On en déduit que s = s2 , ce qui implique bien que s envoie N sur
N 0.
Remarque 6.4. Il est possible de résoudre facilement cette question en utilisant les
nombres complexes. En effet, toute similitude directe f de centre O (où O est l’origine)
est de la forme f (z) = az avec a et z deux nombres complexes. Pour résoudre la ques-
tion, il suffit donc de vérifier que (z + f (z))/2 est de cette forme. Comme (z + f (z))/2 =
(1 + a)/2 · z, z → (z + f (z))/2 est une similitude directe de centre O qui vérifie les
conditions de l’énoncé.
b) Soit K l’intersection des cercles circonscrits des triangles ABP et P CD.
B

C
P
O1
M N

A O2

On sait que K est le centre de la similitude directe t qui envoie [AC] sur [BD], et donc
M sur N et O1 sur O2 . On en déduit que M, P, N, K sont cocycliques. Considérons la
nouvelle similitude s de centre K envoyant tout point U sur le milieu de [U t(U )] (qui
existe par (i)). Elle envoie donc A sur M , B sur N et K sur K. Elle envoie donc le cercle
circonscrit de ABP sur le cercle passant par M, P, N, K. Comme O1 est envoyé sur le
milieu de [O1 O2 ], on en déduit que le milieu de [O1 O2 ] est le centre du cercle passant
par M, P, N, K.
Solution de l’exercice 34

49
Γ2
P4
P1 = P7
A
P2
Γ1
O1
P5 O2

B
F D

E C

Γ3
O3

P6

P3

On considère φB la similitude de centre B qui envoie P1 sur P2 et Γ1 sur Γ2 . De même, on


considère φD la similitude de centre D qui envoie P2 sur P3 et Γ2 sur Γ3 . on définit de manière
similaire φF , φA , φC et φE .
On note Φ = φF ◦ φD ◦ φB ◦ φE ◦ φC ◦ φA . On a alors P7 = Φ(P1 ). Or, l’angle de rotation de
Φ vaut (AO1 , AO2 ) + (CO2 , CO3 ) + (EO3 , EO1 ) + (BO1 , BO2 ) + (DO2 , DO3 ) + (F O3 , F O1 ) = 0
en éliminant les termes correspondants (voir figure). De même, le facteur de dilatation de Φ
vaut r2 /r1 · r3 /r2 · r1 /r3 · r2 /r1 · r3 /r2 · r1 /r3 = 1.
Φ est donc une translation qui envoie Γ1 sur lui-même, donc est l’identité, d’où la conclu-
sion.
Solution de l’exercice 35

M O2
C K

O1 J

Q
A0

K0

50
a) D’après la construction du centre d’une similitude, B est le centre d’une similitude
directe s qui envoie M sur P , C sur Q, le cercle Γ1 sur Γ2 . Par conséquent, s envoie
(M C) sur (P Q) donc, comme (M C) passe par un point fixe K, (P Q) passe par s(K),
noté K 0 . De plus, on peut construire K 0 : c’est l’intersection des tangentes à Γ2 en B et
A0 , où A0 = s(A). Or, A0 est la seconde intersection de Γ2 avec la tangente à Γ1 en A (cela
correspond au cas limite de s(M ) quand M tend vers A).
b) On note O1 et O2 les centres des deux cercles, et J le milieu de [P Q] : les angles O\ 0
2 JK ,
0 0 \0 0 0
O\2 A K et O2 BK sont droits donc O2 , K , A , B et J sont cocycliques sur le cercle de
diamètre [O K ]. De plus, en utilisant que BAA0 et BKK 0 sont semblables :
0 0

1\ 1\ 1\
0 KO = K
K\ 1
\ 0 KB + BKO
\1 = A
\ 0 AB + BKA = KAB + BKA = 90◦
2 2 2
donc K est aussi sur ce cercle.
[ = 180 − BK
On a donc BKJ \ 0 J = 180 − BK
\ 0 P = 180 − BKM
\ donc J, K et M sont
alignés, d’où le résultat.
Solution de l’exercice 36

Z Y

C
X F

D
E

On a clairement une rotation (donc une similitude) de centre le centre du cercle O qui
envoie A sur B, C sur D et E sur F .
En utilisant le théorème 4.14 sur le couple (A, C) 7→ (B, D), A, B, Z et O sont cocycliques.
Or, de même, en l’utilisant sur le couple (A, E) 7→ (B, F ), A, B, Y et O sont cocycliques.
Ainsi, A, B, Z, O, Y sont cocycliques, et de la même manière également C, Z, O, X, D et
E, X, O, Y , F .
Encore une fois, philosophiquement parlant, avec tant de cercles on connaît tous les angles,

51
donc une simple chasse aux angles devrait suffire pour terminer. Effectivement :

ZXY
\ = ZXO
\ + OXY
\
= ODZ
\ + OF
[ Y
= ODB
\ + OF
\ B
= OBD
\ + OBF
\
= DBF
\.

On conclut cycliquement.
Solution de l’exercice 37

A R

E
P D
L

K
C

Q
B

a) Q est le centre de la similitude ρ envoyant B sur D et E sur C. Comme BE = CD,


son facteur de dilatation est 1 i.e. c’est une rotation. En particulier, en appelant Q1 et
Q2 les projections de Q sur (AB) et (CD), comme ρ envoie Q1 sur Q2 , QQ1 = QQ2 ,
i.e. Q est sur la bissectrice (le lecteur attentif remarquera qu’il faut vérifier que c’est
bien la bissectrice intérieure, ce qui se fait facilement par un argument de continuité en
regardant le cas extrémal).
b) On remarque dans un premier temps que ρ envoie K sur L. En particulier, QK = QL,
d’où également RK = RL par Pythagore. De plus, l’angle de la rotation est KQL \ mais
est également, la droite (EB) étant envoyé sur la droite (CD), l’angle entre les droites
(BA) et (CA). En particulier, A, K, Q et L sont cocycliques d’après le théorème de
l’angle inscrit. Comme il est également clair que K, Q, L et R sont cocycliques, K, Q, L,
A et R sont cocycliques.
Le but de l’exercice est donc de montrer que R est le centre de la similitude envoyant
K sur L et B sur C. Soit R0 le centre de cette similitude. R0 comme R est sur le cercle
circonscrit à KQL. De plus, comme KB = LC, cette similitude est une rotation, d’où,
comme pour R, R0 K = R0 L. Ainsi, R et R0 font partie des deux points d’intersection
de la médiatrice de [KL] et du cercle circonscrit à KAL et un argument fumeux de
positionnement (le lecteur pointilleux remarquera que c’est formalisable sans trop de
difficulté) montre que c’est en fait les mêmes. D’où la conclusion.
Solution de l’exercice 38

52
A

P
S

C B

Commençons par nous occuper de P : d’après le théorème de Ceva et celui de Thalès,


(AP ) est une médiane de ABC, ce qui signifie que P[AB ne pourra pas s’exprimer de manière
simple. On va donc utiliser de la trigonométrie.
On remarque que Q est le centre de la similitude directe s qui envoie B sur N et M sur
C. Pour pouvoir faire des calculs trigonométriques sur QAB
[ et QAC,
[ on introduit les projetés
QS
orthogonaux R et S de Q sur (AB) et (AC). On a s(R) = S donc QR est égal au rapport de s,
QS/QA
par Thalès. On en déduit sin QAC
NC AC \ AC
soit BM = AB = QR/QA = AB .
sin QAB
\
D’un autre côté, si M est le milieu de [BC], on a en utilisant plusieurs fois la loi des sinus :

BM
sin P[
AB sin M
\ AB AM
sin ABM
\ sin ABC
[ AC sin QAC
[
= = = = =
sin P[
AC sin M
\ AC CM
sin ACM
\ sin ACB
[ AB sin QAB
[
AM

sin x
Autrement dit, f (P[ [ avec f (x) =
AB) = f (QAC) . On peut vérifier que cette
sin (BAC−x)
\

fonction est strictement croissante, par exemple en la dérivant (exercice), d’où P[


AB = QAC.
[

Remarque 6.5. La trigonométrie est une méthode très puissante. Pour passer de formules sur
des rapports de sinus (obtenues grâce à la loi des sinus et au théorème de Ceva trigonomé-
trique) à des égalités d’angles, le fait que f soit strictement croissante est très utile et à retenir !

Solution de l’exercice 39

53
A

B3
A2

C1
B2

B1
C3

A3

A1
B

C2

On a visiblement de nombreuses similitudes naturelles dans cette figure et qui dit simi-
litudes dit triangles semblables. Après étude de quelques figures, il semble que l’on puisse
montrer que C2 AB ∼ CB3 A3 .
Effectivement, AC\ 2 B = B3 CA3 d’après le théorème de l’angle inscrit. Comme A3 et B3
\
sont plutôt défini en termes de longueur, on cherche également à démontrer que CB3 /CA3 =
C2 A/C2 B. Or, le rapport de dilatation de la similitude de centre C2 envoyant B sur A et A1 sur
B1 vaut selon la manière de le calculer C2 A/C2 B ou AB1 /BA1 qui vaut exactement CB3 /CA3 .
On a donc bien C2 AB ∼ CB3 A3 . Cycliquement, on sait que A2 BC ∼ AC3 B3 et B2 CA ∼
BA3 C3 .
On connaît maintenant très bien tous les angles. Philosophiquement, on sait donc qu’il
suffit de faire une chasse aux angles. Effectivement :

2 A2 C2 = B2 AC2 = BAC2 + B2 AC − BAC


B\ \ \ \ [
3 B3 C + BC3 A3 − BAC = B3 A3 C3 .
= A\ \ [ \

D’où la conclusion en raisonnant cycliquement.

54

    
!!"#$ %'&)(*,+-.&/

0 1 25476/89;:<8%4=2,>72#?A@B8C6/DE8C6F?
3
GBH#IKJMLONKPLOIQHSRKHTUVLWJYXZLWJYT\[]KN_^N`XZNKP^'TYLWa$I`bOHdceTMLfXZN$R`LOTYIQXZTMHSIKbO]KTRKHhgji3k#XZlnmMH%UVTIf[oJMa$Lfg
UVLWJYXZLWJYTpq[obWXZJYT/]KNUYLOJYXoLOJrPXZNeUVLWHNeU[o]a$XZLWNKTrRKH]tsXZlnmMH%UVTuevNIwH]`U\HNR`XZNKNKHJF]KNKH<xoHJMTYLOXoN_I`bO]KT
^xoXZbO]K^HouwvdN*N`XoUVH=y{zn|o}V~bWHJY^TM]KbWUV[U#RKH.b€[RKLxtLWTYLWXZN7RKHEzIf[oJ'}d[oJMJYXZN`RKLB‚_b„ƒ…HNeUVLOHJSTY]KIQ^JMLOH]KJu
† bWXZJYTc
‡$ˆ‰‹Š#ŒF‰ŽB‘“’•”/‰‹ˆF–\‰‹ˆj’— ˜tLXoN™RKLWTYIQXZTYH-š›XZlnmMHUYT*IK[oJYa_L.gœUYLOJMXZLOJMTpS[obWXZJYT7]`NUVLWJYXZLWJ
PXZNeUYLOHNeUh[o];a_XZLONKTžy€šŸ|Zgn~EXZlnmMH%UVTu
Gƒ…[oJMJYXZNKR`LE‚ b„ƒ…HN¡UYLOHJ¢TM]KIQ^JYLWH]KJ;£¤[oLU7lKLOHNTM¥KJ;UVXZ]`UYH-b€[¦£{XZJYPH§R`H§bO[¨Pª©`XZTYHp'TM]KJMUYXZ]`U*TYL
š*«¬gi­kZuuWu
®E¯°ˆFŒF‰‹ŒF²±ž—ž³![JYLOT_PXoa$I`UYH;RKH]ts´a_LObWbOLOXoNKTR“ƒµ©f[ol`LWUª[N¡UYTuj¶hN¸·UVJMH*©¡]Ka$[oLWN¸[`pj[o]¹IKbW]KTp
ºZ»Z»»o»Z» Pª©KH%xZH]ns;TY]KJ<b€[.UY·UYHou † ]žxn]7RKHdPHTSRKXZNKN`^HT.¼{HU<TV[oPª©f[N¡U#PHbO[TYH]KbOHa$HN¡Uª½%ptPXZal`LOHN
RKH³/[oJYLWTYLWHNKT/IwH]`UM¾ŽXZNEUYJYXZ]`xoHJ/¿n]`LtXZNeU!H%s`[oPUYHa_HNeU\bWHa_·a_HNKXZalKJMH<RKHPª©KHxoH]tsTY]KJ/b€[#UV·UYHBÀ

E® ¯°ˆFŒF‰‹ŒFÂÁ¢—;vdN-[.mMHUV^RKHb€[IQHLWNeUV]KJMHNKXZLWJYHTM]KJbOHETYXZbFl`b€[oNKPR“ƒ…]`NKHIKLOÃPHP[oJYJM^HRKH_Ä
a$Ã%UVJMHT#TY]KJhÄtp`N“ƒ…LWa$IQXZJÅUVHdPXZa$a_HNeUuÇÆ*XZNeUVJMHJ#¿¡]“ƒµLObŸH%stLWTMUVHRKH]ts¢IQXZLONeUVTSR`H.bO[a$·a$HPXZ]KbWH]KJ
RKXZNeU'b€[RKLWTMUV[oNKPHHTMU'H%s`[oP%UVHa_HNeUh]KN¢a$Ã%UVJYHu
®E¯°ˆFŒF‰‹ŒFÉÈ*—žÊË^a$XZNeUYJYHÌ¿n]`Hop!IK[oJYa_LbOHTETMUV[oÍZL€[LOJYHTER°ƒ…]KNÎTMUª[ÍZH † N`LOa$[UV©“pŸLWbHN H%stLWTMUVH
RKH]nsž¿¡]KLQPXZN`Nf[oLOTMTYHN¡UH%s`[oPUYHa_HNeUSbWHha_·a_HhN`XZalKJMHhR“ƒ…[o]`UYJYHTTMUV[oÍZL€[LOJYHTh¼¤If[oTj£{XZJYP^a_HNeUbWHT
a$·a$HTETÅUª[oÍoL€[oLWJYHTp“a$[oLOTTMH]KbWHa_HNeUbWH_a_·a_HNKXZal`JYHϽCujÐÑvdN3TY]KIKIQXZTMH¿¡]KHbO[¢JMHbO[UVLWXZN´Ò¢TYH
PXZN`Nf[ÓOUYJYH.Ô7HTÅUhJM^PLOIKJMXt¿¡]KHcfTYLBz_PXZN`Nf[ÓOU'}ÏpÇ[obWXZJYT#}hPXZNKNK[ÓOU#zfuÖÕ
®E¯°ˆFŒF‰‹ŒF¢×=—=¼„ÊhLWØ_PLWbOHouѽ°¶'Na$[ÓOUYJYHFR“ƒµ^Pª©`HPTfmMXZ]KHj[o].a$XoLONKTŸ]KN`HjIK[oJMUYLOHIf[oJtmMXZ]`JpÏa$[oLOTB[o]
IKbO]`TFRKLsIf[oJÅUVLWHT!IK[oJFTYHa[LONKHueÆ*XZNeUYJYHJ!¿n]`HopeTƒµLObmMXZ]KH[oTYTMHÌ#bOXZN`ÍoUVHa$IKTpXoNIwH]`UFUVJMXZ]`xoHJ\]KNKH
TY^JYLOHR`HhmMXZ]KJYTPXZNKTM^P]tUVLW£{TRK]`JV[oNeUbOHTY¿¡]KHbOTLOb![.mMXZ]K^HCsK[PUVHa$HNeUÄٞIf[oJMUYLOHTu¼{ڋNKRKLOP[UVLWXZN¦c
TYXZLU;zZÛbOHÜNKXZalKJMHÜUVXUª[obhRKHIf[oJÅUVLWHT.mMXZ]`^HTEmM]`TY¿¡]“ƒ…[o]ÎmMXZ]KJžÝMpXZNœPª©KHJYPª©KH-ÝHUÞ¨UYHbOT¿n]`H
zàß#«zZÛ`i¸áoâ`uѽ

ã 1 2
6!ä8?ŸDE9¢972#å 2#9@æ4¢ä6)6!ç#:èž6r6!2#9;:<2
3
GBHJV[oLWTYXZNKN`Ha_HNeUËIK[oJËJM^P]`JYJYHNKPHHTMUdb€[_xoHJMTYLOXoNa[qUV©K^a[UYLO¿¡]KHRK]JY[oLOTMXZNKNKHa$HNeUÒ$RKH
IKJYXnPª©KHHN¢IKJMXtPª©KHÔZuKڋbŸTƒµ^N`XZNKPH.PXoa$a_HTY]KLUc
‡$ˆ‰‹Š#ŒF‰ŽBééˆ꓌FëSˆjˆj°Š#ŒFÜ— ˜tXZLOHNeUÎìríîªì\ïCîðððîªìrñ<ðððdR`HT3IKJYXZI`JYLO^%UV^T3a$[UY©K^a$[q¾
UVLW¿n]`HTujvdN¨TV[LWU¿¡]KH¢ì/í_HTÅU_xnJY[oLOHuvdN¹TY[oLWU[o]KTYTML#¿n]`Hop\IQXZ]KJ]KN¹šœ¿¡]KHbWPXZNK¿¡]KHpTYL#ìrñHTMU
xnJV[oLWH[obOXZJMT#ìrñòwï<HTMU'xnJV[oLWH[o]KTYTMLóu † bOXZJMTp`UYXZ]`UYHT#bOHT'I`JYXZIKJMLO^%UV^T#ìBñTYXZNeU#xnJV[oLWHTu
¶hNKH[oIKIKbWLOP[qUVLOXoN¢TMLOa_IKbOHRKH.PHEIKJYLWNKPLWIwH.HTÅUdbO[_RK^%ôfNKLUVLOXoN*If[JhJM^P]`JYJYHNKPHcwTMLrXoNRK^ôKNKLWU
]KNXZl¡mMHUËõÇíI`]KLOT'TYL„pwIQXZ]KJ#UVXZ]tUdHNeUYLOHJ'š!pQXZNR`XZNKNKHE]KNKHa$[oNKLWÃJYHRKHRK^ôfN`LOJ'bóƒµXZlnmMHU'õfñÏòwïh‚
If[oJÅUVLOJ<RKHEb„ƒ…XolnmMHUSõfñtpÇ[obOXoJYTbWHT#XZlnmMH%UVTSõfñTYXZNeU'lKLWHN;RK^ôKNKLOT#IQXZ]KJSUVXZ]tU#š!u
¶hNKH.RK^a_XZNKTÅUVJY[UVLWXZN7If[oJSJM^P]`JYJYHNKPHEPXZNeUYLOHNeU'RKXZN`PUYXZ]qmMXZ]`JYTSRKH]ts¢^UV[oIwHTc
—­G\ƒ…LWNKLWUYL€[obWLOTY[UVLWXZN¨cFPoƒµHTÅUbO[ÜxZ^JMLWôfP[UVLWXZN¨RKH¢ì/íu\ڋbSN`H;£¤[]`UËmY[oa$[oLOTEbóƒµXZ]KlKbWLOHJp/TMLONKXoN´XZN
JV[LOTYXoNKNKHTY]`J'R`]¢xnLOR`HFö
÷
—­GB[ JY^P]KJYJMHNKPH§I`JYXZIKJMHa_HNeU;RKLUVH cXZN TM]KIKIQXZTYH-¿¡]KH-b€[¦IKJMXZIKJMLO^UY^-ìrñ HTMU;xnJV[oLWH¹¼¤XZN
bóƒ…[oIKIQHbWbOHh©  IQXoUV©`ÃTYHERKHJY^P]KJMJYHN`PHϽCpÇHU'XZN;HTMTV[oLWH.R`H.a_XZNeUVJMHJ'ìrñòwï<‚IK[oJMUYLOJSR“ƒµHbWbOHou
®E¯°ˆFŒF‰‹ŒF  7—¢vdNžTMHdRKXZNKN`HdRKH]ns7NKXZalKJYHT EH%USzfufvdNžIQXZTMH °í«zH%UptIwXZ]`JUVXo]`US š 
`p
XZN¨RK^ôKNKLW U wñòwï « wñ i nu\³\JYXoIwXZTMHÌ7]KN`H;£{XoJYa]KbWHTMLOa_IKbOHžIQXZ]KJ wñ=HURK^a$XoN¡UYJYHÌ%¾‹bO[If[oJ
JY^P]KJMJYHN`PHou
®E¯°ˆFŒF‰‹ŒF  *—;vdN7RK^ôKNKLWU wí< «
HUpKIQXZ]KJUYXZ]`USš/pKXZN7IQXZTYH wñE « wñ fïwiΚ!u`GBHTSNKXZal`JYHT
wñÎTYXZNeñ U;…ñÏòw[ïIK IwHbO^TžNKXZal`JYHTžUYJYLO[oNKÍZ]KbO[oLOJMHT*¼¤IQXZ]KJM¿¡]KXZLàÀà½%u<Æ*XZNeUYJYHJ7¿¡]KHpIQXZ]KJ$UYXZ]`U7š/p<XZN [
wñE«  u
vN3N“ƒµHTÅUIf[T.XolKbOLWÍZ^RKHPXoa$a_HNKPHJ ‚
`uŸGB[;JY^P]KJYJMHNKPH$IQH]tU.UYJYÃT.l`LOHN°pŸIf[oJH%stHa$IKbWHop
PXZa_a_HNKPHJ#‚$áXo]*‚_â`pKTMLŸbOHT'I`JYHa_LOHJYTUYHJYa_HTSTMXZNeUËR`HT#H%stPHI`UVLWXZNKTuWuu
®E¯°ˆFŒF‰‹ŒF¢—7vdN=RK^ôfN`LWUrïh« kà|oáHUIQXZ]KJËUYXZ]`Uš ÉátpwXZN=IQXZTYH!wñž«"wñfï/i ñÑñï òwï u
ÊËXZNKN`HJ']KNKH£{XZJYa]KbOHdTMLOa_IKbOHËIwXZ]`#J wñ`u
GBHT\JV[oLWTYXZN`NKHa_HNeUVT!If[oJ!JM^P]KJYJMHNKPH'IQH]txZHNeUIKJM^TMHNeUVHJjR`HT\IKLWÃÍZHTpoHNIf[oJÅUVLOP]KbOLWHJBbOXoJYTY¿¡]KH
b€[_JY^P]KJYJMHNKPHHTÅUa[brLWNKLWUYL€[obWLOTM^Ho%u $]`HIQHNKTMHÌ%¾óxZXZ]`T.R`Hb€[_RK^a_XZNKTÅUVJY[UVLWXZNÜTM]KLWx[oNeUVH$c°XZN*xo[
a$XoN¡UYJYHJS¿n]`HopfRf[oN`TUVXZ]`U#ÍoJYXZ]KIQHdRKHš=IQHJMTYXZNKN`HTpfTƒ…LWb  [[o];a_XZLON`T<]KNKH£{Ha$a_H[obWXZJYTLWb°N“ƒ 
[*¿¡]KHžR`HTE£{Ha$a_HT'u &<HUYUYHžIKJMXZIKJYLW^UY^$HTMUl`LOHN§TY¥KJ.xnJV[oLWH$IQXZ]KJ.šœ« k*c/TMLRf[NKT]KNÎÍZJYXZ]`IwH
R“ƒµ]KNKHTYH]KbOHEIwHJYTMXZNKNKHLWb  [ž]KNKHE£{Ha_a$Hpw[bOXZJMTËLObBN“ƒ  [ž¿n]`HRKHTh£{Ha_a$HTu°˜t]KIKIQXZTYXoNKTRKXZNKP
¿¡]KHb€[IKJMXZIKJMLO^UY^dHTÅUSxnJV[oLWHdIwXo]KJ<UVXo]`USÍZJMXZ]KIQHdRKHšIQHJMTYXZNKN`HTuQvdNžx[a$XoN¡UYJYHJ¿¡]“ƒÑ[bOXZJMTHbWbOH
HTÅU'xnJY[oLOHdIQXZ]KJUYXZ]`U#ÍZJMXZ]KIQHdRKHšiœkIwHJYTMXZNKNKHTuw˜tXZLUSRKXZNKP]`N¢ÍoJYXZ]KIQHRKHšiœkIwHJYTMXZNKNKHT
PXZNeUYHNf[oNeU.]KNKH£{Ha$a_Ho)u ('H%UVLOJMXZNKTdb„ƒ…]`NKHRKHTIQHJMTYXZN`NKHT.¿¡]KLFNKHTYXoLWUIf[Tb€[£{Ha_a_Hop°XoN-[;]`N
ÍZJYXo]KIwH*‚§š­IQHJYTMXZNKNKHTu &<HÜÍZJMXZ]KIQH¢PXZNeUVLWHNeUž]KN`H*£{Ha_a_Hou³![J$©  IwXUV©KÃTYHRKH¢JY^P]KJMJYHNKPHp
PH_ÍZJYXo]KIwHNKH_PXoN¡UYLOHN¡UE¿¡]KH_RKHT.£{Ha_a$HT*u ('[àmMXZ]`UYXZNKT.b€[7IQHJYTMXZNKNKH_¿¡]“ƒµXZN¦[x[oLWU.HNKbOH%xZ^H¢cBTYL
PoƒµHTÅUh]`NKH£{Ha_a$HXZN¢[UVHJYa_LONK^uf˜tLWNKXZN“p`HNKbWHxZXoNKT#RK];ÍZJYXZ]`IwH]KN`H.R`HT#š3[o]`UVJMHTS£{Ha_a_HTuKvdN
XZl`UYLOHNeU#[bOXZJMT<]KN;NKXZ]txZH[]¢ÍoJYXZ]KIQHdRKHš=IQHJMTYXZN`NKHTSPXZNeUVHNK[oNeUË[]7a$XZLWNKT<]`NKHd£{Ha_a$HptH%U#If[oJ
©  IQXoUY©KÃTMHžRKHJM^P]KJYJMHNKPH7LOb\NKH$PXZNeUVLWHNeU¿¡]KH$RKHT.£{Ha_a$HTu/vdN [*R`XZNKPI`JYXZ]`xo^¿¡]“ƒµLObjN“ƒ  [
¿¡]KHER`HTS£{Ha_a$HTuÇ,v +¢HTÅUhb„ƒ…HJYJMH]KJÇÀ
®E¯°ˆFŒF‰‹ŒF.  -¢—¢Æ¢XZNeUVJYHJ¿n]`H$bO[;TMXZa_a$HRKHT.[oNKÍZbWHTR“ƒµ]KN§IQXZb  ÍoXZNKHNKXZN3PJMXZLOTM^_‚¢š¨0P /UV^T
¼¤1
š  âe½x[o]`3U 2{5 š 4¹7á 698\u
Êd[oNKT]`NKHhJM^P]KJYJMHNKPHopKIQXZ]KJja$XoN¡UYJYHJj¿¡]KHdìrñÏòwïFHTÅU<xnJV[oLWHop¡LObÇ[JYJYLxZH'¿¡]KHdìBñEN`HËTY]tØ$TMHdIf[oT
a[LOT¿¡]“ƒµXZN [oLUlwHTYXZLWN“pŸIf[oJ.H%stHa_IKbWHopBRKH_PXZNKNK[ÓOUYJYH_ìrñ;H%UìBñ fïCpBXZ]3HNKPXZJYH$RKH_TV[xZXZLWJ¿n]`H
UVXZ]tUVHT'bOHThI`JYXZIQXZTYLUVLWXZNKT#ìï%îððð%îªìrñ_TYXoN¡U'xnJV[oLWHT'IwXo]KJ#HNÜR`^RK]KLWJYHEìrñòwïC:u &<HUYUYHTMLWUV]K[UVLWXZN;HTMU
[oIKIQHbW^HJY^P]KJMJYHN`PH£{XZJMUYHou
‡$ˆ‰‹Š#ŒF‰ŽB3<3ˆj갌FëSˆjˆj°Š'ŒF<  ;t–jˆf”/Ü— ˜tXZLWHNeUìríîªìïªîðððîªìrñ<ðððqRKHT\IKJMXZIKJMLO^UY^TFa$[UY©K^%¾
a[qUVLO¿¡]KHTu°vdNÜTV[LWUd¿¡]KHì/íHTMUËxnJY[oLOHu“vdNÜTY[oLWUd[o]KTMTYLr¿n]`Hop°IQXZ]KJh]KNš§¿n]`HbOPXZNK¿¡]KHp°TYLrUVXZ]tUVHT
bOHTEIKJMXZIQXZTYLUVLOXoNKT.ì/íîªì\ïCîðððîªìBñ*TMXZNeUxnJV[LOHTpr[obOXZJMT.ìrñÏòwïdHTÅUxnJV[oLWH[]KTYTMLóu † bWXZJYTpBUYXZ]`UYHTbWHT
IKJYXoIKJYLW^UV^T'ìrñTMXZNeU'xnJY[oLOHTu
&<Hb€[3RKXZNKN`H*LWa$a_^RKLO[UVHa$HNeU]KNKH;xo[JYL€[N¡UYH;RKHÜbO[-R`^ôfNKLUVLWXZN´IK[oJ$JM^P]KJYJMHNKPH¦cTYL#XZN [
RK^ôKNKL#]KN´XolnmMHU_õÇí$H%U¿¡]KHpPXZNKNK[oLOTMTV[oNeU_bOHT_XolnmMHUYT_õÇíîMõ“ï%îðððîYõfñ`p\XoN´TY[oLWURK^%ôfNKLWJ_b„ƒ…XolnmMHU
TY]KLx[oNeU'õfñòwïCpÇ[obWXZJYT<UYXZ]KT#bOHT#XZlnmMHUYTSõfñTYXZNeU'lKLOHN;RK^ôfN`LOTu
®E¯°ˆFŒF‰‹ŒF  =*—7˜t]`J]KNKHdÓObOH'RK^TMHJMUYHdxnLWUp`‚bóƒ…[oNKN`^H » pt]KN7PXo]KIKbOHhRKHb€[IKLONKT>u &<©f[o¿¡]KH.[oNKNK^Hop
bOHTFPXZ]`IKbOHTFRKHSb€[IKLONK'T ?oÍZ^T\R“ƒ…[o]a$XoLONKT/RKH]ts[oNKTFTMHSJYHIKJYXnRK]KLWTYHNeU\H%U\HN`ÍZHNKR`JYHNeUj]KNNKXo]`xZH[o]
PXZ]`IKbOHdR`Hdb€[oI`LONKTu † LONKTMLópnbóƒ…[oNKNK^H » LWb  []KN7PXZ]KIKbWAH @`b„ƒÑ[oN`NK^H ÷ pt]KN7PXZ]KIKbWAH @`b„ƒÑ[NKNK^HÄtptR`H]ts
PXZ]`IKbOHT#¼¤bWHIKJMHa_LOHJ!TƒµHTÅU\JYHIKJYXnRK]KLUC:½ @qbóƒ…[oNKN`^HSÙ`pàUVJMXZLOTrPXo]KIKbOHT#¼¤HNKPXoJYHS‚ËP[o]KTMHRK]IKJMHa_LOHJV:½ @
bóƒ…[oNKNK^H BKpKPLWNK¿žPXZ]KIKbWHTE¼¤bWHTSRKH]ts¢IKJYHa$LWHJMTSPXZ]KI`bOHTSTMHETMXZNeU'JYHI`JYXnRK]KLWUYTª½CpKHUYPou
—vdNNKXUVH CBñ'bOHjNKXZalKJMHR`H<PXo]KIKbOHT!b„ƒÑ[oN`NK^Hjš!uÊËXZNKN`HJ!]`NKH£{XZJMa]KbWHjTMLOa_IKbOH\IQXZ]KJ/RK^ôKNKLOJ
Crñ$If[oJhJY^P]KJMJYHN`PH‚IK[oJMUYLOJ'RKDH CBñ fï#H%EU Crñ  uwGr[$TY]KLUVHR`HNKXZalKJYHTd[oLWNKTYLrRK^ôKNKLOHEHTMU
UVJMÃT#PXoNKN¡]KHEH%U'Tƒ…[oIKIQHbObWHb€[TM]KLWUYHRKGH F!LOlQXZNf[oPPL„u
Ä
—Æ*XZNeUYJYHJ#¿¡]KHpQIQXZ]KJUYXZ]`U#š/pfXZNÜ[bO[JYHbO[UVLWXZN Crñ Crñò  «C ñÏ òwï i2 4.k 6 ñ u
—vdN¨TYH7UVJMXZ]`xZHž£¤[PH¢‚]KN¹HTYP[obOLWHJ‚š a$[oJYPª©`HTp!HUXZN¹[=RKHTËmY[oalwHT[oTYTMHÌ;ÍZJV[NKRKHT
IQXZ]KJa_XZNeUVHJEbWHTEa[oJMPª©KHTEIf[oJE]KNKHXo]¦If[oJERKH]tswp/a[oLWTIf[oTIK[oJEUYJYXZLWTu/Æ*XZNeUVJMHJE¿n]`H
bOHNKXoalKJYHRKH.a$[oNKLOÃJYHTSRKL  ^JMHNeUVHTh¿¡]“ƒµXZN[RKH.a$XoN¡UYHJ#bóƒµHTMP[obWLOHJSHTMUh^ÍZ[obB ‚ CBñ`ur¼¤³![oJ
H%stHa$IKbWHop“XoN3IQH]tUa$XZNeUYHJ]KN-HTMP[obWLOHJ‚žUVJYXoLOTda$[oJYPª©`HTRKHUVJMXZLOTËa[oN`LOÃJMHTc°]KNKHIf[oJ
]KNKHpfXZ]¢]KNKHa[JYPª©KHIK]KLWTSRKH]ns°pÇXZ];RKH]ts¢a$[oJYPª©KHT#IK]KLOT#]KN`Houѽ
³![oJÅ£{XZLOTpSLObIQH]`U¢·UYJYH3]`UVLWbOH=RKH-NKH3If[oT¢HTMTV[  HJÜRKH3IKJMXZ]`xZHJ*R`LOJYHPUYHa_HNeUÜbO[ IKJYXZI`JYLO^%UV^
RKHa$[oNKR`^Hopfa$[oLWTR“ƒµHNÜI`JYXZ]`xoHJ#]KNKH.¿¡]KLŸTYXoLWU#]KN;IQH]¢IKbO]KTS£{XZJÅUVHuWuu
®E¯°ˆFŒF‰‹ŒF™± —;Æ*XZNeUVJMHJ#¿¡]KHopÇIQXZ]KJUVXo]`U#š!pfXZN¢[bóƒµLONK^Íe[obWLWUV^
k i k i k  i Ïi
k á
k á â š 
¼¤Ú‹NKR`LOP[qUVLOXoNÜc¡a$XoN¡UYJYHJ<b€[.IKJYXoIKJYLW^UV^'HNJYHa_IKbO
[ [oNeUáEIf[oJá 4¦kà|š*Rf[oNKTbOH#UVHJYa_HËRKHhRKJYXZLUVHu ½

1 ä >72#?B:<2#9@B2
89 *978%2
§
Gr[7RKHTYPHNeUVH_LON`ôKNKLOHHTMU.bOHIKJYLWNKPLWIwHTMHbWXZN=bOH¿n]`HbObWHLOb/N“ƒµH%stLOTÅUVHIf[oTRKHTM]KLWUYHTMUYJYLOP%UVHa$HN¡U
RK^PJYXZLWTYTY[oNeUVHR“ƒµHNeUYLOHJMTIQXZTYLUVL£{Tu &<HÜI`JYLON`PLOIQH¢HTMUJYHRKXZ]`UV[olKbOH¢IQXZ]KJI`JYXZ]`xoHJ$¿n]°ƒ…LWbhN“ƒµH%stLWTMUVH
If[oThRKHTMXZbO]`UYLOXZN¢‚_PHJÅUª[oLWNKT'IKJYXZl`bOÃa_HT#£¤[oLWTV[oNeUhLONeUVHJMxZHNKLOJ'RKHTËNKXZalKJYHTËHNeUYLOHJMTcÇTYL/‚$IK[oJMUYLOJ
R“ƒµ]KNKHTMXZbO]`UYLOXZN°pQXoN=TV[oLUËHN£¤[olKJYLW¿¡]KHJh]KNKH[o]tUVJYHETMUYJYLOP%UVHa$HN¡UdIKbO]KThIwH%UVLUVHa$[oLWT'UYXZ]qmMXZ]KJMTËHN
NKXZal`JYHTËHNeUVLWHJYTpwH%Ud¿¡]“ƒµXZN=IQH]`UËJYHPXZa_a$HNKPHJËLON`RK^ôfN`LOa_HNeUpw[obOXoJYT'bOH.IKJYXZl`bOÃa_H.LONKLUVL€[b“N“ƒ…[
If[oT#RKHTYXobO]`UYLOXZN“uuu
®E¯°ˆFŒF‰‹ŒF ±Ÿ±7— (#^TMXZ]KRKJMHHNN`XZalKJMHTËHNeUVLOHJYThbóƒµ^¿¡]f[UYLOXZN;õ i á ' « 
Ïu/¼¤Ú‹NKR`LOP[qUVLOXoN§c
JYHa[oJM¿¡]KHJ#¿¡]KHõHTMUhIf[oLOJu ½
† ]`UYJYH<£{XZJMa]KbO[UVLWXZNRK]a_·a_HjI`JYLON`PLOIQH'cqUYXZ]`UVHIf[oJÅUVLOHjNKXZNExtLWRKHjRKH ¹[oRKa_HU!]KNIKbW]KT/IQHUYLWU
^bW^a_HNeUutÊËHËa$[oNKLWÃJMHhÍZ^NK^JY[obOHp¡PoƒµHTMU<UVXZ]qmMXo]KJYT]KNKHËlQXZNKN`HËTMUYJV[UY^ÍZLWHope£¤[oPHd‚]KNHNKTMHalKbWHËRKH
IwXoTYTYLWlKLObWLWUY^TpqRKHSPXZa$a_HN`PHJ!If[oJFHCsK[a$LWNKHJFbO[dIKbO]`TFIwH%UVLUVHdcÇÒdTMXZLWU/õbO[dIKbO]KT!IwH%UVLWUYHTYXobO]`UYLOXZNÔZp
XZ]=HN`PXZJMH¢ÒTYXZLU -bOHEIKbO]KTËIwH%UVLWUË£¤[oPUYH]KJdI`JYHa_LOHJËRKHš Ôeuuu“TYXZNeUR`HlwXZN`TdRK^IK[oJMURf[NKTdRKHT
IKJYXolKbOÃa$HTSR“ƒÑ[JYLWUY©Ka_^UVLW¿¡]KHou
 1 2#? 89dä6/8äE9@r?)2@æ?å ç@Ÿ6!8%2#?
3
GBH$IKJYLWNKPLWIwHRKHTLWN¡x[oJML€[oNeUYT.HTMU[oTYTMHÌ$x[oÍo]KHopBPoƒµHTÅUIKbW]`U /oU.]KN`H$a_^UY©KXtR`H$¿¡]“ƒµ]KN¦^NKXZNKP^
a[qUV©K^a[UYLO¿¡]KHdI`JY^PLOTu
‡$ˆ‰‹Š#ŒF‰ŽB­<“’3‰‹Š !Sˆ"‰ !SŠ”!’.— ˜nLr]`NKH¿¡]f[oNeUYLWUV^EHTÅUdPXoNKTYHJMxZ^HIf[oJ']KNKHEPHJÅUª[oLWNKH.Pb€[TYTYH
RKHUVJV[NKTM£{XZJMa[qUVLOXoNKTp°[obOXZJMTLObFHTMU.LOa_IQXZTYTMLOlKbWHRKHIf[oTMTYHJ.R“ƒµ]KNKH_TYLUV]f[UYLOXZN-‚;]KNKH$[o]`UYJYH7X +3b€[
¿¡]f[oNeUVLUV^HTÅUhR`L  ^JYHNeUVHHN¢]`UVLWbOLWTV[oNeUSTMH]KbWHa_HNeU'RKHTSUVJY[oNKTÅ£{XZJYa$[UVLWXZNKTR`H.PHUMUVHPbO[oTYTMHou
&<H*TMXZNeUTMXZ]`xZHNeUbWHTTYH]KbOTa_X  HNKT_¿¡]“ƒµXZN [-IwXo]KJIKJYXo]`xZHJ$¿¡]KH¢PHJMUª[LONKHT_Pª©`XZTYHTTYXoN¡U
LOa_IwXoTYTYLWlKbOHTu°¼„³\JYXo]`xZHJ'¿¡]KH¿¡]KHbO¿¡]KHPª©KXZTMHEHTMU#IQXZTYTMLOlKbWHdHTÅUhTMXZ]`xoHNeUËI`bO]KT£¤[oPLObWHcKLWb°TY]tØ_U'RKH
RKXZNKN`HJ#]KN¢H%stHa_IKbOHu ½
Gr[If[oJMLWUY^dHTÅUËTMXZ]`xZHNeUËb„ƒ…]KN;RKHTha_HLWbObWH]KJMT<LONex[oJML€[oNeUVTS¿¡]KHb„ƒ…XZN;IK]`LOTYTMHUVJYXo]`xZHJuuWu
®E¯°ˆFŒF‰‹ŒF™±wÁ¢—7³/H]tUM¾‹XoN*JMHPXo]`xnJYLOJS]`N*^Pª©KLO¿¡]KLWH$J #%&#$[xoHP.RKHT#RKXZa_LON`XZTd'k %-á\À
®E¯°ˆFŒF‰‹ŒFA±QÈܗž¶hN`H$£{H]`LObObWHRKHIK[oIKLOHJEHTMURK^Pª©KLWJY^HHN§UVJYXoLOT.If[oJMUYLOHT)u (\NKTY]KLUVHp/b„ƒ…]KN`H$RKH
PHT.IK[oJMUYLOHTËHTMU.RK^Pª©KLOJM^HRKHNKXo]`xZH[o]§HN=UVJYXoLOTdIf[JMUVLWHTp°HU.[LONKTMLFRKHTY]`LWUVHuŸ³/H]`UM¾ŽXZN-XZl`UYHNKLWJp
HN;ôfN¢RKHPXZa_I`UVHpf]KN7UVXoUV[ob“RKHPHNeUËIK[oJMUYLOHTfÀ
Ù
E® ¯°ˆFŒF‰‹ŒF™±K×=—7¶hNKHÍZJMHNKXZ]`LObObWHËRKXoa$HTMUVLW¿¡]KH.[^%UV^IKbO[oP^H.IK[oJSTYXZN¢a$[ÓOUYJYHdRf[NKT#]KNKH.TV[obWbOH
RKHlf[oLWNKTP[JYJYHbO^HX7+;TYXZNeU'LONKTÅUª[obWbO^T<]KN¢[olKJML¼ † ½<HU#]KN`H.lK[oLOÍZN`XZLOJMH¼ ½%uKGB[ÍZJMHNKXZ]`LObObWHËTY[o]`UVH

RKH_P[oJMJYH[]§HNÎP[JYJYH[o]¦‚;b€[7JMHPª©KHJYPª©KHžR`H$b„ƒ…HN¡UYJY^H_bO[7lf[oLOÍoNKXZLOJMHIKbOHLONKHR°ƒ…H[o]“u)(\bObOHN`H$TY[oLWU
TV[o]tUVHJ#¿¡]KH.RKHT#a$[oNKLWÃJYHTSLONKR`LO¿¡]K^HT#If[oJSbOHT fÃPª©KHTËTM]KJ#bOHRKHTYTYLWN§c


† JYJMLWxoHJV[ྎUM¾ŽHbWbOH‚If[oTMTYHJhRKHTMXZNÜ[lKJYLŸ‚bO[lK[oLOÍZN`XZLOJMHBÀ
A

®E¯°ˆFŒF‰‹ŒF ±:;— (TÅUM¾ŽLOb¡IwXoTYTYLWlKbOHjRKHJY^IK[oJMUYLOJrbOHTrHNeUYLOHJMT<kZîªátîððð%îªâZâ'HNžkok<ÍZJMXZ]KIQHT!R`LOT{mMXoLONeUVT
RKH;UVJYXoLOT^bW^a_HNeUVT_Pª©f[oP]KN“pjRKH*TMXZJMUYH*¿¡]KH¢Rf[oN`TPª©f[o¿¡]KH*ÍZJYXo]KIwH;b„ƒ…]KN¹RKHT^bW^a_HNeUVT_TYXoLWU_b€[
TYXZa_a_HRKHT#RKH]ts*[o]`UYJYHTfÀ
®E¯°ˆFŒF‰‹ŒF²± *—;ÄZÄ-[oJYlKJMHT$TYXZNeUža_LOTHN JYXZN`ER @jTM]KJPª©K[o¿¡]KH=[oJMlKJYH¢TYH*IwXZTMHÜ]KN´PXZJMlwH[o]“u
 /Xo]`UVHT<bOHTa_LON¡]`UYHTpnR`H]tsPXZJMlwH[o]ts$TYHhRK^I`b€[oPHNeU<Pª©K[oP]KN7TM]KJ<]`Nž[oJMlKJYH#xZXZLWTYLWNRK]$bOH]`Ju (\TMUM¾
LObfIQXZTYTMLOlKbWHSIwXo]KJbWHTPXoJYlQH[o]tswpn[oIKJMÃTj]KNPHJÅUª[oLWN$NKXoalKJYH#RKH'a_LON¡]`UVHTpeRKH'TYHhJV[oTMTYHalKbOHJ\UVXZ]KT
TY]KJ#bWHa$·a$H[oJMlKJYHŸÀ
GBHThT  a$^%UVJYLWHT#TYXoN¡U']KN¢P[T'IK[oJMUYLOP]`bOLOHJ<R“ƒµLONex[oJML€[oNeUc
‡$ˆ‰‹Š#ŒF‰ŽB¨< ’ žêw”!ˆj‰óÜ—æ˜nLQ]`NKHhTMLWUY]f[UVLWXZNHTÅUT  a_^UYJYLO¿¡]KH'HU¿¡]“ƒµXZNžNKH'IwH]`Ub€[UVJV[NKTž

£{XZJYa_HJË¿n]°ƒ…HNÎ]`UYLObOLWTV[oNeUËRKHTUVJY[oNKTÅ£{XZJYa$[UVLWXZNKTdT  a$^%UVJYLW¿¡]KHTpB[obOXZJMTdXZN3NKHIQH]`U.If[oT.[oJMJYLxZHJ‚
]KNKHTYLUV]f[qUVLOXoN7NKXZN¢T  a$^%UVJMLO¿¡]KHou
®E¯°ˆFŒF‰‹ŒF :± ¢—ž¶hN*mMH%UVXZN HTMUIQXZTY^TY]KJPª©K[o¿¡]KH7P[oTMH7R“ƒµ]KN Rf[oa_LOHJ % tu!GBHTP[TYHTR`]
Rf[oa_LOHJ.TMXZNeUJMHIQ^JM^HTRKHbO[;a$[oNKLWÃJMH_TM]KLWx[oNeUYHÜcBb€[;P[oTMHPHNeUVJY[obOH$[*bWHTPXnXZJYR`XZNKNK^H5 T 2 õ´«




`î «
6 @Ÿb€[7PXnXZJYR`XZNKNK^Hõ [o]KÍZa_HNeUVHRKH;kxZHJYT.b€[7RKJYXoLWUVHH%U.b€[7PXnXZJYR`XZNKNK^H [o]KÍZa_HNeUVH
RKH-k7xZHJMT$bWH7©f[o]`Uu † LWNKTYLbO[P[oTMH;HN¹lf[oT_‚=Íe[o]KPª©KH¢HTÅU_b€[P[TYH 2{õ« 4hâ`î ´« 4hâ 6HU_b€[
P[oTMHHN¢©f[o]`Uh‚RKJYXoLWUVH*p 2 õ;«™â`î «™â 6CufÊËH]tsmMXZ]KH]KJYT#IKJMHNKNKHNeUpKUVXo]KJ'‚UYXZ]KJpKRKHT!mMH%UVXZNKT#HN
JYHTYIQHPUV[oNeUËbO[JYÃÍZbOHTM]KLWx[oNeUVHc
— TYL\bWHIKJMHa_LOHJmMXo]KH]KJI`JYHN`R§]KNmMH%UVXZN3TYLUV]K^TM]KJb€[žP[TYH 2{õΫ zfî « } 6%pŸLWb!RKXoLWU[o]`TYTYL
IKJMHNKRKJMHhbWHT“mMHUYXZNKTRKHTP[oTMHT 2{õ7« 4Ë}Ïî «™z 6%p 2{õ7« 4hzÇî « 4h} 6\HU 2 õ;«™}Ïî  « 4Ëz 6:@
— TYLBbWH.R`H]tstLOÃa$HmMXZ]KH]KJ'IKJYHNKR]KNmMHUVXoN*TMLWUV]`^ETM]KJhbO[_P[oTYH 2{õ*«Âzfî _«Â} 6%pÇLObŸR`XZLWU'[o]`TYTYL
IKJMHNKRKJMHdbOHTrmMHUYXZNKTRKHTSP[oTYHT 2{õ7« 4Ëzfî «™} 6%)p 2{õ¢ « 4Ëzfî . « 4Ë} 6H%GU 2{õ7«zfî  « 4Ë} 6%u
GŸH\mMXZ]`H]KJ¿¡]KLQ[.ÍZ[oÍZNK^#HTÅU<bWHSIKJYHa$LWHJ¿¡]KLQ[oJYJMLWxoHh‚.[UMUVHLWNKRKJMHSb€[TYLWUY]f[UYLOXZN7X +_LObfJMHTÅUVH']KN$TMH]Kb
mMHUYXZN“pfTYLUV]K^TM]KJ#b€[P[TY H 2{õ7«•kZî «
6%uK¶hN¢RKHT#RKH]tsmMXZ]KH]KJYT#IQH]`Už‹LOb°Íe[oÍZNKHJÇÀ
GBHT<LWN¡x[oJML€[oNeUYTjIQH]txZHNeU[o]KTYTMLw·%UVJMHhRKH#xZ^JYLUª[olKbWHTj¿¡]f[oNeUVLUV^TjN¡]Ka_^JYLW¿¡]KHTRK^ôfN`LOHT‚EIK[oJMUYLOJ
RKHT'RKXZN`NK^HTu
®E¯°ˆFŒF‰‹ŒF™± -*— ÂIf[oJMUYLOJSR“ƒµ]KN;UVJMLOIKbWHGU 2¤zÇîV}àî 06<XZN¢IQH]`U'H  HP%UV]KHJ#bóƒµXZIw^JV[UYLOXZN7TM]KLWx[oNeUYHc
— XZN;Pª©KXZLWTYLU#RKH]ts¢R`HT#NKXZal`JYHT#RK];UVJMLOIKbWHUp`a_HUYUYXZNKTSõH%U  @


— XZN_JYHa$IKbO[oPHSõIf[oJ 2{õ 4  6M| áHU If[oJ 2{õSi  6Y| átpeHN$b€[oLWTYTY[oNeUbOHSUVJMXZLOTMLOÃa_HNKXoalKJYH


 
LON`Pª©f[oNKÍZ^u

B
/³ H]`UM¾ŽXZN-If[oTYTMHJRK]=UYJYLOI`bOHUËLONKLUVLO[ob 2óátî átîkÏ| á 6Ë[o]UVJMLOIKbWHU2ÅkZî ánîkSi á76hHN-JYHTMIwHPUV[oNeU
   
PHThJMÃÍZbWHTÇÀ
Gr[EIf[oJYLUV^'JYH%xtLWHNeUS‚ERKLWxnLOTMHJjbOHT<NKXZalKJYHT<HNeUYLOHJMT<HN7RKH]tsPXo]KbOH]KJYT<[obWUYHJYNK[UVLxZHa_HNeUhctbWHT
a]KbUVLOI`bOHT!RKHháR°ƒ…]KN`HhPXZ]KbOH]KJpobOHTa]`bWUVLWIKbOHT!RKHhádIKbW]KT#k'R°ƒ…]KN`HË[o]tUVJYH#PXZ]KbOH]KJuevN_IQH]`U£¤[oLWJYH
b€[a_·a_HPª©KXoTYH[xZHP$âc°XZN=PXZbWXZJYLWHETM]KPPHTMTYLWxoHa_HNeUEbOHTHNeUVLOHJYTdHN=UVJYXoLOTdPXZ]KbOH]KJYTp“‚žTY[ÏxoXZLOJ
R“ƒµ]KN0P /oUY^SbOHT\a]KbWUYLOIKbWHTrRKH#â`poR°ƒ…]KN$[o]`UYJYHP /oUV^SbWHT\a]`bWUVLWIKbOHT/R`H#âdIKbO]KTSkopoR“ƒµ]KNUVJMXZLOTMLOÃa_H<P /oUV^
bOHTa]KbWUYLOIKbWHT'RKH_âžIKbO]`T.átuŸ˜tL!RKH]ts=NKXZalKJMHTz¢HU.}ETMXZNeU.RKHb€[a_·a_HPXZ]KbOH]KJp°XZN-RKLWUË¿n]`H
z$HTMU'PXZN`ÍZJY]¢‚}ha_XtR`]KbOXâtuÇvdN¢IQH]tU'[o]KTYTMLŸRK^ôKNKLOJR`HT#PXZN`ÍZJY]KHNKPHTha_XnRK]KbOX KpKXZ]7a_XtR`]KbOX
N“ƒµLOa_IwXZJÅUVHd¿¡]KHbBNKXZalKJMHou
/JMÃT.TYXZ]`xoHNeUpBRf[NKTRKHTIKJMXZlKbOÃa$HTp“]`NKHPHJMUª[LONKH¿¡]f[oNeUVLUV^JMHTMUYHLONex[oJYLO[oNeUVHa_XtR`]KbOXžš
IwXo]KJ]`NPHJMUV[oLON_Dš @nPHbO[xoH]`U<RKLOJMH#¿n]°ƒ…HbObOH'NKH'Pª©f[oNKÍoHË¿¡]KHËIK[oJRKHTa]KbWUYLOIKbWHT\RKH'š!>u &<©KHJYPª©KHJ


]KN¨š UVHbS¿n]`H7PHJÅUª[oLWNKHT¿¡]f[oNeUYLWUV^TNKH7Pª©f[oN`ÍZHNeU$¿n]`H7If[oJRKHT_a]KbWUYLOIKbWHTR`H7š £{XZ]KJYN`LWUUYJYÃT


TYXZ]txZHNeURKHSlQXZNKT\LONex[oJML€[oNeUVTuZGB[Ëx[obOH]KJFRKHSš;‚dUYHTMUYHJ\IQH]`UIK[oJM£{XZLWTF·UYJYHSRKH%xnLONK^H'IK[oJFbóƒµ^N`XZNKP^
¼  [ྎUM¾ŽLOb TMXZJMUYHT#R“ƒµXZlnmMH%UVTÇÀJMHÍe[oJMRKHÌa_XtR`]KbOX tp`IwH]`UM¾Ž·UVJMHa$XnRK]KbWX $Xo ] tuWuu ½Cu
®E¯°ˆFŒF‰‹ŒF ± =ܗ7˜t]`JË]KNKH.ӀbWH.R`^TYHJMUYHxnLWxZHNeU
â $P[oa_^bW^XZNKTu † ]RK^If[oJÅU $TMXZNeUjmY[o]`NKHTp/k


TYXZNeUJYXZ]KÍoHTH%Uk -TYXZNeU$xZHJÅUVTuGŸXZJYTM¿¡]KHÜRKH]ts¸P[oa_^bW^XZNKT_RKH*PXZ]KbWH]KJMTRKL  ^JYHN¡UYHT$TYHÜJMHNt¾

PXZNeUYJYHNeUp/LWbOTIKJYHNKNKHN¡UEUVXZ]`TEbWHT.RKH]ts§bO[;UYJYXZLWTYLOÃa$HPXZ]KbWH]KJuBGBXoJYTY¿¡]KH$TYH$JYHN`PXZNeUVJMHNeUR`H]ts
P[oa_^bW^XZN`ThR“ƒµ]KNKHEa$·a$HPXZ]KbOH]KJ'LObŸNKH.TYHEIf[oTYTMHJMLOHN°u † ]ÜlQXZ]`UhR“ƒµ]KN=[oN¢UVXZ]KT'bOHThP[oa_^bO^XZNKT
TY]KJ'bóƒÖӀbWHdTYXoN¡UhRKH%xZHN¡]KTËRKHb€[a$·a$HPXZ]KbOH]KJuÇGB[o¿¡]KHbObWHBÀž¼¤Ú‹b“£¤[]`U'NKXZN¢TYH]`bOHa_HNeUhRK^UYHJMa$LWNKHJ
b€[PXo]KbOH]KJpKa$[oLWTS[o]KTMTYLBIKJYXo]`xZHJ'¿¡]KH.PoƒµHTÅU'b€[TYH]KbOHIQXZTYTMLOlKbWHouѽ
®E¯°ˆFŒF‰‹ŒF Á —;vN;RKLWTYIQXZTYHR“ƒµ]KNKHIKLWbOHRKH ÷Ï»o»`÷ mMHUYXZNKTuQvdN¢]`UVLWbOLWTYHdbWHTSJYÃÍZbOHTSTY]KLx[oNeUVHT.c
—­[o]I`JYHa_LOHJhPXZ]KI“pwXZNPª©KXoLOTYLUË]`N$mMHUVXoN¿¡]KHbóƒµXZN*^bOLWa$LWNKHRK]$mMH]“pwH%UXZNTM^If[JYHbO[$IKLWbOH
HN¢RKH]ts¢IKLObWHTS[JYlKLUVJV[LOJYHT @
— IK]KLWTpǂPª©f[o¿¡]KH.PXo]KI“pÇXZN;Pª©KXZLWTYLWU#]`NmMH%UVXZN¢¿¡]KHbóƒµXZN;^bOLWa$LWNKHËR`]mMH]“pfHU'XZN¢TY^If[oJYH.]KNKH
IKLWbOH¼{If[oTS£{XZJYP^a_HNeU'PHbObOHR`XZNeU#XZNÜ[HCstUYJV[oLU#bOHmMH%UVXZNǽ<HN*R`H]ts¢IKLWbOHT#[oJYlKLUVJY[oLOJMHTu
³/H]`UM¾ŽXZNTYHR`^lKJMXZ]KLObWbOHJrIQXZ]KJ/¿¡]“ƒÑ‚d]`Na$Xoa$HN¡U/RKXZN`NK^XZNN°ƒÑ[oLU!¿¡]KH<RKHTFI`LObOHT!RKHjUVJMXZLOTfmMHUVXoNKTÇÀ
¼ † UMUVHNeUYLOXZN¨c“TMLF]KNKHIKLWbOHN`HPXZa_IwXoJMUVH¿n]°ƒ…]KNžmMHUYXZN=HU¿¡]“ƒµXZN3JYH%UVLOJMHPHhmMH%UVXZN°p“XZN-PXZN`TYLOR`ÃJYH
¿¡]“ƒ…XoNÜ[RK^TMXZJYa$[oLWT]KNKH.IKLObWH‚ » mMHUVXoN“pfHU'NKXZN;¿¡]KH.b€[IKLWbOH[RKLWTYIf[oJM]“uѽ
¶hN7[o]tUVJYH#UVJM]KP.cnR`ÃT<¿¡]“ƒµLObÇHTMU¿¡]KHTMUYLOXZN$RKHËTMXZa$a_H#RKHTPª©KL  JYHTpnLObK£¤[o]tUIQHN`TYHJ‚.JMHÍe[oJMRKHJ
a$XnRK]KbWX #¢cÇHN*H  HUpQ]`NÜNKXZalKJYH.HUhb€[TYXoa$a_HRKHETMHThPª©KL  JYHTËTYXoN¡U'UVXo]qmMXZ]KJMT'RK[oNKTËbO[a$·a$H
PbO[oTYTMH.a_XnRK]KbOX #¢¼{PoƒµHTMUhPH¿¡]KL“£¤[oLUSa$[oJYPª©KHJ#b€[;ÒIKJYH]`xZH.If[oJ #$ÔK½%u
®E¯°ˆFŒF‰‹ŒF™Á/±—;vdN*^PJMLWUhTY]KPPHTYTYLxZHa$HN¡UUYXZ]KT'bOHThNKXoalKJYHThR`Hk‚]KNa_LObWbOLWXZN“un³\]KLOTpwXZN
JYHa$IKbO[oPHPª©f[¿n]`HNKXZal`JYH.If[oJ'b€[TYXZa_a_H.R`HTYHTËPª©KL  JMHTu°³\]KLWThXZN*JYHPXZa_a$HNKPHpomM]KTY¿¡]“ƒ…‚PH
¿¡]“ƒ…LWbŸN“ƒ  [oLWUIKbO]KT#¿¡]KHRKHThNKXoalKJYHT'‚]`N*Pª©KL  JMHou $d]KHbŸPª©KL  JYH.[oI`If[oJV[qÓOUSbWHIKbO]KTSTMXZ]`xoHNeUQÀ

 ›DEå 4 @rä.252@ 89;:


è;?Ÿ8DE9 K2
ž:
è¢?Ÿ8%D9
;
GBHhIKJYLWNKPLWIwH#R“ƒµLON`PbO]`TYLOXoNt¾‹HCs`PbO]KTMLOXZNIQHJMa$H%U<RKHhPXZa_I`UYHJjbOH#NKXZal`JYH'R“ƒ…XolnmMHUYT¿¡]KLfxZ^JMLWôfHNeU
b€[7IKJMXZIKJYLW^UY^  XZ]§b€[7IKJYXoIKJYLW^UV^ pBTY[oPª©f[oNeUE¿n]`HPHTIKJMXZIKJMLO^UY^TNKH_TYXZNeUEIf[oT.H%stPbW]KTYLxZHTHU
IwH]`xZHNeU·UVJMHJY^[obOLWTY^HT'HN=a_·a_H.UYHa_IKTcQLObB£¤[o]`UË[oRKRKLUVLWXZNKNKHJhbWHENKXoalKJYHER“ƒµXZlnmMHUYT[  [oNeU 
HUEbOH_NKXZal`JYH_R“ƒ…XolnmMHUYT[  [oNeU pBIK]KLOT.JYH%UVLWJYHJbWH$NKXoalKJYHR“ƒµXZlnmMH%UVT[  [oNeU‚¢b€[7£{XZLOT AHU 
¼¤PHT'XZlnmMHUYT#XZNeUh^%UV^PXoa$I`UY^T#RKH]ts7£{XZLWTª½%u
vN=N`XoUV H `bOHN`XZalKJMHR“ƒµ^bO^a$HNeUVTdR“ƒµ]KN=HNKTYHalKbOH uŸ˜nL  H%U æTYXoN¡UdRKHTdHNKTYHalKbWHTp
XZN=NKXoUYH æbOH]KJdJM^]KNKLWXZN¹¼¤b„ƒ…HNKTYHalKbWHRKHT.XolnmMHUYT[oIKIf[oJÅUVHNf[oNeU ‚  XZ]- ‚  Xo]-[o]ts
RKH]nsf½CpÇHU "! bOH]KJSLONeUVHJYTMHPUYLOXZNμ¤b„ƒ…HNKTYHalKbWHRKHT#XZlnmMH%UVTh[oIKIf[JMUVHNf[oNeUh‚ ÂHUh‚ ½%u

#
$‡ ˆ‰‹Š#ŒF‰ŽB²€‰‹Š#Œ`ëS’Z‰Ž–jŠw¯jŒtëS’Z‰‹–Š — vN­NKXoUYH jbWH=NKXZalKJMH-R“ƒ…XolnmMHUYT*R“ƒµ]KN­HNt¾
TYHalKbOHEuǘtL î î TMXZNeUhR`HT'HNKTMHalKbWHTpw[bOXZJMTXZN¢[¢c
   ¬«  i   4 "!
 " ¬«  i  i   4  "!  4  "! 4   !
i "! !

E® ¯°ˆFŒF‰‹ŒFÜÁŸÁ;—;ÊË^a$XoN¡UYJYHÌ\PHTBJMHb€[qUVLOXoNKT\¼ £¤[oLWUYHT“]KNRKHTYTMLONöѽ%pÏIK]KLWTŸÍZ^N`^JV[bOLOTMHÌ\‚SbO[<JY^]KNKLOXoN
RKHš-HNKTMHalKbWHT .ï%îðððî hñtu
®E¯°ˆFŒF‰‹ŒF Á“È*— &<XZalKLWHN  [q¾óUM¾‹LWbwR`HENKXoalKJYHT'‚a_XZLWNKTRKH¿¡]f[UYJYH.Pª©KL  JYHT¼{RKGH
‚ #
# # #e½
¿¡]KLŸNKHTYXZNeUhRKLWxnLWTYLOl`bOHT<NKL“If[oJ'â`p`NKLŸIf[oJ tpfN`L“If[oJ \À¼ ('^IwXoNKTYH_c  nkZu ½
 

®E¯°ˆFŒF‰‹ŒF ÁQ×=—7¶hN P]`lwH¢á
%¨á
%¨á
HTMURKLxnLOTY^H N 7
7

ÜP]KlQHT]`NKLWUY^Tu\vN ^PJMLWU]`N
NKXZal`JYHdRf[NKTPª©f[o¿¡]KHP]KlQH]KNKLWUY^outÊd[oNKTSPª©K[o¿¡]KHbOLWÍZNKH'HUSRf[NKTPª©f[o¿¡]KHPXZbOXoNKNKHËR`H.á
IQHUVLUVT
P]KlQHTp!If[JV[obWbOÃbWH‚*]KN`HRKHT[oJM·UVHTRK]ÎP]KlQHoprb€[¢TYXZa_a$HRKHTNKXoalKJYHTE£¤[oLU_kZurÊd[oNKT]`NÎRKHT
IwH%UVLUVTP]KlwHTpfbWHdNKXZal`JYH^PJYLUSHTÅUk
`uK³/[oJSPHIQHUYLWUSP]KlQHIf[oTYTMHNeU#UVJMXZLOTPXZ]KPª©KHTk %á
%á

If[oJY[obObWÃbOHT[o]ts3£¤[oPHTR`]¦P]KlQHou rJYXZ]`xoHJEb€[*TMXZa_a$HRKH_UVXZ]KT.bOHTNKXoalKJYHTEHNt¾‹R`H©KXZJMTRKH$PHT

UVJMXZLOTSPXZ]KPª©KHTu

D6!ä % 8%@Bç
$]`HbObWHS¿¡]KH#TYXZLU\b€[dTMLWUV]K[UVLWXZN“poXoN_ÍZ[oÍZNKHUYXZ]qmMXZ]KJMT‚dUVHTÅUVHJjb„ƒ…^NKXZNKP^STY]KJRKHSIQHUYLWUYHT\x[obOH]KJYT
RKHT/If[oJY[oa$Ã%UVJMHTpq‚S£¤[oLOJMHRKHTrHTMTV[oLWTu &<Hb€[#IQH]`U/RKXZNKN`HJr]KNKH<LWRK^HR“ƒµ]KNKHj£{XZJYa]KbOH\RKHjJY^P]KJMJYHNKPHp
XZ]3lKLOHN§IQHJMa$H%UYUVJMHRKH$R`LOTMUYLONKÍo]KHJPH¿¡]KLRKL  ^JYHN`PLOHbWHTP[oT.¿¡]KLa[JYPª©KHN¡UERKHTEP[oTE¿¡]KLNKH
a[JYPª©KHN¡U'If[Tu (!stIw^JYLOa_HNeUYHÌFö

º
Stage olympique de Saint-Malo

Cours  Stratégies de base


Lundi 28 juillet 2003

par

Xavier Caruso

Table des matières


1 Les tiroirs 2
1.1 Le principe . . . . . . . . . . . . . . . . . . . . . . . . . . . . . . . . . . . . 2
1.2 Plusieurs façons d'utiliser ce principe . . . . . . . . . . . . . . . . . . . . . . 3

2 Les invariants 5
2.1 La situation . . . . . . . . . . . . . . . . . . . . . . . . . . . . . . . . . . . . 5
2.2 Invariant de parité . . . . . . . . . . . . . . . . . . . . . . . . . . . . . . . . 6
2.3 Invariants et coloriage . . . . . . . . . . . . . . . . . . . . . . . . . . . . . . 7
2.4 Colorier avec plusieurs couleurs . . . . . . . . . . . . . . . . . . . . . . . . . 8

3 Le raisonnement par l'absurde 10


3.1 La situation . . . √. . . . . . . . . . . . . . . . . . . . . . . . . . . . . . . . . 10
3.2 L'irrationalité de 2 . . . . . . . . . . . . . . . . . . . . . . . . . . . . . . . 11

4 Le raisonnement par récurrence 12


4.1 Le principe . . . . . . . . . . . . . . . . . . . . . . . . . . . . . . . . . . . . 12
4.2 Notre premier exemple . . . . . . . . . . . . . . . . . . . . . . . . . . . . . . 13
4.3 Digression sur l'intérêt des formules . . . . . . . . . . . . . . . . . . . . . . . 14

5 Récurrence et suites 16
5.1 Les suites arithmético-géométriques . . . . . . . . . . . . . . . . . . . . . . . 16
5.2 Les suites homographiques . . . . . . . . . . . . . . . . . . . . . . . . . . . . 18
5.3 La suite de Fibonacci . . . . . . . . . . . . . . . . . . . . . . . . . . . . . . . 19
5.4 Le triangle de Pascal . . . . . . . . . . . . . . . . . . . . . . . . . . . . . . . 20
5.5 L'opération  eXclusive OR  . . . . . . . . . . . . . . . . . . . . . . . . . . 22
5.6 Digression sur l'intérêt de la base 2 . . . . . . . . . . . . . . . . . . . . . . . 26
5.7 Parité des coecients binômiaux . . . . . . . . . . . . . . . . . . . . . . . . 28

6 Constructions 32
6.1 Nombres univers et nombres normaux . . . . . . . . . . . . . . . . . . . . . 32
6.2 Nombres rationnels et périodicité . . . . . . . . . . . . . . . . . . . . . . . . 33
6.3 Une fonction pour le moins étrange . . . . . . . . . . . . . . . . . . . . . . . 34
6.4 Le principe du va-et-vient . . . . . . . . . . . . . . . . . . . . . . . . . . . . 36

1
Ce document n'est pas à proprement parler un cours, il s'agit plus d'un recueil de mé-
thodes et d'exemples. Loin d'être exhaustif, le choix des sujets traités reprend évidemment
les fameux incontournables (par exemple la récurrence) mais essaie également d'insister sur
d'autres points, disons plus  originaux 1 , ceci, quelque part, dans le but de ne pas trop
faire redite avec le cours du stage de l'an dernier.
Ce document, bien que sans doute self-contained 2 , gagnera à être complété par un
vrai cours de  stratégies de base . Il est possible d'en trouver par exemple sur la page
d'Animath : http://www.animath.fr/.

1 Les tiroirs
1.1 Le principe
Il s'agit d'une idée fort simple et fort naturelle mais dont les conséquences sont tout
à fait impressionnantes. La situation est la suivante : supposons que l'on ait à ranger 23
chaussettes3 et que l'on dispose pour cela de 5 tiroirs. Il y aura alors forcément au moins
5 chaussettes dans un tiroir. En eet, s'il y avait au plus 4 chaussettes par tiroir, il n'y
aurait pas plus de 20 chaussettes en tout.
Évidemment, la formulation générale n'est pas aussi précise, mais il n'y a de fait rien
de plus à comprendre.

Propriété 1
n
Si n balles sont placées dans k tiroirs, au moins un tiroir contiendra k balles ou plus.

Déjà, il faut dire que si nk n'est pas un entier, contenir au moins nk balles voudra dire
§ ¨en
contenir au moins le premier entier supérieur à nk , entier que l'on notera par la suite nk .

Ce n'est pas tant la propriété précédente qui est intéressante en elle, mais plutôt les
conséquences qu'elle peut avoir et la façon dont on l'utilise pour prouver ces conséquences.
Une première conséquence simple est de prouver qu'il y a au moins deux parisiens qui
ont le même nombre de cheveux sur la tête. Évidemment, il faut des données numériques
pour traiter le problème : disons que la population de Paris est d'environ 20 millions
d'habitants et que les gens n'ont jamais plus d'un million de cheveux sur la tête. Dans ces
conditions, si l'on veut appliquer le principe tel qu'énoncé précédemment, il faut construire
de gigantesques tiroirs, tiroirs que l'on va numéroter avec les nombres compris entre 0 et
1 000 000. On va ensuite répartir les parisiens dans chacun de ces tiroirs, et ce en fonction
du nombre de cheveux qu'il possède. Comme il y a strictement plus de parisiens que de
tiroirs, au moins un tiroir contiendra deux parisiens, et ces deux bons hommes auront donc
le même nombre de cheveux sur le crane.
Évidemment, on peut raner le résultat précédent. D'après le principe, et en reprenant
les
§20 données¨ précédentes (qui sont sûrement fausses) on pourra toujours un tiroir contenant
000 000
1 000 001 = 20 parisiens. Il y a donc au moins un groupe de 20 parisiens qui ont le même
1
Originaux pour un
stage olympique ; les sujets évoqués restent somme toute assez classiques.
2
Ne demandant quepeu des prérequis.
3
Oui, apparemment, elles ne vont pas toutes par deux.

2
nombre de cheveux. Après le club très fermé des  130 de Q.I. 4 , on peut former le club
encore plus fermé des  145 876 cheveux sur le crane .

1.2 Plusieurs façons d'utiliser ce principe


Dans un premier temps, il s'agit de dire que le principe des tiroirs apporte souvent
un secours inespéré lorsque l'on a un grand nombre d'entités du même objet et que l'on
cherche à en isoler certains qui auraient des propriétés sympathiques. L'exemple des pari-
siens chevelus est en ce sens frappant. La diculté dans ce cas sera de trouver quels seront
ces objets et comment les répartir judicieusement dans les tiroirs, l'énoncé ne suggérant
pas toujours si fortement la solution que dans l'exemple précédent.
Il est à noter que cette situation apparaît assez souvent en arithmétique, l'exemple de
base étant le suivant. On se donne un entier n, et n + 1 entiers a0 , . . . , an . Il faut prouver
qu'il existe deux indices distincts i et j tels que ai − aj soit un multiple de n.
On procède comme suit. On considère donc n tiroirs que l'on numérote avec les nombres
compris entre 0 et n − 1. Maintenant, on va placer les ai dans ces tiroirs. Plus précisément,
pour tout indice i, on calcule le reste de la division de ai par n, et si l'on appelle r ce reste,
on place le nombre ai dans le tiroir étiqueté r. D'après le principe des tiroirs, il est bien
clair qu'ainsi au moins deux nombres seront placés dans le même tiroir. Cela signiera qu'il
existe un entier r et deux indices distincts i et j tels que :

ai = qi n + r
aj = qj n + r

les nombres qi et qj étant alors les quotients des divisions faites. Mais on fait alors la
diérence des deux égalités données ci-dessus et on remarque que ai − aj est un multiple
de n, les restes r se simpliant. Cela répond donc à la question.
Donnons nalement sous forme d'exercice deux nouveaux exemples illustrant de nou-
velles applications de cette situation. Un peu comme dans le cas précédent, ici, la diculté
consiste à considérer les bons tiroirs.

Exercice : Sur une table rectangulaire de dimension 2m × 1m sont réparties 500 miettes
de pain. Prouver que l'on peut trouver trois miettes qui déterminent un triangle d'aire
inférieure à 50cm2 .

Solution :
I Apparemment, on a déjà trouvé nos objets que l'on va devoir ranger dans nos tiroirs ; il
s'agit des miettes de pain. Il faut donc encore trouver les tiroirs.
L'astuce, ici, consiste à découper la table en 200 petits carrés de côté 10cm. D'après le
principe des tiroirs, dans au moins un de ces carrés, il y aura trois miettes, et ces miettes
vont déterminer un triangle dont l'aire sera inférieure à la moitié de la surface du carré en
question. Après calcul, on aboutit bien à 50cm2 . J

Exercice (OIM 1984) : On fait une partition de l'ensemble des points du plan orienté en
un nombre ni de parties représentées par autant de couleurs. On xe deux points distincts
O et A de ce plan. À tout point X du plan, distinct de O, on fait correspondre :
\
a) la mesure en radians α (X) de l'angle (OA, OX) prise dans [0, 2π[ ;
4
cf. un sketch de Desproges.

3
b) le cercle C (X) de centre O et de rayon OX + α(X)
OX
Démontrer qu'il existe un point Y du plan avec α (Y ) > 0, tel qu'il existe un point de C (Y )
de la même couleur que Y .

Solution :
I Comme on l'a dit la diculté ici consiste à trouver à quels objets appliquer le principe
des tiroirs, car avec un peu d'entrainement il est passablement clair que c'est ce principe
qu'il va falloir utiliser.
Il ne faut en fait ici pas se focaliser sur les points, mais plutôt sur les cercles de centre
0. À tout cercle, on peut associer l'ensemble des couleurs qui apparaîssent sur ce cercle ;
ils sont là nos tiroirs. Un dénombrement simple nous dit qu'il y a 2n − 1 tiroirs, il nous
sura donc de choisir 2n cercles. En fait, si l'on rééchit un peu au problème, on se rend
facilement compte qu'il faudra choisir ces cercles relativement proches les uns des autres. √
On choisit donc au nal 2n cercles parmi ceux de rayons strictement inférieurs à 2π .
Parmi ces cercles, il va y en avoir deux qui auront le même ensemble de couleurs associé.
Il s'agit maintenant de jouer sur l'angle. Plus précisément si R1 et R2 sont les rayons
des deux cercles choisis, avec par exemple R1 < R2 , il faut trouver Y sur R1 tel que α (Y )
vérie l'équation suivante :
α (Y )
R2 = R1 +
R1
équation dont on vérie facilement qu'elle admet une solution dans l'intervalle ]0, 2π[.
Le cercle C (Y ) est alors le cercle de centre O et de rayon R2 ; la conclusion s'ensuit. J

Toutefois, ce n'est pas le seul cas dans lequel il peut être utile ; il apparaît souvent
au détour d'un raisonnement et c'est alors encore moins évident de le mettre en évidence.
Pour tout commentaire, nous proposons l'exercice suivant :

Exercice (OIM 1997) : Une matrice carrée à n lignes et n colonnes, à éléments dans
l'ensemble S = {1, 2, . . . , 2n − 1}, est appelée une matrice d'argent si, pour tout i =
1, . . . , n, la réunion de la i-ième ligne et de la i-ième colonne contient tous les éléments de
S . Montrer qu'il n'existe pas de matrice d'argent pour n = 1997.

Solution :
I On reconnaît, ici, bien évidemment la situation donnée dans le test de bienvenue5 .
Donnons-nous un entier a compris entre 1 et 2n − 1. Si cet entier apparaît à la position
(i, j), alors il apparaît à la fois dans la i-ième croix6 et dans la j -ième croix. Comme
maintenant a doit apparaître une et une seule fois dans chaque croix, on peut faire la chose
suivante : on écrit les uns à la suite des autres les nombres de 1 et 1997, et on barre au
fur et à mesure les numéros des croix dans lesquelles a apparaît. À la n, toutes les croix
devront être barrées.
On se rend compte que si a n'apparaît pas sur la diagonale, on va barrer les nombres
deux par deux. Mais cela n'est pas possible puisque 1997 est un nombre impair. Il reste
donc à prouver qu'il existe un entier a qui n'apparaît pas sur la diagonale.
C'est la qu'intervient le principe des tiroirs. Il y a 3993 nombres en tout et seulement
1997 places sur la diagonale ; il y a donc au moins un nombre qui ne peut pas apparaître
(en fait, il y en a au moins 1996, mais bon). J

5
Cela
vous touche, je le sais !
6
On appelle i-ième croix la réunion de la i-ième ligne et de la i-ième colonne.

4
2 Les invariants
2.1 La situation
Supposons que l'on ait un gros ensemble regroupant un certain nombre de congu-
rations. Supposons en outre que l'on se soit dicté des règles permettant de passer d'une
conguration à une autre. Ainsi on va regrouper entre elles les congurations qui peuvent
être atteintes l'une de l'autre par application de ces règles.
Schématiquement, on a la chose suivante :

La grosse patate représente l'ensemble des congurations et on a regroupé dans les petites
patates les congurations que l'on pouvait déduire l'une de l'autre par les transformations
autorisées7 .

La situation que l'on va étudier est la suivante : on se donne deux congurations et on


se demande si elles sont ou non dans la même patate, c'est-à-dire s'il y a moyen de passer
de l'une à l'autre par une suite d'opérations autorisées.
Nous allons peut-être d'ores et déjà donner un exemple qui va clarier les choses.
L'ensemble des congurations va être ici l'ensemble des mots (qui ont un sens ou pas)
écrits avec les seules lettres x, y , z et t. On s'autorise les trois transformations suivantes :
i) xy → yyx et yyx → xy
ii) xt → ttx et ttx → xt
iii) yt → ty et ty → yt
La première condition signie par exemple que lorsque l'on a un mot dans lequel apparaît
les deux lettres x et y juste à côté, alors on s'autorise à remplacer ceux deux lettres par
les trois lettres y , y et x. On s'autorise également à revenir en arrière ; c'est la deuxième
condition du i).
Ainsi les mots xxyy et xyyyyx vont être equivalents, grâce à la suite de transformations
suivantes :
xxyy = x xy y → x yyx y = xyy xy → xyy yyx = xyyyyx
Dans le schéma précédent, ces deux-mots là appartiendraient donc à la même petite patate.
On voit que pour prouver que deux mots sont dans la même petite patate, il  sut  d'ex-
hiber une suite de transformations permettant de passer de l'un à l'autre. Mais comment
prouver que deux mots n'appartiennent pas à la même petite patate ? C'est là qu'intervient
la théorie des invariants.
7
Les transformations sont supposées symétriques, ce qui fait que les petites patates seront supposées
disjointes.

5
De façon générale, l'idée consiste à associer à chaque conguration un objet (générale-
ment un entier, ou une propriété) que l'on va appeler son invariant. Cet invariant devra
au moins avoir les deux propriétés sympathiques suivantes :
1. le calcul de l'invariant devra pouvoir se faire de manière simple et systématique
2. deux congurations équivalentes (ie dans la même patate) devront avoir même inva-
riant
Ainsi, pour prouver que deux congurations ne sont pas équivalentes, on calcule les
invariants associés à chacune d'elles : si ces invariants sont diérents, on peut conclure.
Attention, on ne peut pas conclure si les invariants sont égaux !
On voit maintenant qu'une troisième propriété qui serait sympathique pour un inva-
riant serait de prendre des valeurs assez diversiées : un invariant qui a toute position
associe par exemple le nombre 0 est certes facile à calculer, mais ne donnera au nal que
peu d'informations. Au mieux un invariant permet de distinguer deux ensembles de con-
gurations, au plus on dira qu'il est n. Construire des invariants ns est en général un
problème dicile ; toutefois, on verra par la suite que souvent des invariants très grossiers
permettent d'arriver à des résultats déjà intéressants.

Revenons à notre exemple et posons-nous la question suivante : les mots xytx et txyt
sont-ils équivalents ? Pour résoudre cette question, on remarque que les transformations
permises ne modient jamais le nombre d'occurrences de la lettre x dans le mot. Si l'on
veut reprendre le langage introduit précédent, on dira que si l'on associe à un mot le nombre
d'apparitions de la lettre x dans ce mot, on obtient un invariant.
Maintenant, l'invariant du mot xytx vaut 2 et celui du mot txyt vaut 1 ; cela prouve
que ces mots ne sont pas équivalents.
L'invariant que l'on vient de construire permet de distinguer quantité de mots, mais il
est incapable de donner une réponse pour les mots xy et xt. En fait, on peut prouver que
ces deux mots ne sont pas équivalents. Voyez-vous comment ?

2.2 Invariant de parité


Un invariant a priori grossier, mais qui permet souvent d'arriver au résultat, est ce que
l'on appelle un invariant de parité : à une conguration donnée, on va associer soit pair,
soit impair, le problème étant bien sûr souvent de savoir ce qu'il faut compter.

Donnons un exemple qui illustre cela. Supposons que l'on dispose d'une table carrée
sur laquelle sont disposées 64 ampoules dans un carré 8 × 8. Au bout de chaque ligne et
de chaque colonne, il y a un interrupteur. Lorsque celui-ci est actionné, il inverse l'état8 de
chacune des ampoules de la ligne ou de la colonne à laquelle il correspond.
Au début, toutes les ampoules sont éteintes. Est-il possible d'arriver dans la congura-
tion dans laquelle seule l'ampoule marquée est allumée (voir dessin page suivante) ?
Pour répondre à cette question on utilise un invariant de parité : on constate que
lorsque l'on appuie sur un interrupteur la parité du nombre d'ampoules allumés ne change
pas. En eet, si avant sur la ligne ou la colonne aectée par l'opération, il y avait a
ampoules allumées et b ampoules éteintes, il y aura après l'opération b ampoules allumées
et a ampoules éteintes. Ce qu'il faut voir c'est que a et b sont forcément de même parité
puisque a + b = 8.
8
Cela signie que si l'ampoule était allumée, elle s'eteint et si elle était éteinte, elle s'allume.

6
Formellement, on associe à une conguration l'invariant pair si le nombre d'ampoules
allumées est pair et l'invariant impair sinon. On vient de voir que cet invariant n'est bien pas
modié lors d'une transformation autorisée. Au début, toutes les ampoules sont éteintes,
l'imvariant est donc pair. À la n, on souhaite qu'il n'y ait qu'une ampoule allumée et donc
un invariant impair. C'est impossible !

2.3 Invariants et coloriage


Le problème à résoudre ce coup-ci est le suivant. On considère le plateau de jeu repré-
senté ci-dessous :

jeu que l'on souhaite paver avec des dominos de la forme suivante : , dominos que
l'on peut disposer soit horizontalement, soit verticalement.

En fait, cela est impossible et la méthode pour prouver ce fait est la suivante. On
commence par colorier les cases du plateau de jeu de la façon suivante :

On remarque alors que si l'on pose un domino sur le plateau de jeu, il recouvrira
forcément une case blanche et une case noire. On conclut en comptant les cases : il y a 24
cases noires et 22 cases blanches seulement. Un pavage est par le fait impossible.

Cette dernière démonstration est en fait une illustration de la théorie des invariants.
Bien que cela ne soit pas fondamental, nous allons expliquer en quoi.
L'ensemble des congurations sera l'ensemble des ensembles de cases. Cela signie
qu'une conguration sera le choix d'un certain nombre de cases, ces cases étant choisies

7
de façon totalement quelconque ; en particulier, ce choix ne correspond pas forcément à un
pavage par des dominos.
Les transformations autorisées seront celles qui consistent à enlever ou à ajouter deux
cases adjacentes, correspondant donc à l'enlèvement ou à l'ajout d'un domino. Il faut
maintenant dénir l'invariant : étant donné une conguration (ie un choix de certaines
cases), on compte le nombre de cases noires et de cases blanches parmi les cases choisies et
on soustrait ces deux nombres, le résultat pouvant être positif ou négatif.
Il est clair que l'on dénit ainsi un invariant : rajouter un domino rajoute à la fois une
case blanche et une case noire et donc ne modie pas la diérence ; enlever un domino
supprime à la fois une case noire et une case blanche et donc ne modie pas la diérence
non plus. Maintenant l'invariant associé à la position d'origine (celle où l'on choisit toutes
les cases) est 24 − 22 = 2. Pour la position d'arrivée par contre (celle où l'on ne choisit
aucune case), c'est 0.
Cela prouve donc l'impossibilité.

2.4 Colorier avec plusieurs couleurs


Lorsque l'on ne pave plus avec des dominos mais avec des pièces plus grandes ou plus
diormes, il peut parfois être utile d'utiliser un coloriage plus perfectionné. Le premier
exemple à traiter est probablement le suivant.
On considère un rectangle de dimension a × b que l'on veut paver avec des n-ominos
de taille 1 × n. On se demande à quelle condition portant sur les dimensions du rectangle
a et b, un tel pavage est réalisable.
On colorie comme précédemment les petites cases du rectangle, mais ce coup-ci en
utilisant n couleurs, l'idée étant toujours la même : lorsque l'on va poser un n-omino, il va
recouvrir une case de chacune des couleurs. Ainsi, s'il n'y a pas autant de cases de chaque
couleur, le pavage ne sera pas possible. Pour cela, on commence par ordonner de façon
arbitraire les couleurs : il y donc la première, la deuxième, etc. Sur la première ligne, on
dispose les couleurs dans l'ordre et on recommence lorsque l'on a épuisé notre palette. On
fait de même sur la seconde ligne sauf que l'on procède à un décalage d'une couleur (ie on
commence à la deuxième couleur). On continue et termine alors de la même façon, décalant
d'une couleur (toujours dans le même sens) à chaque nouvelle ligne.
Essayons donc de compter le nombre de cases de chaque couleur. Déjà, on remarque
que tout rectangle de taille n × x portera exactement x cases de chaque couleur. Ainsi
pour faire notre décompte, on peut commencer par retirer deux rectangles de sorte que
les dimensions a et b soient toutes les deux strictement plus petites que n. Quitte à faire
pivoter le rectangle obtenu, on peut supposer en outre que a > b.

multiple de n

b
multiple de n a

8
La a-ième couleur est celle qui apparaît le plus à droite sur la première ligne du petit
rectangle en bas à droite. Cette couleur apparaît donc sur toutes les lignes. Par contre la n-
ième couleur, elle, ne peut évidemment apparaître plus d'une fois par ligne, mais n'apparaît
pas non plus sur la première. Finalement, elle apparaît moins que la a-ième et le rectangle
n'est pas pavable. Bien évidemment le raisonnement précédent ne tient pas si a ou b est
nul ; dans ces cas, il n'y a plus de rectangle en bas.
On vient ainsi de donner une première réponse à la question que l'on s'était posée : si
une des dimensions a ou b n'est pas un multiple de n, alors le rectangle n'est pas pavable.
D'autre part, il est clair que si l'une des dimensions est un multiple de n alors le rectangle
est pavable. On vient donc de répondre totalement à la question.

Là encore, ce problème que l'on vient de traiter avec des coloriages peut être vu comme
une application de la théorie des invariants. Comme dans le cas du paragraphe précédent,
une conguration sera le choix d'un certain nombre de cases de notre rectangle a × b. Les n
couleurs vont ce coup-ci être remplacées par des nombres x1 , . . . , xn vériant la condition
x1 + . . . + xn = 0.
L'invariant associé à une conguration sera la somme des nombres associés aux cases
retenues pour la conguration en question. Lorsque l'on passe d'une conguration à une
autre en ajoutant n cases alignées horizontalement ou verticalement, on ne change pas
l'invariant, justement en vertu de la condition x1 + . . . + xn = 0.
Certains choix de valeurs pour les xk simplient grandement les calculs, le plus simple
étant probablement de prendre :
µ ¶
2i (k − 1) π
xk = exp
n

i étant le  −1  des nombres complexes et exp désignant l'exponentielle complexe9 .
Dans ces conditions, un simple calcul10 permet de déterminer l'invariant du rectangle
a × b. On trouve : £ ¡ ¢ ¤£ ¡ ¢ ¤
exp 2iaπ
n − 1 exp 2ibπ
n −1
£ ¡ ¢ ¤2
exp 2iπ
n −1
et l'on sait que si ce nombre n'est pas nul, alors le rectangle n'est pas pavable. Or un
produit de facteurs est nul si et seulement si un des facteurs est nul et les exponentielles
égalent 1 si et seulement si leur argument est un multiple de 2iπ . Cela permet d'arriver de
même que précédemment à la conclusion.

Il est remarquable de noter que cette dernière méthode se généralise directement au cas
continu. Le problème est alors le suivant. Les nombres a et b sont cette fois-ci des réels et
on veut paver un rectangle de taille a × b par des lattes de dimensions 1 × x, la valeur de
x pouvant varier d'une latte à l'autre.
Une conguration sera alors une partie (mesurable) A du rectangle et l'invariant asso-
ciée sera : Z
exp (2i (x + y) π) dxdy
A
Comme précédemment, on montre que si cette quantité est non nulle, alors le rectangle
n'est pas pavable. De cela on déduit que le rectangle est pavable si et seulement si une des
dimensions a ou b est un nombre entier.
9
Le lecteur qui n'est pas familier avec ces notions peut passer directement au chapitre suivant.
10
Si l'on connait la formule de sommation d'une série géométrique.

9
3 Le raisonnement par l'absurde
3.1 La situation
Supposons que l'on ait à démontrer une certaine phrase mathématique. Une façon
d'aborder le problème est de commencer par supposer que cette phrase est fausse. On
regarde ensuite ce qui découle de cette nouvelle hypothèse, le but étant de parvenir à une
contradiction. Si l'on y arrive, cela voudra dire que notre supposition de départ ne pouvait
être valable et ainsi on aura bien démontré notre propriété.
La grande force du raisonnement par l'absurde est d'introduire une hypothèse supplé-
mentaire, ce qui est fort utile lorsque l'on n'a rien ou pas grand-chose pour partir. En outre,
le raisonnement par l'absurde est particulièrement ecace lorsqu'il s'agit de montrer une
propriété négative ( Montrer que telle chose n'a pas telle propriété ). Dans ce cas, on
suppose que cette chose a la propriété en question et on regarde ce qu'il en découle.

Le raisonnement par l'absurde se révèle à la fois ecace et naturel pour tout ce qui fait
partie des problèmes de logique grand public11 , dirions-nous. En voici un exemple :
Le gentil héros se retrouve face à la confrontation nale. Il est dans une salle au fond
de laquelle se trouvent trois portes donnant sur trois prisons gardées par trois vaillants
logiciens. Le héros s'approche et les logiciens parlent tour à tour :
Le gardien de la première porte dit :  Derrière ma porte, il y a la princesse .
Le gardien de la deuxième porte dit :  Il y a un et un seul menteur parmi nous et
derrière ma porte, il y a la princesse .
Le gardien de la troisième porte dit :  Nous sommes tous des menteurs .
La question est bien entendu de savoir où se cache la princesse, sachant que parmi les
trois gardiens, certains disent toujours la vérité et les autres mentent toujours.

N'ayant que peu d'informations au début, pour résoudre ces questions, on est souvent
amené à faire des hypothèses successives et à les tester. C'est exactement le principe du
raisonnement par l'absude.
Commençons par analyser la phrase prononcée par le troisième gardien. Supposons que
ce gardien dise toujours la vérité. Dans ce cas, ce serait un menteur, comme il le dit. C'est
absurde ! Le troisième gardien est donc un menteur et on sait en outre maintenant qu'au
moins un des deux autres gardiens a dit la vérité. Une autre façon de voir les choses est de
dire que le troisième gardien se contredit lui-même ; c'est donc forcément un menteur.
Maintenant que l'on sait cela, focalisons-nous sur le deuxième gardien et supposons
qu'il dise la vérité. Alors, dans un premier temps, il y aurait un et un seul menteur parmi
les trois gardiens, comme il le dit. Mais on l'a déjà trouvé ce menteur, c'est le troisième.
Cela voudrait donc dire que les deux premiers gardiens ont dit la vérité. Mais cela n'est
pas possible puisque chacun arme que la princesse se trouve dans la prison qu'il garde, et
qu'évidemment il n'y a qu'une princesse. Encore, on arrive à une contradiction et le second
gardien est aussi un menteur.
Dès lors, le premier dit forcément la vérité puisque l'on sait qu'ils ne sont pas tous les
trois des menteurs : la princesse est donc retenue dans la première cellule !
11
Le démineur en est encore un exemple : si vous jouez couramment à ce jeu, vous faites sans doute
nombre de démonstrations par l'absurde sans même le savoir.

10
Bien sûr, dans ce cas simple, une étude exhaustive fonctionnera tout aussi bien : il
y a 8 possibilités en tout, chaque gardien pouvant soit mentir soit dire la vérité. Il faut
alors éliminer les cas un par un quand ceux-ci deviennent contradictoires. C'est encore
exactement une application du raisonnement par l'absurde, mais de façon moins subtile
que celle présentée juste avant.

3.2 L'irrationalité de 2
Un nombre rationnel est un nombre qui peut s'écrire
√ comme le quotient de deux entiers,
p
donc une fraction q . On souhaite ici prouver que 2, donc le nombre qui multiplié par lui-
même fait 2, n'est pas rationnel.
C'est un exemple typique où l'on souhaite obtenir une propriété  négative , et donc
un raisonnement par l'absurde va nous permettre de pouvoir partir. On suppose donc, sans
pitié, qu'il existe des entiers a et b tels que :
√ a
2=
b
le dénominateur b étant non nul. Il s'agit maintenant de trifouiller tout cela pour aboutir
à une contradiction. √
La seule information dont on dispose sur le nombre 2 porte sur son carré ; il est
donc naturel d'élever l'égalité précédente au carré. On obtient ainsi après avoir chassé les
dénominateurs a2 = 2b2 . Il s'agit maintenant de comprendre pourquoi cette équation ne
peut pas avoir de solutions, a et b devant être des entiers.
Pour cela, il faut se rappeler un peu d'arithmétique et principalement la décomposition
en facteurs premiers, propriété que nous rappelons ci-dessous.

Propriété 2

Soit n > 2 un entier naturel. Alors il existe des nombres premiers p1 , . . . , pk deux à deux
distincts et des entiers strictement positifs α1 , . . . , αk , le tout tel que :

n = pα1 1 . . . pαk k

En outre cette écriture est unique à l'ordre d'écriture près.

On rappelle que par dénition, un nombre premier est un nombre qui n'admet pas de
diviseurs propres, c'est-à-dire qui n'est divisible que par 1 et lui-même. On rappelle en
outre que par convention, 1 n'est pas considéré comme étant premier.
Il est une façon, peut-être plus agréable pour certains, de réénoncer la propriété précé-
dente :

Propriété 3

Soit n un entier naturel non nul. Alors il existe une unique suite d'entiers positifs ou nuls
(αp ) indexée par les nombres premiers et telle que :
1. l'entier αp est nul pour p susamment grand
Y
2. n = pαp
p premier

11
L'entier αp s'appelle la valuation p-adique de n ; il est souvent noté vp (n).

Q
Il est sans doute nécessaire de faire quelques remarques. Déjà le signe   signie
que l'on fait le produit des tous les termes pαp lorsque p parcourt l'ensemble des nombres
premiers. La première condition assure que ce produit est en fait ni : p0 = 1 pour tout
entier non nul p et multiplier par 1 ne modie pas le résultat.
Finalement, la valuation p-adique de l'entier n peut se dénir directement. Il s'agit du
plus grand entier αp tel que pαp divise n. On voit alors que pour p susamment grand (par
exemple p > n), αp va être nul.
Une dernière remarque, facile à prouver et laissée au lecteur, dit que si a et b sont des
entiers strictement positifs, alors :

vp (ab) = vp (a) + vp (b)

Revenons à nos moutons. On rappelle que l'on était arrivé à l'équation a2 = 2b2 et qu'il
s'agissait de trouver une absurdité. L'idée consiste donc à passer aux valuations 2-adiques ;
on obtient, en vertu de la remarque précédente :

2vp (a) = 1 + 2vp (b)

mais cela est absurde car le membre de gauche de l'égalité précédente est manifestement
un nombre pair, alors que celui de droite est manifestement impair. Ils ne peuvent donc
pas être égaux12 .

On déduit de cela que notre hypothèse de départ ne pouvait être vraie : 2 est bien
un nombre irrationnel.

4 Le raisonnement par récurrence


En guise d'introduction, on se propose de démontrer la partie  existence  de la pro-
priété 2. On prend donc un entier n strictement positif et on souhaite écrire n comme un
produit de nombres premiers.
Il y a deux cas à distinguer : soit n est déjà premier, soit il ne l'est pas. Dans le premier
cas, il n'y a rien à faire : n est déjà écrit comme un produit de nombres premiers (un seul
nombre en l'occurrence).
Maintenant si n n'est pas premier, c'est qu'il existe des entiers a et b strictement plus
petits que n tels que n = ab. C'est le premier pas de notre décomposition et on n'a plus
qu'à continuer ainsi. Si a est premier, c'est très bien, sinon on l'écrit comme produit de
deux nombres et ainsi de suite. Bien sûr on fait pareil avec b.

4.1 Le principe
Voyons comment l'on peut écrire proprement le raisonnement précédent. L'idée consiste
à prouver la propriété 2 pour les entiers les uns après les autres. On sait faire pour 2, c'est
12
Le lecteur un peu embrouillé par l'introduction des valuations pourra décomposer a et b en facteurs
premiers et remplacer dans l'égalité a2 = 2b2 , a et b par leur décomposition respective. Il ne restera plus
alors qu'à comparer les exposants de 2 qui interviennent pour aboutir à la même contradiction.

12
déjà un nombre premier. On peut débuter à 1 si l'on préfère ; il faut alors se convaincre
que 1 s'écrit comme le produit d'aucun nombre premier.
Ensuite, on sait faire pour 3, c'est encore un nombre premier. 4 n'est pas premier mais
il s'écrit 2 × 2 et on sait dejà faire pour 2. Et on continue ainsi de suite.

Rigoureusement le  ainsi de suite  qui précède correspond au principe de récurrence


qui s'énonce comme suit :

Propriété 4

Considérons une famille d'énoncés mathématiques qui dépendent d'un paramètre entier n.
Notons Pn le n-ième énoncé13 .
Si d'une part, l'on sait démontrer P0 et que d'autre part, on arrive à prouver que Pn+1
est une conséquence des énoncés P0 , . . . , Pn , alors tous les énoncés Pn sont vrais.

Faisons tout de suite des remarques. Dans un premier temps, on n'est évidemment pas
obligé de débuter à 0, on peut commencer à n'importe quel entier : si l'on commence à
l'entier k , il faudra prouver que l'énoncé Pk est vrai et que pour tout n > k , l'énoncé Pn+1
est une conséquence des énoncés Pk , . . . , Pn .
Remarquons également que très souvent Pn+1 est simplement une conséquence de
l'énoncé Pn , voire des énoncés Pn et Pn−1 , mais pas vraiment de tous les précédents.
Bref.
Pour faire une démonstration par récurrence, il y a toujours deux étapes :
 L'initialisation qui consiste à prouver la propriété P0
 L'hérédité qui consiste à prouver que Pn+1 est conséquence des propriétés P0 , . . . , Pn

Dans un raisonnement classique par syllogismes, on tente de prouver un résultat général


d'un seul coup pour toutes les valeurs de n indépendamment. Ce qui fait la spécicité du
raisonnement par récurrence est de disposer d'une hypothèse supplémentaire ( l'hypothèse
de récurrence ) qui, lors de l'hérédité, nous donne une information utilisable sur l'entier
n sur lequel on raisonne. Si l'on est capable de conclure sans utiliser cette information,
c'est qu'en fait on n'a pas utilisé un raisonnement par récurrence. Inversement, cela nous
donne aussi une indication sur la méthode à suivre pour mener un tel raisonnement, ou
comment gérer l'hérédité : la clé consiste à se mettre en situation d'utiliser cette donnée
supplémentaire.

4.2 Notre premier exemple


Voyons comment cela fonctionne avec notre premier exemple. Comme on l'a déjà dit,
l'énoncé Pn va être  l'entier n peut s'écrire comme un produit de nombres premiers . On
ne commence ici pas à l'entier 0 mais plutôt à 1.
L'étape d'initialisation consiste à prouver P1 , c'est-à-dire que 1 peut s'écrire comme
un produit de nombres premiers. On a déjà dit que c'était le cas puisque 1 est le produit
d'aucun nombre premier. Encore une fois si cela ne vous plait pas, vous pouvez commencer
à 2.
13
Dans l'exemple précédent, Pn était donc l'énoncé :  l'entier n peut s'écrire comme un produit de
nombres premiers .

13
Considérons maintenant un entier n. On souhaite démontrer Pn+1 , mais en ayant le
droit de supposer P1 , . . . , Pn . Autrement dit, on sait déjà que tous les entiers inférieurs ou
égaux à n peuvent s'écrire comme produit de nombres premiers, et on veut montrer qu'il
en est de même de n + 1.
Si n + 1 est un nombre premier, alors on a gagné. Si par contre ce n'est pas le cas, il
existe des entiers a et b, tous les deux strictement inférieurs à n + 1 (et donc inférieurs
ou égaux à n) tels que n + 1 = ab. Mais pour a et b on sait faire ; on sait par hypothèse
de récurrence comme on dit, qu'il existe des nombres premiers a1 , . . . , ak et b1 , . . . , bl tels
que :
a = a1 . . . ak et b = b1 . . . bl
Mais alors, n + 1 = ab = a1 . . . ak b1 . . . bl et donc n + 1 s'écrit bien comme produit de
nombres premiers.
Tout cela permet de conclure.

Moralement il y a deux façons de voir la récurrence : soit en partant d' en haut , soit
en partant d' en bas . La première est celle que nous avons ébauchée dans l'introduction
du chapitre : on veut écrire n comme produit de nombres premiers, alors on commence par
écrire n comme produit de deux nombres et on continue jusqu'à n'obtenir que des nombres
premiers. Le problème avec cette stratégie est qu'il faut prouver que la suite d'opérations
prend nécessairement n14 . La méthode par  le bas , quant à elle, bien que parfois moins
naturelle a l'avantage d'être sans surprise et de permettre des rédactions plus simples et
souvent plus compréhensibles, ce qui nous ne le dirons jamais assez aide autant le lecteur
que l'élève.

4.3 Digression sur l'intérêt des formules


L'exercice à résoudre est maintenant le suivant : on se donne un réel non nul x tel que le
nombre x + x1 soit un entier. Il s'agit de montrer que pour tout entier n, le nombre xn + x1n
est également un entier.
Voyons ce qu'il se passe pour n = 2. On sait que x + x1 est un entier et on veut prouver
que x2 + x12 en est aussi un. Il nous faut donc trouver un moyen de relier ces deux nombres.
Mais on a la formule suivante :
µ ¶ µ ¶
1 2 2 1 2 1
x+ =x +2+ 2 = x + 2 +2
x x x

et on voit qu'elle permet de conclure directement : le nombre dont on veut voir qu'il est
entier s'écrit comme le carré d'un nombre entier auquel on a enlevé 2.
Comment faire maintenant pour n = 3 ? Ben de la même façon ; on utilise la formule :
µ ¶µ ¶ µ ¶ µ ¶
1 1 1 1 1 1
x2 + 2 x+ = x3 + x + + 3 = x + + x3 + 3
x x x x x x

Comme précédemment, on sait déjà que le membre de gauche de l'égalité précédente est
un entier. Il en est de même par hypothèse de la quantité x + x1 . On en déduit donc bien
ce que l'on veut.
14
Ce qui repose en général toujours sur le fait qu'il n'existe pas de suite strictement décroissante d'entiers,
principe d'ailleurs équivalent à l'énoncé de récurrence.

14
De façon générale, on utilise une récurrence pour prouver le résultat général. L'initiali-
sation correspond au cas n = 1 et est donnée par hypothèse. Supposons donc que chacun
des nombres x + x1 , . . . , xn + x1n soit un entier et essayons de prouver qu'il en est de même
1
de xn+1 + xn+1 . Pour cela, on utilise comme précédemment la formule :
µ ¶µ ¶ µ ¶ µ ¶
n 1 1 n+1 n−1 1 1 n+1 1 n−1 1
x + n x+ =x +x + n−1 + n+1 = x + n+1 + x + n−1
x x x x x x
et on conclut comme les autres fois.
On constate qu'ici nous n'avons utilisé l'hypothèse de récurrence que pour les rangs n
et n − 1. Il faut toutefois faire attention au fait que ces deux valeurs font bien partie de
celles pour lesquelles on a le droit de supposer quelque chose. Ce n'est d'ailleurs ici pas
le cas pour n + 1 = 2 : n − 1 vaut alors 0 et on a commencé notre récurrence à 1. Pour
terminer notre preuve, il faut donc traiter le cas n = 2 à part.

Souvent, des identités purement algébriques comme les précédentes résultent simple-
ment de formules qu'il s'agit de déterminer. Pour illustrer ce propos, nous donnons les deux
exercices suivants :

Exercice : On suppose que les entiers n et m s'écrivent tous deux comme somme de deux
carrés. Montrer qu'il en est de même du produit nm.

Solution :
I Les hypothèses nous disent qu'il existe quatre¡ entiers ¢a¡, b, c et¢d tels que n = a2 + b2
et m = c2 + d2 . Il s'agit donc d'écrire le produit a2 + b2 c2 + d2 comme une somme de
deux carrés et cela se fait  simplement  à l'aide de la formule suivante :
¡ 2 ¢¡ ¢
a + b2 c2 + d2 = (ac + bd)2 + (ad − bc)2

Remarquons qu'il existe des formules analogues pour les sommes de quatre et de huit
carrés. Elles sont :
¡ 2 ¢¡ ¢
a + b2 + c2 + d2 e2 + f 2 + g 2 + h2 =
2 2 2 2
(ae − bf − cg − dh) + (af + be + ch − dg) + (ce + ag − bh + df ) + (ah + de + bg − cf )
et
¡ ¢¡ ¢
x20 + x21 + x22 + x23 + x24 + x25 + x26 + x27 y02 + y12 + y22 + y32 + y42 + y52 + y62 + y72 =
2
(x0 y0 − x1 y1 − x2 y2 − x3 y3 − x4 y4 − x5 y5 − x6 y6 − x7 y7 )
2
+ (x0 y1 + x1 y0 + x2 y4 + x3 y7 − x4 y2 + x5 y6 − x6 y5 − x7 y3 )
2
+ (x0 y2 − x1 y4 + x2 y0 + x3 y5 + x4 y1 − x5 y3 + x6 y7 − x7 y6 )
2
+ (x0 y3 − x1 y7 − x2 y5 + x3 y0 + x4 y6 + x5 y2 − x6 y4 + x7 y1 )
2
+ (x0 y4 + x1 y2 − x2 y1 − x3 y6 + x4 y0 + x5 y7 + x6 y3 − x7 y5 )
2
+ (x0 y5 − x1 y6 + x2 y3 − x3 y2 − x4 y7 + x5 y0 + x6 y1 + x7 y4 )
2
+ (x0 y6 + x1 y5 − x2 y7 + x3 y4 − x4 y3 − x5 y1 + x6 y0 + x7 y2 )
2
+ (x0 y7 + x1 y3 + x2 y6 − x3 y1 + x4 y5 − x5 y4 − x6 y2 + x7 y0 )

Pour information 2, 4 ou 8 sont les seuls entiers pour lesquels on dispose de telles formules.
De la même façon, la formule suivante nous assure par exemple que tout rationnel
s'écrit comme la somme de trois cubes de nombres rationnels :
à !3 µ ¶3 à !3
r6 + 45r4 − 81r2 + 27 −r4 + 30r2 − 9 −6r3 + 18r
r= 2 + + 2
6r (r2 + 3) 6r (r2 + 3) (r2 + 3)

15
Exercice : Soient x, y et z trois nombres réels vériant x + y + z = 0 et x2 + y 2 + z 2 = 2003.
Calculer x4 + y 4 + z 4 .

Solution :
I Cela se fait directement à partir des trois formules suivantes :

(x + y + z)2 = x2 + y 2 + z 2 + 2 (xy + yz + xz)


(xy + yz + xz)2 = x2 y 2 + y 2 z 2 + x2 z 2 + 2xyz (x + y + z)
¡ 2 ¢2 ¡ ¢
x + y2 + z2 = x4 + y 4 + z 4 + 2 x2 y 2 + y 2 z 2 + x2 z 2

La première formule conduit à :


2003
xy + yz + xz = −
2
La seconde implique alors :

20032
x2 y 2 + y 2 z 2 + x2 z 2 =
4
et nalement :
20032 1
x4 + y 4 + z 4 = 20032 − 2 · = · 20032
4 2
Bien évidemment, le lecteur ayant une âme de frimeur pourra combiner les trois formules
précédentes pour n'en n'utiliser qu'une bien plus impressionnante. J

5 Récurrence et suites
Une suite récurrente est une suite dont le n-ième terme, un donc, est déni en fonction
des précédents : u0 , . . . , un−1 . On conçoit facilement que le principe de récurrence va être
particulièrement utile pour montrer de nombreuses propriétés sur de telles suites.
Bien que l'on puisse montrer de nombreuses sortes de propriétés diérentes, nous allons
nous cantonner par la suite à donner des formules explicites pour calculer un , la suite (un )
étant a priori dénie de façon récurrente.

5.1 Les suites arithmético-géométriques


On considère a et b deux réels. On suppose que la suite un vérie la relation de récurrence
un+1 = aun + b pour tout entier n > 1. Bien entendu, cette seule relation ne dénit par
complètement un , il reste encore à choisir une valeur pour u0 15 , mais disons simplement
que u0 vaut u, un certain réel xé à l'avance.
On se propose de prouver que pour tout entier n, le terme un est donné par la formule :
an − 1
u n = an u + b ·
a−1
15
Pour faire un parallèle avec le raisonnement par récurrence, on a donné ici l'équivalent de l'étape
d'hérédité, il reste à se soucier de l'initialisation.

16
On fait naturellement cela par récurrence. Pour n = 0, la formule donne u0 = u, ce
qui est vrai par hypothèse. Supposons maintenant cette formule établie pour tout entier
inférieur ou égal à n et prouvons-là pour l'entier n + 1. On a successivement :
µ ¶
an − 1
un+1 = aun + b = a an u + b · +b
a−1
µ n+1 ¶
n+1 a −a an+1 − 1
= a u+b + 1 = an+1 u + b ·
a−1 a−1
ce qui est bien ce que l'on désirait. On remarque en outre que dans ce cas, on a utilisé
l'hypothèse de récurrence seulement pour le rang n ; ceci est somme toute assez normal
puisque un+1 était déni seulement en fonction de un .

Avant de passer à la suite, faisons plusieurs remarques. Tout d'abord, on a peut-être déjà
remarqué que la formule démontrée est seulement valable dans le cas a 6= 1, n'ayant aucun
sens sinon. Toutefois, il doit être possible de donner une formule, diérente certes, pour le
cas a = 1. Dans ces conditions, la relation de récurrence devient simplement un+1 = un + b
et on peut imaginer directement qu'alors :
un = u0 + nb
Il faut sans doute souligner le fait que cette dernière formule est en fait bien un  cas
particulier  de la formule générale. On peut donner un sens très précis à l'armation
suivante, mais ce qu'il faut remarquer ici, c'est que lorsque a se rapproche de 1, la fraction
an −1
a−1 se rapproche, elle, de n, comme on le constate ecacement en regardant des valeurs
numériques. La formule générale redonne ainsi la formule précédente pour le cas particulier
a = 1.
Une dernière remarque pour nir ce paragraphe. En fait, l'étape dicile dans une
démonstration par récurrence n'est souvent ni l'initialisation, ni l'hérédité mais plutôt la
détermination exacte de la formule Pn que l'on va devoir manier. L'exemple précédent
aurait été bien plus délicat à traiter si la formule à prouver n'était pas donnée a priori, et
pourtant c'est souvent le cas et il faudra alors la deviner.
Voyons que même ici cela n'est pas insurmontable. Pour essayer de deviner quelque
chose, on commence toujours par voir ce qu'il se passe pour les premiers termes. Ici, on a :
u0 = u
u1 = au + b
u2 = a2 u + ab + b = a2 u + b (a + 1)
¡ ¢
u3 = a3 u + ab (a + 1) + b = a3 u + b a2 + a + 1
¡ ¢ ¡ ¢
u4 = a4 u + ab a2 + a + 1 + b = a4 u + b a3 + a2 + a + 1
Ainsi, on imagine sans trop de mal que la formule générale doit être :
¡ ¢
un = an u + b an−1 + . . . + 1
Il ne reste plus qu'à calculer la somme S = 1 + . . . + an−1 . Pour cela, on calcule
aS = a + . . . + an et on fait la diérence ; on obtient aS − S = an − 1. Finalement :
an − 1
S = 1 + . . . + an−1 =
a−1
et on obtient bien la formule voulue. Il s'agit quand même de faire attention : ce qui précède
n'a rien d'une démonstration rigoureuse et implacable, c'est juste un préambule à faire au
brouillon. La vraie démonstration est bien celle que l'on a faite par récurrence juste avant.

17
5.2 Les suites homographiques
Il s'agit des suites dénies par la formule de récurrence suivante :
aun + b
un+1 =
cun + d
les nombres a, b, c et d étant des réels xés. Là encore, il faut xer une valeur pour u0 pour
déterminer complètement la suite ; disons que, comme tout à l'heure, u0 = u un certain
réel xé également.
On se propose une fois de plus de déterminer une formule explicite donnant directement
la valeur de un . Nous allons en fait présenter la méthode via l'exercice suivant :

Exercice : On considère donc la suite (un ) dénie précédemment. On dénit en outre la


fonction f : x 7→ ax+b 16 de la valeur interdite −d mais on retiendra
cx+d . On se souciera peu c
que un+1 = f (un ).
a) Prouver que l'équation f (l) = l admet 0, 1 ou 2 solutions selon les valeurs choisies
pour a, b, c et d.
On supposera dans la suite que l'équation f (l) = l admet deux solutions distinctes que
l'on appelera l1 et l2 .
b) On dénit la suite vn par la formule :
un − l1
vn =
un − l2
Prouver que vn vérie une relation de récurrence simple. (On pourra pour cela commencer
par calculer et simplier f (x) − f (y), x et y étant des réels quelconques).
c) En déduire une formule explicite pour vn puis pour un .
Solution :
I a) L'équation f (l) = l conduit directement à cl2 + (d − a) l − b = 0. Il s'agit d'une
équation de degré 2 (sauf dans le cas où c = 0 ; on remarque que ce cas a d'ailleurs déjà
été traité dans le paragraphe précédent) qui admet 0, 1 ou 2 solutions selon le signe du
discriminant ∆ = (d − a)2 + 4bc.
b) Comme indiqué par l'énoncé, on commence par calculer :
ax + b ay + b (ax + b) (cy + d) − (cx + d) (ay + b)
f (x) − f (y) = − =
cx + d cy + d (cx + d) (cy + d)
acxy + adx + bcy + bd − acxy − bcx − ady − bd
=
(cx + d) (cy + d)
(ad − bc) (x − y)
=
(cx + d) (cy + d)
Et maintenant :
un−1 − l1 f (un ) − f (l1 )
vn+1 = =
un−1 − l2 f (un ) − f (l2 )
(ad − bc) (un − l1 ) (cun + d) (cl2 + d)
= ×
(cun + d) (cl1 + d) (ad − bc) (un − l2 )
cl2 + d
= × vn
cl1 + d
16
Normalement, il faudrait mais bon c'est un problème assez délicat qui n'apporte nalement pas grand-
chose...

18
ce qui est eectivement satisfaisant pour une relation de récurrence simple : si on pose en
outre k = cl 2 +d
cl1 +d , on obtient vn+1 = kvn , chose que l'on sait déjà traiter.
c) En utilisant les formules du paragraphe précédent, on obtient
u − l1
vn = k n v0 = k n ·
u − l2
un −l1
Maintenant il n'est plus bien dicile de trouver un . De la relation vn = un −l2 dénissant
vn , on tire :
l2 vn − l1 l2 k n (u − l1 ) − l1 (u − l2 )
un = =
vn − 1 k n (u − l1 ) − (u − l2 )
ce qui n'est certes pas une formule très agréable, mais bon... J

5.3 La suite de Fibonacci


La suite de Fibonacci est celle qui est dénie de la façon suivante :
½
F1 = F2 = 1
Fn = Fn−1 + Fn−2 pour n > 3

Le but est encore de calculer explicitement la valeur de Fn . En fait, nous n'allons pas
le faire. Nous laissons au lecteur le plaisir immense de prouver par récurrence que :
√ "à √ !n à √ !n #
5 1+ 5 1− 5
Fn = −
5 2 2

Remarquons toutefois qu'une fois cette démonstration faite, elle prouvera ipso facto
que le nombre déni par la formule précédente est un entier pour toute valeur de n, ce qui
n'est pas évident a priori.
Remarquons également que la suite de Fibonacci apparaît dans divers contextes où on
ne l'attend pas toujours. Un exemple est l'exercice suivant : on se demande de combien de
façons on peut monter un escalier de n marches sachant que l'on monte les marches soit
par une soit par deux, bien entendu en pouvant changer d'avis autant de fois qu'on le veut
au cours de l'ascension.
Comme donner des noms aux choses permet souvent de mieux les apprivoiser, on appelle
un ce nombre recherché. Pour des petits escaliers, on sait résoudre le problème. Si l'escalier
a une marche, il n'y a qu'une façon de le gravir ; ainsi u1 = 1. Si l'escalier a deux marches,
il y a deux façons : soit on monte les deux marches à la fois, soit on y va pépère ; ainsi
u2 = 2.
Mettons-nous maintenant devant un escalier de n marches, avec n > 3. Au bas de
l'escalier, on a deux possibilités : soit on monte une marche, soit deux. Si on a monté une
marche, il nous restera évidemment n − 1 marches à gravir, sinon il nous en restera n − 2.
De cela, on déduit la relation :
un = un−1 + un−2
oh ! miracle, on retrouve la relation de Fibonacci. Ainsi on voit facilement que :
 
√ Ã √ !n+1 Ã √ !n+1
5 1+ 5 1− 5 
un = Fn+1 = −
5 2 2

19
5.4 Le triangle de Pascal
On va se concentrer ici sur les suites (un,k ) indexées à la fois par les indices n et k , eux
deux parcourant N. En fait, on va voir plus précisément comment on peut dénir une telle
suite de façon récurrente. Commençons tout de suite par un exemple. Soit donc la suite
(un,k ) dénie par les relations suivantes :

 un,0 = 1 pour tout n
u =0 pour tout k > 1
 0,k
un,k = un−1,k + un−1,k−1 pour n > 0 et k > 0

Commençons par calculer les valeurs des  premiers  termes de la suites. On les reporte
dans le tableau suivant :

k
n 0 1 2 3 4 5
0 1
1 1 1
2 1 2 1
3 1 3 3 1
4 1 4 6 4 1
5 1 5 10 10 5 1

Disons en premier lieu que le tableau que l'on obtient ainsi s'appelle le triangle de
Pascal. Les nombres qui apparaîssent dans ce tableau sont ce que l'on appelle les coecients
binomiaux, cette dernière terminologie s'expliquant avec la formule de binôme que nous
allons voir par la suite. La quantité un,k se note traditionnellement Ckn
On remarque que les conditions données permettent de remplir toutes les cases du
tableau : on commence par exemple par remplir la première ligne, on peut dès lors calculer
les valeurs à mettre sur la seconde et ainsi de suite. Il faut peut-être dire que les cases non
remplies dans le tableau correspondent à des 0.
Comme pour les autres cas traités, il est ici encore possible de donner une formule
explicite pour Ckn . Précisément, on a :

 Ck = n!
n si k 6 n
k! (n − k)!
 k
Cn = 0 sinon

où i! (lire factorielle i) désigne par dénition le produit 1 × 2 × . . . × i et où par convention


0! = 1.
La démonstration de cette dernière armation se fait à nouveau par récurrence. Tou-
tefois, comme ce coup-ci, on a deux indices, il faut faire un peu plus attention à ce que
l'on fait. Ici, ce ne sera pas compliqué : la récurrence ne va concerner que l'indice n. Pré-
cisément, l'énoncé Pn que l'on va considérer sera :  pour tout entier k > 0, le coecient
binômial Ckn est donné par la formule ci-dessus .
L'initialisation est simple : pour n = 0, on vérie directement que la formule donnée
redonne les mêmes valeurs que la dénition. On suppose donc maintenant que tous les
énoncés Pi sont vrais pour i 6 n et on cherche à démontrer Pn+1 .

20
Pour cela, il va bien sûr falloir distinguer plusieurs cas. En premier lieu, si k = 0, la
(n+1)!
formule k!(n+1−k)! vaut bien 1 comme le veut la dénition de C0n+1 .
Si maintenant k est compris entre 1 et n, il s'agit de prouver l'identité suivante :

(n + 1)! n! n!
= +
k! (n + 1 − k)! k! (n − k)! (k − 1)! (n − k + 1)!

ce qui résulte de manipulations élémentaires que nous laissons au lecteur17 .


Finalement, pour les k strictement supérieur à n, il s'agit d'additionner des 0 entre eux,
un 1 venant éventuellement jouer les trouble-fêtes. C'est tout à fait immédiat.
Tout cela prouve l'hérédité et démontre donc la formule annoncée.

Les coecients binômiaux ont un intérêt immense en mathématiques et principalement


en combinatoire. Ils apparaîssent en outre dans la formule du binôme de Newton qui donne
le développement de (a + b)n . Cette formule s'écrit ainsi :
n
X
(a + b)n = Ckn an−k bk
k=0
P
le signe   voulant dire que l'on fait varier k entre 0 et n, que pour chacun de ces k on
évalue le terme écrit à côté du signe, et que l'on somme tous les termes obtenus (il y en a
donc n + 1). Par exemple si on l'applique pour n = 5, on trouve :

(a + b)5 = a5 + 5a4 b + 10a3 b2 + 10a2 b3 + 5ab4 + b5

Il ne nous reste plus qu'à prouver cette formule. Bien sûr, cela va se faire une fois de
plus par récurrence. Pour n = 1, la formule donne a+b = a+b, ce qui est eectivement une
vérité incontestable. On suppose maintenant que la formule est vraie pour tout exposant
inférieur ou égal à n et on peut prouver qu'elle reste vraie pour l'exposant n + 1. On
entreprend pour ce faire le calcul suivant :
à n !
n+1 n
X
k n−k k
(a + b) = (a + b) (a + b) = (a + b) Cn a b
k=0
à n
! Ã n !
X X
= Ckn an−k+1 bk + Ckn an−k bk+1
k=0 k=0
à n
! Ãn−1 !
X X
n+1
= a + Ckn an−k+1 bk + Ckn an−k bk+1 + bn+1
k=1 k=0
à n
! Ã n !
X X
n+1
= a + Ckn an+1−k bk + Ckn an+1−k bk + bn+1
k=1 k=1
à n ³
!
X ´
= an+1 + Ckn + Ck−1
n an+1−k bk + bn+1
k=1
n+1
X
= Ck+1
n a
n+1−k k
b
k=0
17
Hé, hé !

21
Et ceci achève la démonstration. On remarque en outre que si l'on ne connaissait pas à
l'avance les coecients binômiaux Ckn , le calcul précédent permet de trouver la relation de
récurrence qui les dénit. Bien sûr, accéder à la formule explicite est une autre paire de
manches, mais dans d'autres situations il n'y aura pas forcément de formules explicites et
connaître une relation de récurrence simple peut déjà aider énormément.

La chose à laquelle il faut prendre garde avec ces suites à multi-indices, c'est que le calcul
de proche en proche ne se fait pas forcément ligne par ligne. Parfois, c'est plus compliqué
et il faut donc adapter la façon de faire les récurrences ; il est par exemple possible que le
tableau se construise en  diagonale , auquel cas, il faudra privilégier les récurrences sur
la somme n + k .

5.5 L'opération  eXclusive OR 


Présentation de la suite
Voyons un autre exemple de suite récurrente ayant plusieurs indices. La dénition par
récurrence peut paraître ce coup-ci surprenante ; la voici :
¡© ª © ª¢
un,k = mex un0 ,k , n0 < n ∪ un,k0 , k 0 < k (1)

où mexA est le plus petit entier naturel qui n'appartient pas à l'ensemble A. On remarque
qu'un tel entier existe toujours dans les cas précédents puisque tous les ensembles considérés
sont nis.

An de comprendre comment cela fonctionne, le plus simple est sans doute d'essayer
de calculer les premiers termes. Regroupons les résultats dans le tableau suivant :

k
n 0 1 2 3 4 5
0 0 1 2 3 4 5
1 1 0 3 2 5 4
2 2 3 0 1 6 7
3 3 2 1 0 7 6
4 4 5 6 7 0 1
5 5 4 7 6 1 0

Expliquons donc comment on parvient à un tel résultat. La formule nous dit que l'entier
qui doit être écrit dans la case de coordonnées (n, k) doit être le plus petit qui n'est ni écrit
à gauche ni au-dessus de ladite case. Ainsi, on remplit le tableau de gauche à droite et de
haut en bas.
Pour la case (0, 0), il n'y a aucune case qui lui soit située à gauche ou en haut. Il s'agit
donc de prendre le plus petit entier ; c'est 0. Pour sa voisine de droite, il faut prendre le plus
petit entier qui n'est pas 0, c'est donc 1. De même pour la voisine du dessous. Maintenant
pour la case de coordonnées (0, 2), il faudra prendre le plus petit entier qui n'est ni 0, ni
1 ; c'est donc bien 2. Ainsi de suite...

Cet exemple est une mine d'or si l'on veut s'entraîner à faire des récurrences simples :
on s'amuse à repérer des propriétés sur la tableau et on essaie de les démontrer.

22
Par exemple, commençons par prouver que un,0 = n pour tout entier n. L'initialisation
a déjà été faite : on a déjà calculé u0,0 = 0. Maintenant on suppose que un0 ,0 = n0 pour
tout n0 6 n et on veut calculer un+1,0 . Par dénition, étant donné qu'il n'y a pas d'entier
strictement inférieur à 0, c'est le plus petit entier ne s'écrivant pas un0 ,0 pour n0 < n + 1,
c'est-à-dire n0 6 n. Mais on les connait ces entiers justement : un+1,0 est donc le plus petit
entier qui n'est ni 0, ni 1, ..., ni n : c'est bien n + 1. Ceci achève donc l'étape d'hérédité et
la démonstration.

Une deuxième chose que l'on peut remarquer et prouver, peut-être un poil plus dicile,
est que pour tout entier n, un,n = 0. Allons-y. L'initialisation est déjà connue. Passons
directement à l'hérédité : on suppose donc que pour tout n0 6 n, un0 ,n0 = 0 et il s'agit
de calculer un+1,n+1 . C'est par dénition le plus petit nombre qui ne s'écrit ni un0 ,n+1 , ni
un+1,n0 pour n0 6 n. Le problème est qu'apparemment on ne connaît rien sur ces nombres.
Toutefois, ce que l'on souhaite, c'est arriver à la conclusion selon laquelle un+1,n+1 = 0.
Il sut donc de prouver que 0 n'apparaît pas parmi les un0 ,n+1 et les un+1,n0 , n0 étant
toujours inférieur ou égal à n. Voyons donc comment est déni un0 ,n+1 : c'est le plus petit
entier qui ne s'écrit ni sous la forme up,n+1 pour p < n0 ni sous la forme un0 ,q pour q 6 n.
Mais parmi ces entiers, il y a un0 ,n0 qui est nul par hypothèse de récurrence, ce qui assure
la non-nullité de un0 ,n+1 . De la même façon, on prouve la non-nullité de un+1,n0 et la
conclusion en découle.

Prouvons un dernier fait : pour tous entiers n et k , un,k = uk,n . Ce coup-ci, on a deux
indices et il faut choisir comment faire la récurrence. En fait, à peu près toutes les choses
auxquelles on peut penser fonctionnent. Nous allons pour le plaisir18 faire une récurrence
sur la somme n + k , ce qui n'est pas forcément le plus simple, mais pas forcément le plus
compliqué non plus.
Notre énoncé de récurrence va dépendre d'un nouveau paramètre s et sera le suivant :
 pour tout couple d'entiers (n, k) tels que n + k = s, on a un,k = uk,n . Si l'on veut voir
comment les choses se passent  géométriquement , il faut constater que l'on prouve non
pas le résultat ligne par ligne ou colonne par colonne mais en se déplaçant en diagonale
dans le tableau.
Voyons comment la récurrence fonctionne. L'étape d'initialisation est simple. Le seul
couple d'entier (n, k) pour lequel n + k = 0 est le couple (0, 0) et il est incontestable que
u0,0 = u0,0 . Montrer l'hérédité revient à supposer que pour tous les couples (n, k) tels que
n + k 6 s, on a un,k = uk,n et à montrer que la conclusion demeure pour les couples (n, k)
tels que n + k = s + 1. Prenons donc n et k vériant n + k = s + 1 et voyons ce que valent
respectivement un,k et uk,n .
Le premier est déni comme étant le plus petit entier ne s'écrivant ni sous la forme
un0 ,k pour n0 < n, ni sous la forme un,k0 pour k 0 < k . Le second, quant à lui, est déni
comme étant le plus petit entier ne s'écrivant ni sous la forme uk,n0 pour n0 < n, ni sous
la forme uk0 ,n pour k 0 < k . Mais si k 0 < k , k 0 + n < s + 1 ou encore k 0 + n 6 s et donc le
couple (k 0 , n) relève de l'hypothèse de récurrence, ce qui assure uk0 ,n = un,k0 . De la même
façon un0 ,k = uk,n0 . Les deux entiers que l'on veut comparer sont donc dénis de la même
façon ; ils sont par le fait égaux. Cela conclut.

Bien entendu, évoquer un argument de symétrie eût conduit à une démonstration plus
simple. Une façon d'exploiter cette idée est de développer l'argument suivant. On a vu que
18
En fait, surtout pour montrer ce que peut donner ce genre de raisonnements.

23
la relation (1) sut à elle seule à dénir la double suite (un,k ). Ainsi si (vn,k ) désigne une
autre suite et que l'on arrive à prouver que cette suite vérie la même relation (1), on aura
prouvé que pour tous entiers n et k , un,k = vn,k .
Cette dernière remarque s'applique à notre situation en prenant vn,k = uk,n . La conclu-
sion est alors immédiate.

Une méthode de calcul


Il est encore possible de donner une  formule  explicite pour le calcul de un,k . Pour
expliquer cette formule, il va nous falloir dans un premier temps expliquer la base 2. On fait
remarquer que  la base 2  n'est pas quelque chose d'anecdotique et permet de résoudre
nombreux problèmes et de comprendre nombreuses théories.

L'idée est fort simple : on décrète qu'à partir de maintenant, on ne comptera plus
qu'avec deux chires, en l'occurrence 0 et 1. Pour compter, on utilise toujours la même
méthode : on commence à 0 ; on incrémente ensuite tant que l'on peut le chire des unités,
lorsque l'on ne peut plus, on le remet à 0 et on incrémente le chire des dizaines19 ; si l'on
ne peut plus incrémenter ce chire, on le remet à 0 à son tour et on incrémente celui des
centaines et ainsi de suite.
Après application de cet algorithme, on obtient la liste des premiers nombres écrits en
base 2 :

0, 1, 10, 11, 100, 101, 110, 111, 1000, 1001, 1010, 1011, etc.
Le n-ième nombre ainsi listé (on commence à compter à 0) est ce que l'on appelle
l'écriture en base 2 de l'entier n. Ainsi l'écriture en base 2 de 1 est 1, celle de 9 est 1001.
Il serait sans doute bon d'expliquer maintenant comment on retrouve un nombre à partir
de son écriture en base 2 et réciproquement.
Plusieurs remarques simples vont permettre de nous donner de précieux indices. Tout
d'abord dans la liste des nombres, il apparaît évidemment en premier lieu les nombres à un
seul chire, puis ceux de deux chires et ainsi de suite. D'autre part, il y a exactement 2n
nombres qui s'écrivent avec moins de n chires : pour chaque emplacement on a le choix
entre 0 et 1 et bien sûr toutes les suites possibles de chires apparaîssent. De cela, on déduit
que le premier nombre à n + 1 chires (ie 1 suivi de n 0) sera le 2n -ième de la liste20 .
Mais si maintenant, on regarde ce qui vient après ce premier nombre à n + 1 chires, on
constate que c'est exactement la liste prise du début mais avec ce 1 en plus ; évidemment,
c'est la même construction. Dans un langage plus mathématique, on vient de dire que la
position du nombre 1xn−1 xn−2 . . . x0 (où les xi sont des chires pris parmi 0 et 1 est celle
du nombre xn−1 xn−2 . . . x0 augmentée de 2n . Finalement21 , on trouve que la position du
nombre xn . . . x0 est :
xn 2n + xn−1 2n−1 + . . . + x0

Maintenant que l'on sait convertir de la base 2 en base 10 (ie trouver la position
connaissant le nombre), on voudrait s'intéresser à l'opération inverse. Prenons par exemple
l'entier 217 et essayons de trouver quel nombre doit être écrit à la 217-ième place. D'après
19
Il ne faut sans doute plus dire  dizaines  pour parler du deuxième chire en partant de la droite
lorsque l'on compte en base 2, mais bon...
20
Remarquez que cela fonctionne aussi pour n = 0.
21
Le lecteur pas vraiment convaincu pourra prouver le résultat par récurrence lui-même.

24
ce qui précède, il s'agit d'écrire 217 comme une somme de puissances de 2. Quelles sont
donc les puissances de 2 ? Il y a dans l'ordre : 1, 2, 4, 8, 16, 32, 64, 128, 256, etc. Je
m'arrête à 256 car il est clair que lui et les suivants ne pourront pas intervenir pour donner
un résultat aussi minable que 217.
Il y a maintenant deux façons de voir les choses : soit on commence par le haut, soit un
commence par le bas. Commençons par le haut dans un premier temps. On a dit qu'il ne
fallait pas prendre 256 mais 128 on peut, mais en fait c'est même obligatoire car la somme
des autres puissances arrivent à peine à 127. On prend donc 128, il nous reste 217−128 = 89
à faire. On fait pareil : on prend la puissance de 2 immédiatement inférieure à 89, c'est ici
64. Il reste ensuite 25, on prend donc 16. Au nal, on obtient :

217 = 128 + 64 + 16 + 8 + 1

et donc le 217-ième nombre listé sera 11011001.


Si on commence par le bas, maintenant, comment faut-il procéder ? On remarque que
toutes les puissances de 2 sont paires, à l'exception de 20 = 1. Et 217 est impair, il faut donc
obligatoirement prendre ce 1. Il nous reste maintenant 216 à faire et on ne doit prendre que
des puissances de 2 paires ; il nous reste donc 2162 = 108 à faire avec toutes les puissances
de 2. Ce coup-ci 108 est pair, il ne faut donc pas prendre le 1, c'est-à-dire qu'il ne faut pas
prendre le 2 pour 217. On divise encore par 2 et regarde à nouveau la parité et ainsi de
suite.

Fort de savoir écrire les nombres en base 2, on va pouvoir dénir ce que l'on appelle le
ou exclusif (eXclusive OR en anglais) que l'on note souvent XOR et que l'on notera nous
#. La méthode de calcul est la suivante. On part de deux entiers, par exemple 1548 et 217.
On écrit ces nombres en base 2 et on pose l'opération suivante :

11000001100
# 11011001
11011010101
qui est une addition sans retenue. Ainsi lorsque deux chires diérents sont écrits l'un
en dessous de l'autre, le résultat sera 1, sinon il sera 0. Sur notre exemple, on obtient
1548 # 217 = 1749.

Nous allons maintenant prouver que un,k = n # k . Et pour cela, nous allons simplement
démontrer que (n, k) 7→ n # k vérie la relation (1). On prend donc n et k deux entiers et
on veut montrer deux choses :
1. n # k 6= n0 # k pour tout n0 < n et n # k 6= n # k 0 pour tout k 0 < k
2. tout entier a < n # k s'écrit soit sous la forme n0 # k pour un n0 < n, soit sous la
forme n # k 0 pour un k 0 < k

La première chose est plutôt simple à voir. Il s'agit d'une propriété  négative , on
raisonne donc par l'absurde. Que se passerait-il donc si on avait n # k = n0 # k avec n0 < n ?
Il est pas bien dicile, en regardant comme on a déni l'opération # de se convaincre que
l'égalité n # k = n0 # k va entraîner n = n0 , ce qui est absurde.
Une façon plus rigoureuse d'obtenir la dernière implication est de  composer par # k
des deux côtés . Précisément si n # k et n0 # k sont égaux alors il en est de même de
(n # k) # k et (n0 # k) # k mais ces deux dernières quantités sont respectivement égales à
n et à n0 . On en déduit bien ce que l'on voulait.

25
Passons donc à la seconde propriété. On considère un entier a strictement inférieur à
n # k . Dans ces conditions, l'écriture en base 2 de ces deux entiers sera forcément de la
forme suivante :

a : a1 . . . ap 0 . . .
n # k : a1 . . . ap 1 . . .

Ainsi, n et k s'écrivent par exemple :

n : n1 . . . n p 1 . . .
k : k1 . . . k p 0 . . .

Bien sûr le 1 et le 0 peuvent être intervertis, mais il y a forcément deux chires diérents
à cette position puisque leur ou exclusif vaut 1. On traite ce cas pour l'instant, l'autre se
faisant en fait de façon tout à fait similaire.
En outre, on n'oublie pas que l'on a la relation ni # ki = ai , qui implique ai # ki = ni .
Finalement a # k s'écrit :

n0 = a # k : n1 . . . n p 0 . . .

Ce nombre est donc strictement plus petit que n et il vérie en outre n0 # k = a, ce qui est
exactement ce que l'on voulait.
Ceci termine donc la preuve et la relation de récurrence (1) dénit une suite dont le
terme général est :
un,k = n # k
ce que l'on peut s'amuser à vérier sur les premières valeurs calculées dans le tableau.

5.6 Digression sur l'intérêt de la base 2


L'écriture en base 2 est particulièrement intéressante lorsque l'on a aaire à des suites
récurrentes pour lesquelles u2n et u2n+1 sont tous les deux dénis en fonction de un . La
raison en est que si n s'écrit np . . . n0 en base 2, alors 2n va s'écrire np . . . n0 0 et 2n + 1
s'écrira np . . . n0 1. Ainsi très souvent, les propriétés de ces suites se lisent sur leur écriture
en base 2.
Bien entendu, si ce sont les nombres u3n , u3n+1 et u3n+2 qui sont dénis en fonction de
un , il sera judicieux de compter en base 3 et ainsi de suite.

Pour illustrer ces idées, proposons l'exercice suivant :


Exercice (OIM 1988) : On désigne par f l'application de l'ensemble des entiers strictement
positifs dans lui-même dénie par les conditions suivantes :

f (1) = 1, f (3) = 3

et pour tout entier n strictement positif :

f (2n) = f (n)
f (4n + 1) = 2f (2n + 1) − f (n)
f (4n + 3) = 3f (2n + 1) − 2f (n)

26
Déterminer le nombre des entiers n, 1 6 n 6 1988 pour lesquels f (n) = n.

Solution :
I Étant donnée la dénition de f , il est sans doute souhaitable de regarder le comportement
de cette fonction sur les nombres écrits en base 4 ou 2. Commençons par 2, ça ne peut pas
faire de mal.
Comme on voit pas très bien a priori, ce que f pourrait vouloir faire avec les chires
de l'écriture en base 2, on calcule les premières valeurs :

1 7→ 1 1 7→ 1
2 7 → 1 10 7 → 01
3 7 → 3 11 7 → 11
4 7 → 1 100 7 → 001
5 7 → 5 101 7 → 101
6 7 → 3 110 7 → 011
7 7 → 7 111 7 → 111
8 7 → 1 1000 7 → 0001
9 7 → 9 1001 7 → 1001
10 7→ 5 1010 7 → 0101
11 7→ 13 1011 7 → 1101
12 7→ 3 1100 7 → 0011
13 7→ 11 1101 7 → 1011

Rapidement ou non, on constate nalement que f semble inverser l'écriture en base 2. Plus
précisément si a s'écrit en base 2, ap . . . a0 où ap est 1, il semblerait que f (x) soit le nombre
qui s'écrive a0 . . . ap en base 2
Essayons de prouver ce fait. On raisonne évidemment par récurrence. On ne sait pas
trop que faire pour l'étape d'initialisation puisque l'on a une valeur pour f (1) et une pour
f (3) ; disons que l'on vérie jusqu'à 4, ce qui de toute façon a déjà été fait.
Il reste à prouver l'hérédité. Supposons donc que f fasse bien ce que l'on veut d'elle sur
les entiers 1, . . . , n et prouvons le pour l'entier n + 1. Il y a alors trois cas à distinguer.
• Tout d'abord si n + 1 est pair, alors il s'écrit 2k pour un certain entier k . Supposons
que k s'écrive kp . . . k0 en base 2 où kp vaut 1. Alors 2k s'écrit :

2k : kp . . . k0 0

et par hypothèse de récurrence, f (k) s'écrit :

f (k) : k0 . . . kp

ce qui correspond bien à l'écriture renversée. L'hérédité est donc prouvée dans ce cas.
• Maintenant si n + 1 s'écrit 4k + 1 pour un certain entier k . Écrivons encore k en base
2 : kp . . . k0 . Alors 4k + 1 s'écrit :

4k + 1 : kp . . . k0 01

et en utilisant l'hypothèse de récurrence on peut poser l'opération suivante :

2f (2k + 1) : 1k0 . . . kp 0
− f (k) : k0 . . . kp
10k0 . . . kp

27
ce qui est bien ce que l'on veut encore une fois.
• Finalement si n + 1 s'écrit 4k + 3 pour un certain entier k . Écrivons encore k en base
2 : kp . . . k0 . Alors 4k + 3 s'écrit :

4k + 3 : kp . . . k0 11

et en utilisant l'hypothèse de récurrence on peut poser l'opération suivante :

2f (2k + 1) : 1k0 . . . kp 0
+ f (2k + 1) : 1k0 . . . kp
− 2f (k) : k0 . . . kp 0
11k0 . . . kp
la première et la troisième ligne se simpliant bien. Cela conclut l'hérédité.

Il ne reste plus qu'à compter le nombre d'entiers  symétriques en base 2  et inférieurs à


1988. Commençons peut-être par écrire ce nombre en base 2 : c'est 11111000100. Ce nombre
s'écrit avec 11 chires, on n'aura donc pas encore trop de problèmes pour dénombrer le
nombre de solutions de moins de 10 chires.
Avec un seul chire, il n'y a qu'une solution ; c'est 1. Avec deux chires, il n'y a aussi
qu'une seule solution ; c'est 11. Avec trois chires, maintenant, le premier est forcément
xé à 1 et par conséquent le dernier aussi, mais on a libre choix sur celui du milieu, il y a
donc deux solutions.
p−1
De la même façon pour p valant 5, 7 ou 9, il va y avoir 2 2 solutions de p chires. Pour
p−2
les p pairs, donc valant 4, 6, 8 ou 10, il y aura 2 2 solutions. Ainsi parmi les nombres qui
ont moins de 10 chires, on dénombre 1 + 1 + 2 + 2 + 4 + 4 + 8 + 8 + 16 + 16 = 62 solutions.
Voyons les nombres de 11 chires maintenant. Une solution éventuelle doit s'écrire en
base 2 sous la forme suivante :
1_________1
les cinq premiers  _  représentant a priori des chires arbitraires, et les quatre derniers
étant déterminés par le choix des premiers. Toutefois pour que ce nombre reste inférieur
à 11111000100, il faut imposer que les quatre premiers  _  ne¡ 4soient¢ pas simultanément
des 1, et c'est en fait la seule contrainte. On dénombre alors 2 − 1 × 2 = 30 solutions
dans cette situation.
Au nal l'équation proposée admet 92 solutions. J

5.7 Parité des coecients binômiaux


On se demande dans ce chapitre à quelles conditions sur les entiers n et k , le coecient
binômial Ckn est pair. La réponse n'est pas forcément évidente mais est assez élégante. On
commence par écrire n et k en base 2 ; on obtient par exemple :

n : np . . . n0
k : kp . . . k0

Dans ces conditions, Ckn est un nombre pair si et seulement si il existe un indice i compris
entre 0 et p pour lequel à la fois ni = 0 et ki = 1. (On dira alors que le couple (n, k) vérie
le critère (C)).

28
Encore une fois, nous allons prouver ce résultat par récurrence sur n. L'initialisation
est facile. Si n = 0, n s'écrit en base 2 simplement avec des chires 0, donc dès qu'il arrive
un 1 dans l'écriture en base 2, c'est-à-dire dès que k est non nul, la condition que l'on a
donnée ne va pas être vériée. D'autre part, le seul k qui soit tel que Ck0 soit non nul est
k = 0, et alors C00 = 1. Ainsi, Ck0 est pair si et seulement si k est non nul. En mettant les
deux choses précédentes ensemble, on peut conclure pour l'initialisation.
Voyons maintenant l'hérédité. Prenons un entier n et supposons que pour tous les
n0 6 n, l'équivalence donnée précédemment soit vériée. Il s'agit de la montrer pour n + 1.
Il faut traiter le cas k = 0 à part, puisque la dénition de Ckn traite ce cas à part. On a
alors C0n = 1 qui est un nombre impair. En outre, il n'apparaît dans l'écriture en base 2 de
k que des 0 de sorte qu'il n'existe aucun indice i pouvant vérier la condition. On a donc
bien l'équivalence dans ce cas.
On suppose désormais k > 0, et l'on peut ainsi écrire :

Ckn+1 = Ckn + Ck−1


n

La stratégie consiste alors à étudier la parité de chacun de deux termes de la somme


précédente, ceci bien entendu en fonction du critère donné, et à en déduire celle de Ckn+1 .
Commençons donc par écrire n et k − 1 en base 2 :

n : np . . . nN +1 01 . . . 1
k − 1 : kp . . . kK+1 01 . . . 1

où donc les indices N (resp. K ) désigne la position du dernier 0 de l'écriture en base 2 de


n (resp. k − 1). Cela permet évidemment de savoir comment vont s'écrire n + 1 et k en
base 2. Plus précisément, on aura :

n + 1 : np . . . nN +1 10 . . . 0
k : kp . . . kK+1 10 . . . 0

Il y a maintenant trois cas à distinguer selon les positions relatives de N et de K .


• Commençons par le plus simple, celui où N = K . Dans ces conditions, d'après l'hy-
pothèse de récurrence, Ckn sera forcément pair, puisqu'en position N = K , il y aura res-
pectivement un 0 dans n et un 1 dans k .
On distingue maintenant deux sous-cas selon la parité de Ck−1 n . Toujours d'après l'hy-
pothèse de récurrence, si Ck−1
n est pair, alors il va exister un indice i pour lequel ni = 0 et
ki = 1, mais cela ne peut arriver pour les indices i 6 N = K . Ainsi, il va exister un indice
i tel que ni = 0, ki = 1 et N + 1 6 i 6 p, et donc le couple (n + 1, k) va vérier le critère
(C). En outre, Ckn+1 va être un nombre pair, comme somme de deux nombres pairs. On a
bien l'équivalence dans ce cas.
On traite de la même façon le cas où Ck−1n est impair.
• Si N < K , alors en regardant en position N et en utilisant l'hypothèse de récurrence,
on voit que Ck−1
n est un nombre pair.
Comme tout à l'heure, on commence par regarder ce qu'il se passe si Ckn est pair. Dans
ce cas, soit nK = 0, soit il existe un indice i > K + 1 tel que ni = 0 et ki = 1. On constate
alors que quoi qu'il en soit le couple (n + 1, k) vérie le critère (C). De plus, Ckn+1 est pair,
comme somme de deux nombres pairs, ce qui conclut.
On traite l'autre cas de façon analogue.

29
• Finalement, si N > K , alors en regardant en position K , on voit que le couple
(n + 1, k) vérie toujours le critère (C). Il s'agit donc de montrer que les nombres Ckn et
Ck−1
n sont de même parité.
Si Ckn est pair, c'est soit que kN est pair, soit qu'il existe un indice i > N + 1 tel que
ni = 0 et ki = 1. On voit que cela implique que le couple (n, k − 1) vérie le critère (C) et
donc que Cnk−1 est pair. Youpi.
On fait de même si Ckn est impair.

Ceci extermine22 l'hérédité et la récurrence.

Là encore, on peut se demander comment l'on peut penser à un tel critère. Il ne tombe
pas du ciel encore une fois et quelques expérimentations simples permettent de le deviner.
Nous allons essayer de les exposer. Tout d'abord, redessinons le tableau de Pascal en
remplaçant les nombres pairs par des 0 et les impairs par des 1. On obtient :

0 1 2 3 4 5 6 7 8 9 10 11 12 13 14 15 16
0 1
1 1 1
2 1 0 1
3 1 1 1 1
4 1 0 0 0 1
5 1 1 0 0 1 1
6 1 0 1 0 1 0 1
7 1 1 1 1 1 1 1 1
8 1 0 0 0 0 0 0 0 1
9 1 1 0 0 0 0 0 0 1 1
10 1 0 1 0 0 0 0 0 1 0 1
11 1 1 1 1 0 0 0 0 1 1 1 1
12 1 0 0 0 1 0 0 0 1 0 0 0 1
13 1 1 0 0 1 1 0 0 1 1 0 0 1 1
14 1 0 1 0 1 0 1 0 1 0 1 0 1 0 1
15 1 1 1 1 1 1 1 1 1 1 1 1 1 1 1 1
16 1 0 0 0 0 0 0 0 0 0 0 0 0 0 0 0 1

La première remarque à faire consiste à dire que pour construire le tableau précédent,
on n'est pas du tout obligé de regarder la parité des coecients qui apparaîssent dans le
tableau de Pascal. Il sut plutôt de remarquer que la somme de deux nombres pairs ou de
deux nombres impairs donne un résultat pair, alors que la somme d'un nombre pair et d'un
nombre impair donne un résultat impair. On utilise ensuite la dénition par récurrence.
Plus précisément, on construit le tableau comme on construisait le tableau de Pascal,
sauf que lorsque l'on a à faire  1 + 1 , on écrit 0 et pas 2.
Maintenant, il faut observer, contempler et conjecturer. Ce que l'on peut remarquer,
c'est que les lignes 2, 4, 8 et 16, c'est-à-dire les lignes puissances de 2, sont d'une forme bien
particulière : elles semblent commencer et se terminer par un 1 (mais cela est obligatoire et
évident) mais ne contenir que des 0 sinon. Que l'on sache ou non montrer cela23 , prenons-le
pour acquis.
Une conséquence majeure va alors apparaître. Par exemple, les lignes de 8 à 15 vont for-
cément être deux copies mises côte-à-côté des lignes 0 à 7, comme la méthode de construc-
tion du tableau le prouve directement. De la même façon les lignes de 2n à 2n−1 − 1 vont
être deux copies mises côte à côte des lignes de 0 à 2n − 1.
22
Une variante de  achève . Un délire de sup, vous ne pouvez sans doute pas comprendre ce qui peut
m'amuser là-dedans.
23
Ce résultat est bien entendu une conséquence du critère précédemment énoncé, mais on peut le montrer
directement de manière relativement simple.

30
Et lorsque l'on essaie de comprendre ce que peut entraîner tout cela, on pense à écrire les
nombres n et k en base 2 et on obtient nalement le critère énoncé au début du paragraphe.
Il est intéressant nalement de griser les cases impaires et de regarder le dessin obtenu.

Il est intéressant nalement de généraliser le résultat précédent en remplaçant 2 par un


nombre premier quelconque. Le théorème est alors le suivant :

Théorème 5

Soit p un nombre premier. Soient n et k deux entiers dont l'écriture en base p est donnée
par les formules suivantes :
n = nd pd + nd−1 pd−1 + . . . + n1 p + n0
k = kd pd + kd−1 pd−1 + . . . + k1 p + k0
les ni et les ki étant donc des entiers compris entre 0 et p − 1. Alors on a la congruence
suivante :
k
Ckn ≡ Ckndd · Cnd−1 k1 k0
d−1 · . . . · Cn1 · Cn0 (mod p)
cette dernière écriture signiant que la diérence des deux termes écrits de part et d'autre
du signe  ≡  est un multiple de p.

Ce dernier résultat implique donc que s'il existe un indice i tel que ki > ni alors le
nombre Ckn est un multiple de p. C'est exactement cela le critère que l'on avait annoncé
dans le cas p = 2.

31
6 Constructions
Nous allons voir dans ce chapitre que parfois il ne faut pas hésiter à mettre les choses
comme on veut qu'elles le soient.

6.1 Nombres univers et nombres normaux


Un nombre univers (en base 10) est un nombre réel pour lequel on peut trouver n'im-
porte quelle suite de chires dans son développement décimal. Ainsi dans un tel nombre,
on trouvera après la virgule un 3 quelque part, mais aussi un 4, et aussi un 12, et aussi un
154876 et tout ce que l'on veut ainsi.
Ce que l'on peut remarquer d'ores et déjà, c'est que dans un nombre univers, il va
apparaître non seulement un 3, mais en fait une innité de 3 après la virgule. En eet, il
devra apparaître 30, 31, . . . , 39, ce qui fait déjà pas moins de dix 3 (évidemment, ce ne sera
pas les mêmes : un 3 qui est suivi d'un 0 n'est pas suivi d'un 1). Mais il devra apparaître
aussi 300, . . . , 399, ce qui amène déjà le nombre de 3 à cent. En considérant les nombres à
quatre chires, on trouve mille 3 distincts, et ainsi de suite.

Trouver des nombres qui ne sont pas univers est quelque chose de facile. Pour prendre
un exemple bête, les entiers ne sont pas des univers : après la virgule, on ne trouve rien
d'autre que des 0. La fraction 13 n'est pas non plus un nombre univers ; ce coup-ci il n'y a
que des 3.
La question est maintenant d'en exhiber un, de nombre univers. On peut penser aux
fractions mais cela ne marche
√ pas (voir plus loin). On peut ensuite penser à des constantes
plus farfelues comme 2 ou π ... Ces nombres sont selon toute probabilité univers mais
personne aujourd'hui ne sait encore le prouver. On n'est donc guère plus avancé.
Pourtant obtenir un nombre univers est tout ce qu'il y a de plus facile, il sut d'écrire
ce que l'on veut. Et ce que l'on veut c'est le nombre suivant :

0 , 0 1 2 3 4 5 6 7 8 9 00 01 02 03 04 05 06 07 08 09 10 11 12 13 14 15 16 17 18 19 20 21 . . .

On voit bien comment ce spécimen est fabriqué : on commence par écrire  0,  et on ajoute
derrière successivement les chires un par un jusqu'à épuisement, puis les suites de deux
chires jusqu'à épuisement, on mettra ensuite les suites de trois chires et ainsi de suite.
On obtient à l'évidence un nombre univers.

Venons-en maintenant aux nombres normaux. Un nombre normal (en base 10) est un
1
nombre pour lequel les 0 apparaîssent avec une fréquence de 10 (cela signie que si on
appelle z (n) le nombre de zéros rencontrés avant la n-ième décimale, on a limn→∞ z(n)
n =
1 24 1 1
10 ), les 1 apparaîssent avec une fréquence de 10 , les 10 avec une fréquence de 100 , les
1
1847 avec une fréquence de 10000 et de même pour toute suite nie de chires.
De façon évidente, tout nombre normal est aussi un normal univers. Le contraire, par
contre, demande un peu plus de réexion. Existe-t-il des nombres univers qui ne sont pas
normaux ? Le nombre que l'on a construit ci-dessus est-il normal ? Ces deux dernières
questions ne sont a priori pas évidentes. Nous allons toutefois répondre à la première :
la réponse est positive et de fait il est facile comme précédemment de construire un tel
24
Je reste et je resterai volontairement ou sur les limites et les problèmes que cela entraîne, ce genre de
questions très intéressantes au demeurant ne rentrant pas vraiment dans les choses que je veux présenter.
Je ne me soucierai jamais par exemple du problème de la non-existence de ladite limite.

32
nombre. On souhaite que toutes les suites nies de chires soient représentées mais on
souhaite également par exemple que le chire 0 soit sur-représenté25 . Qu'à cela ne tienne,
on prend simplement :

0 , 0 0 1 0 2 0 3 0 4 0 5 0 6 0 7 0 8 0 9 0 00 00 01 00 02 00 03 00 04 00 05 00 06 00 07 00 . . .

Entre deux suites consécutives que l'on veut voir apparaître dans notre nombre univers,
on ajoute autant de 0 que nécessaire pour être sûr qu'au nal ils auront une proportion
supérieure à 21 et donc ne pouvant pas égaler 10
1
.
On a construit un nombre univers qui n'était pas normal, mais peut-on quand même
construire un nombre normal. Évidemment et si l'on excepte les détails techniques en-
nuyeux, les idées sont très similaires aux précédentes. On écrit tout d'abord  0,  et on
s'occupe dans un premier temps des suites d'un seul chire : pour cela, on répète susam-
ment de fois la séquence 0 1 2 3 4 5 6 7 8 9 pour que la proportion de 0, de 1, etc. se stabilise
1
(à quelque chose de petit près), donc à 10 . On s'occupe ensuite des suites de deux chires
et pour cela on répète la séquence 00 01 02 . . . 99, encore jusqu'à avoir une stabilisation
susante.
Bon, ce qui précède n'est que l'idée ; il y a énormément de détails techniques à régler...
nous souhaitons bon courage au lecteur qui veut écrire cela proprement.

6.2 Nombres rationnels et périodicité



On a déjà donné un exemple explicite de nombre irrationnel, en l'occurrence 2, mais il
est remarquable de se rendre compte que les méthodes utilisées précédemment permettent
également d'aboutir à un tel nombre.

Pour cela, nous allons essayer de comprendre comment se comporte la suite des dé-
cimales d'un nombre rationnel. Prenons donc par exemple 17 et posons la division. On
a:
1 7
10 0 , 142 857 1
30
20
60
40
50
10
3
La séquence 142 857 va se répéter inniment. Évidemment, on est retombé sur un
reste déjà rencontré et donc on va répéter les mêmes opérations (et de fait, normalement,
retrouver les mêmes résultats) jusqu'à ce que mort s'ensuive.
Mais cela n'est pas spécique à la division de 1 par 7. Dès que l'on divise un entier par
un autre, le nombre de restes possibles est ni et on sera ainsi forcé de tomber deux fois
sur le même lorsque l'on eectue la division. Ainsi toute fraction est un nombre dont la
partie décimale est périodique (ie au nal, une certaine séquence de chires se répète, ce
caractère répétitif ne commençant pas forcément dès la première décimale, loin de là).
25
Oui, il est plus facile de rajouter pour sur-représenter que d'enlever pour sous-représenter.

33
La réciproque est également vraie : tout nombre dont le développement décimal se
répète au bout d'un moment est en fait une fraction. Voyons comment on prouve cela sur
un exemple. On se donne le nombre 0 , 410 784 153 153 153 (la partie surlignée étant celle
qui se répètera) et on cherche une fraction qui lui soit égale. Si l'on appelle x ce nombre,
l'astuce consiste à calculer 1000x. En faisant ensuite la diérence, la partie répétitive va
s'éliminer. Bref, on a :
1000x = 410 , 784 153 153 153
− x = 0 , 410 784 153 153
999x = 410 , 373 216
De la dernière égalité écrite, on déduit :
410 , 373 216 410 373 216 34 197 773
x= = =
999 999 000 000 83 250 000
trouvant ainsi une fraction égale à notre nombre de départ. Bien évidemment, on comprend
comment cette méthode se généralise à tout nombre périodique.
Ainsi pour construire un nombre non rationnel, il sut de construire une partie décimale
qui ne soit pas périodique. Pour cela, on peut faire la construction suivante :
0 , 1 0 1 00 1 000 1 0000 1 00000 1 000000 1 0000000 1 00000000 1 000000000 . . .
Après le  0, , on met un 1 puis un 0, puis encore un 1 puis deux 0, puis encore un 1
puis trois 0, puis encore un 1 puis quatre 0 et ainsi de suite. J'arme que cette suite ainsi
construite n'est pas périodique.
On a une propriété  négative  à prouver ; faisons un raisonnement par l'absurde.
Supposons donc que ce nombre soit périodique, il s'écrirait donc 0, ABBBB... où A et B
sont deux suites de chires. En premier lieu, on constate que B ne peut pas être constante
égale à 0 : il y a des 1 aussi loin que l'on veut dans notre nombre. Mais alors, le nombre
0, ABBBB... ne pourra pas contenir plus de lg (A) + lg (B) zéros à la suite (lg (X) désigne
le nombre de chires de la suite de chires X ), ce qui n'est pas le cas de notre nombre.
Voici notre contradiction ! Et la conclusion s'ensuit : notre nombre est irrationnel.

On remarque que pour les mêmes raisons que celles exposées dans le raisonnement par
l'absurde précédent, le nombre univers que l'on a construit au tout début est forcément
irrationnel. Plus généralement d'ailleurs un nombre univers ne peut pas être rationnel.

Finalement c'est avec une construction tout à fait analogue à la précédente que Liouville
a exhibé un nombre transcendant. Un nombre transcendant est un nombre qui n'est racine
d'aucun polynôme à coecients entiers. On montre en fait que de telles racines sont soit
rationnelles, soit mal approchées (dans un sens à dénir) par les nombres rationnels ; il sut
donc d'imposer que le nombre transcendant que l'on veut construire ne soit pas périodique,
mais pourtant très proche (encore dans un sens à dénir) d'un nombre périodique. On
ne connaissait avant Liouville aucun exemple explicite de nombre transcendant. On sait
aujourd'hui que π et e en sont, mais les preuves sont franchement complexes comparées à
celle de Liouville.

6.3 Une fonction pour le moins étrange


Le but de ce paragraphe est de construire une fonction f : Q → Q qui soit telle que
l'image de tout intervalle ouvert ]a, b[ (avec a < b) soit Q tout entier. On rappelle, à tout
hasard, que Q désigne l'ensemble des nombres rationnels.

34
On voit qu'une telle fonction doit osciller, et on connaît sans doute des fonctions
¡1¢ qui
ont ce genre de propriétés. On pense peut-être dans un premier temps à sin x qui os-
cille beaucoup en 0. On pourrait se dire qu'en recollant des fonctions de ce genre un peu
partout, on devrait s'en sortir. Ce n'est sans doute eectivement pas désespéré, mais ce
n'est probablement pas la meilleure façon d'aborder le problème, d'autant plus que cela
va impliquer un nouveau problème bien plus dicile à résoudre : la fonction que l'on doit
construire doit prendre ses valeurs dans Q, et on sait que les sinus de nombres rationnels
n'en sont en général pas.

La réponse est encore une fois d'y aller franchement et de dénir exactement ce que
l'on veut. Que veut-on, donc ? On veut par exemple que le rationnel 1 ait un antécédent
dans chaque intervalle ouvert. Qu'à cela ne tienne, mettons-le. On choisit donc un point
dans chaque intervalle ouvert et on dénit f sur ces points en imposant à sa valeur d'égaler
1.
Bien évidemment, dit de la façon précédente, on ne comprend pas bien ce qu'il faut
faire. Plutôt que de choisir un point dans chaque intervalle ouvert26 , on va exhiber un sous-
ensemble de Q qui rencontre manifestement chaque intervalle ouvert. Bien sûr, on pourrait
prendre Q lui-même, mais il n'y aurait alors plus de place pour continuer et la fonction
serait constante égale à 1, ce qui n'est pas ce que l'on souhaite. Il faut donc trouver plus
petit ; et on peut prendre :
np o
?
A1 = , p impair, n ∈ N
2n
le fait que p soit impair n'est pas primordial maintenant, il assure simplement que la fraction
écrite ne peut pas se simplier. Cette condition nous sera utile par la suite.
On pose ensuite, comme annoncé, f (x) = 1 pour tout x ∈ A1 et on a déjà résolu le
problème pour la valeur 1.
Et maintenant, on continue et on traite la valeur 2. On considère un ensemble A2
intersectant encore tout intervalle ouvert et disjoint de A1 , on peut prendre :
np o
?
A2 = , p non multiple de 3, n ∈ N
3n
une puissance non nulle de 2 n'étant jamais une puissance non nulle de 3, et les fractions
ne pouvant se simplier. On dénit ensuite f sur A2 en posant f (x) = 2 pour tout x ∈ A2 .
Et maintenant, on passe à 3. Il faut trouver un ensemble A3 disjoint de A1 et A2 et
intersectant à nouveau tout intervalle ouvert. On prend :
np o
?
A3 = , p non multiple de 5, n ∈ N
5n
il faut faire attention au nombre 4 pour le dénominateur, les puissances de 4 étant mal-
heureusement des puissances de 2 particulières. On voit un premier problème, mais il est
facile à régler, il va sut d'interdire pour le dénominateur les nombres qui sont déjà des
puissances ; si l'on ne veut vraiment pas se tourmenter, on peut ne faire apparaître dans
les dénominateurs que les nombres premiers.
Notons donc pk le k -ième nombre premier et dénissons de façon générale :
½ ¾
p ?
Ak = , p non multiple de pk , n ∈ N
pnk
26
Ce genre de constructions est pourtant tout à fait possible et se révèle souvent très ecace ; ces idées
seront quelque peu exposées dans le paragraphe suivant.

35
On a ainsi toute une ribambelle d'ensembles qui intersectent tout intervalle ouvert, et qui
plus est disjoints deux à deux. Il ne reste donc plus qu'à dénir f comme précédemment.
Le problème qui se pose est maintenant le suivant : si on décide de poser f (x) = k pour
tout x ∈ Ak , f ne va prendre que des valeurs entières (strictement positives qui plus est) et
donc pas toutes les valeurs rationnelles. Il faut donc s'arranger pour lister tous les nombres
rtionnels, on dénira alors pour x ∈ Ak , f (x) comme la k -ième fraction de la liste.
Voyons donc comment l'on peut établir une telle liste. On se cantonne pour débuter
aux rationnels compris entre 0 et 1, disons. On voit alors ce que l'on peut faire : on met
d'abord les rationnels qui ont pour dénominateur 1, puis ceux qui ont pour dénominateur
2 et ainsi de suite. Ainsi la liste obtenue commencera par :
0 1 2 0 1 2 3 0 1 2 3 4 0
0, 1, , , , , , , , , , , , , , ...
2 2 2 3 3 3 3 4 4 4 4 4 5
Bien sûr dans la liste précédente, un même rationnel apparaît de nombreuses fois, on peut
s'amuser à éliminer les doublons si cela nous amuse. Le problème, c'est que l'on ne veut
pas seulement les rationnels compris entre 0 et 1 mais bien tous les rationnels. Qu'à cela
ne tienne, on met d'abord ceux qui sont compris entre −1 et 1 et qui ont un dénominateur
égal à 1, puis ceux qui sont compris entre −2 et 2 et qui ont un dénominateur égal à 2, et
ainsi de suite. Au nal :
4 3 2 1 0 1 2 3 4 9 8
−1, 0, 1, − , − , − , − , , , , , , − , − , . . .
2 2 2 2 2 2 2 2 2 3 3
Quelle que soit la façon retenue, appelons xk le k -ième rationnel. On peut maintenant
dénir notre fonction f en posant :
½
f (x) = xk si x ∈ Ak
f (x) = 0 sinon

Elle convient évidemment.

6.4 Le principe du va-et-vient


Nous allons expliquer dans ce paragraphe, sur un exemple pas forcément simple, com-
ment il est possible de combiner les idées précédentes avec la puissance du raisonnement
par récurrence.
¡√ ¢
Fixons-nous donc un problème. On désigne par Q 2 l'ensemble suivant :
³√ ´ n √ o
Q 2 = a + b 2, a, b ∈ Q

C'est un sous-ensemble de R. La question consiste à¡√


construire
¢ une fonction f : R → R
strictement croissante envoyant Q exactement sur Q 2 .

Là encore, essayer de trifouiller une formule ne va certainement pas faire apparaître


miraculeusement la solution.
¡√ ¢ Il faut littéralement construire cette fonction à la main. On
veut envoyer Q sur Q 2 , faisons-le. On a vu précédemment que l'on pouvait numéroter
les rationnels, nous n'allons pas nous en priver. Appelons xk le k -ième nombre rationnel
(on commence à k = 0 à partir de maintenant, allez hop), pour une liste que l'on s'est xée
à l'avance.

36
¡√ ¢
Il va nous falloir en outre, numéroter les éléments de Q 2 si l'on veut continuer. On
écrit pour ce faire la liste suivante :
√ √ √ √ √ √ √
x0 + x0 2, x0 + x1 2, x1 + x0 2, x0 + x2 2, x1 + x1 2, x2 + x0 2, x0 + x3 2, . . .

On met d'abord les xi + xj 2 pour i + j = 0, puis ensuite ceux pour i + j = 1 et ainsi
de suite. Comme cela, on les obtient bien tous. Dans√la liste précédente, il n'y a pas de
doublons ; c'est une conséquence de l'irrationalité de 2. Cela dit, qu'il y en est ou pas,
n'est pas le problème, on peut décréter qu'on les élimine. Bref, appelons yk le k -ième item
de cette liste.
On
¡√a donc
¢ d'une part une liste des éléments de Q et d'autre part une liste des éléments
de Q 2 . Il est tentant de débuter la dénition de f en posant f (xk ) = yk . Cependant,
cela ne va pas marcher : rien n'est là pour assurer la croissance. Il faut donc faire plus
attention et c'est là qu'intervient la récurrence.

Nous n'allons pas écrire la démonstration rigoureuse et implacable, mais plutôt donner
les idées principales susamment détaillées toutefois et laisser au lecteur le soin de mettre
tout cela au propre s'il souhaite le faire.
On commence par x0 , on veut lui trouver une image. Il n'y a pour l'instant aucune
contrainte, on prend donc naturellement y0 .
On passe à x1 . Il y a cette fois-ci deux cas : soit il est plus grand, soit il est plus petit
que x0 . S'il est plus grand, il faut choisir un yn parmi ceux qui sont plus grands que y0 ; s'il
est plus petit, il faudra
¡√ ¢ en choisir un parmi les plus petits. De toute évidence, de tels yn
existent puisque Q 2  va jusqu'à l'inni  des deux côtés. Disons, pour xer les idées,
que l'on choisit le plus petit indice n tel que yn soit comme on le veut.
Au suivant ! C'est x2 . Il y a maintenant trois cas : soit il est plus petit que le plus petit
de x0 et x1 , soit il est plus grand que le plus grand, soit il est compris entre les deux.
Bref, il est situé quelque part par rapport aux autres : x0 et x1 . Pour chacun de ces cas,
il y a un yn correspondant et encore, pour xer les choses, on choisit le plus petit indice n
convenable.
On continue ainsi. Si on a déni les images de x0 , . . . , xn , on regarde où se situe xn+1
entre tous ces nombres et on lui associe le yn qui va bien.
C'est tout beau, tout mignon, mais ça ne marche pas. Rien n'assure que l'on va tomber
ainsi sur tous les yn et rien ne dit qu'au nal, on ne se retrouve pas avec la fonction
identité. Il faut bien quelque chose pour tenir compte de ces yn . L'idée est alors de faire la
construction dans les deux sens (d'où le nom de va-et-vient ).

Reprenons donc. On commence toujours par x0 et on lui associe toujours y0 . On continue


par x1 comme c'était déjà le cas dans la construction précédente.
Mais maintenant on ne considère pas x2 mais plutôt y1 et on lui cherche un antécédent
s'il n'en a pas déjà. Le nombre y1 se situe d'une certaine façon par rapport aux éléments
qui sont déjà dans l'image de f , on lui choisit son antécédent en conséquence.
Seulement maintenant, on s'occupe de x2 . Soit il a déjà été choisi à l'étape précédente,
et on passe. Soit ce n'est pas le cas, et on le situe non pas par rapport à x0 et x1 mais par
rapport aux éléments qui ont déjà une image par f , c'est-à-dire x0 , x1 et éventuellement
un antécédent de y1 choisi précédemment. Une fois cela fait, on choisit une image pour x2 .

37
Ensuite, on s'occupe de y2 , puis
¡√de¢ x3 , de y3 , 27
de x4 , et ainsi de suite. Tout cela nous
donne une fonction f : Q → Q 2 surjective et strictement croissante. Il reste à
prolonger ce début de fonction à R tout entier.

Mais, si l'on regarde un petit moment dans les yeux la fonction que l'on vient de
construire, on voit qu'on n'a pas le choix. Soit x un réel, disons irrationnel. Par croissance,
pour tout rationnel r < x, on doit avoir f (r) < f (x), et pour ¡√ tout¢ rationnel r > x, on
doit avoir f (r) > f (x). Mais l'ensemble de tous les f (r) est Q 2 donc les conditions
précédentes déterminent au plus un réel et en fait exactement un réel. C'est ainsi que l'on
achève notre construction.
¡√ ¢
Bien entendu, Q et Q 2 étaient des exemples arbitraires, on aurait en fait pu les
remplacer par n'importe quel partie A de R dense (ie entre deux réels quelconques, il y a
toujours un élément de A), dénombrable (ie on peut numéroter les éléments) et  allant
à l'inni  ou plutôt sans extrémités comme on préfère dire (ie pout tout réel positif M ,
il y a dans A un élément plus grand que M et un plus petit que −M ). La démonstration
précédente s'appliquait alors point par point.
¡√ ¢
Ainsi, on aurait pu remplacer Q 2 par l'ensemble des nombres algébriques28 , ce qui
doit paraître peut-être encore plus étonnant.

27 ą√ ć
Celasignie que tout élément de Q 2 admet au moins un antécédent par f .
28
Ie racine d'un polynôme à coecients entiers  voir paragraphe 6.2.

38
Combinatoire énumérative
Igor Kortchemski

- Introduction -

La Combinatoire est un sous-art des mathématiques qui consiste à compter et à étudier


des structures finies. De nombreux problèmes difficiles sont formulés de manière très simple
(mais la résolution nécessite des outils avancés). Le but de ce cours est de présenter quelques
réflexes et idées de bases pouvant être utiles dans la résolution d’exercices de combinatoire
de type olympiades.
On parlera de coefficients binomiaux, double-comptage, injections, surjections, bijections.

1 Coefficients binomiaux
1.1 Définitions
On rappelle qu’un ensemble E est une collection d’éléments dont l’ordre n’a pas d’impor-
tance (ainsi, les ensembles {2, 3} et {3, 2} sont les mêmes ensembles) et comptés sans multipli-
cité (par exemple, 2,2=2). On note x ∈ A si x appartient à l’ensemble A. Si A et B sont deux
ensemble, on écrit A ⊂ B et on dit que A est inclus dans B si chaque élément de A appartient à
B. L’ensemble vide, qui ne contient aucun élément, est noté ∅. On note Card(A) (on prononce
« cardinal de A ») le nombre d’éléments de A. On dit que A est infini si Card(A) = ∞, fini
sinon.
Définition 1. Pour des entiers 0 6 k 6 n, on note n

k
(et on prononce « k parmi n ») le nombre
de manières de choisir un sous-ensemble à k éléments d’un ensemble à n éléments différents.
Pour k > n, on pose n k
= 0.
Il est clair que dans la définition précédente, n

k
ne dépend pas de l’ensemble à n éléments
différents considéré.
Exemple 2. On a 42 = 6, car les sous-ensembles à 2 éléments de {1, 2, 3, 4} sont {1, 2}, {1, 3},


{1, 4}, {2, 3}, {3, 4} et il y en a 6


Pour un entier n > 1, rappelons la notation n! = 1 × 2 × 3 × · · · × (n − 1) × n. On lit "n
factorielle".
Proposition 3. Le nombre de manières d’ordonner n éléments est n!.
Démonstration. Nous avons n possibilités pour choisir le premier élément, n − 1 possibilités
pour le deuxième, et ainsi de suite jusqu’au dernier élément pour lequel nous avons une seule
possibilité. Le résultat en découle.

1
Proposition 4. Pour des entiers 0 6 k 6 n, on a :
 
n n!
= .
k (n − k)!k!

Démonstration. Utilisons le principe de double-comptage, très important en combinatoire. Il


consiste à établir une égalité en comptant de deux manières différentes une certaines quan-
tité. Ici, comptons le nombre de suites à k éléments qu’on peut créer en utilisant les n éléments
d’un ensemble à n éléments différents.
D’une part, comme pour la proposition précédente, nous avons n choix pour le premier
terme de la suite ; n−1 choix pour le deuxième, et ainsi de suite jusqu’au k-ième élément pour
n!
lequel nous avons n−k+1 choix. Finalement, il y a en tout n×(n−1)×· · ·×(n−k+1) = (n−k)!
suites à k éléments.
D’autre part, pour créer une suite à k éléments, on peut commencer par choisir les k élé-
ments qui vont constituer la suite ( nk
possibilités), puis les ordonner (k! manières possibles
de les ordonner). Il y a donc en tout k! · n

k
suites à k éléments.

1.2 Propriétés combinatoires

Exercice 1 Pour des entiers 0 6 k 6 n, on a :


   
n n
=
k n−k
n
 n!
Solution de l’exercice 1 Première méthode : On utilise la formule k
= (n−k)!k!
et le résultat
en découle immédiatement.
Deuxième méthode : On remarque que choisir k éléments parmi n revient à sélectionner les
n − k éléments qu’on ne choisira pas.
L’exercice précédent, bien que facile, est assez représentatif des exercices ayant pour but
de prouver des relations d’égalité entre coefficients binomiaux. Très souvent, il y a toujours
(au moins) deux approches possibles : remplacer les coefficients binomiaux par leur formule
et ramener le problème à un exercice de manipulation de relations algébriques, ou bien inter-
préter de manière combinatoire les deux termes de part et d’autre de l’égalité et prouver qu’ils
sont égaux. La deuxième approche est bien sûr bien plus élégante et fournit très souvent des
preuves courtes, mais requiert davantage d’ingéniosité.

Proposition 5. (formule de Pascal) Soient n et 0 6 k 6 n des entiers (avec (k, n) 6= (0, 0)).
Alors :      
n n−1 n−1
= + .
k k k−1

2
Démonstration. Première méthode : On utilise la formule :
   
n−1 n−1 (n − 1)! (n − 1)!
+ = +
k k−1 k!(n − 1 − k)! (k − 1)!(n − k)!
 
(n − 1)! 1 1
= +
(k − 1)!(n − 1 − k)! k n − k
 
(n − 1)! n
=
(k − 1)!(n − 1 − k)! k(n − k)
n!
=
k!(n − k)!

Seconde méthode : Démontrons ce résultat de manière combinatoire. Considérons l’ensemble


{1, 2, . . . , n} et dénombrons ses sous-ensembles à k éléments. On distingue les ensembles qui
contiennent l’élément n et les ensembles qui ne le contiennent pas. Il y a n−1 k
ensembles qui
ne contiennent pas n (on choisit k éléments dans {1, 2, . . . , n} et il y en a k−1 qui contiennent
n−1

n. Au total on a donc n−1 n−1


 
k
+ k−1
manières de choisir nos k éléments parmi nos n entiers,
d’où le résultat.

La formule de Pascal nous permet ensuite de construire le triangle de Pascal, que vous
connaissez peut-être déjà. La case située dans la k-ième colonne de la n-ième ligne contient le
coefficient binomial n−1
k−1
. D’après la formule de Pascal, on obtient donc chaque case comme
la somme des deux cases qui sont au-dessus.

1
1 1
1 2 1
1 3 3 1
1 4 6 4 1
1 5 10 10 5 1
1 6 15 20 15 6 1
1 7 21 35 35 21 7 1
On constate qu’il y a un lien entre la n-ième ligne du triangle de Pascal et le développement
de (x + y)n :

Proposition 6 (Formule du binôme de Newton). Soient x, y des nombres réels et n > 1 un


entier. Alors :
Xn  
n k n−k
x y = (x + y)n .
k=0
k

Notation. Avant de démontrer cette formule, un petit rappel de notation :

3
Lorsqu’on veut faire une grosse somme de termes qui peuvent s’exprimer selon un entier,
souvent noté
Pk (ou i mais cela n’a pas d’importance), qui varie entre deux valeurs, on utilise
le symbole . Par exemple :

X
n
1 = 1 + 1 + ... + 1 = n + 1
k=0

X
n
n(n + 1)
k = 0 + 1 + 2 + ... + n =
k=0
2
Xn  
n
   
n n

n
  
n
= + + ... + +
k=0
k 0 1 n−1 n
Q
Pour un produit on utilise de la même manière le symbole . Par exemple :

Y
n
k = n!
k=1

Démonstration. Première méthode : par récurrence sur n (s’entraîner à le faire).


Seconde méthode : lorsqu’on développe (x + y) · (x + y) · · · (x + y), pour trouver le coefficient
devant xk yn−k , parmi les n termes (x + y), il faut en choisir k pour lesquels on garde le x et
qui vont donner un terme xk , et les n − k autres termes pour lesquels on sélectionne y (et qui
sont fixés par le choix des k premiers) vont donner le terme yn−k . Le résultat s’ensuit.
Exercice 2 Pour tout entier n > 1, on a
X
n  
n
= 2n .
k=0
k

Exercice 3 Pour 0 6 k 6 n, on a :
   
n n−1
k =n .
k k−1

Exercice 4 Quel est le cardinal moyen d’un sous-ensemble de {1, 2, . . . , n} ?


Exercice 5 Prouver que pour 0 6 m 6 n :

X
n   
n k
 
n n−m
= 2 .
k=0
k m m

Exercice 6 Combien y a-t-il de chemins sur Z2 issus de (0, 0), faisant des pas +(1, 0) ou +(0, 1),
et finissant en (m, n), où m, n > 0 ?
Exercice 7 De combien de manières peut-on placer 5 pièces identiques dans 3 poches diffé-
rentes ?

4
Solution de l’exercice 2 Première méthode : Si on n’a pas d’idée comment commencer de ma-
nière astucieuse, on peut essayer de procéder par récurrence sur n. Pour n = 1, le résultat est
clair. Supposons le résultat acquis au rang n et montrons-le au rang n + 1 en écrivant, avec la
formule de Pascal :
X n + 1
n+1 X n  n 
n+1
= 1+ +
k=0
k k=1
k k−1
Xn  
n Xn  
n
= +
k=1
k k=0
k
= 2n + 2n (par hypothèse de récurrence)
= 2n+1 ,
Seconde méthode : Démontrons ce résultat de manière combinatoire en comptant le nombre
N de sous-ensembles de {1, 2, . . . , n}. D’une part, pour un entier n

Pn 0 6 k 6 n, il y a k
sous-
n

ensembles à k éléments. En sommant le tout, on voit que N = k=0 k .
Mais, pour construire un sous-ensemble de {1, 2, . . . , n}, on a le choix de choisir 1 ou non,
2 ou non, et ainsi de suite jusqu’à n. On a donc n choix à faire entre deux possibilités et ces
choix sont indépendants (le choix de prendre k ou non n’influence pas le choix de prendre k 0
ou non si k 6= k 0 ), d’où N = 2n , ce qui permet de conclure.
En particulier, la solution précédente montre qu’il existe 2n sous-ensembles d’un ensemble
à n éléments.
Troisième méthode : On utilise la formule du binôme de Newton avec x = y = 1.
Solution de l’exercice 3 Première méthode : On utilise la formule exprimant n

k
(on vous laisse
le faire).
Seconde méthode : Démontrons ce résultat de manière combinatoire en comptant de deux
manières différentes le nombre de sous-ensembles de {1, 2, . . . , n} de cardinal k ayant un élé-
ment distingué (qu’on appellera chef).
n

D’une part, il suffit de choisir un sous-ensemble de cardinal k ( k
choix), puis de choisir
n

un chef (k choix indépendants). On obtient donc en tout k k possibilités.
D’autre part, on peut d’abord choisir un chef (n choix) puis compléter les k − 1 éléments à
choisir dans {1, 2, . . . , n} privé du chef (donc un ensemble à n − 1 éléments). Il y a alors n n−1
k−1
éléments.
Solution de l’exercice 4
Première méthode (d’après une idée d’élèves) : On regroupe un ensemble avec son complémen-
taire. Si A ⊂ {1, 2, . . . , n}, on note Ac l’ensemble des entiers de {1, 2, . . . , n} qui ne sont pas dans
A. Notons N ce cardinal moyen. Alors :
1 X 1 X CardA + Card(Ac )
N = n Card(A) = n
2 2 2
A⊂{1,2,...,n} A⊂{1,2,...,n}
1 X n 1 n n
= = n · 2n = .
2n 2 2 2 2
A⊂{1,2,...,n}

Autre méthodes : Il faut évaluer la somme


1 X n
n  
Sn = n k .
2 k=0 k

5
Si on ne sait pas commencer, il faut étudier les premiers cas : n = 1, 2, . . .. On trouve toujours
Sn = n/2. Essayer donc de démontrer cela.
Seconde méthode : Si on n’a pas d’idée, on peut procéder par récurrence sur n.
Troisième méthode,
Pplus avancée. On utilise le résultat de la proposition 6. Considérons le
n n k

polynôme Pn (x) = k=0 k x . D’après la formule du binôme de Newton, Pn (x) = (1 + x)n .
Dérivons cette égalité par rapport à x :
Xn  
n k−1
k x = n(1 + x)n−1 .
k=0
k

Évaluons alors cette quantité en x = 1 :


Xn  
n
k = n2n−1 .
k=0
k
Le résultat en découle.
Solution de l’exercice 5 On remarque d’abord que seuls les k tels que k > m contribuent
de manière non nulle. On va procéder à un double comptage en comptant le nombre N de
sous-ensembles A, B de {1, 2, . . . , n} tels que A ⊂ B et Card(A)= m. En effet, d’une part, pour
n
construire A, B on peut d’abord choisir A de cardinal m ( m choix), puis rajouter un sous-
ensemble quelconque de l’ensemble {1, 2, . . . , n} privédes éléments de A, qui a n−m éléments
n
(et donc 2n−m choix indépendants). En ainsi, N = m 2n−m .
D’autre part, pour construireA, B on peut d’abord choisir B de cardinal quelconque entre
m et n (si B est de cardinal k, n choix), puis choisir A de cardinal m comme sous-ensemble
k P
de B ( m choix si B est de cardinal k). Ainsi, N = n
k n
  k
k=0 k m .
Solution de l’exercice 6 Un tel chemin doit faire m+n pas, dont m fois +(1, 0) et n fois +(0, 1).
Il suffit donc de choisir parmi les m + n pas possibles la position des m qui font +(1, 0). Le
nombre total vaut donc m+n m
.
Solution de l’exercice 7 Considérons la figure suivante (avec deux barres) :

Remplaçons chaque rond soit par une pièce, soit par une barre de sorte qu’il y ait en tout 2
barres et 5 pièces. Les pièces entre les deux premières barres seront contenues dans la première
poche, les pièces entre la deuxième barre et la troisième barre seront contenues dans la seconde
poche, et finalement les pièces entre la troisième barre et la quatrième barre seront contenues
dans la troisième poche.
Ainsi, placer 5 pièces identiques dans 3 poches différentes,
 revient à choisir la position des
2 barres parmi 7 positions possibles. La réponse est donc 72 = 21.
Plus généralement, en procédant de la même façon, on voit qu’il y a a+b−1 = a+b−1
 
b−1 a
manières de placer a pièces identiques dans b poches différentes.

6
2 Principe d’Inclusion-Exclusion
On sait que si A et B sont deux ensembles finis, alors Card(A ∪ B) = Card(A) + Card(B) −
Card(A ∩ B). La formule suivante, dite d’inclusion-exclusion, généralise cela au cas où nous
en avons un nombre quelconque.

Proposition 7 (Formule d’inclusion-exclusion). Si A1 , . . . , An sont des ensembles finis, alors :

X
n X
Card(A1 ∪ A2 ∪ · · · ∪ An ) = (−1)k+1 Card (Ai1 ∩ · · · ∩ Aik ) .
k=1 16i1 <···<ik 6n

Démonstration. Par récurrence (bon courage !)

Exercice 8 Combien y a-t-il d’entiers strictement positifs inférieurs ou égaux à 120 et qui ne
sont divisibles ni par 3, ni par 5, ni par 7 ?
Exercice 9 Les n stagiaires au stage de Montpellier vont se baigner et laissent leurs t-shirts
Animath en vrac sur le sable. Ils reviennent et prennent un t-shirt complètement au hasard.
Quelle est la probabilité que personne ne se retrouve avec son t-shirt ?
Solution de l’exercice 8 Trouvons plutôt le nombre d’entiers strictement positifs inférieurs
ou égaux à 120 divisibles par 3, 5, ou 7. Notons A (respectivement B et C) l’ensemble des
nombres entiers strictement positifs divisibles par 3 (respectivement 5 ou 7). On a :
 
120
|A| = = 40 (1)
3
 
120
|B| = = 24 (2)
5
 
120
|C| = = 17 (3)
7
 
120
|A ∩ B| = =8 (4)
3×5
 
120
|B ∩ C| = =3 (5)
5×7
 
120
|C ∩ A| = =4 (6)
7×3
 
120
|A ∩ B ∩ C| = =1 (7)
3×5×7

D’où d’après la formule d’inclusion-exclusion,

|A ∪ B ∪ C| = 40 + 24 + 17 − 8 − 5 − 3 + 1 = 66.

Ainsi, le nombre cherché vaut 120 − 66 = 54.


Solution de l’exercice 9 On assigne à chaque stagiaire un chiffre différent entre 1 et n, et on
note xi le numéro de l’élève prenant le i-ième t-shirt. Ainsi, (x1 , . . . , xn ) est une permutation

7
de (1, . . . , n). Calculons plutôt la probabilité qu’au moins une personne retrouve son t-shirt
en vue d’utiliser le principe d’inclusion-exclusion. Pour 1 6 i 6 n, soit Ai l’ensemble des
permutations telles que xi = i. Il est clair que pour 1 6 i1 < · · · < ik 6 n, on a :

Card (Ai1 ∪ · · · ∪ Aik ) = (n − k)!.

Ainsi, d’après la formule d’inclusion-exclusion :

X
n X
Card(A1 ∪ A2 ∪ · · · ∪ An ) = (−1)k+1 Card (Ai1 ∩ · · · ∩ Aik )
k=1 16i1 <···<ik 6n
Xn X
= (−1)k+1 (n − k)!
k=1 16i1 <···<ik 6n
Xn  
n
k+1
= (−1) (n − k)!
k=1
k
Xn
n!
= (−1)k+1 .
k=1
k!

La probabilité cherchée vaut 1 − Card(A1 ∪ A2 ∪ · · · ∪ An )/n!, c’est-à-dire :

1 1 1 1
− + + · · · + (−1)n .
2! 3! 4! n!
1
Lorsque n tend vers l’infini, cette probabilité tend vers e
≈ 37%.

3 Injections, surjections, bijections


Définition 8. Une application f : E → F permet d’associer à tout élément x de l’ensemble E un
unique élément, noté f(x) de l’ensemble F.
On dit que E est l’ensemble de départ, F l’ensemble d’arrivée. Si f(x) = y, on dit que x est
l’antécédent de y et y l’image de x.

3.1 Injections et surjections


Définition 9. Soient E, F deux ensembles et f : E → F une application.
(i) On dit que f est injective si pour tous x, y ∈ E avec x 6= y, f(x) 6= f(y).
(ii) On dit que f est surjective si pour tout y ∈ F, il existe x ∈ E tel que f(x) = y.

Pour montrer que f : E → F est injective, on montre très souvent que si x, y ∈ E sont tels
que f(x) = f(y), alors x = y (voir le cours sur les équations fonctionnelles).
On introduit la notation [n] = {1, 2, . . . , n} pour un entier n > 1.

Proposition 10. Il existe une injection [m] → [n] si, et seulement si, m 6 n. Il existe une
surjection de [m] → [n] si, et seulement si, m > n.

8
Démonstration. Exercice.
En pratique, on utilise la proposition précédente en combinatoire comme suit : pour mon-
trer que a 6 b, on construit deux ensembles A, B tels que Card(A) = a, Card(B) = b, ainsi
qu’une injection de A dans B. Ou encore, pour montrer que a > b, on construit deux en-
sembles A, B tels que Card(A) = a, Card(B) = b, ainsi qu’une surjection de A dans B.
Exercice 10 (Olympiades Balkaniques de Mathématiques 1997) Soient m, n > 1 des entiers.
Soit S un ensemble de cardinal n et A1 , A2 , . . . , Am des sous-ensembles de S. On suppose que
pour tous éléments x 6= y de S, il existe 1 6 i 6 m tel que x ∈ Ai et y 6∈ Ai , ou bien x 6∈ Ai et
y ∈ Ai . Prouver que n 6 2m .
Exercice 11
(i) Combien existe-t-il de fonctions de [m] → [n] ?
(i) On suppose m 6 n. Combien existe-t-il d’injections de [m] → [n] ?
(ii) On suppose m > n. Combien existe-t-il de surjections de [m] → [n] ?
Solution de l’exercice 10 À tout élément x ∈ S, on associe le m-uplet (x1 , . . . , xm ) où xi = 0
si x 6∈ Ai et xi = 1 si x ∈ Ai . Cette application est définie sur S, et son ensemble d’arrivée
est {0, 1}m . Par hypothèse, si x 6= y, alors f(x) 6= f(y). Ainsi, f est injective. Le cardinal de
l’ensemble de départ est donc inférieur ou égal au cardinal de l’ensemble d’arrivée.
Solution de l’exercice 11 Pour (i), il y en a clairement nm (n choix pour chacun des m entiers
au départ)
Pour (ii), on a n choix pour l’image de 1, n − 1 choix pour l’image de 2, et ainsi de suite
n!
jusqu’à m pour lequel on a n − m + 1 choix pour son image. La réponse est donc (n−m)! .
Pour (iii), on va utiliser le principe d’inclusion-exclusion et compter le nombre de fonctions
[m] → [n] qui ne sont par surjectives. À cet effet, pour 1 6 i 6 n, notons Ai l’ensemble des
fonctions [m] → [n] telles que i n’est pas atteint par la fonction. Il est clair que pour des entiers
1 6 i1 < · · · < ik 6 n, on a Card(Ai1 ∩ · · · Aik ) = (n − k)m , car chaque élément de [m] peut
être envoyé sur un des n − k entiers de [n] autorisés. Ainsi, d’après le principe d’inclusion
exclusion, si on note s(m, n) le nombre de surjections de [m] → [n], on a :

nm − s(m, n) = Card(A1 ∪ A2 ∪ · · · ∪ An )
Xn X
= (−1)k+1 Card (Ai1 ∩ · · · ∩ Aik )
k=1 16i1 <···<ik 6n
Xn X
= (−1)k+1 (n − k)m
k=1 16i1 <···<ik 6n
Xn  
k+1 n
= (−1) (n − k)m
k=1
k

Ainsi,
Xn  
k n
s(m, n) = (−1) (n − k)m .
k=0
k

9
3.2 Preuves par bijections en combinatoire

Définition 11. Soient E, F deux ensembles et f : E → F une application. On dit que f est bijective
si elle est à la fois injective et à la fois surjective.
Proposition 12. Soient A et B deux ensembles finis. Alors A et B ont même cardinal si, et
seulement si, il existe une bijection entre A et B
Démonstration. Exercice

En combinatoire, cette proposition est souvent utilisée de la manière suivante. Si on veut


montrer que a = b, où a, b > 0 sont des entiers, il suffit de trouver deux ensembles finis A et
B tels que Card(A) = a et Card(B) = b, et de construire une bijection entre A et B.
Pour vérifier qu’une fonction est bijective, il est parfois pratique d’exhiber la fonction réci-
proque. Plus précisément :
Proposition 13. Si f : A → B et g : B → A sont deux fonctions telles que f(g(b)) = b pour
tout b ∈ B et g(f(a)) = a pour tout a ∈ A, alors f et g sont des bijections (on dit qu’elles sont
réciproques, ou inverses, l’une de l’autre).
Démonstration. Montrons d’abord que f est surjective. Soit b ∈ B. On sait que f(g(b)) = b,
ainsi g(b) est un antécédent de b, de sorte que f est surjective. Pour montrer l’injectivité,
soient x, y ∈ E tels que f(x) = f(y) et montrons que x = y. On a x = g(f(x)) = g(f(y)) = y. La
fonction f est donc injective, et comme elle est surjective, elle est bien bijective. Par symétrie,
g est aussi bijective.

Exercice 12 (Canada 2005) Soit un triangle équilatéral dont le côté est de longueur n, divisé en
triangles unitaires tel qu’illustré. Soit f(n) le nombre de chemins allant du triangle de la rangée
du haut jusq’au triangle au centre de la rangée du bas, de façon à ce que des triangles adjacents
partagent une arête commune et que le chemin ne repasse jamais par le même triangle et qu’il
n’aille jamais vers le haut (d’une rangée inférieure à une rangée supérieure). Un tel chemin
est illustré ci-après avec n = 5. Déterminer la valeur de f(2012).

Solution de l’exercice 12 L’application qui à un chemin associe un (n − 1)-uplet (x1 , . . . , xn−1 )


où xi est l’entier x tel que le chemin traverse la i-ième ligne horizontale en partant du haut au
x-ième segment en partant de la gauche. Cette application est clairement une bijection entre
l’ensemble des chemins considérés et l’ensemble [1] × [2] × · · · × [n − 1], qui est de cardinal
(n − 1)!. Ainsi, f(2012) = 2011!.

10

 
  

!#"%$'&)(#*,+.-0/1"2&)3547641(98%&;:2<=-041(

>@?BADCFEHG
IJ?LKNMPOQILRSRUT

V ?SW

X GYCZW WC\[^]`_0abLced0fhg5a

+i:%j0k5476041(mln:poF3qe&;41(

r s'tvuxwQy{z|tu|}~U€‚y{€;~S} ƒ
„†…„ ‡‰ˆ ŠŒ‹ …
……
…'……
……
……
…
……'……
……
…
……
……'…
……
……
……
…'……
……
……
…
… Ž
„†…Ž  ‘ ˆ5’ L“ ”‹5 •‘ ……
……
…
……'……
……
…
……
……'…
……
……
……
…'……
……
……
…
… –
„†…— ˜ š™ ’›……
…'……
……
……
…
……'……
……
…
……
……'…
……
……
……
…'……
……
……
…
… œ
„†… – ‡‰ˆ Š
Œ  ‘%ž ‡‰ˆ Š ŸZ ˆ Ÿ5 …
……
……'…
……
……
……
…'……
……
……
…
…  
„†…œ ¡ ™ ˆ ‘ŒŒ‹ ……
…'……
……
……
…
……'……
……
…
……
……'…
……
……
……
…'……
……
……
…
… ¢

ƒ £itv¤¥e¦Uu|}¥e§¨¤~U¤y{tvu}0©«ª¬€Zt w®­U§{u°¯H±³²´­N§¨ut rµ


Ž…„ ¡ ™ ˆ ‘ŒŒ‹ ……
…'……
……
……
…
……'……
……
…
……
……'…
……
……
……
…'……
……
……
…
…¶„·Ž

¸ ¹ €;§{€Zt y¨¤ºFu|}
' r¸
—…„ » '¼ ˆ 'Š ˆ Ÿ| …
……
…
……'……
……
…
……
……'…
……
……
……
…'……
……
……
…
…¶„·—
—…Ž ½ “  ˆ ŸŠ•†  5  …
……
…
……'……
……
…
……
……'…
……
……
……
…'……
……
……
…
…¶„‘–
—…— ½ )  x¾ •
5Š ˆ Ÿ'L“ ¿ ˆ Š …
……
……'…
……
……
……
…'……
……
……
…
…¶„·œ
—… – ˜  ˆ ÀŒ‹‰L“ ˆ5Á 5Œ‹ ……
……
…
……'……
……
…
……
……'…
……
……
……
…'……
……
……
…
…¶„‚Â
—…œ à | ˆ x‘Ÿ  ™i ˆ ¼ đ•Œ‹hš™ ˆ5’ ™ …
……
……'…
……
……
……
…'……
……
……
…
…¶„‚Â
—…Å ¡ ™ ˆ ‘ŒŒ‹ ……
…'……
……
……
…
……'……
……
…
……
……'…
……
……
……
…'……
……
……
…
…¶„· 

Æ s‰¤tÇ€F­Utv}}v­StÈ­U~#ºZt¤¥N¦Uu ƒZµ
–…„ ˜ Š‘‹É‘ ’‘ˆ ‘‹ …
……
……
…
……'……
……
…
……
……'…
……
……
……
…'……
……
……
…
…,Ž†Ê
–…Ž ˜ Š‘‹%ŠZ 5 ‘‹ ……
…
……'……
……
…
……
……'…
……
……
……
…'……
……
……
…
…,Ž†Ž
–…— ¡ ™ ˆ ‘ŒŒ‹ ……
…'……
……
……
…
……'……
……
…
……
……'…
……
……
……
…'……
……
……
…
…,Ž†—

Ë ÌL€F§Í­S‚y{€F~U} ƒÆ

„
˜ %|F‹ŸH´)  ˆ ¼Ÿ‘@%  ˆ |‘ Œ‹@ ’ Ÿ ‹HZŒ‹H% À´ŸŠ ’ ˆ hŒ‹@ ˆ ŠŒ‹
‘Pp ’ | ˆ ‘ Œ‹ ˆ5’ ‹Ÿ 5‹  •‘|™ …
p‹5  ˆ  ‹5 ˆ ´ pZ ‹P À ˆ †‘ ˆ
‹ ‹% 5‹Ÿ ‹5
 Y“ š™; †‹ ’†…
½ p Œ‘5‘ ˆ  ’ ‹Ÿ ˆ ‘™L“ ‘•‹  ˆ  ‹e)  ˆŸˆ ‰Ÿ ‘•‘| ‹5 ˆ ‘; ˆ 5 ˆ ™ ˆ5’  ’‘ˆ ‘ŒŒ‹H  ’ Œ‹
 À   …

  &;4Pln3qe&;41(0"poF3"(

 !#"%$
˜ ‹Ÿ ’‘ˆ ‹% Œ‹%‘™¿‹ŸŸZŸ ‹h‹Ÿ&†|5Œ‹('

 )  ‹P*  ,+‹52 ˆ -‘|  ˆ ‹H‹5 ‹#.0/213/54m‘@ ˆ ‹@‹ŸŒ‹6+·+||7+ ’ ’ ‘ ˆ  ‘ ’†…


˜ ŠZ 0‹5 ^  Á  ˆ  ˆ ‘  ’   5ŠZ =‹ŸZ ˆ =Œ 5 …8
 • ˆ  ˆ5’  ‹5 ˆ ŒŒ‹
ˆ  ŒŒ ˆ ‘•‘5‹h‹ ‹%|'‘™iŒ 5Œ‹É
‹5 ˆ ‹5‘ …

¼9)
:Y¼ ˆ    ˆ x5‹‰5ŠŒ‹;+F ‰Œ ˆ ‹6+ZZ ˆ Z ˆ Œ‹;+';‘-‘ Á  ˆ ' ˆ ‹
) ˆ  ’ ‹‹´® Á •·  ‹‹ ˆ ‹5=L“ š™x  †‹Ÿ  …S˜ Z‹‹|'Œ‘™®Œ‹´ ˆ 5‹

;‘-‘ɝŒ ŠZ¼5 ˆ +Z Y“ ¼ŸŒ‘Ÿ<PŒ‹Ÿ% ˆ5’  ‹5 ˆ  ˆ ‹Ÿ) ˆ ɋ‹ ˆ ‹|‘h”•  ˆ 'Œ >=
?  …  …  … ‘¿Ÿ ‹|É • ‹%
· ‹É; †‹5‹5¼ 6) …

à BŒ‹ŸÉ ‹‹5@7ZŸ ˆ ‘ SL“ Œ‹5‹·  ˆ AY ˆ ¿Œ‹5‹ŸB'


#CU  ˆ Y“ š™‘ ')+ ¿‹Ÿ x'¼) ‹5ŠZ |‹ ¿Z ˆ  ˆ ³™i‘h5ŠZ ´‹ŸZ ˆ 
ˆ  …
B‘  ˆ ‘  ˆ 5ŠZ ȝ ˆ ³™  5ŠZ  ˆ  Z ˆ Œ‹ ˆ Œ‹0•Œ  ˆ ¼;Œ‹|@‹5
ˆ ‘Œ  ˆ ‘|‰Z ‹ ? ‘  +Z ‹‰  ‘ ˆ ‹ÉB“™iš™ ˆ5’  ’ ‹ …-D  ˆE  …„ ) …
 CN  ˆ Y“ š™‘ ¼9)+Z ®‹Ÿ ¼; ‹'5ŠZ  ˆ ‰Z ˆ  ; ‰  À+Z‹‰ ^
Š ³™    ˆ ¿Œ Œ ˆ Z` À®‹5 ¼; ‹Ÿ JŒ‹ ˆ ‘ ÀŸ ‹‘ ˆ ¿ Œ‹ ˆ x5‹F'1‹   ˆ ‘ 
‘™ ) |5‹=‹Ÿp‘‹‘ ‘•‘|‹·“  ‹;‘-‘ Á  ˆ ` ˆ ‹Ÿ) ˆ  ’ ‹=‘‹5‘'¼  ? Z ˆ š™‘ ,+   …Ž ) …
x % : 2 Œ  ˆ  ˆ  ?   …— ) …» ‹E ˆŸˆ ‹É¿ŠZ ˆ  — Œ
‘™xđ•'5Š ³™°Œ‹Ÿ2 ‹
 ’‘ˆ Œ‹5‹ …
G8H;I JKH;L G M NPOQSRUTWV<Q2VPR O M
8

X Y Z

[K\^]K_ „ ž ½ Œ‹É ˆ ‹2‹5 ‹É‘2 Œ‹% ˆ ‹%‹ŸŒ‹

 A  Y>`Œ ¿ ’  ’‘ˆ  a'


b :c>dSe5f%gh0i Œ‹Ÿ Í ˆ  ’ L  “ ´Z ˆ 2L“ °‘‹5‘'¼ kj ?  B i 9)HL“ ¼Ÿ‘5‹%;‘ ’ ‹Alm>npo
nqh@rsl@+‘ÉL“ ˆ Z ˆ ÉL“  ‘ ‹5‘'¼ kt7L“ue>d2v@rh5l= |ɝŠZ ‘ ˆ ‘ ¿Œ  ‹5 •‘5‹ …

#.7‘w1n‹5 |'‘™ ‹5 •‘5‹6+B   5 ˆ yxz.0/21a{‰ Y“ ˆ5Á 5Hi)  ˆ ˆ †a.7‘


)  ˆ
š™ ˆ5’  ’ 1 …
|  ˆ  ˆ }+L Œ‹' ˆ5Á 5Œ‹~xz.w/21a{‰‘qxz13/2.{´‹5 |
 Z ‹´ Œ‹
 Á •Œ‹ …L ‹
Œ· ‹;+L  
p 2 ˆ ŠhŒ‹Ÿ*m>d€h9r‚ … }+†Z ˆ Œ  ˆ ,+ 2<ƒ ’‘ˆ ‘‘„Ÿ‹HŒŒ‹P‘™ ˆ5Á 5Œ‹ ? )  ˆ 5 ‹
‹ •‘5‹. ‘1…)+  %|B“  BB“ Œ‹52Z ‹% ˆ ‘| ’ ‘*. ‘†1›‹5 É5‹we‡Sˆ@e‰5h%rsl …

 ‹% Œ‹%‘™¿š™‘ Œ‹%‘³Œ‹‹Ÿ‹;+ Œ‹2 ˆ ŠŒ‹É‹5 |ÉZ ‹% ˆ ‘| ’ ‹ …

Ž





 

[ \s]K_ Ž ž C ˆ 5‹h5ŠŒ‹É´ Y“ ¿)‘2•‘Ÿ ˆ '‘‹5‘¼ 







 

[K\^]K_ — ž | Ÿ5‘|Ÿ  C ˆ x5‹ÉŠ|Œ‹  Z ‹2 ’ À ˆ

½ “ ˆ |  ˆ •HL“ H‘| ˆ ‘ ˆ ‹5H 


 ˆ ¼ ˆ 1 ’  ’   †@‹ |SŒ‹Uš™‘ Œ‹U1 ˆ ŠŒ‹
ˆ ‘ ’ ‹ …

à U ˆŸˆ &0 '‹ -‘|´|' Y“  ˆ ‘ ˆ5’ ‹5‘5' Œ‹‰‹5 •‘5‹hZ ˆ Œ‹h; 5‹h‘h Œ‹‰ ˆ5Á 5Œ‹hZ ˆ
 Œ‹2 ˆ Œ‹2´Œ  ˆ ¼)Œ‹;+x Y“ ‘ À Œ‘•‘|ÉŒ‹p) |5‹p Œ‹ ‹pZ ˆpˆ ; ˆ %™° ˆ Œ‹2‘ †Y 

Œ‹´Œ  ˆ ¼)Œ‹´‹·“ |5 ˆ ‹5Œ‘5‘
   ’ …e f9d5€m>d€2‹‹‰; ˆ Œ ?   ˆ 5 ‰ Á •0 ‹
 Q À°|Œ‹5Ÿ Q Ài ÀZ ˆ  ’ ) …U¡ QZ ˆ Ÿ‘  ˆ +S Œ‹
) |5‹'L“ 5 ˆ ‹5Œ‘Ÿ ‹0Œ‹'Œ  ˆ ¼)Œ‹

‹ |%Z ‹%Œ‹É‹5 •‘5‹É‹Ÿ ’ •‘ ˆ Œ‹ …
5 ‹;+‚ xŒ  x‘|e ˆ ‘ ˆ  ˆ B“ ' Á •H ˆ ŠP)‘ Á  ˆ  ˆ ‘ ˆ5’ ‹5‘| ’ @ ‹Ÿ‘ ˆ ‹
Y>`Œ ‹%<ƒ ’‘ˆ ‘|5Œ‹h¿; Ékx´ ’  ’  ˆ | ?   … – )

Y X
Y


Z
Z

[K\^]K_ – ž  ‘™ ˆ ‘  5ˆ ’ ‹5‘|Ÿ ‹‰° Á •


 ˆ Š

 ‹'5 'Œ•@‹Ÿ;+L Œ‹' ˆ ŠŒ‹'Ÿ ‹ ’ ‹'‹5 ˆ '‹Ÿ) †‹ ’ ‹…l€^n†f uh5l+S …  …  … ‹‹
¼) ‘ 
? ´ ˆ5Á 5´| ˆ ‘ ‰ȋ5 •‘  ³Y Á •6)PL ˆ5Á 5Œ‹ ' Ÿ Œ‹ ?  ‹5‘ ˆ ‹p ˆ5Á 5Œ‹p ˆ ‘ ‘
Œ‹'‘™® Á •Œ‹0‹5 •‘5‹;+S‘'‹
 Á •‹5‘‹'‹Ÿ1 • ˆ ŠŒ‹Ÿ' ˆ ‘ ’ )‘;+B‹ 2Œ  ˆ š
·Ÿ +89m>m>d€h9r‚5l

—
 B"

 $ "
½ q‡ hc,dS‚p‡   lm>npnqh@rE.  L“ ˆ5Á 5Œ‹;+  ’ xz.{+F|B %;  ˆ š™ ˆ5’  ’ . …
Œ‹Ÿ2 ´ '¼ ˆ 

s'tv€F¥Ut y r

!#"%$&!$('"*),+.-0/ i1 23)54


6  xz.k{;=<4>!?  xst…{
78:9

@ dShACBh '
D à h‹Ÿ •`F  ˆ ”5ŠZ   ˆ5Á 5 ˆ ‘ ”‘™#‹ •‘5‹`  ˆ Š,+ ‘`  |B“ ‘   Œ‹Ÿ
Œ  ’ 'š™ ‘5‘•‘h‘™ Í ‹É‹2 À‹5 •|Š … E

FHG hdS‰@€z‰5h5IJEe LK h(nae,rsg-‚nae,rz€NMO-h ‡ P le‰ hRQ#STUoVQ#S!TW,X ' ½ 


;  ÀŸ iL“ qx  Àɋ ˆ5’ 

pŸ  ˆ A Á 5 …˜ ŠZ ´; ˆ ‹ ‹5 ˆŸˆ ´ À0¿L“ ¿Œ ˆ ° '¼ ˆ Œ‹hL“ ˆ Œ‹('Y/Z,+ ……… +‘5
‹ … C ˆ - ˆ • ` '¼ ˆ i•) ˆ ‹5 Œ‹Œ |‹5 ˆŸˆ5’ À¿ L“ « ¼ ˆ `Z ˆ 
) ˆ ‹5 Œ‹%Œ‹52Z ˆ·…

m \rz€m> '
[

D
¿Œ ‹ŸÄ ˆ 
´ ˆ Š‹Ÿ  ” ˆ ‘ ’ i + |h Œ‹%‹5 •‘5‹É‹ |% Œ‹Ex  ÀŒ ‹;+‘™
‘ Œ Œ‹ ’ | ˆ ‘  ’ ‹´Z ˆ • ˆ5Á 5‹Ÿ1‘‹5‘ ‘•‘'‹·“  ‹‹50‹5 |
‹ ˆŸˆ5’ À …Là 1‹·“†
  ˆ ‹É ˆ - ˆ %]^,_ 70`acbedOfVacg  xz.{%Œ‹Ÿ2° '¼ ˆ 'Z ˆ·…

hˆ +L“ ˆ ȋ% À0 ˆ  ˆ  ’  ’`„ +   '

4
< >!?  xsta{e; 6  xz.k{H; 6  xz.k{h 6  xz.k{
^,_ ^,_
78:9 7.`dOfVacg 70`iacbedOfVacg

 … … … 6  xz.{H;j<4k>l?  sx t…{nm 6  xz.k{ …


^,_ ^,_
70`iacbedOfVacg 70`dOfVacg

 ‹ 6  xz.{)Œ‹5BZ ˆ ‹5P†“ Œ‹ŸLP‹5 •@e '¼ ˆ Œ ‹UZ ˆ ‹;+Œ  6  xz.{
^O_ ^,_
70`idOfVacg 7.`acbed,fVacgpo
Œ‹52Z ˆ·… E

h dS‰@€z‰5h ' C ˆ - ˆ ,+‹H5 @ ˆ Š‰rqtsu<


‹ •‘5‹;+ F '• ‹ ‘™‹5 •‘5‹
FHG

B |É Œ‹2 Á •Œ‹É‘ ˆ5’ ‹ …

m \rz€m> '
[

D C1‹52 p ˆ ŠÉŒ‹ŸP‹5 ,+† %‘ ˆ5’ L“ ‹ •‘@|‘ Œ ÉZ ˆ Ÿ‘ wv Y/Z>/Oxyxyx&/zqm
Z{ …

CN ˆ Y“¼‹Ÿ ˆ 3'Z‹ŸU Œ‹|q ‹5 •‘5‹h ‰Œ‹‰‘ ˆ5’ ‹´‘™  ‘™”‹ŸŸ‘5‹6+F†“ Œ‹Ÿ‰ B“ 
š™‹Ÿ5È ‹5 •‘°‘ ˆ5’~} ;  ˆ 5  }€ v Y/Z>/Oxyxyx&/zqm‚Z{ …P¡ .Z ˆ Ÿ‘  ˆ +1 %š™‹Ÿ5i
‹ •‘#.\É‘ ˆ5’ Y-+B“ Œ‹Ÿ@  ˆ ‘  ’  ‘`‹5 •‘;+‘P•‹5 •‘ 1 É‘ ˆ5’ qm~Z,+
BŒ‹Ÿ ˆ ‘  ’  5 ‹2 Œ‹É‹5 •‘5‹É ˆ Œ‹%´ ³Y Á •,+‘É‘°Z ˆ Ÿ‘  ˆ  . …Z˜ | ˆ  ‘Ÿ  …
E

–
 €  #"  

%.\‘!1 ‹ |@‘™•‹5 •‘5‹HL“ • ˆ Š†i +† ‰@ghnp€^i.  1 Œ‹ŸP%‹55%L“ ˆ5Á 5Œ‹
x
W /   { )  ˆr} ; Z>/Oxyxyx&/  mZ,+ ; . ‘
k; 1 …
½  '¼ ˆ L“ ˆ5Á 5Œ‹ÉBŒ ; †‹‘|%”Š‘ Œ‹ŸÉ)‘ ’ À &m> c# hA d•¿5Š‘ …

¿Œ  x‘ ˆ |B“ ¿‹ •‘%Œ‹Ÿp5 >  ˆ ‹ ˆ ‘  ’  ³Y Á •Z ˆ °Š‘”´  †‘ ˆ
Ê…
b ® ˆ ŠŒ‹Ÿq‰2m>% h G h  ˆ ‹5=‘™®‹5 •‘5‹‘ Œ Œ‹;‘-‘ Á  ˆ  ˆ ‘  ’ ‹Z ˆ
¿Š‘ …

 . Œ‹Ÿh ‹5 •‘‰L“   ˆ Špi + Y“ ‘‹5‘¼ 5 ‹% Œ‹‰‹5 •‘5‹A1 ;  ˆ Œ‹5‘ ‹%
š™‹Ÿ5´¿Š‘ (.  1 +Œ‹ŸÉ;‘ ’ 
À:‰2m>n†fmle>%rhp‰2m,  h G hp‡ h .0+‘% ” À0 5 7 x
à ÉŒ‹5  ˆ ‹aY ‘ ¿  ’‘ˆ    ˆ |¿‹5*.n‘q1 ‹5 |`‘™.‹ •‘5‹`L“   ˆ Š i ?  
ˆ ‘ ’ )+F  ˆ ‹%‹   7 ;«‹5    7 t;! …
½ Œ‹2‹5 •‘5‹2L“ °5‘ ) ˆ Š´;‘-‘É  ˆ ‹2‹5 ˆ5’ Z ˆ Ÿ ˆ ‘i ˆ ;Œ‹%)‘ ’ ‹(‰2m>n†fmle>%rh5l
‰ m>  h G h5l+ ‘™  ‘™'‹zŸ |5Œ‹;+ ‘™‹5 •‘5‹1Z ˆ 5‘Z    Á •HŒ ) †‹5 ’ 
ˆ ‘  ’ ‹%Z ˆ ”5Š‘ …
" ‘ ‘|5‘+F” ˆ Š'
) †‹5‹5Č
B“ '‹5‘ 'Œ ; †‹5Œ š™'‹5S‘h‹5‘ ‘•‘
‹‚“  BŒ‹Ÿ2 ³Y Á •'Œ š™ …

C ˆ5’ ‘‹5 ‹É‘   |…'


  Y“ $# ’ &%2|‘•‘HŒ‹@ ˆ5Á 5Œ‹H*i® •‹Ÿ Y“  ’ ɝŒ ˆ ‹H2‹5Œ‹@‹5 •‘5‹
N
‘25 5Œ‹É Œ‹É ˆ5Á 5Œ‹%B‹·“ ˆ Ÿ 5ŠZ‘|;+; ” ¼Ÿ‘%¿ -·¿ ˆ Šwi'LBŒ‹Ÿ2l@m  lo
c,d e5f9ghwi …
 b ~c>dSe5f%ghk€^9‡  €zrNZ ˆ i^Œ‹ŸH• ˆ Š‰ ¼5‘|  Z ˆ Ÿ ˆ Ai^‘É| ˆ |p|hŒ ˆ ‹
‹ •‘5‹%‘% Œ‹É ˆ5Á 5Œ‹ÉŸÀ Œ‹ ˆ ‘ À|É Œ‹É‹5 •‘5‹%Œ ‹ ˆ  ’ ‹ …
 L Y“ ¿Y%| ˆ ‘ À Œ ˆ Œ ˆ Œ‹´ ˆ5Á 5Œ‹%wi Z ˆ Œ‹É5Š‘‹É ˆ5’ Œ‹É‘¿| ˆ x³
‹ |Œ‹' -·™ ‹5 •‘5‹|5 ˆ  ’ À ˆ Œ‹ ? Z ˆ š™‘ ,+U  ˆ ‘ À Œ` Y“ ˆ5Á 5 xz.0/21a{´Z ˆ
.)(#xyxyx*
1 + Œ+ ‹  B“ ’ ÉZ ‹hŒ‹É‹5 •‘5‹ÉŸÀ™ )+  ¼Ÿ‘|‰¿ -·” ˆ Š

Œ‹52p*l ,S‡>€ B;€^l5€m> 0i ?   …œ ) …

[K\s] _ œ ži¼)&‹5 ”L“ ¿ ˆ Š

| Ÿ5‘|Ÿ +;;  ˆ 0‹Ÿ¼;&x‹Ÿ +F  B“Z ‹É 0 ˆ ´ ˆ  Ÿ 5 ˆ  5‹ •‘|N‹5 ˆ 
Œ   ‘™\ ˆ5Á 5Œ‹ … CN ˆ š™‘  ‹Ÿ ˆ À  † ˆ  Å +P   ˆ  Š ” ˆ  5 B“ Œ‹Ÿ`Z ‹`
‹5¼;&x‹Ÿ ¿Œ‘ S|Š …

[K\^]K_ Å ž ˜ Œ‘BB“ Œ‹Ÿ2Z ‹%


‹Ÿ¼)&‹Ÿ 

œ
hdS‰@€z‰5h 
FHG ' C ˆ - ˆ |´‹Ÿ iQŒ‹5 i ˆ ŠŒ š™  q ‹5 •‘5‹6+  ˆ ‹i ) †‹5‹5ȍi• ‹
q~m Z ˆ5Á 5Œ ‹ …

[ m \rz€m> '
D CN ˆ%ˆ5’ ‘ ˆŸˆ ‘Œ‹Ÿ ˆ q '
½  ˆ5’ ‹Ÿ ÉŒ‹5 ’ ‘%;  ˆ q ; Z
‘
q ;=< …
x |q s<= ‘|Ÿ ˆ† ™ ’†…9
 ‹Ÿ) †‹5

5 É ˆ ŠŒ š™qQ‹ •‘5‹É) †‹5‹5Č
°• | ‹ q~mZ
 ˆ5Á 5Œ‹ …
x h  ˆ ‹*i ° ˆ Š'Œ š™  q hZ‹ •‘5‹ … x |>`  '¼ ˆ 'L“ ˆ5Á 5Œ‹Éwi …
» 5 ‹%´ ÀŒ š™ ’  ‹5‹Ÿ ˆ 
ŠZ |
‹ •‘ÉŒ‹Ÿ%
‘ ˆ5’ °• ‹ „†…
ŠZ |^‹5 •‘¿Œ‹5i®‘ ˆ5’  • ‹ Ž +%  ˆ ‹i À  ˆ  ˆ  ’  ’ „ Œ   <#> ;
]  xz.k{ su<q„+‘h   > sq …
7 89
=;“  hš™‹Ÿ5°J‹5 •‘”‘ ˆ5’ „†…@» 5 ‹• :. … |  ˆ ‹;+1 ¿ ˆ Š” i ' ¼5‘
‘ ’ Z|k. ‘É Y“ ˆ5Á 50 |‰ UŒ‹5É Y“ š™ ˆ5’  ’ +FŒ‹Ÿh” ˆ Š0Œ š™0  q ‹5 •‘5‹h
) †‹5‹5Č1š™ ‘5‘•‘|NH ˆ5Á 5@1• ‹U#i …· “ ˆ ȋB Y“ Š ) ŸŠȋ5H ˆ5’ ‘ ˆŸˆ ‘Œ,+
i '†) †‹5‹5Č
 ‰`•  ‹ q m Z% ˆ5Á 5Œ‹;+xL“   i^‘`; †‹‹5Čh`• 
‹ q„+ŒÉ; 5ŠÄ@h À
 ’ • ‹5 ˆ Ÿ  …
E

b  ,‰ K‰ uh`Œ‹Ÿ 5Š‘Q=  †‘ ˆ  ^|  • ˆ ‘  ‹5 •‘  ³Y Á • … CN  ˆ
  ’  ˆ Œ‹1 Ÿ ‹;+†•‘ ‘ %‹5 ˆ !‹Ÿ5   ’   
(xyxyx*)  +Œ+5s ? 
 ‹5p p ˆ Š
Œ‹5%‹Ÿ; †‹ ’ ‹Ÿ 
‘2 ” ˆ ‘ ’ )+Z‘É ° Œ‹ ‹5 |É Œ‹%‹5 •‘5‹2Ÿ ‹ ’ ‹6+ ‹%Œ‘É ˆ  ˆ  …

b ye>d ,d2h0Œ‹Ÿ2° ˆ Š'Œ š™


‘%‹‹É‘ x‘  …
» 5 ‹°  ˆ ‹i|®‹ÈD ˆ ¼ ˆ ,+H ´B“ š™‹Ÿ5 |B“  5‹ ‘ 5Š‘D‘| ˆ ^‘™J‹5 •‘5‹
  ’ ‹ ?  e ”‘^`®• ‹‰®Z ˆ Œ š™ ’ +L‘´‹·“  U ‘®;  ‘™”)  ˆ Œ‹‰‹ •‘5‹A.
‘1   ˆ ‹;+Z‘° Œ‹2•‘Ÿ|h¼;   ¼) ;+ ¿)  ˆŸˆ ɝ ˆ5’  ˆ ” ‘ ‘ 6) …

[K\^]K_  ž b ” ˆ ¼ ˆ 

s'tv€F¥Ut y ƒ

      
            
  
2 '"*),+.-0/ q  5/ $ / $H#2 ):" ",/ / $ / N/ 5/ 2.$ / $ O220/ ./ / $ +H /|/ ) $&/ 5/ 2.$

q~m Z ):" $&/

@ dShACBh '
D
°;‘%5 h‹Ÿ5
‹Ÿ) †‹5 ˆ  qtsu< …
  i Œ‹Ÿ2¿ ˆ Š
Œ š™
kq ‹5 •‘5‹É‘É q~mZ
 ˆ5Á 5Œ‹ …
CN ˆ Y“¼‹5 ˆ q'‹Ÿ) †‹5 ‹%(i ; †‹‹5Č´¿‘ x‘ ,+Z‹5 ‹  
(xyxyx*)  …
|  ˆ ‹;+Z‘ ‹Ÿ ˆ h Y“ ˆ5Á 5 x  ; /  ( {+Z  
¼ ˆ ‹5
Z ‹É ÀŒ š™ ’ ” ˆ Š ? ‹5S Y“ 
•¼;Œ‹  Z ‹5‹5 ˆ     
(,+; ” 0Œ | ˆ  ˆ ,+) e‹Ÿ •´ *Y ˆ ' 5  ˆ   À Š‘

Å
 ( xyxyx*
  { …½  -·  ˆ Š' ¼5‘ Œ‹ŸÉ 'Œ š™‘%; †‹‹5Č q ‹5 •‘5‹‰‘
q~m <
 ˆ5Á 5Œ‹6+Œ|BŒ  ˆ ŒÉ  ˆ5’ ‹5 É´ Y“ š™ ˆ ‘Œ' ˆ5’  ’ ‘| …
 ,+9iQ
; †‹5‹Č´Z ‹%
‘ ‘ ,+Œ
|L) ˆ •‘ÉL“ ˆ • ˆ |B“  B‹·“†ɼ‘”L“ ” ˆ ¼ ˆ  …
!C ˆ - ‹h|5‘Z|;+;Z ˆhˆ5’ ‘ ˆŸˆ ‘Œ'‹Ÿ ˆ q„+Z|
‹5 i«Œ‹Ÿ%  ˆ ¼ ˆ   qQ‹5 •‘5‹‰  ˆ ‹
 L) †‹5‹5ȍš™ ‘5‘•‘| q~mZ
 ˆ5Á 5Œ‹ …
½ Œ ‘ ‹Ÿ ”Œ‹5 ’ ‘5
)  ˆ q ;=< …
x 
 qtsu<  ™ ’†… ; †‹ ‹2 À=Œ ‘ ‹Ÿ  ’ ¼ 
;  ˆ 5 É ˆ ¼ ˆ  q ‹5 •‘5‹ …
x †i   ˆ ¼ ˆ   q hZ
‹ •‘5‹ …

: =Œ •‘Œ ˆ Z ˆ  ˆ - ˆ (i ; †‹‹5Č
°• ‹%¿‹5 •‘h
‘ ˆ5’•„ '
CN ˆ Y“¼‹5 ˆ q'‹Ÿ) †‹5 ‹%´5 ‹2 Œ‹%‹5 •‘5‹h‹5 ‘|h
‘ ˆ5’ ‹h°• ‹ Ž…
x i  ˆ ‹   J‹5 •‘i ˆ ¼ ˆ  ˆ  …
\5Š ‹5 )(°Z ˆ ‰ Œ‹`‹ •‘5‹È 5 Œ‘|5‹ 
  +L‘ ®Œ  ˆ = Y“ ˆ5Á 5: 
x   / )(6{ …8
®Š ‹Ÿ  ˆ ‹´^‹5 •‘  ,+L Ÿ Œ‘  )(=‹5‘  
´ ˆ5Á 5‰ È‘Œ ˆ ´Œ  ˆ  ’  ? Œ´|)Œ‹ŸH; †‹5‹5¼ ,+|‹5  x  ( { su<{+‘ iŒ  ˆ ‰ Y“ ˆ5Á 5
 
x )(> /  6{ …¡ 1‹5p‹Ÿ5,+P|p†“ Œ‹Ÿe) †‹5‹Ÿ¼ ,+ ‹Ÿ   ‘PL“ Á  ˆ  Œ ‹5 ˆ P‘eB“  ˆ Œ‹Ÿ5

 ˆ5Á 5'L“ š™x ˆ5’  ’  |LB“0Z ‹h‘Œ ˆ  ’  ’ Œ  ˆ  ’ ,+ ”Š ‹Ÿ%¿‹5 •‘É  ’   
Z ˆ UŒ‘™iB‹5 |É Œ‹É ˆ Œ‹%š™ ˆ5’  ’ ‹É´5‘  Œ‹h ˆ5Á 5Œ‹6+‘É ¿Œ  ˆ  Y“ ˆ5Á 5qx  /    { …
˜ •¿ ° ¼ ˆ  L“ ˆ5Á 5Œ‹ÈŒ‹Ÿ  ‰‘|¿ ° '¼ ˆ  L“ ˆ5Á 5Œ‹ JŒ  ˆ  ’ Œ‹`
L“ 
 ’  5ŠZ  ’ ),+Œ‘Ÿ5 ˆ x ’  ˆ (†=@  ˆ ‹·“ ˆŸˆ5Á 5 ˆ·…
˜ •3i ; †‹‹5ČZ ‹0‘ ‘ ,+B HŒ‹5') †‹5‹Ÿ¼ •B“   Á •`‹5 •‘Z ˆ  À‹5‹5 
‘™3 ‹%‹2 À‹Ÿ5ax  { …
x  ) • ˆ  ˆ ‹5 •‘'Œ ‹5 ˆ  … |  ˆ ‹;+L‹5  x 
,{ s <°‘0| Y“  °Œ  ˆ  ’
B“ @‹‘ @ ˆ5Á 5HL“ š™ ˆ5’  ’ ) +‚†“ Œ‹ŸS H|B“  x‘ ˆ Œ‹Ÿ5@‘Œ ˆ  
• ‹UP 0Œ  ˆ  ’ 
‘%L) ˆ •‘ɍ)  ˆ ‹Ÿ& ˆ ´ À•Œ ‹Ÿ ˆ ‘Ÿ  …;˜  ˆ  ‘Ÿ  …
| ‹Ÿ}+>i”; †‹5‹ČH=• ‹N=‹ •‘e ‘ ˆ5’´„ + ‹5 ‹ . …  ‹1  ˆ ‹;+ ‘=‹Ÿ ˆ |„.
‘e Y“ ˆ5Á 52 1 Œ‹5U2š™ ˆ5’  ’ +†  ¼Ÿ‘|1•‹5 ‹Y ˆ ŠEi ' |Œ‹ŸN5 >  ˆ ‹1Œ š™2‘
‹ ‹ ‘ ‘ ,+ …  …  … ¿ ˆ ¼ ˆ ,+‹2); †‹‹5Čhš™ ‘5‘•‘|Éi‹ •‘p‘p ˆ5Á 5‰• ‹
0i …F½ “ Š| x; ŸŠȋ ˆ5’ ‘ ˆŸˆ ‘Œ0 ‹5‹5 ˆ ' 0|wi ' ) †‹5‹5Č'š™ ‘5‘•‘ q muZ ˆ5Á 5Œ‹6+Z‘
‹5B|(i«‘¿) †‹5‹5ȍš™ ‘5‘•‘| q … E

*hnae>dVMO-h ' ½  ˆ  ˆ  ’  ’'Ž ‘p Y“ š™ ˆ ‘Œ´ ˆ5’  ’ ‘%• | ˆ ‘É|‰ Œ‹% ˆ ¼ ˆ Œ‹2‹5 2 Œ‹p ˆ ŠŒ‹
J

Œ š™Œ‹%™ +F  ‹5‘‹%  '¼ ˆ 'L“ ˆ5Á 5Œ‹ …Z½ • ˆ  ˆ  ’  ’ ‹Ÿ&†|5'• | ˆ 'B“  ‹%) ˆ 
•‘Ÿ5‘ ’ | ‘•‘ hL  
 “ ‘‘ ˆ  ˆ 5 ‹2 Œ‹% ˆ ŠŒ‹ÉŒ š™Œ‹ …

s'tv€F¥Ut y ¸

     
   
    

    
      
0!$ '"*),+.-0/ O220/ ./ +H/ $  i$&/ 2 "w+ ):"€) !$ #2 O/ "$p) 2 20 ",/ ):" $&/ #2 ):" ",/

) i):2 $ N/ 5/|20 ",/ /  5/ $

dShACBh '
@

D
 ˆ ‹5 Z ˆ%ˆ5’ ‘ ˆŸˆ ‘Œ‹Ÿ ˆ  '¼ ˆ %q^
‘ x‘ Œ‹É¿ ˆ Š …
p q3;jY-+  ˆ 
 ˆ ŠŒ‹Ÿ%¿ ˆ ŠŒ š™‹‹%‘ ‘  …à U‹·“†É 'L“   ˆ ¼ ˆ '‘
 LB“ ° ˆ ‘  Y ˆ  …
=x  q s Y J‘|Ÿ ˆ~ ™ ’†… ) †‹5 ‹È|¿  ˆ5’ ‹Ÿ ȋ5  ’ ¼ ‰;  ˆ 5 ` ˆ Š
Œ š™B“· ||ÉZ ‹% ‹É q ‘ ‘ Œ‹ …
x É ° ˆ Š'Œ š™wi«Œ ||  q hZ'‘ x‘ Œ‹ …

®Œ ‹5Ä ˆ =  ˆ ‹´ =‹5 ‹ŸY ˆ Šp
i 'N ¼5‘| ‘^‘ @ |‘•‘ Y“ ˆ5Á 5:x  / )(6{
UZ ˆ 5‘Z  ”‘ ‘    )(:xyxyx*)   …Zà SŒ‹5ɝ‘ À ˆ B“‹ŸU ”¼ ˆ ‹5 • ‹% ‘ ‘ 
Z ˆ PŒ‘™®…i …) 0 ‹;+;5 ‹´ Œ‹‘ x‘ Œ‹… i 'e‹5 |Œ‹‘ ‘ Œ‹pi +) a i 'S) †‹5‹5Č

Â
 ‹
q®‘ ‘ Œ‹ ? )‘ Á  ˆ 
‘”vY ¿¼ ˆ ‹ ’  ‹ÉL“ 9) …¡   + ´ ˆ Šwi ' Œ‹Ÿ%‘Œ ˆ Œ š™,+

‹|%‹5 Y“ a‘PZ ‹5‹5 ˆ    
( ? • ɝŒ | ˆ  ˆ 6)+ ‹Ÿ •@Œ •ɝ‘³Œ‹‹Ÿ‹@ !Y ˆ 

´5  ˆ xÀ= Š‘ )((xOxOx 
  …
 “ ˆ Č‹É Y“ Š| x; ŸŠȋ' ˆ5’ ‘ ˆŸˆ ‘Œ,+F  ‹h|wi'))‘ Á  ˆ ' ¼5‘|  Z ˆ Ÿ ˆ L“   ˆ ¼ ˆ 
Z ˆ  Ÿ L“ «Œ ˆ Q ¼ ˆ ÈL“ ˆ5Á 5Œ‹ …Sà H‹Ÿ •=  ˆ ‹L“ Ÿ 5 ˆ Y“ ˆ5Á 5 x  / )(6{´)  ˆ
ˆ ‘ ˆ - ˆ i +Œ
|S ŠÄ@
À= ’ • ‹Ÿ ˆ Ÿ  … E

J*hnae>dVMO-h ' b . ˆ ¼ ˆ   ; ˆ •‘L“ ¼5‘ ˆ È ˆ ŠiŒ š™ imZ ˆ  Ÿ 5‹=‹ŸŒŒŒ‹5‹Ÿ< ‹
L“ ˆ5Á 5Œ‹Œ‹Ÿ)‘ ’  >e d ,d2h~dSh2‰2m#CB;d e,%r~i …N˜ “ Œ‹Ÿ
  ˆ ¼ ˆ `|@i • Á •‘‹5‘¼ 
‹ •‘5‹H†i …x½  ˆ  ˆ  ’  ’— •  ˆ ´ h%5 H ˆ Š‰Œ š™%) †‹5‹5ȉ• ‹ 
 ˆ ¼ ˆ  ˆ ŒŒ - ˆ  …

 !#"i }" !#"   #   ,"


S‘™”‹5 •‘5‹‰|‘ Œ Œ‹ ? ‹ŸŸ‘5‹)2” ˆ Š i.‹5  ˆ ‘  ’ ‹hZ ˆ  ˆ5Á 5,+F  
´ ´ ˆ Š'Œ‹5w‰2m>n†f uh@r …Z¡ ° ’  ’‘ˆ  }+Z ´ ˆ Š'Œ  ‘  q ‹ •‘5‹ÉŒ‹Ÿ2  ’

[K\s]K_   ž ½  ˆ Š


Œ  ‘  ‹5 •‘5‹

à   ˆ ‹5‘•‘|;+pŒ‹i‹5 •‘5‹iL“ \ ˆ ŠF ˆ •‘|°Œ ¿ Y“ D)‘  ” h-l;hn , &hB‡ h
lm>npnqh@rslk€^9‡ ‚f-h9‡ e>%rsl'‹·“  LB“ š™‹Ÿ5'‘' ˆ5Á 5'‘| ˆ 
‘™¿‘ Œ |Œ‹hL“ ‘| ˆ 
‘™ …
b ° ˆ Š
Œ‹5% ,€ fe>d@rz€srh'  ˆ ‹5‹5Œ‹%‹5 •‘5‹%;‘-‘ Á  ˆ  ˆ5’ Z ˆ Ÿ‹É‘ ‘™È ˆ )Œ‹
‹ 5‹  ‘  7‹5 •‘5‹É ’ ;‘5‹ …
 ‹Œ· ‹;+S` ˆ5Á 5•)‘ ˆ ‘  ˆ B“  ‹ •‘   ‘ ‹5 •‘0  3+L‹
5 ‹
Œ‹Œ  Œ‹
) †‹5‹Ÿ¼ Œ‹´‹5 Z ‹Í ˆ  ’ •‘ ˆ ‘  ’ ‹Z ˆ • ˆ5Á 5 … 1;  ˆ '†“ Œ‹Ÿ 
· ‹6+  ˆ Š
Œ‹ŸÉ  ˆ ‹2  ’
  
  Œ ~;k 4 >l?  !x { ‘ 4 >l?  #x p{ …
 " ; k

” ’ É´Ä ˆ Z  †' Œ‹2 ˆ ŠŒ‹wrzd€ fe>d@rz€srh5l@+‘5 ……

CN ˆ š™‘ ,+ ‰ ˆ Š´Œ ‹Ÿ ’‘ˆ5’ ‹p h ˆ ¼ đ•Œ‹H ˆ ‹ 
 ‹5 ‹p‘2Œ‹  ˆ ‹ ‹ŸŒ‹
? š™‘ ') ‘³Œ‹5‹5‹5)pŒ‹Ÿ2¼Z ˆ Ÿ5 …Fà BŒ ˆŸˆ Œ‹Ÿ) ° Á • 
 …

[ \s]K_ ¢ ž ½  ˆ Š


Œ‹É‹5 ‹%‘ÉŒ‹2‹5Œ‹%Œ‹Ÿ


 
   "   "%$
²ButvÇ|y{ÇuQr
¤ C ˆ - ˆ B“ ° ˆ Š  q ‹ •‘5‹;+5ŠZ ‘ 
‘ ˆ5’ °• ‹ <+; †‹5‹Č´¿‘ x‘  …
 C ˆ - ˆ |'‹ŸL° ˆ Š
) †‹5‹5Č q ‹5 •‘5‹h‘É¿• ‹
q  ˆ5Á 5Œ‹;+F  ˆ ‹% B) †‹5‹5Č
¿‘ ‘  …

²ButvÇ|y{Çu®ƒ

b   Á 5 ˆ ‘ ˆ ; q ) ˆ ‹5 Œ‹;+25ŠZ ‘ Œ ||”m ˆ5’ ‹5‘| ? Y“Ÿ ’ ’  ˆ5’ ‘³
 ˆ x ……… ) …Z ‹É5 h ˆ ;L“ • ‹h ˆ ‹h; ˆ ‹5 Œ‹6+ BB“ ”kŸ‹‰š™ ‘5‘•‘‰‘™
Z ˆ Œ‹ÉL“‹ … C ˆ - ˆ | ŠZ |'; ˆ ‹5 Œ‹5É'Œ'5 5Œ‹2 Œ‹h ˆ Œ‹ …

²ButvÇ|y{Çu ¸  H€F­Ut~U€FyH¯Nu|}Syͧͧ{u|}irFƒ

 ‹HÈZŒ ‹;+| F È• ‹ ZOYZ*x  Œ‹;+x‘HŒ‹ À‹5 ‹p ’‘ˆ ‘Œ‹2 ˆ Œ‘5Œ‹ ?    ˆ  ˆ ‘5  ˆ )
š™‹Ÿ5‘0‘ ˆ °Œ ˆ Œ‹L“ ‘| ˆ i‘  Œ‹ …N½  · È ZŒ ‹=Œ‹Ÿ‹Ÿ ˆ ‘  ’   ZOY Y   Œ‹;+N‘
ŠZ |i ˆ ~  ; †‹5‹Č ZOY À‹5 ‹ ’‘ˆ ‘Œ‹0<ƒ ’‘ˆ ‘|5Œ‹ …Nà @Œ‹Ÿ0) †‹5‹Ÿ¼ q · | ˆ 
B“ ) ˆ 5^‘   x    B“ ) ˆ 5^‘  ® ˆ  Z ˆ Œ‹È À‹5 ‹¿ ’‘ˆ ‘Œ‹;+2‘  ˆ ‹Ÿ
’ ‘Ÿ‘  ‘•‘|HZ ˆ L“ ˆ Œ‹„  Œ‹ … C ˆ - ˆ B“  Œ‹Ÿe; †‹‹Ÿ¼ p#  ˆ • ˆ Àh• Ÿ ’ Œ‹e À‹5 ‹
 ’‘ˆ ‘Œ‹h À=· 
5 %‘” ˆ5’ ‹5 ˆ  % À·Z ‘ ’ ( · | ˆ L“ (    Y“ ˆ  …

²ButvÇ|y{Çu Æ  H€F­Ut~U€FyH¯Nu|}Syͧͧ{u|}irFË

C ˆ - ˆ |,+U‹'5 0 ˆ ;È#Y”) ˆ ‹5 Œ‹;+S ‘ š™x‹Ÿ55 >  ˆ ‹ • ‹0‘™
1 |
® ¼ ˆ •Z ˆ ? ’ ‘|Ÿ‘  ‘•‘ ^)‰Œ Z‹5‹ŒŒ‹
Œ Œ‹'‹ = ˆ )

? À ˆ ‘ ÀŸ  ‰‹5=Œ Z  ˆ   
Œ‹Ÿ´Œ ‹Ÿ ’‘ˆ5’ =Œ • ˆ5’ ‘ ˆ ,+)‘´Š ’  ‹ &+)  =‹50Œ Z  
5‹%‹5 ³Y Á •6) …

²ButvÇ|y{Çu®Ë  H€F­Ut~U€FyH¯Nu|}Syͧͧ{u|}irFË

½ Œ‹2 '¼ ˆ Œ‹ Y/ Z>/ < O/ xyxyx&/ ȋ5 | ’  ˆ 5%
‹ ZOYp ‹%5ŠZ ‘ ‹%°¼ · ZOYtZOY …
C ˆ - ˆ B“  Uš™‹Ÿ5'
†0 ”'Œ  0UŒ |Ÿ‘´ ‹h' ˆ ‹É ¼ ˆ Œ‹‰<ƒ ’ 
ˆ ‘5‹ …

²ButvÇ|y{Çu  H€F­Ut~U€FyH¯Nu|}Syͧͧ{u|}irLr
| . >=5 ˆ 
+ q9‘ 5‹ ’ ‹Ÿ ˆ ‘‹5iZ ˆ  ˆ`’  ¼ ‘•‘|! ¼Z ˆŸˆ Œ‹Š Œ À ’ Œ‹`‘
ŸŒ‹;+‘
'ŒŒ‹2¼Z ˆŸˆ Œ‹%@†| Á  ˆ Œ  ’    ‹hL“  ‹ …
&% q
C ˆ - ˆ |,+Œ‘ ÀŒ‹5 ˆ5’  ‹¼ ‰‹Ÿ)‘H‹5‘ ‘•‘p‹5 '" s q $# …
; q~m ^Œ8x'/zq {

²ButvÇ|y{Çu)(*  H€F­Ut~U€FyH¯Nu|}Syͧͧ{u|}irZµ+
 q, q•· ‹5Œ‹Y ’ ‹;+ 0%‹5 ˆ U '¼ ˆ  …  ‘™
†Œ‹
 ‹N5ŠZ  · ‹5@L“ 0¼ ·0
‘ Œ Œ‹‹5 |5 >Ÿ  ˆ ‹
<ƒ ’‘ˆ ‘|5Œ‹ ?  …  …  …  Pš™‹Ÿ5®• ‹Œ  | ˆ ‘Œ  ˆ 
ŒŒ‹• †Œ‹`‹5‘   Œ‹ '¼ ˆ Œ‹È<ƒ ’‘ˆ ‘5‹5) … C ˆ - ˆ B“  Éš™x‹55°¿Œ  ”° Y“  )‘
ƒB Œ ˆ `‹5 ˆ 5,+S‹ ¼ · q- x q m=Z6{ ˆ Œ‹Ÿ;+B Œ‹ †Œ‹‹ ‘|=‘Œ ˆ ‘™ 
‘™°<ƒ ’‘ˆ ‘|5Œ‹ …

²ButvÇ|y{Çu /.•§0Sw ¥ey¨¤¯Nu21¿~Ny{€F~43ŒÌ)€Sy‚y'5B­Uu®r



b  ˆ Z ’ ‘É2Œ ‹Ÿ ˆ  ˆ 
q x  Œ‹@‘eqm~Z ˆ 5Œ‹@ Œ‹ ˆ ‘ À|p  •2; -  ˆ ‹52 ’  À Œ ˆ
L“   2‘ Œ |  É ˆ  …˜ ŠZ  ˆ 5 ˆ ‘ %‘™p  Œ‹P‹ ‹P‘ ˆ  ˆ ‹5 ˆ L“ ˆ ,+
‘P‘™ ˆ 5Œ‹1B“ PZ ‹1L“ |5 ˆ ‹5Œ‘Ÿ  …½  ˆ  _‘@‹5‹5p Œ‹e ‹1)‘Ÿ5Œ‹P‹ŸŒŒ‹P‘ ˆ 
76 
Œ‹  Œ‹ ? 
 •Œ‹ ˆ 5Œ‹5) ‹5 ‘w Z>/ < / /Oxyxyx&/ (
`
8   •Ä‘ ˆ Œ‹ …¡ ‹ŸŒ
; †‹5‹5¼ ´;  ˆ '
¤ 
 q ; :9

 q ; Z<; :9

¢
²ButvÇ|y{Çu 1 Ì  r 

 Œ‹#Ÿ‘5 ‹h‹5 h À  ’ ‹‰‹Ÿ ˆ  ¼ ·®· ˆŸˆ5’ wq  qQ· ‹5Œ‹Y ’ ‹´w >`Œ   ‹Ÿ‹ Y ˆ 
Œ‹ ˆ đ† Œ‹É‹Ÿ&†|5Œ‹('
? ) 5· ‹5‰|)´Œ |Ÿ‘2Z ‹p0Ÿ‘5 i; †‹‹5ČÉ°Œ¾  ’ Œ '”;Œ´´· ‹5h
‘¿Œ Ÿ‘|‰ …
? ¼9)wCN  ˆ 5 5iZ ˆ °i· ‹5Œ‹ v /  ' {”pŒ |Ÿ‘‘|ȝ5ŠZ ‘° Ÿ‘5 +N pš™‹Ÿ5È
‹55`ȝ· ‹5Œ‹=Œ |5‘Z•5ŠZ ‘i ‘5 +N|pŒ •‘ŒiZ ˆ  ‘=‹55 ˆ ÈZ ˆ  's+N‘
5‘  Œ‹É|
‘™°· ‹5Œ‹%Œ ‹ ’ ‘Ÿ&Œ‹É À=‹Ÿ5' |%5 >Ÿ  ˆ ‹%¿Œ¾  ’ ‘ Œ ' …
C ˆ - ˆ B“  L i”• ‹ /( 6 Ÿ‘5 ‹ …

  &):2<=-41(9<0k5:*:%35&;41(\* Œ"%&)l $0k5476


$0k54e&

 @‘ ‹%‹Ÿ ˆ Y“ š™‘ ') ' ’ Z ˆ w'


¡ ‹ŸY 1) †‹5‹Ÿ¼  ˆ ‘  ˆ 5ŠZ ‘`Œ‹´ ˆ ‹‹5 ‹  ŠZ ‘Œ‹´ ˆ ‹‹5Œ‹
‹5 ˆ 5
‘™°Œ 5Œ‹h‘ Œ |Œ‹%
‹5 ˆ ‹5‘ Ÿ‹ 9

» ‹´ ‹´ ’  Œ ‹Ÿ ’‘ˆ5’ 


 ˆ Š0¼Z ˆ Ÿ50 |‰ Œ‹h‹5 •‘5‹´‹5 h Œ‹h ˆ ‹h‹5 ‹
‘É Œ‹% ˆ ‹%‹5Œ‹;+Z‘É É Œ‹‰ ˆ5Á 5Œ‹h‹5 É Œ‹hŒ 5Œ‹ …Z½ |Œ‹5Ÿ ”Œ‹ŸÉ 
‹  ˆ ‹·“ 
Œ‹5@) †‹5‹Ÿ¼ % ˆ ‘ ˆ5’ ‹5‘|5 ˆ Œ% ˆ Š‰‹H % Àiɋ5 ˆ 5h|É Œ‹H ˆ5Á 5Œ‹p‹5 ‘| ‹Ÿ x'¼) ‹ ’ Œ‹
Z ˆ Œ‹‰ ˆ Œ‹%
Œ  ˆ ¼)Œ‹%L
‹5Œ )‘|%Z ‹ …

b
  ˆ ŠH |U Y“ ')‘ ˆ ‘ ˆ5’ ‹5‘|5 ˆ Œ‹S‹ •‘5‹BZ ˆ Œ‹S; 5‹S
 À0‘B Œ‹U ˆ5Á 5Œ‹UZ ˆ
Œ‹  ˆ Œ‹@%Œ  ˆ ¼)Œ‹@h‹·“ |5 ˆ ‹5Œ‘5‘HZ ‹HŒ‹ŸP~c,d e5f9gh#f &e>9e>€sd2h ? ‘ ‹@ŒŒ‹HŒ Ÿ ‹;+

5‘   ˆ ‘ ˆ5’ ‹5‘|Ÿ ®‹5 ˆ =5 ÀZ ˆ 6) …

| Ÿ5‘|Ÿ +‹5B“ ° Á •‰ ˆ Šh)‘ Á  ˆ  ˆ ‘ ˆ5’ ‹5‘| ’ ‰ ‹Ÿ‘ ˆ ‹ Y>`Œ ‹p<ƒ ’‘ˆ ‘5Œ‹;+
Œ=B“ Œ‹5Z ‹´Z ˆ Œ•|B“ ŒŒ‹ ˆ ‘ ˆ5’ ‹5‘|Ÿ ‹'B“ Œ‹ŸZ ‹ ÀZ ˆ •= 0 ˆ Š= Y“ Œ‹Ÿ
Z ‹ … b ° ˆ ŠŒ‹Ÿ2 ÀZ ˆ   ˆ ‹5B“  U •‘0e#Fn…m,€^-l   h ˆ ‘ ˆ5’ ‹5‘|Ÿ   ÀZ ˆ  …
»  ˆ  ˆ ¼ đ•'Œ ‹Ÿ‹Ÿ5
   ‹   ˆ ‹ŸL  ˆ 
 ˆ Š
Œ‹Ÿ% ÀZ ˆ 
”  …

x Fi D ˆ Š® ÀZ ˆ   ¿  ; †‹5‹Č  ˆ ‘ ˆ5’ ‹‘|Ÿ m ÀZ ˆ  …2½ Œ‹” ˆ5Á 5Œ‹
 ’ 5‘%Œ‹ ˆ5’ † ‹ È À … b e‰ h´i ˆ Š´Œ‹Ÿ   ˆ ‹  ˆ5’ † Èv™x  ? °‹5‘‹
 0 Y“ ‘ ‹Ÿ 9)he0Œ Ÿ‘|1=‹5 •‘|1‹5 ‰ Y“ š™ ˆ5’  ’ L“ = ˆ5Á 5,+)1Z ˆ Ÿ

L“ ˆ5Á 5 …
» 5 ‹eB“ ‘Z ˆ Ÿ‘  ˆ + ‹Ÿi°) †‹5‹5ČH ¼ ˆ    ‹5 •‘5‹1  ˆ ‹U À ˆ ‘ ˆ5’ ‹5‘Ÿ 
 ÀZ ˆ 
; †‹5‹Č´  Œ´ ”¼; ˆ  ’  …

¦€Ztzw u Æ L€Ftw®­N§{u”¯H±³²´­N§¨ut=©\r(Ë


       
i ",/ +."      
         
 $ #2€'"*),+.-0/ %+ N/+ ):2 ) ",/ O220/ / 2 $ #20/ / 2 $p)C$ 2 + ):2 ) ",/+H / 

5/ $ O1 > ):" $&/ / $ ) O/ N" 4

  m > h ;j<

 8i           
2 / $ + ++H O220/ ./ ) +H /    
O %+H ):2.$&/     
O220/ ./ ) N" 4

 mt>kh  ; Z h 

@ dShACBh '
D x …i   ˆ ŠÈ ÀZ ˆ È ’ — +B p‹Ÿ •0 ˆ - ˆ • À
 Œ š™ …N “ ˆ ȋ0 À¿ ˆ  ˆ  ’  °
 ˆ ' pŒ‹Ÿ  ˆ  H)  ˆ Œ‹e ˆ ¼ ˆ Œ‹6+‘e@ À‰Z|Ÿ ’  m%> h  ˆ Œ ‹Ÿ5@   ˆ À5pZ ˆ   eL“ 

„·Ê
 ˆ5Á 5 …
i)‘  5 ˆ h5 2 ˆ Š|;; ˆ •‘2´Œ ‹Ÿ ˆ  ˆ (iQZ ˆ   2L“ ˆ5Á 5Œ‹Ÿp ³
 Á •
 ÀZ ˆ ,+‹5B“  ˆ ‘ ˆ5’ ‹5‘|Ÿ   ÀZ ˆ 

Œ´ ˆ Š'‹5 ’ % ’ À5‘•‘|
L“  ˆ ‘ ˆ5’ ‹5‘|Ÿ   ÀZ ˆ 'wi ‘ ’ ZÉ Œ‹É ˆ5Á 5Œ‹h‘° ˆ  …
CN  ˆ . ˆ ¼ ˆ '
  «ȋ‚“  H   ‹ •‘5‹•  ˆ ‹=  > ;  m Zi ˆ5Á 5Œ‹ ?  …
 ˆ  ˆ  ’  ’Ž ) … ´ ‹6+ À=‹5‘ AY ŒŒ‹Ÿ2 À0  Œ´ °¼) ˆ  ’ ,+‹5´5 5Y Œ‰¼; ˆ  ’   ˆ 

Z ˆ   ˆ '¿‘ x‘  …Z   ; Z …
¡ h‹Ÿ}+    mt>kh  ;  m x  muZ6{ hZ ;=< …
1; Àk ˆ ' ´Œ‹Ÿ! ˆ h;  ˆ  ˆ Š´Œ š™É ÀZ ˆ ki '   ' ‹ •‘5‹;l+ > '  ˆ5Á 5Œ‹p‘
 'Y ŒŒ‹;+F‘‰' Y“ ^ Ÿ 5' ˆ5Á 50‹‹É¼ ˆ ‹ ˆ À ÀZ ˆ  ’†…L½  -·  ˆ Š0) †‹5‹5Č
 ;  ' ‹5 •‘5‹;n + > ; > ' h ZÈ ˆ5Á 5Œ‹ …U ` ‹;+S À” -‘  ° ˆ5Á 5Z ˆ Èq  Œ`‘ ‘™
 -‘  Œ‹wY ŒŒ‹ ? ‹5B“ ‘  • ˆ  ˆ ‹5i‘È ˆ È ˆ5Á 5,+S‘  `Œ‹Ÿ‘ŸÄ ˆ ‘•‘|Œ |5‘
‹h0Œ‹‰‘‘Œ‹AY ŒŒ‹ …Z ' ‹;+Z‹| i ' Œ‹ŸhŒ š™,+;‘=š™ ˆ5’  ’ 0Œ‘Ÿ5
 ˆ5Á 5
B“ ’ %‹5 ’ 6)+‘%  
 -·¿ ˆ Š
) †‹5‹5Č  ;  ' hZAY ŒŒ‹ …
¡ É ”  mt>h  ;  'im Nx > '!hZ6{ h x  'hZ6H { ;=< …
; †‹5 ‹N5‘Z|P i  Œ ; †‹ |5Œ‹1Œ š™Œ‹ … CP‹5|B“ ‘  Œ‹1‹ |e‘™  ‘™
‹ 5Œ‹;+ )‘P Œ‹ ˆ ‘ ˆ5’ ‹5‘5 ˆ 2‹5 ˆ 5%25ŠZ ‘‰Z ˆ Ÿ‘  À†Y Œp •¼) ˆ  ’ %
ŠZ ‘'Œ‹É ˆ Œ‹ …9
¿)‘%Ÿ ‹5 ˆ À  ˆ ' 
 ˆ5’  ’ ‘|5;  ˆ ŠZ ‘'Œ‹%Œ ) †‹5Œ‹
Œ š™Œ‹ …2¡ 9‹ |¿5 5Œ‹°ŒŒ‹ ˆ ‘ ÀŸ ‹6+p À«‹5 •^Œ‹i '¼ ˆ Œ‹  ‹ •‘5‹ ? ˆ Œ‹Ÿ …
L“ ˆ5Á 5Œ‹)@ ´š™ ‘5‘•‘|p ‰ '¼ ˆ ‰5  Lh‹ •‘5‹ ? ˆ Œ‹5 … L“ ˆ5Á 5Œ‹)@i +‘p À‹5 •
Œ‹   ŒŒ‹H %š™ ‘5‘•‘H % '¼ ˆ %5  FK  ŒŒ‹H*i^†•‘| ’   rm Z2 ’ ‹P‹5
ÀwY Œ‰ ȼ) ˆ  ’ 
 ’  ’ Œ  ’  a ‹p‘°5  …
¿ ¼Ÿ‘|%   m3>|h  h x wmZ6H { ;=< +
 …  …  …  mt>kh  ; Zh … E

J*hnae>dVMO-h5l '
`;‘H 5 ˆ % ÀA ˆ ' ‰L“ ¡   ˆ  ‹5‹Ÿ ˆ É‘ ˆ h ˆ ‰|%5 5 ˆ ‘ ˆ5’ ‹5‘v

Ÿ  L“ ” ˆ Š  S ÀZ ˆ 'Œ š™Ÿ ‹5 ˆ   Á •' '¼ ˆ 0kY ŒŒ‹ …F˜ “ Œ‹Ÿh;  ˆ  }+
‹2ŒL‹Ÿ;+ °Z ˆ  ˆ = ˆ Œ‘5‘•‘|‰° '¼ ˆ 
AY ŒŒ‹2L“ ° ˆ Š ÀZ ˆ 
Œ š™
‹ ‹2 ‹KY ˆ  ˆ5’  ’‘ˆ ‘Œ  
|‘ Œ  ˆ ‘ ˆ5’ ‹5‘Ÿ ” ÀZ ˆ  …

.;‘• ˆ - ˆ |i À  ˆ ' ”L“ ¡   ˆˆ Œ‹Ÿ5  ˆ ,+1 Á •¿‹52 i ˆ Š°; †‹‹5Č°Œ‹
¼) ‘ Œ‹ ? ”‹5L Y“ :‘% ˆ  Œ ˆ ´ ˆ Š
‹5 ˆ ‹ŸŠÄ ˆ  5¾ É
‹2 ´ À9) …
FHG h dS‰@€z‰5h 'C ˆ - ˆ B“  FŒ‹ŸP) †‹5‹Ÿ¼  ˆ  ŒŒ ˆ  ˆ Œ‹@ ˆ ‹@‹5 ‹ ™• ˆ ‹P‹ŸŒ‹H‹‹@
‘™°Œ 5Œ‹‰‹5´ ˆ ‹5‘| …
[m \rz€m> '
D CN ˆ Y“¼‹5 ˆ :'B‹Ÿ) †‹5 ‹
= = ˆ ŠŒ š™•Œ‹ ˆ ‹‹ ‹'‘
 ˆ ‹‹5Œ‹
‹5 
 ÀZ ˆ  …

  ;  / > ;  …  H ‹6+v‹|   ˆ Š2Œ‹ŸN¼Z ˆ Ÿ5 ? = ˆ ‘  Z ‹1‘™0‹5 ‹
‘ ˆ `‘  Œ‹6+B@‘™ ‹ŸŒ‹0‘| ˆ È‘  Œ‹5)+N‘`Œ‹(  ŒŒ‹B“ Œ‹Ÿ
  ˆ À† ,+U‘ À Y Œ• 
¼) ˆ  ’ •B“ Œ‹5
Z ‹' Q ‹ ’  ’ `Z ˆ  ˆ ‹0 ˆ5Á 5Œ‹ … | ‹Ÿ}+L5 5q  Œ•Ÿ ‹5` • ‹ –
 ˆ5Á 5Œ‹6+5ŠZ ‘‹5 ˆ  2;  ˆ ‘™~Y ŒŒ‹ … “  ¿ Y“  ’ |  ’ <#>5s  … 
˜ ‘ À«Œ    
( …
hˆ +%L“ ˆ ȋ° À  ˆ '  L“ ¡   ˆ +É m  ; < h > m  ;  …
˜  ˆ  ‘Ÿ  … E

*hnae>dVMO-h ' ˜  ˆ5’ ‹5 ɋ·“ ’  Œa' ½ 


 ˆ Š
J
 ´B“ Œ‹Ÿ%Z ‹2 ÀZ ˆ  …

s'tv€F¥Ut y Ë

  $ i #2~'"*),+.-0/           
%+ N/ + ):2 ) ",/  5/ $ / $ > ):"  

$&/ 23)54

  > m 

„†„
@ dShACBh '
D » 5 ‹%L“ ˆ ȋ% ÀpÍ ˆ  
L“ ¡   ˆ +Z(i«‹5 %Œ š™
° +    h  s >h < …
C1‹50i Œ‹5ɋŸ ,+;5ŠZ ‘0‹5Œ‹  ŒŒ‹hŸ ‹50”• ‹ —  ˆ5Á 5Œ‹ …  ’ ‘ ˆ x|‘•‘;+
ŠZ |
 ˆ5Á 5´5 ˆ ‘h‹Ÿ ˆ ‘™~  ŒŒ‹ … | ‹Ÿ}+ ¿  
<#> …¡ + ˆ ‘; ˆ É‹2 À ˆ ‘ ÀŸ 

L“ ¡   ˆ + x‘| <|h >  h   h


(

> …¡ h‹Ÿ#' > 
 m  …  E

FHG hdS‰@€z‰5h '-C ˆ - ˆ ´  ˆ Š

 B“ Œ‹Ÿ2Z ‹% ÀZ ˆ  …

m \rz€m> '
[

D ½ ^ ˆ 
ZOY 
 Š
2* m


 Œ‹Ÿ°D ˆ  Š Œ š™   ; «‹5 •‘5‹ ‘ ~> ; ZOY\ ˆ5Á 5Œ‹ …ɘ •
+1 2  ’‘ˆ   ° + ‘°;‘• ¿Z ‹ Á  ˆ 
 Z ‹ Y“  ’ |  ’ ° À  ˆ  ˆ  ’  ’ œ @
 ÀZ ˆ  … E

˜ •É …x‘|Hp   ˆ +† ÀA ˆ ' ÉL“ ¡   ˆ ) ˆ •‘@p ˆ - ˆ B“ • ˆ ŠɝŒ š™
B“ Œ‹5BZ ‹B ÀZ ˆ ,+‚‹S‘  PP; ˆ •‘UZ ‹BP ˆ - ˆ |B“ 
 ˆ ŠHŒ‹ŸB ÀZ ˆ  …v½ %Œ‹ŸŸ 
´ À ÀZ ˆ  ’ ¿ ”L“ ¿ ˆ Š ’  ’ˆ5’ ‹5 Z ˆ  ˆ 5 %‹ 8  '
:x‘|É  ˆ ´  

 2


‰‹5 |  ÀZ ˆ Œ‹ …xà ;Œ‹Ÿ„Y ‘ ÉL“ ‘i ’  ˆ h%5   ˆ Š‰|;Œ |Ÿ‘ ˆ 2
‹5¼;&x‹Ÿ ®' Y“ ®Œ‹´ŒŒ‹‰‘™  ˆ ŠŒ‹Œ •0‹ ‹Y ˆ Š0B“ Œ‹ŸhZ ‹‰ ® ‹‰ ÀZ ˆ  …
 
½ ÉŸŠ ’ ˆ đ•´  ˆ 5 %‹ 8 B ˆ •´|É À ˆ5’ ‘ ˆ x|´Œ‹Ÿ  ˆ  … ®‹HŒ‰ŸŠ ’ ˆ đ•´Œ‹Ÿ2 ‹5‹5@7
 ’ ·   ˆ - ˆ + ‹2  ˆ ‹% 
´ •‘Ÿ  ˆ ‘ …

'¦ | €ZtzwQu

¿­St¤·€ •} ByP©\r F¸Zµ  +

 2 '"*),+.-0/ /      
$ + ):2 ) ",/ / $ / N/ 5/ 2.$      
   
20/ O2.$ / 2 $ + ) / 2 /
 !  /


   "    "%$
²ButvÇ|y{ÇuQrxµ4
C ˆ - ˆ ,+B)  ˆ   ˆ Š` ÀZ ˆ `  ‹ •‘5‹0‘  ˆ5Á 5Œ‹‘0 |‘aY Œ

>

B“ Œ‹52¿ ˆ À† ,+  R> <  m …

²ButvÇ|y{ÇuQrZr* /.§0Sw ¥ey{¤¯Uu”¯Uu Ì; 3s‰u·utv} €F­Utvº4


 ‹@•ZŒ ‹;+ Z ZE  Œ‹H‹5 | ˆ ‘  ’ Œ‹@‘™  ‘™• ˆ Œ‘5‘•‘p‹ PZ ˆ h5 ˆ 5h‹5 
Z ˆ 
†0'Š‘®(Í ˆ ? |Í ‘Ÿ ‘´‹h Œ‹‰‘™”‹5‘‹5) … C ˆ - ˆ B“  Uš™‹Ÿ5
 ˆ  ’ •‘|2°) |2‹Ÿ ˆ Œ‘ L‹5 p
5 ˆ 5´Z ‹5‹5Œ‹5‹Ÿ‹2L“ 
 ˆ ,+‹5 p 
Š‘   ˆ Z ‹5‹5
Œ‹‹Ÿ‹%L“ ' ˆ  …

²ButvÇ|y{ÇuQrƒ+
 qtsjZp•‘|Ÿ ˆ·…
•Œ ‹5Ä ˆ 2 p ˆ Š%‹Ÿ i  P Œ‹1‹5 •‘5‹P‹ |P Œ‹1‘Ÿ ˆ ‹
x e
%Z  q„+ Œ‹@‹ •‘5 
‹ >0‘ ’ | ˆ ‘  ’ ‹PZ ˆ ‰ ˆ5Á 5%‹ŸZ‘@‹‘ ‘•‘|H‹5>eh pŒ‹ŸP• ¼ ˆ  
 ˆ ‘ ˆ ? ‘É  „ B“ Œ‹Ÿ%Z ‹ ˆ ‘  ’  ³Y Á •6) …
‹ q^;  ˆ Œ‹|‘ ‹*i Œ‹Ÿ% ÀZ ˆ  …
‰’ 5 ˆ  ˆ Œ‹%‘|Ÿ ˆ |

²ButvÇ|y{ÇuQrx¸4
¤ C ˆ - ˆ |´‹Ÿ i Œ‹ŸpÈ ˆ Š‹5 ‰Œ š™‘p ÀZ ˆ ,+Œ |%i '¼ ˆ   L
‹ •‘5‹;+Z  ˆ ‹*i ) †‹5‹5Č
°• ‹É¿‹5 •‘É
‘ ˆ5’  ’ Z ‹‹|hZ ‹ œ…
 ˜ ‘ À ˆ Œ‹55šYY  ˆ U‹Ÿi B“ Œ‹Ÿ% ‹%‹Ÿ) †‹ ’ Œ š™ 9

„·Ž
²ButvÇ|y{ÇuQrÆ 
x ‘|  O/ xOxOx;/ Œ‹; 5‹'‹ŸŸ‘5‹  À …
  5  Ài ‹
;‘Ÿ5•‹5Œ• 
  
‘| ˆ ‘™iŒŒ‹%; 5‹ … C ˆ - ˆ |B“  LB“ °0Z ‹2 ‹% q~m  Z ˆ Œ‹ v / {
)  ˆ
Œ‹|‘  Œ
‹  ; Z…

²ButvÇ|y{ÇuQrË+

•Œ ‹ŸÄ ˆ p•‘‹5‘'¼    ; 5‹P À+  ˆ ‹5‹P † ’ ‹P‘1|Z ˆ  5‹
Œ x‘ ‘ |Œ‹ …
8. ‘†17‹5 |‰‘™°; 5‹h‹ŸŸ‘5‹´Z ˆ 5‘Z|  ~+Z ” ˆ | ÀZ ˆ  v .0/21 {=Œ‹Ÿ
,2m>  h‹·“  Bš™‹Ÿ5´”‹5A  ˆ  ’ |SŒ Ÿ‘
‘•‘|A. ‘*1 …

¿ 5  x  { ´ ¼ ˆ '¼) Œ‹2Z ˆ Œ‹Í ˆ  ’ Œ‹2Z ˆ Œ‹ ’ ’ •‘|5‹‰
 …
C ˆ - ˆ ,+‹Ÿ ˜  ˆ q x pH{ ; ZOY Y   ˆ ‹ <#Y Y 
 x p{ 
#Y Y …

!#"%k5"2&)35:%8É41(

  !" „   #"


    .  (

b  ‰ m < m,d5€e;c h´` ˆ Š†i^Œ ‹5‹Ÿ5  Ÿ ˆ ¼ ˆ ɝŒ  ‘ ˆ  ŠZ ‘`ɋŒ‹@‹5 •‘5‹ …½ 
Œ  ˆ À ÉŒ‹Ÿ18f d m5f dShÉ  ˆ ‹5p‘™=‹5 •‘5‹H Ÿ Œ‘5‹12‹5 | 5‹P À‰ Á •2Œ  ‘ ˆ·…
;“  )š™‹Ÿ5hhŒ  ˆ Ÿ ° ˆ  ˆ ´ki  Y“ `Œ  ‘ ˆ ‹;+ i ˆ 0AiQŒ‹ŸŒ  ˆ ¼  …
½ • ‹;‘Ÿ0‘|Ÿ ˆ  ? ‹‚“  @š™x‹556)´)  ˆ Œ‘ KiDŒ‹5 Œ  ˆ ¼ ÈŒ‹Ÿ0)‘ ’ F9m>n ,d2h
‰@g d m,n…erzN€ M, h0(i +‘% ¿ ´ 5
 Kxi { …

» 5 ‹0 Y“ š™x‹Ÿ5‘Œ= xi {‰Œ‹Ÿ


 ‹5‹5 ˆ5’ 0‹´ =· ‹ ® 0 ˆ Š=0; †‹5‹Č0B“ 
 ¼ ˆ   B
‹ •‘5‹ ? ‹;+¼‘”‘5‘+ZŒB“ Œ‹52Z ‹2
Œ Ÿ ” ’ ŒŒ‹‹ ˆ 6) …

 @‘ ‹  Y“ š™‘ ¼9)p´ Y“  ˆ ‘Ÿ  '


˜ • ’  x ’   ˆ ‹;+NŒ ‹Ÿ ˆ ‹5 ‹0 ` ˆ ŠÈ  Œ‹=‹5 •‘5‹=‹5  Œ‹ ˆ x5‹0Š³
Œ‹;+U‘™ ’ | ˆ ‘  ’ ‹‹Ÿ ‘‹5‘ ‘•‘3l € Pl… h(f-hACBh%r!f-el Á  ˆ  ˆ ‹Ÿ; ˆ  ’ ‹=‘‹5‘¼  …
ˆ ‹Ÿ) ˆ 5 ˆ iŒ‹@ ˆ x5‹ ‹@i·  ˆ @x‘|  Ÿ ˆ ¼ ˆ `| ’‘ˆ = `hŒ  ‘ ˆ
 Œ ˆ x ? Œ‘   · «Œ ˆŸˆ Œ‹Ÿ; 5) … | ‹Ÿ}+B5 ‹' Œ‹ ˆ 5‹0L“ • Á •ȝŒ 
‘ ˆ ‹5 ˆ |
 ˆ ‹5; ˆ  ’ ‹‹
  Á ••·  …U½ Œ‹
Œ Ÿ ‹0•‹ ’ ‘ ˆ  ’ ) †‹5‘ 
L“Ÿ ˆ ¼ ˆ ŒŒ‹pŒ  ‘ ˆ ‹2´‹5 ˆ 5‰|´‘™ ˆ x5‹% 5 Œ‘|5‹pB“‘|2Z ‹ À Á •´Œ  ‘ ˆ·…
˜  Y‹;+L  Œ ‹Ÿ ˆ '  Œ  ˆ À ˆ  ˆ ^ ˆ Š,+B‘ = ˆ ¼ đ••Œ ‹Ÿ‹Ÿ 
B“ Ÿ ‹ ˆ % h• ‹ ; †‹‹Ÿ¼ h‰· ‹ ˆ @x‘|   ’ 5 ˆ  ˆ h '¼ ˆ ´5Š ˆ Ÿ
 ˆ Š …

FHG hdS‰@€z‰5h ' ´’ 5 ˆ  ˆ 2 '¼ ˆ ‰5Š ˆ ŸhŒ‹@ ˆ ŠŒ‹HÉ À  † ˆ  „·Ê ? Œ‹ ˆ5’ ) ‹5Œ‹@‹ |
  ’ Œ‹‰ —…Ž ) …



 Œ Z eZ ‹N# ˆ ' p‹Ÿ p|; ˆ •‘Ÿ5p  ’ 5 ˆ  ˆ @ ¼ ˆ p5Š ˆ Ÿ
L“ ” ˆ Š'‘ Œ | …à SŒ‹Ÿ%‘ À ˆ |
‹5L  ˆ Š0i ) †‹5‹5Č q ‹5 •‘5‹6+Z  Kxi {  q
? iŒ  ‘ ˆ Z ˆ ‹5 •‘5)‘0` Y“ ’ |  ’ ¿ ‘.‹ŸEi ;
F… CN  ˆ |;+P‘  ’  ’‘ˆ  }+1Œ‘Ÿ5
 Ÿ ˆ Ÿ ®Œ‹Ÿh ˆ ȋh-†‹5,+F‘‰  )‘hŒ‹‹Œ  ˆ ' Y“ ’  ˆ  ˆ ‘ Ÿ ‹|Œ‹‰Œ  ˆ ÀŒ‹
 ‹  Z‹;+ ”‘¿Œ‹5‹ Œ ||‰A  ˆ 
|5 ˆ ‘ ˆ Œ‹%Í ˆ Ÿ ‹‰‹Ÿ ˆ À=‹Ÿ ˆ ‘Ÿ ˆ (i …
½ ´ Z ˆ @Œ‹ ’   ZŸ ‹H
  xi {N ˆ 6‘‘ L“  ˆ ŸŠ•Œ‹ %Œ  ˆ ÀŒ‹ …à ‘•š™‹Ÿ5
¼)·Œ +‘p;  ˆ ‰Z ‹p ‹p ·  ˆ ŒŒ‘Ÿ5Œ‹ŸŸ +x ‹p ‹p¼; ˆ  ˆ ‹   ˆ5’ ‹5‘5 ˆ
´ ‹h ’ đ¼ ˆ 
L“ ‘| ˆ ‘™ …

„·—
X Y





Z
 

[ \s]K_ „·Ê ž ‰’ 5 ˆ  ˆ ´ '¼ ˆ 5Š ˆ ŸŒŒ‹2 ˆ ŠŒ‹

 % u   8   "
    H 

ÃB ‹·“†%
Y“  ˆ ŸŠ•'
Œ  ˆ À
‹5& w'
x  i J ˆ Š  q ‹5 •‘5‹ …
 ˆ   Œ‹`‹ •‘5‹;/  ( /Oxyxyx&/  ‹5‘  \ Œ‹
‘ ˆ5’ ‹h ’  ˆ ‹5‹5‹ ?  …  … )  ˆ 5  }
 x { s  x   {S{ …
|‰|B“  ˆ Œ‹55
Œ‹%‹5 •‘5‹  Œ  ˆ  ˆ + ”š™ ’ ‘5 Œ‹%‘™” ‘Ÿ ‹%‹Ÿ&†|5Œ‹0'
) ˜ Š ˆ 5Š ˆ ‹h À ‹Ÿ5
ˆ   ’ 0Œ‹‰‹5 •‘5‹É ' ˆ ‘ ˆ ‹5 •‘‰  ‘Œ ˆ Œ  ˆ  ’
‘2 'Œ  ˆ  ˆ L“  -‘  'Œ  ‘ ˆ·…
¼9) ˜  ˆ  ˆ ;ŒŒ‘Ÿ5' Á •'Œ  ‘ ˆ +;‘h‘ ˆ Œ‹5;Œ‘h ‘ ˆ ˆ  ˆ '‹h À= ‹Ÿ5,+5 ‹É Œ‹
‹ •‘5‹1 •‘Œ ˆ 2Œ  ˆ  ’ ‹@2‹5 |1Z ‹@ 5 Œ‘|5‹ •‹5 •‘1 ˆ5’  ’ ‘| ‘eZ Ÿ Œ‘5‹
‘ ˆ 
‘™ …

à Œ‹Ÿ‘ À ˆ |,+S‹5• ` '¼ ˆ i‹5 •‘5‹0  Œ  ˆ  ’ ‹0iL“ • ‹0


 ’  5ŠZ w ‹‰| Y“ ^š™ ’ ‘5 Œ‹‰; 5‹´)É‘‰¼9)+FŒ‘Ÿ50 ˆ x ’  ˆ 0†•‹'5 ˆ  ˆ ‘
  ˆ  ˆ ¿Œ  ˆ À' ˆ  ˆ  ?  … ¼9)S)2wi …
 ‹%š™ ‹Y 
L“ ”;‘° ‹É ˆ ȋ('
x   p ˆ  ˆ ‹5 •‘1Œ  ˆ  ’†…  B“‰Z ‹ ’  ’ Œ  ˆ  ’ ; ;+††“ Œ‹Ÿ1| )  ˆ ŠZ ‘
Œ‹@Œ  ‘ ˆ ‹P ˆ5’  ’ ‘|5Œ‹6+|•‹5 •‘  Ÿ Œ‘    ’  ’ Œ  ˆ  ’ %Œ‘Ÿ5ɝŒ  ‘ ˆ·… CN ˆ ‹Ÿ5,+
= ¼ ˆ `Œ  ‘ ˆ ‹Ÿ ‹ ’ Œ‹0†|'•Œ  ˆ  ˆ  =;‘' ’ Z ‹‹5 % ˆ  x  { …S¡ ' ,+B‘
5‘Z%Œ 5
‰ À0Œ  ‘ ˆ   + ° ’ %|‰ ´ ¼ ˆ ´5  BŒ  ‘ ˆ ‹pŸ ‹ ’ Œ‹2Z ˆ
Y“  ˆ ŸŠ•'
 ’ Z ‹5‹5´Z ‹  x   { h Z …

”‘¿ ’ % À= ˆ  ˆ  ’  ’ ‹Ÿ& 5a'

s'tv€F¥Ut y (

 i      
xi {             i
 
 $ #2~'"*),+.-0/ / q  5/ $ 2 20!$&/ N/ /z'" ) ) #2  5/ $ /



N" 4

Kxi { xi { hZ

J *hnae>dVMO-h ' | Ÿ5‘|Ÿ +B Y“  ˆ ŸŠ•†  5  ) ˆ •‘
L“ ¼5‘ ˆ ^Œ  ˆ À ˆ  ˆ  ‹  
Z |' Y“Ÿ ˆ ¼Ÿ ^L“ 'Œ  ‘ ˆ  5ŠZ 0‹5 •‘ … Ce ˆ Œ  ˆ ,+; SB“ ¿‘ ˆ ‹5 
)  ˆ |p pŒ  ˆ ÀÉ‹5 ¼5‘‹5  ˆ5’  ‹ ’ ;Œ2  • ‹PpŒ  ‘ ˆ ‹1; †‹‹Ÿ¼  … |  ˆ ‘•‘
;+ ef9d€m>d5€z+ L
 Z  ‹  xi { …

m \rz€m> WI ‡ h
[
z Gh h@d ‰@€‰ h…‡  W Q   X '

„‘–
D ½ ´ ˆ Š(i %Œ‹Ÿ2Œ‘ BŒ‹p ˆ ‹2‹5 ‹2‘2 ˆ ‹p‹5Œ‹ …Z˜ •´ BŒ‹Ÿp¼Z ˆ Ÿ5,+ L‹Ÿ •
  ˆ `Œ  ‘ ˆ ™^ ˆ ‹‹5 ‹'‘` ˆ `™  ˆ ‹'‹ŸŒ‹;  ˆ ¼5‘ ˆ 
¼Œ  ˆ Ÿ i†i  … | ‹Ÿ}+  xi  { 
 ‘5 ?  ; •'• ‹H
< …½ “  ’ |  ’   ˆ ‹5 ’ | ’ 

 ˆ5Á 56)+Z 
  xi  e
{ ;=< …

b Ÿ ‹5 ‹% Y“  ˆ ŸŠ•


†  5 ”)  ˆ  Ÿ ˆ  ˆ  xi (6{A'
b 
‘ À ‹5‹‘•‘|SŒ‹U‹5 •‘5‹B‹B Y“ ˆ  ˆ @Œ‹B‘ ˆ5’ ‹S ’  ˆ ‹5‹ |5‹SŒ‹5 a/2.0/21~/ / /E/ /54 …

#Ÿ ˆ ¼¿ ° À®Œ  ‘ ˆ    a/ ‘34 … C1‹;+e À®Œ  ‘ ˆ  (  . ‘ … C1‹;+e À
Œ  ‘ ˆ   17‘  …¡ É‘  + À=Œ  ‘ ˆ  (  …%
”‘¿ ’ ‰| Kxi (6{ … 
 ‹;+ ‘• ¼‹5 ˆ †|1   ˆ Š,+ Œ ‹Ÿ52p   ˆ Š2#i ( '  P Œ‹1‹5 •‘5‹P‹ |
.0/ a/ / Œ‹ŸÉ®‹5 ‹Y ˆ Š0Œ  ‘ … C1‹5B“ Œ  ˆ Ÿ ^pi ( h0Œ  ˆ Ÿ 
(i ( ' + ” É;  ˆ Kxi (6{ s …¡   Z ‘•‘;+  xi (6H { ; … E

 *" r 
H  "  # . 
 ( „" 
 !#"


b  ˆ Š#i ’   ™ ’ +Œ '‹51 1PZ ‘Ÿ5PŒ  ‘ ˆ ‹ …


';‘S  ˆ ‹U‹51‘ ˆ
'Œ '¼‘ ( >Œ` ‹É ”)‘+Œ  ˆ  ˆ i …½ • ˆ  ˆ  ’  ’ ‹Ÿ&†|5; ˆ 5Œ‹ ’ ’ •‘|5‹‰
ˆ5’ ) ‹5 …

s'tv€F¥Ut y 

 i                    2 5/ Ol/  
   
      i  qx {
 $ #2R'"*),+.-0/ %+ N/ / q  5/ $ ./ $&/
#2%+e R1 / /z'" q / $ / 2.$

C/
       )  x¾ •5Š ˆ Ÿ  i 
/ 2.$ / " ,1 $&/ 1 +H!#"%$&!$  1 N/
20 ",/ /  O N"*)C$ 2 /  $ 

 /+e 2 5/ / $ ),++e/ N/ /

@ dShACBh '
D Ki9Œ‹Ÿ'  ˆ Š`‹Ÿ È q\‹5 •‘5‹=‘w>  ˆ5Á 5Œ‹;+U   ’ 0 ÀBr}e>€ &hÈ3iDŒ •
’  ’ |   q h> …
  ; xz.0/21a{ ' ˆ5Á 5
0i …
x 
 m>%rzdSe‰rh
d 
‹Ÿ†  hB“  Z ˆ Ÿ ˆ Ai +x iŒ ‹5 ˆ  È -·° ˆ Š,+x  ’ i+x‹
Œ‘ Œ‹0‹ •‘5‹ . ‘ 1 ‹ | ˆ ‘ À  ’ ‹=Z ˆ «‹5 •‘=,+ ˆ ‘  ’ Z ˆ i ˆ5Á 5 
ŠZ ‘ Œ‹%‹ •‘5‹É ’ ‘| Ÿ Œ‘5‹  . F1 +‹2
‹5‘ kÍ ‹%‘h‘ ’ Z|
? ŒU ‹5‹5 ˆ |( i Œ‹Ÿ%‹Ÿ 6) …
CN ˆ Œ • x ’ +Z ” 5 ˆ a i  
‹ ‹Y ˆ Š
wi ¼5‘| ‘ ’ Z|h Y“ ˆ5Á 5!
  …
Z Z Z

X Y X Y X'&yY

 #"%$ )(*$

[ \s]K_ „†„ ž ˜ | ˆ  ‘Ÿ ®L“ ' ˆ5Á 5



:  05‘Z|‰ ˆ - ˆ  ˆ5’ ‹Ÿ ÉZ ˆ2ˆ5’ ‘ ˆŸˆ ‘Œ'‹Ÿ ˆ À=  ,+ (i '
i Œ‹Ÿph  -+H; Z,+†“ Œ‹Ÿ2|B“  )Œ‹ŸpŒ ‹ŸŸŸ ’ L“ i|´‹5 •‘ … ‹pŒ´· ‹;+|)  ˆ
5 h‘|Ÿ ˆ + ´ ¼ ˆ ' Œ  ˆ Ÿ ‹‰Œ‹ŸÉ  ˆ ‹%‘ À ˆ ‘•‘ ’ |   …
x -
 + s Z  ™ ’†…
®‹Ÿ; †‹' ÀȝŒ ‘ ‹Ÿ  ’ ¼ 0;  ˆ 5 ´ ˆ Š=‹Ÿ ='  
 ‹ ’ |   + …

„·œ
x †i ¿ ˆ Š
‹Ÿ 

   + h=Z …
S%i”p) †‹5‹5Č2‘% ˆ5Á 5%  ˆ ‹;+† =;‘@Ÿ ˆ ¼ ˆ B“ ; ˆ 5p À ‘  2Œ‹0Œ  ‘ ˆ ‹
 5ŠZ ‘«Œ% ‹ q; +Hh Z‹5 •‘5‹;+U H‘ ˆ5’ ‹Ÿ 5 ˆ ”5 >  ˆ ‹Œ  ˆ Ÿ  … Ce ˆ ‹55,+B 
 ¼ ˆ ` Œ  ˆ Ÿ ‹=3i9Œ‹Ÿ ˆ Œ‘5‘•‘  +SŒ`| Œ‹Ÿ'¼‘ Q)  x¾ •‘ +S
‘ ˆ5’ q ‘  Œ  ‘‘5‹h‘Ÿ ˆ ‹ …
  i ; †‹5‹Č
¿• ‹2' ˆ5Á 5a' 5 ‹h|B“  ˆ w  ‹ Z >%  
+p‘|
 < q + …
x   %; xz.0/21a{2= ˆ5Á 5pi …F½ Œ‹É ˆ ŠŒ‹ki 0‘i 0‹5 |´ '5 ‹‰ Œ‹‰‘™”Œ‹
 ˆ ŠŒ‹1‹5 Œ‹N   Œ‹ ˆ Œ‹Ÿ)Œ‘Ÿ&Œ‹e  ’ Z ‹5‹|1Z ‹ + …  “ ˆ ȋN Y“ Š ) ŸŠȋ52 ˆ5’ ‘ ˆŸˆ ‘Œ,+
\;‘È  ˆ ‹`Œ ‹Ÿ ’‘ˆ  ˆ Œ‹•)  x¾ •Œ‹`Š ˆ Ÿ|Œ‹i i  ‘3i  +H|° Y“ \ 5 ˆ 
ˆ Œ‹5;Œ‘Ÿ&‘•‘  m‘   …
ȋ H% ‹    ‘  ‹5 |  Œ x ‘‘|5‹p‘|Ÿ ˆ ‹6+   ; q
‘    ; q m Z … % Y“  •  ˆ ”,+e;  ˆ 5  +1 ° ¼ ˆ ” Œ  ˆ Ÿ ‹ÈFi Œ‹Ÿ
qx  ({ m ax  {+ À=Œ ‘ ‹5 ”‹5 ˆ • ‹5‹5 ˆ5’  …
˜ ‹Ÿ ’‘ˆ ‹
Œ‹ Œ  ˆ Ÿ ‹= ˆ ¼ ˆ  ˆ Œ‹'ai -•)  ˆ Œ‹5‘  Œ‹ Œ‹
‹5 •‘5‹w. ‘(1
‹ |ɍŒ  ‘ ˆ ‹h<ƒ ’‘ˆ ‘5Œ‹0'
| Z ˆ Ÿ ˆ L“ ¿5‘   Œ  ˆ Ÿ D i ! +@ \ ¼Ÿ‘i¿  Œ  ˆ Ÿ D i
‘ ˆ5’ ¼ ‹5‹  +FŒ' ˆ  ’  ’ L“ ‹5‹5 x‘ÀŸ  ’ |‘ À ˆ ‘•‘|´¼ ŸŒ‘Ÿ< …;à U ”• =|
Œ  ˆ Ÿ ‹‰(i  '
Œ´ x;B“  B °= Œ  ˆ Ÿ ‹É0i …

˜ ‹Ÿ ’‘ˆ ‹


Œ‹ Œ  ˆ Ÿ ‹= ˆ ¼ ˆ  ˆ Œ‹'ai -•)  ˆ Œ‹5‘  Œ‹ Œ‹
‹5 •‘5‹w. ‘(1
‹ |É´ À= Á •'Œ  ‘ ˆ '
˜ ‘Ÿ5 Í ‹;+  Z ˆ Ÿ ˆ L“  Œ  ˆ Ÿ m®Œ® x;,+ À Œ | ˆ  ‘Ÿ m ®) ˆ •‘¿L“ ‘
 ’  ˆ h  Œ  ˆ Ÿ ¿Ai *+| À'Œ  ‘ ˆ Œ  ’ %Ÿ ˆ ¼ ’ ´È -·i‹5 •‘ …
|  -·+FŒ
 ˆ x ’  ’ L“ ‹5‹5 x‘ÀŸ  Œ‹5%<ÍŒ‹55‘•‘|‰
¼ ŸŒ‘Ÿ  
Y“ ‘‹‘'¼ 0Œ+ ‹ 
Œ  ˆ Ÿ ‹Éi  ´Œ‰ x;´‹Ÿ ˆ Y“ ‘‹5‘¼ 
Œ ‹ Œ  ˆ Ÿ ”ki   +xŒ´B ‹5‹Ÿ ˆ ‰|‰ 
 ¼ ˆ ' Œ  ˆ Ÿ ‹‰w i  
‹hŒ‘™xđ•· ‹%Œ‹Ÿ ’ |  ax  { …

«‘. ’ • i
' qx  { ; ax  z{ h  '¼ ˆ ° Œ  ˆ Ÿ ‹•:i +eŒi| ’ 
  ˆ ' ¼Œ‘Ÿ< … E

FHG hdS‰@€z‰5h ' ´’ 5 ˆ  ˆ ´)  ¾ •


Š ˆ Ÿ|L“   ˆ ¼ ˆ   ^
q ‹5 •‘5‹ …

[ m \rz€m> '
½ ´;  x¾ •
Š ˆ Ÿ|L“   ˆ ¼ ˆ 
kq ‹5 •‘5‹ÉŒ‹Ÿqx {;  xRmZ6{ 76 …

D

 ˆ ‹5 Z ˆ2ˆ5’ ‘ ˆŸˆ ‘Œ‹Ÿ ˆ q '
 q ; Z,+ ” ’  ¿
qx {;! …
x  qtsjZ  ™ ’†…
0‹Ÿ) †‹5P,+·)  ˆ 5 N ˆ ¼ ˆ HHq‹5 •‘5‹6+‚  qx {H;! xmwZ6{ "6  …
x   ˆ ‹ # ˆ ¼ ˆ ¿5  q3h Z°‹5 •‘5‹;+1‘   ‘Ÿ ˆ·…
 ‹• ) †‹5‹5Č¿
• ‹
 ‹5 •‘
‘ ˆ5’°„ ?  … š™ ˆ ‘Œ „ )+L‹5 ‹(. …)¡  ’ Zw.›‘ Y“ ˆ5Á 5 

Œ‹5hš™ ˆ5’  ’ +L ^ ¼Ÿ‘|  ˆ ¼ ˆ  'U q«‹5 •‘5‹ | 0 ¼ ˆ • Œ  ˆ Ÿ ‹
Œ‹5   x  m Z6{ 76  ? L“ ˆ ȋ2 Y“ Š ) ŸŠȋ5 ˆ5’ ‘ ˆŸˆ ‘Œ6) … CN  ˆ ŠZ ‘
´ŒŒ‹ Œ  ˆ Ÿ ‹;+ 
)‘2Œ ‹Ÿ ˆ  ˆ  wm Z Œ  ˆ Ÿ ‹É +‘¿Ÿ ˆ ¼Z|  .mB“ ; ˆ 5‘  ´Œ  ‘ ˆ  ˆ 
'Œ‘  = ’  Ÿ ˆ ¼ ’ ®‹5‘ N‹ •‘‰S eŒ‹Ÿ Ÿ Œ‘ … 5Œ‹‰ Œ ‹ Œ  ˆ Ÿ ‹
¼5‘Œ‹‹ŸS‹5 h‘ À ˆ ‘•‘|‘™  ‘™¿‹ŸŸ‘5Œ‹6+ZŒ'S ¿5  e  x  m Z6{
Œ  ˆ Ÿ ‹p) †‹5‹Ÿ¼ Œ‹p …-
hˆ +| LŒ‹Ÿ ’ ‘2|B“ 
 Œ  ˆ Ÿ ¿ )‘H5 >Ÿ  ˆ ‹ Á  ˆ 
Œ ‹5 ’‘ˆ5’ hŒ •hhŒ  ˆ Ÿ ° '  À ‘  ‰ È0 ‹5‹5 x‘ ’ ɝŒ  ˆ Ÿ i) ˆ Ÿ‘|5´
. +Œ
S ‹5‹Ÿ ˆ ´ 
 ’ Œ 5'‘³Œ‹‹Ÿ‹ ˆ ŒŒ - ˆ 5 5Œ‹% Œ ‹ Œ  ˆ Ÿ ‹‰ … E

FHG hdS‰@€z‰5h ' ´’ 5 ˆ  ˆ ´)  ¾ •


Š ˆ Ÿ|L“ ”· ˆŸˆ5’†…

m \rz€m> '
[

D
°Ÿ ‹5 À0 ’ ŸŠ x
‹5‘¿ À Œ'‹2 ´ŸŠ ’ ˆ đ•,+Z‘”Š ‹Ÿ‹5‹ ; xz.w/21a{'

„·Å
N ^‹Ÿ ˆ • +;  ˆ Š= ¼5‘| Œ‹Ÿ  ˆ ‹´® ˆ ¼ ˆ   Z ˆ 0‹ •‘5‹ …
 x‘|
  ˆ |´ ´ ¼ ˆ ' Œ  ˆ Ÿ ‹‰Œ‘h ˆ ¼ ˆ 
Œ‹Ÿ x mZ6{ …
1 Y“  Œ | ˆ  ‘5) +) = ˆ Š• ¼5‘|QŒ‹5‰^ ˆ À† ,+S …  …  …
…Là e °  ˆ ‹''
Œ  ˆ Ÿ ‹h
´´L“ ˆŸˆ ‘•‘|5‹‰´ ˆ ‹2Œ  ‘ ˆ ‹2Z ˆ B Œ‹ `; †‹‹Ÿ¼ Œ‹ ? Y“ ˆ  ˆ Œ‹Ÿ

) ˆ |‹5B“   


‹   ˆ ‘ P‹5 •‘ ˆ `Œ '|‘  Œ  ‘ ˆ +S‘' ^B“iZ ‹ ˆ '
Š ‹Ÿ ˆ ‘™  ‹p À= Á •
Œ  ‘ ˆ )+Z‹ %   x  mZ6{@x  m <{ …
CN ˆ ‹55,+ ´;  x¾ •
Š ˆ Ÿ|”· ˆŸˆ5’ 
Œ‹Ÿ('
(
~x {H;! x mZ6{ m)  x mZ 6{@x m {
< ;! x mZ6{@x m %h {

 €   } 0"%$ | u,"%$


C  ˆ  ‹;+ Œ‹ ˆ ‘ ÀŸ ‹P‘ ˆ 2 Œ‹1‹5 •‘5‹1‹5 PHZŸ ˆ Œ‹@<ƒ ’‘ˆ ‘5Œ‹ …à x)‘ Á  ˆ  ‘‘™
N
'Œ ‹5 ˆ  ˆ 
Œ¾  ’  ˆ Ÿ'L“  ˆ ‘ ˆ5’ ‹5‘|Ÿ  Z ˆ   ¿ ˆ Š,+Z‹É‘”‹Ÿ x'¼) ‹% Œ‹
& ˆ ‹% x;Œ‹h ˆ ‘ ÀŸ ‹%Z ˆ Œ‹É ˆ5Á 5Œ‹hŒ  ‘ ˆ ‹É<ƒ ‘’ ˆ ‘|5Œ‹ …

FHG h n†f uh '  ‹e2‹5  ˆ5’   À |‘  pZ ˆ Ÿ‘)‘|HqÈ) ˆ ‹5 Œ‹;+ Œ ˆ Œ‹P‹5 @Œ‹;+ L“ ˆ Œ‹
‹ |É‘‘Œ‹h‘%Œ ˆ Œ‹É‹5Œ Z‹‹5‘|hZ ‹ …
C1‹5B“  ; i
 ˆ ‹p¬ )Œ‹2 ˆ ‘ ÀŸ ‹;+ i;‘2Š ‹Ÿ ˆ  ˆ ‘ ˆ5’ ‹5‘|5 ˆ À0‹ŸŸZŸ °Z ˆ 
 ˆ Š‰ i ˆ ‘| ’ ? i‹Ÿ) †‹5hB“  ˆ ‘ ÀŸ °|‘ Œ ´Œ‹Ÿ ‹5  ’  ˆ 6)P q®‹5 •‘5‹;+
ÈZ ˆ ; ˆ ‹ ,+x‘p ˆ ‘  ˆ ŒŒ‹2‹ •‘5‹HZ ˆ ´ ˆ5Á 5‰¼ ‘,+ ˆ ´   ˆ 5´‹5‘  i|h Œ‹
‘™i) ˆ ‹5 Œ‹É‘¿Œ‹ŸŸ ”‹5 hŒ‹;+‘‘Œ‹h ¿<ƒ ’‘ˆ ‘5Œ‹ …

FHG hdS‰@€z‰5h ' C ˆ - ˆ È‹=5 5ȋ  ˆ5’ È Å ) ˆ ‹5 Œ‹;+U ‘. 5 >Ÿ  ˆ ‹0 ˆ ‹|2‹5
Œ Z‹‹5‘|%Ÿ‘  ‘•‘| ° ˆ ‹ÉL
‹5Œ Z‹‹5‘|%Z ‹É‘™  ‘ ™ …

[ m \rz€m> '
D ; Y“ ȝŒ ‹ŸÄ ˆ h h ˆ Š‰Œ  ‘
   Œ‹p‹5 •‘5‹ ‹5  Œ‹H) ˆ ‹5 Œ‹;+x‘@ Œ‹2 ˆ5Á 5Œ‹
‘ ˆ •‘™^; ˆ ‹ Œ‹
|P‹Œ Z‹5‹5‘|0‹5 |‘ ¼ ‘+B‘
Œ‘  Œ‹'‘ ˆ ) ˆ ‹5 Œ‹'1•‹5
Œ Z‹‹5‘|ÈZ ‹`‹ |`‘ ˆ ,+@ ÀQ|Œ‹5Ÿ \Œ‹Ÿi  ˆ ‹`” ˆ - ˆ B“  hš™‹Ÿ5¿\ ˆ À† 
•  x5Š ˆ Ÿ …

hˆ +‹Ÿ .DŒ‹Ÿ %) ˆ ‹5   ™ ’  … CN ˆ ) Œ‹ œ  ˆ5Á 5Œ‹ L“ š™x ˆ5’  ’ . +| É ˆ ‘;‰Œ‹ Ÿ ˆ  ˆ ‹
 ‹‹Ÿ ˆ |B“°• ‹% ˆ ‹%‹5 |hL“  Á •Œ  ‘ ˆ +Z‹5 ‹p¼ ‘ ?  ˆ ‹5 ‘•‘|‰‹·“ 5
‹ ‹%< ‘  ’  Y“ ˆ '· ‹5)p‘h| Œ‹%; ˆ ‹5 Œ‹h‹5 ‘|A13/54A/  … |  ˆ ‹6+‹5 % Y“ 
Œ‹É ˆ ‹
 ˆ5Á 5Œ‹ ˆ ‘ À|K13/549‘  Œ‹ŸP¼ ‘‰‘;+;Œ†. +| qÍ ˆ •‰‹ŸZ` ˆ À† ɼ ‘ … x ;+|ŒŒ‹H ˆ ‹
 ˆ5Á 5Œ‹É‹5 | ˆ Œ‹6+‘E ˆ •‘|h  ˆ ‹É° ˆ À†  ˆ  … E

  > !( C     #


   "9$ "(,   

  ‹ŸY ‹ qD) |5‹# À+1 ˆ ‹k5‹` † ’ ‹ …


  ˆ  Œ¿Œ‹•‹5‘†•‘5‹ ˆ ‘ À
Œ ˆ  ‹iŒŒ‹=) |5‹ …eà 2Œ‹Ÿ0‘ À ˆ ,+e|‘ ‹=i‹5 ‘|= †‹5Š ³™ +  Í ˆ ŒiÈ ˆ  Œ ˆ Œ‹

‹‘†•‘|5‹‰    ˆ Z ˆ   ˆ 0Z ˆ   ˆ 0¿• ‹h” ˆ À†  |‰ Œ‹É‹5 •‘5‹h‹5 |hZ ˆ 
Œ‹%; 5‹ÉŸÀ™ …½ |Œ‹ŸŸ  Œ‹Ÿ%
‹;  ˆ ‘ SŒ‹Ÿ%  ¼ ˆ 
v™ U
‹‘†•‘|5‹É
Y“ .‹‹5È ˆ  Œ ˆ ‹‹|B“ .5‘ 2 ˆ À† °š™‹Ÿ5 …#
«)‘‹5‹ŸK  ˆ È ˆ ¼ đ•¿‹5 ‹= À
 ˆ •a'‘ SŒ‹ŸÉ  '¼ ˆ ' N
‹‘†•‘|5‹‰|U ‹‹Ÿ ˆ 
Y“ š™‹Ÿ5‘Œ
L“   ˆ À† ,+Z‘
Œ‘  
‹5 2 À Y>`Œ ¿ |‰   =‘‹Ÿ5'5Š ‹Ÿ ˆ Œ‹%‹5‘†•‘|5‹ 9

„‚Â
 
  >`Œ ` ’  ’‘ˆ  ,+|` ˆ ¼ đ•hš™ ˆ5’  )Œ‹Ÿ@` ˆ ¼ đ•h` x; ´  ’ 5 ˆ  ˆ É ‹ 
)‘Ÿ ? ¿ ´ ‹É ˆ %)p‘|Ÿ ˆ È;  ˆ Œ‘ B À= ˆ  ˆ  ’  ’ ‹5& 5
Œ‹Ÿ ˆ  …' … …… 
½ “ .Œ‹ ˆ ‘ ˆ ‹= ˆ ¼ đ•Œ‹ȝŒ` x;° ’  ’ˆ5’ ‹5  Z ˆ  ˆ  e… » ‹0` Y“ ’   `Œ ‹
‘P‹
· ‹'Œ‹ ˆ À† Œ‹ ’  x ’ ‘³Œ‹5‹Ÿ‹ ? Œ•· ‹
Z ˆ Ÿ‘  ˆ  † ’  ’ ¼5‘QZ ˆ
®|5‘ U|‘ Œ‹É ’ Œ‹% ‹%5¾ 5) …½ 
· ‹2 ’  ’‘ˆ  NŒ‹Ÿ ˆ ‘  ‚ ’  Y“ š™ ˆ ‘ Œ „· …

'¦ | €Ftvzxw u
  ¤~;·ux§H©\rFµ Utv¤~«©JrZÆBr

 i              
                 
 $ #23'"*),+.-0/ %+ N/ / $202 " / 2 $ q  5/ $ 20/ O2.$&/ 2 ):2 $H+ ) /R$N" ):2i' N/ N" ,1

i 20/ +H /+ ) + /

 ):" $&/ ,1 ) / z'C) $ / $ / N/ 5/ 2 $ q / $ + ) "k/ $ it;     

@ dShACBh '
  (
D ˜ •‘ `Œ ‹2Z ˆ2ˆ ‘ ˆ  ˆ h‹Ÿ  / "`‹5 p‘™ ˆ5’ ‘ ‹;+ ¿~' x ( { s "+x;Œ ’ |  ’
 6 
‹5B‘ɋ5‘ ‘•‘|‰‹Ÿ ~; " …¡  ƒL‘;+Œ‘ À ˆ @x‘|  ˆ ‘ ˆ | ˆ |~x ( { ( s Y-+ZŒ ’ |  ’ ‹Ÿ
‘%‹‘ ‘•‘|h‹5 5; " …
x   ˆ  ‹
”Q‘‹5‘'¼ `•‹5 •‘5‹' ’ ;‘5‹ ?   ˆ„†… – )  v™  …

 5  5; ˜  ˆ q x
{ …
x   3; j 
É Y“ ‘‹‘'¼  Œ‹S‹5 •‘5‹U|†H‹5 |BZ ‹U‹
… |  ˆ ‹ ˜  ˆ  x {; " ; qm  …
8i 
Œ |Ÿ‘ÉZ ‹% ˆ À†   ˆ ‹w'
CU  ˆ 5 H‹5 •‘„.0+ B“ =‰Z ‹1‘™•‹5 •‘5‹;+ Z ˆ FŒ‘™=‹5 |@ 5 Œ‘|5‹  .0+†
‹ ‘|2 Ÿ Œ‘5‹2‘| ˆ ‰‘™ …˜‰…  …  … Œ‹p‹5 •‘5‹ ;‹5 p 5 Œ‘|5‹  . Í ˆ •‘pÈ‘‹5‘¼ 
‹5 •‘5‹h ’ ;‘5‹ …
®‘  ’ h ’ À5‘•‘|
,+)  ˆ 5 ‰‹5 •‘A. ®:'
 xz.{  …

 “ ˆ Z ˆ ;+S5 5È ˆ5Á 53i9i`‹5Œ‹'š™ ˆ5’  ’ ‹0 ‹ ? ‹Ÿ +U‘  ` ˆ =‹5Œ‹
‘™ š™ ˆ5’  ’ ‹' ‹
%+B‘ Œ | ˆ  ‘Ÿ .Œ• … ' 
Œ‹Ÿ ‘‹‘'¼ ‹5 •‘5‹
 ’ )‘|5‹) … Ce ˆ Œ | ˆ ,+@ ÉŒ‹Ÿ) †‹5‹Ÿ¼ ¿°‘™ ‹5 •‘5‹  D‹ ‘| ˆ ‘  ’ ‹Z ˆ 
 ˆ5Á 5 …Z¡ ¿ | > ´ ¼ ˆ 'L“ ˆ5Á 5Œ‹hwi +Œ‘ ÀŒ   Y“  ’ |  ’  ' > ]
7 8
 xz.{ … 
¡   Z ‘•‘|  ' > ]
78

 xz.k{ ]
78
 ;  "
  (
x ( { ;  … 

½ ’ “ |
 
 

  ’ 
 

 ‘
 ”
 Ÿ
‹ S
 ‘
 É
 
‹ ‘
 
 
‘
 •
 ‘
 |
 ´
 Ÿ
‹ B
 5
 Z
Š
 ‘
 
 
 0
 
 Œ
 %
‹ 
  ’  ’ ‹h|5 ˆ  ’ À ˆ Œ‹Œ‹ŸÉ ’ |  ’ +
|
 

 …  …  … ‹ŸP‘'‹5‘ ‘•‘0‹5 ; "t; ( +B•ŠZ |`‹5 •‘0.  #Œ‹Ÿ'•‘ ˆ5’  ‘'


 ˆ •
‹5‹ŸL ‘‹5‘'¼ ‹5 •‘5‹% ’ )‘|5‹ …

˜‰…  …  … q Œ‹ŸÉZ ˆ ‘‰ Œ‹‰‹ •‘5‹‰‹ | ˆ5’ Z ˆ Ÿ‹‘ ‘™¿ ˆ )Œ‹´ ( ‹ •‘5‹;+FŠZ ‘
 ˆ |‰®‘‹5‘'¼ ='‹ •‘5‹h ’ )‘|5‹6+;‘‰ Œ‹´ ˆ5Á 5Œ‹ ˆ ‘ À´ŠZ |=‹5 •‘´L“ 
 ˆ )  ŠZ |
‹ •‘ɍ Y“ ˆ 
‘É‘™°‹5‘ ‹ …
˜‰…  …  … q Œ‹Ÿ2Z ˆ ‘*t i ;
   E
 

   "    "%$


²ButvÇ|y{ÇuQr4
x E
i ¿ h ˆ Š‹Ÿ ´ |p Œ‹2‹5 •‘5‹2‹ |p5 ‹ Œ‹2‘|Ÿ ˆ 2
‹ ‹Ÿ ˆ ‘5‘•‘%; †‹5Ÿ<͋;+| Œ‹

‹ •‘5‹ >°‘ ’ | ˆ ‘  ’ ‹h‹5U‘h‹‘ ‘•‘|‹Ÿ  >%h Œ‹Ÿ%  '¼ ˆ    ˆ ‘ ˆ·…)‰’ 5 ˆ  ˆ  
 ¼ ˆ 'Š ˆ Ÿ|(i  …

²ButvÇ|y{ÇuQr (-  '¦ |€ZtzwQu”¯Nu|}  Ç|€F­N§¨ux­St} 


C ˆ - ˆ i5 • ˆ Š¿‹Ÿ ¿ ÀZ ˆ  i Œ |. ¼ ˆ   É¿‹ •‘5‹;+P #
Kxi {  … 
„· 
²ButvÇ|y{ÇuQr  '¦ |€ZtzwQu”¯Nu St¤~ 
b  5‰ N€ MO-hŒ‹Ÿ´^‹5 ‹Y ˆ Š=Œ  ‘   ‹ •‘5‹ ? Z ˆ š ™‘ ,+))  ˆ  ; 
 ˆ À† 6) …
x ki#  ˆ Š‹Ÿ 0 ® ˆ ‘ ’ %  q«‹5 •‘5‹‘k>” ˆ5Á 5Œ ‹U=Œ Ÿ‘|
Z ‹´
‘  ?    s  ™ ’ )…
C ˆ - ˆ … 
' >
 6 (
 6  

( …

²ButvÇ|y{ÇuQr )¤¥@€F~Dr 



…   ‹‰5 ‰ ˆ ;= ˆ ‹´ ’‘ˆ ) ˆ 5‹;+L • ‹‰‘™
b  ZŒ ‹‰; †‹5‹Č „·¢†¢†   ’‘ˆ ; ˆ 5 ‹ ;
‰‹5 HZ ‹ ˆ ‘  ’ ‹ Z ˆ 3 ; ˆ Œ‘ ?   ˆ  ˆ ‘5  ˆ ) … 
 ‘ ; É ¼ ˆ ‰v™x7† ‹p ˆ Œ‘5‹
‹ÉŒZŒ ‹ 9

²ButvÇ|y{Çu®ƒµ  @€;­St ~S€;y@¯Nu|} Sy{§Í§{u|}ir 




`‹Ÿ) †‹5 Ž„ ; 5‹@‹Ÿ ˆ `Œ ˆ ‘  … C ˆ - ˆ ,+Z ˆ F Œ‹@Œ ˆ Œ‹ | ˆ ‘ ‘ ‘™‘ ³
Œ Œ‹ÉŒŒ‹É; 5‹;+Z¿• ‹ „·Ê†Ê  ’ ‹5‹5‘% † ' Œ‘| ˆ ' Œ ˆ ‘ 
 ’ Z ‹5‹
Z % ‹ Z<#Y †…

²ButvÇ|y{Çu®ƒ)r* sh€;§¨€FºF~Uu r 


(


Q  qjs <¿; 5‹'‹Ÿ ˆ  Œ ˆ ‘  ˆ Œ   „†… C ˆ - ˆ |B“  PB“ iZ ‹' ‹
Z ˆ Œ‹É; 5‹É‹ŸŸ ’ ‹  
‹ŸŒ
‹Ÿ ˆ ‘5‘•‘h‹Ÿ ’‘ˆ ‘ ˆ   < …

²ButvÇ|y{Çu®ƒZƒ /.  ƒµZµF¸



x *.m ‹5 ‹‘‹5‘¼ '´ Y“ ‘‹5‘¼  t; v Z>/ < /OxOxOx /ZHY Y Y Y Y YC{=· ||hš™ ‘5‘•‘| ZOYZ
’ ’ •‘5‹ …   ˆ  ˆ B“  pš™x‹Ÿ5`Œ‹0 '¼ ˆ Œ‹ +  / + (>/OxOxOx6/ + 

‹K+S5‘ ‹0• Œ‹0‘‹5‘¼ Œ‹
.  ; v Rh +     . {;   ˆ  ; Z>/ < O/ xOxOx/ ZOY Y‹5 ‘É‘™  ‘™°‹ 5‹ …

²ButvÇ|y{Çu®ƒ¸4
¤  ‹^ ˆ ; „‚ ; ˆ ‹5 Œ‹6+B‘™®‘ Œ Œ‹'‹5 5 >  ˆ ‹0Œ‹;+L  ‘š
Œ‹6+ i<ƒ ’‘ˆ ‘|5Œ‹2 Y“   Y“ ˆ  ? ŠZ ‘¿´ŒŒ‹ ‹‘|Ÿ•‘|5‹ ’ |pZ ˆ  ’ ‹pZ ˆ Œ‹ ‘™
) ˆ ‹5 Œ‹´‘^|Œ‹5Ÿ 9) … C ˆ - ˆ |B“  1š™‹Ÿ5'® ˆ )= ˆ ‹´) ˆ ‹5 Œ‹N ‘™ 
‘™i Œ‹2 Á •Œ‹h‹5‘|Ÿ•‘5‹É Œ‹2Œ‹h‘  ˆ ‹É Œ‹É ˆ Œ‹ …
  ‹U= ˆ )p „·  ) ˆ ‹5 Œ‹;+‘™0‘ Œ |Œ‹1‹5 U5 >Ÿ  ˆ ‹PŒ‹1 =‘‘Œ‹ …

C ˆ - ˆ B“  Sš™‹Ÿ5'Z ˆ 


'ŒŒ‹É; ˆ ‹ Œ‹hS |‰5 5Œ‹h Œ‹% Á •Œ‹‰‹5‘|Ÿ•‘5‹h Œ‹ÉŒ‹
‘  ˆ ‹% Œ‹É ˆ Œ‹ …


²ButvÇ|y{Çu®ƒÆ  '¦ |€ZtzwQu”¯Nu F~Ny{º+
x EiQÈ ˆ Š|q®‹5 •‘5‹ … C ˆ - ˆ Ai  •‘p‰¼Œ  ˆ Ÿ ¿‹Ÿ;‘2‹5‘ ‘•‘
‹‚“  L´; †‹5‹Č´Z ‹É‘ ‘ 
´  †‘ ˆ Z ˆ  …

²ButvÇ|y{Çu®ƒZË s
tv€F¥P€Z}  .  rZ¸

 ³™ Œ Z†Œ‹` ’‘ˆ ‘Œ‹Œ‹5‹5 ˆ ‘«5  % „·¢† †— x  Œ‹ …e¡ | ˆ °‘™   Œ‹‘ ³
Œ Œ‹;+ Fš™‹Ÿ5%5 >Ÿ  ˆ ‹2`• ‹ ‰Œ Z†‰;  ˆ ƒ ˆ  ˆ ~ ) ˆ Œ‘p‘| ˆ ´ŒŒ‹H‘™
x  Œ‹ ? ‹2 Œ‹p‘™È‹5‘‹) … C ˆ - ˆ |B“°• ‹p´Œ‹2Œ Z†Œ‹É ’‘ˆ ‘Œ‹2 ˆ ) †‹5‰
 ‚ |'BŒ ‹Ÿ‹Ÿ5‘¿¿‘ ˆ ‘*Í ˆ  ’ · ||h¿ '¼ ˆ 'Z ˆ L“ ’ ;Œ‹ …


²ButvÇ|y{Çu®ƒ  °€F~Uº 3 `€F~Uº.r+

 Œ‹ ’ ŸÀ5‹% |pZ ‹5‹ ’ ‰‹ ’‘ˆ ´L“ š™•‘‹p) ˆ |2‹Ÿ ˆ qts ŸÄ ˆ Œ‹ … CU  ˆ ŠZ ‘
Œ‹@ŸÄ ˆ Œ‹;+|š™ ‘5‘•‘|  ˆ ‹ Œ‹ ’ ŸÀ|5‹p H‘ À•‘  ‘ ˆ É 5h‹@Œ‘Ÿ5%ŸÄ ˆ ,+x‘
)  ˆ ‘™¿ŸÄ ˆ Œ‹<ƒ ’‘ˆ ‘5Œ‹|‘ Œ Œ‹;+F ‘‰ ‹‘ ’ ŸÀ`‘ À••‘  ‘ ˆ 0 5
‹ŒŒ‹‘™ ŸÄ ˆ Œ‹ …L´’ 5 ˆ  ˆ 0 ‹´;‘Ÿ q ;  ˆ Œ‘ PŒŒ‹‰Œ Ÿ ‹' |‘
B“ ° Á • ’ ŸÀ´%‘¿ À0•‘  ‘ ˆ ' 5
‹%ŠZ ‘'Œ‹2ŸÄ ˆ Œ‹ …

„·¢
²ButvÇ|y{Çu®ƒ(- Pu ‚¦€Ztzw u ¯Uu}  Ç€;­U§{u­Ut} 
C ˆ - ˆ ,+;  ˆ 5 É ˆ Š'‹Ÿ ´ ÀZ ˆ  «
i  q ‹5 •‘5‹;+ ”Kxi {   …
? à ·Ÿ y' b Ÿ ‹5 ˆ š™ ˆ ‘Œ „·— )

  :2&U"É$'&)(9(Z$& $0 8%&;:2<=-04

#
  „"  p !$ "     " #
$


½ 
 ˆ ¼ đ•0Œ‹%) |5‹‰  ‹5¼; ˆ iŒ‹ŸɝŒ ‹Ÿ ‘’ ˆ5’ Œ •
' ˆ ‘ ˆ  ˆ ¼ đ•Š‹Ÿ5 
ˆ ´Ÿ ‹hš™ ‘5‘•‘|É Œ‹% ˆ ŠŒ‹ …

½ px    ‹Ÿ¼) ˆ •) †‹5‹ ’   ) |5‹6+‹Ÿ) †‹ ’ 2
‹ ‹5‘  ¿ ´ À”´ À  † ˆ  „·Ž…

[ \s]K_ „·Ž ž ½ Œ‹2) |5‹É 


 ‹Ÿ¼) ˆ 

½ P 5Š,+‚ Œ‹LŠZ¼|5‹Ne ÀKx  1‹5e ˆ •‘Z‘|;+‘S‹5 ŠZ‘SŸ ‹5 ˆ 


 Z ˆ Œ  ˆ ‹
  ˆ  ’ Z ‹5‹ |% ‹%‘%'‹5‘ 
‹Ÿ ˆ ŠZ |
;  … ®‹%Œ‹ŸY B) †‹5‹Ÿ¼  ˆ -   ˆ 
 ¿5‘
Z ˆ Œ  ˆ ‹ 9

˜ “ Œ‹Ÿ ¡   ˆ S ˆ5’ ‹5 ”Œ´ ˆ ¼ đ•,+ZZ ˆ À0 ’ |Ÿ& …

b ȝ5Š‘ ? ˆ Œ‹Ÿ … ȝ‘ ‘ 6)P‹5 ˆ h ˆ ŠAiQ‹5 ˆ khA u‚d5€}h ·‹ “  ZŸ ‹5h‰‘H‰‹5‘ 
 ‹%ŠZ |' ˆ5Á 5'(i …
b Fc>dSe5f%g-h…hA u‚d€h@®Œ‹5%° ˆ Š
B; †‹‹5Č´¿‘ x‘ '‘ ’‘ˆ  ‘ …

'¦ | €Ftvzxw u rµ ²´­U§{u|t•©\r(xÆBr


 ¿y{u|t ¦S€;§ u|t©Jr+(¸+

                      
 
2 '"*),+.-0/ O220/ ./ 1 %+ N/ !202 1 / $e/ " / 2 / $ / N/ 5/ 2.$ $&! /  5/ $ 2 $ /

/z'" + ) "

dShACBh '
@

D » 5 ‹É5 hL“¼) ˆ ”  ˆ Š


B“ Œ‹5%Z ‹É‹Ÿ) †‹ ’ ‹5  …  ‹h ¿)‘h‹·“ ˆ •‘ ˆ
‘°‹Ÿ¼)&x‹|25ŠZ ´¼) ‘ ‰‘2ŠZ ‘Œ‹% ˆ5Á 5Œ‹p' Ÿ Œ‹ ? °‹5 •‘2Z ˆ  ˆ5Á 56) …Z˜ ‘ À
••    •‘ ˆ5’ L“‘«Œ‹'‹5 •‘5‹'ŸÀ™+U‘ŠZ ‘ Œ‹‹5 •‘5‹
 ˆ 5‹0Œ‹Ÿ
  ˆ ‹É
‘ ˆ5’ < …» ‹2 ˆ - ‹É 
 ˆ5’ ‹Ÿ É‹% Y“ Š ) ŸŠȋ5
  Œ‹Ÿ%‹Ÿ  …
  .
2 i Œ‹5h‘ ’‘ˆ  ‘+; ”)‘É Œ ‹5 ’‘ˆ  ˆ 
  ‘ ‘ '‘ ’‘ˆ ‘ Z… ½ ˆ ‹5
Y“  Z ˆ Œ  ˆ 
^  ‹É‘ ‘|Ÿ ˆ Z+ ‘ 5ŠZ '‹5 •‘É|5 ˆ  ’ À  ˆ =   ˆŸˆ &
Z ˆ  ' ˆ5Á 5'‘h  ˆ ‘ Z ˆ 

Ž†Ê
Z ˆ h ˆ ,+|Œh|FŒ | ˆ ¼  ‘™• ’ ‹@‹H %‘ ˆ5’ ɝŒɋ5 •‘ …¡ H‹Ÿ Y“  ˆ @‘
‹5 ˆ ŒÈ Á •i‹5 •‘ ‹=  .‹ ÈZ ˆ Œ  ˆ ‹;+e†“ Œ‹Ÿ•‘«Ÿ ‹•Œ‹• ˆ5Á 5Œ‹<ƒ ’‘ˆ ‘5Œ‹
=Œ‘  Œ‹
 ’  Ÿ ‹ ’ Œ‹ ? ‹5= ‘ ‘ Œ‹Ÿ‘ ’‘ˆ ‘9) … | ‹5}+;š™Œ‘ ’’ ‘|Ÿ‘  ‘•‘)  ˆ
‹5 •‘' ’ Z ˆ ‘‰ '‹5 •‘  Z }+;5ŠZ ‹5 •‘  ˆ ŠŒ‹5h0‘ ˆ5’ Z ˆ·…
hˆ +
ŒŒ‹‘™®‹5 •‘5‹š™ ˆ5Á •Œ‹‹5 |Œ  ‹‹5B“  e‹‚“†´L“  ‘ ‘ ,+; 5 ‹Œ‹5 •‘
. …N½ “ ‘‹5‘'¼ iŒ‹ ˆ5Á 5Œ‹= ‘ ‘ ,+Nš™Œ‘ ’ Œ‹0 À” ˆ ‘Ä ˆ È‘0 À  ˆ Ä ˆ  ? ‹0 Y“ ˆ  ˆ i
Z ˆ Œ  ˆ ‹)+SŒ | ˆ ¼‘)  ˆ Q '¼ ˆ •Z ˆ L“  ’ ‹'‹' À   ‘ ˆ   xz.k{ …S¡ ;+BŒ •

 ˆ ŠŒ‹Ÿh‹‹É¼) ‘ ,+%. B“ Œ‹ŸhS ‹5ŒŒ ¿B Y“; ‰ ˆ  ˆ ‹5 •‘;+F‘h '‹‰Œ
· ‘ H Œ‹‘™  ˆ5Á 5Œ‹0š™ ˆ5Á •Œ‹'`‹5 |Z ‹Œ ¼ ‹ ’ Œ‹ … | ‹Ÿ}+S‘ Œ |=ŒŒ‹‘™
 ˆ5Á 5Œ‹6+  ˆ -ɼ‘|  xz.k{1Œ‹ŸPZ ˆ +Œ%F ‹5‹Ÿ ˆ %25 ‹@ Œ‹@‹5 •‘5‹H ˆ Šɋ |
‘ ˆ5’ Z ˆ ‹ …

¿ ˆ - À ˆ5’ ‘ ˆ x|
Z ˆ2ˆ5’ ‘ ˆŸˆ ‘Œ'‹Ÿ ˆ  ¼ ˆ   q^L“ ˆ5Á 5Œ‹É” ˆ Š …
CU  ˆ q~;=Y-+5ŠZ 
‹5 •‘É” ˆ Š
Œ‹Ÿ2‹5 ’ +‘2 SB“ i ˆ ‘  Y ˆ  …
x r qtsY=°‘|Ÿ ˆE ™ ’†…%
¿‹Ÿ) †‹5´|‰5 % ˆ ŠŒ š™´Z ‹2 ‹2 q  ˆ5Á 5Œ‹
 %5 ‹% Œ‹É‹5 •‘5‹%‹5 É
‘ ˆ5’ Z ˆ Œ‹5%‘ ’‘ˆ ‘ …
x |5‘Z°# ˆ Š Œ š™ i ~  q h Z  ˆ5Á 5Œ‹;+  |5 ‹` Œ‹È‹5 •‘5‹`‹ |
‘ ˆ5’ Z ˆ +B‘
 •‹Ÿ ’‘ˆ ‘ ˆ ‹'  ’ |™  Ž ? )  ˆ
ˆ ‹5  Œ š™x ’ ) …e˜ ••‹
Y“ š™ ˆ ‘Œ „ +H  ‘D ’ °| i ) †‹5‹5Č J‘ ‘    … x :i '  ‹ ‹Y ˆ Š® ¼5‘|9‘
’ ZÉ Œ‹É ˆ5Á 5Œ‹É  …Z˜ ‘Ÿ5 ’ ZŸ F % ˆ ‘ ˆ5’ ŠZ |'‹5 •‘ÉL“ 
 ¼ ˆ ¿Z ˆ L“  ’ ‹ ?  ‹•š™ ‘5‘•‘`¿‘™  ‹ i '¼ ˆ  L“Z ˆ Ÿ ‹`$ ‹ D
‹ •‘=Œ ‹5 ’‘ˆ5’ )
Z ˆ=ˆ ) ˆ   ‹F† ‘ ˆ ŸÀ °‹…i … | ‹Ÿ}+N5ŠZ i‹5 •‘ i '
Œ‹5‘Œ ˆ   ‘ ˆ5’ Z ˆ·…K
#)‘È  ˆ ‹`Ÿ ‹5 ˆ Y“ Š ) ŸŠȋ5  ˆ5’ ‘ ˆŸˆ ‘Œ,+H \Z ‹`‹Ÿ ˆ
i 'e|1B“”‘ ˆ ‹5  L“ Á  ˆ •Œ š™,+L‹
‹Ÿ ˆ ŠZ ‘‹5Œ‹'Œ ; †‹5Œ‹'Œ š™Œ‹ …


| ‹Ÿ}+x)  ˆ ŠZ ‘


Œ‹%Œ ) †‹|5Œ‹ÉŒ š™Œ‹%ki ' + °;‘2Œ ‹Ÿ ’‘ˆ  ˆ °‘ ‘ ‘ ’‘ˆ ‘
? ’ ‘Ÿ‘  ‘•‘|`  †‘ ˆÊ +U‹ŸH À”Œ ) †‹5ÈŒ‹5 ˆ5’ 5  «‹5 •‘5) …„
 Œ ¼
  ˆ ‹ŒŒ‹<ƒ ’‘ˆ ‘|5‹‘ x‘ Œ‹‘ ’‘ˆ ‘‹;Œ° i‘ x‘   +e;  ˆ  ˆ • ˆ .‘ x‘ i‘ ’‘ˆ ‘#‹Ÿ ˆ i

À  >`Œ  ‹Ÿ&†|5q'Z ¿Z ˆ Œ  ˆ  Q‘‹É ‹5 •‘%ŸÀ e ˆ ¼ ˆ  ˆ   F‘;+  5ŠZ k ‹
‰ Y“  ˆ ‘Œ | ˆ 'Œ‹%Œ ; †‹ |5Œ‹%Œ š™Œ‹%( i 'F)  ˆ À0 ˆ ‘Ä ˆ  ‹;+ °‹5Ä ˆ 
ȝ‘ ‘ È‘ ’‘ˆ ‘ Œ ‹5 ’‘ˆ5’ ‹5 ˆ Œ‘Ÿ5iŒ ) †‹|5 … ;“†‹‹|•L“ .‘ ‘ ,+U .Œ‹Ÿ• ‹5‹Ÿ ˆ5’ 
) -  ˆ ;  ˆ ‹5& ˆ 0 0Z ˆ Œ  ˆ ‹
 « ˆ ȋŒ= ’ 5  ˆ·…Là eŒ‹5†Y ‘ =  ’‘ˆ    ˆ |B“  i‹Ÿ
¼‘ Œ ‹Ÿ ˆ %¿‘ ‘ 
‘ ’‘ˆ ‘”‹Ÿ ˆ i … E

½ = ˆ ‘-
‘³Œ‹5‹5‹%Œ É ’ À5‘•‘´ ˆ5’ ‹Ÿ h‹Ÿ& w'

¹'€Ftv€;§Í§¨¤y{tu^rZr

                   " +        


          
0!$ '"*),+.-0/R/ " / 2 / O %+e /R/ 2 N/ \1 2~+H/ $ ):"$ " N/ ):" $&/ / 2 N/

/ /  2 $

FHG hdS‰@€z‰5h '  ’ ‹5  ˆ 


´ ˆ ¼ đ•'Œ‹2) |5‹h 
 ‹Ÿ¼) ˆ  …

[ m \rz€m> '
D
«Œ ‹Ÿ ˆ  ` ˆ Ši |= Œ‹0‹5 •‘5‹=‹5 |0 Œ‹ ˆ5’ † ‹0‘0 Œ‹= ˆ5Á 5Œ‹ Œ‹0) |5‹ ?   ˆ
  …„·— ) …
½ •Œ‹ŸŸ ”Œ‹Ÿh'‹;  ˆ ‹·“  Sš™‹Ÿ5 ‘ ˆ ‘h‘ ’‘ˆ ‘ …
hˆ +Z ”Œ ‹Ÿ5(  ‘ ‘•‘‰
Œ‹É‹5 •‘5‹2
‹5 %Z ‹25 ‹%'‘ ˆ5’ Z ˆ +Œ
S ‹5‹Ÿ ˆ B“ ¿5‘ B‘ ˆ ‘%B“ š™‹Ÿ5Z ‹ … E

Ž„
[K\^]K_ „·— ž ½  ˆ Š
Œ‹É; 5‹% 
 ‹Ÿ¼; ˆ 

¹'€Ftv€;§Í§¨¤y{tu^rƒ

               
      
       
2R'"*),+.-0/ O220/ ./n+H /|#2 -0/ 2 / " / 2 / $ / N/ 5/ 2.$ N/H20 ",/ / /  5/ $

                    
/ /z'" %+ ) "/ $ !


                    
%/R+ ,1 N/ '"*),+.-0/ +H / / ) $&/ 5/ 2 $ /  5/ $ / /z'" %+ ) "5) N" $&!$

&-0/ 2t/ " / 2 O 5/ 2 O/w/ 2 #2 / O/  5/ $ / $ /%$&/ " 20/ )C$N",/

@ dShACBh '
D @F5 ‹p Œ‹ ‹5 •‘5‹2i ‹5 p‰‘ ˆ5’ Z ˆ +  x‘|pA  ˆ Ai  •‘Ÿ2ȝ‘ ‘ 
‘ ’‘ˆ ‘+ZŒ
|BŒ‹52 ‹E ˆ %|´ ÀŒ ‘ ‹5 ” ’ ‹Ÿ ˆ5’ ‘ …
P Œ‹
‹5 •‘5‹0. ‘01 ‹ |' Œ‹'‘™^‹‘ ‹
‹5 •‘5‹'~i |@‹ ‘|‘ ˆ5’ Z ˆ·…

®Œ ‹Ÿ ˆ 
  ˆ ‹‰ ‹5 ˆ Y ˆ Š   …i#‘  ˆ ‹  -·^‹5 •‘ | Y“ 
ˆ ‘ '|‘•‘  . ‘*1 …½ 
 ˆ Š  nŒ‹Ÿh‘ À ˆ ‘•‘‰Œ š™'‘h5 ‹É‹5Œ‹h‹5 •‘5‹h‹ |
i‘ ˆ5’ Z ˆ·…1à 2Œ‹Ÿ °‘ ’‘ˆ ‘+1‘•  )‘ °Œ ‹Ÿ ’‘ˆ  ˆ  ‘ ‘ È‘ ’‘ˆ ‘\‹Ÿ ˆ  …

^‘  ’  Š‘Q‘ ’‘ˆ ‘ ‹Ÿ ˆ iJ‘ ’ Z' Œ‹
 ˆ5Á 5Œ‹axz.0 / *{‰‘qxz13 / *{+)|P‹ |
 ’ ŒŒ‹‹ ˆ ‘•‘|Œ ‹ ’ ‘Ÿ&Œ‹=‹ ‘ ‘ ‘ ’‘ˆ ‘«‹5Œ‹ |0 Œ‹‹5‘ Œ‹0H‘|
 
Œ •'š™x ˆ5’  ’†…
  ’ ‘ ˆ x|‘•‘;+Z‹Ÿ8i ) †‹5‹5ȉ”Š‘ ‘ ’‘ˆ ‘   ˆ ‹0'
x 
 @‹·“†'L“  ‘ x‘ •‘ •ŸŠ ’ ˆ đ•‘³Œ‹5‹Ÿ‹0 ‹5‹Ÿ ˆ 5 ‹ Œ‹‹5 •‘5‹'‹5 '
‘ ˆ5’ Z ˆ·… x ;+v†“ Œ‹ŸS05Š‘= - ˆ ;+‚‘S  ˆ ‹5 ‘•‘|e‹Ÿ&x|‹U À2 ˆ ‘-@ŸŠ ’ ˆ đ•
‹‚“ 5
‘¿5 %) |;+F‘2• | ˆ '5 ‹É Œ‹%‹5 •‘5‹É” ˆ Š
‹5 |h
‘ ˆ5’ Z ˆ ‹ 
‹ •‘É
 ’ Z ˆ É‘2 
‹ •‘  Z SB‹5 |É‘™°
‘ ˆ5’ Z ˆ·… E

#
  „"  p !$:„      " #
 $

b °  5Š‘ Œ‹Ÿ2g e>np€ <r}m> €}h¿  ˆ ‹|B“  )Z ‹5‹5‰A ‹2‘p‹5‘ ´Z ˆ 5 ŠZ 
‹5 •‘
° ˆ Š …
b  ‘ ‘ •Œ‹Ÿ
…ge>np€ <r}m> €}hQ  ˆ ‹|B“  PZ ‹‹5=p ‹‘
•‹5‘ =Z ˆ Z Š  |`‹5 •‘

° ˆ Šš™Œ‘ ’ ;  ˆ ‹5 •‘ hŸÀ |L SŒ‹52Ÿ ‹ ’ ‘™3Í ‹ … 
b ° ˆ Š
‹ ˆ g e>np€ r}m>%€h®‹·“  L) †‹5‹5Č´¿‘ ‘ ŠZ 5 ‘ …

FHG hn†f uh5l ' ½ Œ‹`‘™« ˆ ‘ ˆ ‹ ˆ ŠŒ‹ ?   …„‘– )•‹5 |•ŠZ 5 ‘‹6+P‹Z ‹ i ˆ ‹Ÿđ•
?   …„·œ ) …



.¿Œ Z •Z ‹•°Œ Ÿ \ ’ ŒŒ‹5‹ ˆ °‘‹Ÿ ‹ |5¿š™  ¼ ¿‹ À  ˆ Ÿ
)  ˆ  ’ ‘ ˆ ‹ŸU  ˆ Š0Œ‹ŸɊZ 5 ‘^   +F‘´Œ‘Ÿ5Œ‹ŸŸ  Œ‹Ÿ ˆ5’  ’ ;  ˆ´Á  ˆ 

 ˆ ȋ ɝŒ  š™ ;… 
» ‹# ˆŸˆ ‹H5 5 ‹ ‹@ Œ‹ š™ ˆ ‘ŒŒ‹ Œ‹ š™‘ Œ‹ hŒ Ÿ ‹p‹Ÿ ‹ |5Œ‹ ‹Ÿ Œ‹
L) ˆ •‘Ÿ5‘|hL“ ˆ • ˆ |B“ ° ˆ ŠŒ‹Ÿ2ŠZ 5 ‘ …

Ž†Ž
[ \s]K_ „‘– ž
‘
 ‹5 |%5 >Ÿ  ˆ ‹%ŠZ 5 ‘‹

[K\^]K_ „·œ ž b ° ˆ Š ¿ŠZ 5 ‘

#
   "   "%$
²ButvÇ|y{Çu®ƒ 4
C ˆ - ˆ |`‹Ÿ ˆ   ’ 5Š ˆ 
`•‘‹5 ‹ 2 q ?  q‚s Z6{+U HB“ š™‹Ÿ5•Z ‹0
‹ ‘UP‹5‘  ? ‹  À% ˆ ‘Ä ˆ 
Z ˆ Œ  ˆ ‹8  ˆ  ’ L“ ·;   ˆ Z ‹5‹51Z ˆ 5ŠZ @· ‹5@  S
‹|B“ :‘2 ˆ @‘ ˆ ) …

² ButvÇ|y{Çu®ƒ   @€;­St ~S€;y@¯Nu|} Sy{§Í§{u|}ir ;Ë




D † ’ )Œ‹'p  Ÿ¼Z  @Z ˆ Ÿ‘;‘  Q5  ˆ   …S½  ˆ ‘ ˆ Ÿ  ˆ +B5ŠZ ‘È‹Ÿ5
55Š …x½ ɋ5ŒŒ (  ˆ +|5ŠZ  ’ ;„ 2È ˆ %55Š+xŒ | ˆ h ’ )%<ƒ ’‘ˆ ‘5
Œ‘   Àq‘   … C ˆ - ˆ B“ ˆ ȋ´Œ'‹5ŒŒ q  ˆ +F UŒ‹5h) †‹5‹Ÿ¼ 
 ˆ - ˆ   ˆ )
 „·Ê•’ |)Œ‹% |h‘™°‘ Œ Œ‹2
‹5‹5 %Z ‹%‘Œ ˆ  ˆ ‘Œ  ˆ5’ Œ‹ …

²ButvÇ|y{Çu ¸Zµ4
 ‹' ZŒ ‹;+B‹= ˆ -‘| q xqjs <{A  Œ‹6+B‘™ |‘ Œ Œ‹'‹ŸŸ‘5Œ‹ ’ | ˆ ‘  ’ Œ‹
Z ˆ  ˆ 5
 ˆ Œ‘5 …
b  fe>dS‰2#m  d5lÈ †i0 À3  ….  À3  …1 ?  ^ ’ ŒŒ‹5‹ ˆ ‘•‘‹ŸŸ‘5Œ‹)‰Œ ‹Ÿ‹Ÿ5
 Z
  ˆ Ÿ ˆ k.m)  ˆ  ˆŸˆ & ˆ ‘:17‘°Ÿ ‹% Œ‹ ˆ 5Œ‹É‘%‘” ˆ ‹Ÿ| ’ ‘Ÿ‘  ‘•‘|´Z ˆ
Œ ˆ Œ‹Kx  Œ‹;+|‹ ‹ ‹85‹ Ÿ ‹5 ˆ ‘™aÍ ‹H À' Á • ˆ 5 ? •Œ  ˆ ‹2‹pŒ‹p‹5‘‹
) †‹ ’ ‹5) …½  <m, #c  Ah  dp#‡ pfe,d ‰2m  d5l0Œ‹ŸÉ  ˆ ‹%  '¼ ˆ 
 ˆ 5Œ‹%Ÿ ‹ ’ Œ‹ …
C ˆ - ˆ |B“  ˆ |‹5‘ ˆ  Œ  ˆ ‹Œ‹È‘ ‘ ‹Ÿ5Œ‹È)‘`Ÿ ‹5 ˆ 5 5Œ‹È Œ‹ ˆ 5Œ‹`)  ˆ
Œ ‹5 ˆ  ˆ  q€m Z´Z ˆ Œ  ˆ ‹%)‹ |%‰  †‘ ˆ ‹ ˆ Œ‹Ÿ)Œ‘Ÿ&Œ ‹ Z>/ < O/ xyxyx&z/ q~mZ´‘%|)B“ |%‘™
 ‘™”‘ ˆ 5'‘¿Œ ' …

²ButvÇ|y{Çu ¸Lr* Ì;•s‰u·utv} €F­Utvº4


½  ˆ  | ˆ ŸŠ ˆ  ‹·“ ‹5‹Œ  ˆ ‹Œ‹R<qDŠ@   ˆ ‹5  ˆ È À ¼    …P˜ ŠZ ‘ Œ‹
Š@   ˆ ‹´) †‹5‹5Č=® ‹ q m Z‘‘‹´Z ˆ e Œ‹
 ˆ Œ‹´5Š@†  ˆ ‹ … C ˆ - ˆ   ˆ 
Y“ ¡ 5ŠZ|5‘ ˆ )‘P ˆ - ˆ È ˆŸˆ ‘•‘|pŒ‹<q”5Š@†  ˆ ‹ %‹5 ˆ 5%B“‘•%‹5   ‹5‹Ÿ‹
 Œ¾  ’ L“  ”‹5Œ‹`‘‘‹ ? ¼‘J‹S= ˆ +@ Y“• †‹Ÿ ’ Œ‹Ÿ ˆ5’ ‘ ˆ x|,+ ‘`‹‘ ‹• Œ‹È5Š@†  ˆ ‹
‹‚“ ‹5‹5 ‘|‰5  ˆ ´ À=¼ 6) …

Ž†—

²ButvÇ|y{Çu ¸Fƒ 1 ¿ÌNÌ#rZµ+
x 3s Z ‘Ÿ ˆ† ™ ’†…  5ŠZ Œ¾  ’ ‘‰ŠZ |=À Z =L“ 3q)Y Œ  š™,+) 
Ÿ ˆ ¼

Œ  ‘ ˆ 5Š ‹5Z ˆ È) †‹5‹Ÿ¼ Œ‹%‹5 ˆ 5B“‘ †¼ ˆ ‹ ’ Í ˆ  ’  
Œ‹´‹5 •‘5‹´‹5 Œ‹‰‹ •‘5‹‰3q)Y 0‹ ‰Œ  ˆ  ’ =L“ =‹5‘ 0Œ  ‘ ˆ·…  ‘ 1Œ‹Ÿ‰ À
 ‹% ˆ k  ‘ ˆ k  q®;  ˆ À ‘  
Œ‘ À‹ 2; †‹5‹5¼  9

²ButvÇ|y{Çu ¸Z¸ ¹¦UyÍ~Uu^rZµ



à Sš™‹Ÿ5 ‹5‘ ˆ ‹EY>`Œ ‹É
Z ˆ ŸŸ  ˆ ~q)Y Œ  š™'‘ q~m <= ˆ À† Œ‹  Y“
 q~m =
‹5Œ‹%À Z Œ‹;+Z‘™°‘ Œ Œ‹%‹‹2) |ɝŒ  ?  Z ˆ ;+ ’ ‘|Ÿ‘  ‘•‘|;+
Œ‹É‹5 •‘5‹%¿)   6) …
C ˆ - ˆ |B“  1š™‹Ÿ50=ŒŒ‹´Z ˆ ŸŸ ‹´)  ˆ À |‘  = ®‹5‹5=Œ ‹Ÿ ˆ  ˆ 0= †
)  x Z Œ Ÿ|   ˆ  ’ ‘ Ÿ ‹|´0‘h0‹5‘ wÍ ‹‰5ŠZ ‘^Œ‹´Œ¾  ’ ‹´‘‰ŠZ ‘
Œ‹ÉÀ Z Œ‹É Í ˆ •‘% À=Z ˆ ŸŸ +‹5B‘ɋ5‘ ‘•‘|‰‹Ÿ q^Œ‹Ÿ2¿ Ÿ ' …

  "Ék5$
o;35"0(

²ButvÇ|y{ÇuQr
¤  Ÿ5  ‹ ˆ Œ‹5 ˆ ‘ ˆ   ´‹5Œ‹2Œ ) †‹5Œ‹%Œ š™Œ‹6+x °)‘2‹Ÿ) †‹5 ˆ |‰ 
 ˆ Š0Œ‹Ÿ‰Œ š™ … x  >i  '¼ ˆ =‹5Œ‹‰ ˆ5Á 5Œ‹ … |  ˆ ‹ <#>5; ] 7 89  xz.{ s=<q„+Z‘´ 
>  q m Z … “ ˆ ȋ À ˆ  ˆ  ’  ’Ž +|†“ Œ‹5H h ˆ Š´Œ š™h|FB“ Œ‹Ÿ Z ‹@i ˆ ¼ ˆ  … Ce ˆ

Œ ‹ ’ |‘;+ B) †‹5‹5Č


°• ‹% ‘ x‘  …
 F5ŠZ h ˆ ŠÉ Z ˆ ŠZ |‰Œ ) †‹|5´Œ š™%) †‹5‹5Œ@ ‹ph‹5 •‘5‹

”L“ ˆ5Á 5Œ‹6+p  ˆ ‹i‘D Ÿ Z|°   '¼ ˆ ®L“ ˆ5Á 5Œ‹° 5ŠZ  Œ ) †‹|5®Œ š™,+
 ¼Ÿ‘ ˆ  >-% q ? ‘™QŒ ) †‹5Œ‹0Œ š™Œ‹=‹5 |=‘™  ‘™Q‹ 5Œ‹0 ,+N‘
‹ |;+Z °'Œ 5KŸ‹%”‹5 •‘% ‹Ÿ‘ ˆ ‹Í ‹6+S' ˆ5Á 56) …
CN ˆ ‹Ÿ5,+B š™x‹55Œ ) †‹|5ȝŒ š™`P0Q ˆ ŠŒ š™`; †‹5‹ ’ |
®• ‹

L“ ˆ5Á 5Œ‹|==‹5 •‘5‹ …B˜ •‘³Œ‹5‹Ÿ‹6+F ^‘^ ’ 
|0Œ0 ˆ Š
É; †‹5‹Č´¿‘ x‘ ,+‘h  ´ ˆ ŠŸÀ B ) †‹5‹5Č ’ | ‘•‘ …

²ButvÇ|y{Çu®ƒ

˜ ‹Ÿ ’‘ˆ ‹N @ ˆ Š ‹Ÿ  ‘U = ˆ ‘| ’ i¿ U Œ‹N‹5 •‘5‹e‹5 U Œ‹U; ˆ ‹ Œ‹;+v‘™
’  ˆ ‘  ’ Œ‹2Z ˆ 
 ˆ5Á 5‹ŸL‘%‹5‘ ‘•‘ɋŸL‘  Œ‹%‹5 ÉŒ‹ … Ce ˆ Š| x; ŸŠȋ,+5 ɋ5 •‘
Œ‹5È‘ ˆ5’  • ‹ „ ? )+U‘0 @B“ š™‹Ÿ5`Z ‹` ˆ Š`) †‹5‹5Č0š™ ‘5‘•‘| Ž
 ˆ5Á 5Œ‹ ? ¼9) …½ “ ¼ŸŒ‘Ÿ<1Œ‹5% ˆ - ˆ wi Œ‹Ÿ%Œ  ‘ …
½ Œ‹
· k ‹ q; < ? L“ ˆ ȋ ? )S)´‘k  q; ? L“ ˆ ȋ ? ¼9)S)´‹5 | ’ À5‹ …
 ‹Ÿ) †‹5 
 qts … 
CN ˆ Y“¼‹Ÿ ˆ q'Z) †‹5 ‹ÉB“  Sš™‹Ÿ5
‘™¿‹5 •‘5‹;+Z‹5 ‹*. ‘*1 +B
‹5 ‘%Z ‹
ˆ ‘  ’ ‹ …
C1‹5  xz.k{ sjZ,+ Bš™‹Ÿ5´”‹ •‘A4  5 Œ‘|  . …
94 Œ‹ŸP Ÿ Œ‘  1   ˆ ‹6+†L“ ˆ ȋ ? ¼9)+,.#‘„1 ‹5 @ Ÿ Œ‘5‹ ? ‹Ÿ +†‘pŒ ‹Ÿ ’‘ˆ 
= Œ‹'‹ •‘5‹(.w/213/54 Q ˆ '  ˆ Š•') †‹5‹5Č|š™ ‘5‘•‘‘™Q ˆ5Á 5Œ‹5) …
˜  ˆ  ‘Ÿ  …
  4 ‘*1›
‹5 |ÉZ ‹É Ÿ Œ‘5‹ …
 
 Á •,+ Sš™‹Ÿ5
  ˆ ‹%”‹ •‘    5 Œ‘|  1 +‹%Z ‹  .m  4 … ®‹É  ˆ ‹;+
‘ Œ ‹Ÿ ’‘ˆ 0• Œ‹‹5 •‘5‹ .w/213/54A/   ¼Ÿ‘|0Q ˆ Š`0@Œ  ˆ Œ
? ¼9) …
 Z ‘•‘;+%. ‘17‹5 |‰ Ÿ Œ‘5‹;+‘% À=Œ ‘ ‹Ÿ  ‘” ’ Œ   …

Ž–
²ButvÇ|y{Çu ¸+
˜ ‹Ÿ ’‘ˆ ‹=  ˆ Š i9 = Œ‹0‹ •‘5‹0‹ |0 Œ‹0x  Œ‹0‘ Œ‹0 À‹5 ‹ ’‘ˆ ‘Œ‹0 Œ‹
 ˆ5Á 5Œ‹ … x Ei ' % ˆ Š‰2 ¼5‘|iZ ˆ ‹Ÿ ˆ Œ‹5‹Ÿ ih À'· (4 ‘ Œ‹p ˆ5Á 5Œ‹p|; À
Œ Œ ˆ ‘ … Ce ˆ Š| x; ŸŠȋ,+ i Œ‹5pŒ š™ … Ce ˆ Œ | ˆ ,+%i')‰ Y“ Œ‹Ÿ2Z ‹  ˆ  ’ •‘| …   B“ 
‘\‹5 ;+P5ŠZ ”Œ ) †‹5 Œ š™¿ i ' Œ |Ÿ‘¿ • ‹•J‹5 •‘` 5 Œ‘|  4
‹Ai … ‹†i ' +Z5ŠZ ‘®
ŒŒ‹%‹5 •‘5‹hŒ‹ŸÉ
‘ ˆ5’0¢ ? Œ'S ‹5‹Ÿ ˆ ,+‘ ˆ  ˆ 'B“ 
Œ ) †‹5 Œ š™ Œ |Ÿ‘|”\• ‹ „·Ê ‹5 •‘5‹)+H‘È5 ‹È Œ‹È ˆ Œ‹i‹5 È ‘ ˆ5’ ‹
„·Ê… ®‹;+;5ŠZ Œ ; †‹5,+=Œ •=® ˆ Š=Œ š™ ?   ˆ5’   ^‹5 •‘5)+
) †‹5‹5Č« '¼ ˆ ÈZ ˆ i‹5 •‘5‹=È‘ ˆ5’ ‹0Z ˆ ‹;+eŒ`p ‹‹Ÿ ˆ `B“ • ‹0‘™
‹ •‘5‹H‹5 Hh‘ ˆ5’ ‹@Z ˆ ‹ ‹HŠZ |‰Œ ) †‹|5 …˜ •4mŒ‹Ÿ É‘ ˆ5’'„·Ê†Ê ‹
i + p ¼ ˆ É2Œ ) †‹|5Œ‹HŒ š™Œ‹P†i 'x2 ’ Z ‹5‹5É pZ ‹ œ†Ê…
•)‘@  ˆ ‹ ˆ5’ ¼  ˆ
À•Œ š™x ’ ‘ ˆ Œ‹Ÿ5 ˆ |k4 ‘%0 ˆ5Á 5;  ˆ 5ŠZ 'Œ ) †‹|5Œ š™ …F˜ ‘ À ˆ @‘ 
Œ ‹ ˆ  ˆ À=Œ š™x ’ ŸÀ 
wi 5 É‘ ’ Z‰¿• ‹ œ†Ê  ˆ5Á 5Œ‹ …

²ButvÇ|y{Çu Æ4
˜ ‹Ÿ ’‘ˆ ‹ É ˆ Š´‹Ÿ i   Œ‹ ‹5 •‘5‹p‹5 H Œ‹H; ˆ ‹ Œ‹;+‘™ ’ | ˆ ‘  ’ Œ‹

‹5)‘%‹‘ ‘•‘|ɋŸL‘  ‹5´Œ Z À‹5‹‘| …9
”)‘   ˆ  P€}h®5 ɝŠ‘”‰  †‘ ˆhŽ…½ ´¼
° ˆ ¼ đ•Œ‹Ÿ% ˆ - ˆ B“  Sš™‹Ÿ5‘™°‹5 •‘5‹ ˆ ‘  ’ ‹2Z ˆ ¿ '¼ ˆ 'Z ˆ  ‘‹ …
CN ˆ Y“¼‹Ÿ ˆ …') †‹5 ‹2|´‘™i‹5 •‘5‹2|‘ Œ Œ‹2‹5 ‘25 >  ˆ ‹ ˆ ‘  ’ ‹pZ ˆ 
 ¼ ˆ 
Z ˆ ´ ‘‹ …
   ‹5 •‘´ ˆ ¼ ˆ  ˆ  …
 ˆ5’ Z ˆ Ÿ  ˆ ‹h Œ‹´ ˆ Œ‹‰‹ •‘5‹h‘®‘™¿ ˆ ;Œ‹ '
x 
´ ˆ )kt7Œ‘™iB‹5 h Ÿ Œ‘5‹   +‘% ´ ˆ )  JŒ‘™°L´ 
‹5 %Z ‹ …
|  ˆ ‹;+ŠZ |‹5 •‘*.  t B“ ‘ Œ  À  ˆ 
‹5 •‘‰t …à B‘
 ’ Œ  ‰h‹ % ˆ Š‰ Z ˆ t +|Œ‰‹5 •‘K.9Œ‹Ÿ h‘ ˆ5’ Z ˆ·…˜ Œ‘ ’   ˆ 
)  ˆ ŠZ |0.  t + Yh |kt7Œ |Ÿ‘0¿ ¼ ˆ 'Z ˆ 
‹5 •‘5‹ ?   ˆhà )+‘
   x  {pŒ‹Ÿ2Z ˆ·…
˜ •   ’  ’ 5Š ‹Ÿ  ˆ ¼ ˆ  ˆ ‘•‘|;+e  ‘Q ’ '•5ŠZ •‹5 •‘qi Œ‹Ÿ

‘ ˆ5’ Z ˆ·…
 ŒŒ ‹Ÿ ’‘ˆ ‹= °‹5 •‘  …e “ ˆ ȋŒÈp ˆ5’  ’ ,+15ŠZ °‹5 •‘a.  t Œ‹Ÿ
‘ ˆ5’ Z ˆ +xŒ‹Ÿ  5 Œ‘|   +x‘@ ; È '¼ ˆ ‰Z ˆ ‰‹5 •‘5‹ |; F‹ |2 Ÿ Œ‘5‹
‹*t …Zà S ‰ ;  ˆ   '¼ ˆ Z ˆ 
‹5 •‘5‹‰B U‹5 ‰ 5 Œ‘|5‹´ ‹ … Ce ˆ
‹55,+  '¼ ˆ '5  SL“ ˆ5Á 5Œ‹É ˆ ‘ ‘|h¿‹5 •‘Ékt ‘%”‹5 •‘%  JŒ‹Ÿ2Z ˆ·…
¡   +)5 ‹5 •‘  9B“`0 ‘‹
;Œ  wxÀ`®‹ •‘´ t …) ,+; 1 
Á  ˆ  5 Œ‘|  ® '¼ ˆ =Z ˆ ‹=‹5 •‘5‹ t … Ce ˆ ‹Ÿ5, + m ‰ P‹5‹5NŒ 5‘ ˆ
° ¼ ˆ 
Z ˆ '‹5 •‘5‹ …
 ‹  ˆ ‹  >!?  xstan { h  >l?  x K{nh ZŒ‹Ÿ‰Z ˆ +L‘‰)‘´  Á  ˆ  ’ |  œ†Ê…L˜  ˆ v
‘Ÿ  …

²ButvÇ|y{Çu®Ë

CN ˆ Y“¼‹Ÿ ˆ  'x) †‹5 ‹HB“ ˆ ȋH Y“ ’ ZŸ iL“ ‰Œ  ‰|‘ Œ h È ¼Ÿ‘
5 >Ÿ  ˆ ‹É¿• ‹h‘™È †Œ‹É‘|ŸŒ‹ …
|  ˆ ‹;+N)  ˆ 5ŠZ ¿Œ    +e .;‘•Š ‹Ÿ ˆ ‘™ †Œ‹•|%@x‘‘|‘|ŸŒ‹
 ˆ ‹|´ Y“ ”‹5 ˆ •  …9
  ™
°5‘ S5Š ³™°)  ˆ 5ŠZ ‘'Œ‹hŒ  Œ‹ …
˜ ‹Ÿ ’‘ˆ ‹ ¿ ˆ Š i  |` Œ‹‹5 •‘5‹`‹5 | Œ‹• †Œ‹‘™ ’  ˆ ‘  ’ Œ‹È‹Ÿ%‘  Œ‹
Œ ˆŸˆ Œ‹5; ‘  Y“  Œ‹'Š ³™Q‘³Œ‹5‹Ÿ‹ … |  ˆ ‹;+L  ˆ Š| x; ŸŠȋ• ‹ ‹‹Ÿ ˆ ••5ŠZ 
Œ  ' ’ 5 ˆ 'š™ ‘5‘•‘|É ˆ5Á 5 …
˜ 
 ˆ Š0Œ‹Ÿ ’ x‘•‘|‹‹h¼; ‘ ,+Z‹‰‹‹ŸU‹‹´ ˆ5Á 5' Ÿ 3' ¡ ®ƒ)‘;+F‹Ÿ
) †‹5 ‹|`‘™ †Œ‹;+S‹5  ‹ 0‘  ( +U‹5 ‘| ˆ ‘  ’ Œ‹'Z ˆ Œ‹0 ˆ5Á 5Œ‹= ‹5‹5 x‘ ’ Œ‹'L“ `Z ˆ 
 À
 Œ




 
   ‘
 =
 L
 
“ 
 
  ˆ i
 Z
  
ˆ  À
 Œ    ' …P˜ ‘ À®‹Ÿ†  iiL“ iZ ˆ     ‘  (ȋ |
‘Ÿ|Œ‹6+S‹ ’ ‘|Ÿ‘  ‘•‘™^· ‹Œ‹'Z ˆ 5‘Z|   …S¡ L“ ˆ •Z ˆ ;+SB“ ‘  Œ‹'‹ |

Ž†œ
‘Ÿ|Œ‹6+Z‹ ’ ‘|Ÿ‘  ‘•‘'™i· ‹5Œ‹‰Z ˆ 5‘Z|   ' …F˜ •  ‘  ' ‹5 h‹Ÿ) †‹ ’ Œ‹
‹5Ÿ‘5Œ‹;+†“ Œ‹Ÿ% 
´ Œ‹%‘™° †Œ‹É‹ |2‘|ŸŒ‹;+F‘¿Œ | ˆ  ‘Ÿ ^Œ
Y“ ’   ’†…
| ‹Ÿ}+ŠZ |' ˆ5Á 5'Œ ˆŸˆ Œ‹Ÿ;   À=  ’ L“ 'Œ  ,+‘%‘™iŒ  Œ‹É<ƒ ’‘ˆ ‘5Œ‹
 ‘hŒ‹É ˆ5Á 5Œ‹h<ƒ ’‘ˆ ‘5Œ‹ …9
¿‘Ÿ   ˆ   ˆ ‹‰ ˆ5Á 5Œ‹%‘ɝŒ  Œ‹ …
 ,+ i9Œ‹5‹Ÿ ,+U; †‹‹5Č qJ‹5 •‘5‹=‘%  qJ ˆ5Á 5Œ‹ …
«‹ ?  … š™ … à  „ + ¼9)S)'|B“  ˆ ‹…i
) †‹5‹5Čhi‘ x‘    ( xyxyx    ? À' ’‘ˆ Ÿ ¿B“0‘ ˆ ; ˆ %Œ‰ Y“ ˆ  ˆ ´Œ‹ †Œ‹2‹
‰¼ ·9) …  Ÿ5  ˆ5’  ˆ Œ´‘ ‘ ,+ °;‘2‹Ÿ) †‹5 ˆ ´ Œ‹p †Œ‹pŸ ‹ ’ Œ‹%‹5 %‘™ 
‘™”‹5Ÿ‘5Œ‹ …
  5  À`Œ  0 ‹‹5 ‘ ’   Y“ ˆ5Á 5 x  /    {hŒ     ;   … | ‹5}+ Œ‹
Œ  Œ‹   O/ xyxyx&/  =‹5 É‘™  ‘™i‹5Ÿ‘5Œ‹ …
CN ˆ Œ ‹Ÿ ˆ ‘Ÿ +S Œ‹' '¼ ˆ Œ‹‹5 ˆ 5‹'‹5 ˆ Œ‹
†Œ
‹   ‘ (•‹5  ’ |™Q‹2Œ‘™^
‹ |ɋŸ ˆ   …Z¡ ¿Z ˆ Ÿ‘  ˆ +;Œ‘™i‹ ˆ 5‹É‹Ÿ ˆ  =‹5 | ’ |™ …Z 
 Á •,+‹5  ( ;    + Œ‹
 ¼ ˆ Œ‹N‹5 ˆ 5‹S Œ‹U †Œ‹  (P‘  @‘U‹Ÿ ˆ  2‹ | ’ |™ …¡ N‹5xH‹Ÿ5,+‚ 0 ˆ -H@ ˆ Š
‘° ˆ x5Š'|‰5 ‹2 Œ‹2 ¼ ˆ Œ‹%‹5 ˆ 5‹É‹2 À=Œ    '‘%‹Ÿ ˆ Œ‹p †Œ‹ /  ( O/ xyxyx&/   6  /  
‹ | ’ |™ …  ‹  ˆ ‹   ‘  + ’ | ˆ ‘  ’ Œ‹AxÀ  •‹5 h‘Ÿ|Œ‹´Z ˆ 5 ‹H‹5 ˆ   +;‘
L“ ˆ ȋ Œ% ˆ5’  ’ ‰‘  Œ‹P %‹5  ’ | ‘•‘p‹5 ˆ   …¡  Œ‹H‹5 H É5  ‘•‘| ‘|ŸŒ‹;+
‘¿Œ  ˆ  ‘Ÿ ®;Œ
Y“ ’   ’†…
½ Œ ‘ ‹Ÿ ”‘  ’ Œ   …

²ButvÇ|y{Çu+

 Œ ‹Ÿ ˆ =  ˆ Š¼Z ˆ Ÿ5°  Œ‹=‘™ ‘‹‘'¼ Œ‹ȋ5 •‘5‹0 ’ ;‘5‹
‹ |  ‘  ~+F   Œ‹ŸÉ ‘‹5‘'¼ 0 „·Ê ‹5 •‘5‹ ˆ ‘ ˆ5’ ‹5‘|‰ Œ‹ „·Ê  ¼ ˆ Œ‹;+;‘7Œ‹Ÿ
È‘‹5‘'¼ ´ <#Y‹5 •‘5‹ ˆ ‘ ˆ5’ ‹5‘|2 Œ‹ „·Ê †Œ‹p‘@ Œ‹ „·Ê Œ  Œ‹ …¡ H‰ ˆ5Á 5#Ÿ |
0‹5 •‘     ®‹5 •‘ "   ‹ŸU‘‹5‘ ‘•‘´‹5U  '¼ ˆ   Œ‹Ÿ ’  ˆ ‰‹‰ À` †
 Œ    " …à S‹·“†‰  ˆ ‹h
 ˆ - ˆ |B“  ‹5 •‘ "  7Œ‹ŸÉ'‘ ˆ5’ ”• ‹ ’ | 
–… CN  ˆ Œ‘ À+) P‹Ÿ •
= ˆ - ˆ |B“  1 ®°^• ‹ Å„  ˆ5Á 5Œ‹
‹
Œ= ˆ Š,+L 0 ˆ ‘)
Œ‹%Ÿ ˆ  ˆ ‹%) ˆ •‘Ÿ|‰  ˆ ‹É
Œ ‘  ˆ  …
½ ¿‘ ’  Y“ š™ ˆ ‘ŒÈŒ‹Ÿ  ˆ ‹0 ˆ ‘ ˆ  ˆ •ŠZ |    Œ‹Ÿ‘ ˆ5’  • ‹
’ |   ' ¡ ¿ƒL‘;+x;  ˆ    + ° 5 ˆ Œ‹5;Œ‘Ÿ&‘•‘  S!x { ‘  !x { ‰ ¼ ˆ 
´ †Œ‹%‘
Œ  Œ‹|P‹ | ˆ ‘  ’ Œ‹   … |  ˆ ‹;+) =¼ · ˆ  ’ Z ˆ Y“ 5 ˆ ‹5Œ‘Ÿ  Œ ‹ S!x  {‰ †Œ‹
‘  !x {@Œ  Œ‹2Œ Œ ˆ  ’ Œ‹2Œ‹Ÿ@ȼ ·°É   S!x {  !x {H;Œ Ÿ‘|2 Œ‹ ZOYZ ˆ Ÿ ‹
   …L “   S!x {  !x { s ZOY-+UŒ S!x {5/  !x R
{ s Z‘'‘|Ÿ ˆ ‹ …Uà PŒ‹5  ˆ ‹kY ‘ •… ’‘ˆ    ˆ 
S!x  { h  !x r{ s +Œ
|BŒ ˆŸˆ Œ‹5;    !x 
{ s +Œ •  ’†…
 É ˆ - ˆ ‰‹Ÿ; †‹5Ÿ È‹ À ‘  ‰‘‰ †
» ‹H À‹5‹5 ‹2 Œ‘5‘ ˆ h À‹Ÿ ˆ 
%Œ  ”°Œ |Ÿ‘ ‹°Z ˆ °   ¼ ˆ Œ‹<ƒ ’‘ˆ ‘|5‹;+HŒ°|2•  ˆ ”i  ˆ5’ ‹Ÿ 
‘³Œ‹‹Ÿ‹%Œ‹Ÿ% Ÿ …

²ButvÇ|y{Çu)(
x 
 N À0  †‘ ˆ L“ ¼Z ˆŸˆ  …
 sq„+v Œ ‹Ÿ ˆ  Y ‘ ‘•‘|1=Z ˆ  ’ ¼  ‘= À>`·1 Œ‹e¼Z ˆŸˆ Œ‹15Š xŒ À ’ Œ‹
¼)   ¼; ;+Z‹É‘ Œ Z´5 ‹% Œ‹ …Z¡ ;+ZŒ •  s  +  Œ‹Ÿh‹S= ˆ 'Z ‹hŒ ) ˆ
 


¼Z ˆŸˆ  ‹%L“  ‹ …

”‹Ÿ) †‹5´|5‘Z‰ &% q …
 B°Z ˆ  ’ ¼ 
Œ‹Ÿ2) †‹5‹Ÿ¼ …'
˜ ‹Ÿ ˆ ‹5 ‹0 • ˆ ŠȋŸ È‘' « ˆ ‘| ’ iD |= Œ‹0‹5 •‘5‹0‹5 |0 Œ‹%qJ‘ 5‹;+
‘™i‹ŸŸ‘5‹ ’ | ˆ ‘  ’ ‹pZ ˆ ´ ˆ5Á 5‹5)‘2‹5‘ ‘•‘ɋ·“  ‹p |%ŠZ ‘¿i• ˆ Œ·¿L“ 
 Á •`¼Z ˆŸˆ  … C1‹5 % q„+S  ‹0ŠZ |`¼Z ˆŸˆ ÈŒ‹ŸŒ  ’ ,+S‘=Œ •¼Z ˆŸˆ 
B“ Œ‹52Z ‹%Œ  ’ 
 ‹ÉL“  ‹;+ ”= ,   ˆ5Á 5Œ‹ …

Ž†Å
» 5 ‹q4;/Oxyxyx&/54  Œ‹Œ ; †‹ |5Œ‹Œ š™Œ‹:i ? Œ =s Z6{+1‘;  ˆR} ; Z>/Oxyxyx&/ +
” 5 ˆ Œ‹Ÿ)Œ‘Ÿ&‘•‘| q ‘  ¼ ˆ 'L“ ‘Y|5‹‰‘%  '¼ ˆ L“ ˆ5Á 5Œ‹h‹%  ˆ Š

ÉZ ˆ 4 …

C1‹5 Œ‹%Œ ) †‹|5Œ‹hŒ š™Œ‹%‹ |É‘™  ‘™°‹z 5Œ‹;+ ” ' ; ]  ‘

q ; ]  q +Œ s Z‘

 q s <0;  ˆ 5  }…


 = ‹6+);  ˆ ÀÈ Á • ˆ ‹5 +) ÀÈZ ˆ 
ŠZ ‘ Œ‹Ÿ
  ˆ ‹
¼5‘  ˆ ‹
= Ài&x‹Ÿ 
Œ‹%¼Z ˆŸˆ Œ‹ÉƒLŒ‘Ÿ ’ 
‹5‘  ° ÀŒ ) †‹|5'Œ š™  À |‘   SZ ˆ Ÿ‘ …
CU  ˆ 5ŠZ  }  ™ ’ +F Œ‹ q ‘ 5‹‰eŒ ; †‹5‘w4 ‹5
Z ˆ ‘
 =š™ ‘5‘•‘

¼Z ˆŸˆ Œ‹HKY>`Œ  ’ |¼ ,+Œ‘Ÿ5%Z ˆ  ’ @ À´ Á •hL“ ɝŒ ) †‹|5  Y“ ˆ  …| “  +†)  ˆ
 
5  } +   '   ;  + ¿‘Œ ˆ    ;  … ? „ )
 
  %jZ,+ ¿ ’ h|,
  % q …

½ ® ˆ Š®°Z ˆ 5ŠZ ^Œ ; †‹ |5 Œ š™^Œ‹Ÿ° ®D ˆ Š^Œ š™^
) †‹5‹5Č• ‹•‹ •‘5‹L“ ˆ5Á 5Œ‹ …
 ‹|`Œ‘ À° `  ˆ ‹0|  r; q m=Z

)  ˆ 5ŠZ  } …
 
¡ ”‹5 ;+ x‘|w' ; ]  ; ]   { ; q~m
 … ? Ž )
x q m Z6

”
 ‘
 ¿
  ’ 
 
 
 É
 
 
   ;
 



8
x ' / z q {
p 
 &
 
x 
 5
‹   …
6  "6 
 ‹;+)  ˆ 5  } + À ˆ ‘ ÀŸ  ? „ ) ‹·“ ’  ˆ %5‘Z|p'  ; +‘É  Cq ; q …
˜ ‘ À• ‹5‹Ÿ ˆ  i&‹ q …  ? Ž )+Z ¿ ’ É| ¿&‹5 ’ | ‘•‘

 B+‘% 
 
&x‹5 F…
 Z ‘•‘;+Z    ;  ‘ ;q5m Z…

 ’ ‘ ˆ x|‘•‘0'Z+ ; q~m Œ  ; 


x'/zq { …

CU  ˆ ‹5    ˆ Œ‹´ Ÿ ‹;+B  ‹Ÿ) †‹5=|; Z,+L‘
 = À`Z ˆ 
5ŠZ ‘  
  Á  ˆ   …
‰  ; Z:'B  ˆ ‘ ˆ ‘Q = ˆ ‘;•QZ ˆ axQ  ’ Z ˆ ;+S‘ ˆ |
Œ‹
Z ˆ 5‹'
|Š   ˆ 5 … ZɼZ ˆŸˆ ÉŒ‹ŸHŒ  ’ ,+| 2• ˆ Œ·È%|5Š‰Œ‹Ÿ@‹5 H%Z ˆ ;+‹   ˆ ‹

 p ˆ Œ‹Ÿ5È ‹B“  • ˆ Œ· È  †‘ ˆ pi‹5 ˆ  Z ‹ ˆ ŒŒ  ’ +e‹5 Œ  đ5i À
Z ˆ h ˆ5’  ’ ‘|5,+L‘‰Œ‹Ÿ‰  ˆ ‹´
 †‘  ˆ  )  ˆ ®Œ ˆ  ‘Ÿ ˆ  s Z'‘h '• ˆ Œ·®
 ˆ 5´B“ Œ‹5% ´Z ‹ ˆ ŒŒ  ’†…Z˜ ‘ À ‹‹Ÿ ˆ |B“  B‹‚“†2¼‘¿L“ °Z ˆ  ’ |¼ °¬ )
‹ ŠZ ’†…
‰   Z:') ^) †‹5R  q;  q ' ‘,  ;  ' +L‘
  ' ; q ' m Z …8
 ˆ5’ Z ˆ Ÿ0  ˆ ‹ Œ‹
‘Y|5‹‘   ˆ )Œ‹' q ' ‘ Œ‹'¼Z ˆŸˆ Œ‹'‘^Z ‘5‹'  ' …L ‹'5ŠZ ‘ Œ‹ ˆ )Œ‹;+B 
Z ˆ   ˆ ‹ÉŒŒ‹2¼Z ˆŸˆ Œ‹hŒ •'¿· ‹% ˆ5’  ’ ‘h‘2 5  ˆ Œ‹5   ’†…

²ButvÇ|y{Çu+
¤
}+-  ˆE  …„·Å…

 



[K\^]K_ „·Å ž b 'Œ   † ˆ Ÿ  Œ    Œ‹

 »  …
x rq ¿‘Ÿ ˆ ;  ˆ Œ‘ L‰5‘  Œ ‹Ÿ ˆ ‘Ÿ ”Œ‹Ÿp) †‹5‹Ÿ¼  …
¿Œ ‹ŸÄ ˆ ´ ‰ ˆ Š
‹5 •‘'  ˆ ‘| ’ i9 |' Œ‹‹5 •‘5‹0‹5 | Œ‹q x  Œ‹'‘' Œ‹0 ˆ5Á 5Œ‹
Œ‹wq3m=Z ˆ 5Œ‹ …
 “ ˆ ȋ2 Y“ ’   ’ +9i Œ‹5%Œ š™ … CP‹5|B“  L) †‹5‹5ȍš™ ‘5‘•‘||q5m Z
 ˆ5Á 5Œ‹6+†“ Œ‹Ÿ2” ˆ ¼ ˆ 

ŽÂ
‘@ Z%) †‹5‹5ČÉ %Z ‹ %‘ x‘  …½ “  ‹Ÿ i% Y“ ’   ’  À Ÿ È% ‹H;‘Ÿ5É‹ŸŒ
Œ‹5% 
‹Ÿ) ˆ  …
x (.› ‹ •‘ ˆ ¼ ˆ  ˆ  …
  5  = '¼ ˆ px  Œ‹
 
Ài‹ŸŒ  . Œ‹Ÿ
Z ˆ  ? Œ  ˆ ‹K.D‘  šY Á •6) …x h5‘  Œ‹#x  Œ‹ ‹5 ˆ |p5Œ‹,2m>  h5l …
` 5 " É '¼ ˆ ´
x  Œ‹% % À‹ŸŒ  . Œ‹Ÿ%Z ˆ  …
¡ ‘•‘;+F ”  h " ; q … ? „ )
C1‹5B“  ZB“ B“ `‹5‘ ;5Š‘i‘| ˆ ‰‘™…  Œ‹   ’ Œ‹;+ F •š™ ‘5‘•‘  "=Z ˆ Œ‹
v 13/5R 4 {=0  Œ‹´ ´š™ ‘5‘•‘|´=Œ‹Ÿ‰¼;  … CN  ˆ 05‘  Z ˆ ,+; À‹ŸŒ= 1 
4 Œ‹Ÿh  ˆ ‹2Z ˆ  …
 ’ ‘ ˆ x|‘•‘;+;‹5B À‹5Œ‘ ˆ ' Œ‹†  Œ‹A1 ‘k4 Œ‹5ÉZ ˆ ,+;†“ Œ‹Ÿ‰B“ '‘‰ 
_ 76 
‹‘ 1PŒŒ‹B‘™  Œ‹BŒ‹5L¼;  … | ‹Ÿ}+‘‹51 Œ‹B‹5ŒŒ‹S‹5 Bš™ ‘5‘•‘  Z>/ < O/ xyxyx&/ (
` +

†“ Œ‹5%|´Z ˆ NŒŒ‹2 '¼ ˆ Œ‹hš™ ‘5‘•‘_   "i‹5 %Z ˆ ‹ …9


hˆ '
"6  _ "6 
  q ; ° ~  q ; h=Z,+Z  ˆ ‹ (
`
Œ‹Ÿ2Z ˆ +Z‘É   " ; 
`
…
( (
  ? „ )+Z ” ’ h  ˆ ‹%k  q ; q m  " ; !x €m " { …
_ "6   _ 76 
  q ; %h <  q ; %h +Z  ˆ ‹ (
`
Œ‹Ÿ2Z ˆ +‘É   " ; ( x ( ` hZ6{ …
(
  ? „ )+Z ” ’ ɝŒ‘Ÿ5 ‹% q~m <; !x 5m "-{ …
| ‹Ÿ}+‹%5 ‹% Œ‹%· ‹;+‹5 q Œ‹Ÿ2k† ‘ ˆ Œ  ‘Z¼ =  ˆ r ‹ q ~  q~mt<Œ‹Ÿ2 · ˆŸˆ5’†…
˜ •ɝŒ2B“ Œ‹Ÿ@Z ‹P %· ‹P
 q ; Z<;  +|%5‘  ɝŒ ‹Ÿ ˆ ‘Ÿ ÈŒ‹ŸP) †‹5‹Ÿ¼ 2)  ˆ Œ‘Ÿ5
† ‘ ˆ·…

²ButvÇ|y{Çu+
˜ ‹Ÿ ’‘ˆ ‹p É ˆ Ški  |p Œ‹p‹5 •‘5‹p‹5 |p Œ‹pŒ‘| ˆ Œ‹2Œ‹p· ‹5Œ‹ …
°Œ ‹Ÿ ˆ p5 
L“¼) ˆ  ° ˆ5Á 5È‘ ˆ °‘™ · ‹5Œ‹=‹Ÿ Œ‹· ‹5Œ‹=Œ ˆŸˆ Œ‹Ÿ) |5Œ‹•Œ Ÿ‘‘|ȝ5ŠZ ‘i
Ÿ‘5 ^‘Z ˆ ‘|
 Œ¾  ’†… CP‹;+L  ˆ ‘ 5ŠZ · ‹5NŒ Ÿ‘|Z ‹A‘5   
· ‹' ˆ ¼ ˆ  ˆ 'S‘”Œ Ÿ‘|´”‘ÉBZ ˆ 
 Œ¾  ’ Œ‘   ?  …ˆ đ†  ? )S) …F½ “ ’   ’
 ‹‹Ÿ ˆ 
  ˆ ‹%
Œ´ ˆ Š'Œ‹Ÿ%Œ š™ …
x 2  ˆ ‹ ¿ ˆ ¼ ˆ  ˆ ŒŒ - ˆ *i …˜ ‘% ˆ ¼ ˆ ‰; †‹‹5Č|  q ( ‹5 •‘5‹ ? ‘2  >R; q ( m Z
 ˆ5Á 5Œ‹)+P5ŠZ ‘#°‘ ˆ5’ ¿ ’ Z ‹5‹•Z ‹ – ?  2B“ .®| – Œ¾  ’ Z ˆ · ‹ ……… ) …H˜ ‘™«
Œ ˆŸˆ Œ‹5; ‘  Œ‹=· ‹5Œ‹'ȝŒ |5‘ZZ ‹wŸ‘5 ‹0· ||=;  ˆ ‘ ˆ5’ „†…S» 5 ‹0B“ ‘
 Ÿ | Œ‹H ˆ5Á 5Œ‹H ’ ŒŒ‹5‹ ˆ Œ‹H)  ˆ@ˆ ‘ ˆ - ˆ i!+   |B“†•‘5 ˆ É‘ ˆ5’ Œ‹ · ‹5Œ‹
) ˆ |É…‘5  …
à B‹Ÿ •h 
 ˆ - ˆ 5 h ˆ ¼ ˆ    ‹5 •‘5‹ ? ‘h   muZ' ˆ5Á 5Œ‹5)2ŠZ ‘
(
‘ ˆ5’ 
 ’ Z ‹5‹%Z ‹ – +) †‹5‹5Č
°•  ‹  6 ‹5 •‘5‹h‘ ˆ5’ ‹É‹Ÿ ’‘ˆ ‘ ˆ ‹  „†… ?  )

½ “ ˆ Ÿ  ?  ) Œ‹52 ˆ &À ‘•‘‰‹Ÿ‹S 5‹Ÿ   …

CN ˆ Y“¼‹5 ˆ q'Z) †‹5 ‹% Y“ ˆ Ÿ  ?  )p‹ K ‹5‹ …
|  ˆ ‹;+;Z ˆ e5 ‹´ Œ‹
 ˆ ¼ ˆ Œ‹e‹ Ÿ‹ Í ´ Œ‹´ ˆ5’ ‹5‹5Œ‹ ?  )h‹´Z ‹´‹ȝŒ ‘ ‹Ÿ +
`;‘H‘`5Š ‹5 ˆ +|) †‹5‹5ČÉ 2 ‹H;‘ŸH '¼ ˆ ‰ɋ5 •‘5‹ …» 5 ‹ŸY É‘Œ ˆ  +‘
‹ ‹ÉB“  L; †‹5‹Č  ‹ •‘5‹É | 
‘ ˆ5’ ‹h‹Ÿ ’‘ˆ ‘ ˆ ‹  „ +‘É Œ 

h < %

‘   …
x . ®‹ •‘´ ‘ ˆ5’ ‹5 ’‘ˆ ‘ ˆ  „ ? ®5‘ e‹5 •‘š™x‹Ÿ50‹‹´ }+F N
)  ˆŸˆ É ”;  ˆ '‘™”‹ •‘5‹5) …
  ’ 5 ˆ =  ˆ ‹h ‹ •‘A1  h À‹ŸŒ 
 . ?  …  …  … À•  †‘ ˆ ®5Š‘^‘| ˆ  . ‘1…{%‹5 Év™  … C1‹5  xz.{ 
i‘h
   +    Z …F 
 ‹;+  xz1a{ ; Z‹‹% U ¿)  ˆŸˆ É ˆ - ˆ  ‹5 •‘´ Ÿ Œ‘ 
1 ‹5 ˆ   '‹ŸŒ
(. ‹Ÿ ’‘ˆ ‘ ˆ    +Œ|BŒ  ˆ Œ ˆ h À0v™  ’   …

x  4J 2‹5 •‘H 5 Œ‘|  1 …» 5 ‹Pp‹5  jZ,+† `(4 ;  .\‘P  x}40{ jZ … 
|  ˆ ‹;+@5 ‹ Œ‹È‹5 •‘5‹i Ÿ Œ‘5‹  4 ‹
 ? Œ‘ ´|h; ˆ •‘i ˆ ‘  ˆ . ‘~1 +
 5 ‹ŸY   +)N PŒ‹5‰)‘ Á  ˆ Œ  QŒa.k)h‹5 |  ‹ŸŒ   …. ? 5 >Ÿ  ˆ ‹
)  ˆ ·‹5
´v™  ’  })p‘%'‘ ˆ5’ ‹ „ ?  Á • ˆ ‹5  |´)  ˆ 1a) …

Ž† 
x    ˆ ‹ ' % ˆ Šhp ¼5‘|i‘ ’ Z 5 ‹H Œ‹ ‹5 •‘5‹  5 Œ‘|5‹  4…+|‹ 
 …Nà pŒ‹Ÿ‘ À ˆ | '@Œ‹5=‘Œ ˆ È  ˆ ¼ ˆ  … x  +
È '¼ ˆ °i‹5 •‘5‹ ’  ’ ‹ … |  ˆ ‹
  + •‘*.mB“Z ‹ ’  ’’  ’†…

Z

CN ˆ ‹Ÿ5,+ 'e; †‹‹5Č  m +´‹5 •‘5‹;+S;Œ  m  m + %  L5ŠZ ‘ •‘ ˆ5’ 
 ’ Z ‹‹|%Z ‹ – +‘É |  mZ
‹5 ɍ‘ ˆ5’ ‹É‹Ÿ ’‘ˆ ‘ ˆ ‹  „ ? ‹5 •‘A4 Œ‹5%
‘ ˆ5’•„
‹ ' ) …L˜ • -x' m Z6n { h <2%  m   m ++)  Œ ‹5 ˆ ^ ˆ ¼ ˆ =1Œ | ˆ Œ À
  ’  …

²ButvÇ|y{ÇuQrxµ4
½  ˆ ‹ ‘•‘|´Ÿ ‹ ’ ‹h À`‹5 Ÿ ®  ˆ ¼ đ•0Œ‹‰ ˆ ‹É‹ ‹‰‘‰ ˆ ‹h‹ŸŒ‹
Œ‹5´‘Œ ˆ p† À¼ =)  ˆ  ˆ - ˆ |=‹·“  eB“ `Z ‹= ˆ À† •  ˆ ‹> s <  …)¡  ˆ ‘; ˆ 
‹% À Í ˆ  
L“ ¡   ˆ +F ¿ ¼Ÿ‘|0' > h <;  h 


 h ( > + …  …  … > <  m … 
²ButvÇ|y{ÇuQrZr
CN ˆ Y“¼‹Ÿ ˆ :'U) †‹5 ‹'B“  1B“ š™‹Ÿ5Z ‹
|@5 ˆ 5Z ‹5‹5Œ‹5‹5‹
L“ 
 ˆ ,+Lk 

Š‘”  ˆ |LZ ‹5‹
Œ‹5‹Ÿ‹ÉL“  ˆ  …
à N  Z Zp  Œ‹ ˆ ‘  ’ Œ‹‘™  ‘™+)L“   œ†œ À‹5 ‹=‘™ ZŸ ˆ Œ‹ ?  ˆ   ˆ 56) …
 “ ˆ ȋ2 ´ ˆ ‘)Œ‹2Ÿ ˆ  ˆ ‹;+ Bš™‹Ÿ5 
°• ‹ Ž†  À‹ ‹%L“  Á •´ZŸ ˆ  … Ce ˆ
‹5  ’  ˆ Œ‹ ˆ ¾ Œ‹;+ ¿)‘%‹Ÿ; †‹ ˆ |B“  B °°• ‹ Ž†  5 ˆ 5Œ‹ …
˜ ‹Ÿ ’‘ˆ ‹h  ˆ ‹p ´ ˆ Š
‹5 (i  % Œ‹%‹5 •‘5‹É‹5 |2 Œ‹Ex  Œ‹2‘2 Œ‹É ˆ5Á 5Œ‹%‹ |
Œ‹ ˆ 5Œ‹ …e È  ˆ Ȋ| x; ŸŠȋ5i « ’ •| i Œ‹Ÿ ÀZ ˆ  …#
 F.B“  p  ˆ ‹@†
 ’‘ˆ    ˆ À ˆ ‘ ÀŸ  >   m  …

hˆ +‘  m  ;=<  %u<<;  > …;˜ | ˆ  ‘Ÿ  …

²ButvÇ|y{ÇuQrƒ+

˜ ‹Ÿ ’‘ˆ ‹ ‹Ÿ5‰ h ˆ Š´2Z ˆ Y “ ‘‹5‘¼  v < / / -/ /  /  /; //l{ …S˜ “ Œ‹Ÿ2´‹Ÿ¼)³
x‹Ÿ i
 >+ FB“ Œ‹Ÿp ‰Z ‹  ÀZ ˆ  …-
i  ‘i ’ p,+;  ˆ qts + É ˆ Ški B“ Œ‹Ÿ
Z ‹% ÀZ ˆ  …

   

  

[K\s] _ „‚ ž b ¿‹5 ‹Y ˆ Š(i  Œ‹52


‹Ÿ¼)&‹Ÿ ¿


C  ˆ Œ  ˆ ,+) À ˆ ‘ 5ˆ ’ ‹‘|Ÿ Q0 À  † ˆ  „‚ • | ˆ  i=Œ‹Ÿ‰ ÀZ ˆ ,+L‘‹5eB“ 
N
‘¿Œ‹5% Á •)  ˆ 5 rq 
; …

Ž†¢
    



[ \s]K_ „·  ž3i Œ‹52 ÀZ ˆ 

²ButvÇ|y{ÇuQrx¸4
¤ ‘   Ÿ5  ˆ ‹5  ˆ ‹Ÿ ˆ hŒ ) †‹5hŒ š™,+x F‹Ÿ •pÉ ˆ - ˆ  ˆ5’ ‹Ÿ 
)  ˆ i«Œ š™ …

¿ 5  ;O/ xyxyx&/  Œ‹%‹5 •‘5‹ … x |> ´ ¼ ˆ 'L“ ˆ5Á 5Œ‹hwi …
CN ˆ Y“¼‹5 ˆ q'Z) †‹5 ‹%,+;  ˆ 5  } + ”  x { s  …
|  ˆ |

‹ <#> ;j]   x  { s  q„+‘h  q > … 

hˆ +‹|iQŒ‹5H ÀZ ˆ ,+ °‹p| > q m  …˜ ‘ ÀŒ   > > m  x 
+ Œ´|)Œ‹Ÿ
‘ À ˆ ‘•‘‰¼‹Ÿ ˆ  …
½ Œ ‘ ‹Ÿ ”‘  ’ Œ   …

²ButvÇ|y{ÇuQrÆ 
˜ ‹Ÿ ’‘ˆ ‹‰ 
 ˆ Š0i  |‰ Œ‹h‹5 •‘5‹‰‹5 É Œ‹ q ; 5‹;+Z‘™ ˆ ‘  ’ ‹hZ ˆ = ˆ5Á 5
‹5Z‘H‹5‘ ‘•‘ ‹‚“  ‹@‹5   h‹ŸŒ  Y“ È% Y“ ˆ  …
~  ˆ - ˆ †i®Œ‹ŸH ÀZ ˆ  …
CP ‹´ ˆ5’ ‘‹ ’ •‘|;+B †` ˆ - ˆ |0 À ˆ ‘ ˆ5’ ‹5‘Ÿ  …i )  ˆ À ‘  = Y“ ˆ5Á 5 v /  {
Œ‹52 ‹5‘†•‘|  / +Œ‹Ÿ% ÀZ ˆ  …
CN ˆ Y“¼‹Ÿ ˆ  'N) †‹5 ‹;+BZ ˆ š™‘ ,+S• Œ‹0‹5‘†•‘|5‹= - ˆ 5‹  / (  ‘ >/  
‘H h) |  9‘iŒ ' …» 5 ‹H% Œ‹ ‘™`‹5‘†•‘5‹HÉ;‘-‘2‹5 ˆ ‘Œ  ˆ  ˆ B“ ‘
 ‹‹2 L Y“ 
Œ‹%‹ŸŒŒ‹ ( ´  (  ‹5 ˆ ɋŸ ˆ ‘5‘•‘É ’‘ˆ ‘ ˆ   Z…½ “  ’ |  ’
 ˆ À† À ˆ '; ˆ •‘‰  ˆ ‹%L“ ˆ • ˆ … ' 
 h
 ‘  
(eh

 

 ( 
L“  <  ;  (h  ;   h  (h

h 
  h ( 
¡ É ,+ Y“ 
Œ‹É‹ŸŒŒ‹  / (  Œ‹ŸɋŸ ˆ ‘5‘•‘É ’‘ˆ ‘ ˆ    +Œ'|BŒ  ˆ Œ
À0  ’  Z…
¡ ‹Ÿ}+) À ˆ ‘ ˆ5’ ‹5‘Ÿ  ‘³Œ‹‹Ÿ‹
aiJŒ‹Ÿ¼‘  ÀZ ˆ  …B “ ˆ ȋ' ÀÈ ˆ  ˆ  ’  ’`œ +L‘
‹|:i ) †‹5‹5Č5  q9‹5 •‘5‹6+1 i '¼ ˆ  L“ ˆ5Á 5Œ‹•¿ ’ Z ‹5‹iZ ‹ q m  +PŒ¿%Œ‹Ÿ À
Œ ‘ ‹5 ” ’ ‹Ÿ ˆ5’  …

²ButvÇ|y{ÇuQrË+
x   ”‘‹‘'¼ 
kqts 0; 5‹É¿ À”‘°) †‹ŸŸ ¿ ’  ’‘ˆ   …

¿ 5  ‹5 ¿‘ ‘  )'Œ  š™,+‘  
 '¼ ˆ '‹ •‘5‹É  …

: 0 ˆ - ˆ |  x pH
{ ; q~m  m …

; †‹5 ‹ÉŒ ˆ5’ ‹Ÿ  ’ ¼  …


|  ˆ ‹;+‹|   q„+ ¿‘” ’ É<q~m   x p{  …
 
q~m 

¡ ¿Z ˆ Ÿ‘  ˆ +Z‹5 q ; ZOY Y  x‘| <#Y Y  x  { #Y Y …


Q ˆ i|• ‹5|   ˆ  ’ Œ‹Ÿ ,Sm>.‹·“  PŒ Ÿ‘|0 ˆ ‹; 5‹'  ‹Ÿ ˆ ‹5  ) ¼ ˆ ^‘
‘  Y“ | ’‘ˆ ‘ ˆ·…
à NŒ‹Ÿ‰‘ À ˆ ‹5 .0/21 ‹5 ‰‘™ ; 5‹‰ Z ˆ 5‘Z|  ®¼;  ‹5 +;  ˆ ‹‰
‹5 •% ˆ5’  ˆ5’ ‘ ˆ ’  ’‘ˆ ‘•‘| ‹2 ˆ Œ‘Ÿ  Œ  ‘Z¼ ‘•‘‰Š ‹Ÿ)  ˆ  ’ 5 ˆ  ˆ

—†Ê
 ‹5pÍ ˆ  ’ |HŒ Ÿ‘`q.0/21 Œ ••) |5‹0 ~+S‘0  v .0/21 {iŒ‹Ÿ
¼  …
)
| ‹Ÿ}+5 @¼) ‹5%Œ |Ÿ‘pš™ ‘5‘•‘|  ˆ ‹P) |5‹  ?  B“ ´Z ‹H|Z ˆ É; 5‹
Œ x‘ ‘ |Œ‹5)‰  ˆ •‘| ˆ ‹
¼) Œ‹
Z ˆ Œ‹ …B ‹ŒŒ‹
Œ Ÿ ‹;+S   ˆ ¿B“  ‹Í ˆ •‘
)  ,2m>rzd5€e> c uh …

 ’ ‘ ˆ x|‘•‘;+Z‹Ÿ
v .0/21 {Œ‹Ÿ2¼) a'

½ % ˆ 5Axz.†1a{)Z ˆ H 1 À0‘‘™'‘³Y À‹N - ˆ 5‹;+v‘ J‘0‘™


‹5 ‹‘‹5‘¼ Œ‹
‹ 5‹ ? ‘  ˆ ) |h
 B“ Œ‹Ÿ%‹5 ˆ xz.†1a{S)+  ’ ‹   ‘( …

) †‹5   ; v   /Oxyxyx&/ )C{‰‘ (|; v  /Oxyxyx&/{ +xŒ5sjZ‰‘ ’ ‘|Ÿ‘  ‘•‘(Ex …

  (´Œ‹Ÿ2 :xa'

¿; †‹´  ˆ ‹ ;.   17‘ ;.  1 …
‹2; ˆ 5'´ ’  ’‘ˆ   ’ +Z ¿)‘%‹Ÿ) †‹5 ˆ    ;
 ‘  ;
  …

¿ 5 ? ˆ Œ‹Ÿ … ' ) ‹|A  ˆ  ’ ‘ ˆ Œ ‹5 ˆ   . 
s1 ? ˆ Œ‹Ÿ … .  z1a) …
… '  )2=Œ |Ÿ‘Z ‹´L“ ˆ ) |  ? ˆ Œ‹Ÿ … (;)%$
|  ˆ ‹´‹5‘  ? ˆ Œ‹Ÿ   ? ˆ Œ‹Ÿ …




¡   •Œ Ÿ‘|'Z ‹
 ^ ‹
=; ' 
( =‹ŸP‘
‹‘ ‘•‘|'‹Ÿ3m   +) …  …  …
  
h   …
%


 0 À0 Á •
Œ ‘ ‹Ÿ  ‹Ÿ ˆ '  …
 ,+%. ‘*7
1 Z ˆ Ÿ‘‘|´š™ ‘5‘•‘|  ‘™È¼) ‹%‹5Œ‹ …

  (´Œ‹ŸExa'
» 5 ‹%Œ‘ À‹Ÿ†  '|k. ‘17‹5 |h‘™i‹5 •‘5‹hŒ ‹ ’ ‘Ÿ< ‹% …
½  ˆ ‹ ‘•‘|° ˆ5’  ’ ‘| ˆ Œ‹55F† À¼ ”)  ˆ  ˆ - ˆ  . ‘31 Z ˆ Ÿ‘‘| 
š™ ‘5‘•‘|h°¼;  ‹5| …

˜ ‹Ÿ ’‘ˆ ‹@5‘Z|p % ˆ ŠAi^ |H Œ‹H‹ •‘5‹@‹5 @ Œ‹P) |5‹   ‘@ Œ‹  ˆ5Á 5Œ‹
‹ |Éš™ ‘5‘•‘|h Œ‹%‹5‘†•‘|5‹h |% Œ‹hš™x ˆ5’  ’ ‹Í ˆ •‘%¼) ´Z ˆ  …

d mf9d5€}‚@r‚ 
@

) ½  ˆ Š0i Œ ‹Ÿ%‹5  ? ’ ‘5)2‘2 ÀZ ˆ  …


¼9) ½ ^ ˆ Šyi Œ ‹Ÿ¿Œ š™ ‘°5 5Œ‹”‹Œ‹3  ŒŒ‹¿‹ |¿Œ‹° ˆ À† Œ‹ š™Œ‘ ’ ^ À Y Œ
š™ ’‘ˆ ‘ ˆ    …

@ dS,h lBh 
 )N) †‹5 ‹P Y“ ˆ5Á 5  .A1‹5 1Œ  ’ p| ’‘ˆ ‘ ˆ ‘•‘HZ ˆ % ˆ % ˆ5Á 5‹5‘†•‘*'
CN ˆ  5 ‹' Œ‹'; 5‹ .†1   Z ˆ Ÿ‘‘  ` ˆ È ˆ5Á 5,+S   5  Œ‘ 
 % À‹ŸŒ  1 Œ‹52  …
à Sš™‹Ÿ5 
= ˆ5Á 5'‹5‘†•‘   e| ˆ ‘Œ | ˆ   .†1 e‘  +Z‘   v  /  { ? Z ‹
 ˆ ‹%; 5‹É
 7 † ’ ‹5) …
˜ •w. ‘†17‹5 h
Z ˆ h‘hL“ ˆ '    + 
‹‘†•‘|    S
)‘EY ˆ Z ˆ Ÿ
 ¼) ˆ ”  …F “ ˆ ȋ´‘³Œ‹5‹5‹;+Z Nš™x‹55  ) |    75‘ N|   ‹5 ‰”¼) 
 ˆ À† ,+U‘'P‹5 '  Á ••Œ¾  ’ :!x  a{´a1 …S˜ ¼)   ˆ À†     )‘' 
Œ 5‘ ˆ  ’‘ˆ ‘ ˆ ‘•‘|ÉL“ ˆ É;    | F/ !/  … CN ˆ ‹Ÿ5,+ À
 ˆ 5 xz.†1a{1F‘ ˆ 
‹H h ˆ À†    D‘   p‘ ˆ Œ‹‹5 ˆ Ÿ ˆ ‘` ˆ  ˆ ‹2ÈŒ‹ Œ¾  ’ ‹        +Z ˆ
š™‘      ‘
  '…
 ‹É  ˆ + ‹  ' 1 %  1 +Œ
BŒ | ˆ Œh À=  ’   …

¼ )  “ ˆ ȋB‘³Œ‹5‹Ÿ‹;+  Z ˆ B Œ‹B‹5‘†•‘5‹U>Í ˆ •


9  ‘S e¼) ˆ 
  +Œ5 5Œ‹B Œ‹S ˆ Œ ‹N ˆ5Á 5Œ‹
ˆ ‘ ‘ Œ‹•‘'¼ ˆ Œ‹=L“ `¼; Z ˆ  Z ˆ Ÿ ‘  š™ ‘5‘•‘‘™ ¼; ‹' ˆ À† Œ‹;+

—„
5‹ Ÿ ’ ‹ hZ ˆ 2‘ L“ ˆ ´h Y“ ˆ5Á 5´Œ ‹Ÿ ’‘ˆ5’  …%
È‘i ’ 2|h5 5Œ‹p Œ‹!  ŒŒ‹ ? Š ˆ ‹p À
Y Œš™ ’‘ˆ ‘ ˆ 6)2(i«‹5 ÉŒ‹2 ˆ À† Œ‹w'
¡ ”ƒL‘;+‹ŸLY Œ  ’ % Y ,+F;Œ 5s … 
 “ ˆ ȋi)+1 ”;  x   Œ‹Ÿ`‹Ÿ ,+@‘È‘™.‹ •‘5‹`Œ ‹ ’ ‘Ÿ< ‹q. ‘~1   
‹ ˆ ‘| ˆ ‘  ’ ‹h| ˆ ‹2 Y“ š™ ’‘ˆ ‘ ˆ   +ŒSŒ  ˆ ŒÉ À0 ˆ  ˆ  ’  ’ ‘³Œ‹‹Ÿ‹ …

 • 0 ƒL ˆ    ˆ À† ÀŸ ^    Y“''¼; ‹É ˆ À† Œ‹ … É) | 9
 ’  Œ 5‘|°‹  + )  Z ˆ 5‘ ˆ ? ȋ5‘‹@ À ˆ 6)  iɝŒŒ‹H ˆ À† Œ‹ …˜ %%)‘
Á  ˆ %| ’‘ˆ ‘ ˆ ‘•‘|;+x‹5B“  Z‹·“†H2¼; ‹P ˆ À† Œ‹;+Z‹5 ˆ p¼; ˆ ‘‘Š ˆ ‹@Œ‹H‹5 •‘5‹
‹| ›'Œ Ÿ‘|‰Z ‹% ˆ ‹h; 5‹h † ’ ‹ …)˜ “ Œ‹Ÿh '| DŒ‹Ÿ% ‹5 •‘ÉL“ ”¼) 
 ˆ À†  …;˜ ‘ À• ‹5‹Ÿ ˆ  À=Œ š™ ’ wi +‘É ŠÄ@
À0 ˆ ‘-´ À= ˆ  ˆ  ’  ’†…

C1‹5 imŒ‹Ÿ=‹Ÿ È ÀZ ˆ °‘0Œ š™,+e  )‘= `Ÿ ‹ ˆ À: ˆ ' °L“ ¡   ˆ '
 h jmB= . ;=< …

 = ˆ Œ‘5‘•‘|p't; q …
x  ´ '¼ ˆ  ˆ À† Œ‹ … |  ˆ ‹ ‚; hZ …

 “ ˆ  Z ˆ ;+p5ŠZ   ˆ À† ®   ˆ ‹° ˆ5Á 5Œ‹°‘i5ŠZ ^ ˆ5Á 5®Z ˆ Ÿ‘  ‘™
 ˆ À† Œ‹6+2‹Œ‘  Œ‹¿AÍ ˆ •‘°  ¼) ˆ D }+p‘  Œ‹;+ B“Z ˆ Ÿ‘‘|®|B“  D‹5‘
 ˆ À†  …
. ;  x pe
( _
| ‹Ÿ„'-j { ;
 6   ` 6
( h  ; ( … “   ; …
{ m  x  e
( _
` 6
½  ˆ ‘ ÀŸ ”L“ ¡   ˆ ‹·“ ’  ˆ h  ˆ ‹(i
' q hx hZ6e { ;=<
 …  …  …  x  H{ ; q~m  m +Œ •'  ’†…

²ButvÇ|y{ÇuQr4
à BŒ‹Ÿ ’ x‘|h|B“ 
Œ  ˆ Ÿ   ˆ  ˆ 
wi   ’ ŒŒ‹‹Ÿ5'°• ‹É‘™¿Œ  ‘ ˆ ‹ …

à )‹Ÿ •2‘‹Ÿ5´Œ ‹Ÿ5 ˆ ‰‹5.>9/ ɋ |%‰ Á •‰Z ˆ  ’ ‘p‹5Ÿ‘5‹;+  ˆ ‹r> h ÉŒ‹Ÿ 
Z ˆ ‘1 ‹e ˆ | Ž…† p ‹1p‹5 |@Z ‹P Ÿ Œ‘5‹ … Ce ˆ Œ ‹ ’ ‘|;+ =)‘PŒ ‹Ÿ ˆ  ˆ 
 <vŒ  ˆ Ÿ Qai  ‘^Œ  ˆ À
5 ‹´ Œ‹‰ '¼ ˆ Œ‹
Z ˆ ‹‘®¼ ‘^‘5 ‹´ Œ‹‰Z ˆ ‹
‘
ˆ  …Z¡ É 
 Kxi  H { ;=< …

²ButvÇ|y{ÇuQr (

¿ ˆ -  ˆ5’ ‹Ÿ ɋ5 ˆ  '¼ ˆ %q ‹5 •‘5‹ …
CU  ˆ q~; Z,+ ÀŒ ‘ ‹Ÿ ”Œ‹5 ’ ‘5 ? Œ •)  ˆ q   L“  ‘ ˆ ‹5) …

x  q s Z  ™ ’†…E
\‹Ÿ) †‹5¿¿ ÀQŒ ‘ ‹Ÿ DŒ‹Ÿ ’ ¼  ;  ˆ 5 ` ˆ Š ‹Ÿ 
 ÀZ ˆ 
B“Œ ||‰Z ‹2 ‹%  q ‹5 •‘5‹ …
x h  ˆ ‹2¿ ˆ Š
‹5  ÀZ ˆ wi«k  q hZ
‹ •‘5‹ …

 …x  Y“ š™ …Ã5à  – +|
5 É ˆ Š'‹5 
 ÀZ ˆ 0i.; †‹5‹ ’ %”‹5 •‘‰
‘ ˆ5’
 ‹ ’ |  œ… x !.J`5‘ Z‹5 •‘;+|‘Ki ' 2 ˆ Š%p‘‹Ÿ ˆ  . ‘@ Œ‹  ˆ5Á 5Œ‹
Œ ˆŸˆ Œ‹5; 5Œ‹ … “ ˆ ȋp Y“ Š| x; ŸŠȋ ˆ5’ ‘ ˆŸˆ ‘Œ,+5ŠZ Œ ; †‹ |5Œ š™(i ';Œ‹Ÿ
 Œ  ˆ ¼  ? ¼‘ ‹S= ˆ +)  Ÿ ‹ Œ‹´ Á •Œ‹'Œ  ‘ ˆ ‹´;  ˆ 5ŠZ Œ ) †‹|56) …U˜ •….
B“ Œ‹5UZ ‹1 5 Œ‘|   ‹1 œ ‹5 •‘5‹;+|‘  Œ‹eH‹5 ‘e Œ‹NŒ  ˆ Ÿ ‹@5Š ‹ŸŒ‹e‹Ÿ ˆ 5ŠZ 
Œ ) †‹5,+) ˆ Œ‹55 ˆ 5 >Ÿ  ˆ ‹
®• ‹´Œ  ‘ ˆ ‹Ÿ; ¼ =)  ˆÁ  ˆ Ÿ ˆ ¼ ’   . +
‘%Œ ‹5 ˆ  ˆ ‹ŸL  Œ  ˆ Ÿ  wi …

²ButvÇ|y{ÇuQr4

¿ ˆ -  ˆ5’ ‹Ÿ %Z ˆÉˆ5’ ‘ ˆŸˆ ‘Œ'‹Ÿ ˆ  ¼ ˆ 

‹5 •‘5‹|_ q …
 6 (  "6 
 ‘ ˆ  ‹25 ÉL“¼) ˆ ¿|'‹Ÿ  s q hZ,+  ˆ ‹  6   ( s (
` BŒ‹Ÿ2  ¼
ˆ
v™' L“ ˆ5Á 5Œ‹2‰;‘p) †‹5‹5Č ˆ i ˆ  Š‹Ÿ ‰ i ˆ ‘ ’  q®‹ •‘5‹ …à )‹Ÿ •
 '´ ˆ - ˆ  ˆ5’ ‹Ÿ %)  ˆ   q …

CU  ˆ q~; Z,+ LB“ ¿ ˆ ‘    ˆ  …

—†Ž
x  q s <  ™ ’†… ) †‹5 ‹` À Œ ‘ ‹Ÿ  ’ ¼  ;  ˆ 5  ˆ Š L“\ ‹ q m Z
‹ •‘5‹ …
CN ˆ P5 ‹
Œ‹ ˆ ŠŒ‹'Rq.‹5 •‘5‹;+L‹ ‹ ‘ |,+L  Œ ‹5Ä ˆ ^ ˆ Šqi Œ |
°v™' L“ ˆ5Á 5Œ‹ …
; †‹5 ‹É|wi 
Œ Ÿ‘
Z ‹%~x m Z6{‘ |~'
|  ˆ ‹;+ š™‹Ÿ5p‘™0‹5 •‘5‹;+†‹ ‹ . 1‘„. ( +  ˆ ‘  ’ ‹eZ ˆ 2 ˆ5Á 5 ? ‹5 m5Z,%
q { …L¡ Q  'Œ‘Ÿ5 ˆ5Á 5  Œ ‹Ÿ ˆ ‹51^ ˆ Šqi'eRq.‹5 •‘5‹6+L‘Œ |
 ‹
L“ ˆ5Á 5Œ‹É|(i … “ ˆ ȋ2 À0v™  ’ (i!+†“ Œ‹5% |(i ' Œ |Ÿ‘|h ‘ |‘%
Œ‘  š‘eŸ ‹50 Œ‹´‹5 •‘5‹.  ‘+ . ( ? ‹Ÿ +)‘  =‹5 ˆ ‹5‹Ÿe‹i ) … | ;‘  ‹. >/Oxyxyx&/2+ . 
Œ‹p ˆ Œ‹2‹5 •‘5‹p‰Œ‘Ÿ5  ‘  … |  ˆ ‹;+Œ. (,+| ‹# ˆ •‘|p…x kmtZ6{‘ ‰‹Ei +
ŒBŒ | ˆ Œ‰  ˆ Š ) ŸŠȋ5 …
 ,+ i Œ Ÿ‘|%ax  m Z6{‘  … x    Y“ ‘‹5‘¼ 
‹5 •‘5‹%L“ ´5‘  ax  mZ6{
‘ ,+‘ ‚; j  › Y“ ‘‹5‘'¼ 'Œ‹É‹5 •‘5‹%L
‹5 %Z ‹%‹  …
|  ˆ ‹;+| Œ‹@‹5 •‘5‹    ’ ‹‹5‘|pš™ ‘5‘•‘
_  6  _  6 (
`
(
`
 ˆ5Á 5Œ‹ … “ ˆ ÉZ ˆ ;+xL“ ˆ ȋ
Y“ Š ) ŸŠȋ5´ ˆ5’ ‘ ˆŸˆ ‘Œ´Ÿ ‹ ’ h‹5 ˆ h ˆ Š´  2 Œ‹p‹5 •‘5‹p‹5 |2Œ‘™`;‹ |
 ‹  ? ‘0|H `Œ |Ÿ‘|0Z ‹0 ‘ 6)+B Œ‹0‹5 •‘5‹)    ` ’ ‹5‹5‘|
 ( _ "6   
 ‹  66   (
`
 ˆ5Á 5
 Œ
 ‹ …
¡   +S5 >Ÿ  ˆ ‹‹5| i `Œ |Ÿ‘|Z ‹0 ‘ ,+N5ŠZ `‹5 •‘=)   •)‘
Á  ˆ  5 Œ‘|   ‹  m<ȋ5 •‘5‹  … Ce ˆ ‹Ÿ5,+; 0 '¼ ˆ L“ ˆ5Á 5Œ‹
 ˆ ‘ ‘|
‹ •‘É
 ›‘2 ‹5 •‘É   ’ Z ‹‹5´Z ‹0x Rm <{@x q m
%hZ6{ …
½  '¼ ˆ L“ ˆ5Á 5Œ‹É(i  ’ Z ‹5‹
 Z ‹
_ "6   
{ ;  66   (  +·Œ@| ŠÄ@P À2 ’ • ‹5 ˆ Ÿ  …
_  6  _  6 (  6 (  (
ha x  m <{@x qm  h Z6H
` ` `
( h  6   (

J hnae>dzM,-h 
˜  Ÿ ˆ |Œ‹ŸŸ5‘|=  ˆ ‹5 qJŒ‹Ÿ' ' Ÿ ° 5mjZ …N¡ .ƒ)‘;+N‹0Œ`· ‹;+U
‹5 •0 ˆ5’ Z ˆ Ÿ ˆ q\‹5 •‘5‹=‘ 5mjZ ˆ )Œ‹= u;  6  ‹5 •‘5‹;+e‘ȝŒ ‹Ÿ ’‘ˆ  ˆ 
 ˆ Š~x Rm Z6{YZ ˆ Ÿ5'Œ ‹Ÿ‹Ÿ   ˆ  Œ ˆ ' ˆ5Á 5‘ ˆ 
‘™i‹ •‘5‹É‹ŸL‘%‹5‘ ‘•‘h‹Ÿ
ŒŒ‹´‘™®‹5 •‘5‹´0Í ‰Z ‹‰Z ˆ ŸL“ ® Á •= ˆ ) …;à eŒ‹ŸA  ‘ 0  ’‘ˆ    ˆ |B“ ®5‘
 ˆ Š'Œ |Ÿ‘|´š™ ‘5‘•‘|  66   (   ˆ5Á 5Œ‹;+‹2Z ‹É ‘ | …
 (

²ButvÇ|y{ÇuQr4
˜ ‹Ÿ ’‘ˆ ‹% ´ ˆ Šwi  % Œ‹2‹5 •‘5‹%‹5 % Œ‹ „·¢†¢†   ’‘ˆ ) ˆ 5‹;+‘™ ˆ ‘  ’ ‹%Z ˆ  
 ˆ5Á 5 ‹ŸÉ‘‹‘ ‘•‘|i‹·“  ‹‹5  ˆ ‘  ’ ‹`Z ˆ   ‰ ˆ Œ‘ …@ “  ˆ ȋ` Y“ ’   ’ +HŒ° ˆ Š Œ ‹Ÿ
‹ ‹2 ˆ À†  … “ ˆ ȋ% ´ŸŠ ’ ˆ đ•
  ˆ  ? ‘% 5‘ ^)+9i Œ |Ÿ‘´°v™'


 ;  
<<;#Y YZ= ˆ5Á 5Œ‹ …

²ButvÇ|y{Çu®ƒµ4
x †i  ˆ Š
 |É Œ‹%‹5 •‘5‹É‹ |% Œ‹2) |5‹6+‘™ ’  ˆ ‘  ’ ‹%Z ˆ 
 ˆ5Á 5
‹ŸB‘
‹‘ ‘•‘|h‹5L À=Œ ˆ 
|B“  ‹É ’ ‹5‹5‘|‰‹5 ‹Y5‘i † ”Œ‘| ˆ 
 ’ Z ‹‹| Z<#Y  …
x ‘|w.0/213/54  ˆ ‹Œ‹´; 5‹;+L Y“  ‹5; †‹5=‹Ÿ) †‹ ’ ‹´‹Œ‘
ˆ  ˆ ‹Ÿ ˆ =Œ ˆ ‘ 
? ‹‘‹U ˆ    ’  ˆ 6) …˜ •p Y“† 25  ˆ Œ‘| ˆ  JŒ ˆ ‘ 2Œ‹Ÿe  Y  ; : Z<#Y  +
 Œ‹Ÿ   ‘    ’‘ˆ    ˆ | ŒŒ‹N; 5‹e@)‘-‘| Á  ˆ  ‘™  ‘™ ˆ ‘  ’ ‹eZ ˆ p ˆ5Á 5p‹N 
 ˆ Š …|˜ ‘ À´‹5†  2|i Œ‹Ÿ1‹‹e ˆ À†  …½ pŸŠ ’ ˆ đ•É  ˆ  ? ‘e®|5‘ ^)P ‹5‹Ÿ ˆ 2  ˆ ‹
pi ; †‹‹5Č'Z ‹´ ‹´    ˆ5Á 5Œ‹ … CN ˆ ‹Ÿ5,+;  '¼ ˆ   =Œ ˆ Œ‹|e ’ ‹5‹5‘
( 

 † 0= ’ Z ‹5‹Z ‹ Z<#Y  +; …  …  … 0 ¼ ˆ 0Œ  Œ‹´  ˆ ‘  ’ ‹´Z ˆ  ˆ5Á 5,+)Œ‹Ÿ
`• ‹ ’ | 
(  (

( m
(  
 ;  ; < /   …Z˜ • °Œ‹Ÿ ’ ‘•‘ i‘|Ÿ ˆ +x °
 

3sjZOY Y …

—†—
²ButvÇ|y{Çu®ƒ)r
˜ “ Œ‹Ÿ
ÀÈ Á •• ’ |‹' Y“ š™ ˆ ‘Œ ˆ5’  ’ ‘3' à P‹Ÿ •' ˆ ‘ ˆ | ˆ •‹ŸP‘™
) |5‹É‹5 |ɋ5Ÿ ’ ‹  ‹ŸŒ
‹5 ’‘ˆ ‘ ˆ   <-+  ˆ ‹% Y“† '¿Œ‘ ˆ 'B“  ‹% ’ ‹5‹5‘
Œ‹5H‹Ÿ ’‘ˆ ‘ ˆ  #Y  …-
È‘i ’ %‹ ’ •‘|2É h ˆ Š´ |p Œ‹ ‹5 •‘5‹2‹5 | Œ‹ ; 5‹;+
‘™ ’  ˆ ‘  ’ ‹
Z ˆ  ˆ5Á 5‹5@‘'‹5‘ ‘•‘‹·“  ‹‹5 |  ‹ŸŒ`‹Ÿ ’‘ˆ ‘ ˆ    <
Y“   Y“ ˆ ,+
Œ Ÿ‘|‰Z ‹% ‘  …Z½ 
ŸŠ ’ ˆ đ•'  ˆ  ; ˆ •‘h  ˆ ‹hL“ ˆ • ˆ
Œ ˆ ŠŒ Ÿ‘|‰Z ‹2 ‹%   ˆ5Á 5Œ‹ …

²ButvÇ|y{Çu®ƒZƒ+

•Œ ‹ŸÄ ˆ 2   ˆ Š%‹Ÿ i  |@ Œ‹P‹5 •‘5‹P‹5 1 Œ‹ ’ ’ •‘|5‹  K
 +†‘1;  ˆ Œ‘
v  / " {0Œ‹Ÿ2' ˆ5Á 5
‹ŸB‘%‹‘ ‘•‘|h‹5.h  ‘*. h "ȋ |É‹zŸ |5‹ …
½ “ ¼Œ‘Ÿ<2^ ˆ ¼ đ••Œ‹Ÿ •0 ˆ - ˆ aiJŒ |Ÿ‘
 ZOY Y‘ | …e˜ ‘ ÀÈ ‹
 5i   ’    ˆ È '¼ ˆ iL“ ˆ5Á 5Œ‹3i   «~  ˆ ‹Ÿ Z ˆ ŠZ ‹ ˆ Q `ŸŠ ’ ˆ đ•°
 ˆ  ?  … š™ …H„·  )´) ˆ •‘Ÿ ˆ =Z ‹'`Œ ‘  ˆ  …U½  ˆ Š3iD; †‹5‹Č~ZOY  ‹5 •‘5‹0‘
>¿ ˆ5Á 5Œ‹ …;½ Œ ‘ ‹Ÿ ^ ’ ‹Ÿ ˆ5’ 0 ’ Œ  0 ’ À5‘•‘|'0ŒŸŠ ’ ˆ đ•0‹5S Y“ ® ˆ -

>   
  
  …
(


hˆ + v 8/
Œ ‹Ÿ” ˆ5Á 5”‹Ÿ2‘‹5‘ ‘•‘|ȋŸ}+e)  ˆ 5 ‹/zq  .0+P;Œ
" { ; q„+1 #

 m " ; …½ ˆm 5
‹ 
 
 
q
Œ
 p
‹ 


 ¼ ˆ Œ
 É
‹ 
 
 Ÿ
‹ Ÿ
 
 
 ‘
 5
 ‹ ‘
 
r ^
q  
’ ˆ &
 
 ‘
 
 †
 . +
 
´
 

'
 ¼ ˆ 
, m ®
q  ˆ ‘
. ‹ ZOYZ  ZOY Y† ‘ ˆ ‹•<ƒ ’‘ˆ ‘|5Œ‹ …H ,+P;  ˆ     ™ ’ +P 2 «^  ‹ ZOYZ  ZOY Y
’ ’ •‘5‹ "#  |‰®‹5 |iZ ‹° 5 Œ‘|5‹  9‹i   ˆ Š i +pŒ®|´‘| ˆ ^
 mjZm ZOYZuZOY Y
 x!{ s ZOY …P˜ Œ‘ ’ |p ˆ H)  ˆ 5    E+U‹Ÿ Y“ « 5 > ` ¼ ˆ 
L“ ˆ5Á 5Œ‹h(i.  ˆ ‹('
Z Z  
> ; 6 x {
 ! s !ZOY  ZOY mZ
m ZOYZ tZOY Y
< <

8

à ˆ Œ‹Ÿ5   ’‘ˆ    ˆ |  ( ZOY  zZOY  mZrmZOYZ tZOY Y 




 mZ,+Œ
BŒ‹Ÿ%‘ À ˆ ‘•‘† ˆ   …
    


(

+2 …  …  …  ZOY
 
< h-<tZOYZOY YR; ZOY

²ButvÇ|y{Çu®ƒ¸4
¤
 Œ ‹Ÿ ˆ ´ 
 ˆ Š0Œ  ‘  „‚ ‹5 •‘5‹6+F‘‰  Œ  ˆ 0 ˆ5Á 5‘ ˆ ,+;‘
 ˆ p i‘ȼ ‘i‹5‘  i|B“ ‘   ˆ ‘ ‰‘™) ˆ ‹5 Œ‹p|)‹5 |2Œ‹;+‘‘Œ‹2 È<ƒ ’‘ˆ ‘5Œ‹
Y“   Y“ ˆ  …
x *.m¿‹ •‘h ˆ ¼ ˆ  ˆ  …Zà LŒ‹Ÿ%
‘ ˆ5’„·Å ‘% ,+L“ ˆ Č‹É ´ ˆ ‘;'Œ‹2Ÿ ˆ  ˆ ‹;+
`• ‹ Å Œ‹H ˆ5Á 5Œ‹pL“ š™ ˆ5’  ’ .D‹5 HL“ É Á •hŒ  ‘ ˆ +x‹5 ‹ ˆ  … F‘™hŒŒ‹
Å ‹ •‘5‹ ˆ ‘  ’ ‹  .›Z ˆ • ˆ5Á 5 ˆ •‹ |'‘™xY Á •Œ‹ ˆ ‘  ’ ‹Z ˆ • ˆ5Á 5 ˆ ,+B 
` ˆ - ’   ˆ = ˆ )== ˆ ‹ …) ‹´ =· ‹Œ  ˆ  ˆ ,+B  `  Å ‹5 •‘5‹ ˆ ‘  ’ ‹‘™ 
‘™iZ ˆ Œ‹É ˆ5Á 5Œ‹%Œ  ˆ  ’ Œ‹ÉZ ˆ ‹5‘ ‘•‘É‘™iŒ  ‘ ˆ ‹ …9
” ° ˆ5’  ’ ‘•‘‰B“ 
5‘  =Œ   † ˆ Ÿ QŒ 5‘Z ’ ŒŒ‹5‹ ˆ ‘•‘
® ˆ À† •  x5Š ˆ Ÿ,+BŒ=N) ˆ •‘
Œ ‘  ˆ  …

¿Œ ‹Ÿ ˆ h ´ ˆ Š
Œ  ‘  „·  ‹ •‘5‹;+‘% ¿Œ  ˆ ' ˆ5Á 5'‹5 %‘ ˆ 

‹ i‘  ˆ °‹5‘  9B“ ‘   ˆ ‘ ^‘™\) ˆ ‹5 Œ‹°‹5 |”Œ‹¿ D‘‘Œ‹ … x :. 
‹ •‘ ˆ ¼ ˆ  ˆ  …Uà PŒ‹Ÿ'‘ ˆ5’”„‚ ‘' ,+UL“ ˆ ȋ' • ˆ ‘;•Œ‹'Ÿ ˆ  ˆ ‹;+UQ• ‹ ¢
Œ‹% ˆ5Á 5Œ‹ÉL“ š™x ˆ5’  ’ . ‹5 |%L“ ´ Á •Œ  ‘ ˆ +Z‹5 ‹ ˆ  … | )‘  
Œ‹É‹5 •‘5‹%Œ Œ ˆ  ’ ‹ …
‹   O/ xyxyx& /  , / 

É}+)Z ˆ 1 Œ‹    ˆ5Á 5Œ‹'L“ š™x ˆ5’  ’   ? Z ˆ š™‘ 6)´‘´Q ˆ Œ‹) |5‹  +; 1
‘”°• ‹ Å |B‹5 E ˆ 5Œ‹w' |  ˆ ‹;+xZ ˆ B Œ‹ Å ‹5 •‘5‹ÉŒ ˆŸˆ Œ‹Ÿ; ;+Z ¿‹%B“ 
)‘‘^ ˆ - ˆ  ˆ ‹ Í ˆ •‘^ ˆ À† •  Š ˆ Ÿ| … PŒ0 ˆ À† •Œ‹Ÿ ˆ ,+
  ˆ ‹0 È ˆ )q ˆ  ’  . ‘Œ‹ ˆ ‹‹ •‘5‹Œ‹Ÿ« ˆ )i – ) ˆ ‹5 Œ‹= ¬ )

—–
Š ˆ 5Š ’†… LŒ‰ ˆ À† Œ‹Ÿ  ˆ ;+Œ‘Ÿ5†Í ‹2†“ Œ‹Ÿp ‰ ˆ )* ˆ  ’   2‘2Œ‹p ˆ ‹2‹5 •‘5‹
BŒ‹Ÿ%Œ  ‘Z¼  …
˜  ˆ ‹5 ‘•‘|´‹·“ 5 ’ x‘•‘|‰‹ŸL Y“  ˆ ‘ À Œ   Z ˆ   ˆ 
Œ‹+ …
}+ )  ˆ 5ŠZ  } + Z ˆ  Œ‹    ˆ5Á 5Œ‹° ˆ ‘    5ŠZ ‘9Œ‹¿ ˆ Œ‹i  #‘9
š™ ‘5‘•‘| œ |N‹5 |A ˆ 5Œ‹;+)  ˆ ‹É  ˆ Š'‰Z ˆ Œ‹ ¢ ‹5 •‘5‹   /Oxyxyx&/  ,/ 
° ˆ Š  ¢ ‹5 •‘5‹ÉB‹5 %5 ‹%
‘ ˆ5’ Z ˆ ‹6+Œ
BŒ‹Ÿ2; †‹‹Ÿ¼  ?  …Zà ) …
Œ‹Ÿ

eZ Y“ Œ‹ ¢ ) |5‹;+‹5 ‹   +Œ‹Ÿ ˆ ‘  ’  ` ‹ –  ˆ Œ‹HŒ‹  Z ˆ É ˆ5Á 5* ˆ 5 …
|  ˆ ‹;+ ;Œ‹5 ˆ ‘  ’  `• ‹ – Œ‹   ) |5‹ Z ˆ ‰ ˆ5Á 5 ˆ ,+‹5  ‹ )(> /  >/   /  … 
Y“ Œ‹h ˆ5Á 5Œ‹‰| ˆ ‘ ‘|h‘™”'ŒŒ‹ – ‹ •‘5‹ÉŒ‹Ÿ ˆ ,+Z ˆ š™‘ ~x )(> /  6{+;  ˆ ‹


.0 /   / 
(> /  0Í ˆ •‘  ˆ )® x;=Š ˆ 5Š ’†…B ‹´ · ‹´Œ | ˆ  ˆ ,+B†“ Œ‹5  ˆ )
)( /  >/   /   |L) ˆ •‘ɍŒ ‘  ˆ  …

²ButvÇ|y{Çu®ƒÆ 

 †` ˆ - ˆ  ˆ5’ ‹Ÿ 
‹5 ‹´ ˆ   ˆ •,+L‘® ’ •  ˆ |0|ai\Œ‹Ÿ´¼Z ˆ Ÿ5•‹Ÿ
‘%‹‘ ‘•‘|h‹‚“  L; †‹‹5Č´Z ‹%
‘ ‘   †‘ ˆ Z ˆ  …
» 5 ‹É5 hL“¼) ˆ ¿ Y“ ’ &  ‘Œ‘| ˆ 
Œ‹h‘™ ˆ5’ ‹Ÿ 5‹0' ¡ ”ƒL‘;+Z‹Ÿ8i.Œ‹5%¼Z ˆ Ÿ5
 L‹Ÿ •%´Œ  ˆ  ˆ ŠZ ‘¿Œ‹%‘™` ˆ )Œ‹%´‹5 •‘5‹2 ’ )‘|5‹hL“ ‹5‘ ´Œ  ‘ ˆ
)  ˆ ¼5‘ ˆ 
 <vŒ  ˆ Ÿ  …  ’ ‘ ˆ x|‘•‘;+‹Ÿ
 <vŒ  ˆ Ÿ Èš™x‹Ÿ5,+ Œ‹@‘™0 ˆ )Œ‹
 ˆ  ’ ‹2Z ˆ Œ‹É‹5 •‘5‹%L“ 
 Á •'Œ  ‘ ˆ  ‘|h À=¼Z ˆ ŸŸ   ’ ‹Ÿ ˆ5’  …
É„iJŒ‹Ÿ´¼Z ˆ Ÿ5 'C1‹5=5 5Š‘Y
 5 ˆ  ˆ  ‹5 •‘L“ ^ ˆ ;‘´
‹ •‘
 Y“ ˆ ,+L PŒ‹Ÿ‘ À ˆ = Y“ ^=;‘ ˆ @‘ ˆ ‘^^ Á ••‹5 •‘
B“ ‘QŒ |
Ÿ ‹ ’ # ¼ ˆ  Z ˆ L“ ˆ5Á 5Œ‹È|5 ˆ  ’ À ˆ Œ‹ …p¡ i ”B“ \‘ ‘ ¿ )‘`B“ Á  ˆ ”
 †‘ ˆ Z ˆ  …
% i.; †‹‹5Č
Z ‹´'‘ ‘ 0'  †‘ ˆ Z ˆ  '
 †Œ ‹Ÿ ˆ  ˆ 0¼Z ˆ ŸŸ 
ai … CN  ˆ Œ‘ À+S  ˆ ‘ ˆ |•5 'L“¼; ˆ Q|B“  @‹5 •
•Œ ‹Ÿ ˆ  ˆ •=5‘  ¼Z ˆ ŸŸ 
‹5 ˆ ŠZ |Œ ) †‹5
Œ š™,+‹|B“ ‘‹Ÿ5´ ‹%B“ ˆ ‹2 ‹2|B“  ˆ5’ Z ˆ  ˆ  ˆ Œ‹2‘™
 ˆ )Œ‹'`‹5 •‘5‹ ’ ;‘5‹0‹5 ˆ 5ŠZ Œ ) †‹|5`)  ˆ  ˆ • ˆ Œ‹‘™^ ˆ )Œ‹
Š ˆ 5Š ’ ‹p‹Ÿ ˆ i^ ³Y Á • …¡ q ;+ F‹Ÿ •@ h% ˆ - ˆ  ˆ5’ ‹Ÿ H;  ˆ • ˆ ŠAi®
Y“ ”;‘%‹Ÿ) †‹5 ˆ Œ š™ …
x @  ˆ ‹ . •‹5 •‘P ˆ ¼ ˆ  ˆ  … CN  ˆ 5 P‹5 •‘  ? 0Œ  ˆ ‹„.k)+  5  !x F/2.k{
À ‡>€^lr}e>9‰5hÈ‘| ˆ   ‘0.0+U …  …  … À¿  †‘ ˆ  ÈL“  Š‘ ˆ ‘ À  ‘0. ? ;Œ
 xz.w/2.k { ;=Y
Z ˆ  ’ Ÿ 9{ …½ 'Œ š™x ’ †i  ‹5‹Ÿ ˆ %,+)  ˆ 5    +| % ¼ ˆ   !x F/2.k{
Œ‹52¼‘” ’  …

  5 ? ˆ Œ‹Ÿ …
)% Y“ ‘‹5‘'¼ Œ‹´‹5 •‘5‹´ |´ À`‹ŸŒ  . Œ‹ŸhZ ˆ  ? ˆ Œ‹Ÿ … =
Z ˆ 6) …à ;Œ‹5 ‘ À ˆ  ‘
 ˆ •‘|2hZ ˆ ŸŸ ¿h Y“ ‘‹‘'¼ Œ‹p‹5 •‘5‹ … ) †‹5 ‹
B“  %š™‹Ÿ5°‘™«‹ •‘5 ‹ F/   |%‹ ‘| ˆ ‘  ’ Z ˆ ¿ ˆ5Á 5B' |  ˆ ‹;+1‘.Ÿ ‹
®5Š‘   †‘ ˆ  =| ˆ ‘ p.   +F‹´ Y“ ˆ5Á 5 !x  / a{+F‘‘   ®Š‘
0  †‘ ˆ   ˆ ‘    .0+) ^ ˆ5’ ‹Ÿe®‘ ‘ =0  †‘ ˆ Z ˆ ,+LŒ0
Œ  ˆ Œh  ˆ 'Š| x; ŸŠȋ'' ’ Z ˆ  …F ,+ Œ‹5É”‘‹5‘¼ ‹5 •‘5‹% ’ )‘|5‹ …
b  ˆ ‹5 ‘•‘‰Z  †•  ˆ 
|B“  B‘ Œ‹Ÿ2 Á •
%+Œ
B 5ŠÄ@' À= ’ • ‹Ÿ ˆ v
Ÿ  …

²ButvÇ|y{Çu®ƒZË+

: 0 ˆ - ˆ  ˆ5’ ‹Ÿ % ‹2 ’  ’‘ˆ  !'
 ‰ Y“ ‘‹5‘'¼ ®Œ‹° ˆ5Á 5Œ‹i
Œ‹ŸÈZ ˆ ŸŸ  ’ ‘ q ‹5 ‹‘‹5‘¼ Œ‹ .  /Oxyxyx&/2. 
 
+)  ˆ ‹
^• ‹´^Œ‹‹5 ‹Ÿ‘‹5‘'¼ Œ‹(.   ˆ Še; †‹‹5Č0®‘ ‘ 

‚<

´  †‘ ˆ Z ˆ  …


à ‘}+) 0 ˆ Š
Z ˆ +
. pŒ‹ŸÍ ˆ  ’ Œ‹ ‹ •‘5‹‹¼‘ ’ x‘•‘|=Œ‹
 ˆ5Á 5Œ‹É|UZ ˆ Ÿ‘‘  . …
D ·  ‹%5 hL“¼; ˆ ¿Œ •‘h ”‘¿ ’ h‹5 Ÿ ” Y“ š™ ˆ ‘Œa'

—†œ

 Œ ‹5Ä ˆ ®   ˆ Š^Œ  ‘° ° Œ‹°‹5 •‘5‹°‹5 |i Œ‹ ; Z<;   Œ‹;+ ‘i Œ‹
 ˆ5Á 5 Œ‹ ˆ ‘ ˆ5’ ‹5‘5‘| Œ‹0 ‹ ˆ Œ‘5‹ … p‘™QŒ Z†Œ‹ ˆ ) †‹5‘0  Á •q }+N Q)‘
5 >Ÿ  ˆ ‹1B“ ‘Œ ‹5 ’‘ˆ  ˆ B“  …  Y“ Œ‹ŸP·Z¼ h%Œ ‘  ˆ %‹@Œ2· ‹6+  Y“ Œ‹ŸP5 >Ÿ  ˆ ‹
‹' Y“ ˆ  … | ‹Ÿ}+L P‹·“†
¼‘ L“   ˆ Š•‹Ÿ  ? ‹‹ ˆ5Á 5Œ‹'' Ÿ Œ‹5) …
 ˆ5’ Z ˆ Ÿ
  ˆ ‹‰ Œ‹ ˆ5Á 5Œ‹‘ „·Ê  ˆ )Œ‹‹5‘   ÀȝŒ Z†|N ˆ ; †‹'   e‘®Œ‹ŸŸ  … C1‹5
Z<; jZOY< R;j<


+· HŸŠ ’ ˆ đ•H‘³Œ‹5‹5‹N ‹5‹5 ˆ @P Y“  Œ‹UŒ Z†Œ‹e) ˆ •‘NH ’  ˆ  ˆ 


¿‘ ‘ 
´  †‘ ˆ Z ˆ ,+ZŒBŒ‹Ÿ2¼‘”Œ
´ ‹E   ‹ …

C ˆ - ‹%5‘Z|‰  ˆ5’ ‹Ÿ h  ’ +ZZ ˆ2ˆ5’ ‘ ˆŸˆ ‘Œ'‹Ÿ ˆ q '
CU  ˆ q~; Z,+  ˆ5’ ‹Ÿ ÉŒ‹5 ’ ‘%;  ˆ 5   …

s

x 

 qtsjZ´”‘Ÿ ˆE ™ ’†… ; †‹5 ‹%  ˆ5’ ‹Ÿ  ’ ¼ B)  ˆ 5  …

u<

CU  ˆ < + ¿‹5 
|5‘Z|‰´Z ˆ ŸŸ  ´ Y“ ‘‹‘'¼ 'Œ‹h ˆ5Á 5Œ‹%


‘ q h=Z
‹5 ‹‘‹5‘¼ Œ‹*.  O/ xyxyx&/2.  …
 p ˆ Š*i 1PZ ˆ . 1B“ Œ‹Ÿ1Z ‹P¼Z ˆ Ÿ5É  ˆ ‹;+L“ ˆ ȋ1 Y“ š™ ˆ ‘Œ2 ˆ5’  ’ ‘;+
 e •‘h ‘ x‘ 0
 †‘ ˆ Z ˆ ,+;‘É À`Œ ‘ ‹Ÿ ®Œ‹Ÿ‰ ‹5‹5 ˆ5’  …
®‹Ÿ; †‹' '
i  Œ‹50¼Z ˆ Ÿ5,+@‘• «‹È ¿`Z ˆ ŸŸ \i Y“ ‘‹5‘'¼ ¿Œ‹ ‹5 •‘5‹‘«‘™
‹ ‹‘‹5‘'¼ Œ‹6+  ’ ‹  ‘ q+†5‘ ‹@B“‘h ˆ5Á 5h.   ˆ ‘ h‘™‹ •‘5‹@  
‘™ ‹5 •‘5‹   …;½ “ ®0ŒŒ‹´‘™ ‘‹5‘¼ Œ‹;+)‹5 ‹ q+)Œ |Ÿ‘
 ‹´ < ‹5 •‘5‹
? .• ‹  ( )+1‘•5ŠZ ” ˆ5Á 5 ˆ ‘ À`‘™ °ŒŒ‹‹5 •‘5‹Z ˆ Ÿ‘|È   Y“  Œ‹
‘‹‘'¼ Œ‹ .  O/ xyxyx&/2. ;…½ “ Š| x) ŸŠȋ5É ˆ5’ ‘ ˆŸˆ ‘Œ‰ ‹5‹5 ˆ É  ˆ ‹@% Y“ h%ŒŒ‹PZ ˆ ŸŒ‹H
° ˆ Š|L) †‹5‹5Č´¿‘ ‘ 
´  †‘ ˆ Z ˆ  …;˜ S 5ŠÄ@
À• ’ • ‹Ÿ ˆ Ÿ  …

²ButvÇ|y{Çu®ƒ 4
½ = ‹2)‘Ÿ5k† ‘ ˆ kq Œ‹Ÿrq ; ; …
˜ ‹Ÿ ˆ ‹5 ‹H 2 ˆ ŠAi  |@ Œ‹H‹5 •‘5‹ ‹5 |H Œ‹ ’ ŸÀ|5‹6+‘™ ’  ˆ ‘  ’ ‹HZ ˆ 
 ˆ5Á 5‹5)‘%‹‘ ‘•‘|ɋ·“  ‹2 %‘° À•‘  ‘ ˆ  5‹2 ’  ˆ ‘-  ’  … | ‹Ÿ}+5ŠZ 
’  ˆ ‘-2Œ ˆŸˆ Œ‹5;   = ˆ À† H,+‘  •Ÿ ˆ ¼ppŒ  ‘ ˆ <ƒ ’‘ˆ ‘5 … C1‹5
)  ˆ ‘™•ŸÄ ˆ Œ‹p‘ Œ |Œ‹6+| FB“ Z ‹ ‘™ ’ ŸÀ|5‹2F |p‘ Œ‹H•‘  ‘ ˆ Œ‹  5Œ‹
‹ŒŒ‹'‘™ ŸÄ ˆ Œ‹  À  ‹;+BŠZ |È ˆ5Á 5  ˆ Š`Z ˆ Ÿ‘|   ‘
 ‹‘ @Œ‹
 ˆ À† Œ‹h ‹5‹5 x‘ ’ ‹;+‘pŒ‹5p Œ  ˆ  ’ 'L“ ‹5‘ ´Œ  ‘ ˆ·… h ‹6+‹5|´‘™ÈŸÄ ˆ Œ‹
‘ Œ Œ‹ P ^È5 >  ˆ ‹ ‘Q‹5‘ ’ ŸÀ|0|@¿‘^ Œ‹•‘  ‘ ˆ Œ‹
 5Œ‹‹
ŒŒ‹
‘™ ŸÄ ˆ Œ‹;+1‘™Q ˆ À† Œ‹•  x5Š ˆ ŸŒ‹• |5 >Ÿ  ˆ ‹0.‘0«‹5‘ 2‹5 •‘‘
Œ  …Z ‹%Œ
B‹Ÿ;+ # ˆ À† Œ‹ %
Œ‹Ÿ%‘  ‚ ’ )  ˆ # ˆ À† Œ‹%•  x5Š ˆ ŸŒ‹ % …
à 2‹Ÿ ••  ˆ ‹ ˆ ‘ ˆ | ˆ |i‹Ÿp|Z ˆ i ˆ À† Œ‹ |.‹5 •‘   Œ \  ˆ ‹
5 ‹% Œ‹2 ˆ À† Œ‹h ɝŒ‹5 •‘   Z¡  ƒ)‘;+‹É · ‹ÉŒ | ˆ  ˆ ,+Z”‘đ• ˆ À† 
@ ˆ @) †‹5‹5Č ˆ `‹5 •‘pŒ 'i<ƒ ’‘ˆ ‘|2Œ‰5ŠZ ‘°Œ‹@Z ˆ É ˆ À† Œ‹6+|Œ‰|-Y
Zi Á •
°)‘° ˆ ”
‹ •‘5‹2;  ˆ ° ˆ À†  ………
  q s ; |  ˆ ‹;+¿ ˆ À† ' ˆ ¼ ˆ  ˆ  ; †‹‹5Č´¿‹5 •‘ɝŒ  Œ¿• ‹
  ˆ Œ‹' ˆ À† Œ‹ …U½ • ˆ ‘)Œ‹'Ÿ ˆ  ˆ ‹ ‹5‹5 ˆ B“ QŒ‹'‹5 •‘5‹0 Œ‹Ÿ'Œ ' 
 ˆ ‹h ˆ Œ‹% ˆ À† Œ‹;+‘h 
|´ Y“  Œ‹Ÿ%‹2 À‹ŸŸZŸ   ’  ˆ 5
‘³Œ‹5‹Ÿ‹ …
à N‹Ÿ •‰‘‹55 ˆ - ˆ  š™‘ 0‹ŸŸZŸ    ŸÄ ˆ Œ‹ ? ‘‰ ,+;)  ˆ q-% 
’ | ‘•‘5))  ˆ À ‘   PB“ iZ ‹' ’ ŸÀ|=|@ ’  ’ •‘  ‘ ˆ ‹'5ŠZ ‘`Œ‹
ŸÄ ˆ Œ‹ … CN ˆ š™‘  ?  ˆ ³™i À0•‘  ‘ ˆ ' 56)'

—†Å
 ¹  ²  £
r   

ƒ   

¸   

Æ   

Ë   

   

(   

²ButvÇ|y{Çu®ƒ(

: 0 ˆ - ˆ  ˆ5’ ‹Ÿ %Z ˆ%ˆ5’ ‘ ˆŸˆ ‘Œ‹Ÿ ˆ  '¼ ˆ %q^
‹5 •‘5‹ …


 =‘ À ˆ ‘•‘|  xi { 
0)  ˆ q  …

x  q s   ™ ’†…
^‹Ÿ) †‹5 ÀŒ ‘ ‹5  ’ ¼ =)  ˆ 5 ‰ ˆ Š=‹Ÿ =‘‰ ÀZ ˆ 
 q ‹ •‘5‹ …
x h  ˆ ‹*i +¿ ˆ Š
‹5 
‘2 ÀZ ˆ ' q hZ‹ •‘5‹ …
 “ ˆ ȋp Y“ š™ ˆ ‘Œ Ã5à  – +-i ) †‹5‹5ȉ°‹5 •‘.m´‘ ˆ5’ ´ ’ Z ‹5‹%Z ‹ œ… x i ' 
 ˆ ŠpP‘‹Ÿ ˆ #.J‘N Œ‹P ˆ5Á 5Œ‹1‹‚“ ˆ ) ˆ | …
‹5p p ˆ ‘ ˆ5’ ‹5‘Ÿ 
 ÀZ ˆ 'wi +|L ’ ‘•‘É ˆ ‘ ˆ5’ ‹5‘Ÿ   ÀZ ˆ 'w i '…
à NŒ‹Ÿh‘ À ˆ | i ' Œ‹Ÿh‹Ÿ  ÀZ ˆ 0‘É) †‹5‹5Č'š™ ‘5‘•‘| q ‹5 •‘5‹ …Z “ ˆ ȋ‰ Y“ Š| 
) ŸŠȋ5 ˆ5’ ‘ ˆŸˆ ‘Œ,+F °;‘hŒ ‹Ÿ ’‘ˆ  ˆ  œ Œ  ˆ Ÿ  w i '…
=;“  ˆ Œ‹Ÿ5°”Œ  ‘ ˆ ‹Ÿ) ¼ ¿)  ˆ .n‹Œ‘Ÿ5 œ Œ  ˆ Ÿ + ‹‹ ˆ ‘•‘Ÿ ˆ ”‘
Œ‹ŸŸ ° Y%´ 
Œ  ˆ À'  ˆ ‹É ¼5‘ ‹5 2 ˆ  ˆ ,+Z     …
L + 0  xz. 
{ ; %‘N5ŠZ ‘=Œ‹U‹5 •‘5‹ 1  O/ xyxyx&/21 | ‹ |1 Ÿ Œ‘5‹NŒ‹5SŒ  ˆ  ’

;Œ= ÀiŒ  ‘ ˆ   O/ xyxyx&/  … ‹; ˆ 5== ’  ’‘ˆ   ’ +B ^)‘‹Ÿ) †‹5 ˆ |0 ÀÈ ’‘ˆ Ÿ 
 ’  ’ Š ‹Ÿ`ȋ5 ˆ 5  ˆ Œ‹Ÿ)Œ‘5 ˆ À¿ ÀZ ˆ  ’ 3i +U …  …  … |`‹'  ˆ  ˆ ‘ ˆ5’ ‹5‘Ÿ 
 ÀZ ˆ H!i +· Œ‹N ˆ5Á 5Œ‹ xz.0/21 {)‹5 ˆ ‘Œ  ˆ ‘|P‹B H‹5‘‹B ˆ    ’  ˆ H;  ˆ }  ˆ À‹5‹ … 
à  ’‘ˆ Œ‹5‹5 ‹Y ‹1™Œ  ‘ ˆ ‹   ‘  » 5 ‹  Y“ ‘‹5‘'¼ pŒ‹e‹5 •‘5‹   Œ  ‘ ˆ ‹
    i;  ˆ Œ‹|‘ ‹= %š™‹Ÿ5i 5Š‘ ˆ ‘ À|   ‘a1  ? «Œ  ˆ ‹a1  )‘•Œ ‹ŸŸŸ ’
?
‘•‘|1H‹5 •‘5‹UHŒ  ‘ ˆ ‹  e   Œ‘ À ˆ @‘  HŒ ‹5 ’‘ˆ  ˆ P ÀɝŒ ; †‹5
Œ š™0Œ |5‘Z|(1  ‹‰ ' ˆ Š´ |‰ Œ‹‰‹5 •‘5‹´‹5 |‰ Œ‹É) |5‹´Œ  ‘ ˆ ‹
    6{ …
†1  q+   ˆ ‹i  ’ 5ŠZ Œ‹ÈŒ  ‘ ˆ ‹   ‘  ”‹5 ˆ 5 ‹` Œ‹) |5‹i ~+H‘ȋŸ ˆ
‘™ ‹5‘ ‹ …U˜ •,+8x‹  }x‹
Œ‹ ˆ Œ‹0Œ  ‘ ˆ ‹;+   ‘   ‘Œ‹ ˆ ¾ Œ‹'‹Ÿ x ’  ˆ Œ‹'‘
B“‘  ˆ ‹5 •‘h'Œ  ‘ ˆ  h   B“ Œ‹5h Ÿ Œ‘   ‹5 •‘‰ q+Z  -·
Œ  ˆ ÀÉi '‹Ÿ ¼5‘Œ‹Ÿ1‘Œ ˆ 2 œ Œ  ˆ Ÿ ` i ' …  ‹1Œ‘Ÿ5EÍ ‹;+>1  ‘„1 ‹ |
´Œ  ‘ ˆ  +Œ|B ¼ ’‘ˆ5’ ÀŒ  ‘ ˆ  %‘p ‹2) ˆ •‘p´ Y“Ÿ ˆ ¼ ˆ  .   ¿L“ ¼5‘ ˆ
 œ Œ  ˆ Ÿ  (i …
 1  q+  ˆ ‹S š™‹Ÿ5P05Š‘…1  /   / "  /  (>/ " (O/ xyxyx&/  / "  /21 H |N Œ‹U‹ •‘5‹N‹ |
 5 ˆ ZŸ&‘•‘2Œ  ˆ  ’ ‹@Z ˆ   ‘  …|¡ i  !. +| ••‘ ‘ ’ x‘Œ%`‘ ‘ %Ai …¡ ;+
‹|ki Œ‹Ÿp ÀZ ˆ ,+Œ‘ x‘ ´ ’ 5 ˆ5’ † °¼) ˆ  ’ |LŒ Ÿ‘|É Y“ °Œ‹p) |5‹ 1 (
a1  +†‹5 ‹!1 ( + ‹@Z ‹1 Y“ ˆ  …|» 5 ‹PB“  ·‹5%2‹ Œ  ˆ Ÿ + 1 ( B“Z ˆ Ÿ‘| Z ‹
`‘ x‘ h‘`Œ‹ŸŸ  … Ce ˆ ‹Ÿ5,+|Š ) ŸŠ ’ Ÿ|‰5Š‘iŒ ‹ŸŸŸ ’ |‘•‘ h‹5 •‘5‹
ɝŒ  ‘ ˆ ‹  (2    ‘ ˆ ‘ À|  1 (2‘#1  +´ ’ ŒŒ‹5‹ ˆ ‘•‘@%%‹Œ‹@ ˆ5Á 5Œ‹@ ˆ  ˆ ‹%
 ˆ5Á 5p‘ ‘ ,+v‘Œ | ˆ  ‘Ÿ ÈŒp ÀÉ ÀZ ˆ  ’ Ki … Ce ˆ ‹55,+v=5‘ 5Š‘B“ š™‹Ÿ5HZ ‹ …
|  ˆ ‹;+‘ ˆ ‹5 Z|  Z ˆ Ÿ ˆ k 1 (‰‘2Z ˆpˆ ; ˆ É™iŒ  ‘ ˆ ‹  (‰‘   + i;‘ ’ ŠZ ˆ
Œ‹hŒ  ‘ ˆ ‹  ( ‘   ‹5‘   
 ˆ ‘)'Ÿ ‹ ’ ‹h 
 ˆ ‘ ˆ · ‹É)  ˆ  ‰‘  …F˜ ‘ À• ¼)Ä ˆ 
À=Œ  ‘ ˆ  (+Z|´ Y“ ¿)‘2|5‘Z´Ÿ ˆ ¼ ˆ  .0+ZŒ|S ŠÄ@
À ’ • ‹Ÿ ˆ Ÿ  …

—Â
²ButvÇ|y{Çu®ƒ 4
˜ ‘ À ˆ @x‘|   ˆ - ˆ B“  B“ š™x‹Ÿ5`Z ‹=`‘ ‘ `ŠZ 5 ‘ ‹Ÿ ˆ ` ˆ ŠÈ |= Œ‹
‹ •‘5‹É‹5 É Œ‹É· ‹Œ‹;+‘™ ’ | ˆ ‘  ’ Œ‹É‹ŸU‘ɋ5‘ ‘•‘|´‹·“  SŒ‹5%; †‹‹Ÿ¼    ·†  ˆ 
Z ‹‹5 ˆ  ˆ Œ‘5‘•‘|´´ Y“   Y“ ˆ  …
˜  ˆ  ‹È 5 ˆ ZŸ&‘•‘¿‘ ¼ À”‘`  ˆ 5ŠZ ‘ Œ‹`· ‹Œ‹° Y“ ’ Š| ˆ + Œ •
‹5 ˆ  ’ Š| ˆ ‘ À ‹5‹Ÿ,+ZŒ'|8Í  ˆ ‰'Œ  ˆ Ÿ ®” ˆ Š,+;|NŒ‹Ÿ%0Œ  ˆ Ÿ 
 ˆ  ˆ · ˆ ´• -‘•‘hL“ ”·;   ˆ ¼   Z ‹5‹5 ˆ L“ 
· ‹5´¼ ÀŠ  · ‹5´  ˆ 
‘ ˆ5’ ‘ ˆ x‘•‘| …
½ Œ‹ q  Y“ ’ 5Š ˆ ‹5 ˆ5’ Z ˆ Ÿ‹‹5‘|‘®|Z ˆ  †Œ‹%  q · ‹5Œ‹´5ŠZ ‘ ';‘™” †Œ‹
#¬š™ ’‘ˆ ‘ ˆ Œ‹ %2‘P‘™0 †Œ‹ #| ’‘ˆ ‘ ˆ Œ‹ % …
hˆ + p• -‘•‘|H•·;   ˆ ; †‹52pp5 5

· ‹ ‹5Ÿ ’ 2‹Ÿ ˆ p †2š™x ’‘ˆ ‘ ˆ 2 Z ‹5‹ ‹5 ˆ 2· ‹ ‹5Ÿ ’ 2‹Ÿ ˆ p †p| ’‘ˆ ‘ ˆ  … Ce ˆ
‹55,+B‹ŸPQ‘ ‘ ŠZ 5 ‘«š™x‹55,+B5ŠZ · ‹5š™x ’‘ˆ ‘ ˆ `Œ‹Ÿ
 ˆ Œ‘5‘•‘|‹Ÿ&xL“ 
· ‹=| ’‘ˆ ‘ ˆ  …S¡ ;+LŒ • 1 °'· ‹5Œ‹ ’‘ˆ ‘ ˆ Œ‹'· ‹5Œ‹
š™ ’‘ˆ ‘ ˆ Œ‹
‹Ÿ ˆ
Y“ ’ Š| ˆ + †“ Œ‹ŸÈ ”° ”‘ ‘ ¿Œ‹ŸqÍ ˆ  ’ Z ˆ   5 ˆ ZŒ ”· ‹5Œ‹| ’‘ˆ ‘ ˆ Œ‹i‘
š™ ’‘ˆ ‘ ˆ Œ‹ …  ‹  ˆ ‹;+U À Ž Œ  ˆ Ÿ «  ˆ Ši‘| ˆ °• Œ‹=· ‹5Œ‹0š™x ’‘ˆ ‘ ˆ Œ‹‹ |
5 5Œ‹´0 À Á •0Œ  ‘ ˆ ? ‘´| Œ‹´· ‹5Œ‹‰| ’‘ˆ ‘ ˆ Œ‹
‹ |´5 5Œ‹´' Y“ ˆ Œ  ‘ ˆ )+)Œ
;B“ Œ‹ŸH< Œ‹Ÿ5‘•‘2Z ‹H ‰· ‹ …x ,+|°‘ ‘ ɊZ 5 ‘¿i·†  ˆ Œ‹ŸH) †‹5‹Ÿ¼  …

²ButvÇ|y{Çu®ƒ 4

0Œ ‹Ÿ ˆ e0 ˆ ŠEi¿ |e Œ‹e‹5 •‘5‹e‹5 U Œ‹ Ž†Êh’ |)Œ‹;+‘ ˆ ‘ Àe‘™0‹5 •‘5‹
Z ˆ h ˆ5Á 5 ˆ %‹Ÿ‘P‹5‘ ‘•‘H‹·“  ‹1Œ ˆŸˆ Œ‹5; ‘  ‘™ ’ |)Œ‹P‹5p‹5 |  ƒ ˆ  ’ Œ‹
H ˆ ‘ ˆ Ÿ  ˆ +v‘eZ ˆ p ˆ5Á 5  ˆ 5 )  ˆ p‹5ŒŒ †Ÿ  ˆ·… | ‹Ÿ}+5ŠZ 2‹5 •‘NŒ‹5N Y“ š™ ˆ5’ 
 ’ L“ š™ ‘5‘•‘ph ˆ5Á 5 ˆ h‘@‰ ˆ5Á 5* ˆ 5,+‘H h
;  ˆ ‘ ˆ5’hŽ… ‹H5ŠZ 
Œ ) †‹5%Œ š™,+|‘•Z ˆ | L“ •‹5 •‘H ˆ ¼ ˆ  ˆ ,+|‘P‘`‹Ÿ& H 5 ˆ ZŸ&‘•‘| 
 ˆ5Á 5 ˆ ‘
È ˆ5Á 5… ˆ 5,+S  Z ˆ Œ  ˆ ‹ŸP ‘ x‘ •‘ ’‘ˆ ‘+U|HŒ‹Ÿ ’ | ‘•‘|0
‘ x ´ŠZ 5 ‘ …Z½ 0¼Œ  ˆ Ÿ   ‹5‹Ÿ ˆ Œ‘ ‘ Œ‹Ÿ2  †‘ ˆ Z ˆ  …¡ ¿Š ‹Ÿ‹5‹
ȋ5 •‘p‹Ÿ ˆ ‘™‹ 5ŠZ ‘°‰ŒŒ‹H‘ ‘ Œ‹;+ È ¼Ÿ‘2‹5 „·Ê ‹5 •‘5‹@ ’ )‘|5‹6+
 ˆ ‘ ˆ5’ ‹5‘5‘|‰¼‘ „·Ê=’ |)Œ‹% É‘™¿|‘ Œ Œ‹%‹5‹ |%Z ‹%‘Œ ˆ 0 ƒ ˆ  ’ Œ‹ …

²ButvÇ|y{Çu ¸Zµ4
˜ ‹Ÿ ’‘ˆ ‹È ” ˆ Š  J ˆ ‘ ’  i Œ‹`‹5 •‘5‹È‹5 ` Œ‹~  Œ‹È‘È Œ‹È ˆ5Á 5Œ‹` Œ‹
ˆ 5Œ‹ …à ‹‚“†@• ˆ Š%‹Ÿ ɝŒ  ‘ ;…†½ 2¼@ ˆ ¼ đ•ÉŒ‹ŸP hpZ ˆ ŸŸ  ˆ
Y“ ‘‹‘'¼ =Œ‹ ˆ5Á 5Œ‹‰ ‘^Š‘‹w4  /54 (>/Oxyxyx&/54 "6  +F B4 Œ‹ŸÉ®5Š‘ ‘ ’‘ˆ ‘^
 †‘ | ˆ }…
'rq Œ‹Ÿ=Z ˆ ' |  ˆ ‹;+e5ŠZ °‹5 •‘•« ˆ Š¿Œ‹Ÿi‘ ˆ5’ Z ˆ·…1½ iŸŠ ’ ˆ đ•
L“ ¡   ˆ  ‹5‹5 ˆ '  ˆ ‹h' Y“ š™x‹55‘Œ'L“  Š‘ ? ‘h Á •L“  ‘ x‘ 6)%‘ ’‘ˆ ‘ ‹5 ˆ ;…

_ 76 
)‘1 %‹Ÿ ˆ   ˆ Œ‹ (
`
 ˆ5Á 5Œ‹1
  Y>`Œ    ˆ • ˆ Z ˆ Œ  ˆ ‹P  †‘ ˆ
_ "6 
(
`
…xà Z‹Ÿ •2  ˆ ‹H #¬ ’ Œ ; ˆ %‰Œ%Z ˆ Œ  ˆ ‹p‘‰‹ŸŒŒŒ‹5‹Ÿ ÈÉZ ˆ Œ  ˆ ‹pÉ  †‘ ˆ ‹
Z>/ < O / xyxyx&z/ q3mZ,+)  ˆ ¼5‘ ˆ À=Œ ‘ ‹Ÿ   ’ ‹Ÿ ˆ5’ 
‹%Œ'· ‹ …
@0q®Œ‹ŸpZ ˆ ' ˜ ‘Ÿ5A ‹;+5 ‹p Œ‹2‹5 •‘5‹2‹5 p´‘ ˆ5’ ‹pZ ˆ ‹;+‘p  ˆ ‹5 ‘•‘
‘³Œ‹‹Ÿ‹2Œ  x‘|‰ ‹ …
» 5 ‹   /  (>O/ xyxyx&/  Œ‹h‹5 •‘5‹ …

CU  ˆ Y } hZ  q~mZ,+ ”Œ ‹ŸÄ ˆ 
Œ‹2Z ˆ Œ  ˆ ‹w'

    m    ( m   m     m  ( m   (rm  m xyxyx!m     m  ( m   (

¡ ;+)  ˆ Y  } muZ  q~mZ ,+ ¿Œ ‹ŸÄ ˆ ' Œ‹2Z ˆ Œ  ˆ ‹w'

'   6  m   m   m  ( m   6  m  rmxyxyx!m   6  m  ( 6  m  

—† 

  ’‘ˆ     ‘ ‘•‘É,+;  ˆ 5  } +  1Œ‹Ÿ2´  †‘ ˆ ; } h Z,+‘%| ' Œ‹Ÿ%
 †‘ ˆ ' ; } m < …
 % ‹;+†)  ˆ} ™ ’ +† 2Z ˆ Œ  ˆ ‹ ? ˆ Œ‹Ÿ … ' {eB“ Ÿ ‹5É|%Œ‹  ˆ5Á 5Œ‹H  •• ‹H
Œ‹Éš™x ˆ5’  ’ ‹hŒ‹Ÿ         ( ? ˆ Œ‹Ÿ …   6     ) ‘%´Z ‹5‹´Z ˆ ‘”‹5 •‘É 
Y“ ŒŒ‹5p‹5 ’‘ˆ ‘ ˆ  < } ? ˆ Œ‹Ÿ … < } mZ6) ˜ ‘ À ‹5‹Ÿ ˆ ´|´‹Ÿ } ;     ˆ ‹p Œ‹pZ ˆ Œ  ˆ ‹ ‘
L“ ´Z ˆ ;+‹h‹5‹Ÿ ' ‘  ' +‹ŸB e‘ ' ‹5 |2¼‘¿‹zŸ |5‹ ? ‘°5 ˆ •L“ ˆ5Á 5Œ‹5) …
 q ; '
¿Œ ‹Ÿ ˆ ‰|´ Œ‹%Z ˆ Œ  ˆ ‹
/  ' / (>/ ' /Oxyxyx&/  6  …
 q ; %h < '
°'Œ ‹ŸÄ ˆ 
´ Œ‹%Z ˆ Œ  ˆ ‹
/  ' /(>/ ' /Oxyxyx&/ 6  /  ' …
 ‹@ Œ‹P‘™•· ‹;+Œ‘ ÀA  ˆ @Œ‹@Z ˆ Œ  ˆ ‹H‘™  ‘™•‹zŸ |5‹;+‘H2  †‘ ˆ ‹H‘™
 ‘™'‹ŸŸ‘5Œ‹e‹ v Z>/ < O/ xyxyx&z/ q m Z{ …† H ‹;+  Z ˆ   ‘  "6  +5ŠZ  ‹5 •‘N ˆ Š

Z ˆ  š™ ‘5‘•‘|=~Í ‹0Œ •Èš™x ˆ5’  ’ L“ .ŒŒ‹0Z ˆ Œ  ˆ ‹‘;+ ’ ‘|Ÿ‘  ‘•‘|;+
Œ •‰‹5 •‘ |5 ˆ  ’ À ˆ ‹ L“ ˆ Œ‹ Z ˆ Œ  ˆ ‹ … Ce ˆ Œ | ˆ ,+x    "6  B“Z ˆ   
‹e•‘ Œ 2 ŒŒ‹1Š‘‹ … | ‹Ÿ}+ p ˆ Ši ¼5‘|•‘ ’ Z|@5 5Œ‹1 Œ‹P ˆ5Á 5Œ‹
 ’  Ÿ ‹ ’ Œ‹0‹
Œ‹'Z ˆ Œ  ˆ ‹=‘³Œ‹5‹Ÿ‹'Œ‹5'Œ š™ ? Q)‘
Z ‹5‹5 ˆ L“  ‹5 •‘  
 ˆ (À  )  |h Œ‹%‹5 •‘5‹É‹ |2|5‘Z´5 ‹%
‘ ˆ5’ ‹2Z ˆ ‹;+Z‹  ‘  "6  …½ 
ŸŠ ’ ˆ đ•L“ ¡   ˆ ) ˆ •‘'  ˆ ‹
•Œ ‹Ÿ ’‘ˆ  ˆ  5Š‘Q‘ ’‘ˆ ‘ ‹Ÿ ˆ i +L|= Y“ Q ’ Œ )
‘‹55'‘”Z ˆ Œ  ˆ ‹‰
 †‘ ˆ ‹  ’ Z5Œ‹ ? Œ ˆ Œ‹‰  †‘ ˆ ‹´ |‰ ’  ’  ’ Ÿ ‹ ’ Œ‹hZ ˆ
Œ‹%Z ˆ Œ  ˆ ‹É‘³Œ‹5‹Ÿ‹)H;  ˆ Œ ‘  ˆ '‹ÉŒ· ‹ …

²ButvÇ|y{Çu ¸Lr
˜ ‹Ÿ ’‘ˆ ‹ = ˆ Š |' Œ‹‹5 •‘5‹
‹5 ‰ Œ‹'5Š@†  ˆ ‹;+L‘™ ’  ˆ ‘  ’ ‹Z ˆ 
 ˆ5Á 5ɋŸZ‘@‹5‘ ‘•‘H‹·“  ‹P%‹5 |„felÉ‘‘‹ ?  ˆ  h|B“  ‹P‹ | ‹) …à ‹‚“†H 
' ˆ - ˆ 
Œ' ˆ ŠŒ‹ŸɊZ 5 ‘ …L˜ ‘ À ˆ @‘   ˆ - ˆ 
ŸŠ ’ ˆ đ•0‹5& ;+Z=
   ˆ   ? „·¢†œ†Ž )k'

# b J ˆ Š ‹5    s ‹5 •‘5‹;+ 5ŠZ ‘D ‘ ˆ5’ ‹5 ’‘ˆ ‘ ˆ  ’ |  ( +HŒ‹Ÿ
ŠZ 5 ‘ … %

˜ ‹Ÿ ’‘ˆ ‹%


‹5; †‹ŸŸ ” ˆ ¼ ˆ  ˆ Œ‹%5Š@†  ˆ ‹ '

.A1€xyxyxyx .

 .0/21  ’ ‹Ÿ†‘´Œ‹´Š@   ˆ ‹´<ƒ ’‘ˆ ‘|5‹;+;‹‰ Œ‹h‘™ . ˆ ‘ ˆ5’ ‹5‘|5‘ 
 Á •
   ? 
Œ‹5É5  ˆ L“ ¼  ˆ 6) …
 ‹HŒ%F‹Ÿ;+ `Œ ‹Ÿ ’‘ˆ  ˆ 
À
‹Ÿ; †‹5Ÿ `Œ •%h‹Ÿ5%2) ˆ ‹5 Œ‹;+ ˆ   ’ 
´|Š   ˆ 5 …
B‘™  ‹5‹%‹5 |h‘‘‹;+ ” ˆ †“ Œ‹Ÿ2arh@-l5€m> …

;“  BB“ i‘
5‘‹Ÿ +Z†“ Œ‹Ÿ   …
;“  Nš™‹Ÿ55‘‹5 +FZ ˆ ‹Ÿ x ’  ˆ ‘ ˆ ‘ À ˆ ,+L  )‘h5 >Ÿ  ˆ ‹‹Ÿ) †‹5 ˆ  . ‘A1
‹ |É‘‘‹ …
˜ • 1  # ‹ q m Z ‘‘‹6+@ \‘  ’ `F1  #• ‹€q ‹ ?  ‹
³Y Á •6) …
; †‹5 ‹e@‹U ÀÉ‹5; †‹ŸŸ =‘³Œ‹5‹Ÿ‹N‘1 ˆ ȋ 1 +‚ 0H ˆ -5‹e‘™( ‹Ÿ‹
. ' ‘E1 ' ? ‹2Œ‘% ˆ  ˆ 6) L‹5 ‘|É‹ ˆ Œ‹5;Œ‘Ÿ&‘•‘É.m‘%1 |  ˆ ‹;+Œ •´

5‘  0‹Ÿ) †‹ŸŸ  B“ ˆŸˆ &Z ‹h   ‹´; k13+F  ‘  ’ |B“ ‘  B“Z ˆ   Ÿ‹;+;‘
 `|B“   ˆ 5`5ŠZ È q.  ‹ ˆ - ˆ  ‘‘ q1 …U ,+B • ¼ ˆ 
L“ ‘‘‹ 1 Œ‹Ÿ• • ‹ ’ | %« '¼ ˆ °L“‹ . +NŒiH 5qtmjZ s q …
˜  ˆ  ‘Ÿ  …
CN ˆ ‹Ÿ5,+ °)‘p ˆ - ˆ ‘™~ ‹5‹. ' ‘1 ' ? ‹2Œ‘% ˆ  ˆ 6) B‹5 |É‹ ˆ Œ‹Ÿ)Œš
Ÿ&‘•‘hk. ‘%(1 …

—†¢
½ ‹Ÿ) †‹ŸŸ ¿Œ‹ŸÉ  ˆ ‹h´ À Í ˆ •…'

.A1€xyxyx . ' 1 ' xyxyx .

 ‹pŒŒ‹2Œ Ÿ ‹;+ i)‘ ’  ˆ À5‘‹Ÿ °‘ ˆ k.9‘E1 ‘ ˆ ‘  ˆ ‹% Y“ ˆ  ˆ ´Œ‹
) ˆ ‹5 Œ‹%‘ ˆ w1 ‘*. ' '

.pxz1 xyxyx . ' W{ 1 ' xyxyx 


. .pxz. ' xyxyx 1a{W1 ' xyxyx .

˜ ‘ À Í  ˆ •i -‘  ”‹Ÿ; †‹5Ÿ  ‹Œ .• ‹i5‘‹Ÿ #i• ‹•


‹È ÀQ ˆ5’  ’ ‘|5,+p‹5 ‘ À ˆ ‘•‘|° ÀQ•   ·Ÿ D  ˆ5’ ”Z ‹i  -‘  ®5‘‹Ÿ 
? Z ˆ Œ | ˆ ,+e‘  ;‘ ’  ˆ `5‘‹Ÿ  ’ ‘|Ÿ‘  °‘ ˆ ~. ' ‘01 ' { …U¡  ˆ5’  ’ |Œ‘Ÿ5
 ˆ x ’  ˆ °|• ’ ŒŒ‹‹ ˆ  ?   ¼ ˆ   2q ‹=‘ 5 · ‹;+N‹5` ` ¼ ˆ 

ŸÀ U5‘‹Ÿ ‹%Œ‹Ÿ  ^)+Z   %‹5Z ˆ   ˆ 
5 5Œ‹2 Œ‹25‘‹5 ‹;+‘% Y“ ¼ŸŒ‘Ÿ<eŒ‹5ÉŸ5‘| …

²ButvÇ|y{Çu ¸Fƒ+
½ = ‹2 ˆ k† ‘ ˆ ) †‹5‹Ÿ¼ Œ‹Ÿrq ;j<  …
x   q¿ 2 ¼ ˆ hh‹5 •‘5‹@L“ `;  x %)  ˆ Œ‘ ZÉ5‘  ɋŸŸZŸ ÈŒ‹ŸP) †‹5‹Ÿ¼  …
˜ ‹Ÿ ’‘ˆ ‹ È ˆ Š°Œ  ‘
 | Œ‹=‹5 •‘5‹‹5  Œ‹=‹5 •‘5‹•«;  x  …
½ “ ’   ’  ‹5‹5 ˆ ”  ˆ ‹`¿Œ” ˆ Š¿¿) †‹5‹5Č¿Z ‹”‘ ‘ °•  x5Š ˆ Ÿ ? ‘}+@‘
5 ˆ •'L“ ˆ5Á 5Œ‹5) …

   ˆ - ˆ ´ h ¼ ˆ 
L“ ˆ5Á 5Œ‹%Œ  ˆ  ’ Œ‹ÉZ ˆ Œ  ‘ ˆ   ’ ´ ’ Z ‹5‹5‰Z ‹
q~m Zq'

CN ˆ Y“¼‹Ÿ ˆ  ' ) †‹5 ‹È|B“J• ‹ q  ˆ5Á 5Œ‹i‹5 ‘|iL“ ” Á • Œ  ‘ ˆ + ‹5 ‹
 ˆ 5Œ‹ …
˜ ‹Ÿ ’‘ˆ ‹É  ˆ ‹2 ´‹5 ‹Y ˆ Š
¼5‘‹%‘°´Œ ‹5 ˆ †|ɉ Œ‹% ˆ5Á 5Œ‹E ˆ 5Œ‹ …
˜ H‹5 ‹Y ˆ ŠH; †‹‹5ČH  q‹ •‘5‹N‘e'• ‹ qÈ ˆ5Á 5Œ‹ …v “ ˆ ȋU Y“ š™ ˆ ‘Œ à  „ )¬¼9)+‚ ) †‹5‹5Č
  ˆ ‹%”‘ x‘ ,+SŒ‹Ÿ2¿‘ ‘ •  x5Š ˆ Ÿ
;…F˜ _ | ˆ  ‘Ÿ  …
"6 
˜ • 9Ÿ ‹5
  Œ  ‘ ˆ ‹ ‘¿|B“  ' «‘95  (
`
 ˆ5Á 5Œ‹;+% m‘9 ’ ”
_
"6
(

`

x q~mZ6{+‘% 
k
   q <  …

à L ˆ Œ‹Ÿ5
 ‹%|B“   ˆ - ˆ |´)  ˆ q ;< i
5‘  
Œ   † ˆ Ÿ  š™‹Ÿ5a'
CN ˆ ‹5 ‘F%‹5 ‘ ’ + a†‹Ÿ) †‹5 ˆ |2 Y“ ` ˆ †  h;Œ%)  x  ˆ5’ †  ˆ ;Œ
q ; < ”‹5 •‘5‹A. ;/2. ( O / xyxyx&/2. ( +Z‹É(Í ‹h|' Y“   ˆ ‹‰‘ ’ x‘Œ' ÀŒ  ˆ Ÿ 
Œ‹h ˆ5Á 5Œ‹;+ BŒ‹Ÿ%‘ À ˆ Œ‘  š‘U‹ ˆ 0 ˆ ‹Ÿ; †‹ ¼ ;  ˆ 5 É;  x 
Œ  š™ …

QŒ  ˆ  Ài †¼ ˆ ‹ ’ a.  . ( + . (>/2. (  6  xyxyx . >+
.    À°Œ  ‘ ˆ 
… C1‹;+L)  ˆ 5 
 }
  mZ,+Z ”Œ  ˆ ' Y“ 'Œ‘Ÿ5
†¼ ˆ ‹ ’ 'Z ˆ À ˆ Ÿ 


Œ‘| ˆ    ;  x 
‘0L“†   ? †“ Œ‹Ÿ0‘HB“  HŒ‹Ÿ'Ÿ `L“ Ÿ ‹5 ˆ Q)  x  ˆ5’ †  ˆ )(À¿ À¿Œ  ‘ ˆ  …UÃ
B“ Œ‹5É  ˆ ‹%Z ‹%< ‘ 
( ’‘ˆ    ˆ |
Œ  ˆ À' ¼5‘| ‹Ÿ‹ Y2 Œ‹%Œ Ÿ ‹ ˆ Œ‹5Œ‹ …

²ButvÇ|y{Çu ¸Z¸4
˜ ‹Ÿ ’‘ˆ ‹ŠZ ‘Œ‹'Z ˆ ŸŸ ‹Œ • À ˆ ‘ ˆ5’ ‹5‘|Ÿ «L“ Q ˆ Š ÀZ ˆ ~i +
 1 Œ‹P‹5 •‘5‹P‹ |1 Œ‹1‹5 •‘5‹P)  x ,+†‘1 Œ‹P ˆ5Á 5Œ‹P‹5 |1 Œ‹1Œ¾  ’ ‹@‘1 Œ‹1À Z Œ‹
Ÿ ‹ ’ Œ‹ …và ‹‚“†NH ˆ - ˆ |B“ p• ‹e ŒŒ‹SZ ˆ ŸŸ ‹1Œ‹5U‘ ’‘ˆ ‘2‹Ÿx‘N‹5‘ ‘•‘
‹5 q®Œ‹5É&‹5¼ ´Z ˆh—…
 ; †‹5 ‹h|´ Y“  2Z ˆ ŸŸ  ‘ ’‘ˆ ‘~'
 ‹%5 ɝŒ
B‹Ÿ;+ Œ‹%À Z Œ‹h‹5 |ɍŒ‘  Œ‹%Ÿ ‹ ’ Œ‹%‹% À0Z ˆ ŸŸ  …

`‹@ɝŠZ |‰‹5 •‘ Œ‹Ÿ@É‘ ˆ5’ Z ˆ +|‘ •)‘H‹Ÿ) †‹5 ˆ r  q +‹‹@  
 LB“ ° ˆ ‘  Y ˆ  …

 †` ˆ - ˆ |B“ ‘ ’ Z —  ˆ5Á 5Œ‹´¼‘®Š ‹ŸŒ‹;+) 1Œ‹Ÿ´) †‹5‹Ÿ¼ 00 ’ Œ ) †‹5 ˆ i
‘¿‘™¿‹5 ‹ŸY ˆ ŠŒ‹É‘ ’‘ˆ ‘‹%B“· ||‰|B“ ”‹ •‘É‘¿Œ ' …

–|Ê

 Œ •‘ŒiZ ˆ=ˆ ‘ ˆ  ˆ B“ .À Z ÈZ ˆ i )   ÈŸÀ  ‘ ‘™
‹ ‹Y;  x Œ‹  ‘ (,+e‘•|È À  ˆ À† ÀŸ \  ’ À5‘•‘|ÈŒ‹= ˆ À
† ÀŸ ‹ ˆ Œ‹5;Œ‘Ÿ&Œ‹  ‘ ( …)¡  ˆ5’  ’ |Œ‘Ÿ5  ’‘ˆ Ÿ ^‹Ÿ ˆ Y“ ^=ŒŒ‹‰‘™®‹5 ‹
)  x Œ‹
‘'‹ŸP‹Ÿ5,+L    ˆ iZ ˆ  ˆ - ˆ   ˆ À† • |' Œ‹‹5 •‘5‹'‹5  ˆ ‹
‹ •‘5‹'  … ‹'; ˆ 5 ’  ’‘ˆ   ’ +U Q;‘
 `‹Ÿ; †‹ ˆ | • ˆ À† q. .  .+(
Œ‹52  Œ
À0Z ˆ ŸŸ ”ŸÀ  …
C1‹5. ‘#. (2‹5 |@5 ‹@‘™•%‘ ˆ5’ ‹PZ ˆ ‹6+† Zš™‹Ÿ5pÉÀ Z hL“ š™x ˆ5’  ’ + . (
‘2'À Z 'L“ š™ ˆ5’  ’ . B‹5 ˆ ‘Œ | ˆ ‘‰‘”¿ Á •
‹5 •‘*.   <% % q

? ‹5 + Bš™‹Ÿ5 ˆ pAY Œh ’‘ˆ ‘ ˆ '°)   ´;‹ ˆ  Z ‹pi ˆ À† 6) … | ‹Ÿ}+x 
 ˆ À† q. . . (Œ‹Ÿ'‹5‹ŸepY Œ,+L‘'  ’  Œ‹ ˆ5Á 5Œ‹' ’ 5 ˆ  ’ Œ‹
Z ˆ Œ‹
Œ¾  ’ ‹


. .  . ( …)˜ kY‹|;+;   ’ Œ ) †‹5 i ‘ ‘™”‹ ‹Y ˆ ŠŒ‹ÉB“· ||
'‹ •‘†. ‘

Œ  …U¡ ;+S)  ˆ ŠZ ‘ `ŒŒ‹'‹5 ‹Y ˆ ŠŒ‹6+UŠZ ‘ Œ‹‹5 •‘5‹Œ‹5‘Œ ˆ È•‘ ˆ5’
Z ˆ +ZŒ|S ‹5‹5 ˆ 
|B“  ‹É‹5 |%¼‘°5 ‹É Œ‹%‘™°‘ ’‘ˆ ‘‹ …
˜ •”5ŠZ ‘J°ŒŒ‹‘™.‹5 ‹Y ˆ ŠŒ‹`Œ‹Ÿ  ’ Z ˆ  ˆ ‘ ˆ5’ ‹5‘|Ÿ J ÀZ ˆ 
ɝŒ ˆŸˆ Œ‹Ÿ)   #)    Œ  š™” ˆ À† ’ +  )‘È  ˆ ‹ ˆ5’  ’ 5 ˆ Œ‘Ÿ5¿ ˆ x ’  ˆ 
‹ ’ Z ˆ5’ •‘|h‹5 ˆ 5ŠZ ‘®Œ‹%‹5 ‹Y ˆ ŠŒ‹ …Z¡ h‹5S
‹Ÿ5K‹5B“  Œ

Œ‘ À=
‹ % ‹
) †‹5‹Ÿ¼ É;  ˆ ‘°Œ‹H‹ ‹@ ˆ ŠŒ‹p‹ ‘ ’ x‘Œ ?  ŠZ | ’ ),+| À‹Ÿ¼)&‹Ÿ iL“ 

 ˆ Š Y'Z ˆ   ˆ •‘™^‹5 ‹Y ˆ ŠŒ‹'· ||0‹Ÿ ˆ ‘5‘•‘|
• ‹'   ŒŒ‹
0 = ˆ Š
 % ‹É ’‘ˆ &‘;+ZŒ
U ‹5‹5 ˆ | À0 ˆ x ’  ˆ    ˆ ) …
hˆ +‹ŸL À ’ Œ ) †‹ŸŸ ¿B“ Œ‹5% ‹
ˆ5’  ‹ ¼ ,+†“ Œ‹ŸPH Y“ B“‰ ‹P|2Œ‹1‹ ‹Y ˆ ŠŒ‹@‹5 |@‘™xY Á •Œ‹PŒ‹1 ˆ À† Œ‹ …½ 
 ¼ ˆ 
L“ ˆ5Á 5Œ‹É)B“ |hZ ‹ ’  ’’  ’ Œ‹h¿Œ  ˆ ‹%´ À0 ˆ x ’  ˆ 
Œ‹Ÿ% ´° Ÿ 
 —…S˜ •ŠZ | ’ ) ’  —  ˆ5Á 5Œ‹;+L†“ Œ‹Ÿ |0 = ¼ ˆ =ŸÀ PL“ ˆ5Á 5Œ‹
Œ‹Ÿ
³Y Á •¿ Ÿ  —…9
hˆ +ŸÀ ‘•‘;+F U i <q m ` ˆ5Á 5Œ‹ …Z , + q Œ‹Ÿ%¿ Ÿ 
 —…
  ’ ‘ ˆ x|‘•‘0'Z q ; …

: =Œ ‹5 ˆ  ˆ Z ˆ ŸŸ ”‘ ’‘ˆ ‘'Z ˆ%ˆ5’ ‘ ˆŸˆ ‘Œ'‹Ÿ ˆ …
+ ; Z,+  = ˆ Œ‘5‘•‘´ ƒB ˆ   ° ˆ À† ,+Z‘2 À•Œ ‘ ‹Ÿ ”Œ‹Ÿ% ’ À5 …
x  s Z  ™ ’†… ) †‹5 ‹B“  %š™x‹Ÿ5ÈÈZ ˆ ŸŸ #‘ ’‘ˆ ‘°)  ˆ 5  iY 
Œ  š™ …
x p  ˆ ‹H -x' h Z6{Y Œ  š™A.  . ( xyxyx .

… CN ˆ À'À Z   . ;/2. +x `&x‹5
 

0)   ‘^^)‘| q.  . (xyxyx . ‘ iY a.  . .  xyxyx . …)½ “ Š| x) ŸŠȋ5•

ˆ5’ ‘ ˆŸˆ ‘Œ• ‹‹Ÿ ˆ B“  eš™x‹Ÿ5 Z ˆ ŸŸ  ‘ ’‘ˆ ‘ iY  …
®Z ˆ ŸŸ ‘‹Ÿ5
´)‘|   Y“'Œ‹%À Z Œ‹  .  /2. S‘  . >/2.  … 

à Sš™‹Ÿ5'  ˆ ‹h ‘ ‘ 
‘ ’‘ˆ ‘®‹Ÿ ˆ ';‘ ' ˆ & ’ 
Y“ ˆ5Á 5  .  /2. +Z ˆ š™‘ 
 .  . . .  . . (.  …¡  ˆ  ŒŒ ˆ |hŒŒ‹ ‘™°‘ ‘ Œ‹ ‘:.  + iŒ ‹5 ˆ É‹Ÿ)i‘ ‘ ‘ ’ 
ˆ ‘ ‹Ÿ ˆ  -x' hZ6{Y  …

 / Œ/e&)4  N
 41(

 „
N˜‰… "  ˆ ,+ 8g‚Sm>d€}h0‡ h5l†c>dSe5f%gh5l…h@r l;h5l0e5ff €‰2e,rz€m>-l+   x …
 Ž S 0… " … Œ‹Ÿ;+5%rzd m6‡#‰;rz€zm>r}mKdSe5f%g 8ghSm>dAK,+-C ˆ ‘|ŸŒ´  …
 —  … … ˆ ·+5%rzdSm;‡#-‰rz€m>r}mKdSe5f%g 8gh2m>d*K,+  6 ˆ·…
  ˆ 5‹  ‘  + ‡=…  ‘ }+ @ h2e>d Pl€^KdSe5f%g 8gh2m>d*KP ‰2m,n*f dSh5g-h-l5€Bh0‰2m# d l;h@+
 –; »0… ‰ | · ‘
C ˆ Œ‹5‹ …
 œ  … ˜ ) ¼ÀŒ -+ … ˜‰…   -77· -+ FHG e>n†f uh5l e>9‡‰2m  %rh@dSh G e>n†f &hlK€sKd e5f9g 8ghSm>dAK,+ » ˆ ŸŠ
‰  À …
 Å S½P… ½ ;  ‹S7+  m>n ,€^ e,r}m>d€ze f d m , &hn l0e> ‡ h G hdS‰@€ l;h5l+ » ˆ ŸŠ!‰  À …
 " Sà … •Œ‹5‘+ @ dSm , uhn lw€s  m>n ,€s9e,r}m,d5€‰lwe>9‡#Kd ef%g 8ghSm>dAK,+  ‘  à |5 ˆ ‹5‘‘Œ …

–„
–|Ž
Concepts de base en arithmétique
Jean-Louis Tu

Objectifs de ce document
Ce document s'adresse à tout élève de n de collège ou début de lycée souhaitant
s'initier aux exercices d'arithmétique de type olympique. Il rassemble les connaissances
nécessaires et susantes pour pouvoir résoudre des exercices de compétition pour les
juniors. Sa lecture est indispensable à tout élève débutant en arithmétique et souhaitant
participer aux activités de l'OFM (envois d'exercices par correspondance, stages).

Les exercices de ce document sont pour l'essentiel des exercices d'apprentissage, et


permettent d'acquérir certains réexes, mais sont rarement des exercices de compétition.
Par conséquent, pour être performant en situation de concours, l'élève devra les compléter
par d'autres, par exemple le poly d'arithmétique plus volumineux disponible à l'adresse :

http://www.animath.fr/IMG/pdf/cours-arith1.pdf

Table des matières


1 Préliminaires 2
1.1 Notations . . . . . . . . . . . . . . . . . . . . . . . . . . . . . . . . . . . . 2
1.2 Principe de récurrence . . . . . . . . . . . . . . . . . . . . . . . . . . . . . 2
1.3 Mode d'emploi des exercices . . . . . . . . . . . . . . . . . . . . . . . . . . 3

2 Divisibilité 4
2.1 Le théorème de Bézout . . . . . . . . . . . . . . . . . . . . . . . . . . . . . 11
2.2 Le théorème fondamental de l'arithmétique . . . . . . . . . . . . . . . . . 14
2.3 Nombre de diviseurs d'un entier . . . . . . . . . . . . . . . . . . . . . . . . 17

3 Congruences 19
3.1 Dénition et propriétés de base . . . . . . . . . . . . . . . . . . . . . . . . 19
3.2 Critéres de divisibilité . . . . . . . . . . . . . . . . . . . . . . . . . . . . . 20
3.3 Inverses modulo n . . . . . . . . . . . . . . . . . . . . . . . . . . . . . . . 22
3.4 Petit théorème de Fermat . . . . . . . . . . . . . . . . . . . . . . . . . . . 24
3.5 Carrés modulo n . . . . . . . . . . . . . . . . . . . . . . . . . . . . . . . . 25
3.6 Nombres rationnels et irrationnels, développement décimal . . . . . . . . . 26

1
4 Utilisation de factorisations 28
4.1 Identités remarquables . . . . . . . . . . . . . . . . . . . . . . . . . . . . . 28
4.2 L'équation a2 − b2 = n . . . . . . . . . . . . . . . . . . . . . . . . . . . . . 30
4.3 L'équation a2 + b2 = c2 . . . . . . . . . . . . . . . . . . . . . . . . . . . . 30
4.4 Exercices supplémentaires . . . . . . . . . . . . . . . . . . . . . . . . . . . 31

1 Préliminaires
1.1 Notations
On note N = {0, 1, 2, 3, . . .} l'ensemble des entiers naturels, N∗ l'ensemble des entiers
naturels non nuls ;
Z = {. . . , −3, −2, −1, 0, 1, 2, 3, . . .} l'ensemble des entiers relatifs ;
Q l'ensemble des nombres rationnels, c'est-à-dire qui peuvent s'écrire sous la forme ab
avec a ∈ Z et b ∈ N ;

R l'ensemble des nombres réels.


Q∗ désigne l'ensemble des rationnels non nuls, Q∗+ l'ensemble des rationnels strictement

positifs et Q− l'ensemble des rationnels strictement négatifs. On introduit de même les
∗ ∗ ∗
notations Z− , Z , R , R+ , R− .

Si a et b sont deux nombres, alors ab désigne le produit a × b.

1.2 Principe de récurrence


Dans ce document, nous utiliserons fréquemment le principe de récurrence suivant.
Soit P (n) une propriété d'un entier n. On suppose que
(i) (initialisation) P (0) est vraie
(ii) (hérédité) pour tout entier naturel n, si P (n) est vraie alors P (n + 1) est vraie.

P(0) P(1) P(2) P(3) P(4)

Alors P (n) est vraie pour tout n.


Vocabulaire : lorsque l'on essaye de démontrer l'implication P (n) =⇒ P (n + 1), la
propriété P (n) que l'on suppose vraie s'appelle l'hypothèse de récurrence.

Une variante de ce principe est la récurrence forte : supposons que


(i) (initialisation) P (0) est vraie
(ii) (hérédité) pour tout entier naturel n, si P (k) est vraie pour tout k 6 n alors
P (n + 1) est vraie.

Alors P (n) est vraie pour tout n.

2
1.3 Mode d'emploi des exercices
Trois niveaux de dicultés (sans doute subjectifs) sont indiqués pour les exercices.
Les exercices non marqués sont des tests de compréhension immédiate, ou bien de-
mandent simplement d'appliquer une méthode indiquée quelques lignes plus haut.
Les exercices marqués avec le symbole ¶ peuvent demander une petite idée non évi-
dente, ou un peu de calcul.
Les exercices marqués avec ¶¶ nécessitent un temps de réexion un peu plus long.

3
2 Divisibilité
Dénition 2.1.
Soient a, b deux entiers relatifs. On dit que a divise b (ou que b est divisible par a, ou
que b est un multiple de a) s'il existe un entier c tel que b = ac. On note a | b.

Par exemple, 2 | 6, 26 | 7, 1 | n, n | n, n | 0 pour tout entier n.

Proposition 2.2.
Soient a, b, b0 , c, λ des entiers.
(i) (Transitivité) Si a | b et b | c alors a | c.
(ii) Si a | b alors a | bc.
0
(iii) Si a | b et a | b alors a | b + b .
0
0
(iv) Si a | b et a | b alors a | b + λb .
0

(v) Si λ 6= 0 alors a | b si et seulement si λa | λb.


(vi) Si a | b et b 6= 0 alors |a| 6 |b|.
(vii) Si a | b et b | a alors b = a ou b = −a.

Démonstration. (i) Par dénition, il existe u et v tels que b = au et c = bv . On a alors


c = (au)v = a(uv) donc a | c.
(ii) Il est clair que b | bc. D'après (i), on en déduit que a | bc.
(iii) Il existe u et v tels que b = au et b0 = av . On a alors b + b0 = au + av = a(u + v)
donc a | b + b0 .
(iv) D'après (ii), on a a | λb0 , et d'après (iii) on en déduit que a | b + (λb0 ).
(v) Si a | b, u tel que b = au. Ceci entraîne λb = (λa)u donc λa | λb.
alors il existe
Réciproquement, si λa | λb, alors il existe v tel que λb = λau. Comme λ 6= 0, on peut
diviser par λ, ce qui donne b = au, autrement dit a | b.

(vi) Soit u tel que b = au. Comme b 6= 0, on a u 6= 0, donc 1 6 |u|. En multipliant


membre à membre par |a|, on obtient |a| 6 |au| = |b|.
(vii) On suppose que a|b
b | a. et
Si a = 0, alors le fait que a | b entraîne b = 0 donc on a bien b = a ou b = −a. De
même, si b = 0 alors b = a ou b = −a.
Si a 6= 0 et b 6= 0, alors d'après (vi) on a |a| 6 |b| et |b| 6 |a|, donc |a| = |b|, ce qui
donne bien b = a ou b = −a.

Exercice 2.1.
Montrer que si a|b et c|d alors ac | bd.

4
Exercice 2.2.
Quels sont les entiers n∈N tels que n | n + 7?

Exercice 2.3.
Quels sont les entiers n∈N tels que n2 + 1 | n ?

Exercice 2.4.
Montrer que pour tout entier n, l'entier n(n + 1) est pair.

Exercice 2.5.
Montrer que pour tout entier n, l'entier n(n + 1)(n + 2) est divisible par 6.

Exercice 2.6.
Soient a, b, c des entiers. Montrer que si n est un entier vériant an2 + bn + c = 0
alors n | c.

Exercice 2.7.
¶Déterminer les entiers n tels que n5 − 2n4 − 7n2 − 7n + 3 = 0.

Exercice 2.8.
¶Soient a > 1 et n des entiers tels que a | n+2 et a | n2 + n + 5 . Montrer que a=1
ou a = 7.

La division Euclidienne permet de tester si un entier est divisible par un autre.

Théorème 2.3.
Soient a et b deux entiers tels que b > 1. Alors il existe un et un seul couple (q, r)
d'entiers tel que
• a = bq + r ;
• 0 6 r 6 b − 1.

Les entiers q r s'appellent


et respectivement le quotient et le reste de la division Eu-
clidienne de a par b.

5
Démonstration. Montrons d'abord l'unicité. Si a = bq + r = bq 0 + r0 avec 0 6 r 6 b − 1
0 0 0 0 0
et 0 6 r 6 b − 1, alors bq − bq = r − r , donc b|q − q | = |r − r| < b. En divisant par b,
0 0 0 0
on en déduit |q − q | < 1, donc q = q . Il vient r − r = bq − bq = 0, puis r = r .
0

Montrons l'existence. Traitons d'abord le cas a > 0. Remarquons d'abord que


• il existe des entiers x tels que bx 6 a (par exemple x = 0) ;
• si x > a alors bx > a (puisque bx > ba).
Il existe donc un plus grand entier q tel que bq 6 a. Par dénition de q , on a b(q + 1) > a,
c'est-à-dire bq + b > a. Posons r = a − bq , on a alors 0 6 r < b, ce qui prouve l'existence
dans le cas a > 0.
Si maintenant a < 0, on remarque que a − ba = (b − 1)(−a) > 0. On applique la
0 0
division Euclidienne de a − ba par b : il existe q et r tels que a − ba = bq + r et 0 6 r < b.
0
On a alors a = b(a + q ) + r = bq + r avec q = a + q .
0

Exemple : pour a = 25 et b = 7, le quotient est 3 et le reste est 4. Pour déterminer ces


valeurs, on a en fait procédé exactement comme dans la preuve du théorème. On essaye
7 × 1 = 7, 7 × 2 = 14, 7 × 3 = 21, 7 × 4 = 28 dépasse la valeur a. Donc le quotient q est
égal à 3. Pour calculer le reste, on eectue la soustraction 25 − (7 × 3).

Remarque 2.4.
Si b est un entier relatif non nul quelconque, il existe encore un et un seul couple
(q, r) d'entiers tel que a = bq + r et 0 6 r 6 |b| − 1. Le cas b > 1 ayant déjà été
traité, supposons b 6 −1. L'unicité se montre de la même façon. Pour l'existence, on
0
eectue la division Euclidienne de a par −b, ce qui permet d'écrire a = (−b)q + r .
0
On pose alors q = −q et on a a = bq + r .

Par exemple, la division Euclidienne de −17 par −5 s'écrit −17 = (−5) × 4 + 3.

Proposition 2.5.
Soient a, b, d, q, r des entiers. On suppose que a = bq + r. Alors d divise a et b si et
seulement si d divise b et r.

Démonstration. Si d divise a et b alors d divise a − bq d'après la Proposition 2(iv), donc


d divise b et r. De même, si d divise b et r alors d divise bq + r = a.

Soient maintenant a > b > 0. L'algorithme d'Euclide consiste à eectuer des divisions
Euclidiennes successives : on pose a0 = a eta1 = b, puis pour tout k > 0 on eectue la
division Euclidienne de ak par ak+1 et on appelle ak+2 le reste.

6
a0 = a1 q1 + a2
a1 = a2 q2 + a3
a2 = a3 q3 + a4
···
an−2 = an−1 qn−1 + an
an−1 = an qn + an+1
jusqu'à tomber sur un reste nul an+1 = 0. Le processus s'arrête bien car les restes sont
de plus en plus petits : a1 > a2 > a3 > · · · . On note n l'indice tel que an est le dernier
reste non nul.

Dénition 2.6.
Soient ab deux entiers. Un entier d > 0 est appelé
et PGCD (plus grand commun
diviseur) de a et b s'il vérie la propriété suivante :
0 0
pour tout d entier, d divise a et b si et seulement si d
0 divise d.

(Par exemple, pour a = 15 et b = 21, l'entier d = 3 est le PGCD de a et b. En eet,


les diviseurs positifs communs de 15 et 21 sont 1 et 3, et un entier naturel divise 3 si et
seulement s'il est égal à 1 ou à 3.)
Remarquons que le PGCD divise a et b (il sut de prendre d0 = d dans la caractérisa-
tion du PGCD : puisque d divise d, il divise a et b). De plus, le PGCD est unique car si
d1 et d2 vérient la propriété du PGCD alors d1 | d2 et d2 | d1 , ce qui entraîne d1 = d2 .
Le théorème suivant montre l'existence du PGCD :

Théorème 2.7.
Si a et b sont non nuls, alors dans l'algorithme d'Euclide, an est le PGCD de a et b.
Si b = 0 alors a est le PGCD de a et b.

Démonstration. Premier cas : a et b sont non nuls. D'après la proposition précédente, on


a

d0 | a et d0 | b ⇐⇒ d0 | a0 et d0 | a1
⇐⇒ d0 | a1 et d0 | a2
···
⇐⇒ d0 | an et d0 | an+1
⇐⇒ d0 | an
(cette dernière équivalence découle du fait que an+1 = 0 et que d0 | 0 est vrai pour tout
d0 ).
Deuxième cas : b = 0. Alors d0 | a et d0 | b si et seulement si d0 | a, donc a est le PGCD
de a et b.

7
Calculons par exemple le PGCD de 183 et 117 par ce procédé.

183 = 117 ×1+ 66

117 = 66 ×1+ 51

66 = 51 ×1+ 15

51 = 15 ×3+ 6

15 = 6 ×2+ 3

6 = 3 ×2+ 0

(On a encadré les termes a0 , a1 , . . . , an+1 pour les mettre en évidence.) Le dernier reste
non nul est 3, donc le PGCD de 183 et 117 est égal à 3.

Exercice 2.9.
Vérier par l'algorithme d'Euclide que le PGCD de 364 et de 154 est égal à 14.

Exercice 2.10.
¶Combien 10100 et 10121 + 10813 + 10 ont-ils de diviseurs communs ?

P GCD à plusieurs entiers. On a par exemple P GCD(a, b, c) =


On peut généraliser la notion de
P GCD(P GCD(a, b), c) = P GCD(a, P GCD(b, c)). En eet,

(d | a et d | b) et d|c ⇐⇒ d | P GCD(a, b) et d|c


⇐⇒ d | P GCD(P GCD(a, b), c)

et de même d|a et (d | b et d | c) ⇐⇒ d | P GCD(a, P GCD(b, c)).


On dénit également le PGCD de deux (ou plusieurs) entiers relatifs par P GCD(a, b) =
P GCD(|a|, |b|).

Proposition 2.8.
Si a = bq + r alors P GCD(a, b) = P GCD(b, r).

Démonstration. Evident d'après la Proposition 5.

Exercice 2.11.
Déterminer le P GCD de 1000000000 et 1000000005.

L'une des notions les plus importantes en arithmétique est celle de nombre premier :

8
Dénition 2.9.
Un entier p est dit premier si p>2 et si les seuls diviseurs positifs de p sont 1 et p.

La liste des nombres premiers commence par

2, 3, 5, 7, 11, 13, 17, 19, 23, 29, 31, 37, 41, 43, 47.
A noter que 1 n'est pas premier. La raison de cette convention est que l'on veut que
tout entier >2 admette une décomposition unique en facteurs premiers (voir plus loin).

Exercice 2.12.
131 est-il premier ? 221 est-il premier ?

Remarque 2.10.
Si p est premier, alors pour tous entiers naturels a et b, l'égalité p = ab entraîne
(a = 1 et b = p) ou (a = p et b = 1).

Démonstration. p = ab entraîne que a est un diviseur de p, donc a = 1 ou a = p. Si


a=1 alors p = ab = 1 × b = b et si a = p alors p = pb donc b = 1.
Proposition 2.11.
Soit n>2 un entier. Le plus petit diviseur >2 de n est un nombre premier.

Exemple : pour n = 75, les diviseurs > 2 de n sont 3, 5, 15, 25, 75. Le plus petit de ces
nombres est égal à 3, qui est bien premier.

Démonstration. Notons p le plus petit entier tel que


(i) p | n;
(ii) p > 2.
Notons que p est bien déni puisqu'il existe des entiers (par exemple n) vériant les
deux conditions (i) et (ii).
Soit d est un diviseur >2 de p. Commed | p et p | n, on a d | n. De plus, p étant le
plus petit diviseur > 2 de n, on a p 6 d. Or, d 6 p puisque d divise p, donc d = p. Ceci
prouve que l'unique diviseur > 2 de p est p lui-même, donc p est premier.
Remarque 2.12.
Ce rÃ
c sultat permet par rÃ
c currence de montrer que tout entier n ≥ 2 est un
produit de nombres premiers. En eet, 2 admet une dà c composition. Si de plus
k ≤ n admet une dÃ
tout entier c composition, alors n + 1 admet un diviseur premier
p : on Ã
c crit n + 1 = pk . Si k = 1, alors n + 1 est premier et la dà c composition
est immà c diate. Sinon, k ≤ n est un produit de nombres premiers donc n + 1 aussi.

9
Théorème 2.13 (Euclide).
Il existe une innité de nombres premiers.

Démonstration. Supposons par l'absurde qu'il n'existe qu'un nombre ni de nombres

premiers p1 < p2 < · · · < pr avec p1 = 2, p2 = 3, etc. Soit n = p1 p2 · · · pr + 1. On a


n > p1 = 2. Soit p le plus petit diviseur supà c rieur ou à c gal à 2 de n. D'après la
proposition précédente, p est un nombre premier, donc p gure dans la liste {p1 , . . . , pr }.
Par conséquent, p | p1 p2 · · · pr = n − 1. Comme p | n, on en déduit que p | n − (n − 1) = 1
donc p 6 1. Impossible.

Exercice 2.13.
1) Montrer que tout nombre premier >3 peut s'écrire, soit sous la forme 4k + 1, soit
sous la forme 4k − 1.
2) ¶Montrer qu'il existe une innité de nombres premiers de la forme 4k − 1.
[On pourra former un nombre de la forme 4(p1 · · · pr ) − 1.]

Remarque 2.14.
Il y a des résultats bien plus précis sur la répartition des nombres premiers. Par
exemple, le postulat de Bertrand dit que pour tout n > 2, il existe au moins un
nombre premier p tel que n < p < 2n. La démonstration de ce théorème est trop
longue pour être exposée ici.

Dénition 2.15.
Deux entiers a et b sont dits premiers entre eux si l'unique diviseur commun stricte-
ment positif est 1. Autrement dit, P GCD(a, b) = 1.

Par exemple, 6 et 35 sont premiers entre eux, mais 14 et 35 ne le sont pas.

Exercice 2.14.
1) Quels sont les entiers n tels que n et n + 2 sont premiers entre eux ?
2) ¶Quels sont les entiers n tels que n2 − 1 et n2 − 2n + 1 sont premiers entre eux ?

Plus généralement,

Dénition 2.16.
Des entiers a1 , a2 , . . . , an sont dits premiers entre eux dans leur ensemble

si l'unique entier strictement positif les divisant tous est 1. Autrement dit,
P GCD(a1 , a2 , . . . , an ) = 1.

10
Par exemple, 6, 10, 15 sont premiers entre eux dans leur ensemble mais ne sont pas
premiers entre eux deux à deux.

Proposition 2.17.
Soient a et b deux entiers non nuls, et d leur P GCD. Alors on peut écrire a = da0 et
b= db0 avec a0 et b0 premiers entre eux.

(Par exemple, pour a = 15 et b = 21, on écrit 15 = 3 × 5 et 21 = 3 × 7 ; les entiers 5


et 7 sont bien premiers entre eux.)

Démonstration. Comme b, les nombres a0 = ad et b0 = db sont entiers. Si un


d divise a et
entier naturel c
0 b0 , alors
cd divise da0 = a et db0 = b, donc cd divise d, ce qui
divise a et
0 0
entraîne c | 1, puis c = 1. Par conséquent, a et b n'ont pas d'autre diviseur commun que
0 0
1, autrement dit a et b sont premiers entre eux.

2.1 Le théorème de Bézout


Théorème 2.18 (Bézout).
(i) Soient a et b deux entiers. Notons d = a ∧ b. Alors il existe des entiers relatifs
u et v tels que d = au + bv .
(ii) Plus précisément, pour tout entier m il existe u0 et v0 tels que au0 + bv 0 = m si
et seulement si m est un multiple de d.
(iii) En particulier, a et b sont premiers entre eux si et seulement s'il existe u et v
tels que au + bv = 1.

Avant de démontrer ce théorème, commençons par un exemple : reprenons le calcul de


P GCD(183, 117) :

183 = 117 ×1+ 66

117 = 66 ×1+ 51

66 = 51 ×1+ 15

51 = 15 ×3+ 6

15 = 6 ×2+ 3

6 = 3 ×2+ 0

On va écrire les restes successifs en fonction de 183 et 117.


66 = 183 − 117

51 = 117 − 66 = 117 −( 183 − 117 )= 117 ×2− 183

15 = 66 − 51 =( 183 − 117 )−( 117 ×2− 183 )= 183 ×2− 117 ×3

11
6 = 51 − 15 × 3 = 117 × 2 − 183 −( 183 ×2− 117 × 3) × 3
= − 183 × 7 + 117 × 11
3= 15 − 6 × 2 = 183 × 2 − 117 × 3 − (− 183 × 7 + 117 × 11) × 2
= 183 × 16 − 117 × 25.
On a ainsi trouvé que 3 = 183u + 117v avec u = 16 et v = −25.

Exercice 2.15.
Trouver des entiers relatifs u et v tels que 364u + 154v = 14.

Démonstration du théorème. Montrons (i). Il sut d'expliquer formellement l'algorithme

utilisé plus haut. Si a = 0 ou b = 0 l'assertion est évidente. Supposons donc a > 0 et


b > 0. Quitte à échanger a et b, on peut supposer que a > b. On applique l'algorithme
d'Euclide : on pose a0 = a et a1 = b.
On observe que

a0 = au0 + bv0
a1 = au1 + bv1

avec u0 = 1, v0 = 0, u1 = 0, v1 = 1.
L'idée est de chercher à écrire ak sous la forme ak = auk + bvk pour tout k .
On a ensuite a2 = a0 − a1 q1 = au0 + bv0 − q1 (au1 + bv1 ) = (u0 − q1 u1 )a + (v0 − q1 v1 ),
donc
a2 = au2 + bv2
avec u2 = u0 − q1 u1 et v2 = v0 − q1 v1 .
On continue ainsi de proche en proche jusqu'à an = an−2 − qn−1 an−1 ,
an : comme
et comme an−2 et an−1 s'expriment en fonction de a et b, on peut exprimer an en
fonction de a et b. Plus précisément, on a an = aun + bvn avec un = un−2 − qn−1 un−1 et
vn = vn−2 − qn−1 vn−1 . On obtient ainsi d = au + bv avec d = an , u = un et v = vn .
(Cet algorithme s'appelle l'algorithme d'Euclide étendu.)

Montrons (ii). Si m est un multiple de d, alors il existe λ tel que m = λd. On a alors
m = (λu)a + (λv)b.
Réciproquement, si m = au0 + bv 0 , alors comme d divise a et b, il divise au0 + bv 0 donc
d divise m.
Montrons (iii). Si au + bv = 1 alors P GCD(a, b) divise 1, donc est égal à 1. La
réciproque a déjà été prouvée en première partie.

12
Remarque 2.19.
Plus généralement, pour tous a1 , . . . , an , il existe u1 , . . . , un tels que

a1 u1 + a2 u2 + · · · + an un = P GCD(a1 , . . . , an ).

La démonstration n'est pas dicile mais nous l'omettons pour que ce document garde
une longueur raisonnable.

Voici un corollaire important du théorème de Bézout :

Théorème 2.20 (Lemme de Gauss).


Soient a et b des entiers. Soit m un nombre qui est premier avec a, tel que m | ab.
Alors m | b.

Démonstration. Soient a, b, m comme dans l'énoncé. Il existe u et v tels que mu+av = 1.


On multiplie membre à membre par b, ce qui donne mu + (ab)v = b. Comme m divise
mu et (ab)v , il divise le membre de gauche, donc m | b.

Exercice 2.16.
Montrer que si x et y sont des entiers tels que 2x + 1 divise 8y alors 2x + 1 divise y.

Dans le cas particulier où m est un nombre premier, on obtient le résultat suivant :

Théorème 2.21.
Soit p un nombre premier et a, b des entiers. Si p | ab alors p|a ou p | b.

Démonstration. Soit p un nombre entier et a, b des entiers tels que p | ab. Si p ne divise
pas a, alors p est premier avec a, donc p|b d'après le lemme de Gauss.

Remarque 2.22.
L'hypothèse que p est premier est indispensable ici. Par exemple, 6 divise 10 × 21
mais 6 ne divise ni 10, ni 21.

Plus généralement,

Théorème 2.23.
Soit p un nombre premier. Si p divise le produit a1 · · · ar , alors p divise l'un des
nombres a1 , a2 , . . . , ar .

13
Démonstration. On le démontre par récurrence sur r. Si r=1 c'est évident. Supposons
la propriété vraie au rang r − 1.
Si p | a1 · · · ar , alors d'après le lemme de Gauss, p divise a1 ou p divise a2 · · · ar . Or, par
hypothèse de récurrence, p | a2 · · · ar entraîne que p divise l'un des nombres a2 , . . . , ar . Il
vient que p divise l'un des nombres a1 , . . . , ar .

Soient a et b deux entiers premiers entre eux. Revenons à l'équation au + bv = 1


d'inconnues u et v . L'algorithme d'Euclide étendu permet d'en trouver un couple de
solutions. Notons (u0 , v0 ) cette solution particulière : on a donc au0 + bv0 = 1. Cherchons
maintenant toutes les solutions.
Soit (u, v) un couple d'entiers tels que au + bv = 1. On a

au + bv = au0 + bv0
a(u − u0 ) = b(v0 − v). (1)

Or, a divisea(u − u0 ), donc a divise b(v0 − v). Comme de plus a est premier avec b, il
divise v0 − v d'après le Lemme de Gauss. Soit k ∈ Z tel que v0 − v = ak . On reporte
dans (1) :
a(u − u0 ) = bak,
ce qui donne u − u0 = bk , et nalement

u = u0 + bk
v = v0 − ak.
Réciproquement, si u = u0 + bk et v = v0 − ak , on vérie que (u, v) est une solution :

au + bv = a(u0 + bk) + b(v0 − ak) = au0 + bv0 + abk − abk = au0 + bv0 = 1.
Conclusion : (u, v) vérie l'équation au + bv = 1 si et seulement s'il existe k ∈ Z tel
que u = u0 + bk et v = v0 − ak .

Exercice 2.17.
Déterminer toutes les solutions de l'équation 9u − 7v = 1.

Exercice 2.18.
¶Soit n un entier xé. Déterminer tous les couples d'entiers (x, y) vériant l'équation
16x + 26y = n.

2.2 Le théorème fondamental de l'arithmétique


Théorème 2.24.
Tout entier > 2 admet une décomposition en produit de nombres premiers. Cette
décomposition est unique à l'ordre près.

14
Par exemple, 84 = 2 × 2 × 3 × 7 est une décomposition de 84 en facteurs premiers. Les
autres décompositions s'obtiennent en permutant les termes, comme 84 = 3 × 2 × 7 × 2.

Démonstration. Soit P(n) la propriété  n admet une et une seule décomposition en pro-
duit de nombres premiers. Nous allons montrer par récurrence que P(n) est vraie pour
tout n > 2.
La propriété P(2) est vraie : si n = p1 · · · pr est produit de nombres premiers, alors
r=1 puisquen est premier, et par suite p1 = 2.
Soit n > 3, et supposons P(2), P(3), . . . , P(n − 1) vraies. Montrons que P(n) est vraie.
Il faut d'abord prouver l'existence d'une décomposition de n en produit de nombres
premiers.
p le plus petit diviseur de n. On sait que p est premier.
Soit
n = p, alors n est bien produit de nombres premiers.
Si
n
Si p < n, alors comme 2 6
p < n, par hypothèse de récurrence il existe des nombres
n
premiers p1 , . . . , pr tels que
p = p1 · · · pr . Par conséquent, n = pp1 · · · pr admet bien une
décomposition en facteurs premiers.
Montrons maintenant l'unicité de la décomposition. Supposons que

n = p1 p2 · · · pr = q1 q2 · · · qs

avec p1 , . . . , pr et q1 , . . . , qr premiers.
Si n est premier, on a nécessairement r = s = 1, donc p1 = q1 . Supposons donc n non
premier.
D'après le Lemme de Gauss, comme p1 divise q1 q2 · · · qs , il divise l'un des nombres
q1 , . . . , q s dont p1 est égal à l'un de ces nombres. Quitte à changer l'ordre des q1 , . . . , q s ,
on peut supposer que p 1 = q1 . On a alors

p2 · · · pr = q2 · · · qs

Or, P(p1 · · · pr ) est vraie par hypothèse de récurrence, donc (p2 , . . . , pr ) est égal à (q2 , . . . , qs )
à l'ordre près.

Plutôt que d'écrire une décomposition sous la forme 84 = 2 × 2 × 3 × 7, on préfère une


écriture du type 84 = 22 × 3 × 7. Plus généralement, un entier n peut s'écrire sous la
forme
n = pα1 1 pα2 2 · · · pαr r
avec p1 < p2 < · · · < pr premiers. L'entier αj s'appelle la valuation pj -adique de n et se
note vpj (n). Pour tout nombre premier p, l'entier vp (n) est le plus grand entier m tel que
pm divise n.
Par exemple, v2 (84) = 2, v3 (84) = 1, v5 (84) = 0, vp (1) = 0 pour tout p.

Exercice 2.19.
Combien vaut v3 (18100 ) ?

15
Exercice 2.20.
Combien vaut v2 (2100 + 2200 ) ?

Exercice 2.21.
¶Soit n = 1 × 2 × 3 × · · · × 100.
1) Déterminer v7 (n).
2) Déterminer le nombre de zéros à la n de l'écriture décimale de n.

Proposition 2.25.
Soient a et b des entiers > 1.
(i) vp (ab) = vp (a) + vp (b) pour tout p premier.
(ii) a | b si et seulement si pour tout entier premier p on a vp (a) 6 vp (b).
(iii) vp (a ∧ b) = min(vp (a), vp (b)) pour tout p premier.

Démonstration. (i) Soient p1 , . . . , pr les nombres premiers qui divisent a ou b. On écrit

a = pα1 1 · · · pαr r et b = pβ1 1 · · · pβr r avec αi > 0 et βi > 0 pour tout i = 1, 2, . . . , r.


α1 +β1
Alors ab = p1 · · · pαr r +βr , d'où vpi (ab) = α + βi = vpi (a) + vpi (b) pour tout i.
(ii) Si a|b alors il existe c tel que b = ac, d'où vp (b) = vp a) + vp (c) > vp (a).
Réciproquement, si vp (a) 6 vp (b) pour tout entier premier p, alors on peut écrire
a = pα1 1 · · · pαr r et b = pβ1 1 · · · pβr r avec αi 6 βi . Posons γi = βi − αi et c = pγ11 · · · pγr r . Alors
on a b = ac, donc a | b.

(iii) Avec les notations précédentes, posons δi = min(αi , βi ) et d = pδ11 · · · pδrr . D'après
0 0 0
(ii), on a d | a et d | b. D'autre part, pour tout d > 1 on a d | a et d | b si et seulement
0 0
si vp (d ) 6 vp (a) et vp (d) 6 vp (b) pour tout p premier. Ceci équivaut à vp (d ) 6 vp (d)
0
pour tout p premier, ou encore à d | d. Ceci prouve que d = a ∧ b.

Exercice 2.22.
Montrer sans calcul que 87457561565 ne divise pas 100000000000000000000000007.

De la même façon que ci-dessus, on montrerait :

Proposition 2.26.
Soienta, b > 1 des entiers. Il existe un et un seul nombre, appelé le plus petit commun
multiple de a et b, et noté P P CM (a, b) ou a∨b, caractérisé par la propriété suivante :
Soit m un entier non nul. Alors a | m et b | m si et seulement si P P CM (a, b) | m.
De plus, vp (a ∨ b) = max(vp (a), vp (b)).

16
Exercice 2.23.
Montrer que si a2 | b2 alors a | b.

Exercice 2.24.
Montrer que si p est premier et p | a2 alors p2 | a2 .

Exercice 2.25.
Montrer que ab = P GCD(a, b) × P P CM (a, b).

Exercice 2.26.
¶Déterminer tous les couples d'entiers naturels (x, y) vériant :
P GCD(x, y) = 8, P P CM (x, y) = 440, x + y = 128.

Exercice 2.27.
Un nombre est dit parfait si la somme de tous ses diviseurs positifs est égale à 2n.
Montrer que si 2
k+1 −1 k k+1
est un nombre premier alors 2 (2 − 1) est parfait.
¶¶Montrer que tout nombre parfait pair est de cette forme.

2.3 Nombre de diviseurs d'un entier


Théorème 2.27.
Soitn > 1 un entier dont la décomposition en facteurs premiers s'écrit n = pα1 1 · · · pαr r .
Alors n admet (α1 + 1) · · · (αr + 1) diviseurs positifs.

Démonstration. Les diviseurs de n s'écrivent sous la forme pβ1 1 · · · pβr r avec

0 6 β1 6 α1 ,
...
06 βr 6 αr .
Pour β1 on a α1 + 1 choix possibles, pour β2 on a α2 + 1 choix, etc. ce qui donne en tout
(α1 + 1) · · · (αr + 1) choix possibles.

Par exemple, les diviseurs (positifs) de 100 = 22 × 52 sont


20 × 50 , 20
× 51 ,
× 20 52 ,
21 × 50 , 21 × 51 , 21 × 52 ,
22 × 50 , 22 × 51 , 22 × 52 .
Notons τ (n) le nombre de diviseurs (positifs) de n.

17
Exercice 2.28.
Combien vaut τ (1000000) ?

Exercice 2.29.
Quels sont les entiers n 6 1000 tels que τ (n) = 7 ?

Exercice 2.30.

Montrer que τ (n) 6 2 n pour tout n.

Exercice 2.31.
Un jardinier doit planter 480 arbustes en n rangées égales contenant au moins 6
arbustes. De combien de manières peut-il le faire ?

Exercice 2.32.
¶N est divisible par 6, N n'est pas divisible par 8, et N a exactement 15 diviseurs.
Que vaut N?

Exercice 2.33.
A quelle condition un entier n admet-il un nombre impair de diviseurs ?

Exercice 2.34.
Montrer que si m et n sont premiers entre eux alors τ (mn) = τ (m)τ (n).
¶¶Est-il vrai que si τ (mn) = τ (m)τ (n) alors m et n sont premiers entre eux ?

18
3 Congruences
3.1 Dénition et propriétés de base
Dénition 3.1.
Soit n un entier non nul. On dit que a et b sont congrus modulo n si n divise a − b.
On note a ≡ b [n], ou encore a = b (mod n).

Lorsque l'on eectue une division Euclidienne de a par n sous la forme a = nq + r,


on a a ≡ r [n], donc tout entier est congru modulo n à un et un seul entier parmi
{0, 1, . . . , n − 1}. Une autre manière de dénir la congruence modulo n est de dire que
a ≡ b [n] si et seulement si les restes des divisions Euclidiennes de a et de b par n sont
égaux.
La congruence se comporte comme l'égalité :

Proposition 3.2.
Soient a, b, c des entiers et n un entier non nul.
(réexivité) a ≡ a [n].
(symétrie) si a ≡ b [n] alors b ≡ a [n].
(transitivité) si a ≡ b [n] et b ≡ c [n] alors a ≡ c [n].

Démonstration. a − a = 0 est divisible par n, d'où la réexivité.


Si n divise a − b, alors n divise −(a − b) = b − a, d'où la symétrie.
Si n divise a − b et n divise b − c, alors n divise (a − b) + (b − c), donc n divise a − c.
Ceci prouve la transitivité.

La congruence est compatible avec l'addition et la multiplication :

Proposition 3.3.
Soient a, b, c, d des entiers et n un entier non nul. Supposons a ≡ b [n] et c ≡ d [n].
Alors
(i) a + c ≡ b + d [n] ;
(ii) ac ≡ bd [n].

Démonstration. Par hypothèse, n divise a − b et c − d.


(i) n divise (a − b) + (c − d) = (a + c) − (b + d) donc a + c ≡ b + d [n].
(ii) ac − bd = a(c − d + d) − bd = a(c − d) + ad − bd = a(c − d) + (a − b)d est divisible
par n puisque c − d et a − b le sont, donc ac ≡ bd [n].

On raisonne couramment modulo 12 dans la vie de tous les jours. S'il est 9 heures du
matin, alors dans 5 heures il sera 2 heures de l'après-midi : ceci correspond au fait que
9 + 5 ≡ 2 [12].

19
Le fait de raisonner modulo 10 est aussi très intuitif, car il correspond à regarder
le dernier chire d'un entier. Par exemple, 857382993 est congru à 3 modulo 10 car
857382993 − 3 = 857382990 est un multiple de 10.

Proposition 3.4.
Si a ≡ b [n] alors pour tout entier k > 1, on a ak ≡ bk [n].

Démonstration. Il sut d'appliquer k fois la Proposition 3(ii).

Exercice 3.1.
Quels sont les entiers p tels que p et p+1 sont premiers ?

Exercice 3.2.
¶Quels sont les entiers p tels que p, p + 2 et p+4 sont premiers ?

3.2 Critéres de divisibilité


Soit n un entier. On noteak · · · a0 son écriture décimale.
Par exemple, si n = 147 alors k = 2, a0 = 7, a1 = 4, a2 = 1.
On a donc 0 6 aj 6 9 pour tout j , et

n = a0 + 10a1 + 102 a2 + · · · + 10k ak .

Proposition 3.5.
n est congru à a0 + · · · + ak modulo 9 et modulo 3. Par conséquent, n est divisible
par 9 (respectivement) 3 si et seulement si a0 + · · · + ak l'est.

Démonstration. On a 10−1 = 9 donc 10 ≡ 1 [9]. D'après la Proposition 4, on a 10j ≡ 1 [9]


2 k
pour tout j , donc n ≡ a0 + 10a1 + 10 a2 + · · · + 10 ak ≡ a0 + · · · + ak [9].
A fortiori, n est congru à a0 + · · · + ak modulo 3.
Enn, n est divisible par 9 (resp. 3) si et seulement si n est congru à 0 modulo 9 (resp.
3), si et seulement si a0 + · · · + ak est congru à 0 modulo 9 (resp. 3).

Exercice 3.3.
48767621 est-il divisible par 9?

20
Proposition 3.6.
n est congru à a0 − a1 + a2 − a3 · · · modulo 11. En particulier, n est divisible par 11
si et seulement si la somme alternée de ses chires est divisible par 11.

Démonstration. On a 10 ≡ −1 [11], donc 10k ≡ (−1)k [11]. On en déduit que

a0 + 10a1 + 102 a2 + 103 a3 + · · · ≡ a0 − a1 + a2 − a3 + · · · [11].

Exercice 3.4.
98473092 est-il divisible par 11 ?

Proposition 3.7.
n est divisible par 4 si et seulement si a1 a0 est divisible par 4.

Démonstration. a − a1 a0 = ak ak−1 · · · a2 00 est divisible par 100, donc par 4.

Proposition 3.8.
n est divisible par 8 si et seulement si a2 a1 a0 est divisible par 8.

Démonstration. Analogue.

Exercice 3.5.
3645
Sans eectuer la division, montrer que
45 est un entier.

Exercice 3.6.
Le PGCD de 193116 et de 127413 est-il pair ? Est-il divisible par 3? Par 9? Par 11 ?
Par 33 ? Par 99 ?

Exercice 3.7.
¶À quelle condition sur les chires x et y le nombre 27x85y est-il divisible par 33 ?

21
3.3 Inverses modulo n
Dénition 3.9.
Un entier b est appelé inverse de a modulo n si et seulement si ab ≡ 1 [n]. Si a possède
un inverse modulo n, alors on dit que a est inversible modulo n.

Par exemple, 3 est inverse de 5 modulo 7 car 3 × 5 = 15 est congru à 1 modulo 7. Les
nombres . . . , −4, 3, 10, 17, 24, 31, . . . sont tous inverses de 5 modulo 7 car (3 + 7k) × 5 =
15 + 7(5k) ≡ 15 ≡ 1 [7].

Proposition 3.10.
a est inversible modulo n si et seulement si a et n sont premiers entre eux. L'inverse
de a est alors unique modulo n.

Démonstration. Si a est inversible modulo n, alors il existe u tel que au ≡ 1 [n]. Par
dénition, cela signie que au−1 est un multiple de n, donc il existe v tel que au−1 = nv .
On obtient alors une relation de Bézout au + n(−v) = 1, donc a et n sont premiers entre
eux.
Réciproquement, si a et n sont premiers entre eux, alors il existe une relation de
Bézout au + nv = 1. En passant modulo n les deux membres de l'égalité, on en déduit
que au ≡ 1 [n], donc a est inversible modulo n.
Montrons maintenant l'unicité. Supposons que b et b0 sont deux inverses de a modulo
n. Alors
b = b × 1 ≡ b(ab0 ) ≡ (ba)b0 ≡ 1 × b0 = b0 [n]
donc b et b0 sont congrus modulo n.

Remarquons que la démonstration ci-dessus donne un algorithme pour calculer l'inverse


d'un nombre modulo n : il sut de trouver une relation de Bézout grâce à l'algorithme
d'Euclide étendu.

Proposition 3.11.
Si a est inversible modulo n, et si ax ≡ ay [n], alors x ≡ y [n].

Démonstration. Soit b un inverse de a modulo n. On a x ≡ (ba)x = b(ax) ≡ b(ay) =


(ba)y ≡ y [n].

On peut donc simplier par a une congruence modulo n si a est inversible modulo n.
L'hypothèse que a est inversible modulo n est nécessaire, puisque 2 × 3 ≡ 2 × 8 [10]
bien que 3 6≡ 8 [10].

22
Exercice 3.8.
Vérier que les inversibles modulo 8 sont 1, 3, 5, 7, et qu'ils sont tous égaux à leurs
inverses.

Exercice 3.9.
Trouver l'inverse de 37 modulo 53.

Proposition 3.12.
Si a et b sont inversibles modulo p alors ab est inversible modulo p.

L'idée est assez naturelle : si on peut diviser par a et par b, alors on peut diviser par
ab en divisant d'abord par a puis par b.
Démonstration. Il existe a0 et b0 tels que aa0 ≡ 1 [p] et bb0 ≡ 1 [p]. On en déduit
(ab)(a0 b0 ) = (aa0 )(bb0 ) ≡ 1 × 1 ≡ 1 [p], donc ab est inversible modulo p et son inverse
0 0
est a b .

Proposition 3.13.
Soit p un nombre premier et a un entier non divisible par p. Alors a est égal à son
inverse modulo p si et seulement si a ≡ 1 [p] ou a ≡ −1 [p].

Démonstration. a est égal à son inverse modulo p


⇐⇒ a2 ≡ 1 [p]
⇐⇒ a2 − 1 ≡ 0 [p]
⇐⇒ (a − 1)(a + 1) ≡ 0 [p]
⇐⇒ p | (a − 1)(a + 1)
⇐⇒ p | a − 1 ou p | a + 1 (d'après le Lemme de Gauss)
⇐⇒ a − 1 ≡ 0 [p] ou a + 1 ≡ 0 [p]
⇐⇒ a ≡ 1 [p] ou a ≡ −1 [p].

Théorème 3.14 (Wilson).


Si p est premier, alors (p − 1)! ≡ −1 [p].

Démonstration. Rappelons que (p − 1)! = 1 × 2 × 3 × · · · × (p − 1).


Si p = 2, on a (p − 1)! = 1 ≡ −1 [2]. Supposons p > 3.
Notons que p − 1 ≡ −1 [p].
Les nombres 2, 3, . . . , p − 2 ne sont congrus, ni à 1, ni à −1 modulo p. Dans le produit
2 × 3 × · · · × p − 2, on regroupe chaque terme avec son inverse modulo p. On obtient ainsi
2 × 3 × · · · × (p − 2) ≡ 1 [p].
Ainsi, 2 × 3 × · · · × (p − 2) × (p − 1) ≡ p − 1 ≡ −1 [p], ce qui s'écrit (p − 1)! ≡ −1 [p].

23
3.4 Petit théorème de Fermat
Théorème 3.15.
Soit p un nombre premier. Alors pour tout entier a non divisible par p, on a

ap−1 ≡ 1 [p].

Démonstration. Soit A = {1, 2, . . . , p − 1}. Soit B l'ensemble des restes modulo p de


{a, 2a, 3a, . . . , (p − 1)a}.
Tous les éléments de B sont des éléments de A : en eet, si 1 6 k 6 p − 1 alors ka
n'est pas divisible par p, donc son reste modulo p est compris entre 1 et p − 1.
De plus, les éléments a, 2a, . . . , (p − 1)a sont deux à deux distincts modulo p : en eet,
si 1 6 k 6 ` 6 p − 1 et ka ≡ `a [p], comme a est inversible modulo p, en simpliant par
a on obtient k ≡ ` (mod p) donc k = `.
On en déduit que B possède p − 1 éléments. Comme A possède le même nombre
d'éléments, on a B = A, et donc le produit de tous les éléments de A est égal au produit
de tous les éléments de B , ce qui s'écrit

1 × 2 × · · · × (p − 1) ≡ a × (2a) × (3a) × · · · (p − 1)a (mod p).

En simpliant par 1 × 2 × · · · (p − 1), on obtient 1 ≡ ap−1 [p].

Exemple d'application : soit à calculer 21000 modulo 7.


On sait que 26 ≡ 1 [7]. Comme 1000 = 6 × 166 + 4, on a
21000 = 26×166× 24 = (26 )166 × 16 ≡ 1166 × 2 ≡ 2 [7].

Exercice 3.10.
a) Pour quelles valeurs de n ∈ N, 611 + 5n + 2 est-il divisible par 7 ?
b) Quel est le reste de la division de 705432
50 par 11 ?

c) Pour quelles valeurs de n ∈ N, 5


6n n
+ 5 + 2 est-il divisible par 7 ?
5 3
d) Pour quelles valeurs de n ∈ N, 81n − 45n + 4n est-il divisible par 5?

Exercice 3.11.
9
¶Quel est le dernier chire de l'écriture décimale de 73 ?

Exercice 3.12.
2712 est-il divisible par 3, 5, 7, 9, 11 ?

24
3.5 Carrés modulo n
Pour illustrer la notion de carré modulo n, commençons par un exercice :

Exercice 3.13.
Existe-t-il des entiers a et b tels que a2 + b2 = 10100 + 3 ?

Démonstration. La réponse est non. L'idée est de raisonner modulo 4. On a

02 ≡ 0 [4]
12 ≡ 1 [4]
22 ≡ 0 [4]
32 ≡ 1 [4].
On en déduit que pour tous a, b, les entiers a2 et b2 sont congrus à 0 ou à 1 modulo
4. Par conséquent, a
2 + b2 est congru à 0, 1 ou 2 modulo 4. Or, 10100 + 3 est congru à 3
modulo 4 donc ne peut pas être égal à a2 + b2 .

Ceci conduit à la

Dénition 3.16.
Un entier a est un carré modulo n s'il existe b tel que a ≡ b2 [n].

Observons par exemple les carrés modulo 13.


x 0 1 2 3 4 5 6 7 8 9 10 11 12
x2 0 1 4 9 3 12 10 10 12 3 9 4 1

(Dans la deuxième ligne on a écrit le reste de la division Euclidienne de x2 par 13.)


On constate une symétrie : ce n'est pas un hasard puisque 12
2 = (13 − 1)2 ≡ (−1)2 ≡
1 [13], 112 ≡ (13 − 2)2 ≡ 22 [13], etc.
Pour trouver la liste des carrés modulo n, il sut juste de calculer k2 modulo n pour
n
06k6 2.
Dans l'exemple précédent, on voit qu'il y a exactement sept carrés modulo 13, à savoir
0, 1, 3, 4, 9, 10, 13. Plus généralement,

Proposition 3.17.
p+1
Soit p un nombre premier impair. Alors il y a exactement
2 carrés modulo p.

Démonstration. D'après la discussion précédente, tout carré modulo p est congru à k2


p−1 p+1
pour un certain entier 0 6 k 6
2 . On voit déjà qu'il y a au plus 2 carrés modulo
p. Pour conclure, il reste à montrer que si 0 6 k < ` 6 p−1 2 2
2 alors k et ` ne sont pas
congrus modulo p.
En eet, dans le cas contraire, p diviserait `2 − k 2 = (` − k)(` + k). Or, 1 6 ` − k 6
` + k 6 p − 1, donc p ne divise
2 2
ni ` − k , ni ` + k , et par suite p ne divise pas ` − k , ce
qui est contradictoire.

25
Exercice 3.14.
Donner la liste de tous les carrés modulo 5 et de tous les carrés modulo 8.

Exercice 3.15.
Existe-t-il des entiers a, b, c tels que a2 + b2 + c2 = 10100 + 7 ?

Exercice 3.16.
−3 est-il un carré modulo 103 ?

On s'intéresse à la question suivante : −1 est-il un carré modulo n ? Par exemple, pour


n = 5, on a −1 ≡ 4 [5] et 4 = 22 , donc −1 est un carré modulo 5. Par contre, on vérie
facilement que −1 n'est pas un carré modulo 3.

Proposition 3.18.
Si p est un nombre premier impair et −1 est un carré modulo p, alors p ≡ 1 [4].

Démonstration. Comme p est impair, on peut écrire p sous la forme p = 2m + 1.


Soit a tel que a
2≡ −1 [p]. Alors a n'est pas divisible par p (sinon on aurait a2 ≡ 0 [p]),
donc on peut appliquer le petit théorème de Fermat à a :

1 ≡ ap−1 = a2m = (a2 )m ≡ (−1)m [p].

Si m est impair, alors 1 ≡ −1 [p] donc p | 2, ce qui est impossible.


Donc m est pair. On l'écrit sous la forme m = 2k . On en déduit que p = 4k + 1 est
bien congru à 1 modulo 4.

Remarque 3.19.
La réciproque est vraie mais un peu plus dicile à démontrer : si p ≡ 1 [4] alors −1
est un carré modulo p.

3.6 Nombres rationnels et irrationnels, développement décimal


Un nombre est dit décimal s'il a un nombre ni de chires après la virgule. Tout entier
est un nombre décimal. Le nombre 3/8 = 0, 375 est décimal.
Un nombre est dit rationnel s'il est le quotient de deux entiers. Par exemple,

83/70 = 1, 1857142857142857142 · · ·

est rationnel. On constate que son développement décimal est périodique : le motif 857142
se répète. Ceci s'explique par le fait que, lorsque l'on pose la division, il n'y a qu'un

26
nombre ni de restes possibles. Le motif commence à se répéter lorsque l'on tombe sur
un reste déjà obtenu précédemment.
Réciproquement, un nombre dont le développement décimal est périodique est ration-
nel. Sans le prouver formellement, expliquons-le sur un cas particulier.
Soit x = 3, 1212121212 . . ..
309
On a 100x = 312, 12121212 . . ., donc 99x = 100x − x = 309. Il vient x= 99 . Cette
103
fraction n'est pas irréductible, elle se simplie en
33 .

Exercice 3.17.
Ecrire le nombre 3, 157157157157 . . . sous forme d'une fraction irréductible.

Certains nombres n'ont pas de développement décimal périodique.

Proposition 3.20.

2 est irrationnel.

√ 2
Démonstration. Supposons par l'absurde que 2 = ab avec ab irréductible. On a 2 = ab2 ,
donc a2 = 2b2 . On regarde la 2-valuation : 2v2 (a) = v2 (a2 ) = v2 (2b2 ) = 1 + 2v2 (b). Or,
2v2 (a) est pair et 1 + 2v2 (b) est impair. Contradiction.

Plus généralement,

Exercice 3.18.
Soit n > 2. a est un entier positif qui n'est pas la puissance n-ième
Montrer que si

n
d'un entier, alors la racine n-ième de a (notée a) est un irrationnel.
√ √
N.B.
n
a est l'unique nombre réel positif tel que ( n a)n = a.

Exercice 3.19.
√ √
¶Montrer que 2+ 3 est irrationnel.

On peut montrer que π est irrationnel, mais les outils nécessaires pour le démontrer
dépassent largement le cadre de ce document.

27
4 Utilisation de factorisations
Le fait d'écrire un entier n sous la forme n = ab avec a > 2 et b > 2 permet d'une part
de voir que n n'est pas premier, et d'autre part d'utiliser les deux propositions suivantes :
Proposition 4.1.
Soient a et b deux entiers naturels premiers entre eux et x, m deux entiers naturels.
Si ab = xm , alors il existe des entiers y et z tels que a = y m et b = z m .

Démonstration. Ecrivons a = pα1 1 · · · pαr r .


Pour tout nombre
m
premier p, on a vp (ab) = vp (x ) = mvp (x), donc vp (ab) est un
multiple de m.
Comme a et b sont premiers entre eux, vp (a) et vp (b) ne peuvent pas être tous deux
non nuls.
vp (a) = 0 alors m divise évidemment vp (a).
Si
Si vp (b) = 0 alors vp (ab) = vp (a) + vp (b) = vp (a) donc m divise vp (a).
Dans tous les cas, m divise vp (a) pour tout p, donc pour tout j il existe γj tel que
αj = mγj .
γ1 γr m
Soit y = p1 · · · pr . On a a = y . On procède de même pour b.

Proposition 4.2.
Soient a et b deux entiers naturels. On suppose qu'il existe un nombre premier p et
un entier m tel que ab = pm . Alors a et b sont des puissances de p.

Démonstration. Un nombre premier divisant a doit diviser ab = pm , donc doit être égal
à p. Ainsi, p est le seul nombre premier divisant a. Par conséquent, a est une puissance
de p.

4.1 Identités remarquables


Voici quelques identités remarquables à mémoriser et qui sont fréquemment utiles dans
des problèmes d'arithmétique :

(a + b)2 = a2 + 2ab + b2
(a − b)2 = a2 − 2ab + b2
(a + b)3 = a3 + 3a2 b + 3ab2 + b3
(a − b)3 = a3 − 3a2 b + 3ab2 − b3
a2 − b2 = (a − b)(a + b)
a3 − b3 = (a − b)(a2 + ab + b2 )
a3 + b3 = (a + b)(a2 − ab + b2 )
an − bn = (a − b)(an−1 + an−2 b + an−3 b2 + · · · + bn−1 )
an + bn = (a + b)(an−1 − an−2 b + an−3 b2 − · · · + bn−1 ) si n impair

28
Les six premières égalités se vérient en développant. Montrons la septième :

(a − b)(an−1 + an−2 b + an−3 b2 + · · · + bn−1 )


= a(an−1 + an−2 b + an−3 b2 + · · · + bn−1 ) − b(an−1 + an−2 b + an−3 b2 + · · · + bn−1 )
= (an + an−1 b + an−2 b2 + · · · + abn−1 ) − (an−1 b + an−2 b2 + an−3 b3 + · · · + abn−1 + bn )
= an − bn

(tous les termes se sont simpliés excepté le premier et le dernier).

L'identité précédente étant vraie pour tout couple (a, b), elle est vraie pour le couple
(a, −b) donc

an − (−b)n = (a − (−b))(an−1 + an−2 (−b) + an−3 (−b)2 + · · · + (−b)n−1 ).

Si n est impair, on a (−b)n = −bn et (−b)n−1 = bn−1 , d'où la dernière égalité.

Comme cas particulier de la septième égalité, on trouve pour b=1 :

an − 1 = (a − 1)(1 + a + a2 + · · · + an−1 ).

Ceci étant vrai pour tout entier n, l'égalité est vraie pour n + 1 à la place de n, ce qui
donne an+1 − 1 = (a − 1)(1 + a + a2 + · · · + an ). En divisant par a − 1 (si a 6= 1), on
an+1 −1
obtient l'identité 1 + a + a2 + · · · + an = a−1 .

Exercice 4.1.
¶(Nombres premiers de Mersenne) Montrer que si a > 2 et n > 2 sont des entiers tels
quean − 1 est un nombre premier, alors a = 2 et n est premier.

Remarque : on constate que M2 = 3, M3 = 7, M5 = 31, M7 = 127, M11 = 2047,


M13 = 8191, M17 = 131071 et M19 = 524287 sont premiers. En revanche, M23 = 47 ×
178481 est composé. On ignore s'il existe une innité de nombres premiers de Mersenne.

Exercice 4.2.
¶(Nombres premiers de Fermat) Montrer que si 2n + 1 est premier, alors n est une
puissance de 2. (On montrera que n n'admet pas de diviseur impair.)

Remarque : on pose
k
Fk = 22 + 1. On constate que F0 = 3, F1 = 5, F2 = 17,
F3 = 257, F4 = 65537 sont premiers. En revanche, F5 = 641 × 6700417 est composé.
Actuellement, on ne connaît pas d'autre nombre premier de Fermat.

29
4.2 L'équation a2 − b2 = n
A n a2 − b2 = n d'inconnues a, b ∈ N.
xé, on considère l'équation
2 2
Rappelons que a − b = (a − b)(a + b). Comme a + b = (a − b) + 2b, les entiers a + b
et a − b ont la même parité. S'ils sont tous deux impairs, alors n est impair, et s'ils sont
tous deux pairs, alors n est divisible par 4.
Par conséquent, l'équation ne peut avoir de solution que si n est impair ou si n est
divisible par 4.
Supposons que n = (a − b)(a + b). Alors a − b est un diviseur de n. Notons-le d. On a
   
n 1 n 1 n
alors a − b = d et a + b = donc a = d + et a = d −
d 2 d 2 d . Il y a donc autant de
n
solutions que d'entiers naturels d tels que d et
d sont des diviseurs de n de même parité.
r
Ecrivons n = 2 m avec m impair. Si r = 0 alors n'importe quel diviseur de n convient,
j 0
et si r > 2 alors d est de la forme 2 d avec 1 6 j 6 r − 1 et d | m.
0
5 2 j 0
Par exemple, pour n = 2016 = 2 × 3 × 7, d est de la forme 2 d avec 1 6 j 6 4 et
0
d ∈ {1, 3, 9, 7, 21, 63}, ce qui donne 24 solutions en tout.

4.3 L'équation a2 + b2 = c2
Les solutions de cette équation s'appellent les triplets Pythagoriciens, car elles corres-
pondent aux triangles rectangles dont les côtés sont entiers. Le plus connu d'entre eux
est (3, 4, 5) puisque 32 + 42 = 52 .
On remarque tout d'abord que si (a, b, c) est solution, alors pour tout entier d, (da, db, dc)
est une solution. Par exemple, (6, 8, 10) est encore un triplet Pythagoricien. Ceci conduit
à poser la dénition suivante :

Dénition 4.3.
Une solution primitive est une solution telle que a, b, c sont premiers entre eux dans
leur ensemble.

Si (a, b, c) est une solution quelconque, posons d = P GCD(a, b, c). On peut écrire

 a = da0

b = db0
c = dc0

pour certains entiers a0 , b0 , c0 . En divisant l'équation a2 + b2 = c2 par d2 , il vient


(a0 )2 + (b0 )2 = (c0 )2 . De plus, a0 , b0 , c0 sont premiers entre eux dans leur ensemble, donc
(a0 , b0 , c0 ) est une solution primitive.
Par conséquent, le problème revient à chercher toutes les solutions primitives de l'équa-
tion.

Remarquons également que a et b sont premiers entre eux, car s'il y avait un nombre
2 2 2 2 2
premier p divisant a et b, alors p diviserait a +b , donc p diviserait c , ce qui entraînerait
que p divise c, et a, b, c ne seraient pas premiers entre eux.

30
De même, a et c, ainsi que b et c sont premiers entre eux. Autrement dit, a, b, c sont
premiers entre eux deux à deux.
Ainsi, a et b ne peuvent pas être tous les deux pairs.
Ils ne peuvent pas non plus être tous deux impairs, sinon a2 + b2 ≡ 1 + 1 = 2 [4] et on
a vu que 2 n'est pas un carré modulo 4.
Quitte à échanger a et b, on peut supposer que a est impair et que b est pair.

c2 −a2
On a ( 2b )2 = 4 = ( c−a c+a
2 )( 2 ).
c−a c+a
(Comme a et c sont impairs,
2 et 2 sont bien des entiers.)
c−a c+a
Si un nombre p divise à la fois
2 et 2 , il divise leur somme et leur diérence, c'est-
à-dire que p divise c et a. Or, a et c sont premiers entre eux, donc p = 1. Par conséquent,
c−a c+a
2 et 2 sont premiers entre eux.
Or, leur produit est un carré, donc d'après la Proposition 1 ce sont tous deux des
carrés. Ainsi, il existe u et v tels que

c+a
= u2

2
c−a
2 = v2
En faisant la somme des deux égalités, on obtient c2 = u2 + v 2 . En faisant la diérence,
on trouve a
2 = u2 − v 2 , et en faisant le produit, on voit que ( 2b )2 = u2 v 2 donc b = 2uv .
Conclusion : quitte à échanger a et b, les solutions primitives sont données par les
formules
 a = u2 − v 2

b = 2uv
c = u2 + v 2

pour certains entiers u et v.


On vérie qu'on a bien

(u2 − v 2 )2 + (2uv)2 = (u2 + v 2 )2 .

4.4 Exercices supplémentaires


Exercice 4.3.
¶¶Résoudre dans les entiers naturels : x3 − y 3 = 24.

Exercice 4.4.
¶¶Trouver tous les nombres premiers tels qu'il existe des entiers naturels x et y
2 2
vériant x(y − p) + y(x − p) = 5p.

Exercice 4.5.
¶¶Déterminer les entiers naturels m et n tels que 2n − 3m = 1. Même chose pour
m n
l'équation 3 − 2 = 1.

31
Exercice 4.6.
¶¶Déterminer tous les entiers a, b, c > 0 tels 2a 3b + 9 = c2 .

32
Concepts de base en arithmétique :
solutions des exercices.

Thomas Budzinski

1 Préliminaires
Pas d'exercices.

2 Divisibilité
Solution 2.1.

Il existe u et v tels que b = au et d = cv donc bd = (ac)(uv) donc ac | bd.

Solution 2.2.

Si n | n + 7, comme n | n, alors n | (n + 7) − n = 7 donc n vaut 1 ou 7. Dans ces deux


cas, on voit que n divise bien n + 7.

Solution 2.3.

Si n2 + 1|n, alors n = 0 ou n2 + 1 ≤ n. Mais si n > 0, alors n2 + 1 > n2 ≥ n, ce qui


est absurde, donc seul n = 0 est solution.

Solution 2.4.

Si n est pair, alors 2|n et n|n(n + 1) donc n(n + 1) est pair. Si n est impair, alors
2|n + 1 et n + 1|n(n + 1) donc n(n + 1) est pair.

Solution 2.5.

n, n + 1 et n + 2 sont trois entiers consécutifs donc un des trois est divisible par 3,
donc n(n + 1)(n + 2) est divisible par 3 : on peut écrire n(n + 1)(n + 2) = 3u. De
plus, n(n + 1) est pair par l'exercice précédent donc n(n + 1)(n + 2) aussi, donc 3u
est pair. Or, le produit de deux nombres impairs est impairs donc u doit être pair,
donc 6 = 3 × 2 | 3u, d'où le résultat.

1
Solution 2.6.

On a c = −an2 − bn. Or, n | −an2 et n | −bn donc n | c.

Solution 2.7.

On raisonne comme dans l'exercice précédent : 3 = −n5 + 2n4 + 7n2 + 7n et n divise


le membre de droite donc n|3 donc n vaut −3, −1, 1 ou 3. En testant ces quatre
valeurs, on trouve que les seules solutions sont −1 et 3.

Solution 2.8.

On a a | n(n + 2) = n2 + 2n donc a | n2 + n + 5 − (n2 + 2n) = −n + 5. On en déduit


que a | (n + 2) + (−n + 5) = 7, puis que a = 1 ou a = 7.

Solution 2.9.

364 = 154 × 2 + 56
154 = 56 × 2 + 42
56 = 42 × 1 + 14
42 = 14 × 3 + 0

Solution 2.10.

Pour tout entier d, d divise 10100 et 10121 + 10813 + 10 si et seulement si d divise leur
P GCD. On calcule donc ce PGCD avec l'alogrithme d'Euclide :

10121 + 10813 + 10 = 10100 × (1021 + 10713 ) + 10


10100 = 10 × 1099 + 0

Le PGCD vaut 10 donc les diviseurs communs sont les diviseurs de 10, soit −10, −5,
−2, −1, 1, 2, 5 et 10. Il y en a 8.

2
Solution 2.11.

D'après la proposition précédente pour q = 1, b = 109 et r = 5 on a :


P GCD(109 + 5, 109 ) = P GCD(109 , 5) = 5

car 5 | 109 .

Solution 2.12.

131 est premier (la vérication peut être un peu laborieuse...).


221 = 13 × 17 n'est pas premier.

Solution 2.13.

1) Si p ≥ 3 est premier, alors il est impair, donc p − 1 et p + 1 sont deux nombres


pairs consécutifs, et un des deux est divisible par 4. Si c'est p − 1, on peut écrire
p − 1 = 4k donc p = 4k + 1. Si c'est p + 1 on peut écrire p = 4k − 1.
2) Supposons par l'absurde qu'il n'existe qu'un nombre ni de nombres premiers
de la forme 4k − 1 notés p1 < p2 < · · · < pr avec p1 = 3, p2 = 7 etc. Soit
n = 4(p1 . . . pr ) − 1 : on a n ≥ 4p1 − 1 ≥ 11 > 2 donc n est un produit de
nombres premiers. Or, un produit de nombres de la forme 4k + 1 est de la forme
4k + 1 (la vérication est facile), donc n admet un facteur premier de la forme
4k − 1, qui doit être égal à un des pi . Mais alors pi divise 4p1 . . . pr donc pi
divise 1, d'où la contradiction.

Solution 2.14.

1) On a P GCD(n, n + 2) = P GCD(n, 2) donc n et n + 2 sont premiers entre eux


si et seulement si n et 2 sont premiers entre eux ssi n est impair.
2) On a n2 − 1 = (n + 1)(n − 1) et (n2 − 2n + 1) = (n − 1)2 donc n − 1 est un
diviseur commun. Pour que les deux nombres soient premiers entre eux, il faut
donc que n − 1 soit égal à −1 ou 1, donc que n = 0 ou n = 2. Dans ces deux
cas, n2 − 2n + 1 = 1 donc les deux nombres sont bien premiers entre eux.

Solution 2.15.

On reprend les restes successifs de l'exercice 9 :


56 = 364 − 154 × 2
42 = 154 − 56 × 2 = 154 × 5 − 364 × 2
14 = 56 − 42 = 364 × 3 − 154 × 7

3
Solution 2.16.

2x + 1 est impair donc est premier avec 8 (car les seuls diviseurs de 8 sont 1, 2, 4 et
8) donc d'après le lemme de Gauss, si 2x + 1 | 8y alors 2x + 1 | y .

Solution 2.17.

7 et 9 sont premiers entre eux et l'algorithme d'Euclide nous donne la solution parti-
culière (u0 , v0 ) = (−3, −4). D'après ce qui précède, un couple (u, v) est donc solution
ssi il existe k ∈ Z tel que u = 7k − 3 et v = 9k − 4.

Solution 2.18.

On a P GCD(16, 26) = 2. Si n est impair, l'équation n'a donc pas de solution. Si n


est pair, on pose n = 2n0 et on est ramené à l'équation 8x + 13y = n0 avec 8 et 13
premiers entre eux.
De plus, l'algorithme d'Euclide nous donne pour n0 = 1 la solution (x0 , y0 ) = (−8, 5)
donc pour tout n0 on a une solution particulière (−8n0 , 5n0 ). Un couple (x, y) est donc
solution ssi il existe k ∈ Z tel que x = 13k − 8n0 et y = −8k + 5n0 .

Solution 2.19.

18100 = (2 × 32 )100 = 2100 × 3200 donc v3 (18100 ) = 200.

Solution 2.20.

2100 + 2200 = 2100 × (1 + 2100 ) où le deuxième facteur est impair, donc n'admet pas
2 comme facteur premier. On a donc v2 (2100 + 2200 ) = 100.

Solution 2.21.

1) Comptons combien de facteurs sont divisibles par 7 : il y en a b 100 7 c = 14. De


plus, parmi ces 14 facteurs, 49 et 98 sont divisibles par 49 = 7 , et aucun des
2

facteurs n'est divisibles par 73 = 243. On a donc v7 (n) = 14 + 2 = 16.


2) Pour commencer, le nombre de zéros dans l'écriture décimale de n est le plus
grand k tel que 10k | n. Or, 10k | n ssi 2k | n et 5k | n (d'après le lemme de
Gauss) donc on recherche le plus grand k tel que 2k et 5k divisent n. Il s'agit
par dénition de min v2 (n), v5 (n) .


Calculons maintenant v5 (n) : il y a 20 facteurs divisibles par 5 dont 4 divisibles


par 52 = 25 donc v5 (n) = 20 + 4 = 24.
De plus, il y a 50 facteurs pairs donc v2 (n) ≥ 50 > 24 donc la plus petite des
deux valuations vaut 24 : il y a 24 zéros à la n de l'écriture décimale de n.

4
Solution 2.22.

Le premier nombre a une valuation 5-adique non nulle mais pas le second (car son
écriture décimale termine par un 7).

Solution 2.23.

Si b = 0 alors on a bien a | b. Sinon, a 6= 0 car si a était nul on aurait 0 | b2 donc


b2 = 0 et b = 0. On peut donc supposer a, b 6= 0.
L'hypothèse a2 | b2 revient alors à dire que pour tout p premier, 2vp (a) ≤ 2vp (b) donc
vp (a) ≤ vp (b) pour tout p d'où a | b.

Solution 2.24.

On a vp (a2 ) ≥ 1 mais vp (a2 ) = 2vp (a) est pair donc vp (a2 ) ≥ 2 et p2 | a2 .

Solution 2.25.

Pour tout p premier, vp (ab) = vp (a) + vp (b). De plus, vp (P GCD(a,  b)) =


et vp (P P CM (a, b)) donc

min vp (a), vp (b) = max vp (a), vp (b)
vp (P GCD(a, b)P P CM (a, b)) = vp (a) + vp (b) = vp (ab) pour tout p premier.
ab et P GCD(a, b)P P CM (a, b) ont donc la même décomposition en facteurs premiers
donc sont égaux.

Solution 2.26.

On peut écrire x = 8x0 et y = 8y 0 . x + y = 128 donne alors x0 + y 0 = 128 8 = 16 et


l'exercice précédent donne 8x × 8y = 8 × 440 donc x y = 8 = 55 donc x0 et y 0
0 0 0 0 440

valent 1, 5, 11 ou 55. Comme x0 + y 0 = 16, ils doivent valoir 5 et 11 donc les solutions
sont (x, y) = (40, 88) et (88, 40).

5
Solution 2.27.

Si p = 2k+1 − 1 est premier, alors la décomposition du nombre n qui nous intéresse


est 2k p : il s'agit des 2i avec i entre 0 et k et des 2i p avec i entre 0 et k. La somme
des premiers vaut 2k+1 − 1 = p et la somme des seconds (2k+1 − 1)p = p2 . La somme
des diviseurs de n vaut donc p + p2 = p(p + 1) = 2k+1 (2k+1 − 1) = 2n.
Pour la réciproque, on notera s(n) la somme des diviseurs d'un entier n. Soit n
un nombre parfait pair : on peut écrire n = 2k m avec m impair et k = v2 (n). Se
donner un diviseur de n revient alors à se donner un diviseur de 2k (c'est-à-dire un
2i avec i entre 0 et k ) et un diviseur de m et à faire leur produit. En sommant
sur tous les choix possibles, on en déduit s(n) = s(2k )s(m) = (2k+1 − 1)s(m), soit
2k+1 m = (2k+1 − 1)s(m). Or, 2k+1 − 1 est premier avec 2k+1 donc d'après le lemme de
Gauss il divise m. Notons M = 2k+1 m
−1
: si M > 1, alors les nombres de la forme 2i ,
2i M et 2i m sont tous distaincts et sont des diviseurs de n donc s(n) vaut au moins
leur somme, soit :
m
s(n) ≥ (2k+1 −1)(m+M +1) = (2k+1 −1)(m+ )+1 = 2k+1 m−m+m+2k+1 −1
2k+1−1

donc s(n) > 2n et n n'est pas parfait. On doit donc avoir M = 1 donc m = 2k+1 − 1.
La somme des diviseurs de la forme 2i ou 2i m vaut alors exactement 2n et pour qu'il
n'y ait pas d'autre diviseur il faut que m soit premier.

Solution 2.28.

1000000 = 106 = 26 × 56 admet (6 + 1)(6 + 1) = 49 diviseurs.

Solution 2.29.

Soit n tel que τ (n) = 7 : on écrit pα1 1 . . . pαr r sa décomposition en nombres premiers :
on a (α1 + 1) . . . (αr + 1) = 7 mais 7 est premier donc r = 1 et α1 = 6 donc n est de la
forme p6 avec p premier. Si n ≤ 1000, les seules possibilités sont 26 = 64 et 36 = 729.

Solution 2.30.

On peut regrouper les diviseurs de n par paires telles que le produit de chaque pair
soit égal à n. Par exemple, pour 12 on regroupe 1 avec 12, 2 avec √ 6 et 3 avec 4. Si
(a, b) est une de ces paires avec a ≤ b, alors n = ab ≥ a2 donc a ≤ n : toute paire
√ √ √
contient un élément ≤ n donc il y a au plus n paires, soit 2 n diviseurs.

Remarque 2.1.

Cet argument des paires montre aussi que si on veut √vérier qu'un nombre n est
premier, il sut de vérier qu'il n'a pas de diviseurs ≤ n à part 1.

6
Solution 2.31.

480 = 25 × 3 × 5 donc 480 a (5 + 1)(2 + 1)(2 + 1) = 54 diviseurs dont 1, 2, 3, 4, et 5.


Le nombre d'arbres par rangée doit être un de ces diviseurs mais pas 1, 2, 3, 4 ou 5.
Le jardinier a donc 54 − 5 = 49 possibilités.

Solution 2.32.

Écrivons N = pα1 1 . . . pαr r : on a (α1 + 1) . . . (αr + 1) = 15 = 3 × 5 avec 3 et 5 premiers


donc r ≤ 2. Mais 2 et 3 divisent N donc r = 2, p1 = 2 et p2 = 3. De plus, on a
α1 + 1 = 5 et α2 + 1 = 3 ou l'inverse. Mais dans le premier cas, 8 = 23 | N donc seul
le deuxième marche, d'où N = 22 × 34 = 324.

Solution 2.33.

n a un nombre impair de diviseurs ssi tous les αi + 1 sont impairs ssi tous les αi sont
pairs. C'est le cas si n est un carré. Réciproquement, si tous les αi sont pairs, on peut
écrire αi = 2βi et alors n = pβ1 1 . . . pβr r 2 .


Solution 2.34.

Si m et n sont premiers entre eux, on écrit m = pα1 1 . . . pαr r et n = q1β1 . . . qsβs : les pi
et les qj sont tous distincts, de sorte que :
τ (mn) = (α1 + 1) . . . (αr + 1)(β1 + 1) . . . (βs + 1) = τ (m)τ (n)

Si τ (mn) = τ (m)τ (n), écrivons n = pα1 1 . . . pαr r et m = pβ1 1 . . . pβr r (il se peut qu'un pi
divise m mais pas n, auquel cas on prend βi = 0). On a alors :
(α1 + β1 + 1) . . . (αr + βr + 1) = (α1 + 1) . . . (αr + 1)(β1 + 1) . . . (βr + 1) . . .

ce qui se réécrit :
α1 + β 1 + 1 αr + βr + 1
... =1
(α1 + 1)(β1 + 1) (αr + 1)(βr + 1)

Mais pour tout i, on a (ααi +1)(β


i +βi +1
i +1)
αi βi
= 1 − (αi +1)(β i +1)
≤ 1. Pour que le produit soit
égal à 1, il faut donc que chaque terme soit égal à 1 et donc que pour tout i on ait
αi βi = 0. Autrement dit, il n'existe aucun i tel que pi divise à la fois m et n, donc m
et n sont premiers entre eux.

7
3 Congruences
Solution 3.1.

p = 2 convient. Si p ≥ 3, on raisonne modulo 2 : si p ≡ 0 (mod n) alors p est pair et


si p ≡ 1 (mod n) alors p + 1 est pair. Mais p > 2 et p + 1 > 2 donc ils ne peuvent pas
tous deux être premiers. 2 est donc la seule solution.

Solution 3.2.

p = 2 ne convient pas et p = 3 convient. Si p ≥ 4 on raisonne modulo 3 : si p ≡ 0


(mod 3) alors 3 | p. Si p ≡ 1 (mod 3) alors 3 | p + 2. Si p ≡ 2 (mod 3) alors 3 | p + 4.
Un des trois nombres est donc divisible par 3 et ne peut valoir 3 donc les trois nombres
ne sont pas tous premiers, donc 3 est la seule solution.

Solution 3.3.

48767621 est divisible ssi 4 + 8 + 7 + 6 + 7 + 6 + 2 + 1 = 41 l'est, donc il n'est pas


divisible par 9.

Solution 3.4.

98473092 est divisible par 11 ssi −9 + 8 − 4 + 7 − 3 + 0 − 9 + 2 = −8 l'est, donc il


n'est pas divisible par 11.

Solution 3.5.

3 + 6 + 4 + 5 = 18 est divisible par 9 donc 9 divise 3645. 5 le divise aussi et 9 et 5


sont premiers entre eux donc 9 × 5 = 45 divise 3645.

Solution 3.6.

2 ne divise pas 127413 donc le PGCD est impair.


1+9+3+1+1+6 = 21 est divisible par 3 donc 193116 aussi. 1+2+7+4+1+3 = 18
est divisible par 3 donc 127413 aussi, donc leur PGCD est divisible par 3.
1 + 9 + 3 + 1 + 1 + 6 = 21 n'est pas divisible par 9 donc 193116 non plus et le PGCD
non plus.
−1+9−3+1−1+6 = 11 est divisible par 11 donc 193116 aussi. −1+2−7+4−1+3 = 0
est divisible par 11 donc 127413 aussi, et le PGCD aussi.
Le PGCD est divisible par 3 et 11 qui sont premiers entre eux, donc il est divisible
par 33 d'après le lemme de Gauss.
Le PGCD n'est pas divisible par 9 donc il n'est pas divisible par 99.

8
Solution 3.7.

Le nombre est divisible par 33 ssi il est divisible par 3 et 11. Il est divisible par 11 ssi
−2 + 7 − x + 8 − 5 + y = 13 − x + y l'est ssi x − y ≡ 2 (mod 11) ssi x − y vaut 2 ou
−9 (car x et y sont des chires). Cela laisse comme possibilités (2, 0), (3, 1), (4, 2),
(5, 3), (6, 4), (7, 5), (8, 6), (9, 7) et (0, 9).
Il est divisible par 3 ssi 2 + 7 + x + 8 + 5 + y = 22 + x + y l'est ssi x + y ≡ 2 (mod 3).
Les solutions sont donc (2, 0), (5, 3) et (8, 6).

Solution 3.8.

Seuls 1, 3, 5 et 7 sont impairs donc premiers avec 8, donc ce sont les inversibles
modulo 8. De plus, on a 1 × 1 ≡ 1 (mod 8), 3 × 3 ≡ 9 ≡ 1 (mod 8), 5 × 5 ≡ 25 ≡ 1
(mod 8) et 7 × 7 ≡ 49 ≡ 1 (mod 8) donc ces nombres sont tous égaux à leur inverse
modulo 8.

Solution 3.9.

On a besoin d'une relation de Bézout entre 37 et 53. On applique donc l'algorithme


d'Euclide :
53 = 37 × 1 + 16
37 = 16 × 2 + 5
16 = 5 ×3+ 1

Ainsi :
16 = 53 × 1 − 37 × 1
5 = 37 − 16 × 2 = 37 × 3 − 53 × 2
1 = 16 − 5 × 3 = 53 × 7 − 37 × 10

On a donc 37 × 10 ≡ 1 (mod 53) donc l'inverse de 37 modulo 53 est 10.

9
Solution 3.10.

a) 611 + n + 2 ≡ (−1)11 + 2 + n ≡ n + 1 (mod 7) donc ce sont les n congrus à −1


modulo 7.
b) On a 70543250 ≡ (70543210 )5 ≡ 15 ≡ 1 (mod 11), en utilisant à l'avant-dernière
étape le petit théorème de Fermat, donc le reste vaut 1.
c) 56n + 5n + 2 ≡ 1 + 5n + 2 ≡ 5n + 3 (mod 7). Or, si n = 6k + 1 on trouve
5n + 3 ≡ 56k × 5 + 3 ≡ 1 (mod 7). De même, si n = 6k + 2 on obtient 0
(mod 7). Si n = 6k + 3 on obtient 2 (mod 7). Si n = 6k + 4 on obtient 5
(mod 7). Si n = 6k + 5 on obtient 6 (mod 7) et si n = 6k on obtient 4 (mod 7).
Les solutions sont donc les n congrus à 2 modulo 6.
d) Pour tout n entier 81n5 − 45n3 + 4n ≡ n5 − n ≡ 0 (mod 5) en utilisant le petit
théorème de Fermat à la n, donc tous les entiers n marchent.

Solution 3.11.

On étudie les puissances de 7 modulo 10 : 72 ≡ 9 (mod 10), 73 ≡ 3 (mod 10) et


74 ≡ 1 (mod 10). Il faut donc s'intéresser à l'exposant modulo 4 : 39 ≡ (−1)9 ≡ −1
(mod 4) donc on peut poser 39 = 4k + 3, de sorte que :
9
73 ≡ (74 )k × 73 ≡ 1k × 3 ≡ 3 (mod 10)

donc le dernier chire est 3.

Solution 3.12.

2712 = (33 )12 = 336 qui est divisible par 3 et 9 mais pas par 5, 7 et 11.

Solution 3.13.

Corrigé dans le cours.

Solution 3.14.

On a :
02 ≡0 (mod 5)
12 ≡1 (mod 5)
22 ≡4 (mod 5)
32 ≡4 (mod 5)
42 ≡1 (mod 5)
donc les carrés modulo 5 sont 0, 1 et 4. De même, on trouve que les carrés modulo 8
sont 0, 1 et 4.

10
Solution 3.15.

Non. On raisonne modulo 8 : si a, b et c sont solutions, alors a2 + b2 + c2 ≡ 7 (mod 8)


donc au moins un des carrés est impair (a par exemple) donc congru à 1 modulo 8,
ce qui laisse b2 + c2 ≡ 6 (mod 8). Comme b2 est congru à 0, 1 ou 4, c2 doit donc être
congru à 6, 5 ou 2, ce qui est impossible.

Solution 3.16.

Oui : 102 ≡ 100 ≡ −3 (mod 103).

Solution 3.17.

Notons x le nombre considéré : 1000x = 3157, 157157 · · · = 3154 + x donc x = 999 ,


3154

qui est irréductible (algorithme d'Euclide par exemple).

Solution 3.18.

Si n
a = xy , alors xn = ay n donc pour tout p, nvp (x) = vp (a) + nvp (y) donc vp (a) =
n(vp (x) − vp (y)) est divisible par n pour tout p, donc a est la puissance n-ième d'un
nombre entier, ce qu'on avait supposé faux.

Solution 3.19.
√ √ √
Si 2+ 3 = xy avec x et y premiers entre eux, alors x= y 2 (2+3+2 6) soit x2 −5y 2 =
2


2y 2 6 donc (x2 − 5y 2 )2 = 24y 4 soit x4 − 10x2 y 2 + y 4 = 0. Si p est un diviseur premier
de x, alors il divise les deux premiers termes donc il divise y 4 donc p divise y , ce qui
est absurde car on a supposé x et y premiers entre eux. x n'a √ donc pas
√ de diviseur
premier donc x = 1 ou x = −1 et de même pour y . On a donc 2 + 3 égal à 1 ou
à −1, ce qui est absurde.

4 Utilisation de factorisations
Solution 4.1.

On a an − 1 = (a − 1)(an−1 + an−2 + · · · + 1. Le second facteur est toujours strictement


plus grand que 1 donc si an − 1 est premier, le premier facteur vaut 1, soit a = 2.
Si de plus n n'est pas premier, écrivons n = dd0 avec d 6= 1, n : on a 2n − 1 =
(2d − 1)(2d(d −1) + 2d(d −2) + · · · + 1. Le premier facteur est plus grand que 1 car d > 1
0 0

et le second l'est car d0 > 1 donc il n'y a pas que le dernier terme, donc si n n'est pas
premier, an − 1 ne peut pas l'être.

11
Solution 4.2.

Si n n'est pas une puissance de 2, il admet un diviseur impair d. En posant n = dd0


on obtient 2n + 1 = (2d + 1)(2d (d−1) − 2d (d−2) + ... − 2d + 1), où les deux facteurs
0 0 0 0

sont strictement supérieurs à 1, donc 2n + 1 ne peut pas être premier.

Solution 4.3.

L'équation se réécrit (x − y)(x2 + xy + y 2 ) = 24 donc si (x, y) est solution alors


x2 + xy + y 2 | 24. De plus, x ≥ y . Si y ≥ 3 alors x2 + xy + y 2 ≥ y 2 + y 2 + y 2 ≥ 27
ce qui est impossible. y vaut donc 0, 1 ou 2. Si y = 0 on obtient x3 = 0 qui n'a pas
de solution. Si y = 1 on obtient x3 = 25 qui n'a pas de solution. Si y = 2 on obtient
x3 = 32 qui n'a pas de solution. L'équation n'a donc pas de solution.

Solution 4.4.

Solution 4.5.

L'équation se réécrit (y + x)(xy − p) = 5p donc x + y peut valoir 1, 5, p et 5p. Quitte


à échanger x et y , on suppose x ≤ y .
Si x + y = 1 alors par exemple x = 0 et y = 1 ce qui donne 1 − p = 5p, absurde.
Si x + y = 5, alors xy − p = p donc xy = 2p. (x, y) peut valoir (0, 5), (1, 4) ou (2, 3).
Les deux dernières donnent des solutions pour p = 2 et p = 3.
Si x + y = p alors xy − p = 5 soit xy − x − y = 5 donc (x − 1)(y − 1) = 6. Les valeurs
possibles de (x, y) sont (2, 7) et (3, 4). La première donne p = 9 qui n'est pas premier.
La seconde donne une solution pour p = 7.
Si x + y = 5p alors xy − p = 1 soit 5xy + 1 = x + y . Si x, y ≥ 2 alors xy > x, y donc
5xy + 1 > x + y donc on a x = 1 ou x = 0. Si x = 1 alors 5y + 1 = y + 1 et y = 0,
absurde. Si x = 0, on obtient y = 1 et 5p = 1, absurde.
Les p qui marchent sont donc 2, 3 et 7.

Solution 4.6.

Pour n = 0 il n'y a pas de solution. Pour n = 1, m = 0 marche. Pour n = 2,


m = 1 marche. Si n ≥ 3, on raisonne modulo 8 : on a 8 | 2n donc 3m ≡ 2n − 1 ≡ 7
(mod 8). Mais si m = 2k alors 3m ≡ 9k ≡ 1k ≡ 1 (mod 8), et si m = 2k + 1 alors
3m ≡ 9k × 3 ≡ 3 (mod 8). Il n'y a donc pas de solution pour n ≥ 3.
Passons la seconde équation : si n = 0 il n'y a pas de solution. Si n = 1 alors m = 1
est solution. Pour n ≥ 2, on a 3m ≡ 2n + 1 ≡ 1 (mod 4) donc m doit être pair. On
pose donc m = 2m0 . L'équation se réécrit 32m − 1 = 2n , soit (3m + 1)(3m − 1) = 2n .
0 0 0

Les nombres 3m + 1 et 3m − 1 sont donc deux puissances de 2 distantes de 2, donc


0 0

valent 2 et 4, ce qui donne m0 = 1 donc m = 2 et n = 3.

12
Solution 4.7.

L'équation se réécrit 2a 3b = c2 − 9 = (c + 3)(c − 3). Si b = 0, alors c − 3 et c + 3


sont deux puissances de 2 espacées de 6 donc valent 2 et 8, ce qui donne la solution
(4, 0, 5). Si a = 0, alors c − 3 et c + 3 sont deux puissances de 3 espacées de 6 donc
valent 3 et 9, ce qui donne la solution (0, 3, 6).
On suppose donc a, b ≥ 1 : (c + 3)(c − 3) est divisible par 6 et c + 3 ≡ c − 3 (mod 6)
donc tous deux sont divisibles par 6. On écrit c−3 = 6k et c+3 = 6(k +1). L'équation
se réécrit 2a 3b = 36k(k + 1) donc a, b ≥ 2 et 2a−2 3b−2 = k(k + 1). Si k = 1, on trouve
2a−2 3b−2 = 2 donc a = 3 et b = 2 et on trouve la solution (3, 2, 9).
Si k > 1, alors k et k + 1 sont premiers entre eux donc un seul est divisible par 2 et
l'autre par 3, donc a−2 et b−2 vérient une des deux équations de l'exercice précédent.
En utilisant l'exercice précédent, on trouve comme seules solutions (4, 3, 21), (3, 2, 9)
(déjà vue plus haut) et (5, 4, 51).
On a donc nalement 5 triplets solutions : (4, 0, 5), (0, 3, 6), (3, 2, 9), (4, 3, 21) et
(5, 4, 51).

13
Cours d’arithmétique
Première partie
Pierre Bornsztein
Xavier Caruso
Pierre Nolin
Mehdi Tibouchi

Décembre 2004

Ce document est la première partie d’un cours d’arithmétique écrit pour les élèves pré-
parant les olympiades internationales de mathématiques. Le plan complet de ce cours est :

1. Premiers concepts
2. Division euclidienne et conséquences
3. Congruences
4. Équations diophantiennes
5. Structure de Z/nZ
6. Sommes de carrés
7. Polynômes à coefficients entiers
8. Fractions continues
Cette première partie traite les quatre premiers chapitres. Les quatre derniers chapitres
forment quant à eux la deuxième partie de ce cours.
Contrairement à la seconde partie, cette première partie se veut le plus élémentaire
possible. Les notions abstraites, souvent plus difficiles à assimiler, mais qui clarifient les idées
lorsqu’elles sont comprises, ne sont évoquées que dans la seconde partie. Nous conseillons
au lecteur de bien maı̂triser ce premier tome avant de passer à la lecture du second.
Les notions et les théorèmes introduits ici sont généralement tout à fait suffisants pour
traiter les exercices proposées aux olympiades internationales de mathématiques.
Vous trouverez à la fin de chaque chapitre une série d’exercices de difficulté variable mais
indiquée par des étoiles1 . Toutes les solutions sont rassemblées à la fin du document.
Nous vous souhaitons bon apprentissage et bonne lecture.

1
Plus nous avons jugé l’exercice difficile, plus le nombre d’étoiles est important.

1
Liste des abbrévations :
AMM American Mathematical Monthly
APMO The Asian Pacific Mathematics Olympiad
CG Concours général
OIM Olympiades Internationales de Mathématiques
SL Short List
TDV Tournoi Des Villes

Liste des notations :


∅ ensemble vide
N ensemble des entiers naturels (positifs ou nuls)
N? ensemble des entiers naturels strictement positifs
Z ensemble des entiers relatifs
Q ensemble des nombres rationnels
R ensemble des nombres réels
P
Q symbôle de sommation2
symbôle de produit3
a|b a divise b
[x] partie entière de x
{x} partie décimale de x
pgcd plus grand commun diviseur
a∧b pgcd (a, b)
ppcm plus petit commun multiple
a∨b ppcm (a, b)
a ≡ b (mod N ) a est congru à b modulo N
p un nombre premier
vp (n) valuation p-adique de n
d(n) nombre de diviseurs positifs de n
σ(n) somme des diviseurs positifs de n
ϕ fonction indicatrice d’Euler
sb (n) somme des chiffres de n en base b
π (n) nombre de nombres premiers inférieurs ou égaux à n
an . . . a 0 b écriture en base b
n! factorielle de n : n! = 1 × 2 × · · · × n
Ckn coefficient binomial : Ckn = k!(n−k)!
n!

un ∼ v n les suites (un ) et (vn ) sont équivalentes

2
Une somme indexée par l’ensemble vide est égale à 0.
3
Un produit indexé par l’ensemble vide est égale à 1.

2
Table des matières
1 Premiers concepts 4
1.1 Divisibilité . . . . . . . . . . . . . . . . . . . . . . . . . . . . . . . . . . . . . 4
1.2 Nombres premiers . . . . . . . . . . . . . . . . . . . . . . . . . . . . . . . . . 9
1.3 Valuation p-adique . . . . . . . . . . . . . . . . . . . . . . . . . . . . . . . . 12
1.4 Quelques fonctions arithmétiques . . . . . . . . . . . . . . . . . . . . . . . . 14
1.5 Nombres rationnels . . . . . . . . . . . . . . . . . . . . . . . . . . . . . . . . 15
1.6 Exercices . . . . . . . . . . . . . . . . . . . . . . . . . . . . . . . . . . . . . . 17

2 Division euclidienne et conséquences 24


2.1 Division euclidienne et décomposition en base b . . . . . . . . . . . . . . . . 24
2.2 Algorithme d’Euclide . . . . . . . . . . . . . . . . . . . . . . . . . . . . . . . 27
2.3 Algorithme d’Euclide étendu et théorème de Bézout . . . . . . . . . . . . . . 28
2.4 Lemme de Gauss et conséquences . . . . . . . . . . . . . . . . . . . . . . . . 29
2.5 Exercices . . . . . . . . . . . . . . . . . . . . . . . . . . . . . . . . . . . . . . 32

3 Congruences 37
3.1 Définition, premières propriétés . . . . . . . . . . . . . . . . . . . . . . . . . 37
3.2 Critères de divisibilité . . . . . . . . . . . . . . . . . . . . . . . . . . . . . . 38
3.3 Ordre d’un élément . . . . . . . . . . . . . . . . . . . . . . . . . . . . . . . . 39
3.4 Théorème chinois . . . . . . . . . . . . . . . . . . . . . . . . . . . . . . . . . 40
3.5 Congruences modulo p . . . . . . . . . . . . . . . . . . . . . . . . . . . . . . 43
3.6 Congruences modulo pn . . . . . . . . . . . . . . . . . . . . . . . . . . . . . 45
3.7 Coefficients binomiaux . . . . . . . . . . . . . . . . . . . . . . . . . . . . . . 47
3.8 Exercices . . . . . . . . . . . . . . . . . . . . . . . . . . . . . . . . . . . . . . 51

4 Équations diophantiennes 56
4.1 Quelques réflexes . . . . . . . . . . . . . . . . . . . . . . . . . . . . . . . . . 56
4.2 Utilisation des congruences . . . . . . . . . . . . . . . . . . . . . . . . . . . . 59
4.3 Descente infinie . . . . . . . . . . . . . . . . . . . . . . . . . . . . . . . . . . 62
4.4 Équations de degré 2 . . . . . . . . . . . . . . . . . . . . . . . . . . . . . . . 65
4.5 Équations de degré 3 . . . . . . . . . . . . . . . . . . . . . . . . . . . . . . . 68
4.6 Exercices . . . . . . . . . . . . . . . . . . . . . . . . . . . . . . . . . . . . . . 70

5 Corrigé des exercices 75


5.1 Exercices de « Premiers concepts » . . . . . . . . . . . . . . . . . . . . . . . 75
5.2 Exercices de « Division euclidienne et conséquences » . . . . . . . . . . . . . 103
5.3 Exercices de « Congruences » . . . . . . . . . . . . . . . . . . . . . . . . . . 118
5.4 Exercices de « Équations diophantiennes » . . . . . . . . . . . . . . . . . . . 143

3
1 Premiers concepts
Cette section, comme son nom l’indique, présente le concept de base de l’arithmétique,
à savoir la divisibilité. On introduit ensuite les nombres premiers ce qui permet d’énoncer le
théorème fondamental de l’arithmétique (c’est-à-dire la décomposition en facteurs premiers)
dans lequel les nombres premiers jouent le rôle de briques élémentaires pour la fabrication
des nombres.

1.1 Divisibilité
Définition 1.1.1 Si a et b sont deux entiers, on dit que a divise b, ou que b est divisible
par a, s’il existe un entier q tel que b = aq. On dit encore que a est un diviseur de b, ou que
b est un multiple de a. On le note a|b.

Propriétés
a
☞ Si a et b sont deux entiers avec b 6= 0, b divise a si et seulement si la fraction b
est un
entier.
☞ Tous les entiers divisent 0, et sont divisibles par 1.
☞ Un entier n est toujours divisible par 1, −1, n et −n.
☞ Si a|b, et b|c, alors a|c.
☞ Si a|b1 , b2 , . . . , bn , alors a|b1 c1 +b2 c2 +. . .+bn cn , quels que soient les entiers c1 , c2 , . . . , cn .
☞ Si a divise b et b 6= 0, alors |a| 6 |b|.
☞ Si a divise b et b divise a, alors a = ±b.
☞ Si a et b sont deux entiers tels que an |bn pour un entier n > 1, alors a|b.

Toutes les propriétés listées précédemment sont immédiates, à l’exception de la dernière dont
la démonstration n’est pas triviale sans bagage arithmétique. Une preuve possible consiste
à utiliser la caractérisation de la divisibilité par les valuations p-adiques (voir paragraphe
1.3).

Voyons immédiatement deux exercices qui montrent comment on peut manipuler la no-
tion de divisibilité :

Exercice : Soient x et y des entiers. Montrer que 2x + 3y est divisible par 7 si et seulement
si 5x + 4y l’est.
Solution : Supposons que 7 divise 2x + 3y, alors il divise 6 (2x + 3y) − 7 (x + 2y) = 5x + 4y.

Réciproquement si 7 divise 5x + 4y, il divise 6 (5x + 4y) − 7 (4x + 3y) = 2x + 3y.

Exercice : Pour quels entiers n strictement positifs, le nombre n2 + 1 divise-t-il n + 1 ?


Solution : Si n2 + 1 divise n + 1, comme tout est positif, on doit avoir n2 + 1 6 n + 1, ce qui

n’est vérifié que pour n = 1. On vérifie ensuite que n = 1 est bien solution.

4
Parties entières
Définition 1.1.2 Si x est un réel, on appelle partie entière de x, et on note [x], le plus
grand entier inférieur ou égal à x. Ainsi, on a [x] 6 x < [x] + 1.
Remarque. On définit aussi la partie décimale de x, comme la différence x − [x]. La partie
décimale de x est souvent notée {x}. Cette notion est moins utilisée que la notion de partie
entière et les conventions de notations sont moins usuelles à ce propos : lors d’un exercice,
ou d’un exposé, il est toujours de bon goût de commencer par préciser les notations qui vont
être employées par la suite.

Notons qu’il faut être prudent avec les nombres négatifs : autant pour les nombres positifs,
la partie entière correspond au nombre auquel on retire ses chiffres après la virgule, autant
ce n’est pas le cas pour les nombres négatifs. En effet, si on suit la définition, on voit par
exemple que [−3, 5] = −4.

Les parties entières et parties décimales obéissent à quelques propriétés élémentaires que
nous listons ci-dessous :
Propriétés élémentaires
☞ On a toujours x = [x] + {x}.
☞ Pour tout réel x, on a x − 1 < [x] 6 x
☞ Si x est entier, [x] = x et {x} = 0. Et réciproquement si l’une des deux égalités est
vérifiée, alors x est entier.
☞ [−x] = −[x] − 1 sauf si x est entier, auquel cas [−x] = −[x].
☞ Si x et y sont deux réels, [x] + [y] 6 [x + y] 6 [x] + [y] + 1.
x
☞ Si m > 0 est un entier, alors il y a exactement [ m ] multiples de m compris entre 1 et
x.

La démonstration des propriétés consiste en de simples manipulations de la définition et


principalement de l’inégalité [x] 6 x < [x] + 1. Elle est laissée au lecteur. On remarquera
que très souvent les questions faisant intervenir des parties entières se résument à de la
manipulation d’inégalités comme le montre par exemple l’exercice suivant :

Exercice : On suppose que 4n + 2 n’est pas le carré d’un nombre entier. Montrer que pour
n > 0, on a : h√ i h√ i

n+ n+1 = 4n + 2

Solution : Remarquons tout d’abord que l’on a toujours l’inégalité :


√ √ √
n + n + 1 < 4n + 2
√ √
En effet, en élevant au carré, on a à comparer 2n + 1 + 2 n2 + n et 4n + 2, soit 2 n2 + n
et 2n + 1 et l’inégalité devient évidente après une nouvelle élévation au carré.
Il reste à prouver qu’il n’existe aucun entier k tel que :
√ √ √
n + n + 1 < k 6 4n + 2

5
soit, encore en élevant au carré qu’il n’existe aucun entier k tel que :

2n + 1 + 2 n2 + n < k 2 6 4n + 2

Mais il est clair que 4n + 1 < 2n + 1 + 2 n2 + n et un tel entier k vérifirait a fortiori
4n + 1 < k 2 6 4n + 2. Comme k est entier, il vient forcément k 2 = 4n + 2, mais cela n’est

pas possible puisque l’on a supposé que 4n + 2 n’était pas le carré d’un entier.

Remarque. En fait, 4n + 2 n’est jamais le carré d’un entier. En effet, le nombre 4n + 2 est
pair, et s’il était le carré d’un entier, il serait le carré d’un entier pair. Mais alors 4n + 2
devrait être un multiple de 4, ce qui n’est, à l’évidence, pas le cas. L’égalité précédente de
parties entières est donc valable pour tout entier n > 1, sans hypothèse supplémentaire.

Une propriété amusante des parties entières qui montre également que parfois (souvent)
les manipulations d’inégalités ne sont pas faciles est le théorème de Beatty que voici :

Théorème 1.1.3 (Beatty) Soient α et β deux réels strictements positifs. On note Sα


(resp. Sβ ) l’ensemble des entiers strictement positifs qui s’écrivent sous la forme [nα] (resp.
[nβ]) pour un certain entier n.
Les ensembles Sα et Sβ forment une partition de N? si, et seulement si α et β sont
irrationnels et vérifient α1 + β1 = 1.

Démonstration. Commençons par supposer que α et β sont des irrationnels vérifiant α1 +


1
β
= 1. Soit k un entier strictement positif. Il est dans l’ensemble Sα si et seulement s’il
existe un entier n tel que :
nα − 1 < k < nα
l’inégalité de droite étant stricte car α est supposé irrationnel. L’équation se transforme et
donne :
k k 1
<n< +
α α α
¤k k £
Autrement dit, k ∈ Sα si et seulement i si l’intervalle
h , + α1 contient un entier. De même
α α
k ∈ Sβ si et seulement si l’intervalle βk , βk + β1 contient un entier.
¤ £
L’intervalle αk , αk + 1 est de longueur 1 et ses bornes sont irrationnelles, donc il contient
un et un seul entier n. Si n < αk + α1 , alors k ∈ Sα . Sinon, on a l’inégalité :

k 1 k
+ <n< +1
α α α
l’inégalité de gauche étant stricte car k+1
α
est irrationnel et donc ne peut être égal à n.
k k
Comme α = k − β , il vient :
k k 1
<k+1−n< +
β β β
et donc k ∈ Sβ . Si k était à la fois élémenti de Sα et h de Sβ , il y aurait un entier dans
¤k k £
l’intervalle α , α + α1 et un dans l’intervalle βk , βk + β1 et donc par le même raisonnement
¤ £
que précédemment, il y en aurait deux dans l’intervalle αk , αk + 1 , ce qui n’est pas possible.

6
?
Réciproquement, supposons que £kS
¤ α et Sβ forment une partition de N . Considérons un
entier
h ki strictement positif. Il y a α entiers dans {1, . . . , k} qui sont dans Sα . De même, il
k
ya β
entiers dans {1, . . . , k} qui sont dans Sβ . Du fait de la partition, il vient :
· ¸ · ¸
k k
+ =k
α β
pour tout k. En faisant tendre k vers l’infini, il vient :
1 1
+ =1
α β
ce qui démontre la deuxième condition.
Supposons maintenant par l’absurde que α soit rationnel. Alors il en est de même de β
d’après la relation précédente. Écrivons α = ab et β = dc . L’entier ac est élément de Sα (en
prenant n = bc) et également élément de Sβ (en prenant n = ad), ce qui est contradictoire.
¤

Pgcd et Ppcm
Ce paragraphe introduit les définitions de pgcd et ppcm qui sont deux notions fonda-
mentales de l’arithmétique et en donne leurs principales propriétés. Les démonstrations qui
ne sont pas évidentes sont reportées au chapitre 2 et seront vues comme conséquence de la
division euclidienne.

Définition 1.1.4 Soient a et b deux entiers non tous deux nuls. L’ensemble des diviseurs
communs de a et de b est fini et non vide, il possède donc un plus grand élément appelé plus
grand commun diviseur (pgcd) de a et b et noté pgcd (a, b).
Lorsque pgcd (a, b) = 1, on dit que a et b sont premiers entre eux.
De même a et b possèdent un plus petit multiple commun positif, on l’appelle le plus
petit commun multiple (ppcm) de a et de b et on le note ppcm (a, b).

Propriétés
☞ Si d = pgcd (a, b), alors n divise a et b si et seulement si n divise d.
☞ Si m = ppcm (a, b), alors n est un multiple a et de b si et seulement si n est un multiple
de m.
☞ Si a, b et n sont des entiers non nuls ¡ a etb ¢n > 10, alors pgcd (na, nb) = npgcd (a, b). Si
de plus n divise a et b, alors pgcd n , n = n pgcd (a, b).
☞ Si d = pgcd (a, b), on peut écrire a = da0 et b = db0 pour a0 et b0 des nombres premiers
entre eux.
☞ Si a et b sont des entiers, l’égalité pgcd (a, b) = pgcd (a, a + b) est toujours vérifiée
lorsqu’elle a un sens. En particulier, le pgcd de deux nombres consécutifs est 1, et
plus généralement, le pgcd de a et de a + n est un diviseur positif de n.
☞ Plus généralement, si x, y, a, b, a0 et b0 sont des entiers alors :
pgcd (x, y) | pgcd (ax + by, a0 x + b0 y) | (ab0 − ba0 ) pgcd (x, y)
En particulier si |ab0 − ba0 | = 1, alors pgcd (x, y) = pgcd (ax + by, a0 x + b0 y).

7
Ces propriétés sont élémentaires. Souvent, pour prouver l’égalité de deux pgcd, on
montre que chacun des pgcd divise l’autre. C’est la méthode que l’on utilise majoritai-
rement ici. Expliquons comment on procède pour montrer qu’un pgcd en divise un autre en
donnant un preuve de la dernière propriété qui est la plus difficile : notons d = pgcd (x, y).
Alors d divise x et y et donc il divise ax + by et a0 x + b0 y puis leur pgcd. De même, soit
d0 = pgcd (ax + by, a0 x + b0 y), alors d0 divise b0 (ax + by) − b (a0 x + b0 y) = (ab0 − ba0 ) x et
a0 (ax + by) − a (a0 x + b0 y) = (a0 b − b0 a) y. Ainsi d0 divise pgcd ((ab0 − ba0 ) x, (a0 b − b0 a) y) =
|ab0 − ba0 | pgcd (x, y), ce qui conclut.

Citons également des résultats classiques et souvent assez utiles :


Propriétés
☞ Si a et b sont des entiers non nuls alors pgcd (an , bn ) = pgcd (a, b)n pour tout entier
n > 0.
☞ Si a, b et c sont des entiers non nuls, on a :

pgcd (a, ppcm (b, c)) = ppcm (pgcd (a, b) , pgcd (a, c))
ppcm (a, pgcd (b, c)) = pgcd (ppcm (a, b) , ppcm (a, c))

☞ Théorème de Bézout. Si a et b sont des entiers premiers entre eux, alors il existe des
entiers u et v tels que au + bv = 1.
☞ Lemme de Gauss. Si des entiers a, b et c sont tels que a divise bc et a premier avec b,
alors a divise c.
☞ Si deux entiers premiers entre eux a et b divisent n, alors le produit ab divise également
n.

Ces propriétés sont plus difficiles. Les deux premières résultent par exemple directement de
l’expression de pgcd (a, b) en fonction de la décomposition en facteurs premiers de a et de
b (voir la partie sur le théorème fondamental de l’arithmétique dans le paragraphe 1.2). Les
autres résultent des propriétés de la division euclidienne que nous étudions au chapitre 2.
Leur démonstration est donc reportée aux paragraphes 2.3 et 2.4.

Donnons à présent deux exercices qui montrent comment l’on peut manipuler les faits
précédents :
n
Exercice : On définit le n-ième nombre de Fermat par la formule Fn = 22 + 1. Montrer que
les Fn sont deux à deux premiers entre eux.
Solution : On remarque que :
n+1 ¡n ¢¡ n ¢
Fn+1 − 2 = 22 −1 = 22 − 1 22 + 1
³ n−1 ´ ³ n−1 ´¡ n ¢
= 22 − 1 22 + 1 22 + 1 = Fn Fn−1 · · · F0

Soit d un diviseur commun de Fn et Fm . Supposons par exemple n < m. D’après la formule


précédente, comme d divise Fn , il divise Fm − 2 et donc 2. Les Fn sont clairement impairs,

la seule solution est d’avoir |d| = 1. Ceci prouve que Fn et Fm sont premiers entre eux.

8
Exercice : Soient a et b des nombres premiers entre eux. Montrer que ab et a + b sont aussi
premiers entre eux.
Solution : Soit d un diviseur commun de ab et de a + b. Alors d divise a (a + b) − ab = a2 . De
même d divise b2 . D’après une des propriétés précédentes, les entiers a2 et b2 sont premiers

entre eux. Ainsi d = ±1, ce qui conclut.

1.2 Nombres premiers


Définition et exemples
Comme nous l’avons dit dans l’introduction de cette partie, les nombres premiers sont
les briques élémentaires pour fabriquer les nombres. De façon plus précise et moins imagée,
on a la définition suivante :

Définition 1.2.1 Un entier n > 0 est dit premier s’il est différent de 1 et s’il n’admet aucun
diviseur positif différent de 1 et n. Un tel diviseur est appelé diviseur strict.
Un nombre qui n’est pas premier est appelé nombre composé.
Par définition, donc, 1 n’est pas premier. C’est une simple convention mais elle s’avère utile
pour l’énoncé des théorèmes comme vous allez (peut-être) vous en rendre compte. Les entiers
2, 3, 5, 7, 11, 13 sont les premiers nombres premiers. Le nombre 6, n’est par contre pas premier
car on peut écrire 6 = 2 × 3 (et donc 2 (ou 3) est un diviseur strict de 6).

Proposition 1.2.2 Soit n > 1 un entier. Son √ plus petit diviseur d > 1 est un nombre
premier. Si de plus n est composé, alors d 6 n.
Démonstration. Supposons que d ne soit pas premier. Alors par définition, il existe un
diviseur strict d0 de d. Mais alors d0 divise n, d0 > 1 et d0 < d, ce qui contredit la minimalité
de d.
Comme d divise n, on peut écrire n = dd0 . On a d > 1 et comme n n’est pas premier,
d < n. Ainsi d0 est un diviseur de n strictement √supérieur à 1. Par minimalité de d, on
obtient d0 > d et donc n > d2 puis finalement d 6 n. ¤

Remarque. On déduit de la propriété précédente que pour tester si un entier n > 1 est
premier, il suffit de regarder s’il est divisible ou non par un des entiers compris entre 2 et

n. Par exemple, pour vérifier que 37 est premier, il suffit de voir qu’il n’est divisible ni par
2, ni par 3, ni par 4, ni par 5, ni par 6. On aurait également pu éviter les divisions par 4 et
6 si on savait par avance que ces nombres étaient composés.

La remarque précédente nous amène à la méthode suivante, appelée crible d’Ératosthène


pour lister tous les nombres premiers entre 1 et n : on écrit à la suite les uns des autres tous
les entiers compris entre 2 et n. On entoure le premier 2 et on barre tous ses multiples (i.e.
tous les nombres pairs). On entoure ensuite le prochain√ nombre non barré (en l’occurrence 3)
et on barre tous ses multiples. Ainsi de suite jusqu’à n. On entoure finalement les nombres
non barrés. Les nombres entourés sont alors exactement les nombres premiers compris entre
1 et n.

9
Le théorème fondamental de l’arithmétique
On en arrive à présent au théorème fondamental de l’arithmétique. Nous aurons besoin
pour la démonstration du lemme suivant (qui sera démontré dans le paragraphe 2.4) :

Lemme 1.2.3 Si un nombre premier p divise le produit a1 · · · an , alors il divise l’un des ai .

Théorème 1.2.4 (Décomposition en facteurs premiers) Tout entier n > 1 se décom-


pose d’une et d’une seule manière en un produit de nombres premiers. Autrement dit, pour
tout entier n > 1, il existe des nombres premiers deux à deux distincts p1 , . . . , pk et des
entiers strictement positifs α1 , . . . , αk , uniquement déterminés à l’ordre près, tels que :
n = pα1 1 pα2 2 · · · pαk k
Remarque. Le théorème reste bien vrai pour n = 1 : il faut choisir k = 0, le produit d’aucun
entier étant par convention égal à 1.

Démonstration. Commençons par l’existence de la décomposition. On raisonne par récur-


rence sur n. Commençons (pour ne pas perturber le lecteur) à n = 2 qui s’écrit comme un
produit de nombres premiers, étant lui-même premier.
Soit n > 3 un entier. Supposons que tous les entiers strictement inférieurs à n s’écrivent
comme le stipule le théorème et montrons que la conclusion subsiste pour l’entier n. Il y a
deux cas : soit n est premier, soit il ne l’est pas. Le premier cas est vite réglé : n premier
s’écrit bien comme un produit de nombres premiers. Supposons donc que n soit composé.
Ainsi, il s’écrit n = dd0 avec 2 6 d < n et 2 6 d0 < n. Les entiers d et d0 relèvent de
l’hypothèse de récurrence et on peut écrire :
d = p1 p2 · · · pk
d0 = p01 p02 · · · p0k0
pour des nombres premiers pi et p0i . Il ne reste plus qu’à effectuer le produit pour conclure.
Passons désormais à l’unicité. Supposons que :
p1 p2 · · · pk = p01 p02 · · · p0k0
pour certains nombres premiers pi et p0i . On veut montrer que k = k 0 et que les pi sont égaux
aux p0i à l’ordre près. Raisonnons par l’absurde. Parmi les contre-exemples dont on vient de
supposer l’existence, il en est au moins un pour lequel min(k, k 0 ) est minimal. Considérons
un de ceux-ci.
Le nombre premier p1 divise le produit p01 p02 · · · p0k0 donc d’après le lemme 1.2.3, il divise p0i
pour un certain entier i. Or, les diviseurs de p0i (qui est premier) ne sont que 1 et p0i . Comme
p1 6= 1, il ne reste plus que la possibilité p1 = p0i = p. On peut alors simplifier l’égalité :
p1 p2 · · · pk = p01 p02 · · · p0k0
en divisant par p, obtenant ainsi un contre-exemple plus petit. C’est une contradiction et
l’unicité est prouvée. ¤

Le théorème précédent permet de décrire explicitement les diviseurs d’un entier n dont
on connaı̂t la décomposition en facteurs premiers.

10
Proposition 1.2.5 Si la décomposition en facteurs premiers de l’entier n > 1 est n =
pα1 1 pα2 2 · · · pαk k , alors les diviseurs positifs de n sont les entiers de la forme pβ1 1 pβ2 2 . . . pβkk , avec
0 6 βi 6 αi pour tout 1 6 i 6 k.
Comme conséquence, on obtient une expression du pgcd et du ppcm de deux entiers
lorsqu’on connaı̂t leur décomposition en facteurs premiers. Précisément, si :

a = pα1 1 pα2 2 · · · pαk k


b = pβ1 1 pβ2 2 · · · pβkk

où les pi sont deux à deux distincts, mais les αi et βi sont éventuellement nuls, on a :
min(α1 ,β1 ) min(α2 ,β2 ) min(αk ,βk )
pgcd (a, b) = p1 p2 · · · pk
max(α1 ,β1 ) max(α2 ,β2 ) max(αk ,βk )
ppcm (a, b) = p1 p2 · · · pk

Si l’on remarque que pour α et β des entiers (ou des réels), on a toujours min(α, β) +
max(α, β) = α + β, on déduit directement des deux expressions précédentes la proposition
suivante :

Proposition 1.2.6 Si a et b sont des entiers positifs, on a l’égalité :

pgcd (a, b) · ppcm (a, b) = ab

Infinité des nombres premiers et raffinements


Le premier résultat qui remonte à Euclide est le suivant :

Proposition 1.2.7 Il existe une infinité de nombres premiers.


Démonstration. On raisonne par l’absurde. On suppose qu’il n’existe qu’un nombre fini
d’entiers premiers, disons p1 , p2 , . . . , pk . On peut alors exhiber un entier qui n’est divisible
par aucun de ces nombres premiers, ce qui est contradictoire compte tenu du fait que cet
entier possède un diviseur premier. En effet, considérons N = p1 p2 · · · pk +1 : si pi (1 6 i 6 k)
divisait n, alors pi diviserait 1, ce qui est absurde. ¤

La démonstration précédente s’applique pour obtenir des résultats plus précis comme le
montre l’exercice suivant :

Exercice : Montrer qu’il existe une infinité de nombres premiers de la forme 4n + 3.


Solution : On raisonne par l’absurde en supposant qu’il n’existe qu’un nombre fini de premiers
de cette forme, notés p1 , p2 , . . . , pk . On considère alors N = 4p1 p2 . . . pk − 1. Les diviseurs
premiers de n sont distincts de 2 et des pi (1 6 i 6 k), et il en existe un qui est de la forme
4n + 3, car sinon on vérifie immédiatement que N ne pourrait être de la forme 4n + 3 (un
nombre premier qui n’est de la forme 4n + 3 est de la forme 4n + 1 et le produit de tels √
nombres est encore de cette forme).
Remarque. De même, on peut prouver qu’il existe une infinité de nombres premiers de
la forme 6n + 5. Toutefois, ces cas restent anecdotiques : par exemple, la démonstration

11
précédente ne s’applique par pour les nombres premiers de la forme 4n + 1 (qui pourtant
forment bien un ensemble infini).

Une autre propriété utile qui mesure plus ou moins la raréfaction des nombres premiers
est la proposition totalement élémentaire suivante :

Proposition 1.2.8 Il existe des suites arbitrairement longues de nombres consécutifs com-
posés. Autrement dit, pour tout k, il est possible de trouver un entier n tel que les nombres
n + 1, . . . , n + k soient tous composés.
Démonstration. Il suffit de prendre n = (k + 1)! + 1. ¤

Remarque. Comme l’ensemble des nombres premiers est infini, on déduit directement de la
proposition précédente, la proposition suivante plus précise :

Proposition 1.2.9 Pour tout entier k, il existe un nombre premier p tel que tous les
nombres p + 1, . . . , p + k soient composés.

Mis à part ces cas simples, la répartition des nombres premiers est une question qui a
occupé les mathématiciens durant des générations, et de nombreuses questions demeurent
ouvertes. Citons quelques résultats importants qu’il est bon de connaı̂tre même si leur dé-
monstration dépasse de loin le cadre de ce cours :
Propriétés
☞ Postulat de Bertrand. Pour tout entier n > 1, il existe un nombre premier entre n et
2n.
☞ Théorème des nombres premiers. Si on note π(x) le nombre d’entiers premiers inférieurs
ou égaux à x, on a l’estimation π(x) ∼ lnxx (au sens où le quotient des deux membres
tend vers 1 lorsque x tend vers l’infini).
☞ Théorème de Dirichlet. Si a 6= 0 et b sont deux entiers naturels premiers entre eux, la
suite an + b (n entier) contient une infinité de nombres premiers.

1.3 Valuation p-adique


Les valuations sont un moyen systématique et souvent efficace pour utiliser toute la puis-
sance du théorème de décomposition en facteurs premiers. Commençons par une définition :

Définition 1.3.1 Si p est un nombre premier, et n un entier non nul, la valuation p-adique
de n est le plus grand entier k tel que pk divise n. On la note vp (n).
Si n = 0, on convient que vp (0) = +∞ pour tout nombre premier p.

Les propriétés suivantes sont élémentaires mais il est bon de toujours les avoir en tête.
Leur manipulation est simple et puissante.
Propriétés

12
☞ Si n non nul se décompose sous la forme n = pα1 1 pα2 2 . . . pαk k , alors vpi (n) = αi pour tout
1 6 i 6 k, et vp (n) = 0 si p est distinct des pi . Ainsi, vp (n) = 0 sauf pour un nombre
fini de p premiers.
☞ Si m et n sont deux entiers, m divise n si et seulement si vp (m) 6 vp (n) pour tout
nombre premier p.
☞ Si a et b sont des entiers non nuls, on a :

vp (pgcd (a, b)) = min (vp (a), vp (b))


vp (ppcm (a, b)) = max (vp (a), vp (b))

☞ Si m et n sont deux entiers, on a, pour tout nombre premier p :

vp (ab) = vp (a) + vp (b)


vp (a + b) > min (vp (a), vp (b))

et la dernière inégalité est une égalité dès que vp (a) 6= vp (b).

Il est possible de déterminer les valuations p-adiques d’une factorielle. On rappelle, fort
à propos, que par définition n! = 1 × 2 × · · · × n.

Proposition 1.3.2 (Formule de Legendre) Si p est un nombre premier et n est un en-


tier positif, on a :
X∞ · ¸ · ¸ · ¸
n n n
vp (n!) = i
= + 2
+ ···
i=1
p p p
h i
Remarque. Lorsque pi > n, le nombre pni = 0. Ceci assure qu’il n’y a bien qu’un nombre
fini de termes non nuls dans la somme précédente.

Démonstration. Pour un entier positif ou nul i, appelons ni le nombre d’entiers compris


entre 1 et n dont la valuation p-adique est exactement i. On a alors :

vp (n!) = n1 + 2n2 + 3n3 + · · ·


i
D’autre part, les entiers
h i dont la valuation excède i sont exactement les multiples de p et
sont au nombre de pni , d’où :
· ¸
n
= ni + ni+1 + ni+2 + · · ·
pi
Les deux formules précédentes mises ensemble démontrent la proposition. ¤

Classiquement, on illustre le théorème précédent par l’exercice suivant :

Exercice : Par combien de zéros se termine le nombre 2004! ?


Solution : L’entier 10 n’est pas premier : on ne peut donc pas appliquer directement la
formule de Legendre. En décomposant 10 en facteurs premiers, on se rend compte que le

13
plus grand exposant n tel que 10n divise 2004! est le plus petit des deux nombres v2 (2004!)
et v5 (2004!). La formule de Legendre prouve directement que c’est v5 (2004!). Il vaut :
· ¸ · ¸ · ¸ · ¸ · ¸
2004 2004 2004 2004 2004
+ + + + + · · · = 400 + 80 + 16 + 3 + 0 + · · · = 499
5 25 125 625 3125

Le nombre 2004! se termine donc par 499 zéros.

1.4 Quelques fonctions arithmétiques


Les principales fonctions arithmétiques sont les suivantes :
☞ la fonction d qui à n associe le nombre de diviseurs positifs de n ;
☞ la fonction σ qui à n associe la somme des diviseurs positifs de n ;
☞ plus généralement, la fonction σs qui à n associe les somme des diviseurs positifs de n
élevés à la puissance s (les deux cas précédents correspondant à s = 0 et s = 1) ;
☞ la fonction P qui à n associe le produit des diviseurs positifs de n

Remarque. Les notations introduites précédemment sont traditionnelles mais ne sont pas
universelles. Elles seront normalement reprécisées à chaque nouvelle apparition. De même
si vous êtes amenés à utiliser ces fonctions, il est souhaitable de redonner rapidement la
définition avant pour fixer les notations.

La décomposition en facteurs premiers permet de donner les expressions de ces fonctions


arithmétiques :

Proposition 1.4.1 Si la décomposition en facteurs premiers de n est n = pα1 1 · · · pαk k , alors


on a les expressions suivantes :

d(n) = (α1 + 1)(α2 + 1) . . . (αk + 1)


s(α1 +1) s(α +1) s(α +1)
p1 − 1 p2 2 − 1 pk k − 1
σs (n) = · · · ·
ps1 − 1 ps2 − 1 psk − 1
d(n)
P (n) = n 2

Démonstration. On ne démontre que l’expression de P qui est la plus difficile, les autres
se traitant de façon analogue.
Un diviseur positif de n s’écrit pβ1 1 · · · pβkk où 0 6 βi 6 αi . Le produit de tous ces nombres
est de la forme :
pγ11 · · · pγkk
Il suffit donc de calculer les exposants γi . Fixons un entier v ∈ {0, 1, . . . , α1 }. Il y a exacte-
ment (α2 + 1) · · · (αk + 1) diviseurs de n pour lesquels β1 = v. Lorsque l’on multiplie tous
ces diviseurs, on aura donc :
α1
X 1 d(n)
γ1 = (α2 + 1) · · · (αk + 1) v = α1 (α1 + 1) · · · (αk + 1) = α1 ·
v=0
2 2

14
On a bien entendu une formule analogue pour γi . En remettant tout bout à bout, on obtient
la formule annoncée. ¤

Exercice : L’entier n > 0 étant fixé, déterminer le nombre de couples (x, y) d’entiers stricte-
ment positifs vérifiant x1 + y1 = n1 .
Solution : L’équation se réécrit sous la forme :

(x − n)(y − n) = n2

Il y a donc autant de solutions que de diviseurs positifs de n2 en remarquant que puisque



1
x
+ y1 = n1 , on a forcément x > n et y > n.

1.5 Nombres rationnels


a
Définition 1.5.1 Un nombre rationnel est un réel de la forme b
pour a et b entiers, b 6= 0.
Leur ensemble se note Q.
Nous allons voir que certaines propriétés des entiers demeurent inchangées sur les ration-
nels. Précisément il est possible de parler de décomposition en facteurs premiers, et donc de
valuation p-adique pour tout nombre premier p.

Théorème 1.5.2 Soit r un nombre rationnel non nul. Alors r se décompose de façon unique
(à permutation des facteurs près) sous la forme :

r = pα1 1 · · · pαd d

où les pi sont des nombres premiers deux à deux distincts et où les αi sont des entiers
relatifs.
Démonstration. La démonstration est une conséquence presque directe de la propriété
analogue pour les nombres entiers. Elle est laissée au lecteur. ¤

Définition 1.5.3 Si p est un nombre premier, on appelle valuation p-adique du rationnel


r 6= 0, et on note vp (r), l’exposant apparaissant sur le nombre premier p dans la décompo-
sition en facteurs premiers de r. Bien sûr, si p n’apparaı̂t pas dans cette décomposition, on
convient que vp (r) = 0.
Si r = 0, on convient que vp (r) = +∞ pour tout nombre premier p.

Propriétés
☞ Si r est un rationnel non nul, il n’existe qu’un nombre fini de nombres premiers p pour
lesquels vp (r) 6= 0
a
☞ Si b
est une fraction représentant le rationnel r, alors :

vp (r) = vp (a) − vp (b)

En particulier, la valeur vp (a) − vp (b) ne dépend pas de la fraction choisie.

15
☞ Soit r un nombre rationnel. Alors r est entier si, et seulement si vp (r) > 0 pour tout
nombre premier p.
☞ Soient s et t deux nombres rationnels, on a :

vp (st) = vp (s) + vp (t)


vp (s + t) > min (vp (s), vp (t))

et la dernière inégalité est une égalité dès que vp (s) 6= vp (t).

Les extensions précédentes permettent par exemple de démontrer simplement l’irratio-


√ √ ¡√ ¢2
nalité de 2. En effet, si 2 était rationnel, on devrait avoir, du fait de l’égalité 2 =2:
³√ ´
2 · v2 2 =1
¡√ ¢ 1
soit v2 2 = 2 , mais cela n’est pas possible puisque les valuations p-adiques sont toujours
entières.

En utilisant les mêmes concepts, on peut résoudre l’exercice suivant :



Exercice : Montrer que si n > 2 et a > 0 sont des entiers, alors n a est soit un entier, soit
un nombre irrationnel.

Solution : Supposons que n a soit un nombre rationnel. On peut alors écrire pour tout
nombre premier p : ¡√ ¢
nvp n a = vp (a)
√ √ √
et donc vp ( n a) = n1 vp (a) > 0 puisque a est entier. Cela démontre que n a est un entier.

Densité
Une propriété des rationnels souvent utiles pour les passages à la limite (et donc finale-
ment assez peu en arithmétique) est donnée par le théorème suivant :

Théorème 1.5.4 Soit ε > 0 et x ∈ R. Alors il existe y ∈ Q tel que |x − y| 6 ε.


On dit, dans cette situation, que Q est dense dans R.
1
Démonstration. Soit q un entier strictement supérieur à ε
et p = [qx]. On a l’encadrement
p 6 qx 6 p + 1 et donc en divisant par q :
p p 1 p
6x6 + < +ε
q q q q
p
Ainsi le rationnel y = q
convient. ¤

16
1.6 Exercices
Exercice 1 On désigne par d (n) le nombre de diviseurs strictement positifs de l’entier n.
Montrer que d (n) est impair si, et seulement si n est un carré.

Exercice 2 (Saint-Petesbourg 04) Déterminer tous les entiers positifs n tels que 5n−1 +
3n−1 divise 5n + 3n .

Exercice 3 Montrer que pour tout entier n, le nombre n3 − n est un multiple de 6.


21n+4
Exercice 4 (OIM 59) Montrer que la fraction 14n+3
est toujours irréductible.

Exercice 5 Montrer que 2x + 3 est un multiple de 11 si, et seulement si 5x + 2 l’est.

Exercice 6 Soit p > 3 un nombre premier. Montrer que p2 − 1 est un multiple de 12.

Exercice 7 Soient a et b des entiers strictement positifs tels que an divise bn+1 pour tout
entier n > 1. Montrer que a divise b.

Exercice 8 Soit n un entier strictement positif. On appelle k le nombre de diviseurs


premiers de n. Prouver que :
log n > k log 2

Exercice 9 Soient p un nombre premier et n un entier tels que p divise nk . Est-ce qu’alors,
forcément, p divise n ?

Exercice 10* (Baltique 04) Déterminer tous les ensembles X d’entiers strictement positifs
contenant au moins deux éléments et tels que, si m et n sont dans X avec n > m alors il
existe un élément k ∈ X vérifant n = mk 2 .

Exercice 11* (Irlande 98) Déterminer les entiers n ayant exactement 16 diviseurs :

1 = d1 < d2 < . . . < d15 < d16 = n

et tels que d6 = 18 et d9 − d8 = 17.

Exercice 12* Déterminer tous les entiers a, b et c strictement supérieurs à 1 tels que a
divise bc − 1, b divise ca − 1 et c divise ab − 1.

Exercice 13* Pierre et Xavier jouent au jeu suivant. Ils commencent par choisir un nombre
entier n > 0. Puis, Pierre choisit en secret un entier m tel que 0 < m < n. Xavier doit alors
découvrir le nombre secret. Pour cela, il peut proposer un nombre k quelconque à Pierre
qui, en retour, lui indique si le nombre m + k est premier ou non. Prouver que Xavier peut
déterminer le nombre secret de Pierre en moins de n − 1 questions.

Exercice 14* Montrer que les racines cubiques de trois nombres premiers distincts ne
peuvent être dans une même progression arithmétique.

Exercice 15* Soit x un réel. Est-il vrai que :

17
a) Si x7 et x12 sont rationnels alors x est rationnel ?
b) Si x9 et x12 sont rationnels alors x est rationnel ?

Exercice 16* (d’après Autriche 02) Soit a > 9 un entier impair. Montrer que l’équation :
a
x[x] =
2
n’a pas de solution pour x ∈ Q.

Exercice 17* Trouver le plus petit entier x tel que 2|x − 1, 3|x − 2, . . . , 9|x − 8.

Exercice 18* (OIM 02) Les diviseurs strictement positifs de l’entier n > 1 sont 1 = d1 <
d2 < . . . < dk = n. Soit d = d1 d2 + d2 d3 + . . . + dk−1 dk . Montrer que d < n2 et trouver tous
les n pour lesquels d divise n2 .

Exercice 19* (Nombres de Fermat) Montrer que si 2n + 1 est un nombre premier, alors
n est une puissance de 2.

Exercice 20* Si n > 1 est un entier, on note d (n) le nombre de ses diviseurs positifs,
σ (n) la somme de ses diviseurs positifs ou ϕ (n) le nombre de nombres premiers avec n et
compris entre 1 et n.
Trouver tous les entiers n > 1 tels que :

σ(n) + ϕ(n) = n · d(n)

Exercice 21* (OIM 68) Le symbôle [x] désignant la partie entière de x. Calculer :
· ¸ · ¸ · ¸ · ¸
n+1 n+2 n+4 n + 2k
+ + + ... + + ...
2 4 8 2k+1

Exercice 22* On note pn le n-ième nombre premier. En utilisant le théorème des nombres
premiers, montrer que pn ∼ n log n.

Exercice 23* (APMO 04) Déterminer toutes les parties E non vides de N? telles que
a+b
pour tous a et b dans E, le nombre pgcd(a,b) est aussi dans E.

Exercice 24* Trouver tous les entiers n strictement positifs pour lesquels 2n divise 3n − 1.

Exercice 25* (USA 72) Soient a, b et c des entiers strictement positifs. Montrer que :

pgcd (a, b, c)2 ppcm (a, b, c)2


=
pgcd (a, b) pgcd (b, c) pgcd (a, c) ppcm (a, b) ppcm (b, c) ppcm (a, c)

Exercice 26* (Iran 96) Soit k > 0 un entier. Prouver que tout entier n > 0 peut s’écrire
de façon unique sous la forme :

n = Ckak + Ck−1 t
ak−1 + · · · + Cat

18
où ak > ak−1 > · · · > at > t > 1 sont des entiers.

Exercice 27* (Erdös) Soient a1 , . . . , an+1 des entiers deux à deux distincts dans {1, . . . , 2n}.
a) Montrer qu’il existe i et j tels que ai est premier avec aj .
b) Montrer qu’il existe i et j distincts tels que ai divise aj .

Exercice 28* (Australie 96) Si n est un entier, on note σ (n) la somme des diviseurs
positifs de n. Soit (ni ) une suite strictement croissante d’entiers telle que σ (ni ) − ni est
constante. Montrer que tous les ni sont premiers.

Exercice 29* (Iran 99) Déterminer les entiers n pour lesquels d21 + d22 + d23 + d24 = n où
1 = d1 < d2 < d3 < d4 désignent les quatre plus petits diviseurs de n.

Exercice 30* Soient (an ) et (bn ) deux suites d’entiers. On suppose que les suites (an + bn )
et (an bn ) sont arithmétiques. Montrer qu’il existe une constante c tel que pour tout n, on
ait an = c ou bn = c.

Exercice 31* (Corée 98) Trouver tous les entiers strictement positifs `, m, n premiers
entre eux deux à deux tels que :
µ ¶
1 1 1
(` + m + n) + +
` m n
soit un entier.

Exercice 32* (Fonction de Moëbius) On définit la fonction de Moëbius par µ (1) = 1,


µ (n) = 0 est n est divisible par p2 pour un certain nombre premier p, et µ (p1 · · · pr ) = (−1)r
si les pi sont des nombres premiers deux à deux distincts.
a) Montrer que pour tout n > 1, on a :
X
µ (d) = 0
d|n

b) En déduire que si f : N? → N? est une fonction et si g est définie par la formule :


X
g (n) = f (d)
d|n

alors on peut retrouver f à partir de g grâce à la formule :


X ³n´
f (n) = µ g (d)
d
d|n

Exercice 33* Prouver que parmi dix entiers consécutifs, il y en a un qui est premier avec
chacun des autres.

Exercice 34* (AMM) Si n est un entier, on note P (n) le produit des diviseurs de n.
Montrer que si P (n) = P (m) alors n = m.

19
Exercice 35* Déterminer tous les entiers n et m strictement positifs pour lesquels la
somme des entiers de n jusqu’à n + m vaut 1000.

Exercice 36* Déterminer toutes les suites (an ) (n > 1) d’entiers strictement positifs telle
que pgcd (ai , aj ) = pgcd (i, j) pour tous indices i et j.

Exercice 37* (Nombres de Mersenne) Montrer que si 2n − 1 est un nombre premier,


alors n est également premier.

Exercice 38* (URSS 61) Prouver que parmi 39 entiers consécutifs, on peut toujours en
trouver un dont la somme des chiffres (écriture décimale) est divisible par 11.
Est-ce encore vrai pour 38 entiers consécutifs ?

Exercice 39** (Putnam 83) Déterminer un nombre réel x > 0 tel que, pour tout entier
n > 0, le nombre [xn ] a la même parité que n.

Exercice 40** (SL 96) Construire une fonction f : N → N bijective et vérifiant :

f (3mn + m + n) = 4f (m) f (n) + f (m) + f (n)

pour tous entiers m et n.

Exercice 41** En utilisant le théorème de répartition des nombres premiers, montrer que
l’ensemble : ½ ¾
p
, p et q premiers
q
est dense dans R+ .

Exercice 42** (Moscou 95) Montrer qu’il existe une infinité d’entiers composés n pour
lesquels n divise 3n−1 − 2n−1 .

Exercice 43** (Théorème de Miller) Montrer qu’il existe un réel x tel que la suite
définie par x0 = x et xn+1 = 2xn est telle que pour tout n, [xn ] est un nombre premier. (On
pourra utiliser le postulat de Bertrand).

Exercice 44** (OIM 75) Peut-on placer 1975 points sur le cercle unité dont les distances
deux à deux sont toutes rationnelles ?

Exercice 45** On note σ (n) la somme des diviseurs positifs de l’entier n. Pour tout entier
p > 0, on pose :
f (m) = max {n ∈ N? / σ (n) 6 m}
Montrer que, pour tout entier k > 0, l’équation m − f (m) = k a une infinité de solutions.

Exercice 46** (Chine 88) Déterminer le plus petit n > 3 pour lequel pour toute écriture
{3, . . . , n} = A ∪ B, l’équation xy = z a une solution pour x, y et z non nécessairement
distincts, et tous les trois dans A ou tous les trois dans B.

20
Exercice 47** Soit p > 5 un nombre premier. Calculer :
p−1 · 3 ¸ (p−1)(p−2) h i
X k X p
3
et kp
k=1
p k=1

Exercice 48** (Italie 04) a) Montrer qu’il existe 2004 puissances parfaites distinctes en
progression arithmétique.
b) Est-il possible de trouver une suite arithmétique infinie formée exclusivement de
puissances parfaites ?

Exercice 49** Soit n > 0 un entier. Montrer qu’il n’existe pas de rationnels x et y tels
que :
1 1
x + y + + = 3n
x y

Exercice 50** (Moldavie 96) Soit n = 213 × 311 × 57 . Déterminer le nombre de diviseurs
de n2 inférieurs à n et ne divisant pas n.

Exercice 51** Soient a, b et c des entiers strictement positifs, premiers entre eux dans
leur ensemble, et tels que :
1 1 1
+ =
a b c
Prouver que a + b est un carré parfait.

Exercice 52** Un nombre n est dit parfait si σ (n) = 2n (où σ désigne la somme des
diviseurs positifs). Montrer que : ¡ ¢
a) (Euler) l’entier n est parfait pair si et seulement s’il est de la forme 2k−1 2k − 1 avec
2k − 1 premier ;
b) (Sylvester) si n est parfait impair, alors il possède au moins trois diviseurs premiers
distincts.

Exercice 53** (TDV 99) Montrer que si a et b sont des entiers tels que ppcm (a, a + 5) =
ppcm (b, b + 5) alors a = b.
Existe-t-il des entiers strictement positifs a, b et c tels que ppcm (a, b) = ppcm (a + c, b + c) ?

Exercice 54** Soient a, b et c des entiers strictement positifs tels que :


a b c
+ +
b c a
est entier. Montrer que abc est un cube.

Exercice 55** Soit n un entier. On suppose que n = ab = cd pour certains entiers positifs
a, b, c et d. Montrer que ak + bk + ck + dk est un entier composé pour tout entier k positif.

Exercice 56** (OIM 94) Trouver tous les couples (m, n) d’entiers strictement positifs tel
que :
n3 + 1
mn − 1

21
soit entier.

Exercice 57** (SL 99) Montrer que tout rationnel strictement positif peut s’écrire sous
la forme :
a3 + b3
c3 + d3
pour certains entiers a, b, c et d strictement positifs.

Exercice 58** Montrer que tout rationnel compris strictement entre 0 et 1 peut s’écrire
sous la forme :
1 1
+ ... +
n1 nk
pour certains entiers ni deux à deux distincts.

Exercice 59** (Balkans 96) Soit p > 5 un nombre premier. On définit :


© ª
S = p − n2 , n ∈ N, n2 < p

Prouver qu’il existe a et b dans S tels que 1 < a < b et a divise b.

Exercice 60** (OIM 87) On considère le plan euclidien. Soit n > 3 un entier. Montrer
qu’il existe n points vérifiant :
(1) trois quelconques de ces points ne sont pas alignés
(2) la distance entre deux quelconques de ces points est irrationnelle
(3) l’aire du triangle déterminé par trois quelconques de ces points est rationnelle.

Exercice 61** (APMO 98) √ Trouver le plus grand entier n qui soit divisible par tous les
entiers inférieurs ou égaux à 3 n.

Exercice 62** (OIM 92) Trouver tous les entiers a, b, c vérifiant 1 < a < b < c et tels
que (a − 1) (b − 1) (c − 1) divise abc − 1.

Exercice 63** (Inde 98) Soit M un entier strictement positif. On note :


© ª
S = n ∈ N, M 2 6 n < (M + 1)2

Montrer que les produits ab pour a et b dans S sont deux à deux distincts.

Exercice 64** Pour tout entier n > 0, on pose :


1 1 1
Hn = 1 + + + ... +
2 3 n
Montrer que Hn est entier si, et seulement si n = 1. Déterminer les entiers m et n pour
lesquels la différence Hm+n − Hm est entière ?
√ √
Exercice 65** Soient a < b 6 c < d des entiers tels que ad = bc et d − a 6 1. Prouver
que a est un carré.

22
Exercice 66** (OIM 83) Soient a, b et c des entiers strictement positifs et premiers entre
eux deux à deux. Montrer que 2abc − ab − bc − ca est le plus grand entier qui ne peut pas
s’écrire sous la forme xbc + yca + zab avec x, y, z entiers positifs ou nuls.

Exercice 67** (Putnam 95) Pour α un réel strictement positif, on note S (α) = {[nα] , n ∈ N? }.
Montrer que N? ne peut pas s’écrire comme union disjointe de S (α), S (β) et S (γ) pour
trois réels strictement positifs α, β et γ.

Exercice 68** Montrer qu’il Pn’existe pas de partie X ⊂ N infinie telle que pour toute
partie finie I ⊂ X, le nombre x∈I x soit un carré parfait.

Exercice 69** (CG 92) Quelle est le chiffre des unités du nombre suivant :
· ¸
101992
1083 + 7

Exercice 70*** (Yakusk 00) Prouver qu’il n’existe pas d’entiers n > 0 et a1 < . . . < ak
tels que :
1 1 1
+ ··· + = n
a1 ! ak ! 10

Exercice 71*** (OIM 98) Pour tout entier n strictement positif, d (n) désigne le nombre
de diviseurs positifs de n (y compris 1 et n). Trouver tous les entiers strictement positifs k
pour lesquels il existe n tel que :
d (n2 )
=k
d (n)

Exercice 72*** (OIM 84) Soient a, b, c et d des entiers positifs impairs vérifiant a < b <
c < d, ad = bc et a + d = 2k , b + c = 2m pour deux entiers k et m. Prouver que a = 1.

23
2 Division euclidienne et conséquences
2.1 Division euclidienne et décomposition en base b
Les principales propriétés arithmétiques des entiers découlent de l’existence de la division
euclidienne.

Théorème 2.1.1 (Division euclidienne) Soit b un entier non nul. Tout entier a s’écrit
de manière unique sous la forme a = bq + r, avec q entier et 0 6 r < |b|. Les entiers q et r
sont appelés respectivement quotient et reste de la division euclidienne de a par b.
Remarque. Ainsi a est divisible par b si et seulement si r = 0.
Comme pour les parties entières, on prendra garde à ce qui se produit lorsque l’un des
nombres a et b est négatif.

Démonstration. Montrons tout d’abord £a¤ l’existence. On peut supposer b > 0 dans un
premier temps. On prend alors q = b et r = a − bq. De l’inégalité q 6 ab < q + 1, on déduit
aisément 0 6 r < b. Si b < 0, on se ramène au cas précédent en considérant −b.
En ce qui concerne l’unicité, si a s’écrit a = bq + r = bq 0 + r0 , alors b(q − q 0 ) = r0 − r donc
b divise r0 − r. Comme |b| > |r0 − r|, nécessairement r0 − r = 0, d’où r0 = r puis q 0 = q. ¤

Théorème 2.1.2 (Décomposition en base b) Soit b > 2 un entier. Tout entier a > 0
s’écrit de façon unique sous la forme :

a = a0 + a1 b + a2 b2 + · · · + ak bk

où k est un entier, les ai sont des entiers compris entre 0 et b − 1 et où ak 6= 0.
On note parfois a = ak ak−1 . . . a0 b . Cette notation est l’écriture en base b de a.
Remarque. Dans le cas où b = 10, les ai correspondent exactement aux chiffres usuels de
a. On s’aperçoit que 10 ne joue pas un rôle particulier vis-à-vis de la représentation des
nombres : par exemple, on aurait pu noter 143 au lieu de 80 si on avait décidé de compter
en base 7.

Démonstration. La méthode consiste à effectuer des divisions euclidiennes (par b) succes-


sives. On commence par écrire a = bq0 + a0 avec a0 ∈ {0, 1, . . . , b − 1}. Si q0 = 0, on a fini !
Sinon, on continue nos divisions en écrivant q0 = bq1 + a1 avec a1 ∈ {0, 1, . . . , b − 1}. On a
alors :
a = a0 + a1 b + q1 b2
De même si q1 = 0, on a fini. Sinon on continue, construisant ainsi a3 , a4 et ainsi de suite.
On obtient successivement des égalités du type :

a = a0 + a1 b + · · · + ai bi + qi bi+1

La suite des qi est une suite d’entiers positifs strictement décroissante. Elle doit donc s’ar-
rêter, ce qu’ici ne peut être réalisé que si qi = 0. À ce moment, on a bien la décomposition
annoncée.

24
Reste à prouver l’unicité. Supposons que l’on puisse écrire :

a0 + a1 b + · · · + ak bk = a00 + a01 b + · · · + a0k bk

pour des entiers ai et a0i compris entre 0 et b − 1. Alors a0 − a00 est un multiple de b et
|a0 − a00 | < b. D’où a0 = a00 . On simplifie alors par a0 , puis on divise par b. En appliquant le
même argument que précédemment, on obtient a1 = a01 et ainsi de suite. ¤

Ci-dessous, on présente un moyen pratique d’effectuer les calculs pour calculer la décom-
position d’un nombre en base b. Ici a = 80 et b = 7 :

80 7
3 11 7
41
7
En lisant les restes à l’envers, on obtient l’écriture de 80 en base 7, en l’occurrence 80 = 143 .

L’écriture en base b permet de reformuler le théorème de Legendre qui donne la valuation


p-adique d’une factorielle :

Théorème 2.1.3 Soit p un nombre premier. Soit n un entier naturel. On a :

n − sp (n)
vp (n!) =
p−1

où sp (n) désigne la somme des chiffres de n en base p.


Démonstration. Considérons la décomposition de n en base p :

n = nd pd + nd−1 pd−1 + · · · + n1 p + n0

Alors, pour tout entier i, on a :


· ¸
n
= ni + ni+1 p + · · · + nd pd−i
pi

Donc, d’après la formule de Legendre, on a :


¡ ¢
vp (n!) = n1 + n2 p + · · · + nd pd−1
¡ ¢
+ n2 + n3 p + · · · + nd pd−2 + · · · + (nd−1 + nd p) + (nd )
¡ ¢
= n1 + n2 (1 + p) + · · · + nd 1 + p + · · · + pd−1
¡ ¢
n1 (p − 1) + n2 (p2 − 1) + nd pd − 1
=
p−1
n − sp (n)
=
p−1
¤

25
L’énoncé du théorème sous-entend que p − 1 divise toujours la quantité n − sp (n), ce qui
peut se voir facilement par ailleurs. En effet, la factorisation :
¡ ¢
pk − 1 = (p − 1) pk−1 + · · · + p + 1

prouve que p − 1 divise toujours pi − 1. Par ailleurs, on a, en gardant les notations du


théorème : ¡ ¢ ¡ ¢
n − sp (n) = nd pd − 1 + nd−1 pd−1 − 1 + · · · + n1 (p − 1)
et la conclusion en découle directement. On remarque en particulier que la primalité de
p n’intervient pas pour cette dernière propriété. Bref, on vient de prouver la proposition
suivante parfois utile :

Proposition 2.1.4 Soit b > 2 un entier. Si sb (n) désigne la somme des chiffres de l’entier
n écrit en base b, alors le nombre sb (n) − n est toujours un multiple de b − 1.

Décomposition en base b des nombres rationnels


Soit b > 2 un entier. Si x est un nombre réel, on peut définir sa décomposition en base b :
un moyen économique est de définir le n-ième chiffre après la virgule de x comme le dernier
chiffre de l’entier [bn x] ou autrement dit le reste de la division euclidienne de [bn x] par b.

Théorème 2.1.5 L’entier b est toujours fixé. Un nombre réel x est rationnel si, et seule-
ment si sa décomposition en base b est périodique à partir d’un certain rang.
Démonstration. Nous n’allons pas démontrer ce théorème, mais plutôt mettre en valeur
les idées sous-jacentes de la preuve.
Supposons pour simplifier que b = 10 (cela ne change en rien les choses). Nous partons
d’un rationnel r = xy et nous voulons prouver que sa décomposition en base 10 est périodique
à partir d’un certain rang. Pour cela, il suffit de poser la division de x par y. Supposons
pour exemple que x = 5 et y = 14. On écrit :

5 14
50 0 , 357 142 85
80
100
20
60
40
120
80
10

On retombe finalement sur un reste déjà rencontré (ce qui est automatique étant donné
qu’il n’y a qu’un nombre fini (en l’occurrence y) de restes possibles), et donc on retrouve les
mêmes décimales lorsque l’on poursuit l’opération. L’écriture décimale est périodique.
Réciproquement supposons que l’on dispose d’un réel x donc l’écriture décimale est
périodique à partir d’un certain rang. Prenons à nouveau un exemple. Au hasard x =

26
0 , 410 784 153 153 153 (la partie surlignée étant celle qui se répètera). On cherche une
fraction qui soit égale à x. Pour cela, on écrit :

1000x = 410 , 784 153 153 153


− x = 0 , 410 784 153 153
999x = 410 , 373 216

ce qui fournit x = 410 ,999


373 216
= 410 373 216
999 000 000
= 34 197 773
83 250 000
et qui permet de conclure. Le cas général
fonctionne exactement de la même manière. ¤

Remarque. Comme nous l’avons vu dans l’exemple précédent, l’écriture en base b des chiffres
après la virgule d’un nombre rationnel est périodique à partir d’un certain rang et pas
forcément dès le premier chiffre. En réalité, on peut démontrer que la suite des chiffres
après la virgule en base b d’un nombre rationnel r est périodique dès le premier chiffre si, et
seulement si r peut se mettre sous la forme r = xy avec y premier avec b.

2.2 Algorithme d’Euclide


L’algorithme d’Euclide est une méthode efficace pour déterminer le pgcd de deux entiers
donnés. Il est basé sur l’égalité suivante :

pgcd (a, b) = pgcd (b, r)

si a = bq + r pour des entiers a, b, q et r. La démonstration de cette propriété est immédiate.


L’algorithme fonctionne alors ainsi. Supposons donnés deux entiers a et b positifs tels
que a > b. On effectue la division euclidienne de a par b :

a = bq0 + r0

et d’après la propriété précédente, on est ramené à calculer le pgcd des entiers b et r0 .


Deux cas se présentent alors : si r0 = 0, le pgcd cherché est b. Sinon, on effectue la division
euclidienne de b par r0 :
b = r0 q1 + r1
et le pgcd cherché vaut celui de r0 et r1 . Si r1 = 0, on a fini. Sinon, on continue...
Les ri forment une suite d’entiers positifs ou nuls strictement décroissante (d’après les
propriétés de la division euclidienne). Cette suite ne peut pas être infinie, ce qui prouve que
l’algorithme doit s’arrêter. La description de cet algorithme prouve qu’il s’arrête automati-
quement avec un reste nul. À ce moment, le précédent reste fournit le pgcd cherché.

Examinons un exemple. Supposons que l’on désire calculer le pgcd des entiers 56 et 98.
On constitue la liste suivante :
98 , 56 , 42 , 14 , 0
où les deux premiers nombres sont ceux dont on veut calculer le pgcd et où les autres sont
obtenus en calculant le reste de la division euclidienne des deux nombres qui les précèdent
immédiatement. Le pgcd est le dernier entier non nul ainsi écrit ; ici, c’est 14.

27
L’algorithme précédent est également tout à fait adapté pour le calcul de pgcd (an − 1, am − 1)
lorsque a > 2, m > 1 et n > 1 sont des entiers. En effet, si la division euclidienne de n par
m s’écrit :
n = mq + r
alors la division euclidienne de an − 1 par am − 1 s’écrit :
¡ ¢
an − 1 = (am − 1) a(q−1)m+r + a(q−2)m+r + · · · + ar + (ar − 1)

et donc en itérant, on obtient la proposition suivante :

Proposition 2.2.1 Soient a, m et n des entiers strictement positifs. Alors on a l’égalité :

pgcd (an − 1, am − 1) = apgcd(m,n) − 1

En particulier, on constate que l’algorithme d’Euclide peut être utilisé pour déterminer des
pgcd même si les nombres auxquels on s’intéresse ne sont pas donnés sous forme de valeurs
numériques.

2.3 Algorithme d’Euclide étendu et théorème de Bézout


À chaque étape de l’algorithme d’Euclide, on a une égalité de la forme :

ri−2 = ri−1 qi + ri

où par convention r−2 = a et r−1 = b. À l’avant-dernière étape, on a rk = d = pgcd (a, b)


et donc une égalité de la forme :

rk−2 = rk−1 qk + d

soit encore :
d = rk−2 − rk−1 qk
À l’étape précédente, on a de même :

rk−1 = rk−3 − rk−2 qk−1

et donc en réinjectant, on obtient une expression de d comme une combinaison linéaire de


rk−3 et rk−2 . En continuant à remonter, on trouve finalement une égalité de la forme :

d = ur−2 + vr−1 = au + bv

pour des entiers u et v. On en déduit le théorème suivant que nous avons déjà mentionné
(voir paragraphe 1.1) :

Théorème 2.3.1 (Bézout) Soient a et b des entiers non simultanément nuls. Notons d =
pgcd (a, b). Alors il existe des entiers u et v tels que :

d = au + bv

En particulier, a et b sont premiers entre eux si, et seulement s’il existe des entiers u et
v tels que au + bv = 1.

28
Il existe une présentation des calculs pour déterminer efficacement et sans s’embrouiller
les coefficients u et v mentionnés précédemment. On dessine un tableau de quatre colonnes
que l’on commence à remplir comme suit :
Quotient Reste a b
a 1 0
b 0 1
Les lignes suivantes sont obtenues comme l’explique le schéma suivant :
Quotient Reste a b
.. .. .. ..
. . . .
rn−2 un−2 vn−2
rn−1 un−1 vn−1
qn rn un−2 − qn un−1 vn−2 − qn vn−1
où qn et rn désignement respectivement le quotient et le reste de la division euclidienne de
rn−2 par rn−1 . On remarque déjà dans un premier temps que la colonne des restes correspond
exactement aux résultats successifs du calcul explicité dans le paragraphe précédent. Ainsi
le dernier reste non nul fournit le pgcd de a et b. Par ailleurs, on vérifie par récurrence qu’à
chaque ligne, on a :
rn = un a + vn b
ce qui nous donne une expression du pgcd comme combinaison linéaire de a et de b.
Regardons l’exemple a = 153 et b = 71. En suivant les consignes précédentes, on dessine
le tableau suivant :
Quotient Reste a b
153 1 0
71 0 1
2 11 1 −2
6 5 −6 13
2 1 13 −28
5 0
obtenant finalement :
1 = 13 × 153 − 28 × 71

2.4 Lemme de Gauss et conséquences


Le théorème de Bézout implique un autre résultat important :

Théorème 2.4.1 (Lemme de Gauss) Si des entiers a, b et c sont tels que a divise bc et
a est premier avec b, alors a divise c.
Démonstration. Comme a est premier avec b, on peut écrire au + bv = 1 pour des entiers
u et v. Ainsi auc + bvc = c et comme a divise auc (car il divise a) et bvc (car il divise bc), il
divise la somme qui vaut c. ¤
Une première conséquence du lemme de Gauss est le lemme 1.2.3 utilisé lors de la preuve
de l’unicité de la décomposition en facteurs premiers, à savoir :

29
Lemme 2.4.2 Si un nombre premier p divise le produit a1 · · · an , alors il divise l’un des ai .
Démonstration. Supposons que p ne divise aucun des ai . Comme les seuls diviseurs positifs
de p sont 1 et p, les nombres p et a1 sont forcément premiers entre eux. On en déduit, par
le lemme de Gauss, que p divise a2 · · · an (puisque p est premier avec a1 ). Ensuite, p divise
a3 · · · an , puis en itérant il divise an , ce qui est supposé faux. ¤

Deux autres conséquences très importantes et très utiles du lemme de Gauss sont données
respectivement en proposition et en exercice :

Proposition 2.4.3 Si deux entiers premiers entre eux a et b divisent n, alors le produit ab
divise également n.
Démonstration. Comme a divise n, on peut écrire n = ak pour un certain entier k. Mais
alors b divise ak et comme il est premier avec a, il divise k. Ainsi k = bk 0 pour un entier k 0
et puis n = abk 0 , ce qui prouve bien que ab divise n. ¤

Exercice : Soient a et b deux entiers premiers entre eux. On note x0 et y0 des entiers tels que
ax0 + by0 = 1. Soit d un entier. Trouver tous les entiers x et y vérifiant :

ax + by = d

Solution : On remarque dans un premier temps que le couple (dx0 , dy0 ) est solution. Soit
(x, y) une autre solution. On a alors ax + by = d et adx0 + bdy0 = d et donc en faisant la
différence :
a (x − dx0 ) = −b (y − dy0 )
On en déduit que a divise b (y − dy0 ) et comme il est premier avec b, il divise y − dy0 . Ainsi,
il existe un entier k tel que y = dy0 +ka. Finalement, en reportant dans l’équation de départ,
on arrive à x = dx0 − kb.
Réciproquement on vérifie que x et y ainsi définis constituent bien une solution pour
tout entier relatif k. Finalement, les solutions sont les couples (dx0 − kb, dy0 + ka) pour √ k
entier relatif.

Remarque. Résoudre en entiers l’équation ax + by = d revient géométriquement à trouver


les points à coordonnées entières sur la droite d’équation cartésienne ax + by = d.

De façon plus anecdotique, on peut chercher à résoudre l’équation ax+by = d en nombres


entiers naturels. On a, dans ce sens, la proposition suivante :

Proposition 2.4.4 (Coin exchange problem of Frobenius4 ) Soient a et b deux entiers


strictement positifs et premiers entre eux. Le nombre relatif d peut s’écrire sous la forme
ax + by pour des entiers x et y positifs ou nuls si et seulement si le nombre ab − a − b − x
ne peut pas s’écrire sous cette forme.
En particulier, ab − a − b est le plus grand entier qui ne puisse pas s’écrire ax + by où x
et y sont des entiers positifs ou nuls.

30
Démonstration. Notons x0 et y0 des entiers tels que ax0 + by0 = 1. On a vu dans l’exercice
précédent que les solutions (en entiers relatifs) de l’équation ax + by = d sont x = dx0 − kb
et y = dy0 + ka. Ainsi, l’équation admet une solution en nombre entiers positifs ou nuls si
et seulement s’il existe un entier k tel que dx0 − kb ¤ > −1 et dy£0 + ka > −1, autrement dit
dy0 +1 dx0 +1
si et seulement s’il y a un entier dans l’intervalle − a , b .
¤ dy0 +1 dx +1 £
Il s’agit donc de prouver qu’il yia un entier dans l’intervalle
h − a , 0b si et seulement
s’il n’y en a pas dans l’intervalle − (D−d)ya
0 +1 (D−d)x0 +1
, b
où on pose D = ab − a − b. Or, si
n est un entier, les propriétés suivantes sont équivalentes :
(D − d)y0 + 1 (D − d)x0 + 1
− <n<
a b
by0 dy0 1 ax0 dx0 1
−by0 + y0 + + − < n < ax0 − x0 − − +
a a a b b b
En se rappelant que ax0 + by0 = 1, l’inéquation précédente se simplifie considérablement et
devient :
dy0 dx0
−by0 + < n + x0 − y0 < ax0 −
a b
ou encore :
dy0 dx0
< n + x0 − y0 + by0 < 1 −
a b
puis, en passant aux opposés :
dx0 dy0
− 1 < −n − x0 + y0 − by0 < −
b a
¤ £
Désormais, il suffit donc de montrer qu’il y a un entier dans l’intervalle − dy0a+1 , dx0b+1 si et
¤ £
seulement s’il n’y en a pas dans l’intervalle dxb 0 − 1, − dya0 .
Déjà, on remarque que le premier intervalle n’est jamais vide, puisque l’on a bien :
dx0 + 1 dy0 + 1 d+a+b
+ = >0
b a ab
Le second intervalle peut être vide. En effet :
dy0 dx0 d
− − +1=1−
a b ab
qui peut être négatif si d > ab. Seulement dans ce cas, le premier intervalle est d’amplitude
strictement supérieure à 1 et donc contient forcément un entier : l’équivalence est donc bien
vérifiée. ¤ £
Sinon, on remarque que l’intersection de deux intervalles consiste en l’intervalle − dy0a+1 , − dya0
qui
¤ dx ne peut contenir
£ aucun entier, et que par contre
¤ dx la réunion¤ consiste en l’intervalle
dx0 +1 dx0
b
0
− 1, b
qui contient autant d’entiers que b
0
− 1, b
, c’est-à-dire un et un seul.
Ceci démontre la proposition. ¤

Exercice : Soient a et b des entiers strictement positifs et premiers entre eux. Montrer que le
nombre d’entiers positifs qui ne peuvent pas se mettre sous la forme ax + by pour des entiers
positifs ou nuls x et y est donné par la formule :
(a − 1) (b − 1)
2

31
Solution : D’après la proposition précédente tous les nombres strictement supérieurs à d =
ab − a − b peuvent se mettre sous la forme de l’énoncé. D’autre part si n ∈ {0, . . . , d}, on sait
que n peut se mettre sous la forme en question si, et seulement si d − n ne le peut pas. Or
l’application n 7→ d − n réalise une bijection de l’ensemble {0, . . . , d} sur lui-même. On en
déduit qu’exactement la moitié des nombres de cet ensemble s’écrivent sous la forme voulue. √
Cela e fait d+1
2
, soit encore la formule donnée dans l’énoncé.

2.5 Exercices
101010101
Exercice 73 (Théorème de Anning) Montrer que la valeur de la fraction 110010011 dont
le numérateur et le dénominateur sont écrits en base b ne change pas si on remplace le 1
central du numérateur et du dénominateur par un nombre impair quelconque de 1.

Exercice 74 Montrer que tout entier naturel n peut s’écrire de façon unique sous la forme :

n = a1 1! + a2 2! + a3 3! + · · · + ad d! + · · ·

où a1 , a2 , a3 , · · · sont des entiers tels que 0 6 ai 6 i pour tout i.

Exercice 75 Soit a1 > a2 > 0 des entiers. L’algorithme d’Euclide fournit une suite d’en-
tiers :
a1 , a2 , a3 , . . . , an−1 , an , 0
où l’on rappelle que ai+1 est défini comme le reste de la division euclidienne de ai par ai−1
et le dernier reste non nul an est le pgcd de a1 et a2 .
Montrer que l’entier n vérifie l’inégalité Fn−1 6 a2 où (Fn ) est la suite de Fibonacci
définie par F0 = 0, F1 = 1 et Fn = Fn−1 + Fn−2 pour n > 2.

Exercice 76 Trouver tous les entiers a, b et c vérifiant l’équation 5a + 3b + 15c = 2.

Exercice 77 (Canada 85) Trouver tous les entiers n tels que 2n−1 divise n!.

Exercice 78* (Yougoslavie 99) Soit n > 0 un entier. On note sn la somme des chiffres de
l’écritrure décimale de n. Existe-t-il un entier n > 0 tel que s(n) = 1997 et s(n2 ) = 19972 ?

Exercice 79* On note ϕ (n) le nombre d’entiers positifs inférieurs à n et premiers avec n.
Montrer que si n > 2, ϕ (n) est toujours pair.

Exercice 80* (Allemagne 95) Dans le plan, un jeton est déplacé selon les règles suivantes ;
i) De tout point (a, b), on peut le déplacer en (a, 2b) ou (2a, b) ;
ii) De tout point (a, b) avec a > b, on peut le déplacer en (a − b, b). Et si a < b, on peut
le déplacer en (a, b − a).
Le jeton est initialement en (1, 1). Déterminer une condition nécessaire et suffisante sur x et
y pour que l’on puisse amener le jeton en (x, y) en un nombre fini d’étapes.

Exercice 81* Pour tout n strictement positif, on note P (n) le produit des chiffres non
nuls de l’écriture de n en base 10. Un entier n est dit prodigieux lorsque P (n) divise n.
Prouver qu’il n’existe pas 14 entiers consécutifs qui soient tous prodigieux.

32
Exercice 82* (Hollande 04) Soit (un ) une suite d’entiers vérifiant u1 = 2, u2 = 3 et
un+1 = 2un−1 ou un+1 = 3un − 2un−1 pour tout n > 2. Montrer que pour tout n, l’entier un
a au plus deux chiffres non nuls en base 2.

Exercice 83* (Roumanie 97) Soient n > 3 un entier et x un réel positif ou nul. Prouver
que les nombres x, x2 , xn ont la même partie décimale si, et seulement si x est un entier.

Exercice 84* (URSS 71) Démontrer que pour tout entier n > 0 il existe un nombre dont
l’écriture décimale n’utilise que des 1 et des 2, et qui est divisible par 2n .

Exercice 85* (URSS 89) a) Soient a et b des réels distincts tels que, pour tout entier
naturel n, le nombre an − bn soit entier. Les nombres a et b sont-ils rationnels ? Sont-ils
entiers ?
b) Existe-t-il des réels distincts a et b pour lesquels le nombre a + b est rationnel alors
que an + bn est irrationnel pour tout entier n > 2 ?
c) Existe-t-il des réels distincts a et b pour lesquel le nombre a + b est irrationnel alors
que an + bn est rationnel pour tout entier n > 2 ?

Exercice 86* (Devinette) Les martiens sont un peu bizarres quand même. Ma voisine par
exemple en est une. Outre le fait qu’elle ait six doigts à chaque main, j’ai l’impression qu’elle
écrit des inepties sous son cahier de maths. Par l’exemple, l’autre jour, j’ai vu la formule :

(5x + 3) (3x − 7) = 13x2 − 22x − 19

Pouvez-vous m’aider à comprendre, car j’ai entendu dire qu’elle était très forte en maths, et
ne se trompait jamais en calcul ?

Exercice 87* (Problem of the month – Regina) La suite S de Kolakoski :

1, 2, 2, 1, 1, 2, 1, 2, 2, 1, 2, 2, . . .

est un exemple de suite « qui se lit elle-même ». Elle est constituée de groupes de 1 et de
groupes de 2 en alternance, la longueur du n-ième groupe étant la valeur du n-ième terme
de la suite.
Prouver que le nombre x = 0, 122 112 122 122 . . . est irrationnel.

Exercice 88* (OIM 85) Soit n un entier naturel, k un entier premier avec n, 1 6 k 6 n−1,
M l’ensemble {1, 2, . . . , n − 1}. Chaque élément de M est coloré avec l’une des deux couleurs
blanche ou bleue. On suppose :
(1) pour tout i de M , i et n − i ont la même couleur,
(2) pour tout i de M , i 6= k, i et |i − k| ont la même couleur.
Montrer que tous les éléments de M ont la même couleur.

Exercice 89* (Japon 96) Soient m et n des entiers premiers entre eux. Calculer le pgcd
des entiers 5m + 7m et 5n + 7n .

Exercice 90* Soit p1 , p2 , . . . , pn , . . . la suite des nombres premiers. Montrer que le nombre
dont l’écriture décimale est :
0, p1 p2 p3 . . . pn . . .

33
est irrationnel.

Exercice 91* Trouver tous les entiers n > 1 pour lesquels la somme 22 + 32 + · · · + n2 est
une puissance d’un nombre premier.

Exercice 92* Dans ma boı̂te de céréales, j’ai trouvé le jeu suivant. Il se compose des six
cartes que je reproduis ci-dessous.

1 3 5 7 2 3 6 7 4 5 6 7
9 11 13 15 10 11 14 15 12 13 14 15
17 19 21 23 18 19 22 23 20 21 22 23
25 27 29 31 26 27 30 31 28 29 30 31
33 35 37 39 34 35 38 39 36 37 38 39
41 43 45 47 42 43 46 47 44 45 46 47
49 51 53 55 50 51 54 55 52 53 54 55
57 59 61 63 58 59 62 63 60 61 62 63

8 9 10 11 16 17 18 19 32 33 34 35
12 13 14 15 20 21 22 23 36 37 38 39
24 25 26 27 24 25 26 27 40 41 42 43
28 29 30 31 28 29 30 31 44 45 46 47
40 41 42 43 48 49 50 51 48 49 50 51
44 45 46 47 52 53 54 55 52 53 54 55
56 57 58 59 56 57 58 59 56 57 58 59
60 61 62 63 60 61 62 63 60 61 62 63

Pour jouer, je dois demander à un partenaire de penser secrètement à un nombre compris


entre 1 et 63, puis de me montrer les cartes sur lequel son nombre apparaı̂t. Normalement,
je suis supposé savoir retrouver le nombre choisi avec ces seules informations, mais la partie
qui explique cette déduction a été déchirée lors de l’ouverture de la boı̂te.
Peux-tu m’aider à retrouver comment on fait ?

Exercice 93* Soit A l’ensemble des entiers strictement positifs dont l’écriture en base
3 n’utilise pas le chiffre 2. Montrer que trois éléments de A ne sont jamais en progression
arithmétique.

Exercice 94* Trouver tous les entiers a > 0 et b > 2 tel que 2a + 1 soit un multiple de
2b − 1.

Exercice 95* Soit P un polynôme à coefficients entiers. On définit la suite (an ) en posant
a0 = 0 et ai+1 = P (ai ) pour tout i > 0. Montrer que :

pgcd (am , an ) = apgcd(m,n)

34
Exercice 96* (Entiers de Gauss) On note :

Z [i] = {x + iy, x, y ∈ Z}

Soient a, b ∈ Z [i] avec b 6= 0. Montrer qu’il existe des éléments q et r dans Z [i] tels que
a = bq + r et |r| < |b|. Cette écriture est-elle unique ?

Exercice 97* (Ibéroamérique 94) Un entier n > 0 est dit brésilien s’il existe r < n − 1
pour lequel n s’écrit en base r avec des chiffres tous égaux. Montrer que 1994 est brésilien
mais que 1993 ne l’est pas.

Exercice 98* (Crux Mathematicorum) Soit k > 2 un entier fixé. Pour tout entier n > 0,
on désigne par xn le chiffre de gauche dans l’écriture décimale du nombre nk . Prouver que
le nombre :
0, x0 x1 x2 . . . xn . . .
est irrationnel.

Exercice 99* (APMO 94) Dans le première colonne (resp. deuxième colonne) du tableau
ci-dessous, on écrit les nombres 10k en base 2 (resp. en base 5).

1010 20
1100100 400
1111101000 13000
.. ..
. .
Soit n > 1 un entier. Prouver qu’il apparaı̂t dans le tableau précédent un et un unique
nombre de n chiffres.

Exercice 100* (Problem of the month – Regina) Déterminer tous les couples (d, n)
d’entiers strictement positifs tels que d divise n et nd + 1 divise n2 + d2 .

Exercice 101* (URSS 65) Soit n > 0 un entier. Prouver que tout entier inférieur ou égal
à n! peut s’écrire comme la somme d’au plus n diviseurs de n! deux à deux distincts.

Exercice 102** (OIM 91) Soient n > 6 un entier et a1 , . . . , ak tous les entiers compris
strictement entre 0 et n qui sont premiers avec n. On suppose :

a2 − a1 = a3 − a2 = . . . = ak − ak−1 > 0

Montrer que n est soit un nombre premier, soit une puissance entière de 2.
£ ¤
Exercice 103** On définit la suite (an ) par a1 = 2, an+1 = 32 an . Montrer que an est pair
(resp. impair) pour une infinité de valeurs de n.

Exercice 104** (Russie 01) Déterminer tous les entiers n > 1 tels que, pour tous diviseurs
a et b de n premiers entre eux, le nombre a + b − 1 est aussi un diviseur de n.

Exercice 105** (Slovénie 99) Trois boı̂tes contiennent chacune au moins un jeton. Une
opération consiste à choisir deux boı̂tes et à transvaser des jetons de l’une à l’autre de façon

35
à doubler le nombre de jetons dans la boı̂te d’arrivée. Est-il possible de vider l’une des boı̂tes
en un nombre fini d’opérations ?

Exercice 106** Soit x0 ∈ [0, 1]. On définit la suite xn par xn+1 = 1 − |1 − 2xn |. Montrer
que (xn ) est périodique à partir d’un certain rang si, et seulement si x0 est rationnel.

Exercice 107** La suite de Fibonacci est définie par F0 = 0, F1 = 1 et Fn = Fn−1 + Fn−2


pour n > 2.
a) Montrer que Fn+p = Fp−1 Fn + Fn+1 Fp pour tous n et p.
b) Montrer en utilisant la formule précédente que si d = pgcd (m, n), alors pgcd (Fm , Fn ) =
Fd

Exercice 108** (Pologne 96) Pour tout entier k strictement positif, on désigne par p (k)
le plus petit nombre premier ne divisant pas k, et par q (k) le produit de tous les nombres
premiers strictement inférieurs à p (k) (si p (k) = 2, on convient que q (k) = 1).
On définit une suite en posant x0 = 1 et xn+1 = xn p(x n)
q(xn )
. Déterminer les entiers n pour
lesquels xn = 111111.

Exercice 109*** (OIM 88) On désigne par f l’application de N? dans lui-même définie
par :
f (1) = 1, f (3) = 3
et pour tout n > 1 :

f (2n) = f (n)
f (4n + 1) = 2f (2n + 1) − f (n)
f (4n + 3) = 3f (2n + 1) − 2f (n)

Déterminer le nombre d’entiers n, 1 6 n 6 1988, pour lesquels f (n) = n.

Exercice 110*** (Lituanie 94) Si N est un entier, on note S (N ) la somme des chiffres
(en base 10) de N . Montrer que la suite S (2n ) tend vers l’infini.

36
3 Congruences
3.1 Définition, premières propriétés
Tout un chacun sait que l’on peut répartir les entiers en deux catégories : les nombres
pairs, et les nombres impairs. Et que cette répartition est compatible avec les opérations ;
par exemple, la somme d’un nombre pair et d’un nombre impair est impaire, le produit d’un
nombre pair et d’un nombre impair est pair, etc. En fait, cela est souvent bien pratique. Les
congruences généralisent ce type de raisonnement.

Définition 3.1.1 Soit N > 1 un entier. Deux entiers a et b sont dits congrus modulo N
lorsque N divise b − a (ou de façon équivalente a − b). On note a ≡ b (mod N ).
La relation de congruence vérifie les propriétés suivantes (immédiates) :
Propriétés
☞ On a a ≡ 0 (mod N ) si, et seulement si N divise a.
☞ Si a ≡ b (mod N ) et b ≡ c (mod N ), alors a ≡ c (mod N ).
☞ On a a ≡ b (mod N1 ) et a ≡ b (mod N2 ) si, et seulement si a ≡ b (mod ppcm (N1 , N2 )).
☞ Tout entier est congru modulo N à un et un unique élément de l’ensemble {0, . . . , N − 1}.
Il s’agit précisément du reste de la division euclidienne de cet entier par N . On dit
parfois que l’ensemble {0, . . . , N − 1} est un système complet de résidus modulo N .
Un élément d’un système complet de résidu modulo N est parfois appelé un résidu.
☞ Les entiers congrus à a modulo N sont les entiers de la forme a + kN , avec k entier.
☞ Si a ≡ b (mod N ) et a0 ≡ b0 (mod N ), alors a + a0 ≡ b + b0 (mod N ) et aa0 ≡ bb0
(mod N ).

Malgré l’évidence apparente des propriétés précédentes, elles s’avèrent vraiment très
utiles à toutes sortes de moments. Par exemple la finitude d’un système complet de résidus
permet, lorsque l’on a une équation à résoudre faisant intervenir des congruences dont le
modulo est un entier connu, de ne tester qu’un nombre fini de cas. Souvent, il y a des astuces
mais il ne faut jamais désespérer si on n’en trouve pas. Par exemple, en testant tous les cas,
on prouve facilement que le carré d’un entier est toujours congru à 0 ou 1 modulo 4. De
même une étude exhaustive peut permettre de résoudre la question suivante :

Exercice : Trouver tous les entiers x tels que 7 divise x2 + x + 1.


Solution : La condition se réécrit x2 + x + 1 ≡ 0 (mod 7). En essayant les sept résidus, on

voit que les seules solutions sont les entiers congrus à 2 ou 4 modulo 7.

Théorème 3.1.2 Soit N > 1 un entier et c un entier premier avec N . Alors il existe un
entier c0 tel que cc0 ≡ 1 (mod N ).
Un tel entier c0 est appelé un inverse de c modulo N .
Démonstration. Il s’agit d’une simple application du théorème de Bézout. Comme N et c
sont premiers entre eux, on peut écrire une égalité du type uN +vc = 1. On voit directement
que l’entier c0 = v convient pour le théorème.

37
On remarque également que l’algorithme d’Euclide étendu donne un moyen effectif pour
calculer l’inverse de c modulo N . ¤
Remarque. L’implication donnée dans le théorème précédent est en réalité une équivalence :
si c admet un inverse modulo N , alors c est premier avec N . En effet, dire que c admet un
inverse modulo N signifie qu’il existe des entiers k et c0 tels que cc0 = 1 + kN . Le sens facile
du théorème de Bézout permet de conclure.
Si c0 est un inverse de c modulo N et que le contexte ne prête pas à confusion, il arrive
que l’on note c0 = c−1 , voire c0 = 1c . En particulier, si q est un entier premier avec N , la
fraction pq pourra désigner un résidu modulo N .

Le théorème précédent n’est en réalité qu’une reformulation du théorème de Bézout


comme le montre la preuve précédente. Il a une conséquence importante qui est la traduction
en termes de congruences du lemme de Gauss :

Proposition 3.1.3 Soient N > 1 un entier et a, b et c des entiers tels que ac ≡ bc


(mod N ). Si c est premier avec N , alors on peut déduire que a ≡ b (mod N ).
Démonstration. Comme c est premier avec N , il admet un inverse modulo N , disons c0 .
En multipliant par c0 la congruence ac ≡ bc (mod N ), on obtient directement le résultat. ¤

Remarque. Si c n’est pas premier avec N , on a simplement une conclusion plus faible qui
est :
N
a ≡ b (mod )
pgcd (c, N )
On remarquera même que cette dernière congruence est équivalente à ac ≡ bc (mod N ). On
laisse la démonstration de cette équivalence au lecteur.

3.2 Critères de divisibilité


Les critères de divisibilité que l’on apprend dans les petites classes trouvent leur justifi-
cation dans des manipulations simples de congruences. Supposons pour cela que l’on dispose
d’un entier n s’écrivant nd nd−1 · · · n0 en base 10, c’est-à-dire tel que l’on ait :

n = 10d nd + 10d−1 nd−1 + · · · 10n1 + n0

On voit directement sur cette écriture que l’on a toujours n ≡ n0 (mod 10). De même en
regardant modulo 2 et modulo 5, on obtient les critères de divisibilité bien connus suivants :
Critères de divisibilité
☞ Un entier est divisible par 10 si, et seulement s’il se termine par un 0.
☞ Un entier est divisible par 5 si, et seulement s’il se termine par un 0 ou par un 5.
☞ Un entier est divisible par 2 si, et seulement s’il se termine par un 0, un 2, un 4, un 6
ou un 8.

Bien évidemment, ces critères admettent un analogue (dont le démonstration est rigou-
reusement identique) pour une base b quelconque :

38
Théorème 3.2.1 Soit b > 2 un entier et d un diviseur de b. Alors un entier est divisible
par d si, et seulement si le dernier chiffre de son écriture en base b est lui-même divisible
par d.

De même il est possible de retrouver les critères de divisibilité classiques par 3 et 9. En


effet, si N désigne l’un des deux entiers 3 ou 9, on a 10 ≡ 1 (mod N ) et donc la congruence :

n ≡ nd + nd−1 + · · · n1 + n0 (mod N )

qui prouve :
Critères de divisibilité
☞ Un entier est divisible par 9 si, et seulement si la somme de ses chiffres l’est.
☞ Un entier est divisible par 3 si, et seulement si la somme de ses chiffres l’est.

De même, ces critères se généralisent à une base quelconque :

Théorème 3.2.2 Soit b > 2 un entier et d un diviseur de b − 1. Alors un entier est divisible
par d si, et seulement si la somme des chiffres de son écriture en base b est elle-même divisible
par d.

Il est possible d’inventer d’autres critères à perte de vue. Par exemple en remarquant
que 100 ≡ 0 (mod 4) ou que 10 ≡ −1 (mod 11), on obtient les deux critères suivants :
Critères de divisibilité
☞ Un entier est divisible par 4 si, et seulement si le nombre formé par ses deux derniers
chiffres (en base 10) l’est.
☞ Un entier est divisible par 11 si, et seulement si la somme des ses chiffres (en base 10)
de rang pair diminuée de la somme de ses chiffres de rang impair est divisible par 11.

Le lecteur amusé pourra inventer sur le même principe multitude de nouveaux critères de
divisibilité. Ceux-ci, cependant, sont en général peu utiles en pratique.

3.3 Ordre d’un élément


Fixons un entier N > 1 et a un entier premier avec N . Comme il n’y a que N résidus
modulo n, il existe des entiers s et t avec s < t tels que as ≡ at (mod N ). Comme a est
premier avec N , il admet un inverse a0 modulo N . En multipliant la congruence précédente
par a0 s , on obtient at−s ≡ 1 (mod N ). On peut donc poser la définition suivante :

Définition 3.3.1 Soit a un entier premier avec N . On appelle ordre de a modulo N , le plus
petit entier k > 0 tel que ak ≡ 1 (mod N ).
Remarque. Si a n’est pas premier avec N , il n’admet pas d’ordre modulo N . Autrement dit,
il n’existe aucun entier k > 0 tel que ak ≡ 1 (mod N ). En effet, cette dernière congruence
impliquerait que ak−1 est un inverse de a modulo N , ce qui n’existe pas.

39
La notion d’ordre est souvent des plus utiles. Voyons tout de suite une première façon
de s’en servir :

Exercice : Si n est un entier premier avec 10, il possède un multiple qui ne s’écrit qu’avec
des chiffres 1.
k
Solution : On remarque que 1 . . . 1 (k fois) vaut 10 9−1 . Comme 10 est premier avec n, il l’est
aussi avec 9n. Notons k l’ordre de 10 modulo 9n. Alors 9n divise 10k − 1, et finalement, √
k
l’entier 10 9−1 est un multiple de n dont l’écriture en base 10 ne comporte que des 1.

Il est facile mais intéressant de remarquer que si a est premier avec N et si k désigne
l’ordre de a modulo N , alors an+k ≡ an (mod N ) pour tout n. On dit que la suite des an est
périodique modulo N . Par définition, l’ordre correspond exactement à la période de cette
suite. Autrement dit, on a la proposition suivante dont il ne faut pas négliger l’utilité :

Proposition 3.3.2 Avec les notations précédentes, si n vérifie an ≡ 1 (mod N ), alors il


est divisible par k (l’ordre de a modulo N ).

3.4 Théorème chinois


Le théorème chinois s’énonce comme suit :

Théorème 3.4.1 Soient N1 , N2 , . . . , Nk des entiers strictement positifs deux à deux pre-
miers entre eux, et a1 , a2 , . . . , ak des entiers quelconques. Alors il existe un entier a tel que
le système de congruences : 

 x ≡ a1 (mod N1 )

 x ≡ a2 (mod N2 )
..

 .

 x ≡ a (mod N )
k k

soit équivalent à la simple congruence x ≡ a (mod N1 N2 · · · Nk ).


En particulier, le système précédent possède au moins une solution.
Démonstration. On remarque dans un premier temps qu’il suffit de prouver le théorème
lorsque k = 2. Une récurrence directe permettra ensuite de l’avoir dans toute sa généralité.
On cherche à résoudre l’équation x ≡ a1 (mod N1 ) et x ≡ a2 (mod N2 ). La première
condition assure l’existence d’un entier q tel que x = a1 + qN1 et la seconde congruence
s’écrit alors :
a1 + N1 q ≡ a2 (mod N2 )
ce qui fournit :
q ≡ (a2 − a1 ) N10 (mod N2 )
où N10 désigne un inverse de N1 modulo N2 qui existe bien car N1 et N2 sont supposés premiers
entre eux. Ainsi si l’on pose a = a1 + (a2 − a1 ) N10 N1 , on obtient x ≡ a (mod N1 N2 ).
La réciproque est immédiate. ¤

Remarque. La démonstration précédente fournit en réalité (via l’algorithme d’Euclide étendu)


un moyen effectif de calculer l’entier a du théorème.

40
Le théorème chinois est utile principalement dans deux situations. La première se présente
lorsque l’on demande de construire un entier vérifiant un certain nombre de conditions
arithmétiques. Si l’on arrive ainsi à traduire ces conditions en terme de congruence (ou plus
modestement à trouver des conditions plus faibles qui s’expriment en terme de congruence),
on pourra appliquer le théorème précédent pour résoudre la question. Un exemple classique
est donné par l’exercice suivant :

Exercice : Prouver qu’il existe n nombres consécutifs qui ne sont pas des puissances parfaites
(i.e. ne s’écrivent pas sous la forme ak pour des entiers a et k avec k > 1).
Solution : On remarque que si p est un nombre premier et si x ≡ p (mod p2 ), alors x ne
peut pas être une puissance parfaite puisque vp (x) = 1 forcément.
Désignons par p1 , . . . , pn des nombres premiers deux à deux distincts et intéressons-nous
au système suivant : 

 x ≡ p1 − 1 (mod p21 )

 x ≡ p2 − 2 (mod p2 )
2
..

 .

 x ≡ p − n (mod p2 )
n n

Puisque les p2isont premiers entre eux, d’après le théorème chinois il existe une solution à
ce système. Et si x est solution, alors x + i ≡ pi (mod p2i ) et donc n’est pas une puissance√
parfaite. Cela conclut.

Remarque. La difficulté dans cet exercice consiste à transformer la condition de l’énoncé en


condition de congruence. Souvent pour cela, il est nécessaire de faire preuve d’originalité...
mais certains réflexes doivent survenir, comme celui de considérer la suite des nombres
premiers.

La seconde situation se présente lorsque l’on est amené à considérer des congruences mo-
dulo un entier N . Nous allons voir dans les paragraphes suivants que l’étude des congruences
modulo N est plus facile lorsque N est un nombre premier ou une puissance d’un nombre
premier (en effet, on dispose de plus de théorèmes dans ces cas). Ainsi, il peut être intéressant
de décomposer N en facteurs premiers :
N = pα1 1 pα2 2 · · · pαk k
et à considérer la congruence modulo chacun des pαi i . Le théorème chinois permet alors de
tout remettre ensemble.

Notons finalement qu’il existe une généralisation (moins connue) du lemme chinois dans
le cas où les modulos ne sont pas premiers entre eux :

Proposition 3.4.2 Soient N1 , N2 , . . . , Nk des entiers strictement positifs, et a1 , a2 , . . . , ak


des entiers quelconques. Alors le système suivant :


 x ≡ a1 (mod N1 )

 x ≡ a2 (mod N2 )
(S) : ..

 .

 x ≡ a (mod N )
k k

41
admet une solution si, et seulement si pour tous i et j, on a ai ≡ aj (mod pgcd (Ni , Nj )).
Dans le cas où (S) admet une solution, il existe un entier a tel que (S) soit équivalent à la
seule congruence :
x ≡ a (mod ppcm (N1 , N2 , . . . , Nk ))
Démonstration. Supposons dans un premier temps que le système admette une solution
x. Soient i et j deux indices. On a x ≡ ai (mod Ni ) et donc a fortiori, on a x ≡ ai
(mod pgcd (Ni , Nj )). De même, on obtient x ≡ aj (mod pgcd (Ni , Nj )), d’où on déduit
a1 ≡ a2 (mod pgcd (Ni , Nj )).

Faisons la réciproque. On raisonne pour cela par récurrence. Commençons donc par
traiter le cas k = 2. Posons pour cela x0 = x − a1 . Le système est alors équivalent à :
½ 0
x ≡ 0 (mod N1 )
x0 ≡ a2 − a1 (mod N2 )

avec a1 ≡ a2 (mod d) où on a posé d = pgcd (N1 , N2 ). Une solution x0 est alors forcément
0
un multiple de N1 et donc un multiple de d. En posant x00 = xd , le système devient équivalent
à : ½ 00
x ≡ 0 (mod Nd1 )
x00 ≡ a2 −a
d
1
(mod Nd2 )
Les entiers Nd1 et Nd2 sont premiers entre eux. On peut donc appliquer le théorème chinois,
et en utilisant la formule :

pgcd (N1 , N2 ) ppcm (N1 , N2 ) = N1 N2

on obtient la conclusion voulue.


Il reste à traiter l’hérédité. Supposons donc donné le système suivant :


 x ≡ a1 (mod N1 )

 x ≡ a2 (mod N2 )
(S) : ..

 .

 x ≡ a (mod N )
k k

où l’on a les congruences ai ≡ aj (mod pgcd (Ni , Nj )) pour tous i et j. Les deux premières
lignes de (S) sont, d’après le cas traité précédemment, équivalentes à la seule congruence :

x ≡ a1,2 (mod ppcm (N1 , N2 ))

pour un certain entier a1,2 forcément congru à a1 modulo N1 et congru à a2 modulo N2 . Le


système (S) devient donc équivalent à :


 x ≡ a1,2 (mod ppcm (N1 , N2 ))

 x ≡ a3 (mod N3 )
(S 0 ) : ..

 .

 x ≡ a (mod N )
k k

42
Soit i un indice compris entre 3 et k. On a d’une part les congruences :

ai ≡ a1 ≡ a1,2 (mod pgcd (Ni , N1 ))


ai ≡ a2 ≡ a1,2 (mod pgcd (Ni , N2 ))

d’où on déduit :

ai ≡ a1,2 (mod ppcm (pgcd (Ni , N1 ) , pgcd (Ni , N2 )))

Mais d’autre part, on a l’égalité :

pgcd (Ni , ppcm (N1 , N2 )) = ppcm (pgcd (Ni , N1 ) , pgcd (Ni , N2 ))

et donc finalement le système (S 0 ) vérifie l’hypothèse de récurrence. On conclut ainsi. ¤

3.5 Congruences modulo p


On suppose dans ce paragraphe que N = p est un nombre premier et on étudie plus
spécifiquement les congruences modulo p. La propriété fondamentale est la suivante : lorsque
p est premier, tout nombre qui n’est pas divisible par p est premier avec p. Ainsi tous les
résidus non nuls sont inversibles modulo p.
Cela implique par exemple la propriété agréable suivante :

Proposition 3.5.1 Si a et b sont des entiers tels que ab ≡ 0 (mod p), alors soit a ≡ 0
(mod p), soit b ≡ 0 (mod p)
Démonstration. Supposons que a ne soit pas un multiple de p. Alors a est premier avec
p et donc il admet un inverse a0 . En multipliant la congruence ab ≡ 0 (mod p) par a0 , on
obtient bien b ≡ 0 (mod p). ¤

Remarque. Cette propriété n’est pas nouvelle : si l’on regarde bien, c’est exactement celle
énoncée dans le lemme 1.2.3. Toutefois, la formulation précédente permet de la rapprocher
d’une propriété analogues des nombres réels à savoir si un produit est nul, alors l’un des
facteurs est nul. On sait que cette propriété est souvent utilisée pour résoudre des équations
polynômiales de degré 2 ou supérieur... on pourra donc utiliser des méthodes analogues dans
ce contexte modulo p.

Un autre fait important qui peut-être vu comme une conséquence de ce qui précède est
que l’on dispose d’une estimation de l’ordre d’un élément modulo p :

Théorème 3.5.2 (Petit théorème de Fermat) Si p est un nombre premier et a un en-


tier non divisible p, on a ap−1 ≡ 1 (mod p). Ainsi l’ordre de a est un diviseur de p − 1.
Démonstration. Considérons l’ensemble des résidus non nuls modulo p. La multiplication
par a définit une application de cet ensemble dans lui-même. C’est une bijection puisque
l’ensemble des résidus est fini et puisque si ax ≡ ay (mod p), alors x ≡ y (mod p). Ainsi :

(1a)(2a)(3a) · · · ((p − 1)a) ≡ (1)(2)(3) · · · (p − 1) (mod p)

43
c’est-à-dire (p − 1)!ap−1 ≡ (p − 1)! (mod p). Le facteur (p − 1)! est premier avec p, donc
inversible modulo p et le théorème est prouvé. ¤

Remarque. On énonce parfois le théorème de Fermat sous la forme directement équivalente


suivante : pour tout entier a, on a ap ≡ a (mod p). On peut se demander si les nombres
premiers sont les seuls à vérifier de telles congruences pour tout entier a. La réponse est
négative comme le montre l’exemple de 561 = 3 × 11 × 17. Ces exemples sont rares et
s’appellent les nombres de Carmichael.

Le résultat précédent se généralise lorsque le modulo n’est pas premier : si n un entier


strictement positif et si a est un entier premier avec n, alors aϕ(n) ≡ 1 (mod n) où ϕ (n)
désigne le nombre d’entiers compris entre 1 et n et premiers avec n. La fonction ϕ est appelée
fonction indicatrice d’Euler. Toutefois cette formulation est souvent moins agréable car la
restriction sur les entiers a est plus difficile à exploiter et l’exposant ϕ (n) plus difficile à
calculer.

Comme application, citons deux exercices assez proches :

Exercice : Déterminer les entiers naturels n tels que n divise 2n − 1.


Solution : Il est clair que n = 1 convient. Montrons qu’il n’existe pas d’autre solution en
considérant un entier n > 1 tel que n divise 2n − 1. La condition de l’énoncé s’écrit 2n ≡ 1
(mod n). La congruence modulo n n’est pas très aisée à manipuler, c’est pourquoi on se
restreint modulo un diviseur premier p de n. On a alors 2n ≡ 1 (mod p). Par ailleurs, le
petit théorème de Fermat entraı̂ne que l’on a 2p−1 ≡ 1 (mod p). Introduisons δ l’ordre de 2
modulo p : d’après ce qui précède, δ divise n et p − 1. Mais jusqu’à présent, nous n’avons
imposé aucune condition sur p : si on le choisit comme étant le plus petit diviseur premier
de n, δ est strictement inférieur à p et divise n, donc δ = 1 d’où p divise 1, ce qui est absurde

et permet de conclure.

Exercice : Montrer que si n > 1 divise 2n + 1, alors n est divisible par 3.


Solution : Là encore, considérons n > 1 tel que n divise 2n + 1, puis p un facteur premier de
n et δ l’ordre de 2 modulo p. La condition imposée sur n entraı̂ne 2n ≡ −1 (mod p), d’où
22n ≡ 1 (mod p) et δ divise 2n et p − 1. Si on a pris p comme étant le plus petit facteur
premier de n, on a donc δ = 1 ou δ = 2. Le premier cas étant exclu, 22 ≡ 1 (mod p) donc √
p = 3.

Remarque. Les deux exemples précédents montrent bien que quitte à considérer un diviseur
premier de n, on peut ajouter une condition de minimalité pour le même prix.

Il n’est pas possible de conclure ce chapitre sans citer le théorème de Wilson, rarement
utile à vrai dire, mais devant faire partie du bagage culturel :

Théorème 3.5.3 (Wilson) Soit N > 1 un entier. La congruence (N − 1)! ≡ −1 (mod N )


a lieu si et seulement si N est premier.

44
Démonstration. Supposons N composé. On distingue deux cas.
Si N = p2 , on a : · 2 ¸
2 p −1
vp ((p − 1)!) > =p−1
p
Si p > 2, le nombre (p2 − 1)! est multiple de p2 est donc non congru à −1 modulo p2 . On
vérifie que c’est également le cas pour p = 2.
Sinon N n’est pas le carré d’un nombre premier, on peut écrire N = ab pour deux
nombres a et b inférieurs ou égaux à N − 1 et distincts. On voit alors que N divise (N − 1)!
et on conclut comme précédemment.
Si maintenant N = p est premier, on constate que l’on peut regrouper les résidus non
nuls modulo p deux à deux en associant chacun avec son inverse. Seuls 1 et −1 vont rester
seuls. Le produit de tous les résidus sera donc égal à 1 multiplié par −1 multiplié par un
certain nombre de fois 1, ce qui fait bien −1. ¤

3.6 Congruences modulo pn


La notation p désigne encore un nombre premier, et n désigne un entier quelconque
supérieur ou égal à 1. Le résultat principal de ce chapitre est le lemme de Hensel qui permet
de remonter modulo pn les solutions modulo p de certaines équations. Plus précisément, nous
avons :

Théorème 3.6.1 (Lemme de Hensel) Soient des entiers a0 , a1 , . . . , ak des entiers. On


définit le polynôme P par la formule :

P (X) = a0 + a1 X + · · · + ak X k

et le polynôme dérivé de P par la formule :

P 0 (X) = a1 + 2a2 X + · · · + kak X k−1

Soit x1 un entier tel que P (x1 ) ≡ 0 (mod p) et P 0 (x1 ) 6≡ 0 (mod p). Alors il existe un
entier x tel que P (x) ≡ 0 (mod pn ) et x ≡ x1 (mod p).
De plus si x et x0 vérifient les deux conditions précédentes, on a x ≡ x0 (mod pn ).
Remarque. On dit souvent que la solution x1 modulo p se relève en une solution x modulo
pn . La deuxième partie du théorème dit en substance que ce relèvement est unique.

Démonstration. On aura besoin pour cette démonstration de la congruence suivante :

(x + y)j ≡ xj + jxj−1 y (mod y 2 )

qui est une conséquence immédiate de la formule du binôme de Newton rappelée dans le
paragraphe 3.7.
Prouvons l’existence, c’est-à-dire la première partie du théorème. On procède par récur-
rence en construisant une suite d’entiers xi tels que xi ≡ x1 (mod p) et P (xi ) ≡ 0 (mod pi ).
Pour conclure il suffira de prendre x = xn . L’entier x1 est donné par hypothèse et permet
d’initialiser la récurrence.

45
Supposons l’entier xi construit et voyons comment l’on obtient xi+1 . On le cherche sous
la forme xi+1 = xi + pi r pour un certain entier r. Calculons :
¡ ¢ ¡ ¢2 ¡ ¢k
P (xi+1 ) = a0 + a1 xi + pi r + a2 xi + pi r + · · · + ak xi + pi r
≡ a0 + a1 xi + a1 pi r + a2 x2i + 2a2 xi pi r + · · · + ak xki + kak xk−1
i pi r
≡ P (xi ) + pi rP 0 (xi ) (mod pi+1 )

d’après la congruence rappelée au début de la preuve. D’après l’hypothèse de récurrence, il


existe un entier q tel que P (xi ) = qpi . Ainsi si l’on choisit r tel que rP 0 (xi ) ≡ −q (mod p),
on aura fini. Mais cela est possible car comme xi ≡ x1 (mod p), on a P 0 (xi ) ≡ P 0 (x1 )
(mod p) qui est un résidu non nul et donc inversible. Ceci conclut l’existence.
L’unicité est laissée au lecteur : on peut pour la démontrer l’inclure dans l’hypothèse de
récurrence et utiliser le fait que le r trouvé précédemment est uniquement déterminé modulo
p. ¤

Remarque. Il existe des raffinements du lemme de Hensel qui peuvent être très utiles dans
certains cas, mais qui requièrent un énoncé beaucoup plus lourd et difficilement mémorisable.
En pratique, si le lemme de Hensel ne s’applique pas tel quel, il est souvent plus efficace de
refaire la démonstration dans le cas particulier qui nous intéresse.

Les puissances modulo pn


Malgré son apparence un peu complexe et technique, le lemme de Hensel admet de
nombreuses applications. Par exemple, il peut être utile pour déterminer quels sont les
résidus quadratiques (c’est-à-dire les carrés de résidus) modulo pn . On a de fait la proposition
suivante :

Proposition 3.6.2 Soient p est un nombre premier impair et x un entier premier à p.


Alors x est un résidu quadratique modulo pn si, et seulement s’il en est un modulo p.
Démonstration. Le sens direct est immédiat : si x ≡ y 2 (mod pn ), alors on a également
x ≡ y 2 (mod p).
Pour la réciproque on utilise le lemme de Hensel. On pose P (X) = X 2 − x, et on a
P 0 (X) = 2X. Par hypothèse il existe y tel que x ≡ y 2 (mod p), i.e. P (y) ≡ 0 (mod p). De
plus on a forcément y 6≡ 0 (mod p), et donc P 0 (y) = 2y 6≡ 0 (mod p), puisqu’on a supposé
p impair. Le lemme de Hensel fournit la conclusion attendue. ¤

Remarque. Évidemment cette proposition ne résout pas complètement le problème des ré-
sidus quadratiques puisqu’il reste à voir ce qui se passe modulo un nombre premier p. Ces
résultats sont bien connus et détaillés dans le second tome de ce cours.

La proposition précédente se généralise directement à une situation plus vaste. On ob-


tient :

Proposition 3.6.3 Soient k un entier et p un nombre premier ne divisant pas k. Soit x un


entier premier à p. Alors, pour tout entier n, le nombre x est une puissance k-ième modulo
pn si, et seulement s’il en est une modulo p.

46
On peut alors se demander ce qu’il se passe lorsque k n’est plus premier à p. Dans ce
cas, la situation est un peu plus complexe comme le montre le lemme suivant :

Lemme 3.6.4 Si a et b sont des entiers tels a ≡ b (mod p) alors ap ≡ bp (mod p2 ). Plus
j j
généralement, si pour un entier i, on a a ≡ b (mod pi ), alors ap ≡ bp (mod pi+j ) pour
tout entier j.
Démonstration. Pour le cas j = 1, on écrit b = a + pi q pour un certain entier q et on
applique la congruence rappelée dans la démonstration du lemme de Hensel qui permet de
conclure directement. Le cas général s’en déduit par récurrence immédiate. ¤

Si la proposition 3.6.3 s’étendait pour les k non premiers à p, tous les nombres premiers
à p devraient être des puissances p-ième modulo p2 , puisqu’ils le sont tous modulo p (on
rappelle que par le petit théorème de Fermat, on a ap ≡ a (mod p) pour tout entier a). Or
le lemme précédent implique en particulier qu’il n’y a que p puissances p-ième modulo p2 .
Cependant, on dispose quand même d’un énoncé analogue à la proposition 3.6.3 pour le
cas k = p :

Proposition 3.6.5 Soit x un entier premier à p. Soit n > 3 un entier. Alors x est une
puissance p-ième modulo pn si, et seulement s’il en est une modulo p3 .
Démonstration. Le sens direct est évident. Pour la réciproque, on ne peut pas utiliser
le lemme de Hensel directement, mais on peut adapter la démonstration. Voyons sur cet
exemple comment cela peut fonctionner.
On suppose qu’il existe x3 tel que xp3 ≡ x (mod p3 ). On construit à nouveau par récur-
rence une suite (xn ) d’entiers tels que xn ≡ x3 (mod p) et xpn ≡ x (mod pn ). Cependant,
ici, on cherche xn+1 sous la forme xn+1 = xn + pn−1 r. On obtient la relation :
¡ ¢p
xpn+1 = xn + pn−1 r ≡ xpn + pn r (mod pn+1 )

puisque on a n > 3 (et donc 2 (n − 1) > n+1). Par hypothèse de récurrence on a xpn = a+pn q
pour un certain entier q. Il suffit pour conclure de choisir r = −x0n q où x0n désigne un inverse
de xn modulo p (les entiers x0n et p sont bien premiers entre eux, car on a supposé x premier
avec p). ¤

3.7 Coefficients binomiaux


Définition 3.7.1 Soient n et k deux entiers. On pose :
½ n!
k k!(n−k)!
si 0 6 k 6 n
Cn =
0 sinon

Propriétés immédiates
☞ Pour tout entier n, on a C0n = Cnn = 1
☞ Pour tous entiers n et k, on a Ckn = Cn−k
n
☞ Pour tous entiers n et k, on a kCkn = nCk−1
n−1

47
Les nombres Ckn sont appelés coefficients binomiaux. Ils ont une interprétation combi-
natoire (Ckn compte le nombre de parties à k éléments d’un ensemble de cardinal n) et ils
apparaissent dans la formule du binôme de Newton que nous rappelons :
n
X
n
(a + b) = Ckn ak bn−k
k=0

Les remarques précédentes assurent que Ckn est toujours un entier, ce que nous allons dé-
montrer avec des arguments arithmétiques.

Proposition 3.7.2 Soient n et k des entiers. Alors Ckn est un entier.


Démonstration. Soit p un nombre premier. On utilise la formule de Legendre pour évaluer
la valuation p-adique de Ckn . On a :
∞ µ· ¸
X · ¸ · ¸¶
n k n−k
vp (Ckn ) = vp (n!) − vp (k!) − vp ((n − k)!) = − i −
i=1
pi p pi

On a vu que pour tous réels x et y, [x + y] > [x] + [y], et donc chaque terme de la somme
précédente est positif ou nul. Il en est donc de même de vp (Ckn ). Ceci étant valable pour tout
nombre premier, on en déduit que Ckn est entier. ¤

On peut parfois affiner la proposition précédente comme le montre l’exercice suivant (qui
est un résultat important à retenir) :

Exercice : Soient n = pr une puissance d’un nombre premier p et k tel que 1 6 k 6 pr − 1.


Montrer que Ckpr est un multiple de p.
Solution : On reprend :
∞ µ· r ¸
X · ¸ · r ¸¶
p k p −k
vp (Ckpr ) = − i −
i=1
pi p pi

Comme précedemment, chaque terme de la somme est positif ou nul. Mais cette fois-ci,
lorsque i = r, on a : · r¸ · ¸ · r ¸
p k p −k
− r − =1−0−0=1
pr p pr
ce qui assure que la somme totale est supérieure ou égale à 1 et donc la conclusion. √

Remarque. On aurait pu également utiliser kCkpr = pr Ck−1


pr −1 .

Triangle de Pascal
La formule suivante valable pour tous entiers n et k (qu’on laisse au lecteur le soin de
vérifier) :
Ckn = Ckn−1 + Ck−1
n−1

48
permet de calculer les coefficients binomiaux de proche en proche.

Pour des raisons pratiques, on rassemble souvent les Ckn non nuls dans le triangle de
Pascal :
k
n 0 1 2 3 4 5

0 1
1 1 1
2 1 2 1
3 1 3 3 1
4 1 4 6 4 1
5 1 5 10 10 5 1

La formule Ckn = Ckn−1 + Ck−1


n−1 implique qu’un nombre du tableau précédent est obtenu
en additionnant les deux nombres placés au-dessus (celui juste au-dessus et son voisin de
gauche).
La construction précédente s’avère très intéressante lorsque l’on souhaite déterminer les
coefficients binomiaux modulo un nombre entier N . En effet, on remplit le tableau exac-
tement de la même manière mais on ne tient compte que des restes modulo N lors des
additions.

Lorsque N = p est un nombre premier, on dispose en outre d’un théorème pour déter-
miner simplement Ckn modulo p :

Théorème 3.7.3 (Lucas) Soit p un nombre premier. Soient n et k des entiers avec 0 6
k 6 n. Écrivons les décompositions en base p de n et de k :
n = nd pd + nd−1 pd−1 + · · · + n1 p + n0
k = kd pd + kd−1 pd−1 + · · · + k1 p + k0
où kd peut valoir 0. Alors :
Ckn ≡ Ckndd Cknd−1
d−1
· · · Ckn00 (mod p)
Démonstration. Avec des dessins...
Notons M le tableau carré p × p formé par les p premières lignes (donc pour n variant
entre 0 et p − 1) du triangle de Pascal modulo p. D’après l’exercice précédent, la première
ligne qui n’est pas dans M ne contient que des 0 (modulo p) sauf pour les termes des colonnes
0 et p qui sont égaux à 1. Le procédé de construction du triangle de Pascal implique que les
2p premières lignes sont données par :
1

M
1 1
1 1

M M
1 1 1 1

49
Mais alors, en continuant la construction, on voit que la ligne suivante ne contient que des
0 sauf à l’extrémité gauche (où il y a un 1), à la colonne p (où il y a un 2) et à la colonne
2p où il y a un 1. On obtient alors le dessin suivant :
1

M
1 1
1 1

M M
1 1 1 1
1 2 1

M 2M M
1 1 2 2 1 1

k
Et ainsi de suite... et donc le terme
¡ C n se retrouve être le nombre écrit à la position ¢ (n0 , k0 )
dans le bloc situé à la position nd pd−1 + · · · + n2 p + n1 , kd pd−1 + · · · + k2 p + k1 , soit :
d−1
Ckn = Cknddppd−1+···+k 2 p+k1
Ck0
+···+n2 p+n1 n0

La conclusion découle alors d’une récurrence immédiate.


Avec des calculs...
On utilise ici la formule du binôme et l’égalité :
dn
(1 + X)n = (1 + X)n0 (1 + X)pn1 · · · (1 + X)p d

r
Le fait que p divise Ckpr pour 1 6 k 6 pr − 1 prouve que (1 + X)p ≡ 1 + X p (mod p) pour
r

tout entier r. Ainsi :


³ ´
d nd
(1 + X)n = (1 + X)n0 (1 + X p )n1 · · · 1 + X p

On compare les termes en X k . À gauche, le coefficient vaut Ckn et à droite, comme k a une
unique décomposition en base p qui est k = kd pd + · · · + k1 p + k0 , ce coefficient vaut :
Ckndd Cknd−1
d−1
· · · Ckn00
et la congruence est prouvée. ¤
Exercice : Soient p un nombre premier et r un entier strictement positif. Montrer que :
Cppr ≡ pr−1 (mod pr )

Solution : On utilise la formule bien connue pCppr = pr Cpp−1


r −1 qui donne :

Cppr = pr−1 Cp−1


pr −1

En base p, k = p−1 a un seul chiffre, donc en reprenant les notations précédentes, k0 = p−1
et ki = 0 pour tout i > 0. D’autre part, n = pr − 1 s’écrit avec r chiffres p − 1, i.e.
n0 = . . . = nr−1 = p − 1 et ni = 0 pour i > r. Finalement, d’après le théorème de Lucas :
Cp−1 p−1 0
pr −1 ≡ Cp−1 (Cp−1 )
r−1
≡ 1 (mod p)

Cela termine l’exercice.

50
3.8 Exercices
Exercice 111 Prouver que le produit de k entiers consécutifs est divisible par k!.

Exercice 112 Soit n un entier positif ou nul tel que pour tout 0 6 k 6 n, Ckn est impair.
Montrer que n = 2m − 1 pour un entier m.

Exercice 113 Soient n > 2 progressions arithmétiques d’entiers, infinies dans les deux sens
(c.à.d. indexées sur Z), telles que deux quelconques d’entre elles aient toujours au moins un
terme commun. Prouver qu’il existe un entier qui appartient à chacune des progressions
arithmétiques.

Exercice 114 On note ϕ (n) le nombre d’entiers compris entre 1 et n et premiers avec n.
Prouver que : X
ϕ (d) = n
d|n

Exercice 115 La légende raconte que les chinois procédaient de la façon suivante pour
compter leur armées. Le « général » demandait aux soldats de se mettre en rang deux par
deux, et notait s’il restait un soldat isolé ou non. Il leur demandait ensuite de se mettre en
rang trois par trois et notait encore le nombre de soldats isolés qu’il restait. On continuait
ainsi, en se mettant en rang ensuite cinq par cinq, puis sept par sept, puis onze par onze,
puis treize par treize et dix-sept par dix-sept.
Montrer que pour des armées de moins de cinq cent mille hommes, cette méthode permet
effectivement de compter les soldats.

Exercice 116 (URSS 64) Soient a, b et n des entiers strictement positifs tels que, pour
tout entier k > 0 avec k 6= b, le nombre a − k n soit divisible par b − k. Prouver que a = bn .

Exercice 117 Montrer que pour tout entier n, 9n − 2n est divisible par 7.

Exercice 118 Déterminer tous les nombres premiers p tels que 4p + 1 et 7p − 4 soient
également premiers.

Exercice 119* (TDV 04) Existe-t-il une permutation {a1 , . . . , a2004 } de {1, . . . , 2004} de
sorte que ai + · · · + ai+9 soit un multiple de 10 pour tout i compris entre 1 et 1995 ?

Exercice 120* (AMM, France 03) On se place dans le plan rapporté à un repère ortho-
normal (O, i, j). On dit qu’un point à coordonnées entières A est invisible si le segment [OA]
contient un point à coordonnées entières distinct de O et de A. Soit L un entier naturel.
Montrer qu’il existe un carré dont les côtés sont parallèles aux axes et ont une longueur égale
à L et tel que tous les points à coordonnées entières intérieurs au carré soient invisibles.

Exercice 121* Calculer la somme des diviseurs de 104060401.

Exercice 122* Montrer que pour tout nombre premier p > 3, le numérateur de la fraction :
1 1
1+ + ... +
2 p−1

51
est divisible par p.

Exercice 123* Existe-t-il deux puissances de 2 distinctes ayant exactement les mêmes
chiffres (comptés avec multiplicité) en base 10. (Les éventuels zéros au début d’un nombre
ne sont pas comptés comme des chiffres).

Exercice 124* (Putnam 50) Soit n un entier. Prouver que le nombre de k tel que Ckn est
impair est une puissance de 2.

Exercice 125* Prouver qu’il n’existe pas de suite infinie (xn ) de nombres premiers telle
que, pour tout n, on ait |xn+1 − 2xn | = 1.

Exercice 126* (Russie 95) Existe-t-il une permutation {a1 , a2 , a3 , . . .} de N? telle que,
pour tout n, le nombre a1 + a2 + · · · + an soit divisible par n ?

Exercice 127* Soient 111 entiers relatifs de somme nulle. Montrer que la somme de leur
puissances 37-ièmes est divisible par 399.

Exercice 128* (OIM 79) Soient a et b des entiers strictement positifs vérifiant :
a 1 1 1 1 1
= 1 − + − + ... − +
b 2 3 4 1318 1319
Montrer que 1979 divise a.

Exercice 129* (Crux Mathematicorum) Trouver tous les nombres premiers p pour
lesquels il existe une base b > 2 dans laquelle l’écriture de p utilise une et une seule fois tous
les chiffres. (Le chiffre 0 peut se positionner au début de l’écriture).

Exercice 130* (OIM 86) Soit d un entier strictement positif n’appartenant pas à l’en-
semble {2, 5, 13}. Montrer que l’on peut trouver un couple (a, b) d’éléments distincts de
l’ensemble {2, 5, 13, d} tel que ab − 1 ne soit pas le carré d’un entier.

Exercice 131* (Estonie 00, France 04) Existe-t-il un entier n pour lequel on puisse
partitionner l’ensemble {n, . . . , n + 17} en deux parties A et B telles que le produit des
éléments de A vaut celui des éléments de B ?

Exercice 132* Pour x et y dans N, on pose :

B = x (y + 1) − (y! + 1)
y − 1 ¡¯¯ 2 ¯ ¡ ¢¢
f (x, y) = B − 1¯ − B 2 − 1 + 2
2
Montrer que, lorsque x et y décrivent N, la fonction f décrit exactement l’ensemble des
nombres premiers, et que chaque nombre premier impair est atteint une seule fois.

Exercice 133* Soient p un nombre premier différent de 2 et 5 et n un entier strictement


positif. Montrer que la somme :
1 1 1
+ + ··· +
n n+1 n+p−1

52
n’est jamais un nombre décimal (i.e. sa partie décimale est finie). Que se passe-t-il pour
p = 2 et p = 5 ?

Exercice 134* (Bosnie 97) a) Prouver que, pour tout entier n > 0, il existe un en-
semble Mn contenant exactement n entiers strictement positifs et tel que : les moyennes
arithmétiques et géométriques des éléments de toute partie non vide de Mn sont des entiers.
b) Existe-t-il un ensemble infini d’entiers naturels ayant la même propriété ?

Exercice 135* (Roumanie 94) On construit une suite d’entiers en posant u0 = 20002003
et pour tout n > 0, un+1 = un + 7 si un est impair, un+1 = u2n si un est pair.
Quel est le plus petit entier atteint par cette suite un ?

Exercice 136* (SL 84) Montrer que pour tout n > 0, Cn2n divise ppcm (1, . . . , 2n).

Exercice 137* Soit p un nombre premier. Prouver qu’il existe un diviseur premier de pp −1
qui est congru à 1 modulo p.

Exercice 138* Soient n1 , . . . , nk des entiers positifs. Montrer que le quotient :

(n1 + . . . + nk )!
n1 ! . . . n k !
est entier.

Exercice 139* (Nombres de Catalan) Montrer que pour tout n, Cn2n est divisible par
n + 1.

Exercice 140* Trouver tous les entiers strictement positifs n tels que 17 divise 3n − n.

Exercice 141* (OIM 72) Montrer que (2m)! (2n)! est un multiple de m!n! (m + n)! pour
tous entiers positifs ou nuls m et n.

Exercice 142* (SL 85) Soient k > 2 et n1 , . . . , nk des entiers strictement positifs tels que :

n2 divise 2n1 − 1
n3 divise 2n2 − 1
..
.
nk−2 divise 2nk−1 − 1
n1 divise 2nk − 1

Prouver que n1 = n2 = · · · = nk = 1.

Exercice 143* (Iran 94) Soit p > 5 un nombre premier. Montrer que 43 divise 7p − 6p − 1.

Exercice 144* Déterminer tous ls entiers a > 0 et b > 2 tels que 2a + 1 soit divisible par
2b − 1.

53
Exercice 145** (Balkan 89 (non utilisé)) Montrer que pour tout n > 3, le nombre :
nn n
nn − nn

est divisible par 1989.

Exercice 146** (France 04) On note P l’ensemble des nombres premiers. On considère
une partie M de P ayant au moins 3 éléments. On suppose que, pour tout sous-ensemble
fini non vide strict A de M , les facteurs premiers de l’entier :
¡Y ¢
p −1
p∈A

appartiennent à M . Montrer que M = P.

Exercice 147** (SL 91) Déterminer le pgcd des nombres n37 − n pour n décrivant Z.

Exercice 148** (SL 91) Prouver que le dernier chiffre non nul de n! forme une suite non
périodique (même à partir d’un certain rang).

Exercice 149** (Moscou 73) Douze peintres vivent dans douze maisons d’une même
rue circulaire, peintes certaines en bleu et les autres en blanc. Chaque fin de mois, l’un des
peintres quitte sa maison avec ses pots de peinture et repeint chaque maison de la couleur
contraire, en commençant par la sienne et dans l’ordre des aiguilles d’une montre. Il s’arrête
dès qu’il a repeint une maison blanche en bleu. Durant une année, un peintre donné ne fait
cela qu’une seule fois. Prouver que, si au début de l’année l’une au moins des maisons était
peinte en bleu alors, à la fin de l’année, chacune retrouvera sa couleur initiale.

Exercice 150** (Quadrature) Trouver toutes les puissances de 2 qui sont encore des
puissances de 2 lorsqu’on efface leur chiffre de gauche.

Exercice 151** (Russie 96) Prouver qu’il n’existe pas d’entiers a et b strictement positifs
tels que, pour tous nombres premiers distincts p et q strictement supérieurs à 1000, le nombre
ap + bq soit aussi premier.

Exercice 152** (Turquie 96) Soient a et n des entiers strictement positifs. Prouver que :
n−1
Y
(an − ak )
k=0

est divisible par n!.

Exercice 153** (OIM 75) Soient A la somme des chiffres (en base 10) de 44444444 , et B
la somme des chiffres de A. Calculer la somme des chiffres de B.

Exercice 154** (D. J. Newman) Soient a et b des entiers strictement positifs. Montrer
que le nombre : µ ¶n µ ¶n
1 1
a+ + b+
2 2

54
est un entier pour seulement un nombre fini de valeurs de n.

Exercice 155** Soit n > 1 un entier. Prouver qu’il existe un entier k > 0 tel que 2k + 1
divise n + k! ou n − k!. (On pourra utiliser, après l’avoir prouvé, qu’un diviseur premier
impair de n2 + 1 est toujours congru à 1 modulo 4).

Exercice 156** Soient n et m deux entiers. Montrer que :


(nm)!
m! (n!)m
est un entier.

Exercice 157*** (Erdös et al.) Soit n > 1 un entier. Prouver que parmi 2n − 1 entiers,
on peut toujours en trouver n dont la somme est divisible par n.

Exercice 158*** n enfants sont assis en cercle. Dolpha donne un bonbon au premier
enfant, saute le second, donne un bonbon au troisième, saute les deux suivants, donne un
bonbon au prochain enfant, puis saute les trois suivants, et ainsi de suite. Pour quelle valeur
de n, tous les enfants auront-ils au moins un bonbon au bout d’un certain nombre de tours ?

Exercice 159*** (Erdös) Prouver que le produit des nombres premiers inférieurs ou égaux
à n est inférieur ou égal à 4n .

Exercice 160*** (USA 82) Montrer qu’il existe un entier k tel que k2n + 1 est toujours
composé pour tout n.

Exercice 161*** (SL 98) Déterminer tous les entiers n > 0 pour lesquels il existe un
entier m tel que 2n − 1 divise m2 + 9.

Exercice 162*** (France 02) Soit p un nombre premier impair. Montrer qu’il existe p
entiers strictement positifs a1 , a2 , · · · , ap inférieurs ou égaux à 2p2 tels que les p(p−1)
2
sommes
de la forme ai + aj (où i < j) soient distinctes.

Exercice 163*** Trouver tous les entiers n > 1 tels que n divise 2n−1 + 1.

Exercice 164*** (SL 93) Soit b > 1 un entier. Soit a un entier strictement positif tel que
bn − 1 divise a. Montrer qu’en base b, l’entier a possède au moins n chiffres non nuls.

Exercice 165**** (SL 97) Montrer que toute progression arithmétique infinie d’entiers
positifs qui contient un carré et un cube contient une puissance sixième.

Exercice 166**** Pour tout entier n > 1, on écrit :


1 1 a (n)
1+ + ... + =
2 n b (n)
avec a (n) et b (n) premiers entre eux. Montrer qu’il existe une infinité d’entiers n pour
lesquels a (n) n’est pas une puissance d’un nombre premier.

55
4 Équations diophantiennes
On appelle équation diophantienne 5 toute équation dont on cherche les solutions en
nombres entiers. Par exemple x2 = 4k + 3, x2 + y 2 = z 2 , 1! + 2! + · · · + n! = x2 sont des
équations diophantiennes.
Notez qu’en général, les équations diophantiennes font intervenir plusieurs et souvent
un grand nombre d’inconnues. Notez également que les techniques utilisées pour aborder
les équations sont très souvent radicalement différentes des techniques classiques d’attaque
pour les équations algébriques.
Résoudre celles-ci est souvent très difficile et les mathématiques actuelles sont encore
loin de savoir proposer des méthodes dans tous les cas. Toutefois, certaines pistes sont très
bien balisées et ce sont elles que nous allons présenter par la suite.
Finalement, ne soyez pas effrayés : s’il est fort probable que si vous inventiez une équation
diophantienne un peu tordue, elle soit complètement inabordable même pour les plus grands
chercheurs, il est aussi évident que les exercices que l’on vous propose disposent de solutions
que vous avez les moyens de trouver.

4.1 Quelques réflexes


Les propriétés des entiers et les notions de divisibilité sont essentielles dans la résolution
des équations diophantiennes. Rappelons tout de suite quelques propriétés qu’il est bon
d’avoir constamment en tête :
Quelques idées à tester systématiquement
☞ Si le produit ab est une puissance d’un nombre premier p, alors a et b sont également
des puissances de ce nombre premier. Si le produit ab est une puissance d’un entier n,
il peut être intéressant de décomposer n en facteurs premiers.
☞ Si le produit ab est un carré et que a et b sont premiers entre eux, alors a et b sont
des carrés. Plus généralement si d = pgcd (a, b), a s’écrit dx2 et b s’écrit dy 2 pour des
entiers x et y. Rappelons à ce niveau que le pgcd de deux entiers dont la différence est
n est un diviseur de n. En particulier, cette propriété est forte utile pour les situations
faisant intervenir des produits a (a + n) ou plus souvent (a − n) (a + n) = a2 − n2 .
☞ Un entier strictement positif est supérieur ou égal à 1. De même, si n est entier et
n 6 x, alors n 6 [x]. Un bon reflexe à avoir à ce niveau est de ne jamais (ou du moins
le plus rarement possible) conserver des inégalités strictes entre nombres entiers : elles
peuvent toujours être améliorées.
☞ On dispose de la factorisation :
¡ ¢
an − bn = (a − b) an−1 + an−2 b + · · · + bn−1

et si n est impair, de la factorisation analogue :


¡ ¢
an + bn = (a + b) an−1 − an−2 b + · · · + bn−1
5
Du nom du mathématicien Diophante.

56
Ces factorisations s’avèrent très utiles lorsque l’on a besoin de modifier l’aspect d’une
équation diophantienne, le plus souvent en faisant des manipulations algébriques. Si-
gnalons également que toute expression de la forme αx + βy + γxy + δ peut en général
se factoriser sous la forme :
(ax + b) (cx + d) + e
pour certains rationnels (pas forcément entiers même si α, β, γ et δ le sont) a, b, c, d
et e.

Voyons sur des exemples simples comment utiliser ces idées. Cherchons dans un premier
temps à résoudre :
2n + 1 = x2
Pour cela, on fait passer le 1 de l’autre côté de l’égalité et on factorise :

2n = (x + 1)(x − 1)

et donc d’après une des propriétés rappelées précédemment, à la fois x + 1 et x − 1 doivent


être des puissances de 2. Or, des puissances de 2 qui diffèrent de 2, il n’y a que 2 et 4. Donc
x = 3 est la seule solution, et fournit n = 3.
Cet exemple illustre de façon parfaite le fait mentionné précédemment stipulant qu’il
peut parfois être intéressant de faire des manipulations algébriques simples sur l’équation
pour lui donner un aspect plus propice à sa résolution. Souvent savoir factoriser un membre
de l’égalité s’avère déterminant.

Lorsque seulement deux valeurs interviennent, un premier pas éclairant consiste souvent
à comparer les ordres de grandeur de ces valeurs. Pour exemple, considérons l’équation :

x2 = 2 + 6y 2 + y 4

Sans trop réfléchir, on voit que si cette équation admet une solution, x doit être de l’ordre
de y 2 et même plus précisément l’écriture suivante :

x2 = (y 2 + 3)2 − 7

nous dit que x ne doit pas être loin de y 2 + 3. Précisément en fait, on a x < y 2 + 3. On a
également :
x2 = (y 2 + 2)2 + 2y 2 − 2
et donc dès que 2y 2 − 2 > 0, on doit avoir x > y 2 + 2. Comme il n’y a pas d’entiers entre
y 2 + 2 et y 2 + 3 l’équation n’admet pas de solution. Les seules solutions éventuelles seraient
alors obtenues pour les y tels que 2y 2 − 1 > 0, c’est-à-dire y = −1, y = 0 et y = 1. On vérifie
ensuite au cas pas cas.
La morale est que lorsque l’équation a un petit nombre d’inconnues, des techniques d’in-
égalité, peuvent permettre de restreindre l’étude à un nombre fini de cas. Lorsque l’exercice
est bien fait, ce nombre est petit, et on peut donc traiter ces cas un par un.

57
Une autre illustration simple de ce dernier principe est le suivant : trouver tous les entiers
positifs n tel que 3n + 7 divise 5n + 13. Le quotient 5n+13
3n+7
est toujours compris strictement
entre 0 et 2 et donc, comme il est entier, il ne peut en fait valoir que 1. Il ne reste alors plus
qu’à résoudre l’équation 5n + 13 = 3n + 7 qui admet pour solution n = −3. Ce nombre n’est
pas positif, donc il n’existe aucun n répondant à notre question.

Remarquons pour finir que l’utilisation d’inégalités peut s’avérer efficace même si le
nombre d’inconnues est plus important. Pour exemple, nous donnons l’exercice suivant :

Exercice : Trouver tous les entiers strictement positifs x, y et z tels que :


1 1 1
+ + =1
x y z

Solution : Comme les inconnues jouent un rôle symétrique, on peut supposer 0 < x 6 y 6 z.
Dans ces conditions, on a :
1 1 1 3
1= + + 6
x y z x
et donc x 6 3. Il ne peut valoir 1, il vaut donc 2 ou 3.
On traite les deux cas séparemment en utilisant à nouveau la même méthode. Si x = 2,
l’équation devient :
1 1 1
+ =
y z 2
puis par le même argument x = 2 6 y 6 4. On teste alors les cas un par un trouve que les
seules solutions sont y = 3, z = 6 et y = 4, z = 4.
Pour x = 3, on obtient :
1 1 2
+ =
y z 3
puis x = 3 6 y 6 3. La seule solution est, dans ce cas, x = y = 3.
Finalement, les solutions sont les triplets (2, 3, 6), (2, 4, 4), (3, 3, 3) et toutes leurs per-

mutations.

Confrontés à une équation, on peut également se demander s’il n’y a pas une manipulation
simple qui permet de transformer une solution en une autre. Par exemple, il est possible qu’en
multipliant tous les entiers d’une solution par une même valeur d, on obtienne une nouvelle
solution. On dit souvent alors que l’équation est homogène. Ce cas se produit par exemple
lorsque l’équation proposée est une somme de mônomes, tous de même degré. Souvent la
solution obtenue ainsi est « plus grande ». Cependant cela peut-être encore plus intéressant
si elle se trouve « plus petite » (voir 4.3).
Ces manipulations permettent par exemple de faire des hypothèses supplémentaires sur
une solution recherchée. Par exemple, dans le cas « homogène » présenté précédemment, on
peut supposer que les inconnues sont premières entre elles dans leur ensemble. Ces solutions
sont souvent appelées fondamentales. Les autres solutions (différentes de (0, 0, . . . , 0) qui
convient toujours dans ce cas) s’obtiennent alors par multiplication à partir d’une solution
fondamentale. Ainsi, si on trouve toutes les solutions fondamentales, on aura trouvé toutes
les solutions.

58
Cette √
dernière remarque est par exemple appliquée dans la preuve usuelle de l’irration-
nalité de 2. On se ramène directement à montrer que l’équation diophantienne :

a2 = 2b2

n’a pas de solution non nulle. On remarque que l’équation est homogène et il suffit donc de
chercher les solutions avec pgcd (a, b) = 1. On remarque ensuite que a2 est pair, donc a doit
être pair. Ceci implique que a2 est un multiple de 4, et donc b2 est pair. Ainsi b est pair, et
il n’y a pas de solution avec a et b premiers entre eux.
La remarque sur l’homogénéı̈té implique alors qu’il n’y√a aucune solution hormis la
solution triviale a = b = 0. Cela démontre l’irrationnalité de 2.

Noter que parfois, la manière dont on transforme une solution en une autre est moins
évidente. Par exemple, pour l’équation suivante :

x3 + y 5 = z 2

il faut remarquer que si (x, y, z) est solution et si a est un entier quelconque, alors le tri-
plet (a10 x, a6 y, a15 z) est aussi solution. Si l’on demande ensuite simplement de prouver que
cette équation admet une infinité de solutions en entiers strictement positifs, on conclut en
remarquant que (2, 1, 3) est solution.
On a ainsi prouvé que pour tout entier a, le triplet (2a10 , a6 , 3a15 ) est solution, ce qui en
fait bien une infinité.
De façon plus générale, lorsque l’on souhaite prouver que telle équation admet une infinité
de solutions, il s’agit souvent de trouver une formule. L’exemple de l’équation :

x3 + y 3 + z 3 + t 3 = 3

est frappant. Pour conclure, il suffit de sortir de son chapeau l’identité :


¡ ¢3 ¡ ¢3 ¡ ¢3
4 + 24n3 + 4 − 24n3 + −24n2 + (−5)3 = 3

On pourrait objecter qu’on ne voit pas trop comment on peut arriver à une telle formule.
En réalité, si l’on sait ce que l’on cherche, à force de patience et avec un peu de pratique,
on arrive assez bien à bricoler des coefficients qui conviennent.

4.2 Utilisation des congruences


Une méthode, souvent efficace, pour prouver qu’une équation diophantienne n’a pas de
solution est de considérer la même équation modulo un entier N et de prouver qu’il n’y a
pas de solution dans cette nouvelle situation.
Le premier exemple à considérer est celui de l’équation :

x2 = 4k + 3

S’il existait un couple (x, k) solution, alors il vérifierait la congruence :

x2 ≡ 4k + 3 ≡ 3 (mod 4)

59
mais on a vu que les seuls carrés modulo 4 sont 0 et 1. On en déduit que l’équation de départ
n’a pas de solution.
Il n’y a pas beaucoup de théorie à faire sur le sujet, le point délicat est de trouver un
entier N qui amènera une contradiction lorsque l’on regarde modulo N . Pour cela, on a
quelques principes généraux :
☞ Lorsque l’équation fait intervenir des carrés, il est souvent intéressant de regarder
modulo 4, voire 8 ou 16 (et il est peu utile d’aller au delà – voir proposition 3.6.3).
Retenez que les carrés modulo 4 sont toujours congrus à 0 ou 1, qu’ils sont toujours
congrus à 0, 1 ou 4 modulo 8 et qu’ils sont toujours congrus à 0, 1, 4 et 9 modulo 16.
(Il est utile de connaı̂tre ces résultats par cœur bien que ce soit facile de les retrouver
en dressant une table).
☞ De façon plus générale, si p est un nombre premier et que l’équation fait intervenir des
puissances de p, il peut être bon de regarder modulo p2 ou p3 , voire les suivants...
☞ Lorsque l’on fait la liste des carrés modulo un nombre premier p, l’égalité x2 = (−x)2
entraı̂ne que l’on ne pourra pas obtenir tous les restes possibles. En réalité, on en
obtient exactement p−1 2
. Lorsque l’on a des puissances n-ièmes qui apparaissent (disons
avec n impair), cet argument ne fonctionne plus. Par contre, il reste vrai que de
nombreux restes ne sont pas des puissances n-ième lorsque le modulo p est un nombre
premier congru à 1 modulo n. Cette remarque est on ne peut plus intéressante lorsque
l’équation fait intervenir deux types de puissances : par exemple, si on a un terme en
x2 et un en y 3 , on pourrait être tenté de regarder modulo 7.
☞ Si l’équation fait intervenir un terme de la forme 2n , on pourra également regarder
modulo 2, 4, 8, etc. Ainsi, le terme en question va s’annuler à partir d’une certaine
valeur de n. De même si on a un terme de la forme 3n , on aura intérêt modulo 3, 9, et
ainsi de suite.
☞ De façon un peu plus générale, si une constante, semble-t-il un peu étrange, inter-
vient comme un facteur multiplicatif, il peut être opportun de regarder modulo cette
constante. En effet, on tuera ainsi le terme correspondant.
☞ Encore si l’on a un terme de la forme an (où a est fixé et n est l’inconnue), on peut
chercher un modulo N (pas démesurément trop grand) diviseur de ak − 1 pour un
certain entier k (pas trop grand, par exemple k = 1). En effet, dans ce cas, on aura
ak ≡ 1 (mod N ) et donc la suite des an modulo N sera périodique de période (divisant)
k. Ainsi si k n’est pas choisi trop grand, le terme an ne pourra prendre qu’un petit
nombre de valeurs modulo N . Par exemple, dans le cas des puissances de 2, on pourra
choisir N = 3, N = 5 ou N = 31.

Les heuristiques données précédemment ne doivent pas etre considérées comme parole
d’évangile. Parfois, il peut être préférable de suivre son intuition.

Exercice : Trouver tous les entiers relatifs x et y tels que :

x2 = y 5 − 4

60
Solution : Comme les exposants qui interviennent dans l’équation sont 2 et 5, il peut être
intéressant de l’examiner en réduction modulo un nombre premier p tel que 2 et 5 divisent
p − 1. Le premier qui se présente est p = 11.
Les puissances cinquièmes modulo 11 sont 0, 1 et −1, donc le second membre de l’équation
est congru à 7, 8 ou 6 modulo 11. Par ailleurs, les carrés modulo 11 sont 0, 1, 4, 9, 5 et 3. √
Il
en résulte immédiatement que l’équation n’a pas de solution.

Remarquons pour finir qu’il existe des équations diophantiennes n’admettant pas de
solutions, mais qui en admettent modulo N pour tout entier N . L’exemple de base est
donné par l’exercice suivant :

Exercice : Montrer que l’équation :


¡ 2 ¢¡ ¢¡ ¢
x − 2 x2 + 7 x2 + 14 = 0

n’admet pas de solution entière mais admet une solution modulo N pour tout entier non nul
N . On pourra utiliser le critère d’Euler (qui sera discuté dans le second tome) qui affirme
que si p est un nombre premier impair et a un entier premier à p, alors a est un carré modulo
p si, et seulement si :
p−1
a 2 ≡ 1 (mod p)

Solution
√ : Déjà, il est facile de voir que les seules solutions réelles de l’équation sont 2 et
− 2 qui ne sont pas des entiers.
Considérons p un nombre premier impair et montrons que soit 2, soit −7, soit −14 est
p−1
un carré modulo p. Remarquons que pour tout a premier à p, le carré de a 2 est congru à
1 modulo p et donc on a la factorisation :
³ p−1 ´ ³ p−1 ´
a 2 −1 a 2 + 1 ≡ 0 (mod p) (1)

Ainsi p divise un des deux facteurs. Supposons que ni 2, ni −7 ne soit un carré modulo p.
Alors d’après le critère d’Euler, p ne divise pas le premier facteur de (1) lorsque a = 2 ou
a = −7. C’est donc qu’il divise le second facteur et que l’on a les congruences :
p−1
2 2 ≡ −1 (mod p)
p−1
(−7) 2 ≡ −1 (mod p)

et en multipliant ces deux congruences, on arrive à :


p−1
(−14) 2 ≡1 (mod p)

ce qui prouve que −14 est un carré modulo p.


On vient donc de prouver que 2, −7 ou −14 est un carré modulo p, et donc l’équation :
¡ 2 ¢¡ ¢¡ ¢
x − 2 x2 + 7 x2 + 14 ≡ 0 (mod p)

a bien une solution. D’après la proposition 3.6.3, cette équation a également une solution
modulo pn pour tout entier n (on utilise ici encore une fois le fait que p est impair).

61
Soit N = 2k une puissance de 2. Montrons que −7 est un carré modulo N . Pour k 6 3,
on vérifie que l’on a 32 ≡ −7 (mod 2k ). On applique ensuite le proposition 3.6.5 qui conclut
directement.
En résumé, on a prouvé que notre équation admet une solution modulo pk pour tout
nombre premier p et tout exposant k. Notons P (X) = (X 2 − 2) (X 2 + 7) (X 2 + 14). Soit N
un entier et N = pα1 1 · · · pαd d sa décomposition en facteurs premiers. On sait qu’il existe des
entiers xi tels que :

P (x1 ) ≡ 0 (mod pα1 1 )


P (x2 ) ≡ 0 (mod pα2 2 )
..
.
P (xd ) ≡ 0 (mod pαd d )

Soit x un entier tel que x ≡ xi (mod pα1 1 ) pour tout entier i (qui existe d’après le lemme

chinois). Il vérifie bien P (x) ≡ 0 (mod N ).

Lorsqu’il existe un entier N pour lequel une équation diophantienne donnée n’admet pas
de solution modulo N , on dit qu’il y a obstruction locale. Nous venons donc de voir qu’il
peut y avoir des obstructions d’autre nature pour empêcher l’existence de solutions entières.
L’étude de ces obstructions a occupé et occupe encore beaucoup de mathématiciens en
géométrie algébrique.

4.3 Descente infinie


La descente infinie est une méthode introduite et abondamment utilisée par Fermat. Le
but est de prouver qu’une certaine équation diophantienne n’admet pas (ou très peu) de
solutions. Pour cela, on part d’une solution hypothétique et on en construit une nouvelle,
strictement plus petite dans un certain sens.
On obtiendrait ainsi une suite strictement décroissante de solutions, ce qui n’est en
général pas possible (de la même façon qu’il n’existe aucune suite infinie d’entiers positifs
strictement décroissante).

Un premier exemple simple qui illustre ce principe est à nouveau l’irrationalité de 2.
On est à nouveau amené à considérer l’équation :

a2 = 2b2
¡a b¢
On prouve que a est pair puis¯ b ¯ que b l’est, et on voit que 2 , 2 est une nouvelle solution.
D’autre part si b 6= 0, on a ¯ 2 ¯ < |b| (voici notre condition de décroissance).
Ainsi, si l’on part d’une solution (a0 , b0 ) avec b0 6= 0, on peut construire une nouvelle
solution (a1 , b1 ) (en l’occurrence a1 = a2 et b1 = 2b ) avec |b1 | < |b0 |. Puis on continue, on
construit (a2 , b2 ), (a3 , b3 ), et ainsi de suite. On construit ainsi une suite (bi ) telle que :

|b0 | > |b1 | > |b2 | > · · ·

ce qui constitue une contradiction.

62

Ainsi on a forcément b = 0 puis directement a = 0, et 2 est à nouveau irrationnel.

Voici, en exercice, un autre exemple tout à fait similaire :

Exercice : Trouver tous les entiers x, y et z tels que :

x3 + 9y 3 = 3z 3

Solution : On part d’une éventuelle solution (x, y, z) distincte du triplet (0, 0, 0). L’équation
implique que x3 est multiple de 3, et donc x l’est aussi. Mais alors x = 3x0 et l’équation
devient (après simplification par 3) :
3
9x0 + 3y 3 = z 3

et on déduit de cela que z est multiple de 3. On écrit z = 3z 0 , l’équation devient :


3 3
3x0 + y 3 = 9z 0

et on obtient 3 divise y. Posons y 0 = y3 . On vérifie que le triplet (x0 , y 0 , z 0 ) est encore solution
de l’équation de départ et qu’il est plus petit dans le sens :

|x0 | + |y 0 | + |z 0 | < |x| + |y| + |z|

Le principe de descente infinie permet alors de conclure que l’unique solution est x = y √
=
z = 0.

Notons que ces deux cas relèvent encore de ce que l’on appelle une obstruction locale.
Ici, il y a une solution, donc évidemment,
√ il y a une solution modulo N pour tout entier
N . Cependant, pour l’irrationalité de 2 par exemple, on a exactement ( !) prouvé que si
a2 ≡ 2b2 (mod 2k ), alors a ≡ b ≡ 0 (mod 2k ). Ainsi une éventuelle solution devrait être telle
que a ≡ b ≡ 0 (mod 2k ) pour tout entier k et les seuls entiers vérifiant cela sont a = b = 0.
L’argument est exactement le même pour le second exemple, sauf que 2 est remplacé par
3.

Obstruction globale
Le principe de descente infinie s’applique toutefois dans des situations différentes. L’exemple
le plus classique est celui du cas particulier de l’équation de Fermat pour n = 4. Pour l’illus-
trer nous allons avoir besoin du théorème suivant :

Théorème 4.3.1 (Triplets pythagoriciens) Soient x, y et z des entiers positifs premiers


entre eux dans leur ensemble vérifiant :

x2 + y 2 = z 2

Alors, soit x, soit y est pair. Dans le cas où c’est x qui l’est, il existe des entiers m et n
premiers entre eux et de parité contraire tels que x = 2mn, y = m2 − n2 et z = m2 + n2 .

63
Remarque. On vérifie immédiatement que réciproquement, les triplets fournis par le théorème
précédent sont effectivement solutions de l’équation x2 + y 2 = z 2 . On a ainsi entièrement
résolu cette équation.

Démonstration. Si d est un diviseur commun de x et y, alors d2 divise x2 + y 2 et donc


z 2 . Ainsi d divise z et d est un diviseur commun de x, y et z et d’après l’hypothèse d = 1.
Finalement, x et y sont premiers entre eux. De même on prouve que x, y et z sont premiers
entre eux deux à deux.
Si x et y étaient tous les deux impairs, on aurait x2 ≡ y 2 ≡ 1 (mod 4), et donc z 2 =
x2 + y 2 ≡ 2 (mod 4), ce qui est impossible. Au moins l’un des deux est pair. Supposons que
ce soit x et écrivons x = 2x0 .
L’équation devient alors :
4x02 = (z − y) (z + y)
Les deux facteurs z − y et z + y sont de même parité et donc tous les deux pairs. En outre,
si d est un diviseur commun de z − y et de z + y, il divise leur somme et leur différence,
c’est-à-dire 2z et 2y et donc 2 puisque y et z sont premiers entre eux. Cela prouve que les
entiers z−y
2
et z+y
2
sont premiers entre eux. Leur produit est un carré, ce sont donc tous les
deux des carrés :
z−y z−y
= m2 ; = n2
2 2
pour m et n des entiers positifs premiers entre eux. En reportant dans l’équation, il vient
x2 = 4m2 n2 puis x = 2mn (puisque x est supposé positif).
Finalement m et n sont de parité contraire, car sinon y et z seraient tous deux pairs. ¤

On est maintenant prêt pour donner la descente infinie faite par Fermat pour prouver
qu’il n’existe aucun triplet (x, y, z) d’entiers strictement positifs tels que x4 + y 4 = z 4 .
En réalité, on prouve un résultat légèrement plus fort : il n’existe pas d’entiers x, y et z
strictement positifs tels que x4 + y 4 = z 2 .
Supposons qu’un tel triplet existe. Déjà on peut supposer que x, y et z sont premiers entre
eux dans leur ensemble, sinon on obtient directement une solution plus petite en divisant
par le pgcd. Dans ces conditions, en appliquant le théorème précédent, et quitte à échanger
les rôles de x et de y, il existe des entiers m et n premiers entre eux tels que :

x2 = 2mn ; y 2 = m2 − n2 ; z = m2 − n2

La deuxième égalité fournit m2 = n2 +y 2 et les entiers m, n et y sont premiers entre eux dans
leur ensemble. En outre, y est impair (puisque x est pair), et donc le théorème précédent
s’applique et donne l’existence d’entiers u et v premiers entre eux tels que :

n = 2uv ; y = u2 − v 2 ; m = u2 + v 2

On obtient x2 = 2mn = 4uv (u2 + v 2 ). Si d est un diviseur commun de u et de u2 + v 2 , il


divise v 2 et donc vaut 1 puisque u et v sont premiers entre eux. Ainsi les nombres u, v et
u2 + v 2 sont premiers entre eux deux à deux et leur produit est un carré. Chacun d’eux est
alors un carré et il existe des entiers x0 , y 0 et z 0 tels que :
2 2 2
u = x0 ; v = y0 ; u2 + v 2 = z 0

64
On tire directement de là x0 4 + y 0 4 = z 0 2 et donc une nouvelle solution.
D’autre part, on a l’argument de descente :

z 0 6 z 02 = m < m2 + n2 = z

l’inégalité stricte résultant du fait que n > 0 (x étant supposé non nul).
Cela conclut la preuve.

4.4 Équations de degré 2


Il est important de noter que l’on a des méthodes assez générales pour déterminer toutes
les solutions rationnelles d’une équation de degré 2 ayant deux inconnues.
Nous allons présenter la méthode sur un exemple simple. Supposons que l’on ait à dé-
terminer les rationnels x et y tels que :

x2 + y 2 = 1

Autrement dit, on cherche à trouver tous les points à coordonnées rationnelles sur le cercle
unité. Pour cela, il nous faut déjà connaı̂tre un point particulier A (à coordonnées ration-
nelles) sur le cercle : ici, c’est facile, on prend celui de coordonnées (1, 0) par exemple. Si
l’on trace une droite ∆ (non verticale) passant par A, elle recoupe le cercle en un point B.

Le fait est que le point B est à coordonnées rationnelles si, et seulement si la pente
de la droite est rationnelle. En effet, si ∆ n’est pas verticale, une équation de ∆ est de la
forme y = t (x − 1). Les points d’intersection de ∆ et du cercle vérifient donc le système
d’équation :
x2 + y 2 = 1
y = t (x − 1)
ce qui nous donne :
x2 + t2 (x − 1)2 = 1
ou encore : ¡ ¢ ¡ ¢
1 + t2 x2 − 2xt2 + t2 − 1 = 0

65
Cette équation admet à l’évidence x = 1 comme solution (puisque A est un point commun à
2t2
∆ et au cercle) et la somme des deux racines est donnée par 1+t 2 . L’autre racine vaut donc :

t2 − 1
x= 2
t +1
Le y correspondant est donné via la formule y = t (x − 1) = t−2t
2 +1 . Ces deux nombres sont

bien rationnels si t l’est.


Réciproquement, il est clair que si les points A et B (distincts) sont à coordonnées
rationnelles, la pente de la droite (AB) est rationelle. Ainsi, on a prouvé que toutes les
solutions rationnelles, hormis la solution x = 1, y = 0 sont données par les formules :

t2 − 1 −2t
x= ; y=
t2 + 1 1 + t2

Avant de continuer, remarquons que cela donne une nouvelle preuve du théorème 4.3.1.
En effet, si x, y et z sont solutions de x2 + y 2 = z 2 , on obtient :
³ x ´2 ³ y ´2
+ =1
z z
et on applique la résolution obtenue précédemment.

Notons que dans le cas général d’une équation de degré 2, on n’a plus forcément affaire
à un cercle, mais à une conique (qui peut-être soit une ellipse, soit une parabole, soit une
hyperbole). Toutefois la méthode se généralise mot pout mot : on trouve une solution par-
ticulière, puis on regarde l’intersection de la conique avec les droites de pente rationnelle
passant par cette solution. On obtient ainsi toutes les solutions rationnelles de l’équation.

Application à la descente infinie


Les idées précédentes s’appliquent encore, du moins partiellement, lorsque l’équation fait
intervenir un plus grand nombre de variables mais que certaines d’entre elles apparaissent
en degré inférieur ou égal à 2. On isole alors ces variables en considérant les autres comme
paramètres et on résout l’équation comme on l’a expliqué dans le paragraphe précédent.
Cette méthode ne permet pas en général de trouver directement toutes les solutions
rationnelles de l’équation, mais permet à partir de l’une d’entre elles d’en construire une
nouvelle. Elle peut donc être utilisée lors d’une descente infinie pour prouver qu’une équation
donnée n’a pas de solution. Ou, au contraire, elle peut aussi permettre de prouver qu’une
équation a une infinité de solutions : on part de l’une d’entre elles qui saute aux yeux, à partir
de celle-ci on en construit une nouvelle, puis une autre, et ainsi de suite. Il faut finalement
prouver que toutes les solutions obtenues sont distinctes mais c’est souvent le cas.

Équation de Pell-Fermat
L’équation de Pell-Fermat est la suivante :

x2 − dy 2 = ±1

66
où d est un entier que l’on suppose sans facteur carré.
Intéressons-nous en premier lieu à l’équation :

x2 − dy 2 = 1

Pour la résoudre on peut être tenté d’appliquer la méthode vue précédemment. On constate
que le couple (1, 0) est toujours solution. On introduit donc un nombre rationnel t tel que
y = t (1 − x). L’équation devient alors :

x2 − dt2 (1 − x)2 = 1

et la seconde solution de cette équation est :

dt2 + 1
x=
dt2 − 1
ce qui nous donne :
−2t
y=
dt2 − 1
On a ainsi déterminé toutes les solutions rationnelles. Si l’on s’intéresse désormais aux so-
lutions entières, il faut se demander pour quels rationnels t = ab , les fractions x et y sont
entières, c’est-à-dire pour quels entiers a et b, da2 − b2 divise da2 + b2 . Le pgcd de ces deux
nombres divise 2da2 et 2b2 qui est un diviseur de 2d puisque a et b et donc a2 et b2 sont
premiers entre eux.
Donc, déjà dans le cas où d = 2, on est ramené à déterminer les entiers a et b pour
lesquels le dénominateur da2 − b2 est un diviseur de 4... et on revient ainsi presque à la case
départ.
Nous voyons sur cet exemple que la méthode des équations de degré 2 si elle fonctionne
sans bavure pour les solutions rationnelles, peut être mise en défaut si l’on ne recherche que
les solutions entières.

Toutefois, on peut résoudre l’équation de Pell-Fermat et plus précisément, on a le théo-


rème suivant :

Théorème 4.4.1 Soit d un entier sans facteurs carrés. L’équation (que l’on cherche à
résoudre en entiers positifs) :
x2 − dy 2 = 1
admet toujours
√ au moins une solution. Notons (x0 , y0 ) une solution non nulle pour laquelle
x0 + y0 d est minimal. Une telle solution s’appelle une solution fondamentale.
Les autres solutions de l’équation sont les couples (xn , yn ) définis par :
√ ³ √ ´n
xn + yn d = x0 + y0 d

pour tout entier relatif n.

67
Démonstration. Nous allons admettre dans cette preuve l’existence d’une solution non
triviale (c’est-à-dire pour laquelle x0 6= 0 et y0 6= 0) et donc l’existence d’une solution
fondamentale. Cette existence est prouvée dans le second tome, dans le chapitre sur les
fractions continues.
Ensuite, il est facile de voir que xn et yn définis par :
√ ³ √ ´n
xn + yn d = x0 + y0 d

forment toujours une solution. En effet, on multiplie par la quantité conjuguée pour obtenir :
¡ ¢n
x2n − dyn2 = x20 − dy02 = 1

Montrons que ces solutions sont √ les seules. La méthode consiste à partir
√ d’une solution
(x, y). On considère le nombre x + y d et on veut le diviser par x0 + y0 d :
√ ³
x+y d √ ´³ √ ´ √
√ = x + y d x0 − y0 d = (xx0 − dyy0 ) + (yx0 − xy0 ) d
x0 + y0 d
√ √ √
Comme x0 + y0 d > 1, on a (xx0 − dyy0 ) + (yx0 − xy0 ) d < x + y d. D’autre part, les
couples (x0 , y0 ) et (x, y) sont solutions de l’équation, et donc on a :
µ ¶2 µ ¶2
x0 1 x 1
=d+ 2 ; =d+ 2
y0 y0 y y
et comme y > y0 et d > 1, il vient yx0 − xy0 > 0 et xx0 − dyy0 > 0. On obtient par le fait
une solution plus petite.
À ce niveau, on a tous les éléments pour conclure et on laisse le lecteur le faire proprement.
¤

4.5 Équations de degré 3


Certaines méthodes du paragraphe précédent s’appliquent encore aux équations de degré
3. Supposons donné un polynôme à deux variables P (x, y) ne faisant intervenir que des
termes dont le degré total est inférieur à 3 et intéressons à l’équation diophantienne :

P (x, y) = 0

dont on cherche au choix les solutions entières ou rationnelles.


Dans cette situation, il est bien plus difficile de donner une méthode générale pour
déterminer toutes les solutions, mais il reste possible, dans certains cas, de construire à partir
de nouvelles solutions à partir d’anciennes. Là encore, cela peut se combiner fructueusement
à une descente infinie ou permettre de montrer qu’une équation donnée admet une infinité
de solutions.
Voyons comment on fait la construction sur un exemple. Supposons que l’équation dio-
phantienne soit la suivante :
y 2 = x3 − 5x

68
et donc P (x, y) = y 2 − x3 + 5x. On cherche des points à coordonnées rationnelles sur la
courbe6 donnée par cette équation :

On commence par trouver un point évident sur la courbe. Ici, le point A de coordonnées
x = 1, y = −2 convient. Si l’on trace une droite passant par A, et que l’on cherche les autres
intersections de cette droite avec la courbe, on va être amené à résoudre une équation de
degré 2 qui donc à toutes les chances de faire intervenir des racines carrées dans sa résolution.
Ce n’est pas agréable.
Il y a deux moyens de contourner ce problème. Le premier est de considérer une droite
qui passent par deux points à coordonnées rationnelles de la courbe. Le troisième point
d’intersection sera donné par une équation de degré 1 et donc à coordonnées rationnelles.
Cependant, cela nécessite de connaı̂tre deux points de la courbe.
Le second moyen, que nous allons illustrer ici, est de choisir une droite particulière passant
par le point A, en l’occurrence la tangente à la courbe. L’équation de la tangente7 est (de
façon très générale) donnée par la formule :
∂P ∂P
(x0 , y0 ) · (x − x0 ) + (x0 , y0 ) · (y − y0 ) = 0
∂x ∂y

où x0 et y0 sont les coordonnées du point duquel on cherche la tangente (et où ∂P
∂x
désigne
la dérivée du polynôme P par rapport à la variable x, et donc en supposant y constant). Ici
x0 = −1 et y0 = −2. On calcule les dérivées partielles et on obtient :
∂P ∂P
(x, y) = −3x2 + 5 ; (x, y) = 2y
∂x ∂y
Par application de la formule, on voit que la tangente à la courbe en A a pour équation :

2 (x + 1) = 4 (y + 2)
6
Une courbe donnée par une équation de degré 3 est appelée une cubique.
7
Il se peut que la formule donnée n’aboutisse pas à une véritable équation de droite, lorsque les deux
coefficients ∂P ∂P
∂x (x0 , y0 ) et ∂y (x0 , y0 ) sont simultanément nuls. Dans ce cas, on dit que le point (x0 , y0 ) est
singulier. Un point qui n’est pas singulier est dit régulier. Une courbe d’équation y 2 = x3 + ax + b (avec a
et b réels) dont tous les points sont réguliers est appelée une courbe elliptique.

69
soit encore :
1 3
y = x−
2 2
En reportant désormais dans l’équation diophantienne, on obtient l’équation polynômiale :
1 7 9
x3 − x2 − x − = 0
4 2 4
Elle admet évidemment (−1) comme racine puisque la tangente passe par le point A. Mais
en réalité la racine (−1) est double (ceci est justement lié à la condition de tangence). Le
polynôme se factorise donc par (x + 1)2 et la troisième solution est rationnelle :
9
x=
4
Le y correspondant est − 38 . On a ainsi construit une seconde solution de l’équation. Graphi-
quement, on a :

A0

B
A

Désormais on a deux points à coordonnées rationnelles sur la courbe. Cependant la droite


droite (AB) ne recoupe pas la courbe, puisque A est point d’intersection double. Mais on
peut regarder la droite (A0 B) où A0 est le symétrique de A par rapport à l’axe des abscisses
par exemple. On peut également si l’on préfère considérer la tangente en B qui elle aussi,
comme précédemment, fournit un nouveau point. De cette façon, on obtient une infinité de
solutions rationnelles à l’équation diophantienne de départ.

Notons que parfois cette méthode est inefficace. Par exemple si l’on considère l’équation
y = x3 − x et que l’on essaie de faire la construction en partant de la solution x0 = 1,
2

y0 = 0 par exemple, l’équation de la tangente sera x = 1, ce qui ne fournit pas de nouvelles


solutions (le problème étant que les termes en x3 se sont simplifiés).

4.6 Exercices
Exercice 167 a) Prouver que le produit de deux entiers consécutifs n’est jamais un carré
parfait non nul.
b) Prouver que le produit de trois entiers consécutifs n’est jamais un carré parfait non
nul.

70
c) Prouver que le produit de quatre entiers consécutifs n’est jamais un carré parfait non
nul.

Exercice 168 Trouver tous les rationnels x et y vérifiant :

x2 + 3y 2 = 1

Exercice 169 a) Trouver tous les entiers n et a strictement positifs tels que 5n = a2 .
b) Trouver tous les entiers n et a strictement positifs tels que 5n = a2 − 1.
c) Trouver tous les entiers n et a strictement positifs tels que 5n = a2 − 2.

Exercice 170* (France 02) On considère 2002 rationnels x1 , . . . , x2002 tels que, pour tout
sous-ensemble I de {1, . . . , 2002} de cardinal 7, il existe un sous-ensemble J de {1, . . . , 2002}
de cardinal 11 vérifiant :
1X 1 X
xi = xj
7 i∈I 11 j∈J
Prouver que tous les xi sont égaux.

Exercice 171* (Biélorussie 99) Prouver qu’il existe une infinité de triplets de rationnels
non entiers positifs (x, y, z) tel que :
© 3ª © 3ª © 3ª
x + y = z

où {t} = t − [t] désigne la partie décimale de t.

Exercice 172* Quelle est la valeur minimale positive de 12m − 5n pour m et n des entiers
strictements positifs.

Exercice 173* (Hongrie 98) Trouver tous les entiers strictement positifs x, y et z tels
que z > 2 et :
(x + 1)2 + . . . + (x + 99)2 = y z

Exercice 174* Montrer que tout entier relatif peut s’écrire comme la somme de cinq cubes
d’entiers relatifs d’une infinité de manières différentes.

Exercice 175* Trouver tous les entiers strictement positifs x et y tels que xy = y x .

Exercice 176* (Irlande 96) Soient p un nombre premier et a et b des entiers positifs.
Prouver que si 2p + 3p = an , alors n = 1.

Exercice 177* (Lituanie 94) Trouver les entiers m, n et k tels que k > 2 et :

1 + 2! + 3! + . . . + n! = mk

Exercice 178* (Italie 94) Trouver tous les entiers x et y pour lesquels :

y 2 = x3 + 16

71
Exercice 179* (OIM 81) Soient m et n deux entiers (1 6 m 6 1981 et 1 6 n 6 1981)
vérifiant : ¡ 2 ¢2
n − mn − m2 = 1
Déterminer le maximun de m2 + n2 .

Exercice 180* (Putnam 73) Un fermier possède un troupeau de 1973 vaches. Chaque
vache a une masse qui est un nombre entier, et chaque fois que l’on retire une vache du
troupeau, on peut séparer le groupe restant en deux groupes de 986 vaches de masse égale.
Montrer que toutes les vaches du troupeau ont la même masse.

Exercice 181* (Saint Petersbourg 97) Soient x, y et z des entiers strictement positifs
tels que 2xx + y y = 3z z . Prouver que x = y = z.

Exercice 182* (Irlande 95) Pour quelles valeurs de a l’équation :

x2 + axy + y 2 = 1

admet-elle une infinité de solutions dans Z ?

Exercice 183* (Afrique du sud 95) Soit x et y des entiers positifs tels que :

x2 + y 2 + 1
A=
xy
est entier. Montrer que A = 3.

Exercice 184* (Bac 2003) Trouver tous les entiers x, y et z tels que :

x2 + y 2 = 7z 2

Exercice 185* (Kömal) Trouver tous les entiers relatifs a et b tels que a4 + (a + b)4 + b4
soit un carré parfait.

Exercice 186* (CG 90) a) Trouver trois nombres entiers naturels a, b, c distincts ou non,
tels que :
1 1 1 1
= 2+ 2+ 2
4 a b c
b) Déterminer tous les entiers naturels n tels qu’il existe n nombres entiers naturels
x1 , . . . , xn distincts ou non, vérifiant :
1 1 1
1= 2
+ 2 + ... + 2
x1 x2 xn

Exercice 187* (SL 83) Trouver tous les entiers relatifs x pour lesquels 1 + x + x2 + x3 + x4
est un carré.

72
Exercice 188** Trouver tous les entiers positifs ou nuls a, b, c et d vérifiant :

a2 + 5b2 − 2c2 − 2cd − 3d2 = 0

Exercice 189** (Putnam 76) Trouver tous les nombres premiers p, q et les entiers r, s > 2
vérifiant :
|pr − q s | = 1

Exercice 190** (SL 02) Quel est le plus petit entier t pour lequel il existe des entiers
x1 , . . . , xt vérifiant :
x31 + . . . + x3t = 20022002

Exercice 191** Trouver tous les entiers x et y vérifiant :

y 2 = x3 − 3x + 2

Exercice 192** (Taı̈wan 98) Existe-t-il une solution de :

x2 + y 2 + z 2 + u2 + v 2 = xyzuv − 65

avec x, y, z, u et v des entiers supérieur à 1998 ?

Exercice 193** (Inde 98) Trouver tous les entiers strictement positifs x, y et n tels que
pgcd (x, n + 1) = 1 et :
xn + 1 = y n+1

Exercice 194** (OIM 97) Trouver tous les couples (a, b) d’entiers a > 1, b > 1 vérifiant
l’équation :
2
ab = ba

Exercice 195** (OIM 88) Soient a et b deux entiers strictement positifs tels que ab + 1
2 +b2
divise a2 + b2 . Montrer que aab+1 est un carré parfait.

Exercice 196** (Équation de Markov) Pour quels entiers positifs ou nuls n l’équation :

a2 + b2 + c2 = nabc

admet-elle une solution en entiers strictement positifs ?

Exercice 197** (Biélorussie 00) On considère l’équation :

(aa )n = bb (?)

a) Pour quelles valeurs de n, (?) admet-elle une solution avec a, b > 1.


b) Résoudre (?) pour n = 5.

73
Exercice 198*** (SL 95) Trouver tous les entiers strictement positifs x et y tels que :

x + y 2 + z 3 = xyz

où z = pgcd (x, y).

Exercice 199*** (Fermat) a) Trouver tous les triangles rectangles à côtés entiers dont
l’aire est le carré d’un nombre entier.
b) Trouver tous les entiers positifs x, y et z vérifiant :

x4 − y 4 = z 2

Exercice 200**** (Moscou 99) Trouver tous les entiers n, k, `, m tels que ` > 1 et :
¡ ¢`
1 + nk = 1 + nm

74
5 Corrigé des exercices
5.1 Exercices de « Premiers concepts »
Solution de l’exercice 1 : On sait que si la décomposition en facteurs premiers de n est :

n = pα1 1 · · · pαk k

alors d (n) est donné par la formule :

d (n) = (α1 + 1) · · · (αk + 1)

Ce dernier nombre est impair si, et seulement si chacun des facteurs est impair, c’est-à-dire
si, et seulement si αi est pair pour tout i. Ceci est bien équivalent au fait que n soit un carré.

Solution de l’exercice 2 : On remarque que :


¡ ¢ ¡ ¢
3 3n−1 + 5n−1 < 3n + 5n < 5 3n−1 + 5n−1
n
3 +5 n
et donc le quotient 3n−1 +5n−1
supposé entier ne peut valoir que 4. L’égalité donne 3n−1 = 5n−1
et la seule solution est obtenue pour n = 1.

Solution de l’exercice 3 : Nous allons montrer qu’il est multiple de 2 et de 3. Comme 2 et 3


sont premiers entre eux, cela conclura. La factorisation :
¡ ¢
n3 − n = n n2 − 1 = n (n − 1) (n + 1)

nous montre que n3 − n s’écrit comme le produit de trois nombres consécutifs. Parmi eux,
il y a forcément un multiple de 2 et un multiple de 3 (pas forcément distincts bien sûr). Le
produit est donc à la fois multiple de 2 et de 3 comme on le voulait.

Solution de l’exercice 4 : Il suffit de remarquer que :

3 (14n + 3) − 2 (21n + 4) = 1

Le théorème de Bézout assure que numérateur et dénominateur de la fraction sont premiers


entre eux et donc qu’elle est irréductible.

Solution de l’exercice 5 : Cela résulte directement des formules :

5x + 2 = 8 (2x + 3) − 11 (x + 2)
2x + 3 = 7 (5x + 2) − 11 (3x + 1)

Solution de l’exercice 6 : On factorise p2 − 1 = (p − 1) (p + 1). Comme p est un nombre


premier différent de 2, il est impair. Ainsi p − 1 et p + 1 sont tous les deux pairs et le produit
est un multiple de 4.
De même p > 3 et donc p ne peut-être un multiple de 3. On en déduit que soit p − 1,
soit p + 1 est un multiple de 3, et donc p2 − 1 en est également un.
En conclusion, p2 − 1 est multiple de 3 et de 4, et donc de 12.

75
Solution de l’exercice 7 : Soit p un nombre premier. L’hypothèse nous dit que nvp (a) >
(n + 1) vp (b), soit encore : µ ¶
1
vp (a) > 1 + vp (b)
n
et par passage à la limite vp (a) > vp (b) pour tout nombre premier p. On en déduit que a
divise b.

Solution de l’exercice 8 : La décomposition en facteurs premiers nous donne :

n = pα1 1 · · · pαk k

pour certains nombres premiers pi distincts deux à deux et certains exposants αi strictement
positifs. On a pi > 2 et donc a fortiori pαi i > 2. On en déduit que n > 2k et puis l’inégalité
proposée en prenant les logarithmes.

Solution de l’exercice 9 : Comme p divise nk , on a vp (nk ) > 0 et en particulier vp (n) =


1
v (nk ) > 0. On en déduit que p divise n.
k p

Solution de l’exercice 10 : Notons a le plus petit élément de X et b le deuxième plus petit


(et donc b > 1). D’après l’hypothèse, il existe k ∈ X tel que b = ak 2 . Il est évidemment
que l’on ne peut pas avoir k > b, c’est donc que k = a puis b = a3 . Notons qu’alors a 6= 1
puisque b 6= a.
Supposons que X contienne un autre élément c que l’on choisit encore minimal. Encore
d’après l’hypothèse, il doit exister k et k 0 dans X tels que c = ak 2 et c = bk 02 = a3 k 02 . De
là, on déduit que ak 2 = a3 k 02 puis (ak 0 )2 = k 2 et finalement ak 0 = k.
Par ailleurs, les entiers k et k 0 sont deux éléments de X avec k 0 < k, donc à appliquant
une nouvelle fois l’hypothèse, il doit exister k 00 ∈ X tel que k = k 0 k 002 . On en déduit que
a = k 002 et donc que, puisque a > 1, il vient k 00 < a. Cela contredit la minimalité de a.
On en déduit que X est réduit à {a, a3 } puis que les ensembles de cette forme sont les
seules solutions.

Solution de l’exercice 11 : Comme n est un multiple de 18, il s’écrit :


Y γ
n = 2α 3β pi i
i

pour certains entiers α > 1, β > 2 et γi > 0, et pour certains nombres premiers pi > 5 deux
à deux distincts. D’autre part on constate que 18 a exactement 6 diviseurs (qui sont 1, 2,
3, 6, 9 et 18). Comme un diviseur de 18 doit être un diviseur de n, on a forcément d1 = 1,
d2 = 2, d3 = 3, d4 = 6, d5 = 9 et d6 = 18. En particulier, 4 ne divise pas n et α = 1.
Le nombre de diviseurs de n est donné par la formule :
Y
2 (β + 1) (γi + 1)
i

Ce nombre doit faire 16 et comme β + 1 > 3, les seules solutions sont β = 3 (et γ1 = 1) et
β = 7 (et γ1 = 0). Les éventuelles solutions sont donc, soit 2 × 37 , soit 2 × 32 × p pour un
nombre premier p > 19.

76
On calcule les diviseurs successifs de 2 × 37 : d7 = 27, d8 = 54, d9 = 81. On a d9 − d8 > 17
et ce nombre n’est donc pas solution.
De même, on calcule les diviseurs de 2 × 32 × p. Si 19 6 p < 27, on a d7 = p, d8 = 27,
d9 = 2p, ce qui fournit l’équation 2p − 27 = 17 et donc p = 22 qui n’est pas premier. Si
27 < p < 54, on a d7 = 27, d8 = p, d9 = 54 et donc 54 − p = 17 puis p = 37, qui convient.
Si p > 54, on a d7 = 27, d8 = 54, d9 = p, ce qui donne p = 71.
Finalement, il y a deux solutions : n = 2 × 33 × 37 = 1998 et n = 2 × 33 × 71 = 3834.
Solution de l’exercice 12 : Un diviseur commun à a et à b doit diviser bc − 1 et donc 1. Ainsi
a et b sont premiers entre eux. De même, a, b et c sont premiers entre eux deux à deux.
Si a divise bc − 1, il divise également bc + ac + ab − 1. De même, b et c doivent diviser
bc + ac + ab − 1. Comme a, b et c sont premiers entre eux deux à deux, on en déduit que abc
divise ab + bc + ca − 1. Le quotient :
ab + bc + ca − 1 1 1 1 1
= + + −
abc a b c abc
3
est inférieur ou égal à 2 . Il ne peut donc valoir que 1.
On est amené à résoudre :
1 1 1 1
+ + − =1
a b c abc
On ne peut pas avoir a, b et c supérieurs ou égaux à 3. Sans perte de généralité, on peut
supposer a 6 b 6 c, et donc a 6 3 et donc a = 2 ou a = 3. Si a = 2, l’équation devient :
1 1 1 1
+ − =
b c 2bc 2
et comme précédemment b et c ne peuvent pas être simultanément supérieurs ou égaux à 4.
On a donc b = 2 ou b = 3. Pour b = 2, l’équation n’a pas de solution en c. Pour b = 3, on
obtient c = 5. On vérifie que le triplet (2, 3, 5) est solution.
De même, on traite le cas a = 3. L’équation devient alors :
1 1 1 2
+ − =
b c 3bc 3
Ici, on ne peut avoir b et c supérieurs ou égaux à 3 et comme ils sont supposés supérieurs à
a, il n’y a pas de solution.
Finalement, les solutions sont le triplet (2, 3, 5) et toutes ses permutations.
Solution de l’exercice 13 : On sait qu’il existe un nombre premier p qui est tel que tous les
nombres p + 1, . . . , p + n soient composés.
Xavier peut alors tester les entiers de l’ensemble {1, . . . , n − 1} les uns après les autres.
Pour tester 1, il propose le nombre p − 1 : si Pierre répond oui, alors le nombre choisi est 1,
sinon ce n’est pas 1. Ensuite, il teste 2 et proposant le nombre p − 2, et ainsi de suite.
Il arrive donc à conclure en moins de n − 1 questions, puisqu’il est inutile de tester n − 1 :
si aucun des nombres p − k n’est premier pour k 6 n − 2, c’est donc que c’est m = n − 1.
Solution de l’exercice 14 : Notons p, q, ` les trois nombres premiers en question. Si leurs
racines cubiques étaient dans une même progression arithmétique de raison r, il existerait
des entiers m et n non nuls et distincts tels que :
√ √ √ √
3
3
p − 3 q = mr et 3 p − ` = nr

77
On a donc : √
√ n √ 3 √
3
p− `=
( 3 p − 3 q)
m
et donc il existe des rationnels non nuls α et β tels que :
√ √ √3
α3p+β 3q = `

En élevant au cube, il vient donc :


√ √ √
` = α3 p + β 3 q + 3αβ(α 3 p + β 3 q) 3 pq
p
= α3 p + β 3 q + 3αβ 3 pq`

Il en résulte que t = 3
pq` est rationnel. Mais on doit avoir vp (t3 ) = 3vp (t) = 1, ce qui est
absurde.

Solution de l’exercice 15 : a) Oui. Les entiers 7 et 12 sont premiers entre eux, donc d’après
le théorème de Bézout, il existe des entiers u et v tels que 7u + 12v = 1. On a alors :
¡ ¢u ¡ ¢v
x = x7u+12v = x7 × x12

Ainsi x s’écrit comme un produit de nombres rationnels. Il est donc rationnel.


Remarque. Le raisonnement précédent ne permet pas de déduire que si x7 et x12 sont entiers,
alors x est forcément entier : en effet, parmi u et v il y a au moins un nombre négatif et
donc le produit (x7 )u × (x12 )v cache en réalité un quotient. Toutefois le résultat est quand
même vrai. On vient de prouver que x est rationnel, et on sait que x7 est entier. Un résultat
du cours prouve alors que x est entier.

b) Non. Cette fois-ci les entiers 9 et 12 ne sont pas premiers entre


√ eux : leur pgcd est
3
3. On chosit x irrationnel tel que x est rationnel (par exemple x = 3). Alors x9 = (x3 )3
3

est bien rationnel, ainsi que x12 = (x3 )4 .

Solution de l’exercice 16 : En écrivant la valuation 2-adique des deux membres de l’équation,


il vient :
[x]v2 (x) = −1
ce qui impose [x] = ±1. Si [x] = 1, on a 1 6 x < 2 et donc x[x] < 2 ce qui est incompatible
avec l’équation. Si [x] = −1, x est négatif et x[x] aussi, ce qui est encore incompatible.

Solution de l’exercice 17 : La condition est équivalente à x + 1 à la fois multiple de 2, de


3, et ainsi de suite jusqu’à 9. Elle est donc également équivalente à x + 1 multiple de
ppcm (2, 3, 4, 5, 6, 7, 8, 9) = 2520. La plus petite solution positive est x = 2519.
n
Solution de l’exercice 18 : On remarque que si d est un diviseur de n alors d
aussi. L’indexa-
tion des diviseurs donne alors di dk+1−i = n. Donc :
k−1
X k−1
X k−1
X µ ¶
2 1 1 1
d= di di+1 = n 6 n2 −
i=1 i=1
di di+1 i=1
di di+1

78
la dernière inégalité étant obtenue car di+1 − di > 1. On en déduit que :
µ ¶ µ ¶
2 1 1 2 1
d6n − =n 1− < n2
d1 dk n

Si n = p est un nombre premier, alors d = p qui divise n2 . Si n est composé, alors k > 2.
Soit p le plus petit diviseur premier de n. On a :
n2
d > dk−1 dk =
p
et donc :
n2
1< <p
d
2
Mais alors nd est un diviseur de n2 strictement inférieur à p, ce qui n’est pas possible.
Les seules solutions sont donc les nombres premiers.

Solution de l’exercice 19 : Soit p un nombre premier impair divisant n. On peut écrire n = pk


et : ¡ ¢¡ ¢
2pk + 1 = 2k + 1 2k(p−1) − 2k(p−2) + · · · + 1
Évidemment 1 < 2k + 1 < 2pk + 1. On a donc obtenu une factorisation non triviale.
On en déduit que le seul diviseur premier de n est 2 et donc que n est une puissance de
2.
n
Commentaire. Les nombres de la forme Fn = 22 + 1 s’appelent les nombres de Fermat.
Fermat avait conjecturé que tous les nombres de cette forme étaient premiers. C’est le cas
de F0 , F1 , F2 , F3 et F4 . Malheureusement, Euler démontra en 1732 que F5 est composé.
Depuis, on n’a pas trouvé d’autres nombres de Fermat premiers.

Solution de l’exercice 20 : Soit n > 1 un entier. On constate que les entiers 1 et n sont
toujours des diviseurs distincts de n et que tous les autres diviseurs de n sont strictement
plus petits que n. On en déduit l’inégalité :

σ(n) 6 1 + n + (d(n) − 2) n

et cette égalité est stricte dès que n admet plus de deux diviseurs, c’est-à-dire dès que n est
composé.
D’autre part, il ne peut y avoir plus de n − 1 entiers premiers avec n dans l’intervalle
[1, n] (en effet, n n’est pas premier avec lui-même). On en déduit que ϕ (n) 6 n − 1. En
combinant avec l’inégalité obtenue précédemment, on arrive à :

σ(n) + ϕ(n) 6 n · d(n)

et cette inégalité est stricte si n est composé.


On en déduit que les seuls n susceptibles de répondre à la question sont les nombres
premiers. On vérifie par ailleurs que si p est premier, d(p) = 2, σ(p) = p + 1 et ϕ(p) = p − 1,
et donc que l’on a bien l’égalité σ(p) + ϕ(p) = p · d(p). Les solutions sont donc exactement
les nombres premiers.

79
Solution de l’exercice
h 21 i : Il est clair que cette somme ne possède qu’un nombre fini de
n+2k
termes non nuls, 2k+1 est nul dès que 2k+1 > n + 2k , donc 2k > n. Cet énoncé se résout
£ ¤
facilement si l’on connaı̂t le lemme : pour tout £ réel¤ x, [2x] = [x] + x + 12 qui se vérifie
1 1 1
immédiatement
£ ¤ : si n 6 x < n + 2 , [x] = n = x + 2 et [2x] = 2n, et si n + 2 6 x < n + 1,
1
[x] = n, x + 2 = n + 1, et [2x] = 2n + 1. Il en résulte que :
· ¸ · ¸ h i h
n + 2k n 1 n n i
= + = −
2k+1 2k+1 2 2k 2k+1

La somme s’écrit donc :


³ h n i´ ³h n i h n i´ ³h n i h n i´
[n] − + − + − + ...
2 2 4 4 8
et par « simplification téléscopique », elle vaut [n], soit n si n est un entier.

Solution de l’exercice 22 : Par définition π (pn ) = n. Ainsi :

n log n = π (pn ) log (π (pn )) (2)

D’après le théorème des nombres premiers, le quotient π (pn ) · logpnpn tend vers 1 quand n tend
vers l’infini (car pn tend vers l’infini) et donc en passant au logarithme, il vient :

log (π (pn )) − log pn + log (log pn ) → 0

puis en divisant tout par log pn , il vient :

log (π (pn )) ∼ log pn

En appliquant une nouvelle fois le théorème des nombres premiers, la formule (2) donne :
pn
n log n ∼ · log pn = pn
log pn

Solution de l’exercice 23 : Soit E un tel ensemble, et a ∈ E quelconque. Alors d’après


l’hypothèse, (a + a)/a = 2 ∈ E. On peut remarquer que le singleton {2} est en fait une
solution du problème. On suppose dorénavant que E contient au moins un autre élément
que 2.
Si 1 est dans E, alors pour tout a ∈ E, (a + 1)/1 = a + 1 ∈ E, donc une récurrence
immédiate montre que E = N∗ , qui est bien une solution.
Dans le cas contraire, soit m le plus petit élément de E autre que 2. Si m était pair,
disons m = 2k avec k > 2, on aurait (2k + 2)/2 = k + 1 ∈ E. Or 2 < k + 1 < 2k, ce qui
contredirait la minimalité de m. Donc m est impair, et m + 2 est aussi dans E, et de même
pour m + 2p pour tout p > 0. Tous les nombres impairs à partir de m sont donc dans E, et
en particulier km pour tout k > 1 impair. Ainsi, (km + m)/m = k + 1 ∈ E, ce qui montrer
que E contient tous les entiers pairs, et en particulier 4. La minimalité de m assure alors que
m = 3 et donc que E contient tous les entiers au moins égaux à 2, ce qui réciproquement
fournit bien une solution au problème.

80
Finalement, il y a trois solutions, qui sont {2}, N∗ et N∗ \ {1}.

Solution de l’exercice 24 : On a la factorisation :


¡ ¢
3n − 1 = 2 3n−1 + 3n−2 + · · · + 1

Si n est impair le facteur entre parenthèses est une somme de n termes impairs et donc est
impair. On en déduit que 3n − 1 ne peut être un multiple de 4 et donc a fortiori de 2n pour
n > 1. Le cas n = 1 convient comme on le vérifie à part.
Supposons
¡ k ¢ ¡nk pair¢ et posons donc n = 2k. La condition se réécrit 22k divise 32k −
1 = 3 − 1 3 + 1 . Les deux nombres 3k − 1 et 3k + 1 sont distants de 2 et donc
leur pgcd
¡ k est kun diviseur
¢ de 2. D’autre part, ces nombres sont toujours pairs et donc
pgcd 3 − 1, 3 + 1 = 2. On en déduit que soit 3k − 1, soit 3k + 1 est divisible par 22k−1 .
Cela implique en toutes circonstances 3k + 1 > 22k−1 ce qui n’est plus vrai pour k > 3. On
vérifie à la main que k = 0, k = 1 et k = 2 sont solutions.
Finalement les seules solutions sont n = 1, n = 2 et n = 4.

Solution de l’exercice 25 : Soit p un nombre premier. On va prouver que les valuations p-


adiques des deux membres de l’égalité à prouver sont toujours égales. Notons α = vp (a),
β = vp (b) et γ = vp (c). Il s’agit de montrer que :

2 inf (α, β, γ) − inf (α, β) − inf (β, γ) − inf (α, γ)


= 2 sup (α, β, γ) − sup (α, β) − sup (β, γ) − sup (α, γ)

Comme les rôles de α, β et γ sont similaires, on peut supposer α 6 β 6 γ et donc ce cas les
deux membres de l’égalité précédentes sont égaux à −β. Ce qui conclut.

Remarque. On ne peut évidemment pas déduire de ce qui précède que les quotients de
l’énoncé sont égaux à 1b . Pourquoi ?

Solution de l’exercice 26 : Pour construire une telle décomposition, on utilise l’algorithme


glouton : on commence par déterminer le plus grand entier, disons ak , tel que Ckak 6 n. Puis,
on recommence avec n et k remplacés respectivement par n − Ckak et k − 1. Comme, par
construction, on a n < Ckak +1 c’est donc que :

n − Ckak < Ck−1


ak

ce qui assure que la suite des ai est bien strictement décroissante

On prouve maintenant l’unicité. Supposons que n possède deux représentations distinctes


associées respectivement aux suites ak , . . . , at et bk , . . . , br . On considère le plus grand indice
pour lequel ces deux suites différent. Quitte à éliminer des termes, on peut supposer que cet
indice est k et que ak > bk . Mais alors :

n 6 Ckbk + Ck−1 1 k k
bk−1 + · · · + Cbk −k+1 < Cbk +1 6 Cak 6 n

ce qui constitue une contradiction.

81
Solution de l’exercice 27 : a) D’après le principe des tiroirs parmi les ai , il y en a deux
consécutifs, qui sont donc premiers entre eux.
b) Écrivons pour tout i, ai = 2αi bi pour un certain nombre impair bi , forcément compris
entre 1 et 2n. D’après le principe des tiroirs, puisqu’il y a n nombres impairs entre 1 et 2n,
il existe i et j tels que bi = bj . On peut supposer αi 6 αj et dans ce cas ai divise aj .

Solution de √ l’exercice 28 : Si n n’est√ pas premier, il admet un diviseur d différent de n et


supérieur à n. Ainsi, σ (n) − n > n. La suite σ (ni ) − ni est constante, disons égale à k.
D’autre part, on prouve par récurrence que ni > i pour tout i. Considérons un i > k 2 . Si ni
n’était pas premier, on aurait :

σ (ni ) − ni > i > k

ce qui est impossible. Ainsi ni est premier et k = 1. On conclut en remarquant que si


σ (n) = n + 1, alors forcément n est un nombre premier.

Solution de l’exercice 29 : Déjà d1 = 1. Si n était impair, tous ses diviseurs seraient impairs
et donc d21 + d22 + d23 + d24 = n serait pair. Ce n’est pas possible. Donc n est pair et d2 = 2.
Si d3 et d4 étaient de même parité, la somme d21 + d22 + d23 + d24 serait impaire, ce qui n’est
pas possible non plus.
Supposons que 4 divise n. Les nombres d3 et d4 sont alors 4 et un nombre premier p à
l’ordre près et on est ramené à l’équation :

21 + p2 = n

Or p divise n, donc p divise 21 et ainsi p vaut 3 ou 7. On vérifie qu’aucun des deux ne fournit
une solution.
Si 4 ne divise pas n, on a d3 = p où p est le plus petit diviseur impair de n, et d4 = 2p
puisque d4 doit être pair. L’équation devient :

5 + 5p2 = n

Donc p divise 5 puis p = 5. Cela conduit à n = 130 qui convient.

Solution de l’exercice 30 : Écrivons an + bn = α + nr et an bn = β + ns pour des entiers α,


β, r et s. Pour tout n, les nombres an et bn sont les racines du polynôme :

X 2 − (α + nr) X + (β + ns)

Le discriminant ∆n = (α + nr)2 − 4 (β + ns) est donc un carré parfait pour tout entier
n > 0. On a l’égalité :
r2 ∆n = (nr2 + αr − 2s)2 + d
où d = −4s2 + 4rsα − 4βr2 est indépendant de n. Si r 6= 0, pour n suffisamment grand, on
a l’inégalité :
(nr2 + αr − 2s − 1)2 < r2 ∆n < (nr2 + αr − 2s + 1)2
Comme r2 ∆n doit être un carré, on doit forcément avoir r2 ∆n = (nr2 + αr − 2s)2 et donc
d = 0. Mais alors, notre trinôme admet toujours pour racine c = rs et donc pour tout n, on
a an = c ou bn = c.

82
Solution de l’exercice 31 : En réduisant au même dénominateur, la condition est équivalente
à `mn divise (` + m + n) (mn + `n + `m). En particulier, ` divise (m + n) mn et par le
lemme de Gauss ` divise m + n.
Pareillement m divise ` + n et n divise ` + m. Les rôles des variables étant symétriques,
on peut supposer que n est le plus grand des trois. Du coup, ` + m 6 2n et la condition de
divisibilité impose ` + m = n ou ` + m = 2n. Dans ce dernier cas, on a forcément ` = m = n
et cette valeur commune est 1 car ils sont premiers entre eux. Le triplet (1, 1, 1) est bien
solution.
Sinon, ` + m = n, et en remplaçant, le nombre :
µ ¶
1 1 1
2 (` + m) + +
` m `+m

doit être entier, ce qui équivaut à :


µ ¶
1 1 2 (` + m)2
2 (` + m) + =
` m lm

est entier. Comme ` et m sont premiers entre eux, on peut supposer ` impair. Alors ` divise
(` + m)2 et donc divise m2 , ce qui n’est possible que si ` = 1. Alors m divise 2, et m = 1 ou
m = 2.
On déduit de ce qui précède que les solutions sont les triplets (1, 1, 1), (1, 1, 2), (1, 2, 3)
et toutes leurs permutations.

Solution de l’exercice 32 : a) Décomposons n > 1 en facteurs premiers :

n = pα1 1 pα2 2 · · · pαd d


Q
Les diviseurs d de n pour lesquels µ (d) 6= 0 s’écrivent d = i∈I pi pour I un certain sous-
ensemble de {1, . . . , d}. La valeur de µ (d) est alors (−1)Card I .
Pour conclure, il faut donc juste voir que l’ensemble {1, . . . , d} possède autant de sous-
ensembles de cardinal pair que de sous-ensembles de cardinal impair. Mais à chaque ensemble
de cardinal pair, on peut associer un ensemble de cardinal impair en lui ajoutant 1 s’il
n’appartient pas à l’ensemble de départ ou en lui otant s’il appartient. Cette association est
bijective : il y a donc autant de sous-ensembles de cardinal pair que de cardinal impair, et
la formule de sommation est prouvée.
b) La somme proposée se réécrit :
X X ³n´
µ f (d0 )
0
d
d|n d |d

Dans la somme précédente le terme f (d0 ) apparaı̂t pour tout entier d tel que d0 |d|n. Ainsi
le coefficient qui restera au final de f (d0 ) est :
X ³n´
µ
0
d
d |d|n

83
En écrivant d = d0 x, cette somme s’écrit encore :
X ³ n ´ X
µ 0 = µ (x)
n
dx n
x| d0 x| d0

la dernière égalité provenant du changement de variable x 7→ dn0 x . D’après a), cette somme
fait toujours 0 sauf si dn0 = 1 (i.e. n = d0 ) auquel cas elle fait 1. On trouve ainsi bien la
formule annoncée.

Solution de l’exercice 33 : Parmi ces 10 entiers, il y en a 5 impairs. Parmi ces 5 impairs, il


y en a au plus 2 qui sont divisibles par 3 (noter que les multiples de 3 sont alternativement
pairs et impairs). Parmi ces mêmes 5, il y en a au plus 1 qui est multiple de 5 et au plus 1
multiple de 7.
Donc il y a au moins un des dix entiers qui n’est divisible ni par 2, ni par 3, ni par 5,
ni par 7. Appelons-le n. Le pgcd de n et de n + k est un diviseur de k, donc si k est non
nul et est compris entre −9 et 9, pgcd (n, n + k) doit être divisible par un nombre premier
strictement inférieur à 10. Mais par construction n n’est divisible par aucun tel nombre
premier. L’entier n convient donc.

Solution de l’exercice 34 : Écrivons :

n = pα1 1 · · · pαd d
m = pβ1 1 · · · pβd d

où les pi sont des nombres premiers deux à deux et les exposants αi et βi sont des entiers
positifs ou nuls.
On a vu dans le cours que le produit des diviseurs de n s’écrit :

pγ11 · · · pγdd

pour :
1
γi = αi (α1 + 1) · · · (αd + 1)
2
L’hypothèse de l’énoncé assure que pour tout i :

αi (α1 + 1) · · · (αd + 1) = βi (β1 + 1) · · · (βd + 1)

et donc il existe un rationnel q, indépendant de i, telle que αi = qβi . Quitte à intervertir m


et n, on peut supposer q > 1. L’hypothèse se réécrit alors :

q (qβ1 + 1) · · · (qβd + 1) = (β1 + 1) · · · (βd + 1)

et on voit directement que si q > 1, le membre de gauche est strictement supérieur à celui
de droite. On a donc q = 1 et m = n.

Solution de l’exercice 35 : D’après les formules classiques, la somme en question vaut :


(2n + m) (m + 1)
2

84
et on est donc ramené à l’équation (2n + m) (m + 1) = 2000. Les nombres 2n + m et m + 1
sont donc des diviseurs associés de 2000. On remarque en outre d’une part que la somme de
ces diviseurs vaut 2n + 2m + 1 est donc doit être impaire, et d’autre part que 1 < m + 1 <
2n + m.
La décomposition en facteurs premiers de 2000 est 2000 = 24 × 53 . En vertu de ce qui
précède les seules possibilités sont :
☞ m + 1 = 5, 2n + m = 400, soit m = 4 et n = 198
☞ m + 1 = 25, 2n + m = 80, soit m = 24 et n = 28
☞ m + 1 = 16, 2n + m = 125, soit m = 15 et n = 55
qui effectivement conviennent.

Solution de l’exercice 36 : La condition de l’énoncé assure que pour tout n, on a pgcd (a2n , an ) =
pgcd (2n, n) = n. Ainsi n divise an et on écrit an = nbn pour un entier bn > 0.
Supposons par l’absurde qu’il existe n tel que bn > 1 et soit p un facteur premier de bn .
Comme p divise an et ap , il divise pgcd (an , ap ) = pgcd (n, p) et donc p divise n. Soit pa
la plus grande puissance de p qui divise n. Alors pa+1 divise an et puis pgcd (an , apa+1 ) =
pgcd (n, pa+1 ) = pa . C’est une contradiction.
Finalement, bn = 1 pour tout n, et donc an = n pour tout n.

Solution de l’exercice 37 : Supposons que n soit composé et que d soit un diviseur strict de
n. On peut donc écrire n = dd0 et :

n
¡ d ¢ ³ d(d0 −1) d(d0 −2)
´
2 −1= 2 −1 2 +2 + ··· + 1

Le facteur 2d − 1 est non trivial, et donc 2n − 1 est composé.

Commentaire. Les nombres de la forme 2p − 1 où p est un nombre premier sont appelés
nombres de Mersenne. On dispose d’algorithmes spécifiques pour tester leur primalité. C’est
pourquoi les plus grands nombres premiers connus à ce jour sont des nombres de Mersenne,
le plus grand étant :
224 036 583 − 1
découvert le 15 mai 2004.

Solution de l’exercice 38 : Notons n le plus petit multiple de 10 qui apparaı̂t parmi les 39
entiers consécutifs. Il est immédiat que tous les entiers compris entre n et n + 29 font partie
des 39 entiers consécutifs.
Notons s la somme des chiffres de n. Les entiers n + 1, . . . , n + 9 sont parmi les 39 entiers
consécutifs et ont pour somme des chiffres respectivement s + 1, . . . , s + 9. D’autre part, si
on suppose que n ne se termine pas par 90, l’entier n + 19 est également parmi les 39 entiers
consécutifs et a pour somme des chiffres s + 10. Or, parmi les sommes s, . . . , s + 10, il y en
a forcément une qui est multiple de 11, ce qui termine ce cas.
Si n se termine par 90, alors n + 10 ne se termine pas par 90 et on peut appliquer le
raisonnement précédemment en remplaçant n par n + 10.

85
En analysant la preuve précédente, il est facile de trouver un contre-exemple lorsqu’il est
question de 38 entiers. Par exemple, on peut prendre les entiers compris entre 999 981 et
1 000 018.

Solution de l’exercice 39 : Soient a un entier impair et b un entier pair tels que b < a.
Notons u et v les racines de x2 − aX − b = 0, avec v < u. On vérifie facilement que u > 1
et −1 < v < 0.
Pour n > 0 entier, on note Sn = un +v n . Alors S1 = a, S2 = a2 +2b et Sn+2 = aSn+1 +bSn .
Puis, par récurrence on a Sn impair pour tout n. Or un = Sn − v n donc :
– si n est impair, on a Sn < un < Sn + 1 et donc [un ] = Sn est impair.
– si n est pair, on a Sn − 1 < un < Sn et donc [un ] = Sn − 1 est pair.
Le nombre u répond ainsi au problème.

Remarque.
√ Si on veut une valeur particulière, on peut choisir a = 3 et b = 2, ce qui fournit
u = 3+2 17 .

Solution de l’exercice 40 : On utilise ici le fait que :

(3n + 1) (3m + 1) = 3 (3mn + m + n) + 1

ce qui incite à poser n0 = 3n + 1 et m0 = 3m + 1. Ce changement de variable effectué,


l’équation fonctionnelle se réécrit :
µ 0 0 ¶ µ 0 ¶ µ 0 ¶ µ 0 ¶ µ 0 ¶
mn −1 m −1 n −1 m −1 n −1
f = 4f f +f +f
3 3 3 3 3
Ainsi, si, pour n un entier de la forme 3k + 1, on pose :
µ ¶
n−1
g (n) = f
3
on obtient l’équation fonctionnelle :

g (xy) = 4g (x) g (y) + g (x) + g (y)

ou encore en utilisant le même type de factorisation :

4g (xy) + 1 = (4g (x) + 1) (4g (y) + 1)

Posons finalement h (x) = 4g (x) + 1 pour obtenir h (xy) = h (x) h (y). La fonction h n’est
définie que sur l’ensemble 3N + 1 et prend ses valeurs dans l’ensemble 4N + 1 et elle réalise
une bijection entre ces deux ensembles. D’autre part, si on arrive à construire une telle
fonction h, on en déduira facilement une fonction f solution en posant :
h (3n + 1) − 1
f (n) =
4
Le problème devient donc de construire h.
Notons pour cela p1 , . . . , pn , . . . la suite des nombres premiers congrus à 1 modulo 3. No-
tons également p01 , . . . , p0n , . . . celle des nombres premiers congrus à 2 modulo 3, q1 , . . . , qn , . . .

86
celle des nombres premiers congrus à 1 modulo 4 et q10 , . . . , qn0 , . . . celle des nombres premiers
congrus à 3 modulo 4. Ces suites sont toutes infinies, par exemple d’après le théorème de
Dirichlet.
Soit n un élément de l’ensemble 3N + 1. Sa décomposition en nombres premiers peut
s’écrire :
α0 α0
n = pα1 1 . . . pαd d p0 1 1 . . . p0 d0d0
puisque tout nombre premier sauf 3 est, de façon exclusive, soit un pi , soit un p0i . Cependant,
3 ne peut apparaı̂tre dans la décomposition de n puisque n étant congru à 1 modulo 3, il
n’est pas divisible par 3. D’autre part, comme n ≡ 1 (mod 3), le nombre α10 + . . . + αd0 doit
être pair. Définissons :
α0 α0
h (n) = q1α1 . . . qdαd q 0 1 1 . . . q 0 d0d0
Comme la somme α10 + . . . + αd0 est paire, on a bien h (n) ≡ 1 (mod 4)
La fonction h ainsi définie vérifie immédiatement h (xy) = h (x) h (y) pous tous entiers
x et y congrus à 1 modulo 3. En outre, elle est bijective, car on peut reconstruire n à partir
de h (n) en effectuant le même procédé à l’envers. Fini !

Solution de l’exercice 41 : Soit α > 0 fixé. Pour j > 2, on pose :


· ¸
αj
nj =
log j
On a log nj ∼ log j. Par ailleurs :

pnj ∼ nj log nj ∼ nj log j ∼ αj


h i pnj
Soit alors mj = logj j . Comme précédemment pmj ∼ j et donc la suite de terme général pmj
est à valeurs dans l’ensemble considéré et a pour limite α.

Solution de l’exercice 42 : On cherche n sous la forme 3d − 2d car la factorisation :


¡ ¢¡ ¢
3kd − 2kd = 3d − 2d 3(k−1)d + 3(k−2)d · 2 + · · · + 2(k−1)d

assure que si d divise n, alors 3d − 2d divise 3n − 2n . De plus si d est un entier composé, alors
il en sera de même de 3d − 2d encore par la même remarque.
On est donc ramené à trouver une infinité d’entiers d divisant n − 1 = 3d − 2d − 1. En
fait, les puissances de 2 conviennent. En effet, si d = 2t , on a directement d divise 2d . Il reste
à voir que d divise 3d − 1.
On montre ce dernier fait par récurrence sur l’entier t. Pour t = 1, on a bien d = 2 divise
d
3 − 1 = 2. Supposons que ce soit vrai pour l’entier t et remarquons que :
t+1
³ t ´³ t ´
32 − 1 = 32 − 1 32 + 1

Le premier facteur est divisible par 2t par hypothèse de récurrence et le second est pair à
l’évidence. Le produit est donc divisible par 2t+1 .
t t
Finalement, on a démontré que tous les entiers n de la forme 32 − 22 sont composés
pour t > 1 et conviennent, ce qui en fait indéniablement une infinité.

87
Remarque. En fait, tout nombre premier n > 5 convient également. C’est une conséquence
directe du petit théorème de Fermat (voir 3.5). De façon plus générale, tous les nombres
de Carmichael (voir 3.5) sont également solutions et on peut prouver qu’il en existe une
infinité. Cependant cette dernière preuve est bien plus difficile que celle que l’on vient de
donner pour résoudre l’exercice.

Solution de l’exercice 43 : Avant de commencer, définissons ϕ (x) = log2 x = log x


log 2
pour tout
réel strictement positif x. Posons également ϕk = ϕ ◦ · · · ◦ ϕ (k fois)
³ partout
´ où c’est défini.
(k) (k)
Construisons par récurrence des réels x tels que la suite xn définie comme dans
h i h i
(k) (k) (k−1)
l’énoncé soit telle que xk soit un entier et un nombre premier et xn = xn pour
tout k < n. Il est évident de construire x(0) , par exemple prenons x(0) = 2.
Faisons l’hérédité. D’après le postulat de Bertrand, il existe au moins un nombre premier
(k) (k)
p compris strictement entre 2xk et 2xk +1 . Posons finalement x(k+1) = ϕk+1 (p) (dont on
(k+1)
vérifie facilement qu’il est bien défini). Directement, xk+1 = p qui est par construction un
(k) (k)
nombre premier. D’autre part 2xk < p < 2xk +1
et donc, en appliquant ϕ1 , on arrive à :
(k) (k)
xk < ϕ1 (p) < xk + 1
h i
(k) (k+1) (k)
et donc [ϕ1 (p)] = xk ou encore xk = xk . De même soit 2 6 m 6 k un entier. En
appliquant ϕm , on obtient :
³ ´ ³ ´
(k) (k)
ϕm−1 xk < ϕm (p) < ϕm−1 xk + 1

D’autre part, on constate qu’il ne peut pas y avoir d’entier entre les deux membres extrêmes
de la précédente inégalité, car sinon il y aurait une puissance de 2 comprise strictement entre
deux entiers consécutifs. On en déduit que :
h ³ ´i
(k)
[ϕm (p)] = ϕm−1 xk

ou encore : h i h i
(k)+1 (k)
xk+1−m = xk−m+1
ce qui achève la récurrence sauvagement. ¡ ¢
Pour conclure, après avoir remarqué que la suite x(k) est décroissante, on note x sa
limite. On laisse au lecteur le soin de rédiger l’argument de continuité montrant que x
convient.

Solution de l’exercice 44 : Oui, c’est possible. Remarquons pour cela que la distance entre
deux points A et B sur le cercle unité de centre O est donné par :
¯ Ã !¯
¯ AOB ¯¯
[
¯
2 ¯sin ¯
¯ 2 ¯

D’autre part, on remarque que si tan θ est rationnel, alors il en est de même de sin (2θ) en
vertu de la formule :
2 tan θ
sin (2θ) =
1 + tan2 θ

88
Finalement, si tan α et tan β sont rationnels, il en est de même de tan (α + β) pourvu qu’il
soit défini, à cause cette fois-ci de la formule :
tan α + tan β
tan (α + β) =
1 − tan α tan β

Considérons donc θ un angle incommensurable avec π et dont la tangente est rationnelle8 .


Plaçons A1 sur le cercle unité puis les points Ai (2 6 i 6 1975) tels que l’angle A\
1 OAi vaille
4 (i − 1) θ. Comme θ est incommensurable avec π, les points Ai sont deux à deux distincts.
En outre, la distance Ai Aj est donnée par la formule :
¯ Ã !¯
¯ A\ OA ¯
¯ i j ¯
2 ¯sin ¯ = 2 |sin (2 (i − j) θ)|
¯ 2 ¯

qui est un nombre rationnel, en application des premières remarques.

Solution de l’exercice 45 : Fixons k. On cherche à prouver qu’il existe une infinité d’entiers
m tels que f (m) = m − k. On demande donc que σ (m − k) 6 m et σ (s) > m pour tout
s > m − k.
Déjà si s > m, comme 1 et s sont diviseurs de s, on aura σ (s) > s√ + 1 > m + 1 > m.
D’autre
√ part, si s est composé, il admet un diviseur strict supérieur à s et donc σ (s) >
s + s.
L’idée est donc de choisir m tel que m − k soit premier et tous les entiers compris entre
m − k + 1 et m soient composés avec m > k 2 + k − 1. En effet, on aura alors σ (m − k) =
m − k + 1 6 m. Si s est compris entre m − k + 1 et m − 1, on aura :
√ √
σ (s) > s + s > m − k + 1 + m − k + 1 > m

Finalement si s > m, on aura forcément σ (s) > m.


Soit p > k 2 un nombre premier et soit N = p (k + 1)! + 1. Le nombre N + 1 est pair, le
nombre N +2 est multiple de 3, et de même pour tout i compris entre 1 et k, le nombre N +i
est multiple de i + 1 et donc composé. Soit m1 − k le plus grand nombre premier inférieur à
N . Ce nombre est supérieur à p et donc à k 2 . Les nombres compris entre m1 − k + 1 et m1 − 1
sont tous composés et d’après ce qui a été vu précédemment m1 vérifie bien m1 −f (m1 ) = k.
On recommence alors la même construction en partant de p un nombre premier supérieur
à m1 . On trouve ainsi une nouvelle solution m2 > m1 . Ainsi de suite, on construit une suite
strictement croissante de solutions, ce qui prouve qu’il en existe une infinité.

Solution de l’exercice 46 : La solution est 243 = 35 .


Définissons pour cela Sn = {3, . . . , n} et montrons dans un premier temps que S243
possède la propriété. En effet, soit X, Y une partition de S243 . On peut supposer que 3 ∈ X.
Si 9 ∈ X, c’est gagné. On peut donc supposer que 9 ∈ Y . Si 81 ∈ Y , c’est gagné, on peut
donc supposer que 81 ∈ X. Si 27 ∈ X, comme 3 × 27 = 81, c’est gagné, on peut donc
supposer que 27 ∈ Y . Enfin 243 = 3 × 81 = 9 × 27, et on gagne dans chacun des cas.
¡ ¢
On peut montrer que l’angle arctan 43 convient, la preuve étant laissée au lecteur qui pourra judicieu-
8
¡ ¢n
sement prouver que le nombre complexe 3+4i
5 ne vaut jamais 1.

89
Pour pouvoir conclure, il suffit de montrer que S242 ne possède pas la propriété, c’est-à-
dire d’exhiber une partition X, Y adéquate (pour n < 242, il suffira de prendre la partition
Sn ∩ X, Sn ∩ Y ). Pour cela nous allons définir l’ensemble P des nombres S-premiers :

P = {4, 8, p, 2p où p est un nombre premier ≥ 3} ∩ S242

Les nombres S-premiers sont en fait les nombres qui ne sont pas le produit de nombres de
S242 .
Comme le plus petit nombre S-premier de S242 est 3, chacun des nombres de S242 peut
s’écrire comme produit d’au plus 4 nombres S-premiers d’au moins une façon (la décompo-
sition en produit de nombres S-premiers n’est pas unique).
On note X la réunion des nombres S-premiers et des nombres de S242 qui peuvent s’écrire
comme produit de 4 nombres S-premiers exactement, Y = S242 \ X.
Un nombre de Y possède au moins 2 facteurs S-premiers, donc un produit de 2 nombres
de Y en possède au moins 4 ; s’il en a plus il est trop grand (car 44 > 242). L’ensemble Y
ne possède donc pas de triplet problématique. Maintenant, un produit de 2 nombres de X
sera 6 242 seulement s’il s’agit d’un produit de 2 nombres S-premiers. En regardant 4, 8,
p1 , 2p2 , on s’aperçoit qu’un produit deux nombres S-premiers ne peut se factoriser en un
produit de 4 nombres S-premiers. L’ensemble X ne contient pas de triplet problématique
non plus.

Solution de l’exercice 47 : Intéressons nous à la première somme et notons-la A. En inversant


les indices de sommation, on constate que :
p−1
" # p−1 µ · 3 ¸¶
X (p − k)3 X −k
2 2
A= = p − 3kp + 3k +
k=1
p k=1
p

Or si x n’est pas un entier, on a [−x] + [x] = −1 et donc, comme p est premier :


p−1
X ¡ ¢ ¡ ¢ p (p − 1) p (p − 1) (2p − 1)
2A = p2 − 3kp + 3k 2 − 1 = p2 − 1 (p − 1) − 3p +3
k=1
2 6

Après simplification, on trouve :


(p − 1) (p − 2) (p + 1)
A=
4
Appelons B la seconde somme. On remarque que les valeurs prises par le terme que l’on
somme
£√ ¤ sont comprises entre 0 et p − 2. Soit n un entier compris entre 0 et p − √ 2. On a
3
kp = n (pour k compris entre√ 1 et (p − 1) (p√− 2) si, et seulement si (puisque 3
kp n’est
pas un entier, p étant premier) kp − 1 < n < kp et puis :
3 3

n3 (n + 1)3
<k<
p p
De plus on a : " #
(n + 1)3
< (p − 1) (p − 2)
p

90
pour tout n compris entre 0 et p − 2. En effet, il suffit de le vérifier pour n = p − 2, ce qui
revient à voir que :
(p + 1)3
< (p − 1) (p − 2) + 1
p
ce que l’on vérifie directement en développant. h i h 3i
3
Ce qui précède assure que dans la somme B il y a exactement (n+1) p
− np termes
égaux à n. et donc :
p−2
Ã" # · ¸! " #
X (n + 1)3 n3 (p − 1)3
B= n − = (p − 1) −A
n=0
p p p
h 3
i
Un calcul analogue à celui qui a déjà été fait prouve que (p−1)
p
= (p − 1) (p − 2), et donc
que :
(p − 1) (p − 2) (3p − 5)
B=
4
3
Remarque. La courbe représentée sur le dessin suivant est celle d’équation y = xp . Dans ces
conditions, on constate que A calcule le nombre de points à coordonnées entières dans la
région en gris foncé, alors que B calcule le nombre de points à coordonnées entières dans la
région gris clair :

y = x3 /p

(p − 1)3 /p

0 1 p−1

Comme la courbe ne passe par aucun point à coordonnées entières, on en déduit que :
" #
(p − 1)3
A + B = (p − 1) = (p − 1)2 (p − 2)
p

91
Solution de l’exercice 48 : a) Par récurrence, nous allons construire pour tout n une progres-
sion arithmétique de longueur n formée exclusivement de puissances parfaites. L’initialisation
pour n = 1 (ou n = 2) est immédiate.
Supposons donc que l’on dispose d’un entier x et d’une raison r tels que, pour tout
k ∈ {0, . . . , n − 1}, on ait x + kr = abkk pour des entiers ak et bk avec bk > 1. Notons b le
ppcm de tous les bk , définissons x0 = (x + nr)b x et r0 = (x + nr)b r. Soit k ∈ {0, . . . , n − 1}.
Alors :
x0 + kr0 = (x + nr)b (x + kr) = (x + nr)b abkk
et comme par définition bk divise b, le nombre x0 + k 0 r est bien une puissance parfaite.
D’autre part x0 + nr0 = (x + nr)b x + n (x + nr)b r = (x + nr)b+1 et est également une
puissance parfaite. Cela conclut l’hérédité et la récurrence.

b) Montrons par l’absurde que ce n’est pas possible et donc qu’il existe un entier x et
une raison r tels que x + nr soit une puissance parfaite pour tout entier positif n. Soit
d = pgcd (x, r). Il existe des entiers y et s premiers entre eux tels que x = dy et r = ds.
Pour tout n, on a alors x + nr = d (y + ns).
D’après le théorème de Dirichlet, il existe un nombre premier p > d de la forme y + ns.
Mais alors vp (x + nr) = 1 et x + nr ne peut pas être une puissance parfaite. Contradiction.

Solution de l’exercice 49 : Supposons qu’il existe des rationnels x et y vérifiant l’équation.


Soit p un nombre premier. Notons α = vp (x) et β = vp (y) et supposons que l’un des deux
nombres soient non nuls. La somme
1 1
x+y+ +
x y
doit être entière donc de valuation positive ou nulle, mais la valuation d’un des termes est
strictement négative. La seule possibilité est d’avoir α = ±β, et ce pour tout p.
Ceci prouve qu’il existe des rationnels non nuls a et b tels que x = ab et y = ab . Écrivons
a = pq et b = st pour des entiers non nuls p, q, s et t tels que pgcd (p, q) = pgcd (s, t) = 1.
L’équation devient :
ps qt pt qs
+ + + = 3n
qt ps qs pt
soit après simplification : ¡ ¢¡ ¢
p2 + q 2 s2 + t2 = 3npqst
Ainsi 3 divise l’un des deux facteurs du membre de gauche, par exemple p2 + q 2 . On voit
directement que cela implique que 3 divise p et 3 divise q, mais cela est impossible puisque
p et q étaient supposés premiers entre eux.
Finalement, il n’y a bien aucune solution.

Solution de l’exercice 50 : On va prouver que si n > 0, alors il y a exactement :


1 ¡ ¡ 2¢ ¢
d n − 2d (n) + 1
2
diviseurs positifs de n2 qui sont inférieurs à n et ne divisent pas n.

92
n2
En effet, si d est un diviseur de n2 , alors d0 = d
en est aussi un et d < n si et seulement
si d0 > n. Il y a donc exactement :
1 ¡ ¡ 2¢ ¢
d n −1
2
diviseurs de n2 qui sont strictement inférieur à n (et autant qui sont strictement supérieurs).
Parmi ces diviseurs, il y a d (n) − 1 diviseurs de n qu’il faut éliminer pour notre dénombre-
ment. La formule annoncée en découle.
Dans le cas particulier de n = 213 × 311 × 57 , on calcule :
¡ ¢
d n2 = 27 × 23 × 15 = 9315
d (n) = 14 × 12 × 8 = 1344

Le nombre cherché est 3314.

Solution de l’exercice 51 : Notons d = pgcd (a, b) et soient a0 et b0 des entiers tels que a = da0
et b = db0 . L’équation devient :
c (a0 + b0 ) = da0 b0
d’où on déduit dans un premier temps que c divise da0 b0 . Comme a, b et c sont premiers
entre eux dans leur ensemble, c est premier avec d et donc par le lemme de Gauss c divise
a0 b0 .
D’autre part, on déduit également que a0 divise c (a0 + b0 ), mais comme a0 est premier
avec b0 il est premier avec a0 + b0 et donc a0 divise c. De même, on trouve que b0 divise c.
Encore une fois parce que a0 et b0 sont premiers entre eux, cela implique a0 b0 divise c.
Les entiers a0 b0 et c sont positifs et se divisent mutuellement, donc a0 b0 = c et a0 + b0 = d.
En remultipliant par d, on trouve finalement a + b = d2 , ce qui conclut.
¡ ¢
Solution de l’exercice 52 : a) Supposons que n soit de la forme 2k−1 2k − 1 avec 2k − 1
premier. On dispose de la décomposition en facteurs premiers de n et donc de l’expression
de σ (n) :
¡ ¢2
2k − 1 2k − 1 − 1 ¡ k ¢¡ ¢
σ (n) = · k = 2 − 1 2k − 1 + 1 = 2n
2 − 1 (2 − 1) − 1
Les nombres de cette forme sont donc bien parfaits et pairs (car la condition 2k − 1 premier
implique k > 2).
Réciproquement, supposons que n soit un nombre parfait pair. Alors n s’écrit n = 2k−1 A
pour un certain entier k > 2 et un certain entier positif
¡ A¢impair. Par exemple, en décom-
posant A en facteurs premiers, on arrive à σ (n) = 2k − 1 σ (A) ce qui conduit, puisque n
est supposé parfait, à : ¡ ¢
2k A = 2k − 1 σ (A) (3)
Comme 2k −1 est impair (et donc premier k k
¡ k avec¢ 2 ), le lemme de Gauss assure que 2 −1 divise
A et donc que l’on peut écrire A = a 2 − 1 pour un certain entier positif a. L’égalité (3)
assure σ (A) = A + a. Puisque k > 2, les entiers A et a sont distincts et tous deux diviseurs
de A. Ainsi, comme 1 est également diviseur de A, on doit avoir a = 1 pour que l’égalité

93
σ (A) = A + a soit satisfaite. Cela implique A = 2k − 1 et σ (A) = A + 1 et donc A premier.
Finalement, n s’écrit bien sous la forme annoncée.

b) Supposons que n soit impair et ait au plus deux facteurs premiers. Alors on peut
écrire n = pa q b pour des nombres premiers impairs distincts p < q et des entiers a et b
positifs ou nuls. On a :

σ (n) pa+1 − 1 q b+1 − 1 pa+1 q b+1 p q


= a b
< a b
=
n p (p − 1) q (q − 1) p (p − 1) q (q − 1) p−1q−1
σ(n) 3
D’autre part, on a forcément p > 3 et q > 5. Mais alors on obtient n
< 2
× 54 < 2 et donc
n ne peut pas être parfait.

Solution de l’exercice 53 : a) Notons A = pgcd (a, a + 5) et B = pgcd (b, b + 5). Ce sont


des diviseurs de 5, ils sont donc égaux soit à 1, soit à 5.
Si A = B, en vertu de l’égalité ppcm (x, y) · pgcd (x, y) = xy, l’équation devient
a (a + 5) = b (b + 5), ce qui implique a = b puisque a et b sont positifs.
Par symétrie, on peut supposer A = 1 et B = 5 pour le cas restant. Dans ce cas, b est
un multiple de 5, disons b = 5b0 et l’équation devient : a (a + 5) = b0 (5b0 + 5) = 5b0 (b0 + 1).
On en déduit que 5 divise a, ce qui contredit A = 1.
b) La réponse est non. Supposons, par l’absurde qu’il existe a, b et c solutions. Notons
m = ppcm (a, b) = ppcm (a + c, b + c). Soit p un nombre premier diviseur commun de a et
de b. Alors p divise m et donc p divise a + c ou b + c. Dans un cas, comme dans l’autre, il
divise c. En posant a0 = ap , b0 = pb et c0 = pc , on obtient un nouveau triplet solution. En itérant
le procédé, on peut supposer que a et b sont premiers entre eux dès le commencement.
Dans ces conditions, m = ab. Soit p un diviseur premier de a + c et b + c. Alors p divise
m, et donc il divise soit a, soit b. Il divise donc c et donc a et b. Cela est impossible car a
et b sont premiers entre eux. Du coup, a + c et b + c sont également premiers entre eux et
l’équation devient :
ab = (a + c) (b + c)
qui n’a manifestement pas de solution en entiers strictement positifs.

Solution de l’exercice 54 : Soit p un nombre premier fixé, et α, β, γ les valuations p-adiques


de a, b et c. On veut montrer que α + β + γ est un multiple de 3.
Pour cela, remarquons que :
µ ¶
a b c
vp + + >0 (4)
b c a

et que l’on a la relation :


(α − β) + (β − γ) + (γ − α) = 0
Il en résulte que l’un au moins des trois entiers α − β, β − γ, γ − α est négatif. Si aucun
n’est strictement négatif, c’est que α = β = γ et α + β + γ est bien muliple de 3. Sinon, pour
que (4) soit satisfaite, le minimum des trois valeurs doit être atteint au moins deux fois. Par
exemple, on a α − β = β − γ, ce qui assure que α + β + γ = 3β est encore multiple de 3.

94
Solution de l’exercice 55 : Montrons dans un premier temps qu’il existe des entiers x, y, z
et t tels que a = xy, b = zt, c = xz et d = yt. Définissons x = pgcd (a, c), y = xa , z = xc et
t = zb = yd comme on le vérifie immédiatement. Les quatre égalités voulues sont immédiates,
et les nombres x, y et z sont manifestement des entiers. Le seul point qui n’est pas immédiat
est de prouver que t est entier. Pour cela, considérons p un nombre premier. On a :

vp (t) = vp (b) − vp (z) = vp (b) − vp (c) + min (vp (a) , vp (c))


= vp (d) − vp (y) = vp (d) − vp (a) + min (vp (a) , vp (c))

Or min (vp (a) , vp (c)) vaut soit vp (a) et alors la première ligne prouve que vp (t) > 0, soit
vp (c) et c’est alors la deuxième ligne qui permet de conclure.
Forts de cette remarque, nous pouvons terminer l’exercice. On écrit simplement pour
cela : ¡ ¢¡ ¢
ak + bk + ck + dk = xk y k + z k tk + xk z k + y k tk = xk + tk z k + y k
et les deux facteurs précédents sont supérieurs ou égaux à 2.

Solution de l’exercice 56 : Supposons que le couple (m, n) soit solution. Alors le quotient
n3 +1
mn−1
est entier, et donc il en est de même de :

n3 + 1 (mn)3 − 1 m3 + 1
m3 = +
mn − 1 mn − 1 mn − 1
m3 + 1
= (mn)2 + (mn) + 1 +
mn − 1
On en déduit que (n, m) est également solution. Les rôles des deux variables sont donc
symétriques et on peut ne chercher que les solutions pour lesquelles m > n.
n3 +1
Soit K = mn−1 > 1. On a n3 + 1 = K (mn − 1) et donc n divise K + 1, puis K = nx − 1
pour un certain entier x > 1. Mais alors pour n > 1 :

n3 + 1 1
nx − 1 = K 6 2
=n+
n −1 n−1
Si de plus n > 3, il vient n (x − 1) < 2 et donc n (x − 1) = 0 (puisque c’est un multiple de
n) puis x = 1. L’équation devient :

n3 + 1
n−1=
mn − 1
d’où :
2
m=n+1+
n−1
ce qui implique que n − 1 divise 2. Comme n > 3, la seule possibilité est n − 1 = 2, soit
n = 3. Dans ce cas, m = 5 donne une solution.
Reste à regarder les cas n = 1 et n = 2. Pour n = 1, on doit avoir m − 1 divise 2 et donc
m = 2 ou m = 3. Pour n = 2, il vient 2m − 1 divise 9, puis m = 2 et m = 5 (car on suppose
toujours m > n = 2).
Finalement, les solutions sont (2, 1), (3, 1), (2, 2), (5, 2), (5, 3), (1, 2), (1, 3), (2, 5) et (3, 5).

95
Solution de l’exercice 57 : Soit r un nombre rationnel compris strictement entre 12 et 2.
Posons r = m n
. On remarque alors que a = 2m − n, b = m + n, c = 2n − m et d = m + n
sont des entiers strictement positifs qui conviennent.
Pour le cas général, quitte à considérer l’inverse, on peut supposer r > 1. Il existe alors
n > 0 tel que : µ ¶3n µ ¶3n+3
1 3 1 3
<r6
2 2 2 2
¡ 2 ¢3n
Ainsi r 3 est un rationnel strictement compris entre 12 et 2. On utilise le résultat précé-
dent : µ ¶3n
2 a3 + b3
r = 3
3 c + d3
pour certains entiers strictement positifs a, b, c et d. On en déduit :
(3n a)3 + (3n b)3
r=
(2n c)3 + (2n d)3
ce qui assure la conclusion.

Solution de l’exercice 58 : Soit r = p1 /q1 le rationnel que l’on veut écrire comme somme
d’inverses d’entiers. On va voir que « l’algorithme glouton » consistant à soustraire à r la
plus grande fraction 1/n possible fournit bien une solution au problème, et se termine en au
plus p1 étapes.
Plus précisément, construisons par récurrence les suites (pk ), (qk ) et (nk ) de la façon
suivante : en supposant pk > 0 et qk construits, on note nk le plus petit entier (nécessairement
supérieur ou égal à 2) tel que 1/nk 6 pk /qk , et pk+1 et qk+1 sont le numérateur et le
dénominateur de la fraction :
pk 1

qk nk
écrite sous forme irréductible. En particulier, pk+1 est un diviseur de pk nk − qk . Or on a :
1 pk 1
6 <
nk qk nk − 1
ce qui donne 0 6 pk nk − qk < pk . Il en résulte que 0 6 pk+1 < pk , et la suite (pk ) est
donc strictement décroissante, et atteint en particulier la valeur 0 (car sinon l’algorithme
continue).
Lorsque pk+1 = 0, la construction s’arrête, et l’on a :
1 1 1
r= + + ··· +
n1 n2 nk
Reste à voir que les ni sont tous distincts. En fait, ni+1 > ni . En effet, si l’on avait
ni+1 6 ni , on aurait 2/ni 6 pi /qi . Mais 1/(ni − 1) 6 2/ni , et donc ni − 1 aurait dû être
choisi à la place de ni .

Remarque. La décomposition précédente n’est pas unique par exemple grâce à l’égalité :
1 1 1
= +
n n + 1 n (n + 1)

96
Solution de l’exercice 59 : Traitons d’abord le cas où 1 ∈ S. Alors il existe un entier n tel que
p = n2 + 1. Cela implique n > 2 et n pair. On pose a = p − (n − 1)2 = 2n et b = p − 1 = n2
qui conviennent.
Maintenant si 1 6∈ S. Alors il existe n > 2 tel que :

n2 + 1 < p < (n + 1)2 − 1 = n (n + 2)

l’inégalité de droite étant stricte, car p est premier. On pose a = p − n2 ∈ S. On a a − n =


p − n (n + 1) qui est non nul puisque p est premier et strictement inférieur à n en valeur
absolue. On pose b = p − (a − n)2 ∈ S. On vérifie que b = a (1 + 2n − a). On a a < 2n et
donc 1 + 2n − a > 1 puis a < b. Le couple (a, b) convient.

Solution de l’exercice 60 : Pour n > 0, on considère le point An de coordonnées (n, n2 ). La


distance entre An et Am est donnée par la formule :
q q
(n − m) + (n − m ) = |n − m| (n + m)2 + 1
2 2 2 2

Si n 6= m, on a |n − m| = 6 0, et n + m > 1 ce qui implique que (n + m)2 + 1 ne peut pas être


un carré. La racine carrée écrite précédemment n’est pas un entier, c’est donc forcément un
nombre irrationnel : en effet, s’il existe x ∈ Q tel que x2 = n pour un certain entier n fixé,
on a 2vp (x) = vp (n) > 0 pour tout nombre premier p et donc x est entier.
Il reste à voir que si n, m et p sont trois entiers distincts, alors l’aire du triangle dont les
sommets sont An , Am et Ap est rationnelle et non nulle. L’aire d’un tel triangle se calcule
via :
1 ³−−−→ −−−→´ 1 £¡ 2 ¢ ¡ ¢ ¤
det An Am , An Ap = m − n2 (p − n) − p2 − n2 (p − n)
2 2
1
= (m − n) (p − n) (m − p)
2
qui est bien non nul et rationnel.

Solution de l’exercice 61 : Soit
√ n un entier. Notons m = [ 3
n]. Dire que n est divisible par
tous les entiers inférieurs à n revient à dire que n est divisible par ppcm (1, 2, . . . , m).
3

Deux nombres consécutifs sont forcément premiers entre eux. Ainsi le ppcm (m, m − 1) =
m (m − 1) et ppcm (m − 2, m − 3) = (m − 2) (m − 3). Un nombre premier p > 5 ne peut
diviser chacun de ces deux produits. Il en est de même de 4 et de 9. Ainsi le pgcd de
m (m − 1) et de (m − 2) (m − 3) est un diviseur de 6. On en déduit que :

ppcm (m, m − 1, m − 2, m − 3) = ppcm (m (m − 1) , (m − 2) (m − 3))

est un multiple de :
m (m − 1) (m − 2) (m − 3)
6
Ainsi n doit également en être un, et on obtient une inégalité :

m (m − 1) (m − 2) (m − 3)
(m + 1)3 > n >
6

97
que l’on peut réécrire :
µ ¶µ ¶µ ¶
2 3 4
m66 1+ 1+ 1+
m−1 m−2 m−3

ce qui n’est plus vérifié pour m > 13, et ne nous laisse qu’un nombre fini de vérifications à
faire.
En réalité m ne peut valoir 13 : on calcule pgcd (1, . . . , 13) = 360 360 et donc une
solution n doit être multiple de 360 360 inférieur à 143 = 2744. C’est impossible. De même
on élimine les valeurs de m comprises entre 8 et 12.
Pour m = 7, on trouve ppcm (1, . . . , 7) = 420 et il existe bien (au moins) un multiple de
420 inférieur à 83 = 512. Le plus grand est 420, c’est la réponse au problème !

Remarque. On peut en réalité légèrement ruser sur la fin pour éviter quelques calculs... mais
bon.

Solution de l’exercice 62 : Tout d’abord, il est clair que le quotient :


abc − 1
q=
(a − 1) (b − 1) (c − 1)

ne peut valoir 1. Comme c’est un entier, il est au moins égal à 2.


x
Remarquons ensuite que pour tout réel x > 5, on a x − 1 > √ 3 . Ainsi pour a > 5, on a :
2

2 (a − 1) (b − 1) (c − 1) > abc > abc − 1

et donc abc − 1 ne peut être divisible par (a − 1) (b − 1) (c − 1). On a donc prouvé que
2 6 a 6 4.
Supposons q = 2. Alors abc est impair et d’après ce qui précède, a qui doit être impair
ne peut valoir que 3. L’équation se réécrit 4 (b − 1) (c − 1) = 3bc − 1, soit bc + 5 = 4b + 4c,
qui se réecrit (b − 4) (c − 4) = 11 qui conduit directement à la solution b − 4 = 1, c − 4 = 11,
soit b = 5 et c = 15.
Supposons maintenant q > 3. Si a = 2, il vient q (bc − b − c + 1) = 2bc − 1, soit
(q − 2) bc + (q + 1) = qb + qc qui se factorise en :

[(q − 2) b − q] [(q − 2) c − q] = q + 2 (5)

On a b > 3, et si c > 4, on a (q − 2) b − q > 2q − 6 et (q − 2) c − q > 3q − 8. Ainsi le pro-


duit [(q − 2) b − q] [(q − 2) c − q] > (2q − 6) (3q − 8) et ce dernier minorant est strictement
supérieur à q + 2 dès que q > 4. Il ne reste plus que la possibilité q = 3. Dans ce cas, il vient
(b − 3) (c − 3) = 5 qui conduit à b = 4 et c = 8.
Si a = 3, on a 2q (bc − b − c + 1) = 3bc − 1, soit (2q − 3) bc + (2q + 1) = 2qb + 2qc.
Comme b > 4, on a (2q − 3) bc > (8q − 12) c > 4qc > 2qc + 2qb, il n’y a pas de solution.
Si a = 4, on écrit (3q − 4) bc + (3q + 1) = 3qb + 3qc. Mais b > 4, et donc (3q − 4) bc >
(12q − 16) c > 6qc > 3qb + 3qc, et il n’y a pas non plus de solution.
En conclusion, les seules solutions sont a = 2, b = 4, c = 8 et a = 3, b = 5, c = 15.

98
Solution de l’exercice 63 : Soit A = M 2 + M , et a, b, c, d quatre éléments de S tels que
ab = cd. On pose a = A + x, b = A + y, c = A + z et d = A + t de sorte que x, y, z et t sont
en valeur absolue inférieurs ou égaux à M . L’égalité imposée s’écrit alors :

(x + y)A + xy = (z + t)A + zt

Si l’on a x + y = z + t, il vient xy = zt et donc les paires {x, y} et {z, t} sont égales, et c’est
terminé. Sinon, on écrit :

2M 2 > |xy − zt| = |(x + y) − (z + t)| · |A| > |(x + y) − (z + t)| · M 2

ce qui impose que les sommes x + y et z + t diffèrent de 1 exactement, par exemple x + y =


z + t + 1, et par suite xy = zt − A. Alors x et y sont racines du trinôme :

X 2 − (z + t + 1)X + (zt − A)

Ls racines sont : √
z+t+1± ∆
2
avec :
∆ = (z + t + 1)2 − 4zt + 4A > 4A = (z − t + 1)2 + 4d > 4M 2
En fait, il ne peut y avoir égalité car sinon d = M et z − t + 1 = 0, ce qui implique t = −M
et z = −M − 1, ce qui est interdit. Ainsi ∆ > 4M 2 .
Donc si z + t + 1 > 0, la racine correspondant à +∆ est strictement supérieure à M , et si
z + t + 1 6 0, la racine correspondant à −∆ est strictement inférieure à −M . Dans tous les
cas, on obtient que x ou y dépasse M en valeur absolue, ce qui est absurde. D’où le résultat.

Solution de l’exercice 64 : Soit r le plus grand entier naturel tel que 2r 6 n. Alors pour tout
k compris entre 1 et n, v2 (k) 6 r avec égalité si et seulement si k = 2r . On en déduit que :
µ ¶
1 1
v2 (Hn ) = v2 1 + + · · · + = −r
2 n

et donc Hn n’est pas entier si r > 1, ce qui se produit dès que n > 2.
L’entier m > 1 étant fixé, notons d’autre part un = Hm+n − Hm . Notons an = v2 (un ).
Nous allons montrer par récurrence que l’on a :

an = − max v2 (m + k)
16k6n
¡ 1 ¢
C’est vrai pour a1 = v2 m+1 . Supposons alors le résultat au rang n. Soit v = v2 (m + n + 1).
Si v 6= −an , on a an+1 = inf(an , −v) et le résultat s’ensuit. Sinon, cela signifierait qu’il
existe un entier k compris entre m + 1 et m + n et de valuation v. Mais alors 2v divise k
et m + n + 1, et donc m + n + 1 > k + 2v . Mais v2 (k + 2v ) > v + 1 (car k est de la forme
2v (2k 0 + 1)), ce qui contredit le fait que an = −v.
Finalement, on a montré que la différence Hm+n − Hm n’était entière que pour m = 0 et
n = 1.

99
Solution de l’exercice 65 : On a d − a > c − b d’où en élevant au carré a2 − 2ad + d2 > c2 −
2bc + b2 . En ajoutant 4ad = 4bc des deux côtés de l’inégalité, on obtient (a + d)2 > (b + c)2 ,
soit encore :
a+d>b+c+1
√ √
Si d − a < 1, on obtiendrait :
√ √
a + d < 1 + 2 ad = 1 + 2 bc 6 1 + b + c 6 a + d
√ √
ce qui est absurde. On a donc forcément d − a = 1. On a alors :
√ √
a + d = 1 + 2 ad = 1 + 2 bc 6 1 + b + c 6 a + d

et donc toutes les inégalités précédentes sont des égalités. Ainsi b = c et donc ad = b2 . Notons
p un diviseur premier commun à a et à d. Forcément p divise b et p divise a + d = 2b + 1, ce
qui est impossible. Ainsi a et d sont premiers entre eux. Comme leur produit est un carré,
a est également un carré.

Solution de l’exercice 66 : Montrons dans un premier temps que 2abc − ab − bc − ca ne peut


s’écrire sous la forme xbc + yca + zab. En effet supposons que ce soit le cas. On aurait alors :

(x + 1) bc = 2abc − ab − ca − yca − zab

et donc comme a, b et c sont premiers entre eux deux à deux, a devrait diviser x + 1.
Ce dernier nombre ne peut être nul, puisque x doit être positif et donc on devrait avoir
l’inégalité x > a − 1. De même, on prouve y > b − 1 et z > c − 1, mais alors :

xbc + yca + zab > (a − 1) bc + (b − 1) ac + (c − 1) ab


= 3abc − bc − ac − ab > 2abc − ab − bc − ca

d’où la contradiction.
Il faut désormais montrer que tout nombre strictement supérieur à 2abc − ab − bc − ca
peut s’écrire sous la forme voulue. Puisque c et ab sont premiers entre eux, on sait que tout
nombre strictement supérieur à abc − ab − c, disons d, peut s’écrire sous la forme tc + zab,
pour t et z des entiers positifs. Maintenant, le nombre ab − a − b + 1 + t est strictement
supérieur à ab − a − b et donc en appliquant à nouveau le même résultat, on voit que l’on
peut écrire :
ab − a − b + 1 + t = xb + ya
pour certains entiers x et y encore positifs. Calculons :

xbc + yac + zab = (abc − ac − bc + c) + (tc + zab) = abc − ac − bc + c + d

On a bien prouvé que tout nombre de la forme abc−ac−bc+c+d pour d > abc−ab−c, c’est-
à-dire tout nombre strictement supérieur à abc−ab−bc+c+abc−ab−c = 2abc−ab−bc−ca,
s’écrit de la forme voulue.

Solution de l’exercice 67 : On raisonne par l’absurde et on suppose qu’il existe α, β et γ


strictement positifs tels que S (α), S (β) et S (γ) forment une partition de N? . Les réels α,
β et γ sont strictement supérieurs à 1, car si 0 < x < 1, on a S (x) = N.

100
On peut supposer 1 ∈ S (α) et donc α < 2. Il existe alors un entier m > 2 :
1 1
1+ 6α<1+
m m−1
Alors [nα] = n pour tout n 6 m − 1 et [mα] = m + 1. Ainsi m est le plus petit entier
n’appartenant pas à S (α). De plus deux éléments consécutifs n’appartenant pas à S (α)
diffèrent de m ou de m + 1.
Sans perte de généralité, on peut supposer m ∈ S (β), ce qui assure que [β] = m. Ainsi
deux éléments consécutifs de S (β) diffèrent d’au plus m + 1. Soit n = [γ]. Il est élément de
S (γ) et la minimalité de m assure qu’il existe x et y deux éléments consécutifs de S (β) tels
que x < n < y. La différence y − x ne peut excéder m + 1. Mais les différences n − x et y − n
sont au moins égales à m. Ceci constitue une contradiction.

Solution de l’exercice 68 : Soit a2 un élément de cet hypothétique ensemble infini X. Alors,


pour tout autre élément x2 de X, il existe un entier y > 0 tel que a2 + x2 = y 2 , c.à.d.
(y − x)(y + x) = a2 ce qui implique x = p−q 2
pour p et q des diviseurs de a2 (en l’occurrence
y − x et y + x). Bref, tout élément de X différent de a2 s’écrit comme la demi-différence de
deux diviseurs de a2 . Comme il n’y a qu’un nombre fini de tels diviseurs, il ne peut donc y
avoir qu’un nombre fini de tels x. D’où la conclusion.

Solution de l’exercice 69 : On remarque immédiatement que 1992 est un multiple de 83, en


l’occurrence 1992 = 83 × 24. Posons x = 1083 . Le nombre dont on veut calculer la partie
entière est :
à µ ¶2 µ ¶22 µ ¶23 ¡ 7 ¢24 !
x23 7 7 7 7
= x23 1 − + − ··· + − + x 7
x+7 x x x x 1+ x
724
= x23 − 7x22 + 72 x21 − · · · − 723 +
x+7
Le dernier terme est compris strictement entre 0 et 1, donc la partie entière cherchée est
donnée par :
x23 − 7x22 + 72 x21 − · · · − 723
Tous les termes sont des multiples de 10 sauf le dernier. En posant la soustraction, on voit
que le chiffre cherché est le complémentaire à 10 du chiffre des unités de 723 .
Le dernier de 7 est 7, celui de 72 est 9, celui de 73 est 3, puis 1 puis à nouveau 7. Comme
pour calculer le dernier chiffre d’un produit, on multiplie les derniers chiffres des facteurs,
on voit que la suite des derniers chiffres des puissances de 7 est périodique de période 4.
Ainsi 723 se termine par 3 et la réponse à la question est 7.

Solution de l’exercice 70 : Supposons, par l’absurde, qu’il y ait une solution. Tout d’abord
ak ne peut être pair. En effet, si c’était le cas, v2 (ak !) > v2 (ai !) pour tout i < k et donc :
µ ¶ µ ¶
1 1 1
−n = v2 + ··· + = v2 = −v2 (ak !)
a1 ! ak ! ak !
D’autre part : µ ¶ µ ¶
1 1 1
−n = v5 + ··· + > v5 = −v5 (ak !)
a1 ! ak ! ak !

101
On en déduit que v5 (ak !) > v2 (ak !), ce qui est impossible puisque ak > 2. Ce raisonnement
fonctionne encore pour ak impair et ak−1 < ak − 1, ainsi que dans le cas k = 1.
Donc ak doit être impair et ak−1 = ak − 1. En réduisant au même dénominateur, en
l’occurrence ak !, on trouve que ak ! = c10n où :
c = 1 + ak + ak (ak − 1) · · · (ak−2 + 1) + · · · + ak (ak − 1) · · · (a1 + 1)
On remarque que ak et c sont premiers entre eux. Or ak divise 10n c donc il divise 10n .
Comme il est impair, il existe α positif ou nul tel que ak = 5α . On a ak−1 = ak − 1 = 5α − 1
est un multiple de 4. Ainsi v2 (c) = 1. On en déduit que v2 (ak !) = v2 (c10n ) = 1 + n. De
même v5 (ak !) = n. Ainsi :
v2 (ak !) = 1 + v5 (ak !)
ce qui est impossible pour ak > 4. On sait que ak est une puissance de 5, donc ak = 1 qui
est exclus par n > 0.

Solution de l’exercice 71 : Si la décomposition en facteurs premiers de n est :


n = pα1 1 · · · pαd d
le quotient qui nous intéresse vaut :
d (n2 ) 2α1 + 1 2αd + 1
= ···
d (n) α1 + 1 αd + 1
Il s’agit donc simplement de voir quels nombres entiers peuvent s’écrire de cette façon.
On constate tout d’abord que le numérateur est forcément impair, il en est donc de même
du quotient.
Réciproquement, nous allons prouver par récurrence que tout nombre impair peut être
obtenu. Pour les premières valeurs, on a :
5 9 5 5 9
3= × ; 5= × ×
3 5 3 3 5
Pour l’hérédité, notons m le nombre impair que l’on cherche à obtenir. Notons k la valuation
2-adique de m + 1 et posons :
¡ k ¢
2 −1 m−1
a1 =
¡ k 2¢
2 −1 m−1
a2 =
4
..
. ¡ ¢
2k − 1 m − 1
ak =
2k
k
Tous ces nombres sont des entiers, puisque ak +1 = 2 2−1k (m + 1) l’est. De plus a2kk −1
+1
= m+1
2k
<
m+1
m. D’après la définition de k, 2k est impair et il relève donc de l’hypothèse de récurrence
et on conclut en remarquant :
2a1 + 1 2ak + 1 ak + 1
m= ··· × k
a1 + 1 ak + 1 2 −1

102
Solution de l’exercice 72 : L’astuce est d’utiliser judicieusement la condition ad = bc, en
écrivant : ¡ ¢ ¡ ¢
(b − a) (b + a) = b2 − a2 = b2 + bc − a2 + ad = 2m b − 2k a
D’autre part, on a k > m car a + d > b + c puisque a (a + d − b − c) = (a − b) (a − c) > 0.
Ainsi (b − a) (b + a) est multiple de 2m . Comme b est impair, (b − a) + (b + a) = 2b n’est
pas multiple de 4, et donc il est impossible que b − a et b + a soient tous les deux multiples
de 4. On en déduit que l’un d’entre eux est divisible par 2m−1 . Mais :
b+c
0<b−a<b< = 2m−1
2
c’est donc (a + b) qui est un multiple de 2m−1 . On a une autre inégalité :

b + a < b + c = 2m

et donc b + a = 2m−1 .
Si d est un diviseur commun de a et de b, d doit diviser a + b et donc être une puissance
de 2. Puisque a et b sont impairs, il vient d = 1, c’est-à-dire que a et b sont premiers
entre eux. De même, on prouve que a et c sont premiers entre eux en remarquant que
c − a = (c + b) − (b + a) = 2m − 2m−1 = 2m−1 .
Finalement a divise bc et est premier avec b et c. Donc a = 1.

5.2 Exercices de « Division euclidienne et conséquences »


b
Solution de l’exercice 73 : Le nombre 1010 1 . . . 1 0101 (avec 2n + 1 chiffres 1 au milieu) ne
possède que des 1 à part aux positions 1, 3, 2n + 5 et 2n + 7 (le chiffre de droite étant en
position 0). Ainsi on a :

b b2n+9 − 1
1010 1 . . . 1 0101 = − b − b3 − b2n+5 − b2n+7
b−1
la première fraction correspondant à un nombre de 2n + 9 chiffres ne contenant que des 1.
De même, on obtient :

b b2n+9 − 1
1100 1 . . . 1 0011 = − b2 − b3 − b2n+5 − b2n+6
b−1
Il suffit donc de montrer que le quotient suivant :

b2n+9 − 1 − (b − 1) (b + b3 + b2n+5 + b2n+7 )


b2n+9 − 1 − (b − 1) (b2 + b3 + b2n+5 + b2n+6 )

ne dépend pas de n, et donc par exemple est égal à sa valeur pour n = 0, qui se simplifie
(on n’est pas obligé de le voir, mais bon) :

b9 − 1 − (b − 1) (b + b3 + b5 + b7 ) b4 − b3 + b2 − b + 1
=
b9 − 1 − (b − 1) (b2 + b3 + b5 + b6 ) b4 − b2 + 1

103
Pour cela, on calcule les produits en croix et on vérifie (péniblement) qu’ils sont égaux.

Solution de l’exercice 74 : On raisonne comme pour la décomposition en base b.


On remarque que si n s’écrit sous la forme :

n = a1 1! + a2 2! + a3 3! + · · · + ad d! + · · ·

avec 0 6 ai 6 i, alors a1 est forcément le reste de la division euclidienne de n par 2 puisque


la somme a2 2! + a3 3! + · · · est un multiple de 2. On pose donc la division euclidienne de n
par 2 et on écrit :
n = 2q1 + a1
pour des entiers q1 et a1 avec 0 6 a1 6 1. Il s’agit maintenant d’écrire q1 sous la forme :
2! 3! d!
q1 = a2 + a3 + · · · ad + · · ·
2 2 2
et pour cela on considère la division euclidienne de q1 par 3. On écrit :

q1 = 3q2 + a2

pour des entiers q2 et a2 avec 0 6 a2 6 2. On veut alors écrire q2 sous la forme :


2! 4! d!
q2 = a3 + a4 + · · · ad + · · ·
3! 3! 3!
et on considère donc la division euclidienne de q2 par 4, obtenant ainsi q3 et a3 .
La suite des qi est une suite d’entiers positifs et si qi > 0, alors qi+1 < qi . Il existe donc
un entier d tel que qd = 0 et à ce moment on a l’égalité :

n = a1 1! + a2 2! + a3 3! + · · · + ad−1 (d − 1)!

Pour l’unicité, on remarque en analysant la construction précédente, qu’à chaque étape,


on n’avait qu’un seul choix pour ai .

Solution de l’exercice 75 : Dans un premier temps, on a an > 1 = F1 . D’autre part, par


les propriétés de la division euclidienne, la suite des ai est strictement décroissante. Ainsi
an−1 > 2 = F2 et le quotient de la division euclidienne de ai par ai−1 est toujours au moins
1. De cela, on déduit l’inégalité :
ai+1 6 ai−1 − ai
Une récurrence immédiate permet alors de prouver que pour tout i ∈ {1, . . . , n}, on a
an−i > Fi+1 . Pour i = n − 2, on obtient l’inégalité de l’énoncé.

Remarque. Cette inégalité assure la rapidité de l’algorithme √d’Euclide. En effet, on √peut


montrer√ que l’on a une expression de Fn sous la forme Fn = 55 (ϕn − ϕ̄n ) où ϕ = 1+2 5 et
ϕ̄ = 1−2 5 . Ainsi, on obtient :

5 n
a2 > Fn−1 > (ϕ − 1)
5

104
Ainsi le nombre d’étapes dans l’algorithme d’Euclide est majorée par un nombre de l’ordre
de logϕ (a2 ).

Solution de l’exercice 76 : Les entiers 3 et 5 sont premiers entre eux et liés par la relation
de Bézout 2 × 3 − 5 = 1. Fixons un entier c. D’après le cours, l’équation :

5a + 3b = 2 − 15c

admet des solutions pour tout c qui sont données par a = 15c − 2 − 3n et b = 4 − 30c + 5n
pour n décrivant Z.
L’ensemble des solutions est donc l’ensemble des triplets (15m − 2 − 3n, 4 − 30m + 5n, m)
pour m et n décrivant Z.

Remarque. L’ensemble des triplets (a, b, c) précédemment déterminés correspondent aux


points à coordonnées entières sur le plan d’équation 5a + 3b + 15c = 0.

Solution de l’exercice 77 : On sait que si n est un entier et que si sn désigne la somme des
chiffres de n en base 2, alors :
v2 (n!) = n − sn
Ici, par hypothèse, v2 (n!) > n − 1. La seule possibilité est d’avoir sn = 1, ce qui impose à n
d’être une puissance de 2.

Solution de l’exercice 78 : On choisit un nombre n formé simplement avec des 0 et des 1 et


suffisamment espacés pour que l’élévation au carré ne fasse pas intervenir de retenues.
On peut prendre :
1997
X 2i
n= 102
i=1

Alors déjà l’écriture décimale n ne contient que des 0 mis à part 1997 fois le chiffre 1. On a
donc bien s(n) = 1997. D’autre part, on aura :
1997
X X
2 2i+1 2i +22j
n = 102 + 2 · 102
i=1 16i<j61997

Les exposants qui apparaissent sont deux à deux distincts, comme on le voit en regardant
leur écriture en base 2 par exemple. Il s’ensuit que n2 possède 1997 chiffres 1, 1997×1996
2
chiffres 2 et sinon que des 0. On en déduit que s(n2 ) = 19972 .

Solution de l’exercice 79 : On remarque que si l’entier k vérifie 1 6 k 6 n et est premier avec


n, alors il en est de même de l’entier n − k. Ainsi on peut regrouper les nombres premiers
avec n et inférieurs à n deux à deux, sauf éventuellement si k = n − k.
Ce dernier cas équivaut à k = n2 . Déjà, il ne peut se produire que si n est pair. Mais si
n > 2 est pair, les nombres n2 et n ne sont pas premiers entre eux.
Tout cela prouve que ϕ (n) est toujours pair pour n > 2.

Solution de l’exercice 80 : Le déplacement ii) fait penser à l’algorithme d’Euclide. On se


doute alors que la condition nécessaire et suffisante doit porter sur le pgcd de x et de y.

105
Plus précisément, nous allons montrer que le point (x, y) est atteignable si, et seulement si
x et y sont strictement positifs et pgcd (x, y) est une puissance de 2.
On remarque dans un premier temps que si le couple (a, b) est formé d’entiers strictement
positifs tels que pgcd (a, b) est une puissance de 2 alors il en est de même des couples (a, 2b),
(2a, b), (a − b, a) si a > b et (a, b − a) si b > a. Cela implique la nécessité de la condition :
toute case (x, y) pouvant être atteinte par un jeton est telle que x et y sont strictement
positifs et pgcd (x, y) est une puissance de 2.
Il reste à prouver que toutes ces cases peuvent effectivement être atteintes. Nous montrons
cela par récurrence : au rang n, notre hypothèse de récurrence stipule que tout couple (x, y)
vérifiant la condition précédente et tel que x + y 6 n peut être atteint par un jeton. On
initialise notre récurrence à n = 2, auquel cas le seul couple (x, y) d’entiers strictement
positifs tel que x + y 6 2 est le couple (1, 1) qui est bien atteignable.
Traitons l’hérédité. Donnons-nous donc un couple (x, y) d’entiers strictement positifs
tels que pgcd (x, y) soit une puissance de 2, et supposons que x + y 6 n + 1. Si x est pair,
l’hypothèse de récurrence s’applique pour le couple ( x2 , y) et la transformation i) permet
de conclure. On raisonne de même si y est pair. Sinon c’est que x et y sont impairs et
donc premiers entre eux. S’ils sont égaux à 1, il n’y a rien à faire. Sinon, ils sont forcément
distincts. Supposons par exemple x < y. Dans ce cas, on regarde le couple (x, x+y 2
) qui relève
de l’hypothèse de récurrence comme on le vérifie directement. Il peut donc être atteint. Une
transformation de type i) permet alors de passer au couple (x, x+y) puis une transformation
de type ii) au couple (x, y) ce qui conclut.

Solution de l’exercice 81 : Supposons que ces 14 entiers existent. Parmi eux, il en existe un,
disons k, qui n’est ni le premier, ni l’un des trois derniers, qui est un multiple de 10. Ainsi
k − 1, k, k + 1, k + 2 et k + 3 sont tous les cinq prodigieux. Nous allons montrer que cela
est impossible.
Comme k est un multiple de 10, on a directement P (k + i) = iP (k) pour 0 < i < 10. En
particulier, P (k + 1) = P (k) et ce nombre doit diviser k +1 puisque k +1 est prodigieux. Or
k est également prodigieux, donc P (k) divise k et finalement P (k) = 1. Ainsi P (k + 3) = 3
divise k + 3 et donc k est un multiple de 3.
Comme P (k) = 1, l’entier k ne s’écrit qu’avec des 0 et des 1 et le chiffres de ses unités
est 0. Ainsi k − 1 ne termine par un 9 et P (k − 1) est un multiple de 3, et donc il en est de
même de k − 1. Mais cela est impossible puisque déjà k était un multiple de 3.

Solution de l’exercice 82 : Nous allons montrer par récurrence qu’il existe des suites (an ) et
(bn ) telles que un = 2an + 2bn pour tout n et les nombres an+1 − an et bn+1 − bn sont soit 0,
soit 1. Il est clair que cela suffira pour conclure.
Pour n = 1 et n = 2, il suffit de prendre a1 = b1 = a2 = 0 et b2 = 1. Passons à l’hérédité.
Si un+1 = 2un−1 , il suffit de prendre an+1 = an−1 + 1 et bn+1 = bn−1 + 1. Sinon, on a :

un+1 = 3un − 2un−1 = 2an + 2bn + 2an +1 + 2bn +1 − 2an−1 +1 − 2bn−1 +1

Alors si an = an−1 + 1, on pose an+1 = an + 1 et sinon (c’est-à-dire si an−1 = an ), on pose


an+1 = an . On fait de même avec bn et cela termine l’hérédité et l’exercice.

106
Solution de l’exercice 83 : Supposons que les nombres x, x2 et xn aient tous les trois la même
partie décimale. Déjà, on ne peut pas avoir 0 < x < 1, car sinon les trois nombres seraient
dans l’intervalle [0, 1[ et par le fait devraient être égaux.
Supposons donc x > 1. Alors xn > x2 > x et la condition nous dit que les deux nombres
x2 − x et xn − x sont entiers. On peut donc écrire x2 = x + a pour un certain entier a > 0.
En multipliant par x, on obtient successivement :

x3 = x2 + ax = x (a + 1) + a
¡ ¢
x4 = x2 (a + 1) + ax = x (2a + 1) + a2 + a
..
.

Et finalement par récurrence, on obtient xk = uk x + vk pour des entiers uk > a et vk . Pour


k = n, on obtient xn = un x + vk et donc :

xn − x = (un − 1) x + vk

ce qui assure que (un − 1) x est entier et donc que x est rationnel.
Supposons que x ne soit pas entier. Alors il existerait p un nombre premier tel que
vp (x) < 0. Mais alors vp (x2 ) = 2vp (x) < vp (x) et donc vp (x2 − x) = vp (x2 ) < 0, ce qui n’est
pas possible puisque x2 − x est un entier. On en déduit que x est entier.

Solution de l’exercice 84 : On définit une suite (an ) par récurrence de la façon suivante. On
pose a0 = 1 et a1 = 2. Puis an+1 = 1an (on ajoute 1 à gauche de l’écriture décimale de an )
si 2n+1 ne divise pas an , et an+1 = 2an dans le cas contraire.
Il est clair que l’écriture décimale de an n’utilise que des 1 et des 2. On montre alors par
récurrence que an est un multiple de 2n . L’initialisation est immédiate. Et si on suppose que
an = 0 (mod 2n ) alors :
– si an 6= 0 (mod 2n+1 ) alors an = 2n · bn avec bn impair. D’où an+1 = 10n + an =
2n (5n + bn ) = 0 (mod 2n+1 ) ;
– si an = 0 mod [2n+1 ] alors an+1 = 2 · 10n + an = 0 mod 2n+1 .

Solution de l’exercice 85 : a) Les deux réponses sont « oui ». Les nombres a2 − b2 et a − b


sont entiers, donc leur quotient a + b est rationnel. Par suite la somme (a − b) + (a + b) = 2a
est également rationnelle et donc a est rationnel. On en déduit directement que b est aussi
rationnel.
Montrons désormais que a et b sont entiers. On raisonne par l’absurde en supposant qu’il
existe un nombre premier p tel que vp (a) < 0. Comme a − b est entier, il existe un entier k
tel que a = b + k et on a alors :

an − bn = (b + k)n − bn = nbn−1 k + C2n bn−2 k 2 + C3n bn−3 k 3 + · · · + k n

Choisissons à partir de maintenant n premier à p, suffisamment grand. Alors on a :

vp (nbn−1 k) = (n − 1) vp (b) + vp (k)

qui est strictement négatif. De plus pour tout i compris entre 2 et k, on a :

vp (Cin bn−i k i ) − vp (nbn−1 k) = vp (Cin ) + (i − 1) vp (k) − (i − 1) vp (b)

107
Le deux premiers termes du membre de droite sont positifs ou nuls et le troisième est
strictement positif. On en déduit l’inégalité :

vp (Cin bn−i k i ) > vp (nbn−1 k)

et donc :
vp (an − bn ) = vp (nbn−1 k) < 0
ce qui est contradictoire.
√ √
b) Oui. Prenons par exemple a = − 2 et b = 2 + 2. La somme a + b = 2 est bien
rationnelle. D’autre part, on vérifie par récurrence qu’il existe pour tout n des entiers un et
vn tels que : ³ √ ´n √
1 + 2 = un + v n 2
où un > 1 et vn > 0 dès que n > 2. On a ainsi :
³√ ´n ³ √ ´
bn = 2 un + v n 2

Si n = 2k est un nombre pair (avec k > 1), on en déduit :


³ √ ´
n n k
a + b = 2 1 + un + v n 2

qui est bien irrationnel puisque vn est non nul.


Si n = 2k + 1 est un nombre impair (avec k > 1), on obtient :
√ ³ √ ´ √
an + bn = 2k 2 −1 + un + vn 2 = 2k+1 vn + 2k (un − 1) 2

qui est encore irrationnel puisque un > 1 et donc un − 1 est non nul.
Remarque. La méthode précédente est agréable car elle ne nécessite que peu de calculs.
Toutefois, si on ne la trouve pas, on peut développer l’expression de an + bn grâce à la
formule du binôme de Newton et parvenir plus laborieusement à la même conclusion.

c) Non. Si l’un des deux nombres (disons a) est nul, alors :

a3 + b3
b=
a2 + b2
est rationnel.
Sinon, on peut écrire :

(a3 + b3 )(a2 + b2 ) − (a5 + b5 )


a+b =
a2 b2
1 2 1
a2 b2 = (a + b2 )2 − (a4 + b4 )
2 2
La deuxième égalité prouve que a2 b2 est rationnel et alors la première prouve qu’il en est de
même de a + b.

108
Solution de l’exercice 86 : Il faut bien entendu comprendre que comme la martienne a six
doigts à chaque main, elle a appris à compter en base 12. Les nombres de la formule sont
donc écrits en base 12 : de fait 13 correspond à 15, 22 à 26 et 19 à 21. La formule de la
martienne devient alors, version humaine :

(5x + 3) (3x − 7) = 15x2 − 26x − 21

qui est un développement parfaitement correct.

Solution de l’exercice 87 : Il s’agit de montrer que la suite de Kolakoski ne devient jamais


périodique. Notons un le n-ième terme de cette suite. Raisonnons par l’absurde et suppose
qu’il existe un entier N et une période t tels que un+t = un pour tout n > N . Choisissons t
minimal pour cette propriété. Toute période est alors un multiple de t.
Notons a (resp. b) le nombre d’indices i (N 6 i < N + t) pour lesquels ui = 1 (resp.
ui = 2). On a bien sûr a + b = t. D’autre part, par définition de la suite a + 2b est également
une période et donc un multiple de t = a + b. Mais si ni a ni b n’est nul, on a :

a + b < a + 2b < 2 (a + b)

et donc a + 2b ne peut être un multiple de a + b. D’autre part a = 0 est absurde car cela
signifierait qu’il n’y a que des 1 dans la suite à partir d’un certain rang. De même b = 0 est
absurde.
On a finalement obtenu une contradiction : la suite (un ) n’est pas périodique à partir
d’un certain rang et le nombre x est irrationnel.

Solution de l’exercice 88 : Soit a un entier compris entre 1 et n − 1. On va montrer que a


et k + a (considéré modulo n) ont la même couleur. Si k + a < n, il faut montrer que a et
k + a ont la même couleur, mais c’est évident puisque |(k + a) − k| = a.
Si k + a > n, il faut montrer que a et k + a − n ont la même couleur. Mais k + a − n a la
même couleur que n − a d’après la condition (2). Et n − a a la même couleur que a d’après
la condition (1).
On a ainsi prouvé que a a la même couleur que (le reste de la division euclidienne par n
de) a + αk pour tout entier α. Comme k et n sont premiers entre eux, d’après le théorème
de Bézout, il existe des entiers α et β tels que αk = 1 + βn, soit :

a + αk = (a + 1) + βn

Ainsi a a toujours la même couleur que a + 1. Cela conclut.

Solution de l’exercice 89 : Posons sn = 5n + 7n . On remarque que :

sn = sm sn−m − 5m 7m sn−2m si n > 2m


sn = sm sn−m − 5m 7m s2m−n si n < 2m

Ainsi pgcd (sn , sm ) = pgcd (sm , sn−2m ). En effectuant l’algorithme d’Euclide, et en utilisant
le fait que m en n sont premiers entre eux, on voit que :
– si m et n sont de même parité (donc impairs), pgcd (sm , sn ) = pgcd (s1 , s1 ) = 12
– si n et m sont de parité contraire, pgcd (sm , sn ) = pgcd (s0 , s2 ) = 2

109
Solution de l’exercice 90 : Si ce nombre était rationnel, son écriture décimale serait pério-
dique à partir d’un certain rang. Notons N un entier tel que pN tombe dans la partie
périodique. Il en sera de même de tout nombre premier pn avec n > N .
Notons r la période. Par exemple, d’après le postulat de Bertrand (avec la constante 10
remplaçant 2 – ce qui est moins fort), il existe un nombre premier de k chiffres pour tout
k. En particulier, il existe un nombre premier pn (avec n > N ) dont le nombre de chiffres
est un multiple de r, supérieur à 2r. Ce nombre doit être une répétition d’une séquence de r
chiffres et par le fait doit être divisible par 1 . . . 1 (r fois). Ainsi, il n’est pas premier. C’est
une contradiction.

Autre solution. Soit n un entier. D’après le théorème de Dirichlet, il existe une infinité de
nombres premiers congrus à 1 modulo 10n+1 . Ces nombres premiers possèdent une suite
de n zéros consécutifs dans leur écriture décimale. Ainsi, le réel dont il est question a des
suites de zéros consécutifs arbitrairement longues dans sa partie décimale. Si ce nombre
était rationnel, sa période serait uniquement composée de zéros... or il est bien connu que
les nombres premiers ne sont pas constitués exclusivement de zéros.

Solution de l’exercice 91 : On rappelle que l’on dispose d’une formule :


n (n + 1) (2n + 1)
12 + 22 + · · · + n2 =
6
de laquelle on déduit :
(n − 1) (2n2 + 5n + 6)
22 + · · · + n2 =
6
et la condition dit alors simplement que ce dernier quotient doit être une puissance d’un
nombre premier p. Ainsi, à part un facteur 2 et un facteur 3, le nombre premier p doit être le
seul qui intervient dans la décomposition en facteurs premiers du produit (n − 1) (2n2 + 5n + 6).
Si n > 8, chacun des facteurs est strictement supérieur à 6 et donc doit faire intervenir le
nombre premier p dans sa décomposition en facteurs premiers. Ce p doit donc aussi intervenir
dans la décomposition en facteurs premiers de pgcd (n − 1, 2n2 + 5n + 6).
On peut calculer ce pgcd par l’algorithme d’Euclide. On commence par écrire :

2n2 + 5n + 6 = (n − 1) (2n + 7) + 13

ce qui prouve que pgcd (n − 1, 2n2 + 5n + 6) = pgcd (n − 1, 13). Puisque 13 est un nombre
premier, forcément, p = 13 et l’équation devient :
¡ ¢
(n − 1) 2n2 + 5n + 6 = 6 × 13k

On sait (toujours sous l’hypothèse n > 8) que l’on doit avoir v13 (n − 1) > 1. Supposons
v13 (n − 1) > 2 ou autrement dit que n ≡ 1 (mod 169). Alors on vérifie que 2n2 +5n+6 ≡ 13
(mod 169) et donc que v13 (2n2 + 5n + 6) = 1. D’autre part, on a vu que v2 (2n2 + 5n + 6) 6
1, v3 (2n2 + 5n + 6) 6 1 et v` (2n2 + 5n + 6) = 0 pour tout ` différent de 2, 3 et 13. On en
déduit l’inégalité 2n2 + 5n + 6 6 2 × 3 × 13 = 78, qui n’est jamais vérifiée pour n > 8.
On en déduit que v13 (n − 1) = 1 et donc que n − 1 ne peut valoir que 13, 13 × 2, 13 × 3
ou 13 × 6, ce qui ne laisse qu’un nombre fini de cas que l’on vérifie à la main.

110
Les solutions sont les entiers n = 2, n = 3, n = 4 et n = 7.

Solution de l’exercice 92 : En réalité, après une étude attentive des cartes, on constate que
la première carte réunit exactement les nombres dont l’écriture en base 2 se termine par un
1 (c’est-à-dire les nombres impairs), que la deuxième carte réunit les nombres dont l’avant-
dernière chiffre de l’écriture en base 2 est un 1, et ainsi de suite.
Ainsi lorsque le partenaire montre les cartes, il donne l’écriture en base 2 du nombre
qu’il a choisi. Il ne reste plus qu’à faire la conversion vers la base 10. Cela peut se faire
simplement de la façon suivante : on additionne tous les nombres écrits en haut à gauche
des cartes montrées, la somme est le nombre choisi.

Solution de l’exercice 93 : Notons a, b et c trois éléments distincs de A. Il s’agit de montrer


que l’on ne peut pas avoir a + c = 2b. Le nombre b ne s’écrit qu’à l’aide de 0 et de 1 en base
3, ainsi le nombre 2b n’utilise, lui, que des 0 et des 2.
Notons de plus que l’addition de a par c ne peut pas faire intervenir de retenus. Comme
a et c sont distincts, il existe un rang n tel que le n-ième chiffre (en partant de la droite) de
a et de c diffèrent. Ainsi l’un vaut 0 et l’autre 1, et la somme fait 1. Forcément, donc, dans
la somme a + c écrite en base 3 possède un 1 ; elle ne peut donc égaler 2b.

Solution de l’exercice 94 : Effectuons la division euclidienne de a par b : il existe q et r tels


que a = bq + r et 0 6 r < b. On a alors :
¡ ¢ ¡ ¢
2a + 1 = 2bq+r + 1 = 2bq+r − 2r + (2r + 1) = 2r 2bq − 1 + (2r + 1)

Or la factorisation :
¡ ¢¡ ¢
2bq − 1 = 2b − 1 2b(q−1) + 2b(q−2) + · · · + 1

montre que 2bq − 1 est un multiple de 2b − 1. Ainsi, pour notre situation, il faut que 2r + 1
soit un multiple de 2b − 1, et donc en particulier 2r + 1 > 2b − 1. De plus r < b, donc en
regroupant on obtient 0 < 2b − 2r 6 2, qui n’a pas de solution puisque l’on a supposé b > 2.

Solution de l’exercice 95 : Pour tout k > 0, on note Pk le polynôme P ◦ · · · ◦ P obtenu en


composant P k fois avec lui-même. En particulier, pour tout i, ai+k = Pk (ai ). On pose de
plus d = pgcd(am , an ), et si par exemple m > n, on note m = n + k. Il vient ainsi, avec des
notations évidentes : Ã !
X
am = Pk (an ) = an αr ar−1
n + Pk (0)
r>1

Il en résulte que d divise Pk (0) = Pk (a0 ) = ak = am−n . Par une récurrence du type de
l’algorithme d’Euclide, il en résulte aussitôt que d divise apgcd(m,n) .
Il s’agit alors réciproquement de voir que apgcd(m,n) divise am , ou plus généralement que
pour tout i et tout r > 1, ai divise ari . Mais c’est clair par récurrence : ai se divise lui-même,
et si ai divise ari , alors le calcul précédent montre qu’il divise aussi Pi (ari ) = a(r+1)i puisque
c’est le coefficient constant de Pi .
On a donc bien montré que pour tous m, n, on a :

pgcd(am , an ) = apgcd(m,n)

111
Solution de l’exercice 96 : Notons z = ab = x + iy. Notons x0 (resp. y 0 ) un entier tel que
|x − x0 | 6 12 (resp. |y − y 0 | 6 12 ). Posons finalement q = x0 + iy 0 et r = a − bq. On a :
¯r¯ q √
¯ ¯ 2 2 2
¯ ¯ = |(x − x0 ) + i (y − y 0 )| = (x − x0 ) + (y − y 0 ) 6 <1
b 2
Ainsi |r| < |b| comme on le souhaite.
La décomposition n’est pas unique comme le montre l’exemple suivant :

1=0×2+1=1×2−1

Notons que ce contre-exemple est déjà valable dans Z : la condition de positivité est donc
essentielle à l’unicité et il est difficile d’en donner un équivalent convaincant sur Z [i].

Commentaire. Un ensemble de nombres sur lequel on dispose d’une telle division est un
anneau euclidien. Comme dans l’exemple de Z, cette propriété en implique de nombreuses
autres bien utiles : existence de pgcd, existence et unicité de la décomposition en facteurs
premiers...

Solution de l’exercice 97 : On remarque aisément que 1994 = 997 × 2 s’écrit 22 en base 996.
Supposons que 1993 soit brésilien. Dans ce cas, il existerait des entiers a, b et k avec
1 6 a < b < 1992 tels que :
bk − 1
1993 = a ·
b−1
Or, 1993 est un nombre premier. Donc, forcément a = 1. En écrivant 1993 en base 2, . . . , 6,
on voit que forcément b > 7. Ceci implique k 6 3. D’autre part k > 3 car les hypothèses
faites assurent que le nombre ne peut avoir un chiffre ou deux chiffres égaux à 1. Il ne reste
plus qu’à résoudre :
1 + b + b2 = 1993
et on vérifie que les solutions de cette équation ne sont pas entières.

Solution de l’exercice 98 : On va montrer que parmi les termes de la suite, on peut trouver
une série de 1 aussi longue que l’on veut. Comme d’un autre côté, on va prouver que la suite
n’est pas constante à partir d’un certain rang, cela permettra directement de conclure.
Prenons n = 10j . Alors nk = 10jk commence par 1, donc xn = 1. Mais le nombre
£ j √ ¤k
10 × k 2 < 2 × 10jk commence par un 1 également. De façon plus générale, pour tout i
£ √ ¤
compris entre 10j et 10j × k 2 , on a xi = 1. Le nombre de i concernés par cette dernière
propriété croı̂t indéfiniment avec j, ce qui assure la première partie de la conclusion.
¡ £ √ ¤¢k
D’autre part, pour tout j, le nombre 1 + 10j × k 2 commence par un 2, ce qui suffit
pour conclure.

Solution de l’exercice 99 : Le nombre de chiffres d’un entier n en base b est donné par :
· ¸
log n
1+
log b

112
Ainsi, si l’on définit pour n > 0 :
· ¸ · ¸
log 10k log 10
uk = = k
log 2 log 2
· ¸ · ¸
log 10k log 10
vk = = k
log 5 log 5
il s’agit de prouver que l’ensemble des valeurs prises par les suites uk et vk forment une
partition de N? . D’après le théorème de Beatty, il suffit pour cela de vérifier que log 10
log 2
et
log 10
log 5
sont irrationnels et que :
log 2 log 5
+ =1
log 10 log 10
La deuxième condition est immédiate. Pour vérifier que le nombre log 10
log 2
est irrationnel,
on suppose qu’il s’écrit m
n
pour certains entiers m et n strictement positifs, ce qui donne
m n
10 = 2 . C’est impossible.

Solution de l’exercice 100 : Notons n = dd0 . La deuxième condition de l’énoncé devient


d2 d0 + 1 divise d2 (d02 + 1). Les nombres d2 d0 + 1 et d2 sont premiers entre eux, donc le lemme
de Gauss assure que d2 d0 + 1 divise d02 + 1.
On a d02 + 1 = (d2 d0 + 1) + d0 (d0 − d2 ) et donc d2 d0 + 1 doit divise d0 (d0 − d2 ) puis d0 − d2
en appliquant à nouveau le lemme de Gauss. On a :
d0 − d2 d0 d2
= −
d2 d0 + 1 d2 d0 + 1 d2 d0 + 1
et les deux termes du membre de droite sont compris strictement entre 0 et 1. Leur différence,
qui doit être un entier, est donc forcément nulle. On en déduit que d0 = d2 puis n = d3 .
Réciproquement, on vérifie immédiatement que ces solutions conviennent.

Solution de l’exercice 101 : Soit 0 6 x < n! un entier. Nous allons montrer qu’il existe des
entiers ai tels que :
n! n! n!
x = a1 n! + a2 + a3 + · · · + an
2! 3! n!
avec 0 6 ai < i. Cela conclura car on vérifie directement que chacun de termes non nuls
ai n!
i!
est un diviseur de n! et que si i < j et si ai et aj sont non nuls alors ai n!
i!
> aj n!
j!
. Par
ailleurs, il est facile de traiter le cas x = n! à part.
Pour démontrer la propriété, on raisonne par récurrence sur n. On commence par poser
la division euclidienne de x par n qui s’écrit x = nq + an avec 0 6 an < n. Il s’agit alors
d’écrire l’entier q sous la forme :
(n − 1)! (n − 1)! (n − 1)!
q = a1 (n − 1)! + a2 + a3 + · · · + an−1
2! 3! (n − 1)!
mais c’est exactement l’hypothèse de récurrence : il suffit de vérifier que q < (n − 1)! ce qui
est immédiat.

Solution de l’exercice 102 : Si n est impair, alors 1 et 2 sont premiers avec n et donc il en
est de même de tous les entiers strictement inférieurs à n. Donc n est premier.

113
Si n = 4k, alors 2k − 1 et 2k + 1 sont premiers avec n et inférieurs à n, et donc il en est
de même de tous les entiers impairs inférieurs à n. Ainsi n est une puissance de 2.
Si n = 4k + 2, alors pgcd (2k + 3, n) = pgcd (2k + 3, 2k + 1) car 2k + 3 est impair. Ce
pgcd doit diviser 2 mais ne peut être pair, c’est donc 1. De même 2k + 5 est premier avec
n. De plus comme n > 6, il est inférieur à n et le même argument que précédemment prouve
que n ne peut-être une solution.

Solution de l’exercice 103 : Supposons que an soit pair et notons k sa valuation 2-adique.
Le nombre 32 an est alors entier et de valuation 2-adique k − 1. On voit directement par
récurrence que pour i 6 k, v2 (an+i ) = k − i et donc an+k est impair. Ceci démontre qu’il y
a une infinité de termes impairs.
De même, si an est impair, on note k la valuation 2-adique de an − 1. Le nombre 32 an
n’est pas entier et donc sa partie entière vaut an+1 = 32 an − 12 = an + an2−1 . Ainsi :
3
an+1 − 1 = (an − 1)
2
et donc la valuation 2-adique de ce nombre est k − 1. Comme ci-dessus, il vient an+k − 1 est
impair et an+k est pair.

Remarque. Les propriétés précédentes se voient lorsque l’on écrit les nombres an en base 2.

Solution de l’exercice 104 : Clairement, les puissances de nombres premiers conviennent. Soit
n une solution qui ne n’est pas une puissance de nombre premier. Soit p le plus petit diviseur
premier de n. On pose n = pα k où k est premier avec p. Notons que k > 2.
Puisque k est un diviseur de n, tout diviseur de k est aussi un diviseur de n. Les entiers
p et k sont des diviseurs de n premiers entre eux, on doit donc avoir p + k − 1 divise n. Si
` est un nombre premier divisant p + k − 1, il doit diviser n. Si ` 6= p, il divise en outre k
et donc p − 1, ce qui n’est pas possible. Ainsi p est le seul diviseur premier de p + k − 1 et
donc p + k − 1 = pa pour un certain entier a > 2. Ainsi p2 divise n.
Supposons k et p + 1 premiers entre eux. Alors k et p2 − 1 sont également premiers entre
eux. En appliquant l’hypothèse p2 + k − 1 divise n. Comme ci-dessus, on en déduit que
p2 + k − 1 = pb pour un certain entier b strictement supérieur à a. Dans ces conditions, il
vient :
p2 − p = pb − pa
et comme b > a > 2, on en déduit que p2 divise p, ce qui est absurde.
Par suite, d = pgcd (k, p + 1) = pgcd (k, p2 − 1) > 1. Or, si d divise p + 1 et k, il divise
aussi n et la minimalité de p assure alors que p = 2 et d = 3 et que k est impair. De plus,
on sait que k + 1 = 2a . Cette fois, on a p2 + k − 1 = k + 3 qui est divisible par 2, par 3, mais
ni par 9 (puisque sinon k serait également divisible par 9 et donc 3 le serait aussi) ni par
aucun autre nombre premier q > 3 qui divise n. Par suite k + 3 = 3 · 2b , où b > 1. Ainsi, il
vient 2 + 2a = 3 · 2b et donc facilement a = 2 et b = 1, puis k = 3 et n = 3 · 2α , avec α > 2.
Si α > 3 alors 23 + 3 − 1 = 10 doit diviser n, ce qui est absurde. Donc α = 2 et n = 12 qui
effectivement est bien une solution.
Finalement, les solutions sont les puissances de nombres premiers et n = 12.

Solution de l’exercice 105 : Appelons A, B et C les trois boı̂tes. Elles contiennent respec-
tivement a, b et c jetons. Sans perte de généralité, on peut supposer que 1 6 a 6 b 6 c.

114
On pose b = aq + r avec 0 6 r < a et q > 1 la division euclidienne de b par a. Soit
q = m0 + 2m1 + · · · + 2k mk l’écriture de q en base 2 (avec donc mk = 1 et mi ∈ {0, 1} pour
tout i).
Comme c > b, il y a au moins 2k a jetons dans la boı̂te C. On double alors k fois de suite
le nombre de jetons dans A par les transvasements suivants : à l’étape i, si mi = 1 on prend
les 2i a jetons dans B, et si mi = 0, on les prend dans C. Il y en aura bien suffisamment dans
C car 2k > 1 + 2 + · · · + 2k−1 .
À l’issue de ces opérations, on se retrouve avec 2k+1 a jetons dans A, r jetons dans B et le
reste dans C. En particulier, la boı̂te qui contient le moins de jetons en contient strictement
moins que a. En répétant la procédure précédente, on finira par obtenir une boı̂te vide.

Solution de l’exercice 106 : Commençons par exprimer les termes de la suite en fonction de
x0 . On peut le faire à l’aide du développement en base 2.
On peut bien sûr supposer que x0 < 1, et on écrit alors le développement en base 2 de
x0 sous la forme :
x0 = 0, a1 a2 . . .
Alors on a 2x0 = a1 , a2 a3 . . . Si a1 = 1, il vient donc :

|1 − 2x0 | = 2x0 − 1 = 0, a2 a3 . . .

et si à l’inverse a1 = 0, on a :

|1 − 2x0 | = 0, a¯2 a¯3 . . . où l’on note 0̄ = 1 et 1̄ = 0

On obtient ainsi : ½
0, a2 a3 . . . si a1 = 0
x1 = 1 − |1 − 2x0 | =
0, a¯2 a¯3 . . . sinon
Par une récurrence immédiate, on obtient le terme de rang n de la suite :
½
0, an+1 an+2 . . . s’il y a un nombre pair de 1 parmi a1 , . . ., an
xn =
0, an+1
¯ an+2
¯ . . . sinon

Cela étant, si x0 est rationnel, son développement en base 2 est périodique à partir d’un
certain rang N , de période k. Alors pour tout n > N , il y a nécessairement un nombre pair
de 1 parmi les chiffres an+1 , . . . , an+2k qui forment deux périodes consécutives, et bien sûr
an+2k+` = an+` pour tout `. Il en résulte que xn = xn+2k pour tout n > N , et donc (xn ) est
périodique de période au plus 2k à partir du rang N .
Réciproquement, supposons (xn ) périodique à partir du rang N , et notons k une période.
En comparant les chiffres de xn et xN +k , il vient (selon la parité du nombre de 1 parmi
aN +1 , · · · , aN +k ) ou bien an+k = an pour tout n > N + 1, ou bien an+k = a¯n pour tout
n > N + 1. Dans tous les cas, an+2k = an pour tout n > N + 1, et donc la suite des chiffres
binaires de x0 est périodique à partir du rang N + 1, ce qui entraı̂ne que x0 est rationnel.
Finalement, la suite (xn ) est périodique si et seulement si x0 est rationnel.

Solution de l’exercice 107 : a) On raisonne par récurrence sur p. Pour p = 1, la formule dit
simplement que Fn+1 = Fn+1 , ce qui est certainement vrai. Pour p = 2, la formule redonne

115
la relation de récurrence qui définit la suite de Fibonacci. Supposons donc la formule vraie
aux rangs p et p + 1, et prouvons-là pour le rang p + 2. On calcule :

Fn+p+2 = Fn+p+1 + Fn+p = Fp Fn + Fn+1 Fp+1 + Fp−1 Fn + Fn+1 Fp


= Fn (Fp + Fp−1 ) + Fn+1 (Fp+1 + Fp )
= Fn Fp+1 + Fn+1 Fp+2

ce qui est bien ce que l’on désirait.

b) On montre par récurrence que pour tout n, les nombres Fn et Fn+1 sont premiers
entre eux : cela est vrai pour n = 1, et l’hérédité s’obtient en remarquant qu’un diviseur
commun à Fn et Fn+1 doit également diviser Fn+1 − Fn = Fn−1 . La formule prouvée en a)
implique alors pour tous p et n, on a pgcd (Fn+p , Fp ) = pgcd (Fn , Fp ) : en effet, un diviseur
d commun à Fn+p et Fp doit diviser Fp−1 Fn d’après la formule, mais il est également premier
avec Fp−1 car Fp l’est, et donc le lemme de Gauss assure que d divise Fn ; l’autre sens se
traite de façon analogue.
Une récurrence immédiate prouve que si on a une égalité du type a = bq + r, alors :

pgcd (Fr , Fb ) = pgcd (Fb+r , Fb ) = · · · = pgcd (Fqb+r , Fb ) = pgcd (Fa , Fb )

Le principe de l’algorithme d’Euclide prouve alors que si d = pgcd (a, b), on a pgcd (Fm , Fn ) =
Fd .

Solution de l’exercice 108 : Notons pk le (k + 1)-ième nombre premier, de sorte que p0 = 2


désigne le premier nombre premier. Nous allons montrer par récurrence sur n que si nd . . . n0
est l’écriture de n en base 2 (ie si n = n0 + 2n1 + · · · + 2d nd ), alors :

xn = pn0 0 · · · pnd d

Pour n = 0, c’est vrai. Supposons que ce soit vrai pour n et montrons-le pour n + 1.
Notons nd . . . n0 l’écriture en base 2 de n. Soit i le plus petit indice tel que ni = 0 (s’il
n’existe pas, on convient que i = d + 1 bien entendu). Dans ce cas, l’écriture en base 2 de
n + 1 est :
nd . . . ni+1 10 . . . 0
D’autre part p (xn ) = pi et q (xn ) = p0 . . . pi−1 . On vérifie alors directement l’hypothèse de
récurrence.
La factorisation de 111 111 est 3 × 7 × 11 × 13 × 37. L’entier n tel que xn = 111 111 s’écrit
en base 2, d’après la propriété que l’on vient de prouver :

100000111010

Le nombre cherché est donc 4218.

Solution de l’exercice 109 : Après certains essais, on constate que f inverse l’écriture en base
2. Plus précisément si a s’écrit en base 2, ap . . . a0 où ap est 1, il semble que f (x) soit le
nombre qui s’écrive a0 . . . ap en base 2

116
Montrons cela par récurrence. On vérifie que la propriété précédente est vraie aux rangs
1, 2, 3 et 4. Pour l’hérédité supposons que f agisse comme on l’a dit sur les entiers compris
entre 1 et n et regardons l’entier n + 1. Il y a trois cas à distinguer :
Tout d’abord si n + 1 est pair, alors il s’écrit 2k pour un certain entier k. Supposons que
k s’écrive kp . . . k0 en base 2 où kp vaut 1. Alors 2k s’écrit :

2k : kp . . . k0 0

et par hypothèse de récurrence, f (k) s’écrit :

f (k) : k0 . . . kp

ce qui correspond bien à l’écriture renversée. L’hérédité est donc prouvée dans ce cas.
Regardons à présent le cas où n + 1 s’écrit 4k + 1 pour un certain entier k. Écrivons
encore k en base 2 : kp . . . k0 . Alors 4k + 1 s’écrit :

4k + 1 : kp . . . k0 01

et en utilisant l’hypothèse de récurrence on peut poser l’opération suivante :

2f (2k + 1) : 1k0 . . . kp 0
− f (k) : k0 . . . kp
10k0 . . . kp

ce qui est bien ce que l’on veut encore une fois.


Finalement si n + 1 s’écrit 4k + 3 pour un certain entier k. Écrivons encore k en base 2 :
kp . . . k0 . Alors 4k + 3 s’écrit :

4k + 3 : kp . . . k0 11

et en utilisant l’hypothèse de récurrence on peut poser l’opération suivante :

2f (2k + 1) : 1k0 . . . kp 0
+ f (2k + 1) : 1k0 . . . kp
− 2f (k) : k0 . . . kp 0
11k0 . . . kp

la première et la troisième ligne se simplifiant bien. Cela conclut l’hérédité.


Il ne reste plus qu’à compter le nombre d’entiers « symétriques en base 2 » et inférieurs
à 1988. Ce nombre en base 2 s’écrit 11111000100. Il a onze chiffres.
Avec un seul chiffre, il n’y a qu’une solution ; c’est 1. Avec deux chiffres, il n’y a aussi
qu’une seule solution ; c’est 11. Avec trois chiffres, maintenant, le premier est forcément fixé
à 1 et par conséquent le dernier aussi, mais on a libre choix sur celui du milieu, il y a donc
deux solutions.
p−1
De la même façon pour p valant 5, 7 ou 9, il va y avoir 2 2 solutions de p chiffres. Pour
p−2
les p pairs, donc valant 4, 6, 8 ou 10, il y aura 2 2 solutions. Ainsi parmi les nombres qui
ont moins de 10 chiffres, on dénombre 1 + 1 + 2 + 2 + 4 + 4 + 8 + 8 + 16 + 16 = 62 solutions.
Voyons les nombres de 11 chiffres maintenant. Une solution éventuelle doit s’écrire en
base 2 sous la forme suivante :

117
1_________1

les cinq premiers « _ » représentant a priori des chiffres arbitraires, et les quatre derniers
étant déterminés par le choix des premiers. Toutefois pour que ce nombre reste inférieur à
11111000100, il faut imposer que les quatre premiers « _ » ne soient pas simultanément des
1, et c’est en fait la seule contrainte. On dénombre alors (24 − 1) × 2 = 30 solutions dans
cette situation.
Au final l’équation proposée admet 92 solutions.

Solution de l’exercice 110 : Montrons d’abord que si k 6 n, alors le nombre formé par les
k derniers chiffres de l’écriture décimale vaut au moins 2k . En effet, les nombres 2n et 10k
sont divisibles par 2k . Il en est donc de même du reste de la division de 2n par 10k , qui est
justement formé par les k derniers chiffres de 2n . Ce reste est non nul, et donc on a bien la
propriété annoncée.
On note ai le i-ième chiffre (le chiffre de droite étant le 0-ième) de l’écriture décimale de
n
2 . Alors :

2n = a0 + 10a1 + 100a2 + · · · + ai 10i + · · ·


S (2n ) = a0 + a1 + a2 + · · · + ai + · · ·

Soit k tel que 1 6 4k 6 n. D’après ce que l’on vient de voir, le nombre formé par les chiffres
a4k−1 . . . a1 a0 vaut au moins 24k > 10k . Il possède donc au moins un chiffre non nul de rang
supérieur ou égal à k. Autrement dit, au moins un parmi ak , ak+1 , . . . , a4k−1 est non nul.
Soit n > 4 un entier, et soit m tel que 4m 6 n < 4m+1 . Chacun des intervalles
[1, 3] , [4, 15] , . . . , [4m−1 , 4m − 1] contient donc au moins un entier i tel que ai 6= 0. On en
déduit que :
S (2n ) > m > log4 n − 1
ce qui permet de conclure.

5.3 Exercices de « Congruences »


Solution de l’exercice 111 : Pour voir que le produit de k entiers consécutifs positifs n, n + 1,
. . ., n + k − 1 est divisible par k!, il suffit de remarquer que :

n(n + 1) · · · (n + k − 1)
= Cnk
k!
est entier. S’ils sont tous négatifs, on considére les opposés. Sinon, c’est que l’un d’entre eux
est nul, et donc le produit également.

Solution de l’exercice 112 : Soit n = nr . . . n1 n0 l’écriture de n en base 2. Pour tout k, si


k = kr . . . k1 k0 en base 2, on a :
r
Y
Cnk ≡ Cnkii (mod 2)
i=0

118
donc Cnk est impair si et seulement si Cnkii = 1 pour tout i, c’est-à-dire lorsque l’on a pas,
pour un certain i, ki = 1 et ni = 0.
Cette condition est automatiquement vérifiée si tous les ni sont égaux à 1, et réciproque-
ment, si l’on a nj = 0 pour un certain j 6 r, alors en prenant k = 2j < n, il vient que Cnk
est pair.
Finalement, Cnk est impair pour 0 6 k 6 n si et seulement si tous les chiffres de l’écriture
en base 2 de n valent 1, ce qui revient à dire que n est de la forme 2r+1 − 1.

Solution de l’exercice 113 : Notons k le nombre de suites et pour i compris entre 1 et k,


notons Ni la raison de la i-ième suite et ai un de ses termes. L’ensemble des valeurs prises
par la i-ième suite est alors l’ensemble des entiers x tels que :

x ≡ ai (mod Ni )

Il s’agit donc de prouver que le système suivant :




 x ≡ a1 (mod N1 )

 x ≡ a2 (mod N2 )
(S) : ..

 .

 x ≡ ak (mod Nk )

admet une solution. Mais pour cela, on a vu dans le cours qu’il suffit de vérifier que pour
tous i et j, on a ai ≡ aj (mod pgcd (Ni , Nj )). Or cela est vrai, puisque par hypothèse, le
système formé par les i-ième et j-ième lignes de (S) admet une solution.

Solution de l’exercice 114 : Pour tout d diviseur de n, notons Ad l’ensemble des éléments de
{1, . . . , n} dont le pgcd avec n faut exactement nd .
Déjà, il est évident que les Ad sont deux à deux disjoints et que leur réunion est égale à
tout {1, . . . , n}, puisque pour tout i ∈ {1, . . . , n}, le pgcd de n et de i est un nombre de la
forme nd pour d un diviseur de n.
Soit x ∈ Ad . L’entier xd n
est un élément de {1, . . . , d} et il est premier avec d. Récipro-
quement, tous les éléments de Ad sont obtenus ainsi. Il s’ensuit que Card Ad = ϕ (d). La
conclusion découle de tout ce qui précède.

Solution de l’exercice 115 : C’est en fait une application du théorème chinois9 . Appelons x
le nombre de soldats. La méthode des chinois permet au général d’accéder à des nombres
x2 , x3 , x5 , x7 , x11 , x13 et x17 tels que :

x ≡ xi (mod i)

pour tout i ∈ {2, 3, 5, 7, 11, 13, 17}. On constate que les nombres de l’ensemble précédent
sont premiers entre eux deux à deux, et donc le théorème chinois s’applique et affirme qu’il
existe un entier n tel que le système précédent soit également à :

x≡n (mod 510 510)


9
D’où le nom...

119
On conclut en remarquant qu’il existe au plus un entier inférieur à 500 000 et satisfaisant
cette dernière congruence.

Solution de l’exercice 116 : Dans un premier temps, on remarque que a = bn est bien solution
du problème.
Supposons qu’il y ait deux solutions, disons a et a0 . Soit k > b un entier. Les nombres
a − k n et a0 − k n doivent tous deux être divisibles par b − k et donc en particulier congrus
modulo b−k. On en déduit que pour tout k > b, on doit avoir a ≡ a0 (mod b−k). Autrement
dit, a et a0 doivent être congrus modulo tous les entiers strictement positifs. Mais cela n’est
possible que si a = a0 . D’où la conclusion.

Solution de l’exercice 117 : Cet exercice est évident avec les congruences. Comme 9 ≡ 2
(mod 7), on a 9n ≡ 2n (mod 7) pour tout n, ce qui est bien ce que l’on veut. Si l’on préfère,
on peut également utiliser la factorisation :
¡ ¢
9n − 2n = (9 − 2) 9n−1 + 9n−2 · 2 + · · · + 2n−1
qui permet de conclure directement également.

Solution de l’exercice 118 : On a les congruences 4p + 1 ≡ p + 1 (mod 3) et 7p − 4 ≡ p + 2


(mod 3). Ainsi forcément l’un des trois nombres p, 4p + 1 et 7p − 4 est un multiple de
3. Comme 4p + 13 et 7p − 4 sont tous les deux forcément strictement supérieur à 3, c’est
forcément p le multiple de 3 et puis p = 3.
On vérifie réciproquement que pour p = 3, on a 4p + 1 = 12 et 7p − 4 = 17 qui sont bien
tous premiers.

Solution de l’exercice 119 : Non. Supposons qu’une telle permutation existe. Des deux éga-
lités :
ai + · · · + ai+9 ≡ 0 (mod 10)
ai+1 + · · · + ai+10 ≡ 0 (mod 10)
on déduit ai ≡ ai+10 (mod 10) pour tout i pour lequel cela a un sens. Comme les ai doivent
atteindre tous les résidus modulo 10, on en déduit qu’il en est de même de l’ensemble
{a1 , . . . , a10 }.
Mais cet ensemble a dix éléments, et il y a dix résidus modulo 10 et donc {a1 , . . . , a10 }
doit être une permutation modulo 10 de l’ensemble {1, . . . , 10}. En particulier, on doit avoir :
a1 + · · · + a10 ≡ 1 + · · · + 10 ≡ 5 (mod 10)
ce n’est pas en accord avec l’hypothèse.

Solution de l’exercice 120 : Un point A de coordonnées entières (x, y) est invisible si et


seulement si pgcd (x, y) > 1. On cherche donc des entiers x et y tels que pour tous entiers
i et j dans {0, . . . , L}, les nombres x + i et y + j ne soient pas premiers entre eux. (Le point
(x, y) sera alors le coin inférieur gauche d’un carré convenable).
Considérons, pour i et j variant entre 0 et L, des nombres premiers pij deux à deux
distincts. D’après le lemme chinois, il existe un entier x tel que x ≡ −i (mod pij ) pour tous
i et j. De même il existe un entier y tel que y ≡ −j (mod pij ) pour tous i et j.

120
Pour un tel choix d’entiers, on a bien pij divise à la fois x + i et y + j.

Solution de l’exercice 121 : Il suffit de reconnaı̂tre les coefficients binomiaux C4k dans l’écri-
ture de 104060401 :
X4
104060401 = C4k 102k = 1014
k=0

Or 101 est premier, donc les diviseurs de 1014 sont les 101k , 0 6 k 6 4. Ainsi :

1015 − 1 10510100501 − 1
σ(104060401) = = = 105101005
100 100

Solution de l’exercice 122 : Notons S la somme envisagée. On écrit :


µ ¶ µ ¶ µ ¶ µ ¶
1 1 1 1 1 1
2S = 1+ + + + ··· + + + ··· + +1
p−1 2 p−2 i p−i p−1
· ¸
1 1 1
= p + ··· + + ··· +
p−1 i (p − i) p−1

Le terme entre crochets est représentée par une fraction dont le dénominateur est premier à
p. Ceci termine l’exercice.

Solution de l’exercice 123 : Notons 2n et 2m , avec n < m, deux éventuelles puissances de 2


ayant exactement les mêmes chiffres. En particulier 2n et 2m ont le même nombre de chiffres
et donc m − n < 4. La différence m − n vaut donc 1, 2 ou 3.
D’autre part, on doit avoir 2n ≡ 2m (mod 9), soit 2m−n ≡ 1 (mod 9). Mais on vérifie
que 21 ≡ 2 (mod 9), 22 ≡ 4 (mod 9) et 23 ≡ 8 (mod 9). Aucun ne vaut 1, il n’y a donc pas
de solution.

Solution de l’exercice 124 : Soit n = nr . . . n1 n0 et k = kr . . . k1 k0 les écritures de n et k en


base 2. Dans la solution de l’exercice 5.3, on a vu que Cnk était impair si et seulement si pour
tout i tel que ni = 0, on avait ki = 0. Par conséquent, si n a s zéros dans son écriture binaire,
il en résulte que le nombre d’entiers k compris entre 0 et n qui conviennent est exactement
2r+1−s . En particulier, c’est bien une puissance de 2.

Solution de l’exercice 125 : S’il existait une telle suite, elle contiendrait à l’évidence un
nombre premier strictement supérieur à 7. On peut donc supposer x0 = p > 7. Un tel
p est forcément congru à 1 ou 5 modulo 6.
Supposons p ≡ 1 (mod 6). Alors 2p + 1 ≡ 3 (mod 6) et donc 2p + 1 est un multiple de 3.
Ainsi on a forcément x1 = 2p − 1. En outre 2p − 1 ≡ 1 (mod 6), et en appliquant à nouveau
le même raisonnement, il vient x2 = 2x1 − 1. Par récurrence, on prouve que xn = 2xn−1 − 1,
ce qui donne la formule générale :

xn = 2n p − 2n + 1

Mais alors xp−1 = 2p−1 p − 2p−1 + 1 ≡ 0 (mod p) d’après le petit théorème de Fermat. Ceci
constitue une contradiction.

121
On traite de même le cas où p ≡ 5 (mod 6).

Solution de l’exercice 126 : La réponse est affirmative. Pour construire une telle permutation,
on commence par poser a1 = 1. Soit ensuite k > 1 fixé. Supposons que l’on ait déterminé
des entiers strictement positifs deux à deux distincts a1 , · · · , ak tels que pour tout i 6 k on
ait :
Xi
aj = 0 (mod i)
j=1

Soit alors n le plus petit entier non encore utilisé. Puisque k + 1 et k + 2 sont premiers
entre eux, le lemme chinois assure de l’existence d’un entier m arbitrairement grand, et
donc différent de n et de chacun des ai déjà choisis, tel que :

m ≡ −(a1 + · · · + ak ) (mod k + 1)
m ≡ −(a1 + · · · ak + n) (mod k + 2)

On pose alors ak+1 = m et ak+2 = n. La suite ainsi construite vérifie clairement la condition
de divisiblité, et la minimalité du n considéré à chaque étape assure que l’on utilisera bien
chaque entier une et une seule fois.

Solution de l’exercice 127 : Notons x1 , . . . , x111 nos entiers. Ils sont tels que x1 +· · ·+x111 = 0.
Dans un premier temps on remarque que 399 = 3 × 7 × 19. Nous allons donc montrer que
le nombre :
x37 37
1 + · · · + x111

est à la fois multiple de 3, 7 et 19. Par le petit théorème de Fermat, on a x2 ≡ 1 (mod 3)


pour tout x premier avec 3. En élévant à la puissance 18, on obtient x36 ≡ 1 (mod 3), puis
x37 ≡ x (mod 3), cette dernière congruence étant en fait valable pour tout x. Ainsi :

x37 37
1 + · · · + x111 ≡ x1 + . . . + x111 = 0 (mod 3)

On raisonne de la même façon dans les autres cas.

Solution de l’exercice 128 : On a :

1 − 21 + ¡13 − 14 + · · · − 1318
1
+
¢
1
1319¡ ¢
= ¡1 + 12 + · · · + 1319
1
¢ − 2¡ 12 + 14 + · · · + 1318
1
¢
= 1 + 12 + · · · + 13191
− 1 + 12 + · · · + 6591
1 1
= 660 + · · · + 1319
1 1 1979
On a 660 + 1319 = 660×1319 et ainsi de suite. Comme la somme précédente comporte un
nombre pair de termes, on obtient au final :
· ¸
1 1 1
1979 + + ··· +
660 × 1319 661 × 1318 989 × 990

Le terme entre crochets est une fraction dont le dénominateur est premier avec 1979 (car
0
1979 est premier). On a donc obtenu une écriture de la somme sous la forme ab0 avec a0

122
multiple de 1979 et b0 premier avec 1979. Si ab est une autre écriture, on a l’égalité ab0 = b0 a
et donc 1979 divise ab0 , puis a par le lemme de Gauss.
Remarque. Si l’on sait que les fractions dont le dénominateur premier avec p peuvent
se voir dans Z/pZ (voir la seconde partie, chapitre 5), on peut procéder différemment. La
somme S que l’on cherche à estimer est :
1 1 1 1 1
1+ + ··· + + + + ··· +
2 1319 −1 −2 −659
et l’on remarque qu’il s’agit de la somme des inverse des entiers : −659 ≡ 1320 (mod p),
−658 ≡ 1321 (mod p) et ainsi de suite. On conclut alors en utilisant l’exercice 5.3.

Solution de l’exercice 129 : Un nombre premier p convenable s’écrit :

p = p0 + p1 b + · · · + pb−1 bb−1

où (p0 , . . . , pb−1 ) est une permutation de l’ensemble {0, . . . , b − 1}. On a alors :

b (b − 1)
p ≡ 0 + 1 + · · · + (b − 1) = (mod b − 1)
2
et donc si b est pair, p est un multiple de b − 1. Si b > 2, p a au moins deux chiffres non nuls
et donc p > b − 1. Ceci prouve que p ne peut-être un nombre premier. Pour b = 2, on voit
directement que le nombre s’écrivant 01 en base 2 (i.e. 1) n’est pas premier, alors que celui
s’écrivant 10 (i.e. 2) l’est.
Si b est impair, on pose n = b−1
2
. On a b ≡ 1 (mod n) et donc :

b (b − 1)
p ≡ 0 + 1 + · · · + (b − 1) = ≡ 0 (mod n)
2
Or si b > 3, on a comme précédemment p > n, ce qui est contradictoire car p est premier.
Il ne reste plus qu’à vérifier pour les nombres écrits en base 3. Il y a 012 (5), 021 (7), 102
(11), 120 (15), 201 (19) et 210 (21).
Finalement les nombres premiers p qui conviennent sont 2, 5, 7, 11 et 19.

Solution de l’exercice 130 : Il suffit de prouver que pour tout d, 2d−1, 5d−1 ou 13d−1 n’est
pas un carré. Supposons qu’ils soient tous les trois des carrés. Comme 2d − 1 est impair, il
faut qu’il soit congru à 1 modulo 4, et donc d doit être impair. On pose d = 2d0 + 1. Comme
2d − 1 = 4d0 + 1 est congru à 1 modulo 4, il doit être congru à 1 modulo 8, et donc d0 est
pair. On pose donc d0 = 2d00 et ainsi les deux nombres 20d00 + 4 et 52d00 + 12 sont des carrés.
Ces deux nombres sont congrus respectivement à 4d00 + 4 et 4d00 + 12 modulo 16. On obtient
une contradiction en remarquant que les seuls résidus quadratiques multiples de 4 modulo
16 sont 0 et 4 ; ils ne peuvent donc différer de 8.

Solution de l’exercice 131 : Non, un tel entier n’existe pas. En effet, supposons qu’il existe.
Dans un premier temps, on constate qu’au plus un nombre de l’ensemble {n, . . . , n + 18}
peut être un multiple de 19. Ainsi s’il y a un tel élément, il sera soit dans A, soit dans B.
Supposons que ce soit dans A. Alors 19 divise le produit des éléments de A mais pas celui
des éléments de B puisqu’il est premier. C’est une contradiction.

123
On en déduit qu’aucun élément n’est multiple de 19. Ainsi modulo 19, l’ensemble {n, . . . , n + 18}
est exactement l’ensemble {1, . . . , 18}. Si l’on note P le produit des éléments de A (qui est
aussi celui des éléments de B), on a donc :

P 2 ≡ 1 × 2 × . . . × 18 (mod 19)

et d’après le théorème de Wilson (ou un calcul si on ne connaı̂t pas ce théorème), P 2 ≡ −1


(mod 19). Mais en calculant tous les résidus quadratiques modulo 19, on vérifie que cette
congruence est impossible.

Solution de l’exercice 132 : Le nombre |B 2 − 1| − (B 2 − 1) est non nul si, et seulement si


B 2 − 1 < 0, c’est-à-dire puisque B est un entier si, et seulement si B = 0. Ainsi si B 6= 0,
on a toujours f (x, y) = 2 qui est premier.
Si B = 0, on a |B 2 − 1| − (B 2 − 1) = 2 et f (x, y) = y + 1. Le fait que B soit nul implique
y! + 1 = x (y + 1), soit encore y! ≡ −1 (mod y + 1), ce qui assure, d’après le théorème de
Wilson, que y + 1 = f (x, y) est premier. Ainsi la fonction f ne prend que des valeurs qui
sont des nombres premiers.
Considérons à présent p un nombre premier impair. D’après ce qu’il précède, s’il s’écrit
f (x, y), c’est que y = p − 1 et que y! + 1 = x (y + 1), soit x = (p−1)!+1
p
. Le nombre premier
p ne peut donc être atteint qu’une unique fois. Et il est effectivement atteint, lorsque l’on
choisit x et y comme précédemment : la fraction qui définit x est bien un entier, encore
d’après le théorème de Wilson.

Solution de l’exercice 133 : Soit p un nombre premier différent de 2 et de 5. Alors parmi les
nombres n, n+1, . . . , n+p−1, il y en a un et un seul qui est un multiple de p. Autrement dit,
il existe un unique entier k ∈ {n, n + 1, . . . , n + p − 1} tel que vp (k) > 1. Pour les autres,
on a vp (k) = 0. Cela prouve que :
µ ¶
1 1 1
vp + + ··· + 6 −1
n n+1 n+p−1

Un nombre décimal est un nombre de la forme 10as . En tout cas, sa valuation p-adique (pour
p différent de 2 et 5) est nulle. Cela résout la première partie de la question.
Traitons maintenant le cas p = 2. Si ` est un nombre premier différent de 2 et 5, par le
même argument que précédemment, on prouve qu’aucun des deux nombres n et n + 1 ne
0 0
peut être multiple de `. Il en résulte que l’on peut écrire n + 1 = 2a 5b et n = 2a 5b , pour des
entiers a, b, a0 et b0 positifs ou nuls. On est donc amené à considérer l’équation :
0 0
2a 5b = 2a 5b + 1

Si on a a > 0, alors le membre de gauche de l’égalité précédente est pair. Pour que le membre
de droite le soit aussi, il faut a0 = 0. De même on prouve que si b > 0, alors on doit avoir
b0 = 0 et des énoncés analogues en échangeant soit a et a0 , soit b et b0 soit les deux. Bref, on
est ramené à résoudre séparément les deux équations suivantes :
0 0
2a = 5b + 1 et 5b = 2a + 1

124
Pour la première équation, on regarde modulo 4 : si a > 2, le terme de gauche vaut toujours
0 et celui de droite toujours 1. On a donc forcément a = 1 (car a = 0 ne convient pas) et
donc b0 = 0. Regardons l’autre équation. On vérifie d’abord que a0 = 1 ne fournit aucune
solution, contrairement à a0 = 2 qui donne b = 1. Ensuite, on regarde l’équation modulo 3 :
0
on obtient 2b ≡ 2a + 1 (mod 3), ce qui n’est possible que si b est impair. Posons b = 2u + 1.
L’équation devient :
0
5 × 25u = 2a + 1
0
Maintenant si a0 > 3, on a 2a +1 (mod 8), mais 5×25u ≡ 5 (mod 8), d’où une contradiction.
Finalement, les seuls nombres consécutifs de la forme 2x 5y sont d’une part 1 et 2 et d’autre
part 4 et 5. On en déduit que les seules sommes qui conviennent sont :
1 1 1
1+ = 1, 5 et + = 0, 45
2 4 5

Reste le cas p = 5. Comme pour le cas p = 2, on montre qu’il n’est pas possible qu’un
nombre premier ` > 5 divise un des nombres n, n + 1, n + 2, n + 3 et n + 4. D’autre part,
encore par le même argument, on montre que 3 doit forcément diviser deux nombres de la
liste précédente, donc soit n et n + 3, soit n + 1 et n + 4. Quoi qu’il en soit, les deux nombres
intermédiaires sont consécutifs et non divisibles par 3 et donc de la forme 2x 5y . De l’étude
faite précédemment, on déduit facilement que la seule solution est :
1 1 1 1 1
+ + + + = 1, 45
2 3 4 5 6

Solution de l’exercice 134 : a) Soit n > 0 un entier. On pose Mn = {a1 , . . . , an } où ak =


(k ·n!)n! pour tout k. Soit r ∈ {1, . . . , n}. On note que chacun des ak est un entier strictement
positif divisible par r et que c’est également une puissance r-ième d’entier. Cela assure que la
somme de r quelconques d’entre eux est un multiple de r et que le produit de r quelconques
d’entre eux est une puissance r-ième, et donc que l’ensemble Mn convient.

b) Non. On raisonne par l’absurde en supposant qu’un tel ensemble infini existe. Appelons-
le M . Soient a et b deux éléments de M , distincts. Alors, il existe un nombre premier p qui
ne divise pas a − b. Soient x1 , x2 , · · · , xp−1 des éléments de M \{a, b} deux à deux distincts.
Alors :

x1 + · · · + xp−1 + a ≡ 0 (mod p)
x1 + · · · + xp−1 + b ≡ 0 (mod p)

Mais alors a ≡ b (mod p), ce qui est supposé faux.

Solution de l’exercice 135 : Si un > 7, soit un est pair et un+1 = u2n est inférieur à un . Soit
un est impair, un+1 = un + 7 est pair, donc un+2 = un2+7 < un . Il en résulte que la plus
petite valeur ne peut pas être strictement supérieure à 7. Mais elle peut être égale à 7, car
si un = 7, les termes suivants sont 14, 7, 14, 7, . . . et la suite ne descend plus. Si un = 2,
4 ou 6, alors un+1 = u2n est strictement inférieur à un . Si un = 5, les termes suivants sont
12, 6, 3, 10, 5, 12, . . . et la suite descend jusqu’à 3. De même si un = 3. Enfin, si un = 1, un

125
ne peut pas descendre plus bas, les termes suivants de la suite étant 8, 4, 2, 1, 8, 4, . . .. Le
plus petit entier atteint par cette suite peut être soit 1, soit 3, soit 7, tout dépend quel est
le nombre de départ.
Pour atteindre 7, il faut que tous les termes de la suite soient multiples de 7, car un+1
est divisible par 7 si et seulement si un est divisible par 7. Or il est clair que 20002003 n’est
pas multiple de 7 car 2000 n’est pas multiple de 7. Plus précisément, 2000 ≡ 5 (mod 7), car
1995 = 7 × 285. On a 52 ≡ 4 (mod 7), 53 ≡ 6 (mod 7), 54 ≡ 2 (mod 7), 55 ≡ 3 (mod 7) et
(comme permettait de le prévoir le théorème de Fermat) 56 ≡ 1 (mod 7). Pour tout k, 56k
sera donc congru à 1 modulo 7, en particulier 51998 (puisque 1998 = 6 × 333). D’où 52003 est
congru à 3 modulo 7, puisque 2003 = 1998 + 5. Il en résulte que 20002003 ≡ 3 (mod 7).
Or l’algorithme transforme un nombre congru à 3 modulo 7 :
– s’il est impair, en un autre nombre congru à 3 modulo 7 (puisqu’on ajoute 7)
– s’il est pair, en un nombre congru à 5 modulo 7 (puisque 2 × 5 congru à 3 modulo 7)
Un nombre congru à 5 modulo 7 est transformé, s’il est impair, en un autre nombre
congru à 5 modulo 7, et s’il est pair, en un nombre congru à 6 modulo 7. Un nombre congru
à 6 modulo 7 est transformé, s’il est impair, en un autre nombre congru à 6 modulo 7, et
s’il est pair, en un nombre congru à 3 modulo 7.
La boucle est bouclée : à partir d’un nombre congru à 3 modulo 7, en réitérant autant
de fois que l’on veut l’algorithme, on ne peut obtenir que des nombres congrus à 5, 6 ou 3
modulo 7. On ne peut donc jamais atteindre le nombre 1 : le plus petit entier atteint par
cette suite est 3.

Solution de l’exercice 136 : Soit p un nombre premier quelconque. La valuation p-adique de


an = ppcm(1, 2, . . . , 2n) est le plus grand entier v tel que pv 6 2n. Mais alors pour tout
k > v, on a : · ¸ · ¸
2n n
k
−2 k =0
p p
Par conséquent, on a :
X · 2n ¸ n
¸ ·
n
vp (C2n ) = k
−2 k
k>1
p p
Xv · ¸ · ¸
2n n
= − 2
k=1
pk pk
v
X
6 1=v (puisque [2x] 6 2[x] + 1)
k=1

n n
On a donc montré que pour tout nombre premier, vp (C2n ) 6 vp (an ), ce qui assure que C2n
divise an .

Solution de l’exercice 137 : Soit ` un diviseur premier de pp − 1. On a alors pp ≡ 1 (mod `).


D’après le petit théorème de Fermat, on a p`−1 ≡ 1 (mod `) et donc l’ordre de p modulo `
est un diviseur de pgcd (p, ` − 1). C’est soit 1, soit p.
Si c’est p, comme p`−1 ≡ 1 (mod `), il vient p divise ` − 1 et donc ` ≡ 1 (mod p) comme
on le veut.

126
Si c’est 1 cela signifie que p ≡ 1 (mod `), c’est-à-dire que ` divise p − 1.
Il suffit donc de prouver qu’il existe un diviseur premier ` pour lequel cet ordre est p,
c’est-à-dire ne divisant pas p − 1. Pour cela, on introduit la factorisation :
£ ¤
pp − 1 = (p − 1) pp−1 + pp−2 + · · · + p + 1

Le facteur entre crochets est congru à 1 modulo p − 1 et donc est premier avec p − 1. Ainsi
tout nombre premier ` divisant ce facteur est un diviseur premier de pp − 1 ne divisant pas
p − 1. Il ne reste plus qu’à prouver que ce facteur est strictement supérieur à 1 mais c’est
évident.

Solution de l’exercice 138 : Le quotient :


(n1 + · · · + nk )!
n1 ! · · · nk !
peut se voir comme le nombre de partitions ordonnées (E1 , . . . , Ek ) d’un ensemble E à
n1 + · · · + nk éléments en k ensembles E1 , . . ., Ek , avec Card Ei = ni . Il en résulte en
particulier que c’est un entier.

Remarque. On laisse au lecteur le soin de traiter cet exercice par les valuations p-adiques.
n
Solution de l’exercice 139 : Soit p un nombre premier quelconque. On veut voir que vp (C2n )>
`
vp (n + 1). Pour cela, notons ` = vp (n + 1). Ainsi n + 1 s’écrit p m avec m premier à p. Si
` = 0 le résultat est clair. Sinon, on remarque que pour 1 6 k 6 `, on a :
· ¸ · ¸
n 1
= p m − k = p`−k m − 1
`−k
pk p
et de même : · ¸ · ¸
2n 2
k
= 2p m − k = 2p`−k m − 1
`−k
p p
car pk > 2. Par conséquent, on a la minoration suivante de vp (C2n
n
):
X µ· 2n ¸ · ¸¶
n
n
vp (C2n ) = k
−2 k
k>1
p p
X̀ µ· 2n ¸ · ¸¶
n
> k
−2 k
k=1
p p

> 1=`
k=1

ce qui conclut.

Autre solution. Un calcul prouve que :


1
Cn = Cn2n − Cn+1
n + 1 2n 2n

127
et voilà...
1 n
Remarque. L’entier cn = n+1 C2n s’appelle le n-ième nombre de Catalan. Il apparaı̂t dans
un certain nombre de problème de dénombrement. Par exemple, c’est le nombre de bons
parenthésages que l’on peut faire avec n parenthèses ouvrantes et n parenthèses fermantes.

Solution de l’exercice 140 : On calcule les puissances successives de 3 modulo 17 :

30 ≡ 1 (mod 17) ; 31 ≡ 3 (mod 17) ; 32 ≡ 9 (mod 17)

33 ≡ −7 (mod 17) ; 34 ≡ −4 (mod 17) ; 35 ≡ 5 (mod 17)


36 ≡ −2 (mod 17) ; 37 ≡ −6 (mod 17) ; 38 ≡ −1 (mod 17)
39 ≡ −3 (mod 17) ; 310 ≡ −9 (mod 17) ; 311 ≡ 7 (mod 17)
312 ≡ 4 (mod 17) ; 313 ≡ −5 (mod 17) ; 314 ≡ 2 (mod 17)
315 ≡ 6 (mod 17) ; 316 ≡ 1 (mod 17) ; 317 ≡ 3 (mod 17)
La suite obtenue est périodique de période 16. On raisonne alors comme suit. Si n ≡ 0
(mod 16), alors 3n ≡ 1 (mod 17), et donc on doit avoir n ≡ −16 (mod 272) d’après le
lemme chinois.
Si n ≡ 1 (mod 16), alors 3n ≡ 3 (mod 17) et donc on doit avoir n ≡ −31 (mod 272).
On traite ainsi tous les cas. Toutes les solutions trouvées précédemment conviennent
comme on le vérifie immédiatement.
Finalement les solutions sont les entiers congrus à −16, −31, 162, 163, 132, 5, 134, 57,
152, −71, 42, 75, 140, 29, −66 ou −33 modulo 272 (sauf erreur !).

Remarque. Pour accélérer les calculs, on peut remarquer que le nombre 17a − 16b est congru
à a modulo 16 et à b modulo 17.

Solution de l’exercice 141 : Pour montrer que :

(2m)!(2n)!
a(m, n) =
n!m!(m + n)!
m
est entier, on peut par exemple procéder par récurrence sur n. En effet, a(m, 0) = C2m est
entier pour tout m. Supposons que pour un certain n, a(m, n) soit entier pour tout n, et
remarquons que l’on a :

2(n + 1)(2n + 1)(2m)!(2n)! 2(2n + 1)


a(m, n + 1) = = a(m, n)
(n + 1)(m + n + 1)m!n!(m + n)! m+n+1

On a de plus la relation symétrique en échangeant m et n, si bien que :

2(2n + 1) + 2(2m + 1)
a(m + 1, n) + a(m, n + 1) = a(m, n) = 4a(m, n)
m+n+1
Ainsi a(m, n + 1) = 4a(m, n) − a(m + 1, n) est entier d’après l’hypothèse de récurrence.

128
Finalement, on a bien montré que a(m, n) était entier pour tous m, n. Notons que l’on
aurait également pu conclure en établissant la positivité de vp (a(m, n)) pour tout nombre
premier p.

Solution de l’exercice 142 : Supposons que n1 , . . . , nk forment une solution. Clairement, si


l’un des ni est égal à 1, on en déduit par réaction en chaı̂ne que tous sont égaux à 1.
On suppose donc qu’aucun des ni n’est égal à 1. Pour tout i, on note pi le plus petit
diviseur premier de ni . Alors pi divise 2ni−1 − 1 (avec n0 = nk ). Notons que cela assure en
passant que pi est impair. Soit mi l’ordre de 2 modulo pi . Alors mi divise ni−1 et mi divise
pi − 1 (d’après le petit théorème de Fermat). En particulier, 1 < mi 6 pi − 1 < pi et donc
le plus petit diviseur pi−1 de ni−1 est inférieur à pi . D’où pk > pk−1 > · · · > p1 > pk . Il n’y a
donc bien pas d’autre solution.

Solution de l’exercice 143 : On commence par calculer la suite des puissances de 7 modulo
43. On obtient :

70 ≡ 1 (mod 43) ; 71 ≡ 7 (mod 43) ; 72 ≡ 6 (mod 43)

73 ≡ −1 (mod 43) ; 74 ≡ −7 (mod 43)


5 6
7 ≡ −6 (mod 43) ; 7 ≡1 (mod 43)
La suite est donc périodique de période 6. De plus, on constate que 72 ≡ 6 (mod 43) et
donc la suite des puissances de 6 est périodique de période 3. Un nombre premier p > 5 est
congru soit à 1, soit à 5 modulo 6.
Si p ≡ 1 (mod 6), on a 7p ≡ 7 (mod 43) et 6p ≡ 6 (mod 43) et donc 7p − 6p − 1 est un
multiple de 43. Si p ≡ 5 (mod 6), on a 7p ≡ −6 (mod 43) et 6p ≡ −7 (mod 43) et donc
7p − 6p − 1 est aussi un multiple de 43. Cela conclut.

Solution de l’exercice 144 : Posons N = 2b − 1. Alors évidemment, 2b ≡ 1 (mod N ). Soit


r le reste de la division euclidienne de a par b. On a alors a ≡ r (mod b) et donc 2a ≡ 2r
(mod N ).
Ainsi si N divise 2a + 1, il divise également 2r + 1. Mais si b > 2, on a forcément, puisque
r < b, 0 < 2r + 1 < 2b − 1 et donc 2r + 1 ne peut diviser N . Il n’y a donc finalement pas de
solution au problème.

Solution de l’exercice 145 : Remarquons dans un premier temps que 1989 = 32 × 13 × 17. Il
suffit donc de prouver que la différence est un multiple de 9, de 13 et de 17.
n
Commençons par 13. On veut prouver que na − nb (avec a = nn et b = nn ) est un
multiple de 13, c’est-à-dire que na ≡ nb (mod 13). Comme pour tout x non multiple de
13, x12 ≡ 1 (mod 13) (d’après le petit théorème de Fermat), il suffit de prouver que a ≡ b
(mod 12).
On recommence. On a 12 = 3 × 4, donc on doit montrer que a ≡ b (mod 3) et a ≡ b
(mod 4). Pour 3, encore d’après le petit théorème de Fermat, il suffit de voir que nn et n
sont de même parité, ce qui est évident. Pour 4, c’est légèrement plus compliqué parce que ce
n’est pas un nombre premier. On distingue deux cas : si n est pair, a et b sont des multiples
de 4 et c’est fini. Sinon, on constate que pour tout entier impair x, on a x2 ≡ 1 (mod 4).
L’argument précédent s’applique alors à nouveau.

129
On raisonne de même pour 17. Il faut comparer a et b modulo 16. Là, encore, ce n’est
pas un nombre premier. Si n est pair, a et b sont des multiples de 16 (car n > 3). Sinon,
comme pour tout x impair, on a x4 ≡ 1 (mod 16), on est amené à comparer n et nn modulo
4. L’égalité de ces nombres résulte du fait que n est impair.
Il ne reste plus que 9. Si n est un multiple de 3, na et nb sont des multiples de 9 et donc
leur différence aussi. Sinon, on est amené à comparer a et b modulo 6 (en effet, pour tout x
premier à 3, on a x6 ≡ 1 (mod 9)), c’est-à-dire modulo 3 et modulo 2. Et on a déjà traité
ces deux cas.

Solution de l’exercice 146 : Comme M contient au moins deux éléments, il contient un


nombre premier impair p. En prenant A = {p}, on obtient 2 ∈ M car il divise p − 1.
Soit p un élément de M distinct de 2. Si p ≡ 1 (mod 3), on constate comme précédem-
ment que 3 ∈ M . Si p ≡ 2 (mod 3), on voit que 3 ∈ M car il divise 2p − 1. Sinon, p = 3 et
dans tous les cas 3 ∈ M .
Montrons maintenant que M est infini. On suppose par l’absurde que M est fini. Notons
P le produit des éléments de M . Soit p un élément de M . Le nombre Pp est le produit des
éléments de M différents de p, donc d’après l’hypothèse sur M , le nombre Pp −1 se décompose
en produit d’éléments de M . Comme il est premier avec les éléments de M distincts de p,
c’est une puissance de p. En particulier :

P − 2 = 2α et P − 3 = 3β

pour des entiers α et β. Comme P > 2 × 3 × 5 = 30, on voit que α > 4 et a fortiori P ≡ 2
(mod 8). Cela entraı̂ne 3β ≡ −1 (mod 8), ce qui est absurde. Ainsi M est infini.
Montrons maintenant que M = P. Soit q un nombre premier. Comme M est infini, il
existe q − 1 éléments de M ayant même résidu modulo q, forcément non nul. D’après le petit
théorème de Fermat, le produit de ces nombres est congru à 1 modulo q. Si A est la partie
formée par ces q − 1 nombres, on voit directement que q ∈ M . Cela conclut.

Solution de l’exercice 147 : On remarque en premier lieu que d’après le petit théorème de
Fermat 37 divise n37 − n pour tout entier n. Le pgcd cherché est donc au moins un multiple
de 37.
Cepedant, si p est un nombre premier tel que (p − 1) divise 36, on va avoir 36 = (p − 1) k
et, si n est premier avec p :
¡ 36 ¢ ³¡ ¢ ´
37 p−1 k
n −n=n n −1 =n n − 1 ≡ 0 (mod p)

la dernière congruence résultant du petit théorème de Fermat. On constate que cette congruence
reste vrai si n est un multiple de p. Ainsi, tout nombre premier p tel que p − 1 divise 36 doit
être du pgcd cherché. Les tels nombres premiers sont 2, 3, 5, 7, 13 et 19 et 37.
De plus, pour tout nombre premier p, on vérifie immédiatement que p2 ne peut pas
diviser p37 − p. Le pgcd est donc sans facteur carré.
Il reste à voir si les facteurs premiers trouvés précédemment sont les seuls qui appa-
raissent. Pour cela, on factorise 237 − 2 :
¡ ¢ ¡ ¢¡ ¢ ¡ ¢¡ ¢¡ ¢
237 − 2 = 2 236 − 1 = 2 218 − 1 218 + 1 = 2 29 − 1 29 + 1 218 + 1

130
On a 29 − 1 = 511 = 7 × 73, 29 + 1 = 513 = 33 × 19. Dans le dernier facteur, il doit y
avoir au moins les nombres premiers 5, 13 et 37, on trouve ainsi la factorisation 218 + 1 =
5 × 13 × 37 × 109. Ainsi :

237 − 2 = 2 × 33 × 5 × 7 × 13 × 19 × 37 × 73 × 109

Il ne reste donc plus qu’à tester les nombres premiers 73 et 109. On calcule pour cela 337
modulo ces deux nombres. La méthode suivante est assez efficace, on calcule successivement :

32 ≡ 9 (mod 109)
¡ ¢2
34 = 32 ≡ 92 ≡ 81 (mod 109)
¡ ¢2
39 = 34 × 3 ≡ 812 × 3 ≡ 63 (mod 109)
¡ ¢2
318 = 39 ≡ 632 ≡ 45 (mod 109)
¡ ¢2
337 = 31 8 × 3 ≡ (45)2 × 3 ≡ 80 (mod 109)

et donc 337 − 3 n’est un multiple de 109, ce qui élimine ce nombre premier. En faisant un
calcul analogue, on voit (hélas !) que 337 ≡ 3 (mod 73), et on ne peut donc pas conclure
directement. Pour éliminer 73, il faut calculer 537 −5 modulo 73 : on trouve 63, ce qui permet
d’éliminer le dernier récalcitrant.
Le pgcd cherché est finalement 2 × 3 × 5 × 7 × 13 × 19 × 37 = 1 919 190.

Remarque. Si l’on sait que Z/pZ est cyclique pour p premier, on peut prouver sans faire de
calcul que les seuls facteurs premiers qui apparaissent dans le pgcd sont ceux que l’on a
trouvés.

Solution de l’exercice 148 : Définissons la suite (un ) par :

un = n si n n’est pas un multiple de 5


un = 3un/5 sinon
On montre par récurrence que le dernier chiffre non nul de n! est congru à u1 · · · un
modulo 5. Pour n = 1, c’est évident. Supposons que ce soit le cas pour n. Remarquons que
si n + 1 n’est pas un multiple de 5, alors le nombre de zéros terminant n! sera le même que
le nombre de zéros terminant (n + 1)!. On en déduit directement la propriété voulue dans
le cas où n + 1 n’est pas un multiple de 5. Sinon, notons v = v5 (n + 1). On peut en outre
écrire n! = 10k r où r est un entier congru à u1 · · · un modulo 5 et donc premier avec 5. Dans
ce cas, on a :
n+1 r
(n + 1)! = 10k+v · · v
5v 2
On vérifie facilement que :
µ ¶
n+1 r
k + v + v2 · v = v2 ((n + 1)!) > v5 ((n + 1)!) = k + v
5v 2
n+1 r
Ainsi 5v
· 2v
est entier et le dernier chiffre non nul de (n + 1)! est congru modulo 5 à :
n+1 r
· v ≡ u1 · · · un 3v u n+1 ≡ u1 · · · un+1 (mod 5)
5v 2 5v

131
comme on le veut.
Montrons que la suite (un ) n’est pas périodique modulo 5 (même à partir d’un certain
rang). Supposons que ce soit le cas et notons k une période. Supposons que k soit un
multiple de 5. Dans ce cas, on a pour tout entier n (suffisamment grand), u5n = u5n+k puis
en multipliant par 3 :
un ≡ un+ k (mod 5)
5

k
et donc est également une période. On peut donc supposer que k est premier avec 5. Pour
5
tout entier n (suffisamment grand), on a alors :

un ≡ 2u5n ≡ 2u5n+k = 2 (5n + k) ≡ 2k (mod 5)

ce qui est absurde.


Comme un est toujours premier avec 5, ce dernier résultat implique que la suite des
derniers chiffres non nuls de n! n’est pas périodique à partir d’un certain rang.

Solution de l’exercice 149 : Choisissons une maison et partant de celle-ci, attribuons succes-
sivement tous les entiers positifs ou nuls (dans l’ordre) aux maisons successives en tournant
dans le sens des aiguilles du montre. Ainsi, la maison choisi aura le nombre 0 mais également
les nombres 12, 24, etc. La maison qui est immédiatement après elle dans le sens des aiguilles
d’une montre portera les numéros 1, 13, 25, etc. Si x et y sont deux nombres d’une même
maison, on aura x ≡ y (mod 12).
À toute coloration des maisons, on peut associer un entier de la façon suivante. Pour
toutes les maisons peintes en bleu, on regarde un nombre 2n où n est un numéro quelconque
de la maison, et on somme tous ces nombres. La somme obtenu ainsi n’est pas uniquement
définie, mais par contre elle définit un résidu modulo N = 212 − 1 (car si x ≡ y (mod 12),
alors 2x ≡ 2y (mod N )).
On a une propriété intéressante, conséquence directe de l’unicité de la décomposition en
base 2 : si n1 et n2 sont des nombres associés à deux colorations, et que n1 et n2 définissent le
même résidu modulo N (i.e. si n1 ≡ n2 (mod N )), alors les deux colorations sont identiques
ou l’une est entièrement bleu et l’autre entièrement blanche.
Notons ci le nombre associé à la coloration après le passage de i-ième peintre et c0 le
nombre associé à la coloration initiale. Soit i un entier compris entre 1 et 12. Si le i-ième
peintre repeint les maisons i, i + 1, . . . , i + t − 2 en blanc et la maison i + t − 1 en bleu. On
a alors :
¡ ¢
ci+1 ≡ ci − 2i + 2i+1 + · · · + 2i+t+2 + 2i+t+1 = ci − 2i+t+1 − 2i + 2i+t+1 = ci + 2i (mod N )

On en déduit que :

c12 ≡ c0 + 20 + 21 + · · · + 211 = c0 + N ≡ c0 (mod N )

et donc soit les colorations au début et à la fin sont identiques, soit l’une est totalement
bleue, et l’autre totalement blanche.
Cependant le dernier cas ne peut pas se produire, car aucune d’entre elle ne peut être
totalement blanche. En effet, par hypothèse, celle de départ ne peut pas l’être. Et celle
d’arrivée non plus, car un peintre termine toujours son œuvre en repeignant une maison en
bleu. Cela résout l’exercice.

132
Solution de l’exercice 150 : On est ramené à considérer une équation de la forme :

2n = a · 10d + 2m

ou a est un entier compris entre 1 et 9 et où d + 1 désigne le nombre de chiffres de 2n en


base 10. On a n > m et posons n = m + k où k est un entier strictement positif. L’équation
devient : ¡ ¢
2m 2k − 1 = a10d
et donc 5d divise 2k − 1.
Si d > 1, alors il faut 2k ≡ 1 (mod 25). En regardant les puissances successives de
2 modulo 25, on voit que cela implique que k est un multiple de 20. On a alors 2k ≡ 1
(mod 11) ce qui signifie que 11 divise 2k − 1. Mais cela est impossible car le nombre premier
11 n’apparaı̂t pas dans 10 et ne peut apparaı̂tre dans a 6 9.
Reste le cas d = 1. Cela entraı̂ne que 2m a un seul chiffre et 2n en a deux. Par une
recherche exhaustive, on prouve que les seules puissances qui conviennent sont 32 et 64.

Solution de l’exercice 151 : Supposons par l’absurde que de tels entiers existent.
Soit ` un nombre premier. Nous allons montrer que soit ` divise a, soit ` divise b. Si ce
n’était pas le cas, d’après le théorème de Dirichlet, on pourrait choisit des nombres premiers
arbitrairement grands, p congru à a1 modulo ` et q congru à − 1b modulo `. Alors ap + bq ≡ 0
(mod `), et donc ap + bq n’est pas premier (si on a choisi p et q suffisamment grands). C’est
une contradiction.
On en déduit que tout nombre premier divise soit a, soit b. Mais cela est une absurdité
sans nom ! Les entiers a et b n’existent donc tout simplement pas.

Autre solution. Cette autre solution, bien plus longue, a l’avantage de ne pas faire appel
au théorème de Dirichlet.

Un couple (a, b) vérifiant les conditions de l’énoncé sera dit convenable. On suppose, par
l’absurde, qu’il existe au moins un couple convenable, disons (a, b).
Lemme. Si (x, y) est convenable alors x et y sont distincts et premiers entre eux et tout
diviseur premier de x (resp. de y) est inférieur à 1000.
Prouvons le lemme. Si x = y alors, pour tous p et q premiers distincts et supérieurs à
1000 le nombre xp + yq = x(p + q) est pair et supérieur à 1000. Il ne peut donc pas être
premier. Donc x et y sont distincts.
Si p est un diviseur premier de x supérieur à 1000 alors pour tout q premier, distinct de
p et supérieur à 1000, on a xq + yp = k, avec k premier et k > p. Or p divise k, ce qui est
absurde. Donc, ni x ni y n’a de diviseur premier supérieur à 1000.
Si d est un diviseur premier commun à x et y. Alors, d’après le point précédent, on a
d < 1000. Soient p et q deux nombres premiers distincts supérieurs à 1000. Alors d divise
xp + yq qui est premier et supérieur à 1000 et donc à d, ce qui est impossible. Donc x et y
sont premiers entre eux.
Le lemme est donc acquis. On prouve maintenant que, si l’on définit les suites (an ) et
(bn ) par a0 = a, b0 = b et, pour tout entier n > 0 :

an+1 = a2n + b2n et bn+1 = 2an bn

133
alors pour tout n > 0, le couple (an , bn ) est convenable.
Bien entendu, il suffit de prouver que (a1 , b1 ) est convenable, puisqu’alors une récurrence
immédiate conduira impitoyablement à la conclusion désirée. Soient p et q deux nombres
premiers distincts et supérieurs à 1000. On a ap + bq = k1 et aq + bp = k2 où k1 et k2 sont
premiers et supérieurs à 1000. De plus, puisque k1 − k2 = (a − b)(p − q), on déduit du lemme
que k1 6= k2 . Mais alors ak1 + bk2 = k3 avec k3 premier, soit (a2 + b2 )p + 2abq = k3 . D’où
(a2 + b2 , 2ab) est convenable.
On note que, pour tout n, on a an+1 , bn+1 > 2 et donc que an+1 > an . Ceci assure entre
autre que les couples (an , bn ) sont deux à deux distincts, et donc que l’on a ainsi construit
une infinité de couples convenables.
D’autre part, pour tous i 6= j, les nombres ai et aj sont premiers entre eux. En effet, on
a clairement :
bj = 2j ba0 a1 · · · aj−1
Or, d’après le lemme, on sait que aj est premier avec bj . Par suite aj est premier avec tous
les ak où k < j.
Pour conclure, on choisit pn un diviseur premier de an . D’après ce qui précède, la suite
(pn )n>1 est une suite infinie de nombres premiers deux à deux distincts. Mais d’après le
lemme, les pn sont tous inférieurs à 1000, ce qui est absurde.

Solution de l’exercice 152 : Pour des raisons pratiques, on commence par traiter à part les
cas n = 1 et n = 2 qui sont évidents. On suppose donc à partir de maintenant que n > 3.
Soit p un nombre premier. Nous allons montrer que :
Ãn−1 !
Y
n k
vp (n!) 6 vp (a − a )
k=0

La formule de Legendre nous donne une formule pour vp (n!) :


· ¸ · ¸ · ¸
n n n n n n n
vp (n!) = + 2 + 3 + ··· 6 + 2 + 3 + ··· =
p p p p p p p−1

On en déduit, puisque vp (n!) est un entier, que :


· ¸
n
vp (n!) 6 (6)
p−1

Réécrivons le produit dont on veut calculer la valuation sous la forme suivante :


n−1
Y n−1
Y n−1
¡ k ¡ n−k ¢¢ n(n−1) Y ¡ ¢
n k
P = (a − a ) = a a −1 =a 2 an−k − 1
k=0 k=0 k=0

On distingue deux cas :


– si vp (a) > 0, alors le premier facteur dans l’écriture précédente prouve que vp (P ) >
n(n−1)
2
. Comme n > 3, on obtient vp (P ) > n et donc d’après la majoration (6), on a
bien vp (P ) > vp (n!) comme on le voulait ;

134
– si vp (a) = 0, les entiers a et p sont premiers entre eux, et donc d’après le petit théorème
n−k
de Fermat, tous les entiers k tels que n − k est multiple de p − 1 sonth tels i que a −1
n−k n
est multiple de p, ou encore que vp (a − 1) > 1. Comme il y a p−1 tels entiers k,
h i
n
il vient vp (P ) > p−1 et donc la conclusion encore par la majoration (6).

Solution de l’exercice 153 : On se rappelle que si S (n) désigne la somme des chiffres de n,
on a S (n) ≡ n (mod 9). Le nombre cherché ici est donc congru à 44444444 modulo 9.
Calculons donc 44444444 modulo 9. Déjà il est congru à 74444 . Les premières puissances
de 7 modulo 9, sont :

70 ≡ 1 (mod 9) ; 71 ≡ 7 (mod 9) ; 72 ≡ 4 (mod 9)

73 ≡ 1 (mod 9) ; 74 ≡ 7 (mod 9)
et la suite ainsi obtenue est périodique de période 3. Comme 4444 est congru à 1 modulo 3,
on a finalement :
44444444 ≡ 7 (mod 9)
Maintenant, on remarque que 44444444 a 1 + 4444 log10 (4444) < 16212 chiffres. Ainsi
S (44444444 ) 6 9 × 16211 = 145899. Ainsi S ◦ S (44444444 ) < 1 + 9 × 5 = 46, puis S ◦ S ◦
S (44444444 ) < 4 + 9 = 13.
Il ne reste plus qu’une possibilité : le nombre cherché est 7.

Solution de l’exercice 154 : Notons x = 2a + 1 et y = 2b + 1, qui sont des entiers impairs. La


condition est équivalente à 2n divise xn + y n . Si n est pair, xn et y n sont congrus à 1 modulo
4 et donc la somme n’est pas un multiple de 4. On en déduit que n vaut forcément 2.
Si n est impair, on factorise :
£ ¤
xn + y n = (x + y) xn−1 − xn−2 y + · · · + y n−1

Le facteur entre crochets est une somme de n nombres impairs, il est donc impair. On en
déduit que 2n divise x + y, ce qui ne peut se produire que pour un nombre fini de n.

Solution de l’exercice 155 : Commençons par prouver l’énoncé donné en indication. Soit p
un diviseur premier impair de n2 + 1. Ainsi on a la congruence n2 ≡ −1 (mod p) et en
élevant à la puissance p−1
2
, on obtient :
p−1
np−1 ≡ (−1) 2 (mod p)

D’autre part, n est évidemment premier à p et donc d’après le petit théorème de Fermat,
np−1 ≡ 1 (mod p). Cela implique p−1 2
pair et donc p ≡ 1 (mod 4).
Nous voulons considérer un diviseur premier impair de n2 + 1. Montrons que cela existe
bien. Si ce n’était pas le cas, on aurait n2 + 1 = 2v pour un certain entier v. De n > 1, on
déduit v > 2 et donc n2 ≡ −1 (mod 4). Mais cette dernière congruence ne peut pas être
réalisée. Un tel diviseur premier p existe donc bien et d’après ce qui précède, la fraction
k = p−1 2
est un entier pair.

135
On en déduit directement (en utilisant p − x ≡ −x (mod p)) que :

(p − 1)! ≡ (k!)2 (mod p)

D’après le théorème de Wilson, ces nombres sont congrus à −1 modulo p et donc :

(n − k!) (n + k!) = n2 − (k!)2 ≡ 0 (mod p)

L’un des deux facteurs du membre de gauche est alors un multiple de p = 2k + 1 ; cette
remarque termine l’exercice.

Solution de l’exercice 156 : Le nombre que l’on considère :


(nm)!
m!(n!)m

est le nombre de façons de partitionner l’ensemble à nm éléments E = {1, 2, . . . , mn} en m


parties à n éléments, et c’est donc en particulier un entier. Pour le voir, on peut remarquer
que chaque permutation σ de E fournit une telle partition {E1 , . . . , Em } en posant :

Ek = {σ(kn − k + 1), . . . , σ(kn − 1), σ(kn)}

et bien sûr toutes les partitions sont obtenues de cette façon. En outre, si l’on permute les
éléments de chacun des ensembles Ek ou que l’on permute les m ensembles Ek entre eux, on
obtient la même partition. Une partition donnée correspond donc à m!(n!)m permutations
distinctes de E exactement. Comme il y a (nm)! permutations de E en tout, cela conclut.

Solution de l’exercice 157 : On commence par prouver que si le résultat est vrai pour n et
m alors il est vrai pour nm. On considère donc 2nm − 1 entiers. D’après notre hypothèse, si
l’on en choisit 2n − 1 quelconques parmi ces entiers, on peut en trouver n dont la somme s1
est divisible par n. On écarte ces n entiers et on recommence avec les 2nm − 1 − n restants.
On forme ainsi 2m − 1 groupes de n entiers, dont les sommes respectives s1 , · · · , s2m−1 sont
toutes divisibles par n. Soit d le pgcd de m et n. Pour tout i, on pose alors di = sdi . Par
hypothèse, parmi les 2m − 1 nombres di , on peut en trouver m dont la somme est divisible
par m, disons d1 , · · · , dm . Cela assure que la somme des mn nombres entiers de départ qui
appartiennent aux groupes si , où i = 1, . . . , m, est divisible par mn, ce qui prouve bien que
le résultat est vrai pour mn.
On en déduit qu’il suffit de prouver le résultat pour n = p premier. Considérons dans ce
cas de entiers a1 , · · · , a2p−1 . Par l’absurde, supposons que l’on ne puisse en trouver p dont
la somme soit divisible par p. Soit alors :
X
S= (ai1 + · · · + aip )p−1

où la somme porte sur toutes les parties {i1 , . . . , ip } à p éléments de {1, 2, · · · , 2p − 1}.
Notons qu’il y a exactement t = Cp2p−1 telles parties et qu’alors :

(2p − 1)(2p − 2) · · · (p + 1)
t= 6≡ 0 (mod p)
(p − 1)!

136
D’après le petit théorème de Fermat, on a donc S = 1 + 1 + · · · + 1 = t 6≡ 0 (mod p).
D’autre part, si l’on développe chaque terme (ai1 + · · · + aip )p−1 de la somme S, on
e ei
obtient une somme de termes de la forme ai1i1 · · · aipp , où les eij sont des entiers positifs ou
nuls dont la somme est égale à p − 1. Intéressons-nous à un terme fixé de cette forme, disons
ae11 · · · aerr , où r représente donc le nombre de ai qui apparaı̂ssent avec un exposant non nul.
Ce terme apparaı̂t dans S exactement autant de fois que l’on peut choisir p − r termes parmi
2p − 1 − r (ceux qui ont au contraire un exposant nul dans le développement). Le coefficient
de ae11 · · · aerr est donc égal à :

(2p − 1 − r) · · · p · · · (p − r + 1)
Cp−r
2p−1−r = ≡ 0 (mod p)
(p − r)!

Donc, tous les coefficients sont divisibles par p, ce qui assure que S elle-même est divisible
par p. Nous avons la contradiction désirée, et le résultat demandé en découle.

Solution de l’exercice 158 : On voit facilement que le k-ième enfant qui reçoit un bonbon est
celui qui est numéroté k(k+1)
2
modulo n. Il s’agit donc de savoir si tout résidu modulo n peut
k(k+1)
s’écrire sous la forme 2 .
Nous allons montrer que cela n’est vrai que si n est une puissance de 2. Soit un entier n
et supposons que n admette un diviseur premier p > 3. Si tout résidu modulo n s’écrit sous
la forme k(k+1)
2
, il en est de même de tout résidu modulo p. Comme p est impair, il existe
un entier d (par exemple d = p+1 2
) tel que 2d ≡ 1 (mod p) et :

k (k + 1)
≡ dk (k + 1) (mod p)
2
Si k ≡ 0 (mod p), la quantité précédente est nulle. Et il en est de même si k ≡ −1 (mod p).
Il est dans ces conditions impossible que toute valeur soit atteinte par cette formule. Cela
est une contradiction, et donc si tout résidu modulo n est de la forme k(k+1)
2
, n est forcément
une puissance de 2.
Réciproquement, on procède par récurrence, en s’inspirant du lemme de Hensel, sur
l’exposant de la puissance de 2. Pour n = 2 ou n = 4, on vérifie facilement à la main.
Passons à l’hérédité. Donnons-nous un entier a. On cherche à construire k tel que :

k (k + 1)
≡ a (mod 2m )
2
ou encore :
k (k + 1) ≡ 2a (mod 2m+1 )
On sait par hypothèse de récurrence, qu’il existe un entier k 0 tel que :

k 0 (k 0 + 1) ≡ 2a (mod 2m )

Cherchons k sous la forme k 0 + 2m r. On calcule :

k (k + 1) = k 2 + k = k 02 + 2m+1 k 0 r + 22m r2 + k 0 + 2m r ≡ k 0 (k 0 + 1) + 2m r (mod 2m+1 )

137
Or, par choix de k 0 , il existe un entier r0 tel que k 0 (k 0 + 1) = 2a + 2m r0 . Il suffit donc pour
conclure de prendre r0 = −r, ou r0 = 2t − r pour t suffisamment grand si l’on veut un entier
k positif.

Autre solution. On indique ici une autre solution plus conceptuelle pour la seconde partie de
la preuve. On a besoin pour cela de la définition et des propriétés de Z/nZ qui sont détaillées
dans la seconde partie du cours (chapitre 5). On suppose que n = 2m est une puissance de
2 et on considére l’application suivante :

f: Z/2m+1 Z → Z/2m Z
x 7→ x(x+1)
2

Noter que l’on est obligé de choisir 2m+1 pour l’ensemble de départ : l’application n’est pas
correctement défini si l’on prend 2m : à cause de la division par 2, la valeur de f (x) ne
dépend pas que de la classe de x modulo 2m , mais modulo 2m+1 .
Supposons que f (x) = f (y), alors directement :

x (x + 1) ≡ y (y + 1) (mod 2m+1 )

ce qui signifie que (x − y) x + y + 1 est un multiple de 2m+1 . Si x − y est pair, x + y + 1 est


impair, et donc on peut simplifier dans Z/2m+1 Z par ce facteur. Ainsi x = y dans Z/2m+1 Z.
Si maintenant x − y est impair, alors on peut simplifier par x − y et on obtient x + y + 1 = 0
dans Z/2m+1 Z.
Ce qui précède entraı̂ne que tout élément de Z/2m Z admet au plus deux antécédents par
f . Par des considérations simples de cardinalité, on voit qu’il en admet en fait exactement
deux. En particulier, toutes les valeurs de Z/2m Z sont atteintes.

Solution de l’exercice 159 : On raisonne par récurrence sur l’entier n. On vérifie à la main
pour n 6 4. Supposons le résultat vrai pour tout k < n pour un certain entier n quelconque
mais néanmoins fixé. Si n est pair, c’est évident. Si n est impair, notons A (resp. B) le
produit des nombres premiers inférieurs ou égaux à n+1
2
(resp. supérieurs strictement à n+1
2
et inférieurs ou égaux à n).
D’après l’hypothèse de récurrence, on a :
n+1
A64 2

n+1
D’autre part, chaque nombre premier apparaissant dans B divise Cn 2 et donc B divise ce
coefficient binomial et en particulier est plus petit. On en déduit :
n+1 n+1
AB 6 4 2 Cn 2

Par ailleurs, on a :
n
X n
X
n+1
2Cn 2 6 2 Ckn = Ckn = 2n
k= n+1
2
k=0

On en déduit le résultat voulu.

138
Solution de l’exercice 160 : On remarque dans un premier temps que si n est impair, 2n ≡ 2
(mod 3) et donc si on choisit pour k ≡ 1 (mod 3), on aura k2n + 1 ≡ 0 (mod 3) qui ne
pourra donc jamais être premier (du moins si k > 1). On a ainsi réglé le cas n impair.
Si n est pair, on aimerait trouver un nombre modulo N premier avec 3 pour faire la même
manipulation. Il serait intéressant de le choisir tel que 2 soit d’ordre 2 modulo N . Seulement
cela signifie que 22 = 4 ≡ 1 (mod N ) et donc N divise 3. Ce n’est pas bon. On choisit donc
un modulo (premier) pour lequel 2 est d’ordre 4. Ces modulos sont les diviseurs premiers de
24 − 1. On choisit p2 = 5 (car on ne peut pas prendre 3). Ainsi, si n ≡ 3 (mod 4), on aura :

2n ≡ 23 (mod p2 )

et puisque p2 est un nombre premier impair, il existe un entier k2 tel que si k ≡ k2 (mod p2 ),
on ait k2n +1 multiple de p2 . Choisissant k ainsi, on règle le problème des puissances congrues
à 3 modulo 4.
Mais il en reste encore. Et on ne peut plus prendre de modulo N pour lesquels 2 est
d’ordre 4 (car 24 − 1 = 3 × 5 et 3 et 5 ont déjà été choisis). On cherche donc des modulos
pour lesquels 2 est d’ordre 8, c’est-à-dire des diviseurs de 28 − 1 = 3 × 5 × 17. On choisit
p3 = 17 et comme précédemment, on obtient un k3 qui reglèra le cas des puissances congrues
à 7 modulo 8.
On remarque quand même que :
n
22 − 1 = F1 F2 · · · Fn−1
k
où Fk = 22 +1 désigne le k-ième nombre de Fermat. Ainsi si Fn−1 est premier, il restera tou-
jours un cas qui nous échappe. Mais si Fn−1 fait intervenir deux nouveaux facteurs premiers,
on sera sauvé.
Précisément, comme précédemment on obtient des nombres premiers p4 = 257, p5 =
65537 et des entiers k4 et k5 tels que pour tout i compris entre 1 et 5 (avec p1 = 3 et k1 = 1),
k ≡ ki (mod pi ) et n ≡ −1 (mod 2i ), on ait pi divise k2n + 1. Lorsque l’on arrive à p6 , on
se rend compte que F5 est composé et fait intervenir deux nouveaux facteurs premiers p6 et
p06 . On choisit alors k6 et k60 de sorte que :
– pour tout k ≡ k6 (mod p6 ) et tout n ≡ −1 (mod 26 ), on ait p6 divise k2n + 1
– pour tout k ≡ k60 (mod p06 ) et tout n ≡ 25 − 1 (mod 26 ), on ait p06 divise k2n + 1
Finalement si l’on choisit k vérifiant k ≡ ki (mod pi ) et k ≡ k60 (mod p06 ), l’entier k2n +
1 sera toujours divisible soit par l’un des pi , soit par p06 . On peut finalement choisir k
suffisamment grand pour que le nombre premier qui apparaı̂t ne soit pas le seul facteur.
Cela termine l’exercice.

Solution de l’exercice 161 : Montrons tout d’abord que n doit être une puissance de 2. En
effet, supposons par l’absurde qu’un n autre qu’une puissance de 2 convienne, et soit m un
entier tel que 2n − 1 divise m2 + 9. Alors n possède un diviseur impair ` > 3, et comme
2` − 1 divise 2n − 1, on a aussi m2 ≡ −9 (mod 2` − 1). Or 2` − 1 ≡ −1 (mod 4), donc 2` − 1
possède un diviseur premier p ≡ −1 (mod 4). Si p 6= 3, on a, d’après le petit théorème de
Fermat :

1 ≡ mp−1 ≡ (m2 )(p−1)/2 ≡ (−9)(p−1)/2 ≡ (−1)(p−1)/2 3p−1 ≡ −1 (mod p)

ce qui est absurde. Et si p = 3, 2` ≡ (−1)` 6≡ 1 (mod 3), d’où encore une contradiction.

139
Si n est solution, il doit donc être une puissance de 2. On va voir qu’en fait, la réciproque
est vraie. Soit n = 2k . On a :
2 k−1
2n − 1 = 3(22 + 1)(22 + 1) · · · (22 + 1)
`
Or les 22 + 1 pour ` = 0, . . . , k − 1 sont deux à deux premiers entre eux. En effet, si a > b,
a b
soit d le pgcd de 22 + 1 et 22 + 1. Alors d est impair, et l’on a :

a
³ b ´2a−b
−1 ≡ 22 ≡ 22 ≡ 1 (mod d)

donc d = 1. D’après le théorème chinois, il existe donc un entier c tel que l’on ait :
`−1 `
c ≡ 22 (mod 22 + 1)

pour ` = 1, . . . , k − 1. Alors 2` − 1 divise c2 + 1 pour ` = 1, . . . , k − 1, et donc 2n − 1 divise


(3c)2 + 9.
Finalement, les entiers cherchés sont exactement les puissances de 2.

Solution de l’exercice 162 : Face à une telle question, il est assez naturel de penser à poser
ai = 2pi + bi , où bi ∈ {0, 1, · · · , p − 1} vérifiant bi ≡ i2 (mod p), pour i = 1, 2, · · · , p. On
vérifie facilement que ces nombres sont bien deux à deux distincts et que ap = 2p2 et que
0 < ai 6 2p(p − 1) + p − 1 < 2p2 pour tout i < p. De plus, pour tous i et j on a :
· ¸
ai + aj
=i+j (7)
2p

Soient 1 6 i, j, k, l 6 p des entiers tels que i 6= j, k 6= l et ai + aj = ak + al . De (7), on déduit


que i + j = k + l. Par suite bi + bj = bk + bl , c.à.d. i2 + j 2 ≡ k 2 + l2 (mod p). Mais alors,
toujours d’après (7), on a :

(i − k)(i + k) ≡ (l − j)(l + j) ≡ (i − k)(l + j) (mod p) (8)

Si i = k alors j = l. Et si i 6= k, alors −(p − 1) 6 i − k 6 p − 1 avec i − k 6= 0, ce qui assure


que i − k est premier avec p. De (8), il vient alors i + k ≡ j + l (mod p). En réutilisant (7), il
vient alors k = j et i = l. Ainsi, dans tous les cas, si ai + aj = ak + al c’est que (i, j) = (k, l)
ou (i, j) = (l, k). Et donc les sommes ai + aj pour i < j sont bien deux à deux distinctes.

Solution de l’exercice 163 : Déjà, n est forcément impair. Supposons n > 2. Soit :

n = pα1 1 · · · pαk k

la décomposition en nombres premiers de n (on a alors k > 1). Les pi sont impairs. On pose
pi = 2a1 b1 + 1 où les bi sont des entiers impairs. On peut supposer que a1 est le plus petit
des ai . On a alors :
n = (2a1 b1 + 1)α1 · · · (2ak bk + 1)αk
On en déduit que n ≡ 1 (mod 2a1 ).

140
D’autre part, p1 divise 2n−1 + 1, et donc 22n−2 ≡ 1 (mod p1 ). Soit g l’ordre de 2 modulo
p1 , il divise 2n − 2 et d’après le petit théorème de Fermat, g divise également p1 − 1. Par
contre, g ne divise pas n − 1 car sinon on aurait 2n−1 ≡ 1 (mod p1 ) et donc p1 = 2. Donc :

v2 (g) = 1 + v2 (n − 1) > 1 + a1 > a1 = v2 (p1 − 1)

ce qui est incompatible avec g divise p1 − 1.


Finalement n = 1 est la seule solution.

Solution de l’exercice 164 : Soit s le nombre minimal de chiffres non nuls (en base b) d’un
multiple strictement positif de bn − 1. Parmi tous les multiples de bn − 1 ayant s chiffres non
nuls, on en choisit un, disons A, dont la somme des chiffres est minimale. Posons :

A = a1 bn1 + . . . + as bns

avec n1 > · · · > ns . Montrons que les ni sont deux à deux distincts modulo n. Supposons
par l’absurdes qu’il existe des indices i et j distincts tels que ni ≡ nj (mod n). Notons r un
entier strictement supérieur à n1 et congru modulo n à ni et nj . On a alors br ≡ bni ≡ bnj
(mod bn − 1) et donc le nombre :

B = A + (ai + aj ) br − ai bni − aj bnj

est un multiple de bn − 1. Si ai + aj < b, ce nombre a s − 1 chiffres non nuls, ce qui contredit


la minimalité de s. Donc b 6 ai + aj < 2b. Si l’on note a = a1 + · · · + as la somme des chiffres
de A, celle de B vaut :

a − ai − aj + 1 + (ai + aj − b) = a − b + 1 < a

ce qui contredit la minimalité de a.


Maintenant que l’on sait que les ni sont deux à deux distincts modulo n, on en déduit
que s 6 n (ce qui n’est pas du tout ce que l’on veut). On va montrer qu’en réalité s = n.
Notons ri le reste de la division euclidienne de ni par n. Soit :

C = a1 br1 + . . . + as brs

qui est alors un multiple non nul de (bn − 1). Puisque les ri sont deux à deux distincts, ce
qui précède est l’écriture en base b de C. On a donc un multiple de bn − 1 avec pas plus de
n chiffres (éventuellement nuls). La seule possibilité est donc d’avoir C = bn − 1 et dans ce
cas l’unicité de la décomposition en base b assure que s = n (et ai = b − 1 pour tout i).

Solution de l’exercice 165 : La progression arithmétique est constituée des entiers de la forme
ak + b pour k entier positif.
Supposons qu’il existe des entiers x et y tels que x2 et y 3 soit de cette forme. Alors
x6 ≡ b3 (mod a) et y 6 ≡ b2 (mod a). Si y est premier avec a (ce qui équivaut à : si b premier
avec a, car tout nombre premier divisant b et a divise y, et tout diviseur commun de y et a
divise b), d’après Bézout il existe u tel que uy ≡ 1 (mod a) avec u > 0. Alors u6 y 6 x6 ≡ b3
(mod a), soit (ux)6 ≡ b (mod a), et le problème est résolu... dans ce premier cas.
Mais la difficulté, c’est lorsque b et a ne sont pas premiers entre eux. Et une idée astucieuse
est de construire une récurrence sur a. L’hypothèse de récurrence s’énonce ainsi : si une

141
progression arithmétique infinie de raison a 6 n contient un carré et un cube, elle contient
une puissance sixième. Pour n = 1, c’est évident. C’est presque aussi évident pour n = 2,
puisqu’une progression arithmétique de raison 2 contient tous les nombres pairs ou tous les
nombres impairs à partir d’un certain rang. Supposons que ce soit vrai pour n, et montrons
que si une progression arithmétique infinie de raison a = n + 1 contient un carré et un cube,
elle contient une puissance sixième. En se limitant au cas où b et a ne sont pas premiers entre
eux : appelons d leur pgcd et p un facteur premier de d, pi étant la plus grande puissance
de p divisant d. Deux cas à envisager : soit p ne divise pas ad , soit p ne divise pas db , car si p
divise ad et db , d n’est pas le pgcd de a et b.
Si p ne divise pas ad , les congruences x2 ≡ b (mod a) et y 3 ≡ b (mod a) entraı̂nent, a
fortiori x2 ≡ b (mod pai ) et y 3 ≡ b (mod pai ). Mais pai 6 n, donc, d’après l’hypothèse de
récurrence, il existe z tel que z 6 ≡ b (mod pai ). Et d’après le théorème chinois, il existe t > b
tel que :
t ≡ z (mod pai )
t ≡ 0 (mod pi )
a
puisque pi est premier avec pi
. Il en résulte :

t6 ≡ z 6 ≡ b (mod pai )
t6 ≡ 0 ≡ b (mod pi )
puisque b est divisible par d, donc par pi . L’entier (t6 − b) est divisible par pai et par pi , donc
par a puisque pai est premier avec p. Donc t6 convient.
Si p divise ad mais ne divise pas db , alors pi+1 divise a mais pas b, donc tous les termes
ak + b de la progression arithmétique sont divisibles par d donc par pi , mais aucun n’est
divisible par pi+1 . En particulier, la plus grande puissance de p divisant x2 est pi (ce qui
prouve que i est pair), et la plus grande puissance de p divisant y 3 est également pi (ce qui
prouve que i est divisible par 3). L’entier i est donc multiple de 6 : i = 6j. Et l’on a x
³ ´2 ³ ´
divisible par p3j : x2 − b étant divisible par a, px3j − pb6j est divisible par pa6j . De même,
³ ´3
y est divisible par p2j , et py2j congru à pb6j modulo pa6j . Comme pa6j 6 n, l’hypothèse de
b a
récurrence s’applique : il existe z tel que z 6 congru à p6j
modulo p6j
. Si la différence z 6 − pb6j
a 6
est divisible par p6j
, le nombre (zpj ) − b est divisible par a, on a donc trouvé une puissance
6
sixième, en l’occurrence (zpj ) , dans la suite arithmétique. Ce qui achève la démonstration.

Solution de l’exercice 166 : On procède par l’absurde en supposant qu’il n’existe qu’un
nombre fini de tels n. En particulier, il existe un rang N à partir duquel tous les a(n)
sont des puissances d’un nombre premier. On note par ailleurs h(n) = a(n)b(n)
.
On commence par remarquer que pour tout n, b(n) > n/2. En effet, si l’on note k le plus
grand entier tel que 2k 6 n, il n’y a dans la somme 1 + 1/2 + · · · + 1/n qu’un et un seul
terme 1/m avec m divisible par 2k , ce qui montre que 2k > n/2 divise b(n).
Soit alors p un nombre premier impair plus grand que N + 1. D’après l’exercice 5.3, p
divise a(p − 1), et donc que a(p − 1) est une puissance de p. Mais si l’on avait a(p − 1) = p,
p
il viendrait h(p − 1) < (p−1)/2 = 2 + 2/(p − 1) 6 3. En choisissant p assez grand10 pour que
10
C’est possible, puisque h(n) tend vers +∞ quand n tend vers +∞.

142
h(p − 1) > 3, on a donc nécessairement a(p − 1) > p, et donc a(p − 1) est une puissance de
p au moins égale à p2 .
p étant toujours choisi supérieur à N + 1 et tel que h(p − 1) > 3, on peut ensuite montrer
par récurrence que a(pk − 1) est une puissance de p au moins égale à p2 pour tout k > 1.
En effet, supposons le résultat pour un certain k. Alors on peut écrire :
pk −1
X 1 X 1
k+1
h(p − 1) = +
m=1
pm m
16m6pk+1 −1
p6|m
k
p −1 p−1
h(pk − 1) X X 1
= +
p q=1 r=1
pq + r

Comme a(pk − 1) est divisible par p2 au moins, le numérateur du premier terme est divisible
par p et en appliquant à nouveau le résultat de l’exercice 5.3 (en remarquant que le numé-
1
rateur de la fraction pq+r − 1r est un multiple de p, et que son dénominateur est premier à
p), on trouve que p divise a(pk+1 − 1) et donc que a(pk+1 − 1) est une puissance de p. Puis,
par le même argument que précédemment, la condition h(pk+1 − 1) > h(p − 1) > 3 assure
que ce n’est pas exactement p, ce qui achève la récurrence.
Par ailleurs, pour tout k > 2, on a :
p−1
X 1
h(pk − p) = h(pk − 1) +
r=1
r − pk

donc a(pk − p) est aussi divisible par p.


Choisissons alors un entier n tel que pn ne divise pas le numérateur de 1 + 1/2 + · · · +
1/(p − 1), et un entier k > n tel que pk − p > 2pn . Alors a(pk − p) est une puissance de p
telle que a(pk − p) > b(pk − p) > (pk − p)/2 > pn , donc pn divise a(pk − p), et aussi a(pk − 1)
pour la même raison. Donc il divise le numérateur de la différence h(pk − p) − h(pk − 1),
c’est-à-dire de :
1 1 1
k
+ k
+ ··· +
1−p 2−p p − 1 − pk
Mais le numérateur de cette fraction est congru modulo pk à celui de la fraction :
1 1 1
1+ + + ··· +
2 3 p−1
mais pn ne divise pas ce numérateur par définition de n. C’est une contradiction et le résultat
s’ensuit.

5.4 Exercices de « Équations diophantiennes »


Solution de l’exercice 167 : a) Supposons que n (n + 1) soit un carré parfait. Comme n et
n + 1 sont premiers entre eux, ils doivent être tous les deux des carrés parfaits Mais ceci
n’est pas possible pour des carrés non nuls.

143
Remarque. La même démonstration s’applique pour prouver que le produit de deux entiers
consécutifs n’est jamais une puissance parfaite.

b) Supposons que (n − 1) n (n + 1) soit un carré parfait. Comme n et (n − 1) (n + 1) =


2
n − 1 sont premiers entre eux, ils doivent être tous les deux des carrés parfaits. Mais pour
n2 − 1 soit un carré parfait, il faut n = ±1 et alors le produit (n − 1) n (n + 1) est nul.

c) L’expression (n − 1) n (n + 1) (n + 2) est de degré 4 et donc sa racine carrée doit être


proche d’une expression de degré 2. Plus précisément, on vérifie que l’on a la formule :
¡ ¢2
(n − 1) n (n + 1) (n + 2) = n2 + n − 1 − 1

On obtient à nouveau deux carrés qui différent de 1 ; à nouveau la seule possibilité est que
le produit soit nul.

Solution de l’exercice 168 : On applique pas à pas la méthode du cours. On commence par
remarquer que x = 1, y = 0 est une solution évidente. On introduit donc un rationnel t tel
que y = t (x − 1). L’équation devient :

x2 + 3t2 (x − 1)2 = 1
2
Les solutions sont x = 1 et x = 3t −1
3t2 +1
. Le y correspondant vaut y = − 3t22t+1 .
Finalement, les solutions sont les couples :
µ 2 ¶
3t − 1 2t
,−
3t2 + 1 3t2 + 1

pour t décrivant l’ensemble des nombres rationnels et le couple (1, 0).

Solution de l’exercice 169 : a) La condition implique que a doit être une puissance de 5,
donc a = 5k . L’équation se réécrit alors 2k = n, ce qui implique que n doit être pair. Ainsi
il y a une solution pour tout entier n pair, le a correspondant étant a = 2n/2 .
b) On peut factoriser l’expression fournit et obtenir :

5n = (a − 1) (a + 1)

Ainsi les deux nombres a − 1 et a + 1 doivent être des puissances cinquièmes. Or il n’existe
pas deux puissances cinquièmes qui diffèrent de 2. L’équation n’a pas de solution.
c) En regardant modulo 4, on obtient a2 ≡ 3 (mod 4) puisque 5 ≡ 1 (mod 4). Or on sait
qu’un carré modulo 4 ne peut être congru qu’à 0 ou 1. L’équation n’a donc pas de solution
non plus.

Solution de l’exercice 170 : Quitte à multiplier tous les xi par un même facteur, on peut
supposer que tous les xi sont entiers. Quitte à permuter les xi , on peut supposer que x1 est
le plus petit d’entre eux. Posons yi = xi − x1 > 0. On a y1 = 0 et la famille des yi vérifient
la même hypothèse que la famille des xi .

144
Soit 2 6 i 6 2002 un entier. Notons I un sous-ensemble de {2, . . . , 2002} de cardinal
7 contenant i et I 0 l’ensemble I obtenu en remplaçant i par 1. Par hypothèse, il existe des
ensembles J et J 0 de cardinal 11 tels que :
X X X X
11 yi = 7 yj et 11 yi = 7 yj
i∈I j∈J i∈I 0 j∈J 0

En soustrayant ces deux égalités, on voit que 11yi est un multiple de 7 et donc d’après le
lemme de Gauss qu’il en est de même de yi . Ceci est vrai pour tout i.
La famille constitué des y7i est encore solution du problème. Le principe de descente
infinie assure que l’unique solution est alors yi = 0 pour tout i. Cela entraı̂ne bien que tous
les xi sont égaux.

Solution de l’exercice 171 : On trouve en premier lieu une solution particulière : x0 = 35 ,


y0 = 45 et z0 = 65 .
Comme un dénominateur commun de x30 , y03 et z03 est 125, si on multiplie x30 , y03 et z03 par
un nombre congru à 1 modulo 125, on ne changera pas la partie décimale. On cherche donc
des cubes congrus à 1 modulo 125. Les nombres de la forme (125k + 1)3 s’imposent.
On est finalement amené à considérer les nombres :

x = x0 (125k + 1) ; y = y0 (125k + 1) ; z = z0 (125k + 1)

dont il est facile de vérifier qu’ils conviennent.

Solution de l’exercice 172 : Soit x = 12m − 5n le minimum cherché. On a x ≡ −5n (mod 6),
donc x n’est divisible ni par 2, ni par 3. De même x n’est pas divisible par 5. Par conséquent,
on a x = 1 ou x > 7 = 12 − 5. Il reste donc à exclure le cas x = 1. Pour cela, on peut
remarquer que :
12m − 5n ≡ −1 6≡ 1 (mod 4)
Finalement, on a x = 7.

Solution de l’exercice 173 : La somme de tous les carrés modulo 9 vaut 9·(9+1)(18+1)
6
≡ 3·5·1 ≡
6 (mod 9). La somme des carrés de 99 entiers consécutifs x + 1, x + 2, . . ., x + 99 vaut donc
11 × 6 ≡ 3 (mod 9). En particulier, la valuation 3-adique de cette somme est exactement 1,
et ce n’est donc jamais une puissance parfaite.
L’équation proposée dans l’énoncé n’a donc aucune solution.

Solution de l’exercice 174 : Nous partons de la formule :

(t + 1)3 + (t − 1)3 + (−t)3 + (−t)3 = 6t

qui prouve déjà que tout multiple de 6 peut s’écrire comme somme de quatre cubes. Soit n
un entier. On veut réussir à écrire n (d’une infinité de façon) comme somme de cinq cubes.
Mais, d’après ce qui précède, chaque fois que l’on arrive à trouver un cube congru à −n
modulo 6, on obtient une telle écriture. En effet, si l’on a n = x3 + 6t pour des entiers t et
x, on aura de fait :
n = x3 + (t + 1)3 + (t − 1)3 + (−t)3 + (−t)3

145
Or on vérifie facilement que tout résidu modulo 6 est un cube (en réalité, on a a3 ≡ a
(mod 6) pour tout a). On obtient donc bien une infinité de x qui fournissent, par le fait, une
infinité de solutions.

Solution de l’exercice 175 : En prenant le logarithme, l’équation s’écrit aussi y log x = x log y,
soit encore :
log x log y
=
x y
Une étude de fonction montre que la fonction t 7→ logt t est croissante sur l’intervalle [1, e]
et décroissante sur l’intervalle [e, +∞[. Plus visuellement, la résentation graphique de la
fonction est :

1/e

0 1 e

En particulier, si x et y sont deux entiers distincts (disons x < y) tels que xy = y x , on


doit avoir x < e, et donc x = 2, ce qui conduit à y = 4.
Finalement les solutions sont les couples (x, y) avec x = y et les deux couples (2, 4) et
(4, 2).

Solution de l’exercice 176 : Pour commencer, comme 22 + 32 = 13 n’est pas une puissance
parfaite, on peut supposer p impair. On a alors la factorisation :

2p + 3p = 5(2p−1 − 3 · 2p−2 + · · · + 3p−1 )

En particulier, 5 divise 2p + 3p = an , donc 25 aussi. Mais 3 ≡ −2 (mod 5), donc on a :

0 ≡ 2p + 3p = 5 · p2p−1 (mod 25)

Cela impose que 5 divise p, c’est-à-dire que p = 5. Mais 25 + 35 = 275 = 52 · 11 n’est pas
une puissance parfaite.
On a bien montré que 2p + 3p n’était une puissance parfaite non triviale pour aucun
nombre premier p.

Solution de l’exercice 177 : Notons an = 1 + 2! + · · · + n!. On peut remarquer que pour


tout entier n et tout n > N − 1, on a an ≡ aN −1 (mod N !). Ainsi, pour tout n > 2,
an est divisible par 3. Si par chance an n’était pas divisible par 9 pour n assez grand,
on obtiendrait immédiatement que an n’est pas une puissance parfaite. Malheureusement
a5 = 153 est divisible par 9. Mais si l’on poursuit courageusement les calculs jusqu’à a8 , on
trouve que :
a8 = 46233 ≡ 9 (mod 27)
Cela signifie que si pour n > 8, an est une puissance parfaite, alors ça doit être un carré
parfait, puisque sa valuation 3-adique est 2.

146
D’autre part, on peut remarquer que a4 = 33 ≡ 3 (mod 5) n’est pas un carré modulo 5.
Il en résulte avec ce qui précède que l’équation n’a pas de solution pour n > 8.
On peut examiner les cas restants à la main. On a :

a1 = 1 qui est une puissance k-ième pour tout k.


a2 = 3 qui n’est pas une puissance parfaite.
a3 = 9 qui est un carré parfait.
a4 = 33 qui n’est pas une puissance parfaite.
a5 = 153 = 9 · 17 qui n’est pas une puissance parfaite.
a6 = 873 = 9 · 97 qui n’est pas une puissance parfaite.
a7 = 5913 = 81 · 73 qui n’est pas une puissance parfaite.

donc les triplets (m, n, k) solutions sont (1, 1, k) pour tout k > 2 et (3, 3, 2).

Solution de l’exercice 178 : Soit (x, y) une éventuelle solution de l’équation y 2 = x3 + 16, qui
s’écrit encore (y − 4)(y + 4) = x3 . Si y est impair, y − 4 et y + 4 sont premiers entre eux,
et sont donc deux cubes impairs distants de 8, ce qui n’existe pas. Donc y = 2y 0 est pair, et
par suite x = 2x0 aussi. L’équation devient donc :

(y 0 + 2)(y 0 − 2) = 2(x0 )3

En réduction modulo 4, il vient donc (y 0 + 2)2 ≡ 0 ou 2 (mod 4), et donc forcément (y 0 + 2)2
est divisible par 4, et ainsi y 0 est pair. Donc on peut encore réécrire y 0 = 2s, x0 = 2t, et il
vient (s + 1)(s − 1) = 4t3 . Mais alors s + 1 et s − 1 sont pairs, donc s = 2u + 1 est impair,
et l’on obtient finalement :
u(u + 1) = t3
Cela impose que u et u + 1, qui sont premiers entre eux, soient tous les deux des cubes, et
donc u = −1 ou 0 et t = 0.
En remontant, on trouve que l’équation initiale possède exactement deux solutions en-
tières, qui sont (x, y) = (0, ±4).

Solution de l’exercice 179 : L’examen des premiers cas suggère que les couples de nombres
de Fibonacci consécutifs sont solutions : (1, 2), (2, 3), (3, 5), (5, 8), etc. et il est facile de le
vérifier de manière générale. En effet, si (m, n) convient, alors :

(m + n)2 − n(m + n) − n2 = m2 + 2mn + n2 − mn − 2n2 = −(n2 − mn − m2 )

et donc (n, m + n) convient aussi. En calculant les termes de la suite de Fibonacci inférieurs
à 1981 :
1, 2, 3, 5, 8, 13, 21, 34, 55, 89, 144, 233, 377, 610, 987, 1597
on obtient que m2 + n2 peut atteindre 15972 + 9872 . On aimerait que cette valeur soit
effectivement le maximum.
Pour le voir, il suffit d’effectuer l’opération (m, n) 7→ (n, m + n) dans l’autre sens ! En
effet, soit (m, n) une solution avec 1 6 m < n, alors (n − m, m) aussi. En itérant ce procédé,
on se ramène nécessairement à une solution de la forme (1, n). Or pour m = 1 l’éqution
devient :
n2 − n = 0 ou 2

147
dont la seule solution strictement positive est n = 2. Il en résulte que toutes les solutions
sont obtenues à partir de (1, 2) par itération de (m, n) 7→ (n, m + n), donc sont toutes de la
forme (Fn−1 , Fn ).
Finalement, le maximum possible de m2 + n2 est exactement 15972 + 9872 .

Solution de l’exercice 180 : Quitte à soustraire une masse commune à toutes les vaches, on
peut supposer que l’une d’entre elles a une masse nulle. Il s’agit alors de montrer qu’elles
ont toutes une masse nulle.
Pour cela, soit M la somme des masses de toutes les vaches du troupeau. On sait que
la masse m d’une vache quelconque est telle que M − m soit pair, puisque c’est la somme
des masses de deux groupes de vaches de même masse totale. Il en résulte que les masses
de toutes les vaches sont de la parité de M . Comme l’une des vaches a une masse nulle,
toutes les vaches ont une masse paire. Mais alors un troupeau composé de vaches de masses
moitié vérifie encore les hypothèses de l’énoncé, et donc toutes les masses moitié sont encore
paires. La descente infinie qui s’amorce ainsi montre clairement que toutes les masses sont
nécessairement nulles.

Solution de l’exercice 181 : Déjà, si z = 1, on a forcément x = y = 1. Supposons donc z > 2.


Si on n’a pas x = y = z, au moins un des deux nombres parmi x et y doit être strictement
plus grand que z, et donc plus grand ou égal à z + 1.
Si c’est x, on obtient :

2xx > 2 (z + 1)z+1 > 2z z+1 > 4z z

ce qui est absurde.


Si c’est y, on écrit :

y y > (z + 1)z+1 > z z+1 + (z + 1) z z = (2z + 1) z z > 5z z

ce qui est tout aussi absurde.

Solution de l’exercice 182 : On remarque que x = 1, y = −a est toujours solution de l’équa-


tion.
D’autre part, si (x, y) est une solution de l’équation, x est racine du polynôme :
¡ ¢
X 2 + ayX + y 2 − 1 = 0

et l’autre racine de ce polynôme est donc −ay − x. On obtient ainsi une nouvelle solution
qui est le couple (−ay − x, y). De même, on a la solution (x, −ax − y).
D’autre part, si |a| > 2, et si x et y sont non nuls, on a :

min (|−ay − x| , |y|) > min (|x| , |y|) ou min (|x| , |−ax − y|) > min (|x| , |y|)

Supposons dans un premier temps |x| 6 |y|, alors :

|−ay − x| = |ay + x| > |ay| − |x| > |a| |y| − |x| > (|a| − 1) |y| > |y| > min (|x| , |y|)

De même, si |y| 6 |x|, on a :

|−ax − y| > |x| > min (|x| , |y|)

148
Posons alors x0 = 1 et y0 = −a. Grâce à ce qui a été fait précédemment, on construit
une suite de solutions (xn , yn ) telles que :

min (|xn+1 | , |yn+1 |) > min (|xn | , |yn |)

Cette dernière condition assure que toutes les solutions sont deux à deux distinctes. L’équa-
tion admet donc dans ce cas une infinité de solutions.
Si a = 2, l’équation se réécrit (x − y)2 = 1 qui admet également une infinité de solutions.
De même si a = −2, il y a une infinité de solutions. Si a = 1, l’équation fournit x2 +y 2 = 1−xy
et (x + y)2 = 1 + xy. Les nombres 1 − xy et 1 + xy doivent donc être positifs et cela ne peut
se produire pour une infinité de couples d’entiers (x, y). On raisonne de même si a = −1.
Finalement pour a = 0, l’équation n’admet clairement qu’un nombre fini de solutions.
La réponse à la question de l’énoncé est finalement l’ensemble des entiers a tels que
|a| > 2.

Remarque. L’équation se réécrit sous la forme :


³ µ ¶
a ´2 a2
x+ y + 1− y2 = 1
2 4

On voit directement sous cette écriture que si a2 < 4, l’équation ne peut admettre qu’un
nombre fini de solutions. En outre, on reconnaı̂t (du moins lorsque a est pair) une équation
de Pell-Fermat. Le résultat est donc une conséquence de la résolution de cette équation.
Notez cependant que l’existence d’une solution non triviale à cette équation n’est pas une
évidence (et n’a pas été traité dans le cours pour l’instant). Cependant, ici, on dispose de
cette solution non triviale donnée par x = 1 et y = −a.

Solution de l’exercice 183 : Pour A 6 2, on a :

Axy 6 x2 + y 2 < x2 + y 2 + 1

donc l’équation x2 + y 2 + 1 = Axy n’a pas de solution entière. Il s’agit donc de montrer
qu’elle n’en a pas non plus pour A > 4.
En effet, soit (x0 , y0 ) une telle solution. Alors en considérant le second point d’intersection
de la conique avec la droite horizontale (resp. verticale) passant par (x0 , y0 ), on voit que
(x0 , Ax0 − y0 ) (resp. (Ay0 − x0 , y0 )) est encore solution. On peut donc, par descente infinie,
construire une solution (x, y) vérifiant x 6 Ay − x et y 6 Ax − y. Il vient alors :

x2 + y 2 + 1 = Ax · y > 2y 2 et x2 + y 2 + 1 = Ay · x > 2x2

Ainsi, on obtient |x2 − y 2 | 6 1, ce qui impose x = y ou (x, y) = (1, 0) ou (0, 1). On vérifie
aisément que ces différents cas ne fournissent pas de solution à l’équation.
2 2 +1
Finalement, on a bien montré que si x +y xy
est un entier, alors c’est 3.

Solution de l’exercice 184 : Soit (x, y, z) une éventuelle solution non nulle à l’équation x2 +
y 2 = 7z 2 . Si 7 divise x ou y, alors clairement 7 divise x et y, et donc x2 + y 2 est divisible par
72 et finalement 7 divise z. Par conséquent (x/7, y/7, z/7) est encore une solution entière

149
non nulle, et une descente infinie permet donc d’obtenir une solution (x, y, z) avec x et y
premiers à 7. Mais il vient alors :
2
x2 + y 2 ≡ 0 (mod 7) donc (xy 0 ) ≡ −1 (mod 7)

où y 0 est l’inverse de y modulo 7. Mais l’on vérifie facilement que −1 n’est pas un carré
modulo 7.
Finalement, la seule solution de l’équation x2 + y 2 = 7z 2 est la solution nulle (x, y, z) =
(0, 0, 0).

Solution de l’exercice 185 : Soit (a, b, c) 6= (0, 0, 0) une éventuelle solution de l’équation :

a4 + (a + b)4 + b4 = c2

Si a et b sont pairs, alors 24 divise le membre de gauche, donc aussi c2 , et c est multiple de 4.
Alors (a/2, b/2, c/4) est encore solution. Une descente infinie permet alors de supposer que
a ou b est impair.
Si exactement l’un des deux nombres a et b, par exemple a, est pair, on remarque alors
que :
a4 + (a + b)4 + b4 ≡ 0 + 1 + 1 ≡ 2 (mod 4)
ce qui ne saurait être un carré. Par conséquent, on doit avoir a et b impair, mais alors :

a4 + (a + b)4 + b4 ≡ 1 + 0 + 1 ≡ 2 (mod 4)

fournit une nouvelle contradiction.


Finalement, le seul couple (a, b) tel que a4 + (a + b)4 + b4 soit un carré est (0, 0).

Solution de l’exercice 186 : a) On peut supposer a 6 b 6 c. On obtient 41 6 a32 et donc


a2 6 12. Ainsi a = 1, a = 2 ou a = 3. On ne peut évidemment ni avoir a = 1, ni a = 2.
Si a = 3, l’équation devient :
1 1 1 1 5
+ = − =
b2 c2 4 9 36
Comme précédemment, on montre que b2 6 72 5
puis b = 3. Il vient alors c = 6.
Les solutions sont donc les triplets (3, 3, 6), (3, 6, 3) et (6, 3, 3)
b) Comme précédemment, on montre que ni n = 2, ni n = 3 ne conviennent. Pour n = 4,
il suffit de prendre x1 = x2 = x3 = x4 = 2.
On remarque que si (x1 , . . . , xn ) convient alors (2x1 , 2x1 , 2x1 , 2x1 , x2 , x3 , . . . , xn ) convient
également, ce qui montre que si n est solution alors n + 3 l’est aussi.
D’autre part, la solution du a) fournit (2, 2, 2, 3, 3, 6), qui fournit ensuite (2, 2, 2, 3, 3, 9, 9, 18).
Ainsi n = 6 et n = 8 sont solutions. Par suite, tous les entiers autres que 2, 3 et peut-être 5
sont solutions.
Reste à prouver que n = 5 n’est pas une solution. Si (a, b, c, d, e) convenait, avec a 6
b 6 c 6 d 6 e, alors comme précédemment, on déduirait que 1 < a2 6 5 et donc a = 2. On
réinjecte, et il vient successivement par les mêmes arguments b = 2, c = 2, d = 2 et donc
1
e2
= 0, ce qui est absurde.

150
Solution de l’exercice 187 : On remarque que pour x 6= 0 on a l’encadrement :
³ x ´2 ³ x ´2
x2 + < 1 + x + x2 + x3 + x4 < x2 + + 1
2 2
Si x est pair, le membre central est compris entre deux carrés consécutifs et donc ne peut pas
être un carré. Si x est impair, le seul entier compris entre x2 + x2 et x2 + x2 + 1 est x2 + x2 + 12
et donc on doit forcément avoir :
µ ¶2
2 x 1
x + + = 1 + x + x2 + x3 + x4
2 2

ce qui conduit à x2 − 2x − 3 = 0 qui admet pour solution x = 1 et x = −3.


Finalement les solutions sont x = 0, x = 1 et x = −3.

Solution de l’exercice 188 : Soit (a, b, c, d) des entiers non nuls vérifiant :
µ ¶2
2 2 2 2 d 5
a + 5b = 2c + 2cd + 3d = 2 c + + d2
2 2
soit, en multipliant par 4 :

4a2 + 20b2 = 2 (2c + d)2 + 10d2

Modulo 5, cette équation devient :

4a2 ≡ 2 (2c + d)2 (mod 5)

Comme les carrés modulo 5 sont 0, 1 et −1, cela impose nécessairement que a et 2c + d
soient divisibles par 5. Or :

4a2 − 2 (2c + d)2 = 10d2 − 20b2

Le premier membre étant divisible par 25, il en est de même du second et donc on obtient
que d2 ≡ 2b2 (mod 5), ce qui impose encore que 5 divise b et d. Par conséquent, 5 divise a,
b, c, d, et (a/5, b/5, c/5, d/5) fournit une nouvelle solution entière.
Une descente infinie permet alors immédiatement de conclure qu’il n’y a pas de solution
non nulle à l’équation.

Solution de l’exercice 189 : Pour des raisons de parité, l’un des deux nombres p ou q vaut
2. Par symétrie des rôles, on suppose q = 2. Par suite, p est un nombre premier impair tel
que :
pr ± 1 = 2s
Si r est impair, pr ± 1 est un multiple de :

pr−1 ± pr−2 + pr−3 ± · · · ± p + 1

qui est un nombre impair strictement supérieur à 1. Il ne peut donc pas diviser 2s . On en
déduit que r est pair.

151
Posons r = 2t. Le nombre p2t est alors un carré impair qui est donc congru à 1 modulo
4. Ainsi pr + 1 ≡ 2 (mod 4) et donc pr + 1 = 2s entraı̂ne s = 1, ce qui est exclus.
Il reste à étudier p2t − 1 = 2s . Le produit (pt − 1) (pt + 1) est une puissance de 2 et donc
il en est de même de chacun des facteurs. Deux puissances de 2 qui diffèrent de 2 ne peuvent
être que 2 et 4. Ceci entraine p = 3 et t = 1, donc r = 2.
Les deux seules solutions sont p = 3, q = 2, r = 2, s = 3 et p = 2, q = 3, r = 3, s = 2.

Solution de l’exercice 190 : On remarque d’abord que :

20022002 = 20022001 · 2002 = (2002667 )3 · (103 + 103 + 13 + 13 )

donc 20022002 s’écrit comme somme de quatre cubes. On va voir que cette valeur est optimale.
La présence de cubes nous invite à examiner l’équation modulo une puissance de 3.
Modulo 9, on a déjà :

20022002 ≡ 42002 ≡ 46·333+4 ≡ 44 ≡ 4 (mod 9)

car ϕ(9) = 6. Or les cubes modulo 9 sont 0, 1 et −1, ce dont on déduit aussitôt que 4 n’est
pas somme de trois cubes ou moins.
Le résultat est donc t = 4.

Solution de l’exercice 191 : On remarque que l’équation se factorise sous la forme :

y 2 = (x − 1)2 (x + 2)
y
Si (x, y) est une solution, posons t = x−1 . On obtient alors : x = 2 − t2 et donc t est entier.
Cela donne directement toutes les solutions qui sont :
¡ ¡ ¢¢
2 − t2 , t 1 − t2

pour t décrivant Z.

Remarque. Voici l’allure de la courbe d’équation y 2 = x3 − 3x + 2.

−2 0 1

Ce n’est pas une courbe elliptique. On remarque que le point (1, 0) joue un rôle particulier
(il n’y a pas clairement une tangente en ce point). On dit que l’on a affaire à un point
singulier. L’existence d’un tel point permet d’appliquer la méthode des équations de degré
2 pour décrire tous les autres points de la courbe à coordonnées rationnelles : on trace une
droite passant par ce point et on cherche les autres intersections de cette droite avec la

152
courbe. Le fait que le point soit singulier implique en réalité que l’on va avoir une racine
double alors de la résolution de l’équation qui va apparaı̂tre, et donc un unique autre point
d’intersection, forcément rationnel.
La rédaction donnée ci-dessus correspond exactement à l’application de cette méthode :
le paramètre t correspond à la pente de la droite tracée.

Solution de l’exercice 192 : On commence par chercher une petite solution à l’équation. On
peut par exemple remarquer que :

12 + 22 + 32 + 42 + 52 = 55 = 120 − 65 = 1 · 2 · 3 · 4 · 5 − 65

Il s’agit ensuite, à partir d’une solution (x, y, z, u, v), avec au moins quatre des cinq entiers
supérieurs à 2, de trouver une autre solution plus grande. On considère alors le plus petit
entier (ou l’un des plus petits) parmi x, y, z, u, v, disons x, et l’on regarde l’équation :

x2 + y 2 + z 2 + u2 + v 2 = xyzuv − 65

comme une équation du second degré en x. On obtient immédiatement que (yzuv−x, y, z, u, v)


fournit une autre solution, et l’on a yzuv > 8y > x, donc c’est bien une solution « plus grande
».
Plus précisément, le minimum des quatre nombres ne diminue pas quand on applique ce
procédé, et il augmente strictement au bout d’au plus cinq étapes (dans le cas hypothétique
où l’on serait parti d’une solution avec x = y = z = u = v). On finit donc bien par aboutir,
après un certain nombre d’itérations, à une solution avec x y, z, u et v tous supérieurs à
1998.
En fait, six itérations suffisent, et l’on obtient alors la solution suivante, qui convient, et
que l’on ne recopie que pour l’amusement du lecteur :

x = 7138
y = 16 988 437
z = 72 151 760 667 066
u = 1 041 175 313 471 572 184 867 943 319
v = 9 109 630 532 627 114 315 851 511 163 018 235 051 842 553 960 810 405

Solution de l’exercice 193 : La relation xn + 1 = y n+1 s’écrit encore :

xn = (y − 1)(1 + y + · · · + y n ) (9)

Dès lors, si p est un diviseur premier de y − 1, p divise x, et donc ne divise pas n + 1, qui
est premier avec x. Mais on a :

1 + y + · · · + yn ≡ n + 1 (mod y − 1)

et par conséquent p ne divise pas non plus 1 + y + · · · + y n . Ainsi, y − 1 et 1 + y + · · · + y n sont


premiers entre eux. Il en résulte, d’après (9), que 1 + y + · · · + y n est une puissance n-ième.
Mais c’est impossible, puisque c’est un entier strictement compris entre les deux puissances
n-ièmes consécutives y n et (y + 1)n : il n’y a donc pas de solution.

153
Solution de l’exercice 194 : Posons x = ba2 et montrons que x est entier.
2 2
Si a > b, alors ab = ba 6 aa et donc a > b2 . Donc a2b = b2a 6 aa et donc a > 2b2 . Par
suite :
2 2
xb = ba−2b
est entier. On en déduit que x est entier (puisqu’il est rationnel). L’équation devient :
2 x
bxb = bb

soit x = bx−2 . Si b = 1, on a directement x = 1. Si b > 2, l’inégalité ne peut avoir lieu pour


x > 4. Pour x = 3, on trouve b = 3 et pour x = 4, on trouve b = 2. Les solutions obtenues
ainsi sont (1, 1), (16, 2) et (27, 3).
2
Si a 6 b, on a ab = ba 6 bb et donc ab 6 b, ce qui est impossible si a > 2. On obtient
ainsi la solution (1, 1) déjà trouvée.
Finalement les solutions sont (1, 1), (16, 2) et (27, 3).

Solution de l’exercice 195 : Soit (a, b) une solution de l’équation a2 + b2 = n(ab + 1) en


entiers strictement positifs. On peut supposer a minimal parmi toutes les solutions en entiers
strictement positifs. Alors en particulier, comme (b, a) est une telle solution, on a a 6 b.
D’autre part, (na − b, a) est encore une solution, donc si na − b > 0, on doit avoir
na − b > a. Mais alors en multipliant par b il vient :

ab 6 nab − b2 = a2 − n 6 ab − n

ce qui est absurde. Donc na − b 6 0. Si l’inégalité est stricte, on a b > na + 1 > n et donc :

n = a2 + b2 − nab = a2 + b(b − na) > a2 + n · 1

ce qui est encore absurde. Il en résulte que b − na = 0, et donc n = a2 .


Finalement, si l’équation a2 + b2 = n(ab + 1) possède des solutions en entiers strictements
positifs, n doit bien être un carré parfait.

Solution de l’exercice 196 : Si a2 + b2 + c2 = nabc avec a 6 b 6 c, alors c est l’une des racines
du polynôme :
P (X) = X 2 − nabX + a2 + b2
La somme des racines étant nab, l’autre racine c0 est également entière et positive (puisque
le produit des racines l’est). D’autre part P (b) = (3 − na) b2 + a2 − b2 qui est strictement
négatif sauf si na < 3 ou si on a à la fois na = 3 et a = b. Le premier cas est exclu. En effet,
pour na 6 2, il vient :

a2 + b2 + c2 − nabc > a2 + (b − c)2 > 0

Le second cas conduit aux solutions n = 3, a = b = 1 (donc c = 1 ou c = 2) et n = 1,


a = b = 3 (et donc c = 3 ou c = 6), que l’on appelera solutions minimales.
si n > 1 et n > 3, puisque P (b) < 0, la seconde racine est strictement inférieure à b,
et donc à c. Ainsi, on construit à partir d’une solution (a, b, c) non minimale une solution
(a, b, c0 ) plus petite. Le principe de descente infinie prouve que si n 6= 1 et n 6= 3, l’équation

154
n’admet aucune solution. Pour n = 1 ou n = 3, elle en admet une infinité qui s’obtiennent
en remontant à partir des solutions minimales par la même transformation.

Solution de l’exercice 197 : a) Tous les entiers n > 1 conviennent, sauf n = 2. En effet, si
n = 1, il suffit de choisir a = b = 2. Si n > 3, il suffit de choisir a = (n−1)n−1 et b = (n−1)n .
Prouvons désormais que n = 2 ne convient pas. Par l’absurde, supposons qu’il existe des
entiers a, b > 2 tels que
(aa )2 = bb (10)
Une telle relation montre que tout nombre premier qui divise a divise également b. On peut
aussi noter que si b 6 a alors bb 6 aa < (aa )2 , et que si b > 2a alors bb > (2a)2a = 22a (aa )2 >
(aa )2 . Par suite, on doit avoir a < b < 2a.
Soit donc p un nombre premier qui divise a (et donc b). On note α (resp. β) l’exposant
de p dans la décomposition de a (resp. de b) en facteurs premiers. La relation (10) conduit
alors à 2aα = bβ, c’est-à-dire α/β = b/2a < 1, et donc α < β.
Par suite, tout nombre premier qui divise a divise également b et ce, selon une puissance
supérieure. Cela entraine que b est un multiple de a ce qui est impossible puisque a < b < 2a :
contradiction !
b) On a déjà la solution évidente (1, 1). Supposons maintenant que a, b > 2 soient des
entiers tels que
(aa )5 = bb (11)
La démarche du a) s’adapte en tout point pour prouver que a < b < 5a et que a divise
b. Par suite, b = ka où k ∈ {2, 3, 4}, et (11) s’écrit a5a = (ka)ka , c’est-à-dire a5−k = k k . Pour
k = 2, cela conduit à a3 = 4, ce qui est impossible. Pour k = 3, il vient a2 = 27, ce qui est
toujours impossible. Et si k = 4, on obtient a = 44 puis b = 45 , et on vérifie qu’ils forment
effectivement une solution de (11).
Finalement, les solutions sont (1, 1) et (44 , 45 ).

Solution de l’exercice 198 : Posons x = zc et y = zb où b et c sont des entiers premiers entre
eux. L’équation devient :
c + zb2 + z 2 = z 2 cb
On en déduit que z divise c et donc c = za pour un certain entier a. L’équation se transforme
à nouveau a + b2 + z = z 2 ab, c’est-à-dire :

b2 + z
a=
z2b − 1
puis :
b + z3
z2a = b +
z2b − 1
3
Le terme zb+z2 b−1 doit donc être un entier strictement positif, (donc supérieur ou égal à 1), et
z 2 −z+1
donc b 6 z−1 . Ce majorant est inférieur strictement à z + 1 dès que z > 3. Ainsi, si z > 3,
2
on récupère b 6 z et donc a 6 zz2 +z−1
< 2. Par suite a = 1 et l’équation devient :

1 + b2 + z = z 2 b

155
C’est une équation du second degré en b dont le discriminant est z 4 − 4z − 4. Il ne peut être
2
un carré puisqu’il est strictement compris entre (z 2 − 1) et z 4 . On n’a donc aucune solution
dans ce cas.
Il ne reste plus que les cas z = 1 et z = 2. Pour z = 1, on a :

b2 + 1 2
a= =b+1+
b−1 b−1
donc b = 2 ou b = 3. On obtient alors deux solutions : (x, y) = (5, 2) ou (x, y) = (5, 3). Si
z = 2, on écrit :
16b2 + 32 33
16a = = 4b + 1 +
4b − 1 4b − 1
donc b = 1 ou b = 3. On obtient encore deux solutions : (x, y) = (4, 2) ou (x, y) = (4, 6).
Les solutions sont (5, 2), (5, 3), (4, 2), (4, 6).

Solution de l’exercice 199 : a) On suit la méthode de descente de Fermat. Donnons-nous


des entiers strictement positifs x, y et z tels que x2 + y 2 = z 2 et xy
2
soit un carré. On peut
supposer x, y et z premiers entre eux, quitte à diviser par leur pgcd, ce qui fournit une
solution plus petite. Dans ce cas, ils sont premiers entre eux deux à deux.
Quitte à échanger x et y, on peut écrire :

x = u2 − v 2 ; y = 2uv ; z = u2 + v 2

où u et v sont strictement positifs et premiers entre eux de parité contraire. L’aire du triangle
est alors uv (u − v) (u + v). Comme u et v sont de parité contraire, u+v et u−v sont impairs
et donc premiers entre eux. Ainsi les quatre facteurs du produit sont premiers entre eux et
donc chacun un carré. Il existe des entiers a, b et des entiers impairs c et d tels que :

u = a2 ; v = b2 ; u + v = c2 ; u − v = d2

On a 2b2 = c2 − d2 ≡ 0 (mod 4) donc b est pair. On pose b = 2b0 d’où :


µ ¶µ ¶
c+d c−d 2
= 2b0
2 2

On a pgcd (c + d, c − d) = 2 (puisque ces deux nombres sont pairs) et donc un et un seul


des deux facteurs précédents est pair.
Si c’est c+d
2
. Alors : µ ¶µ ¶
c+d c−d 2
= b0
4 2
Comme les deux facteurs sont premiers entre eux, il existe r et s tels que :

c + d = 4s2 ; c − d = 2r2

On vérifie que a2 = r4 + 4s4 , ce qui prouve que le triplet (r2 , 2s2 , a) est une nouvelle solution
dont on vérifie qu’elle est plus petite au sens où a < z.
Le principe de descente infinie permet de conclure qu’il n’y a pas de solution.

156
b) Soient x, y et z des entiers strictement positifs vérifiant :
x4 − y 4 = z 2
On pose X = x4 − y 4 , Y = 2x2 y 2 et Z = x4 + y 4 . Alors X, Y et Z sont les côtés d’un
triangle rectangle dont l’aire est un carré. D’après la question précédente n’est possible que
si X = 0, c’est-à-dire x = y et par le fait z = 0. Ceci n’est pas une solution acceptable.
¡ ¢p
Solution de l’exercice 200 : Soit p un diviseur premier de `. Alors 1 + nk − 1 divise
¡ ¢`
1 + nk − 1 = nm . Or, d’après la formule du binôme :
¡ ¢p 1
1 + nk − 1 = pnk + p (p − 1) n2k + M n3k
2
où M est un entier. Notons :
1
A = p + p (p − 1) nk + M n2k
2
m
C’est un diviseur de n strictement supérieur à p. Si p ne divisait pas n, alors n serait
premier avec A, ce qui contredit que A divise nm . Donc p divise n et il divise aussi A. Le
quotient Ap est un entier strictement supérieur à 1 divisant nm . On a en outre :
A 1 M
= 1 + (p − 1) nk + n2k
p 2 p
et chacun des trois termes est entier (pour le terme central, si p = 2, alors n est pair, et
sinon p − 1 est pair). Le dernier terme est même multiple de n. Si k > 1 ou p est impair,
n divise 21 (p − 1) nk , et donc n est premier avec Ap , ce qui contredit le fait que Ap soit un
diviseur de nm .
Donc k = 1 et p = 2. Et donc ` = 2s pour un certain entier s > 1. L’équation se réécrit :
nm = (1 + n)` − 1 = n` + n2 M 0
pour un certain entier M 0 > 0. On en déduit que m > 2 puis que n divise `. Ainsi n est
aussi une puissance de 2 : n = 2t pour un certain entier t.
On remarque que :
s
³ s−1 ´ ³ s−1 ´
X2 − 1 = X2 + 1 X2 − 1
³ s−1 ´ ³ s−2 ´ ³ s−2 ´
= X2 + 1 X2 + 1 X2 − 1
..
. ³ ´ ³ s−2 ´
s−1
= X 2 + 1 X 2 + 1 · · · (X + 1) (X − 1)
En l’appliquant à X = 1 + n, on obtient :
³ s−1
´³ s−2
´
2tm = n (1 + n)2 + 1 (1 + n)2 + 1 · · · (n + 2)
Ainsi n et n + 2 sont tous les deux des puissances de 2, ce qui n’est possible que si n = 2.
Le facteur (1 + n)2 + 1 vaut alors 10 qui n’est pas une puissance de 2. Donc s = 1 et n = 2.
Il s’ensuit m = 3.
Finalement la seule solution est m = 3, n = 2, k = 1 et ` = 2.

157
Propriétés de Z/nZ

Louis Nebout

Le but de ce cours est de présenter le point du vue moderne sur l’arithmétique, issu de
l’algèbre. Rien de ceci n’est officiellement au programme des olympiades et une partie des
résultats vous est certainement déjà connue sous une formulation différente, mais une bonne
connaissance de cette théorie permet de mieux comprendre ce qui se passe, et de prouver
quelques résultats très puissants.

1 L’anneau Z/nZ
Soit n > 2 un entier naturel. Quelle est précisement la nature de la formule a ≡ b [n] ?
Ce n’est pas une vraie égalité : cela veut dire qu’il existe une certaine relation d’équivalence,
la relation de congruence, pour laquelle a et b sont en relation. Maintenant, si a ≡ b [n] et
c ≡ d [n], on sait bien que a + c ≡ b + d [n], et de même avec la multiplication. Ainsi, cette
relation possède en fait des propriétés tout à fait similaires à l’égalité, et on aimerait bien
dire que « on peut additionner et multiplier les modulo », mais cette phrase n’a aucun sens
mathématique. Pour lui donner du sens, on aimerait bien la « transformer » en une véritable
égalité, en « faisant de deux entiers congrus modulo n un seul et même nombre ».
Si x est un entier, on appelle classe d’équivalence de x modulo n l’ensemble des entiers congrus
à x modulo n. On note x la classe de x. Attention, si x ≡ y (mod n), alors x et y sont deux
notations pour un seul et même objet. On obtient exactement n classes d’équivalence : 0, 1,
. . . , n − 1, et on note Z/nZ l’ensemble de ces classes d’équivalence. On munit Z/nZ de deux
opérations + et × en posant x + y = x + y et x × y = x × y. Il y a une subtilité : il faut
prouver que ces opérations sont bien définies, c’est-à-dire que les résultats de ces opérations
ne dépendent pas des choix des représentants x et y de x et y, par exemple pour +, que si
x = x 0 at y = y 0 , alors x + y = x 0 + y 0 : c’est une simple reformulation du fait que la relation
de congruence est compatible avec les opérations.
La construction de Z/nZ peut paraître conceptuellement difficile la première fois qu’on la
voit, mais en fait, la manipulation de cet ensemble est très simple en pratique : écrire x + y = z
est rigoureusement équivalent à écrire x+y ≡ z mod n, par exemple. Pour passer d’une écri-
ture à l’autre, on enlève les barres et on remplace l’égalité par une relation de congruence. Mais
l’énorme avantage conceptuel de l’utilisation de Z/nZ est, dans le cas de Z/5Z par exemple,
le fait que 2 et 7 sont un seul et même nombre, et non plus simplement congrus. De plus, Z/nZ
possède une certaine structure algébrique, qui nous permet de réaliser toutes nos opérations
en restant à l’intérieur de Z/nZ, et donc sans avoir à repasser par les entiers.
L’ensemble Z/nZ est donc muni de deux opérations, une addition et une multiplication,
toutes deux commutatives et associatives, et telles que

1
— La loi + admet un élément neutre, 0, tel que pour tout x ∈ Z/nZ, x + 0 = x ;
— Tout élément x de Z/nZ admet un opposé noté −x, tel que x + (−x) = 0 (celui-ci est
unique).
— × est distributive sur + ((x + y) × z = x × z + y × z),
— La loi × admet un élément neutre, 1, tel que pour tout x ∈ Z/nZ \ {0}, x × 1 = x.
En algèbre, on appelle un tel ensemble un anneau (commutatif). Les anneaux sont fondamen-
taux, car ils apparaissent dans bien des domaines, et les mathématiciens ont donc développé
une théorie générale traitant de ce type d’objets. Je n’en dirai pas plus pour l’instant.

2 Inversibilité
Proposition 1. On dit que a ∈ Z/nZ est inversible s’il existe b ∈ Z/nZ, appelé l’inverse de a et
noté a−1 , tel que a × b = 1. Les inversibles de Z/nZ sont exactement les k, où k est un entier
premier avec n.
Démonstration. C’est une reformulation du théorème de Bézout, en effet on a les équivalences
suivantes.
Il existe b ∈ Z tel que ab ≡ 1 mod n
⇔ il existe b ∈ Z et k ∈ Z tels que ab = kn + 1
⇔ a est premier avec n.
Remarque 2. Si on sait que ab = ac dans Z/nZ, on peut donc conclure b = c dans le cas où
a est premier avec n : il suffit de multiplier des deux côtés par l’inverse de a. C’est faux en
général. Par exemple, 2 × 1 = 2 × 3 = 2 dans Z/4Z mais il est faux que 1 = 3.
Si p désigne un nombre premier, on a ainsi que tous les éléments de Z/pZ autres que 0 sont
inversibles. On appelle corps un anneau vérifiant cette propriété. Dans un corps, on dispose
donc d’une opération fondamentale qui n’existe pas dans les anneaux : la division. Ainsi, les
corps sont des objets algébriques beaucoup plus riches. Par exemple, la théorie des polynômes
fonctionne très bien sur les corps, et nous allons donc étudier les polynômes à coefficients dans
Z/pZ. Noter que de tels polynômes seraient délicats à définir sans l’introduction de Z/pZ :

3 Polynômes sur Z/pZ


Définition 3. Un polynôme sur Z/pZ est une expression de la forme :

a0 + a1 X + a2 X2 + ... + ad Xd

avec les ai dans Z/pZ.


On note Z/pZ[X] l’ensemble de ces polynômes.
Remarque 4. Sur R, on peut assimiler un polynôme et la fonction de R dans R qui lui corres-
pond. Sur Z/pZ il faut être plus prudent. Par exemple, ap − a = 0 pour tout a donc la fonction
correspondant au polynôme Xp − X est la fonction nul. En revanche, ce n’est pas le polynôme
nulle : un polynôme est défini par ses coefficients.
Lemme 5. Soient a et b dans Z/pZ \ {0}, alors a × b est dans Z/pZ \ {0}.

2
Remarque 6. On dit que Z/pZ est intègre.
Démonstration. Si on avait a × b = 0, en multipliant par a−1 et b−1 on obtiendrait 1 = 0, c’est
absurde.
Ce lemme facile nous permet de définir une notion satisfaisante de degré sur Z/pZ [X],
l’ensemble des polynômes à coefficients dans Z/pZ. En effet, si P et Q ont pour termes domi-
nants a · Xk et b · Xl , alors ab · Xk+l sera non nul, et sera le terme dominant de P · Q. Nous
sommes maintenant en mesure de prouver :
Proposition 7. Il y a une notion de division euclidienne sur Z/pZ [X] : soient A et B dans
Z/pZ [X] avec B non nul, alors il existe un unique couple (Q, R) de polynômes de Z/pZ [X] tels
que A = Q · B + R, avec deg(R) < deg(B).
Démonstration. La preuve est la même que dans R [X]. Pour l’unicité, soit (Q 0 , R 0 ) un deuxième
tel couple, alors B · (Q − Q 0 ) = R 0 − R, puis Q = Q 0 en examinant les degrés.
Pour l’existence, on vérifie que l’algorithme usuel fonctionne, car le coefficient dominant
de B est inversible. Par exemple, pour diviser A = 5 · X3 + 2 · X2 + 5 · X par B = 3 · X2 + 6 · X + 2
−1
dans Z/7Z, on commence par retrancher 5 · 3 · X · B à A, le coefficient dominant de Q doit
−1
donc être 5 · 3 · X = 5 · 5 · X = 4 · X. Il reste A − 4 · X · B = 6 · X2 + 4 · X. On retranche donc
−1
6 · 3 · B. Au final, on obtient A = Q · B + R avec Q = 4 · X + 2 et R = 6 · X + 3.
Corollaire 8. Soit P dans Z/pZ [X], et a une racine de P. Alors P est divisible par (X − a), i.e.
il existe Q dans Z/pZ [X] tel que P = (X − a) · Q.
Démonstration. Soit P = (X − a) · Q + R la division euclidienne de P par (X − a). Alors R est de
degré inférieur strictement à 1, donc constant, et l’évaluation de l’expression précédente en a
nous donne R = 0.
Corollaire 9. Un polynôme de degré n dans Z/pZ [X] a au plus n racines.
Démonstration. Soit P de degré n dans Z/pZ [X]. Supposons qu’il admette n racines a1 , a2 , . . . ,
an . D’après le corollaire précédent, il existe une constante c non nulle tel que
P = c · Πn
i=1 (X − ai ).

Soit alors a une racine de P. On a


0 = c · Πn
i=1 (a − ai ),

et, d’après le lemme, un des (a − ai ) est nul, donc a est l’un des ai .
Soit a dans Z/pZ, en appliquant cela au polynôme Xk −a, on obtient un résultat important :
a a au plus k racines k-ièmes ! La section suivante en donne une importante application.
Exercice 1
Résoudre dans Z/12Z l’équation x2 + 3x + 2 = 0.
Exercice 2 (Théorème de Wilson)
Soit p > 2 un entier naturel. Montrer que p est premier si et seulement si (p − 1)! ≡ −1
mod p.
Exercice 3
1 1 a
Soit p > 5 un nombre premier. Soient a, b ∈ Z tels que 1 + + ... + = . Montrer
2 p−1 b
que p2 | a.

3
4 Carrés dans Z/pZ
Définition 10. Soit x ∈ Z. On dit que x est un résidu quadratique modulo p (ou encore que
x est un résidu quadratique dans Z/pZ) si x n’est pas divisible par p et si x est le carré d’un
élément de Z/pZ.
On note : 
   1 si x est un résidu quadratique modulo p
x
= 0 si p | x
p 
−1 sinon
 
x
Le symbole s’appelle le symbole de Legendre.
p

Théorème 11 (Critère d’Euler). Soit p un nombre premier impair, et x ∈ (Z/pZ)∗ . Alors x est
p−1 p−1
un résidu quadratique
 si et seulement si x
2 = 1. Sinon, on a x 2 = −1.
x p−1
Autrement dit, = x 2 pour tout x ∈ Z/pZ
p
Démonstration. Commençons par dénombrer les résidus quadratiques de (Z/pZ)∗ . Soit a un
résidu quadratique, disons que a = y2 avec y ∈ (Z/pZ)∗ . On a alors aussi a = (−y)2 , or
y 6= −y puisque p est impair, donc a est le carré d’au moins deux éléments de (Z/pZ)∗ . En
fait, c’est le carré d’exactement deux éléments, car le polynôme X2 − a est de degré 2, donc
admet au plus deux racines dans Z/pZ. Puisque (Z/pZ)∗ possède p − 1 éléments, et puisque
chaque résidu quadratique est le carré d’exactement deux de ces éléments, on en déduit qu’il
y a exactement p−1 2
résidus quadratiques.
p−1
Tous ces résidus quadratiques vérifient x 2 = 1, puisqu’en les écrivant x = y2 , on obtient
p−1
x 2 = yp−1 = 1, par petit Fermat. Il s’agit de montrer que c’est les seuls. Mais le polynôme
p−1
X 2 − 1 a au plus p−1
2
racines dans Z/pZ, et tous les résidus quadratiques, qui sont au nombre
p−1
de 2 , en sont racines. Donc ce sont les seules, ce qui conclut la première affirmation du
théorème.
p−1
Pour démontrer la seconde partie, il suffit de montrer que la fonction f(x) = x 2 ne prend
que les valeurs 1 et −1 lorsque x parcourt (Z/pZ)∗ . Mais f(x)2 = xp−1 = 1, donc les valeurs
prises par f sur (Z/pZ)∗ sont des racines carrées de 1 : ce sont donc 1 et −1.

Cette preuve, ou du moins le premier paragraphe, est à connaître, car elle donne des
informations sur la répartition des résidus quadratiques : leur nombre, et le fait que cha-
cun soit le carré d’exactement deux éléments opposés. On peut en déduire, par exemple, que
2
12 , 22 , ..., p−1
2
forme un système complet de représentants des résidus quadratiques de
Z/pZ, car ils sont au nombre de p−1 2
et sont deux-à-deux non-opposés.
Une autre remarque importante est que le critère d’Euler peut se reformuler de la façon
suivante à l’aide
  du symbole de Legendre : pour tout nombre premier impair p et pour tout
x p−1
x ∈ Z, on a ≡ x 2 mod p (on remarquera que ceci marche même si p | x). On en déduit
p
immédiatement que le symbole de Legendre est complètement multiplicatif par rapport à son

4
    
x y xy
argument supérieur, autrement dit, pour tous x, y ∈ Z, on a = . En par-
p p p
ticulier, le produit de deux résidus quadratiques est un résidu quadratique, et l’inverse d’un
résidu quadratique est un résidu quadratique (on dit que l’ensemble des résidus quadratiques
est un sous-groupe de (Z/pZ)∗ ), mais aussi, le produit de deux non-résidus quadratiques est
un résidu quadratique, et le produit d’un résidu quadratique et d’un non-résidu quadratique
n’est pas un résidu quadratique.

Voici enfin un célèbre résultat dû à Gauss, peu utile en pratique dans les exercices mais
qu’il est toujours bon de connaître :

Théorème 12 (Loi de réciprocité quadratique). Soient p et q deux nombres premiers impairs.


— Si au moins un des deux nombres p et q est congru à 1 modulo 4, alors q est un résidu
quadratique modulo p si et seulement si p est un résidu quadratique modulo q ;
— Si les deux nombres p et q sont congrus à 3 modulo 4, alors q est un résidu quadratique
modulo p si et seulement si p n’est pas un résidu quadratique modulo q.
À l’aide du symbole de Legendre, on peut reformuler ce résultat de la manière suivante :
pour tous nombre premiers impairs p et q, on a :
   
q p−1 q−1 p
= (−1) 2 2
p q

Ce théorème ne dit rien du cas où p = 2. Pour cela, on a la proposition suivante :

Proposition 13. Soit p un nombre premier impair. Alors 2 est un résidu  quadratique
 modulo
2 p2 −1
p si et seulement si p est congru à 1 où à −1 modulo 8. Autrement dit, = (−1) 8 .
p

Avec la loi de réciprocité quadratique ainsi que la proposition précédente, on peut détermi-
ner très rapidement si un entier est où non un résidu quadratique modulo un nombre premier
p. On peut aussi, pour simplifier les calculs (même si ce n’est en réalité pas nécessaire), utili-
ser le fait que −1 est un résidu
 quadratique modulo p si et seulement si p est congru à 1 où 2
−1 p−1
modulo 4, autrement dit = (−1) 2
p

Exercice 4
(1) Trouver tous les nombres premiers p vérifiant la propriété suivante : pour tous entiers
a, b ∈ Z, si p | (a2 + b2 ) alors p | a et p | b.
(2) Montrer qu’il existe une infinité de nombres premiers congrus à 1 modulo 4.

Exercice 5
219 est-il un résidu quadratique modulo 383 ?

5
5 Ordre dans (Z/nZ)∗
Passons à une étude plus poussée de l’opération la plus intéressante dans Z/nZ : la mul-
tiplication. Seulement, cette multiplication possède quelques propriétés pénibles, comme le
fait que le produit de deux éléments non nuls puisse être nul, qui empêchent de dire grand
chose d’intéressant. Ainsi, il est naturel de restreindre notre étude à l’ensemble des éléments
inversibles de Z/nZ, que l’on notera (Z/nZ)∗ . Attention ! (Z/nZ)∗ n’est pas l’ensemble des
éléments non nuls !
Algébriquement, cet ensemble est muni d’une opération, la multiplication, qui possède un
élément neutre 1, et chaque élément possède un inverse. Un tel ensemble s’appelle un groupe,
une structure mathématique très importante. Un autre exemple de groupe est Z/nZ tout en-
tier, muni de son addition. Nous prouverons plus loin que dans certains cas ces groupes ont
en fait une structure très proche.
J’insiste sur ces questions de structure, car elles sont fondamentales. Selon les ensembles
ou les opérations intervenant dans un problème donné, le cadre naturel dans lequel se place
le problème change. Ainsi, un problème ne faisant intervenir que des multiplications modulo
n « vit » dans (Z/nZ)∗ , et les outils que l’on peut utiliser pour aborder le problème seront ceux
de la théorie des groupes, qui sont très différents de ceux de la théorie des corps par exemple.
Commençons par un rappel : la forme générale du petit théorème de Fermat.

Théorème 14. Soit a dans (Z/nZ)∗ . Alors aϕ = 1. (On rappelle que ϕ désigne l’indicatrice
d’Euler, et que ϕ(n) est le nombre d’entiers inférieurs à n et premiers avec n, qui est aussi le
cardinal de Z/nZ∗ , d’après notre reformulation du théorème de Bézout).

Démonstration. L’idée est d’utiliser le fait que la multiplication par a est une bijection. Appe-
lons x1 , x2 , . . . , xϕ(n) les éléments de (Z/nZ)∗ : si axi = axj , comme a est premier avec n on
peut multiplier par son inverse. On obtient xi = xj d’où l’injectivité. De plus, a(a−1 xi ) = xi
avec a−1 xi ∈ (Z/nZ)∗ d’où la surjectivité.
La multiplication par a est donc une bijection de (Z/nZ)∗ dans lui-même donc :

{x1 , x2 , . . . , xϕ(n) } = {ax1 , ax2 , . . . , axϕ(n) },


Q
Le produit des éléments à gauche vaut x∈(Z/nZ)∗ x et à droite :
Y Y
(ax) = aϕ(n) x
x∈(Z/nZ)∗ x∈(Z/nZ)∗

Q
On obtient le résultat en simplifiant par x∈(Z/nZ)∗ x, ce qu’on peut faire car ce nombre est
inversible.

Proposition 15. Soit x dans (Z/nZ)∗ . La suite (xk )k∈Z est périodique. Appelons ω(x) sa pé-
riode. Alors xl = 1 ⇔ ω(x)|l.

Démonstration. La seule chose à prouver est la périodicité. Or la suite (xk )k∈N prend ses va-
leurs dans un ensemble fini, il existe donc par principe des tiroirs p > q tels que xp = xq , et
alors xp−q = 1 (on simplifie par xq qui est inversible), et la suite est p − q périodique.

6
Attention, dans cette preuve p − q n’est pas nécessairement l’ordre de x. Calculer l’ordre
d’un élément x n’est pas un problème facile : en général, il n’y a pas de méthode plus beaucoup
plus intelligente que le calcul des puissances de x. Introduire cet ordre peut pourtant s’avérer
très fructueux. Il y a un cas particulièrement agréable : si on dispose d’une relation de type
xl = 1 : on saura alors que l’ordre divise à la fois l et ϕ(n), ce qui peut permettre de le
déterminer.

6 (Z/pZ)∗ est cyclique


Étudier les ordres est plus agréable dans (Z/pZ)∗ , grâce aux bonnes propriétes des poly-
nômes de la forme Xk − 1 montrées dans la troisième partie.
Définition 16. Un générateur de (Z/nZ)∗ est un élément x de (Z/nZ)∗ tel que la suite des
puissances de x recouvre tout (Z/nZ)∗ .
Théorème 17. Pour tout p premier, (Z/pZ)∗ possède un générateur. On dit aussi que (Z/pZ)∗
est cyclique.
Pour prouver ce théorème, il faut trouver un moyen de construire des éléments d’ordre
donnés. Je commence par un lemme allant dans ce sens. il est vrai de manière générale sur
(Z/nZ)∗ , je l’énonce donc dans ce cadre.
Lemme 18. Soient a et b dans (Z/nZ)∗ tels que ω(a) ∧ ω(b) = 1. Alors ω(ab) = ω(a)ω(b).
Démonstration. Tout d’abord, (ab)ω(a)ω(b) = 1, donc ω(ab)|ω(a)ω(b). Pour terminer, il suffit
de prouver que si k est tel que (ab)k = 1, alors k est multiple de ω(a) et de ω(b). Or, si
(ab)k = 1, en élevant à la puissance ω(a) on trouve que bkω(a) = 1, donc que ω(b) divise
kω(a), donc ω(b) divise k par lemme de Gauss.
Lemme 19. Posons m := PPCM((ω(x))x∈(Z/nZ)∗ ). Alors il existe dans (Z/nZ)∗ un élément
d’ordre m.
Démonstration. Décomposons m en facteurs premiers : m = pα 1 α2 αk
1 p2 . . . pk . Soit xi un élément
dont l’ordre est un multiple de pα αi
i : ω(xi ) = ki pi (un tel xi existe par définition du PPCM).
i

Posons yi := xki i . On vérifie que yi est d’ordre pα


i et, en utilisant de façon répétée notre lemme
i

précédent, le produit des yi est d’ordre m.


Démonstration. Il est temps de finir la preuve du théorème annoncé. Appliquons donc les lem-
mes à (Z/pZ)∗ . Soit donc x un élément d’ordre m. On sait que p − 1 est multiple de tous les
ordres des éléments de (Z/pZ)∗ , il est donc multiple de leur PPCM : m, et donc m 6 p − 1.
Or, on sait que le polynôme Xm − 1 a au plus m racines, et que les p − 1 éléments de Z/pZ∗
sont racines de ce polynôme (car m est multiple de tous les ordres), donc on a p − 1 6 m, puis
p − 1 = m, et notre élément d’ordre m est notre générateur recherché.
Que veut dire ce théorème ? Choisissons x un générateur. Alors

(Z/pZ)∗ = {1, x, x2 , . . . , xp−2 }.

La multiplication de telles puissances de x est très facile : il suffit d’ajouter les exposants mo-
dulo p − 1. En fait, ce choix de x permet même d’identifier (Z/(p − 1)Z, +) avec (Z/pZ∗ , ×),

7
via la fonction k 7→ xk . Ainsi, la structure multiplicative du groupe (Z/pZ∗ , ×) n’est pas plus
compliquée que la structure additive du groupe (Z/(p − 1)Z, +). Attention, tout n’est pas si
rose, car trouver un générateur x n’est pas facile. Toutefois, l’introduction d’un générateur
peut faire des miracles dans un problème plutôt théorique. Mentionnons, enfin, qu’il existe
des résultats plus généraux, plus difficiles, explicitant pour tout n la structure de (Z/nZ)∗ . En
particulier, on a :

Théorème 20. Le groupe (Z/nZ)∗ possède un générateur si et seulement si n vaut 2, 4, pk ou


2pk , où p est un nombre premier plus grand que 3.

- Quelques exercices -

Exercice 6 Combien y a-t-il de classes a modulo 343 telles que a70 ≡ 1[343] ?
Exercice 7 Soit n un entier naturel. Montrer que les diviseurs premiers du nième nombre de
n
Fermat 22 + 1 sont tous de la forme k · 2n+1 + 1.
Exercice 8 Soit p un nombre premier. Trouver tous les entiers k tels que p divise 1k + 2k + . . . +
(p − 1)k .
Exercice 9 Combien y-a-t’il d’élements d’ordre k dans (Z/pZ)∗ ?
Exercice 10 Soit p un nombre premier impair. Prouver que si q est un diviseur premier de
xp−1 + xp−2 + · · · + 1 alors p = q ou p divise q − 1.
Exercice 11 Soit n divisible par deux premiers impairs distincts. Montrer que Z/nZ n’est pas
cyclique, i.e n’a pas de générateur.
Exercice 12 Trouver tous les entiers n > 1 impairs tels que n divise 3n + 1.
Exercice 13 Trouver tous les p, q premiers tels que pq divise 2p + 2q .

7 Solutions des exercices


Solution de l’exercice 1 En factorisant, on obtient (x + 1)(x + 2) = 0. Mais attention, on ne peut
pas en déduire que x = −1 ou x = −2, car l’anneau Z/12Z n’est pas intègre ! Dans un tel
anneau, un polynôme de degré 2 peut avoir bien plus de deux racines. Il faut éviter d’essayer
d’utiliser des résultats classiques sur les polynômes dans un anneau non intègre, car en gé-
néral, très peu sont vrais. Ici, le seul moyen de résoudre l’équation est de tester tous les cas
possibles, et de cette façon on obtient que l’ensemble des solutions est {2, −5, −2, −1}.
Solution de l’exercice 2
Si p est composé, on choisit a ∈ J1, p − 1K divisant p. p et (p − 1)! ont alors a pour facteur
commun, donc ne sont pas premiers entre eux. p n’est donc pas premier, sinon il diviserait
(p − 1)! donc aussi un entier inférieur à p − 1.
Réciproquement, supposons p premier. Il existe alors deux méthodes pour obtenir la congruence
demandée.
Première méthode. On va partitionner (Z/pZ)∗ en paires d’éléments deux à deux inverses.
Pour cela, il faut connaître les x ∈ (Z/pZ)∗ qui sont leur propre inverse ; ceux-là sont racines
du polynôme X2 − 1, de degré 2, donc il y en a au plus deux : ce sont donc 1 et −1 = p − 1.

8
 
On regroupe les autres par paires d’inverses xi , x−1
i , de sorte que {1}, {p − 1} et les xi , x−1
i
forment une partition de (Z/pZ)∗ .
En réorganisant l’ordre des facteurs du produit, on a alors

(p − 1)! = 1 · p − 1 · (x1 x−1 −1


1 ) · ... · (xr xr ) = p − 1 = −1.

Seconde méthode. Pour p = 2, le résultat est vrai. Supposons maintenant que p > 3. Consi-
dérons le polynôme P(X) = Xp−1 − 1 ∈ Z/pZ[X]. Il est unitaire et de degré p − 1, et par le
petit théorème de Fermat, tous les p − 1 éléments de (Z/pZ)∗ en sont racines. Par la remarque
précédant l’exercice, on en déduit que P(X) = (X − 1)(X − 2)...(X − p − 1). Le polynôme P étant
de degré pair, son coefficient constant est produit de ses racines, autrement dit (p − 1)! = −1.
Solution de l’exercice 3 L’égalité de l’énoncé se réécrit (p − 1)! · a = bc1 , où c1 = 2 · 3 · ... · ... ·
(p − 1) + 1 · 3 · ... · (p − 1) + ... + 1 · 2 · ... · (p − 2) est le coefficient du terme en X du
polynôme P(X) = (X − 1)...(X − (p − 1)). Pour résoudre le problème, il suffit de montrer que
p2 | c1 , car alors comme p2 est premier avec (p − 1)!, on en déduit par Gauss que p2 | a.
Si on tente d’appliquer directement une méthode simillaire à la seconde méthode du pro-
blème précédent, on obtiendra, en réduisant P modulo p que c1 ≡ 0 mod p, ce qui n’est pas
suffisant pour résoudre le problème. On va en fait utiliser une autre méthode : si on note
X
p−1
Xp−1
P(X) = ck Xk , en évaluant P en p, on obtient (p − 1)! = ck pk , en remarquant que
k=0 k=0
X
p−1
c0 = (p − 1)! et simplifiant par p, on obtient ck pk−1 = 0. En réduisant modulo p2 , on ob-
k=1
tient c2 p + c1 ≡ 0 mod p2 . Il suffit donc de montrer que c2 est divisible par p, mais ceci est
clair en réduisant P modulo p.
Solution de l’exercice 4
(1) Il est clair que cette propriété n’est pas vérifiée par 2, en prenant par exemple a = b = 1.
p−1
Soit maintenant p un nombre premier impair. On a (−1) 2 = 1 si p ≡ 1 mod 4 et −1
si p ≡ 3 mod 4, donc par le critère d’Euler, −1 est un résidu quadratique modulo p si
et seulement si p ≡ 1 mod 4. Ceci montre immédiatement qu’aucun nombre premier
congru à 1 modulo 4 ne vérifie la propriété demandée, car un tel nombre premier divise
un entier de la forme n2 + 1 (prendre pour n un représentant de la classe dont le carré
vaut −1).
Montrons maintenant que tout nombre premier congru à 3 modulo 4 vérifie la propriété
demandée. Supposons qu’il existe a, b ∈ Z, avec a non divisible par p, tel que p | (a2 +
b2 ). Il est alors clair que p ne divise pas b non plus, donc a et b sont inversibles dans
2 −1
Z/pZ. On a alors a2 = −b , donc (ab )2 = −1, ce qui contredit le fait que −1 ne soit pas
un résidu quadratique modulo p.
(2) Supposons que l’ensemble des nombres premiers congrus à 1 modulo 4 soit fini et notons-
le {p1 , ..., pn }. Posons alors N = (2p1 ...pn )2 + 1. Il est clair que ni 2 ni aucun des pi ne
divise N, donc tous ses diviseurs premiers sont congrus à 3 modulo 4. Soit p un des di-
viseurs premiers de N ; comme p divise (2p1 ...pn )2 + 12 , alors par la question précédente
il divise 1, absurde.

9
    
219 3 73
Solution de l’exercice 5 Par multiplicativité du symbole de Legendre, on a = .
384
  383 383  
219 383 383
Par deux applications de la loi de réciprocité quadratique, on en déduit que =− .
     383   3 73
219 −1 18 18
Puis en réduisant modulo 3 et 73 respectivement, = − = . Une
383 3 73 73
     2  
219 2 3 2
nouvelle fois par multiplicativité, on a = = , puis par la proposi-
  383 73 73 73
2
tion 13, on finit par en déduire que = 1, donc que 219 est un résidu quadratique modulo
73
383.
Solution de l’exercice 6 Tout d’abord, 343 = 73 , donc (Z/343Z)a st est cyclique, de cardinal 294
(294 = 6 · 72 ). Soit x une racine primitive et α tel que a = xα :

a70 ≡ 1[343] ⇐⇒ 70α ≡ 0[294] ⇐⇒ 21|α,

ce qui nous donne 14 classes d’équivalences mod 294 pour α qui correspondent à 14 classes
d’équivalence modulo 343 pour a.
n n n+1
Solution de l’exercice 7 Soit p un diviseur premier de 22 +1. Alors 22 ≡ −1 mod p et 22 ≡ 1
mod p, donc l’ordre de 2 modulo p divise 2n+1 mais pas 2n , donc c’est 2n+1 . Il s’ensuit que
2n+1 | (p − 1), ce qui est le résultat voulu.
Solution de l’exercice 8 La somme des puissances k-ièmes va être difficile à manipuler telle
quelle. L’idée est d’introduire un générateur x de Z/pZ∗ . En effet,

1k + 2k + . . . + (p − 1)k ≡ 1 + xk + x2k . . . + x(p−2)k [p] .

On reconnaît la somme des termes d’une suite géométrique. Ainsi, si (p − 1)|k, chaque terme
de la somme vaut 1, et la somme vaut p − 1 qui est non nul. Sinon, xk est différent de 1, xk − 1
(p−1)k
est inversible, et la somme vaut x xk −1−1 , qui est nul par théorème de Fermat.
Solution de l’exercice 9 Soit x un générateur de Z/pZ∗ (le résultat de l’exercice implique l’exis-
tence d’un générateur, donc, à moins de vouloir tout reprouver, il faudra de toute façon utiliser
ce résultat). Les éléments de Z/pZ∗ sont donc les xk avec k entre 0 et p − 2. Quel est l’ordre
d’un tel xk ? C’est le plus petit a tel que ak soit multiple de p − 1, c’est donc PPCM(p−1,k)
k
, au-
p−1 k p−1
trement dit PGCD(p−1,k) . Soit donc d un diviseur de p − 1. L’élément x est d’ordre d si et
seulement si PGCD(p − 1, k) = d, donc si k est de la forme αd avec α premier avec p−1 d
. Il y a
p−1
ϕ( d ) tels k.
Deux remarques : la notion d’ordre fonctionne aussi dans (Z/nZ, +), l’ordre de x étant
le plus petit entier n tel que nx soit nul (où nx est défini comme valant x + x + . . . + x n
fois). Un raisonnement identique montre alors que, pour d divisant n, il y a ϕ(d) éléments
d’ordre d (moralement, notre preuve consistait à, via le choix d’un générateur, se placer dans
(Z/(p − 1)Z, +). On en déduit en comptant selon leur ordre les éléments de Z/nZ, la très jolie,
et importante, identité combinatoire suivante :

ϕ(n) = Σd|n ϕ(d).

10

Solution de l’exercice 10 Un tel diviseur q divise xp − 1 = (x − 1) xp−1 + xp−2 + ... + 1 donc
l’ordre de x modulo q divise p premier. Si cet ordre est p, comme d’après le théorème de
Fermat il divise également q − 1, p divise q − 1. Si l’ordre est 1, pour tout k, xk ≡ 1 [q] donc la
somme des p termes : xp−1 + xp−2 + ... + 1 ≡ p [q] est divisible par q si et seulement si q divise
p, soit q = p.
Solution de l’exercice 11 Soit n = pα1 α2 αk
1 p2 . . . pk la décomposition en facteurs premiers de n. Je
vous laisse utiliser le théorème chinois pour montrer que pour tout a, si on note ωj (a) l’ordre
de a mod j :  
ωn (a) = ppcm ωpα1 (a), pα 2
2
(a), . . . , p αk
k (a) .
1

Comme l’ordre d’un élément mod j divise ϕ(j), on a que


 
ωn (a) ppcm (p1 − 1)pα 1 −1
, (p − 1)p α2 −1
, . . . , (p − 1)p αk −1

1 2 2 k k

Or on veut un élément dont l’ordre soir ϕ(n), qui est le produit des termes de droite. Le ppcm
de ces termes est égal à leur produit ssi ils sont tous premiers entre eux deux à deux, mais il
est facile de voir que si n est divisible par deux premiers impairs distincts, deux de ces termes
sont pairs. Donc Z/nZ n’est pas cyclique.
Solution de l’exercice 12 Soit n > 1 tel que n divise 3n + 1. Une autre façon de le dire est que
3n ≡ −1 [n], on est donc face à un problème purement multiplicatif. On aimerait bien utiliser
le théorème de Fermat, mais il est difficile à exploiter car on contrôle mal ϕ(n). Soit donc p un
facteur premier de n, qui est impair. On a 32n ≡ 1 [p]. Soit ω l’ordre de 3 modulo p. Alors ω
divise 2n. D’autre part, d’après le petit théorème de Fermat, 3p−1 ≡ 1 [p]. Ainsi ω divise p − 1.
On en déduit que ω divise PGCD(2n, p − 1).
Comme on est libre de choisir le diviseur premier p de n qu’on veut, il est "assez classique"
de choisir un diviseur "extrême". Si on impose donc de plus que p soit le plus petit facteur
premier de n, alors nécessairement ω = 1 ou 2 (car un facteur premier de p − 1 ne peut pas
diviser n par minimalité). Dans le premier cas de figure, 3 ≡ 1 [p] et donc p = 2, ce qui est
exclu. Dans le deuxième cas, 32 ≡ 1 [p] et donc p divise 8, ce qui est exclu également. On en
déduit que n = 1.
Solution de l’exercice 13 Remarquons tout d’abord que si p = 2, 2q divise 4 + 2q si et seulement
si soit q = 2, soit 2q divise 6, puisque, pour tout q impair, q divise 2q−1 − 1, donc 2q divise
2q − 2. D’où les solutions : (p, q) = (2, 2), (2, 3) ou (3, 2). On supposera désormais p et q
impairs. Appelons ωp et ωq les ordres de 2 modulo p et q respectivement. Supposons que p
divise 2p + 2q , donc 2p−1 + 2q−1 (car 2 est inversible modulo p, on peut donc diviser par 2),
comme p divise 2p−1 − 1, p divise 2q−1 + 1, donc p divise 22(q−1) − 1. Dès lors, ωp divise p − 1
et 2(q − 1) mais ne divise pas q − 1. Appelons v2 la valuation 2-adique. Le fait que ωp divise
p − 1 implique que v2 (ωp ) 6 v2 (p − 1), et le fait que ωp divise q − 1 mais pas 2(q − 1) implique
que v2 (ωp ) = 1 + v2 (q − 1) (pour vous en convaincre, regardez les décompositions en facteurs
premiers). Or, symétriquement, on a v2 (ωq ) 6 v2 (q − 1), et donc v2 (ωp ) = 1 + v2 (q − 1) >
v2 (ωq ). Symétriquement, v2 (ωq ) > v2 (ωp ), c’est absurde.

11
Autour des diviseurs premiers de an ± bn
Igor Kortchemski∗

Résumé
Nous nous intéressons à plusieurs propriétés des diviseurs premiers de an ±bn . En particulier, en utilisant les
polynômes cyclotomiques, nous démontrons le théorème de Zsigmondy, qui s’intéresse aux diviseurs premiers
de an − bn ne divisant aucun des entiers aj − bj pour 1 6 j < n. Les notions abordées sont illustrées par de
nombreux exercices issus de diverses olympiades de mathématiques.

1 Introduction
Ce texte présente des résultats concernant l’étude des facteurs premiers de an ± bn autour de quatre thèmes.
Nous commençons par démontrer le résultat suivant, où pour un entier relatif non nul n ∈ Z et un nombre premier
p, on note vp (n) l’exposant de la plus grande puissance de p divisant n.
Théorème 1 (Théorème « Lifting The Exponent » (LTE)). Soit p un nombre premier impair. Soient a, b des
nombres entiers relatifs distincts et un entier n > 1. On suppose que p divise a − b mais que p - a, p - b. Alors :

vp (an − bn ) = vp (a − b) + vp (n).

Si ce théorème, redécouvert de nombreuses fois, fait partie du « folklore » mathématique, il semble qu’il
apparaisse en 1878 dans les travaux de Lucas [6, Section XIII] (avec une petite erreur dans le cas p = 2), alors
professeur au lycée Charlemagne. Il est maintenant popularisé sous le nom de « Théorème LTE ».
Nous poursuivons par des propriétés de l’ordre multiplicatif d’un entier dans Z/nZ, qui est souvent utile pour
étudier des questions de divisibilité.
Nous définissons ensuite les polynômes cyclotomiques et étudions en particulier des propriétés concernant leurs
diviseurs premiers en lien avec l’ordre multiplicatif.
Finalement, si a, b ∈ Z et n > 2 est un entier, nous étudions la question de l’existence de diviseurs premiers
primitifs de an − bn . Par définition, un diviseur premier p de an − bn est dit primitif si, pour tout entier 1 6 j < n,
p ne divise pas aj − bj , et non primitif sinon. Par exemple, 5 est un diviseur primitif de 24 − 14 , 3 n’est pas
diviseur primitif de 24 − 14 , et 26 − 16 n’admet pas de diviseur premier primitif. En utilisant le théorème LTE
ainsi que les polynômes cyclotomiques, nous démontrons enfin le théorème de Zsigmondy, qui garantit l’existence
de diviseurs premiers primitifs, sauf exceptions.
Théorème 2 (Théorème de Zsigmondy). Soient a > b > 1 des entiers premiers entre eux et n > 2 un entier.
Alors an − bn admet au moins un diviseur premier primitif à l’exception des deux cas suivants :
(i) 26 − 16 ,
(ii) n = 2 et a + b est une puissance de 2.
Ce résultat est dû à Bang [3] (1886) dans le cas b = 1, et Zsigmondy [10] (1892) dans le cas général. Ce
théorème a été redécouvert plusieurs fois avec des variantes de preuve, par exemple par Birkhoff & Vandiver [4]
en 1904, Dickson [5] en 1905 (dans le cas b = 1) et Artin [2] en 1955. Cependant, toutes ces preuves utilisent les
polynômes cyclotomiques.
Ce théorème implique que hors cas exceptionnels (i) et (ii), a, n > 2 étant fixés, il existe un nombre premier
p tel que l’ordre de a modulo p vaut n (voir la Définition 1 ci-dessous pour la définition de l’ordre). Il est aussi
intéressant de noter que la démonstration de Wedderburn du théorème éponyme [7], qui dit que tout corps fini
est commutatif, utilise le théorème de Bang. Mentionnons également que le théorème de Zsigmondy trouve des
applications dans la théorie des groupes finis (voir par exemple [2]).

DMA, École Normale Supérieure, Paris igor.kortchemski@normalesup.org

1
À la fin des quatre parties relatives respectivement au théorème LTE, à l’ordre multiplicatif dans Z/nZ, aux
polynômes cyclotomiques et au théorème de Zsigmondy se trouvent des exercices issus de diverses olympiades
de mathématiques (de niveau lycée) afin d’illustrer ces notions, et dont les solutions figurent en fin de texte. Si
bien entendu des solutions différentes sans les deux derniers outils avancés existent, nous verrons que ceux-ci
fournissent peut-être des approches plus naturelles.

2 Théorème LTE
2.1 Preuve du théorème
On commence par le lemme suivant.
Lemme 1. Soient x, y des entiers relatifs distincts et n > 1. Soit p un nombre premier ne divisant pas n, tel que
p | x − y mais tel que p - x, p - y. Alors
vp (xn − y n ) = vp (x − y).
Démonstration. On écrit xn − y n = (x − y)(xn−1 + yxn−2 + · · · + y n−1 ). Comme x ≡ y (mod p), on remarque que
xn−1 + yxn−2 + · · · + y n−1 ≡ nxn−1 (mod p). Comme p ne divise ni x, ni y, il s’ensuit que nxn−1 6≡ 0 (mod p)
(et, en particulier, xn 6= y n ). Donc p ne divise pas xn−1 + yxn−2 + · · · + y n−1 et le résultat en découle.
Nous sommes maintenant en mesure d’établir le théorème LTE.
Preuve du Théorème 1. Étape 1. On montre d’abord que

vp (ap − bp ) = vp (a − b) + 1. (1)

À cet effet, notons A = ap−1 + bap−2 + · · · + bp−1 . Le même raisonnement que dans la preuve du Lemme 1 fournit
A ≡ pap−1 ≡ 0 (mod p). Étudions maintenant A modulo p2 . Comme p divise a − b, il existe k ∈ Z tel que
b = a + kp. Alors, tout entier 0 6 i 6 p − 1, on a
 
i  
X i
bi ap−1−i = (a + kp)i ap−1−i = ai + ikpai−1 + (kp)j ai−j  ap−1−i ≡ ap−1 + ikpap−2 (mod p2 ).
j=2
j

Il en découle que
p−1
X p−1
X
bi ap−1−i ap−1 + ikpap−2 (mod p2 )


i=0 i=0
 
p−1 p−1
≡ pap−1 + · kp2 ap−2 (mod p2 ) est entier car p est impair
2 2
≡ pap−1 (mod p2 ) 6≡ 0 (mod p2 ) (car p - a).

Ceci établit (1).


Étape 2. Par une récurrence immédiate, on obtient que
i i
vp (ap − bp ) = vp (a − b) + i. (2)

pour tout entier i > 1. Écrivons à présent n = pα N avec p ne divisant pas N . Alors
 α 
N α N α α
vp (an − bn ) = vp ap − bp = vp (ap − bp ) = vp (a − b) + α,

où on a utilisé le Lemme 1 pour l’avant-dernière égalité et (2) pour la dernière égalité.


Lorsque n est impair, en changeant b en −b on en déduit immédiatement le résultat suivant.
Théorème 3 (Théorème LTE bis). Soit p un nombre premier impair. Soient a, b des nombres entiers (non
nécessairement positifs) et un entier n > 1 impair . On suppose que p divise a + b mais que p ne divise ni a ni b.
Alors vp (an + bn ) = vp (a + b) + vp (n).

2
Nous encourageons le lecteur à étudier le cas p = 2.

Voici un exemple d’application : trouver tous les nombres premiers p tels que (p − 1)p + 1 soit une puissance
de p.
Pour répondre à cette question, on exclut d’abord le cas p = 2 qui convient bien, et on remarque qu’on peut
alors appliquer le théorème LTE : vp ((p − 1)p + 1) = vp (p − 1 + 1) + vp (p) = 2. Donc (p − 1)p + 1 = p2 , ou encore
(p − 1)p−1 = p + 1. Donc p − 1 divise p + 1, et donc p − 1 divise p + 1 − (p − 1) = 2. Donc p = 3. On vérifie
réciproquement que p = 3 convient aussi.

2.2 Exercices
Exercice 1 (Compétition UNESCO 1995) Soient a, n deux entiers strictement positifs et p un nombre premier
impair tel que ap ≡ 1 (mod pn ). Montrer que a ≡ 1 (mod pn−1 ).
Exercice 2 Soit k un entier strictement positif. Trouver tous les entiers strictement positifs n tels que 3k divise
2n − 1.
Exercice 3 (Olympiades Balkaniques de Mathématiques 1993) Soit p un premier impair et m un entier tel qu’il
existe des entiers x, y > 1 vérifiant (xp + y p )/2 = ((x + y)/2)m . Montrer que m = p.
Exercice 4 Trouver toutes les solutions entières strictement positives de x2009 + y 2009 = 7k .
Exercice 5 (Olympiade Iran 2008) Soit a un entier strictement positif. On suppose que 4(an + 1) est le cube
d’un entier pour tout entier positif n. Trouver a.

3 Un lemme utile
On établit ici le lemme utile suivant (qui est probablement un résultat bien connu lorsque b = 1) :
Lemme 2. Soient a 6= b des entiers relatifs premiers entre eux. Soient m, n > 1 des entiers. Alors

PGCD (an − bn , am − bm ) = aPGCD(m,n) − bPGCD(m,n) .

Démonstration. On montre que chaque terme de l’égalité divise l’autre. Pour simplifier les notations, posons
Vn = an − bn pour n > 1. Tout d’abord, comme PGCD (m, n) divise m, VPGCD(m,n) divise Vm . De même,
VPGCD(m,n) divise Vn . On en déduit que VPGCD(m,n) divise PGCD (Vm , Vn ).
Ensuite, si m = n, il n’y a rien à faire. Sinon, supposons m > n. On vérifie que

am − bm − (an − bn ) = an (am−n − bm−n ) + (an − bn )(bm−n − 1) = an Vm−n + Vn (bm−n − 1).

Il en découle que PGCD (Vm , Vn ) divise an Vm−n . Comme a et b sont premiers entre eux, PGCD (Vm , Vn ) divise
Vm−n . Ainsi, PGCD (Vm , Vn ) divise PGCD (Vm−n , Vn ).
Si m < n, on montre de même que PGCD (Vm , Vn ) divise PGCD (Vn−m , Vn ). Or on sait que PGCD (m − n, n) =
PGCD (m, n − m) = PGCD (m, n). Par récurrence (par exemple sur m + n) on en déduit que PGCD (Vm , Vn )
divise PGCD (m, n), ce qui conclut.

4 Ordre multiplicatif d’un entier


4.1 Définition
Dans cette partie, on considère a ∈ Z et n > 1 des entiers premiers entre eux.
Définition 1. L’ordre de a modulo n est le le plus petit entier non nul, noté ωn (a), tel que aωn (a) ≡ 1 (mod n).
Cette définition a un sens, car il existe bien un entier k > 1 tel que ak ≡ 1 (mod n). En effet, comme l’ensemble
des résidus modulo n est fini, il existe deux entiers distincts 1 6 r < s tels que as ≡ ar (mod n). Alors n divise
ar (as−r − 1), donc aussi as−r − 1 car a et n sont premiers entre eux.
L’utilité de cette notion provient essentiellement du théorème suivant, dont la preuve est laissée au lecteur :

3
Théorème 4. Soient a, n des entiers naturels premiers entre eux et k > 1 un entier vérifiant ak ≡ 1 (mod n).
Alors ωn (a) divise k.
Dans la suite, φ désigne la fonction indicatrice d’Euler. On rappelle que φ(n) est le nombre d’entiers, compris
au sens large entre 1 et n, premiers avec n, que φ(ab) = φ(a)φ(b) lorsque a et b sont des entiers premiers entre
eux, et que aφ(n) ≡ 1 (mod n) (théorème d’Euler). En particulier, ωn (a) divise φ(n) (ce qui, dans le cas où n = p
est premier, donne ωp (a) | p − 1). Ceci est en particulier utile lorsqu’on cherche l’ordre d’un entier modulo n à la
main : il suffit de tester les diviseurs de φ(n).
Voici un exemple d’application.
(i) Trouver tous les entiers n > 1 tels que n divise 2n − 1.
(ii) Trouver tous les entiers n > 1 impairs tels que n divise 3n + 1.
Solution.
(i) Soit n > 1 tel que n divise 2n − 1. Il est clair que n est impair. Soit p le plus petit facteur premier de n,
qui est donc impair. Alors 2n ≡ 1 (mod p). Soit ω l’ordre de 2 modulo p. Alors ω divise n. D’autre part,
d’après le petit théorème de Fermat, 2p−1 ≡ 1 (mod p). Ainsi ω divise p − 1. D’après la condition sur p,
on a nécessairement ω = 1. Alors 2 ≡ 1 (mod p), ce qui est absurde. On a donc n = 1.
(ii) Soit n > 1 tel que n divise 3n + 1. Soit p le plus petit facteur premier de n, qui est donc impair, de sorte que
p > 3. Alors 32n ≡ 1 (mod p). Soit ω l’ordre de 3 modulo p. Alors ω divise 2n. D’autre part, d’après le petit
théorème de Fermat, 3p−1 ≡ 1 (mod p). Ainsi ω divise p − 1. On en déduit que ω divise PGCD(2n, p − 1).
D’après la condition sur p, on a nécessairement ω = 1 ou 2. Dans le premier cas, 3 ≡ 1 (mod p) et donc
p = 2, ce qui est exclu. Dans le deuxième cas, 32 ≡ 1 (mod p) et donc p divise 8, ce qui est exclu également.
On en déduit que n = 1.
Pour les exercices qui suivent, il peut être utile de se rappeler que si a et n sont des entiers premiers entre
eux, alors il existe un entier b tel que ab ≡ 1 (mod n).

4.2 Exercices
Exercice 6 Existe-t-il des entiers n > 1 tels que 9 divise 7n + n3 ?
Exercice 7 Trouver tous les entiers m, n > 1 tels que mn divise 3m + 1 et mn divise 3n + 1.

Exercice 8 Soient p, q deux nombres premiers tels que q divise 3p − 2p . Montrer que p divise q − 1.
Exercice 9 (Olympiade Chine 2006) Trouver les entiers a, n > 1 tels que n divise ((a + 1)n − an ).
Exercice 10 Soient a, b > 1 impairs tels que a + b = 2α avec α > 1. Montrer qu’il n’y a pas d’entiers k > 1 tels
que k 2 divise ak + bk .
Exercice 11 Trouver tous les entiers n tels que 19 divise 23n+4 + 32n+1 .
Exercice 12 Soient a, b, n des nombres entiers strictement positifs avec a > b. Montrer que n divise φ(an − bn ).
Exercice 13 Soient n, k > 2 des entiers tels que n divise k n − 1. Peut-on avoir PGCD(n, k − 1) = 1 ?
Exercice
n n
14 Soient x et y deux entiers positifs premiers entre eux. Si k est un entier impair positif qui divise
x2 + y 2 avec n > 1, alors il existe un entier m tel que k = 2n+1 m + 1.
Exercice 15 Trouver tous les p, q premiers tels que pq divise 2p + 2q .
Exercice 16 (Olympiade Irlande 1996) Soient p un nombre premier et a, n des entiers strictement positifs.
Prouver que si 2p + 3p = an , alors nécessairement n = 1.
Exercice 17 Soit n > 1 un entier impair. Si m > 1 est un entier, montrer que n ne divise pas mn−1 + 1.
Exercice 18 (Olympiades Internationales de Mathématiques 1990) Trouver tous les entiers n > 1 tels que n2
divise 2n + 1.
Exercice 19 (Olympiade Bulgarie 1997) Pour un entier n > 2, 3n − 2n est une puissance d’un nombre premier.
Montrer que n est premier.
Exercice 20 (Olympiade États-Unis 2003) Trouver tous les nombres premiers p, q, r tels que p divise 1 + q r , q
divise 1 + rp et r divise 1 + pq .

4
5 Polynômes cyclotomiques
Avant d’introduire les polynômes cyclotomiques, nous introduisons les racines primitives de l’unité et la fonction
de Möbius.

5.1 Racines primitives de l’unité


Définition 2. Soit n > 1 un entier. Un nombre complexe z tel que z n = 1 est appelé racine n-ième de l’unité.
Il y a n racines n-ièmes de l’unité : ce sont les n nombres complexes e2iπk/n pour 0 6 k 6 n − 1. S’il existe un
entier n tel que z est racine n-ième de l’unité, on dit simplement que z est racine de l’unité.
Si z est une racine de l’unité, le plus petit entier k > 1 tel que z k = 1 est appelé ordre de z, et est noté ord(z).
Si un nombre complexe z, racine de l’unité, est d’ordre k, on dit que z est une racine primitive k-ième (de l’unité).
Nous mentionnons la propriété utile suivante, dont la démonstration est laissée au lecteur.
ord(z)
Proposition 1. Soit z une racine de l’unité. Alors ord(z k ) = . En particulier, si z est une
PGCD (k, ord(z))
k
racine primitive n-ième, alors z est une racine primitive n/PGCD (k, n)-ième.
En utilisant ce résultat, il est possible d’en déduire qu’il existe φ(n) racines n-ièmes de l’unité.

5.2 Fonction de Möbius


Définition 3. Soit µ : N∗ → {−1, 0, 1} la fonction définie comme suit :

1
 si n = 1
µ(n) = (−1)k si n est le produit de k nombres premiers distincts

0 sinon.

La fonction µ est appelée fonction de Möbius, et on remarque que µ(ab) = µ(a)µ(b) si a et b sont premiers entre
eux. L’utilité de la fonction de Möbius provient, entre autres, du théorème d’inversion suivant, dont la preuve est
laissée au lecteur.
Théorème 5 (Inversion multiplicative de Möbius). Soient F, f : N∗ → R∗ deux fonctions telles que F (n) =
Y Y  n µ(d)
f (d) pour tout entier n > 1. Alors f (n) = F pour tout entier n > 1.
d
d | n d | n

Mentionnons qu’il existe une formule similaire d’inversion « additive » de Möbius.

5.3 Définition et premières propriétés


Définition 4. Pour tout entier n > 1, on pose
Y
Φn (X) = (X − z) .
z est racine primitive n-ième de l’unité

Le polynôme Φn , de coefficient dominant égal à 1, a priori à coefficients complexes, est appelé n-ième polynôme
cyclotomique. Rappelons deux propriétés bien connues des polynômes cyclotomiques :
Théorème 6. Pour tout entier n > 1,
Y
(i) on a X n − 1 = Φd (X) ;
d|n

(ii) le polynôme Φn est à coefficients entiers.


Mentionnons ensuite deux corollaires immédiats du Théorème 6, mais utiles.
Corollaire 1. Soit n > 1 un entier.
X
(i) On a n = φ(d).
d|n

5
(ii) Soient a ∈ Z et p un nombre premier. Si p | an − 1, alors il existe un diviseur d de n tel que p | Φd (a).
Le théorème 6 permet également d’obtenir différentes formules faisant intervenir des polynômes cyclotomiques :
Y
Proposition 2. (i) Si n > 1 est impair, on a X n + 1 = Φ2d (X).
d|n
Y n µ(d)
(ii) Pour tout entier n > 1, on a Φn (X) = X d −1 .
d|n

Φn (X p ) si p | n
(iii) Si p est un nombre premier et n > 1 un entier, on a Φpn (X) = Φn (X p )
 si p - n.
Φn (X)
(iv) Si n > 3 est un entier impair, alors Φ2n (X) = Φn (−X).
Y
(v) Si a, n > 1 sont des entiers premiers entre eux, alors Φn (X a ) = Φnd (X).
d|a

Démonstration. Pour (i), d’après le Théorème 6 (i), on a


Y Y Y Y
(X n − 1)(X n + 1) = X 2n − 1 = Φd (X) = Φd (X) · Φ2d (X) = (X n − 1) · Φ2d (X),
d|2n d|n d|n d|n
n
d’où le résultat en divisant par X − 1.
Compte tenu du Théorème 6 (i), la seconde assertion est une conséquence immédiate du théorème 5.
La troisième assertion découle aisément de la seconde, et on laisse les détails au lecteur.
D’après (iii) on a Φ2n (X) = Φn (X 2 )/Φn (X). Or si n > 3 est impair, on a Φn (X 2 ) = Φn (X) · Φn (−X). En
effet, si z 2 est racine n-ième de l’unité, on vérifie aisément que soit z soit −z est racine n-ième de l’unité, ce qui
fournit (iv) puisque Φn (−X) est unitaire, tout comme Φ2n (X).
Pour la dernière assertion, en utilisant le Corollaire 1 (i), on vérifie que les degrés des deux polynômes unitaires
sont les mêmes. Il suffit donc de montrer que si z a est une racine primitive n-ième, il existe un diviseur d de a tel
que Φnd (z) = 0. À cet effet, appliquons la Proposition 1 :
n · PGCD (a, ord(z)) = ord(z). (3)
0
En particulier, ord(z) divise an. Puisque a et n sont premiers entre eux, on peut donc écrire ord(z) = dn avec
d | a et n0 | n. En injectant dans (3), on obtient nd = dn0 . Donc n = n0 , et ord(z) = dn. Ainsi, z est racine
primitive dn-ième avec d | n, ce qui conclut.
Si p est un nombre premier et k, n > 1 sont des entiers, la Proposition 2 (iii) et une récurrence fournissent
l’égalité 
pk
Φn (X )
 si p | n
k
Φpk n (X) = p
Φn (X ) (4)
 si p - n.
Φn (X pk−1 )

Pour calculer Φn (X), (4) nous permet de nous ramener au cas où n n’a pas de facteur carré.
Il est instructif d’utiliser les formules précédentes pour vérifier que
Φ1 = X − 1, Φ2 = X + 1, Φ3 = X 2 + X + 1, Φ4 = X 2 + 1, Φ5 = X 4 + X 3 + X 2 + X 2 + X 1 , Φ6 = X 2 − X + 1,
k−1 k−1 k−1
que Φp (X) = X p−1 + · · · + X + 1 si p est premier, et que Φpk (X) = Φp (X p ) = X (p−1)p + · · · + Xp + 1.
Finalement, nous aurons besoin de minorer et majorer des polynômes cyclotomiques.
Lemme 3 (Encadrement des polynômes cyclotomiques). Soient a ∈ C et n > 1. On a
||a| − 1|φ(n) 6 |Φn (a)| 6 (|a| + 1)φ(n) .
De plus, lorsque n > 2, ces inégalités sont strictes.
Démonstration. En prenant le module dans la définition de Φn (a), on a
Y
Φn (a) = |a − z| .
z est racine primitive n-ième de l’unité

Le résultat découle alors de l’inégalité triangulaire. Lorsque n > 2, les racines primitives n-ièmes ne sont pas
alignées avec 0, ce qui implique l’existence d’une racine primitive n-ième z telle que ||a| − 1| < |z −a| < |a|+1.

6
5.4 Lien avec les propriétés d’ordre multiplicatif
Lemme 4. Soient a, n > 1 des entiers et p un nombre premier. Supposons qu’il existe un polynôme P ∈ Z/pZ[X]
tel que l’égalité
X n − 1 = (X − a)2 · P (X)
ait lieu dans Z/pZ[X]. Alors p divise n.
Démonstration. On dérive l’égalité apparaissant dans l’énoncé du lemme :

nX n−1 = 2(X − a)P (X) + (X − a)2 P 0 (X).

On évalue en X = a pour obtenir que nan−1 = 0 dans Z/pZ[X]. Donc p divise nan−1 . Or 0 n’est pas racine de
X n − 1 dans Z/pZ[X], donc p ne divise pas a. Donc p divise n.
On en déduit le résultat suivant, concernant les diviseurs premiers de deux polynômes cyclotomiques évalués
au même entier.
Lemme 5. Soient n > 1 un entier et p un nombre premier. Soit d un diviseur de n avec d 6= n. On suppose que
p | Φn (a) et que p | Φd (a). Alors p divise n.
Démonstration. Comme p divise Φd (a), a est racine de Φd dans Z/pZ[X]. Donc il existe un polynôme P1 ∈
Z/pZ[X] tel que Φd (X) = (X − a)P1 (X) dans Z/pZ[X]. De même, il existe un polynôme P2 ∈ Z/pZ[X] tel que
Φn (X) = (X − a)P2 (X) dans Z/pZ[X]. D’après le Théorème 6 (i), on a X n − 1 = (X − a)2 R(X) dans Z/pZ[X]
pour un certain polynôme R ∈ Z/pZ[X]. Le Lemme 5 implique alors que p divise n.
Ce lemme va nous permettre d’établir les deux théorèmes suivants concernant les polynômes cyclotomiques
évalués en des entiers.

Théorème 7. Soient m, n > 1 des entiers, a ∈ Z et p un nombre premier. On suppose que p divise Φm (a) et que
p divise Φn (a). Alors il existe k ∈ Z tel que
m
= pk .
n
De plus, PGCD (Φm (a), Φn (a)) est une puissance de p.
Démonstration. On écrit m = pα M et n = pβ N avec p - M et p - N . On va montrer que M = N . Tout d’abord,
p | Φm (a) | am − 1, donc p ne divise pas a. Montrons que p divise ΦM (a). On peut supposer α > 1 (car sinon
m = M et il n’y a rien à faire). Alors, d’après la Proposition 2 (iii),
α
ΦM (ap )
Φm (a) = .
ΦM (apα−1 )
α α α
Donc p divise ΦM (ap ). Or ap ≡ a (mod p) d’après le petit théorème de Fermat. Donc 0 ≡ ΦM (ap ) ≡ ΦM (a)
(mod p). On montre de même que p divise ΦN (a).
Maintenant, raisonnons par l’absurde en supposant M 6= N . Sans perte de généralité, supposons que M > N
et posons g = PGCD (M, N ). On a p | ΦM (a) − 1 | aM − 1 et p | ΦN (a) − 1 | aN − 1. Donc

p | PGCD aM − 1, aN − 1 |ag − 1


d’après le Lemme 2. Le Corollaire 1 (ii) fournit alors l’existence d’un diviseur d de g tel que p | Φd (a). Or
p | ΦM (a) et on a d | M, d 6= M . D’après le Lemme 5, ceci implique que p divise M , ce qui est absurde. Le fait
que PGCD (Φm (a), Φn (a)) soit une puissance de p est une conséquence immédiate de la première assertion.
On peut remarquer que le Lemme 5 est un cas particulier du théorème 7.
Théorème 8. Soit p un nombre premier, n > 1 et a ∈ Z.
(i) Si p | Φn (a), alors p ≡ 1 (mod n) ou p | n.
(ii) Si n = pα N avec p premier avec N et p | Φn (a), alors l’ordre de a modulo p vaut N .
(iii) Si p et n sont premiers entre eux, p | Φn (a) si, et seulement si, l’ordre de a modulo p vaut n.

7
Démonstration. Pour (i), on remarque d’abord que p | Φn (a) | an − 1 et donc p ne divise pas a. Soit ω l’ordre de
a modulo p. Comme an ≡ 1 (mod p), ω divise n.
Premier cas : ω = n. D’après le petit théorème de Fermat, ap−1 ≡ 1 (mod p). On en déduit que n = ω | p − 1,
de sorte que p ≡ 1 (mod n).
Deuxième cas : ω < n. Comme p | aω − 1, le Corollaire 1 (ii) implique qu’il existe un diviseur d de ω tel que
p | Φd (a). Or p divise Φn (a) et d < n (car d 6 ω < n). D’après le Lemme
α
5, p divise n.
Pour (ii), notons ω l’ordre de a modulo n. On a 1 ≡ an = (aN )p ≡ aN (mod p). Donc ω | N . Si ω < N , on
raisonne comme dans la preuve de (i) : puisque p | aω − 1, le Corollaire 1 (ii) implique qu’il existe un diviseur d de
ω tel que p | Φd (a). Or p | Φn (a) et n/d n’est pas une puissance de p car d 6 ω < N . Ceci contredit le Théorème
7, et donc ω = N .
Pour (iii), le sens direct provient du deuxième point avec α = 0. Pour la réciproque, supposons que l’ordre
de a modulo p vaille n. Alors p divise an − 1, et d’après le Corollaire 1 (ii), il existe un diviseur d de n tel que
p | Φd (a). D’après le sens direct, l’ordre de a modulo p vaut d. Donc d = n, ce qui conclut.
Comme Φp (X) = 1 + X + X 2 + · · · + X p−1 , on en déduit le corollaire suivant.
Corollaire 2. Soit x ∈ Z. Si p, q sont deux nombres premiers tels que q divise 1 + x + · · · + xp−1 , alors q ≡ 1
(mod p) ou q = p.

5.5 Une application


Théorème 9 (Théorème de Dirichlet). Soit n > 2. Il existe une infinité de nombres premiers p tels que p ≡ 1
(mod n).
Démonstration. Par l’absurde, supposons qu’il n’en existe qu’un nombre fini. Notons T le produit de ces nombres,
multiplié également par tous les diviseurs premiers de n. Comme T > 1, il existe un entier k > 1 tel que
Φn (T k ) > 1. Soit alors p un diviseur premier de Φn (T k ). D’après le Théorème 8 (i), ou bien p ≡ 1 (mod n), ou
bien p divise n. Or p | Φn (T k ) | T nk − 1, donc p est premier avec T . Donc p est premier avec n, ce qui implique
p ≡ 1 (mod n) et est absurde.

5.6 Exercices
Exercice 21 Soit p un nombre premier. Montrer que pp − 1 admet un diviseur premier congru à 1 modulo p.
Exercice 22 Soient n, b > 2 des entiers. Montrer que si (bn − 1)/(b − 1) est une puissance d’un nombre premier,
alors n est une puissance d’un nombre premier (on verra à l’Exercice 40 qu’en fait n est un nombre premier).
n n−1
Exercice 23 Soit n > 1 un entier. Prouver que 22 + 22 + 1 est divisible par au moins n nombres premiers
n n−1
différents. Quel est le plus petit entier n > 1 tel que 22 + 22 + 1 est divisible par au moins n + 1 nombres
premiers différents ?
Exercice 24 (Liste courte Olympiades Internationales de Mathématiques 2002) Soit n > 1 un entier et soient
n−1
p1 , . . . , pn des nombres premiers impairs distincts. Montrer que 2p1 p2 ···pn + 1 a au moins 22 diviseurs.
Exercice 25 (Olympiades Iran 2013) Soit p un nombre premier et d un diviseur de p − 1. Trouver le produit de
tous les éléments de Z/pZ dont l’ordre vaut d.
Exercice 26 (Liste courte Olympiades Internationales de Mathématiques 2006) Trouver tous les entiers relatifs
x7 − 1
x, y tels que = y 5 − 1.
x−1
2
√ une infinité d’entiers positifs n tels que les diviseurs premiers de n + n + 1
Exercice 27 Prouver qu’il existe
sont tous inférieurs ou égaux à n.

6 Théorème de Zsigmondy
6.1 Un deuxième théorème de Zsigmondy
Tout d’abord, du Théorème 2 on peut aussi aisément déduire la version suivante :

8
Théorème 10 (Théorème de Zsigmondy bis). Soient a > b > 1 des entiers strictement positifs premiers entre
eux et n > 2 un entier. Alors an + bn admet au moins un facteur premier qui ne divise pas ak + bk pour tout
1 6 k < n, à l’exception du cas 23 + 13 .
Démonstration. Supposons (a, b, k) 6= (2, 1, 3). On peut alors appliquer le théorème de Zsigmondy à a2n − b2n : il
existe un nombre premier p divisant a2n −b2n mais pas aj −bj lorsque 1 6 j < 2n. Donc p divise (an −bn )(an +bn ).
Comme p ne divise pas an − bn , il divise nécessairement an + bn . Soit maintenant 1 6 j < n. Comme p ne divise
pas a2j − b2j = (aj − bj )(aj + bj ), on en déduit que p ne divise pas aj + bj , ce qui conclut.

Le reste de cette partie est consacré à la preuve du Théorème 2 en adaptant les références [1, 8, 9]. On fixe
dans la suite a > b > 1 des entiers strictement positifs premiers entre eux et n > 2 un entier.
Prouvons déjà le théorème de Zsigmondy dans le cas n = 2, qui n’est pas difficile.
Preuve du Théorème 2 dans le cas n = 2. Supposons que n = 2 et que a + b n’est pas une puissance de 2. Soit p
un diviseur premier impair de a + b. Alors p ne divise pas a − b. En effet, si p | a − b, alors p | a + b + (a − b) = 2a et
p | a + b − (a − b) = 2b. Or a et b sont premiers entre eux, donc p = 2, ce qui contredit le fait que p soit impair.
Dans la suite, on suppose n > 2 et on fixe des entiers a > b > 1 premiers entre eux.

6.2 Quelques propriétés des diviseurs premiers primitifs


Lemme 6. Soit p un nombre premier divisant an − bn . Alors p est non primitif si, et seulement si, il existe un
diviseur d de n tel que d < n et p | ad − bd .
Démonstration. La réciproque est claire par définition, on se concentre donc sur l’implication. Soit p un diviseur
premier non primitif de ak − bk avec k < n. Soitd = PGCD (k, n). En particulier, d | n et d < n. En utilisant le
Lemme 2, on obtient p | PGCD an − bn , ak − bk = ad − bd .
Lemme 7. Soit p un nombre premier divisant an − bn . Si p est primitif, alors p ≡ 1 (mod n).
Démonstration. Comme a et b sont premiers entre eux, p ne divise ni a, ni b. Il existe donc un entier c tel que
a ≡ bc (mod p). Alors, pour j > 1, aj − bj ≡ bj (cj − 1) (mod p). Donc l’ordre de c modulo p vaut n. D’après le
petit théorème de Fermat, cp−1 ≡ 1 (mod p), et donc n divise p − 1.

6.3 Idées de la preuve et résultats préliminaires


L’idée est d’introduire l’entier a
Ψn = bφ(n) Φn .
b
En effet, pour b = 1, le théorème de Zsigmondy implique l’existence d’un nombre premier p tel que l’ordre de a
modulo p vaut n, et compte tenu du Théorème 8 (iii), il est naturel de considérer Φn (a).
L’identité clé est la suivante : Y
a n − bn = Ψd . (5)
d|n

Pour la prouver, on écrit, en utilisant le Théorème 6 (i) et le corollaire qui le suit,


Y Y  a   a n 
Ψd = bφ(d) Φd = bn − 1 = a n − bn .
b b
d|n d|n

Il en découle en particulier que Ψn | an − bn et que


Y n n µ(d)
Ψn = ad − bd . (6)
d|n

en vertu du Théorème 5.
On utilisera aussi l’inégalité suivante :

(a − b)φ(n) < Ψn < (a + b)φ(n) , (7)

9
avec inégalités strictes car n > 2. Cela se démontre exactement comme le Lemme 3 en remarquant que
Y a  Y
Ψn = bφ(n) −z = (a − bz).
b
z est racine primitive n-ième de l’unité z est racine primitive n-ième de l’unité

Concluons cette partie par deux égalités utiles ultérieurement. Si p est un nombre premier divisant n, écrivons
n = pα N avec p ne divisant pas N . Posons Ψn (x, y) = y φ(n) Φn (x/y) pour des entiers x, y > 1 quelconques. Alors
α α
ΨN ap , bp  α−1 α−1 
Ψn (a, b) =  , Ψ n (a, b) = Ψ pN ap , bp . (8)
ΨN apα−1 , bpα−1

Cela se démontre aisément en utilisant (4) ; montrons par exemple la première égalité en utilisant le fait que
φ(pα N ) = φ(pα )φ(N ) = pα−1 (p − 1)φ(N ) :
α α
φ(pα N )
a
pα−1 (p−1)φ(N ) ΦN ap /bp
Ψn (a, b) = b Φp α N =b 
b ΦN apα−1 /bpα−1
α φ(N ) α α
bp ΦN ap /bp
= φ(N ) 
bpα−1 ΦN apα−1 /bpα−1
α α
ΨN ap , bp
= .
ΨN apα−1 , bpα−1

La deuxième égalité apparaissant dans (8) se démontre de la même manière. Pour simplifier, on écrira Ψn à la
place de Ψn (a, b).

6.4 Preuve du Théorème 2


αk
Soit an − bn = pα n n
1 · · · pk la décomposition en facteurs premiers de a − b . Soient pi1 , . . . , pij les facteurs
1

n n
premiers primitifs de a − b . On pose alors
α αi
Pn = pi1i1 · · · pij j ,

qui est la “partie primitive” de an − bn (si an − bn n’a pas de facteurs premiers primitifs, on pose Pn = 1). Nous
allons montrer que Pn > 1.

Étape 1. On montre que Pn | Ψn .


En effet, soit p un diviseur premier primitif de an − bn . D’après le Lemme 6, si d | n et d 6= n, alors p ne
Ydivise
pas ad − bd , et donc p ne divise pas Ψd non plus d’après (5). On en déduit que p est premier avec Ψd .
d|n,d6=n
Par (5), on en déduit que vp (an − bn ) = vp (Ψn ). Ceci étant vrai pour tout diviseur premier primitif de an − bn ,
c’est-à-dire pour tout diviseur premier de Pn , cela implique que Pn | Ψn . 

Soit alors λ > 1 l’entier tel que


Ψ n = λ · Pn . (9)
Tout d’abord, on remarque qu’on a bien Pn > 1 dans le cas λ = 1. En effet, d’après (7), on a Pn = Ψn >
(a − b)φ(n) > 1. On suppose donc λ > 1 dans la suite.

Étape 2. Soit p un nombre premier qui divise λ. On montre que :


(i) Le nombre premier p n’est pas primitif. En particulier, PGCD (λ, Pn ) = 1.
(ii) On a p | n.
Pour (i), il suffit de remarquer que par définition de Pn , Ψn /Pn = λ n’a pas de diviseurs premiers primitifs.
Prouvons (ii). Par (i), p n’est pas primitif, et d’après le Lemme 6, il existe d0 6= n tel que d0 | n et p | ad0 − bd0 .
Compte tenu de (5), on a Y
an − bn = Ψn · ad0 − bd0 ·

Ψd .
d|n,d6=n,d-d0

10
Donc p | Ψn | (an − bn )/(ad0 − bd0 ).
Premier cas : p 6= 2. Dans ce cas, le théorème LTE donne

vp (an − bn ) = vp (ad0 − bd0 ) + vp (n/d0 ).

Ainsi, si p ne divise pas n, alors vp (n/d0 ) = 0 et donc p ne divise pas (an − bn )/(ad0 − bd0 ), ce qui est absurde.
Deuxième cas : p = 2. Dans ce cas, ad0 − bd0 est pair. Comme a et b sont premiers entre eux, cela entraîne
que a et b sont impairs. Par l’absurde, supposons que n soit impair. Alors on peut écrire
 dn  dn
an − bn = ad0 0
− bd0 0
= (ad0 − bd0 ) · A,

où A est une somme de n/d0 termes impairs. Donc A est impair et 2 ne divise pas (an − bn )/(ad0 − bd0 ), ce qui
est absurde. 

Étape 3. On montre que λ est une puissance du plus grand nombre premier divisant n.
Soit p un nombre premier divisant λ. D’après l’étape 2, p divise n et on peut écrire n = pα N avec p ne divisant
pas N . Alors, par (8),
α α
ΨN ap , bp
p | Ψn = .
ΨN apα−1 , bpα−1
 α α α α
Donc p divise ΨN ap , bp . Or, d’après le petit théorème de Fermat, ap ≡ a (mod p) et bp ≡ b (mod p), ce
qui entraîne que  α α
0 ≡ ΨN ap , bp ≡ ΨN (a, b) (mod p).

Donc p divise ΨN . D’après l’étape 1 (appliquée avec N à la place de n), on peut écrire ΨN = λ0 · PN , où PN est
la partie primitive de aN − bN et λ0 est un entier. Si p divise λ0 alors p divise N d’après l’étape 2, ce qui n’est pas
possible. Comme p divise ΨN , cela implique que p divise PN . Donc p est un facteur premier primitif de aN − bN .
Donc
p ≡ 1 (mod N ) (10)
par le Lemme 7. En particulier p > N . Ceci nous donne bien que p est bien le plus grand facteur premier de n,
et donc que le seul diviseur premier de λ est p. 

Dans la suite, p désignera le plus grand diviseur premier de n et on écrit n = pα N avec p ne divisant pas N .

Étape 4. On montre que λ = p.


Pour cela, comme Ψn = λ·Pn et que PGCD (λ, Pn ) = 1 (d’après l’étape 2), il suffit de montrer que vp (Ψn ) = 1.
Considérons un entier d > 1 tel que d | n et p | ad − bd . En particulier, comme a et b sont premiers entre eux,
p ne divise pas b. Soit c un entier tel que bc ≡ 1 (mod p). Nous avons déjà vu que p divise Ψn . Ainsi
a
0 ≡ Ψn = bφ(n) Φn ≡ bφ(n) Φn (ac) (mod p).
b
Donc p | Φn (ac). Le Théorème 8 (ii) entraîne que l’ordre de ac modulo p vaut N . Or p | an −bn (car ad −bd | an −bn ).
Donc (ac)d ≡ 1 (mod p) et N divise d.
Intéressons nous maintenant aux termes divisibles par p dans le produit (6). Compte tenu de ce qui précède,
n n
si d | n et si p divise a d − b d , alors N | n/d, ce qui implique que d = pi pour un certain entier i > 0. Comme
µ(pi ) = 0 dès que i > 2, la factorisation (6) entraîne que
 n
a − bn

vp (Ψn ) = vp n n .
ap − bp
Premier cas : p 6= 2. Alors le théorème LTE donne immédiatement vp (Ψn ) = 1.
Deuxième cas : p = 2. Nous avons déjà établi qu’alors a et b sont impairs, et que p est le plus grand diviseur
premier de n. Donc n est une puissance de 2. Comme n > 2, écrivons n = 2k avec k > 2. Mais alors
k k
an − bn a2 − b2 2k−1 k−1
n n =
2k−1 2k−1 = a + b2 ≡2 (mod 4).
a −b
p p a −b

11
Ainsi v2 (Ψn ) = 1. 

Étape 5. Étude du cas λ = p : fin de la preuve du théorème.


Tout d’abord, si a − b > 2, alors en utilisant successivement (9) et (7), on a
α−1
1 1 2φ(n) 2p (p−1)φ(N )
2p−1
Pn = Ψn > (a − b)φ(n) > = > > 1,
p p p p p
et donc Pn > 1 dans ce cas.
Supposons donc a − b = 1. Raisonnons par l’absurde en supposant Pn = 1. Alors Ψn = p. De plus, comme p
divise (b + 1)n − bn , p est impair.
Premier cas : α > 1. En vertu de successivement (8) et (7), on a

φ(pN ) p−1  

pα−1 pα−1
 
pα−1 pα−1 p
X
p p i
p = ΨN = ΨpN (b + 1) ,b > (b + 1) −b > (b + 1) − b = b > p,
i=0
i

ce qui est absurde.


p−1  
p p
X p i p p
Deuxième cas : α = 1. On remarque d’abord que a − b > a + b car a − b = b > pb + 1 > 2b + 1.
i=0
i
Alors, en vertu de (8) et (7), on a
φ(N ) p−1   p−1  
ΨN (ap , bp ) ap − bp ap − bp

1 X p k b X p
p = ΨN = > > = b >
ΨN a+b a+b 2b + 1 k 2b + 1 k
k=0 k=1

p−1  
X p
Or b/(2b + 1) > 1/3 pour tout entier b > 1 et = 2p − 2. Ainsi, 3p > 2p − 2, ce qui force p = 3.
k
k=1
La congruence (10) entraîne que N divise 2. Ainsi, n = 3 ou n = 6. Traitons d’abord le cas n = 3. On a

a a2
 
3 = Ψ3 = b2 1 + + 2 = a2 + ab + b2 = 1 + 3b + 3b2 ,
b b

ce qui est impossible. Finalement, si n = 6, on a

a a2
 
2
3 = Ψ6 = b 1 − + 2 = a2 − ab + b2 = 1 + b + b2 .
b b

Ceci entraîne b = 1 et a = 2, qui est précisément la dernière exception du théorème de Zsigmondy. Ceci conclut
(enfin !) la preuve de ce théorème.
Remarque 1. Dans la preuve précédente, nous avons en particulier établi qu’en toute généralité, on a soit
Ψn = Pn , soit Ψn = p · Pn , où p est le plus grand diviseur premier de n.

6.5 Exercices
Exercice 28 Pour un entier n > 2, notons an le nombre entier dont l’écriture décimale comporte n fois le chiffre
1. Soit n > 1. Existe-t-il un nombre premier p divisant an mais pas an−1 , . . . , a1 ?
Exercice 29 (Olympiades Italie 2003) Trouver tous les entiers strictement positifs (a, b, p) avec p premier tels
que 2a + pb = 19a .
Exercice 30 (D’après olympiade Russie 1996) Trouver tous les entiers strictement positifs (x, y, n, k) tels que x
et y soient premiers entre eux et 3n = xk + y k .
Exercice 31 (Olympiades Iran) Soit A un ensemble fini de nombres premiers et soit a > 2 un entier. Montrer
qu’il n’existe qu’un nombre fini d’entiers positifs n tels que tous les facteurs premiers de an − 1 appartiennent à
A.

12
Exercice 32 (D’après liste courte Olympiades Internationales de Mathématiques 2002) Soit n > 1 un entier et
soient p1n, p2 , . . . , pn des nombres premiers distincts tous supérieurs ou égaux à 5. Montrer que 2p1 p2 ···pn + 1 a au
moins 22 diviseurs différents.
Exercice 33 (Liste courte Olympiades Internationales de Mathématiques 2004) Trouver tous les entiers stricte-
ment positifs a, m, n tels que am + 1 divise (a + 1)n .
Exercice 34 (Olympiades États-Unis 2001) Trouver tous les entiers strictement positifs x, r, p, n tels que p soit
premier, n, r > 1 et xr − 1 = pn .
Exercice 35 (Compétition Tchéco-Slovaque 1996) Trouver tous les entiers strictement positifs x, y, p tels que
px − y p = 1 avec p premier.
Exercice 36 (Olympiade Pologne 2010) Soient q, p deux nombres premiers tels que q > p > 2. Montrer que
2pq − 1 a au moins trois facteurs premiers distincts.
Exercice 37 (Olympiade Japon 2011)Trouver tous les entiers strictement positifs a, n, p, q, r tels que an − 1 =
(ap − 1)(aq − 1)(ar − 1).
Exercice 38 (Olympiades Balkaniques de Mathématiques 2009) Trouver tous les entiers strictement positifs
x, y, z tels que 5x − 3y = z 2 .
Exercice 39 Trouver tous les nombres strictement positifs a, p, n tels que pa − 1 = 2n (p − 1), où p est un nombre
premier.
Exercice 40 (Olympiade Estonie 2007) Soient n, b > 2 des entiers. Montrer que si (bn − 1)/(b − 1) est une
puissance d’un nombre premier, alors n est un nombre premier.
Exercice 41 Trouver tous les entiers strictement positifs a, m, n tels que (a+1)(a2 +a+1) · · · (an +an−1 +· · ·+1) =
am + am−1 + · · · + 1.
Exercice 42 (Olympiade Roumanie 1994) Montrer que la suite an = 3n − 2n ne contient pas trois termes d’une
même suite géométrique dont la raison est un entier au moins égal à 2.
Exercice 43 (Olympiade Angleterre 1996) Trouver les entiers positifs x, y, z tels que 2x + 3y = z 2 .
Exercice 44 Résoudre l’exercice 4 en vous aidant du théorème de Zsigmondy.
Exercice 45 Résoudre l’exercice 19 en vous aidant du théorème de Zsigmondy.
Exercice 46 (Liste courte Olympiades Internationales de Mathématiques 1997) Soient b, m, n des entiers stric-
tement positifs avec b > 1 et m 6= n. Prouver que si bm − 1 et bn − 1 ont les même facteurs premiers, alors b + 1
est une puissance de 2.
k k
Exercice 47 (Olympiade Iran 2006) Soient a, b, c, k > 1 des entiers. On pose n = ac − bc . Si c est divisible par
au moins q nombres premiers différents, montrer que n est divisible par au moins qk nombres premiers différents.

Exercice 48 Existe-t-il une infinité de couples (p, q) de nombres premiers tels que pq | 2p−1 + 2q−1 − 2 ?
Exercice 49 Soit p un nombre premier. Si m > 1 est un entier, on pose |m|p = pk si pk | m et pk+1 - m.
(i) (Théorème de Feit) Soit N > 1 fixé. Alors pour tous les couples d’entiers (a, n) avec a > 1 et n > 2, sauf
éventuellement pour un nombre fini d’entre eux, il existe un diviseur premier primitif p de an − 1 tel que
|an − 1|p > nN + 1.
(ii) Si m > 1 est un entier, on note maintenant [m]p le plus grand diviseur de m qui n’est pas divisible par p.
Montrer que si p est un nombre premier et que si a > 2 est un entier, alors

[an − 1]p
−→ ∞.
n n→∞

13
7 Solutions des exercices
Solution de l’exercice 1 Il est clair que a et p sont premiers entre eux. D’après le petit théorème de Fermat,
ap−1 ≡ 1 (mod p). Comme ap ≡ 1 (mod p), on en déduit que a ≡ 1 (mod p). On peut donc utiliser le théorème
LTE et on obtient vp (a − 1) + 1 = vp (a − 1) + vp (p) = vp (ap − 1). Par hypothèse, le dernier terme est supérieur
ou égal à n. Il en découle que vp (a − 1) > n − 1, ce qu’il fallait démontrer.
Solution de l’exercice 2 Soit k > 1 un entier tel que 3k divise 2n − 1. En raisonnant modulo 3, on voit que n est
pair. Écrivons donc n = 2m avec m > 0. Alors 3k divise 4m − 1. Comme 3 divise 4 − 1, on peut appliquer le
théorème LTE pour obtenir v3 (4 − 1) + v3 (n) = v3 (4n − 1) > k. On en déduit que v3 (n) > k − 1. Ainsi 2 × 3k−1
divise n.
Réciproquement, le même raisonnement nous donne que 3k divise 2n − 1 si 2 × 3k−1 divise n.
Solution de l’exercice 3 Par convexité de x 7→ xp , on a (xp + y p )/2 > ((x + y)/2)p . Par hypothèse, il s’ensuit que
m > p. Soit d = PGCD(x, y), x = dX, y = dY . L’équation se réécrit

2m−1 (X p + Y p ) = dm−p (X + Y )m . (11)

Premier cas : X + Y n’est pas une puissance de 2. Soit alors q un diviseur premier impair de X + Y . Par le
théorème LTE, vq (2m−1 (X p + Y p )) = vq (X + Y ) + vq (p) et d’autre part vq (dm−p (X + Y )m ) > mvq (X + Y ). Donc
vq (X + Y ) + vq (p) > mvq (X + Y ). Ceci implique m 6 2, et donc m = p = 2 car m > p, ce qui est exclu car p est
impair.
Deuxième cas : X + Y est une puissance de 2. Comme p est impair, X + Y divise X p + Y p et donc v2 (X + Y ) 6
v2 (X p + Y p ). À présent, en prenant la valuation 2-adique dans l’égalité (11), on obtient m − 1 + v2 (X + Y ) >
mv2 (X + Y ). Ainsi v2 (X + Y ) 6 1, X + Y 6 2, et donc X = Y = 1 et m = p.
Solution de l’exercice 4 Déjà, 2009 = 72 × 41. Comme x + y divise x2009 + y 2009 , x + y est une puissance de 7.
On remarque aussi que si x et y sont multiples de 7, on peut tout diviser par 7 et juste changer l’exposant k ;
on peut donc supposer que x et y sont premiers avec 7. Le théorème LTE nous garantit que v7 (x2009 + y 2009 ) =
v7 (x + y) + v7 (2009) = v7 (x + y) + 2, donc x2009 + y 2009 = 49(x + y), donc

x2009 + y 2009
= x2008 − x2007 y + x2006 y 2 − · · · + y 2008 = 49
x+y
Mais il est facile de vérifier que ce terme est beaucoup plus grand que 49. Par exemple, si on suppose x > y, on
aura toujours x2008 − x2007 y > 1, x2006 y 2 − x2005 y 3 > 1 et ainsi de suite, de sorte que la somme totale sera au
moins égale à 1004. Il n’y a donc pas de solutions.
Solution de l’exercice 5 Il est clair que a = 1 convient. Montrons que c’est le seul. Supposons donc a > 1.
Choisissons n = 2m et remarquons que a2 +1 n’est pas une puissance de 2 car congru à 1 ou 2 modulo 4. Soit donc p
un nombre premier impair tel que p divise a2 +1. Alors d’après le théorème LTE, vp (4(an +1)) = vp (a2 +1)+vp (m).
On choisit m de sorte que ce dernier terme soit congru à 1 modulo 3. Alors 4(an +1) n’est pas un cube, contradiction.

Solution de l’exercice 6 Soit n > 1 tel que 9 divise 7n + n3 . Comme un cube est congru à 0, −1 ou 1 modulo 9,
on en déduit que n6 ≡ 1 mod 9 et donc que 72n ≡ 1 (mod 9). Or l’ordre de 7 modulo 9 est 3. On en déduit que 3
divise 2n. Ainsi 3 divise n. Il faudrait donc que 3 divise 7n , ce qui est absurde. Il n’y a donc pas de tels entiers.
Solution de l’exercice 7 On suppose m, n > 2. Soit p le plus petit diviseur de n. Alors 32n ≡ 1 mod p. Soit ω
l’ordre de 3 modulo p. Alors ω divise 2n. D’autre part, d’après le petit théorème de Fermat, 3p−1 ≡ 1 (mod p).
Ainsi ω divise p − 1. On en déduit que ω divise PGCD(p − 1, 2n). D’après la condition sur p, on a nécessairement
ω = 1 ou 2. Dans le premier cas, 3 ≡ 1 (mod p) et donc p = 2. Dans le deuxième cas, 32 ≡ 1 (mod p) et donc
p = 2. On en déduit que n est pair. On montre de même que m est pair. Alors 4 divise 3m + 1, ce qui n’est pas
possible car m est pair.
Il reste à examiner le cas où m ou n vaut 1 et il vient que les solutions sont (1, 1), (1, 2) et (2, 1).
Solution de l’exercice 8 Il est clair que q > 5. Notons ω l’ordre 3/2 modulo q (ici, et similairement dans la suite,
1/2 désigne l’inverse de 2 modulo q). Alors ω divise p, donc ω = 1 ou p. Le premier cas n’étant pas possible, on a
donc ω = p. Or d’après le petit théorème de Fermat, (3/2)q−1 ≡ 1 (mod q). On en tire que ω divise q − 1, d’où
le résultat.

14
Solution de l’exercice 9 Supposons que n > 2. Soit p le plus petit facteur premier de n. Alors p divise (a+1)n −an .
En d’autres termes, ((a + 1)/a)n ≡ 1 (mod p). Soit ω l’ordre de (a + 1)/a modulo p. Alors ω divise n. D’autre
part, d’après le petit théorème de Fermat, ((a + 1)/a)p−1 ≡ 1 (mod p) de sorte que ω divise p − 1. D’après la
condition sur p, nécessairement ω = 1. Ceci implique a + 1 ≡ a (mod p), ce qui est absurde.
Les solutions sont donc n = 1 et a quelconque.
Solution de l’exercice 10 Raisonnons par l’absurde et considérons un entier k > 1 tel que k 2 divise ak + bk . En
raisonnant modulo 4 on voit que k est impair. Comme a + b est une puissance de 2, il en découle que a et b sont
premiers entre eux. Soit p le plus petit facteur premier de k qui est donc différent de 2 et ne divise ni a, ni b.
Soit ω l’ordre de −a/b modulo p. Comme ak + bk ≡ 0 (mod p), on a (a/b)k ≡ −1 (mod p), soit, puisque k
est impair, (−a/b)k ≡ 1 (mod p). Ainsi, ω divise k, mais aussi p − 1 d’après le petit théorème de Fermat. Par
définition de p, k et p − 1 sont premiers entre eux. Donc ω = 1. Ainsi, a + b ≡ 0 (mod p), ce qui est absurde et
conclut la solution.
Solution de l’exercice 11 Les conditions de l’énoncé impliquent que 9n ≡ 8n (mod 19). Mais l’inverse de 8 modulo
19 est 12. On en déduit que 13n ≡ 108n ≡ (9 × 8)n ≡ 1 (mod 19). Or 13 est racine primitive modulo 19. Les
entiers recherchés sont donc les multiples de 18.
Solution de l’exercice 12 Traitons d’abord le cas où a et b sont premiers entre eux. Alors a et b sont premiers avec
an − bn et il est clair que l’ordre de a/b modulo an − bn est n. On en déduit que n divise φ(an − bn ).
Si d > 1 est le PGCD de a et de b, notons u = a/d et v = b/d de sorte que u et v sont premiers entre eux.
D’après ce qui précède, n divise φ(un − v n ). En utilisant la formule exprimant φ(N ) en fonction des facteurs
premiers de N , on voit que φ(un − v n ) divise φ(dn (un − v n )) = φ(an − bn ), ce qui conclut.
Solution de l’exercice 13 Soit p le plus petit facteur premier de n. Modulo p, l’ordre de k divise n puisque k n ≡ 1
(mod p). Par ailleurs, d’après le théorème de Fermat, l’ordre de k modulo p divise p − 1. Or p est le plus petit
facteur premier de n : le seul diviseur de n strictement inférieur à p est 1. L’ordre de p, diviseur de n inférieur ou
égal à p − 1, vaut donc nécessairement 1, ce qui prouve précisément que k ≡ 1 (mod p), donc que p divise k − 1,
de sorte que PGCD(n, k − 1) vaut au moins p. La réponse est donc non.
Solution de l’exercice 14 k n’est pas supposé premier, mais si tous ses facteurs premiers vérifient le résultat, alors
un produit de nombres congrus àn 1 (mod 2n+1 ) sera lui-même ≡ 1 (mod 2n+1 ). Il suffit donc de démontrer
2n
que tout facteur premier 2
p nde x + y vérifie p ≡ 1 (mod 2n+1 ). Par ailleurs, si p divisait x, comme par
2n
hypothèse il divise x + y 2 , il diviserait également y : x et y ne seraient pas premiers entre eux. Donc x et
p sont premiers entre eux, et y et p sont premiers entre eux. Notons 1/x l’inverse de x modulo p, de sorte que
2n 2n
n n n

x2 + y 2 ≡ x2 1 + (y/x) ≡ 0 (mod p) équivaut à : (y/x) ≡ −1 (mod p). Donc cet élément y/x a pour
k
2
ordre 2n+1 , car 2n+1 est la première puissance de 2 vérifiant (y/x) ≡ 1 (mod p), et 2n+1 n’a pas d’autre diviseur
p−1
que des puissances de 2. Comme (y/x) ≡ 1 (mod p), 2n+1 divise p−1, ce qui est précisément le résultat cherché.
Un cas particulier important : pour n = 1, tout diviseur d’une somme de deux carrés premiers entre eux est congru
à 1 modulo 4.
Solution de l’exercice 15 Remarquons tout d’abord que si p = 2, 2q divise 4+2q si et seulement si soit q = 2, soit 2q
divise 6, puisque pour tout q impair q divise 2q−1 −1, donc 2q divise 2q −2. D’où les solutions : (p, q) = (2, 2), (2, 3)
ou (3, 2). On supposera désormais p et q impairs. Appelons ωp et ωq les ordres de 2 modulo p et q respectivement.
Si p divise 2p + 2q , donc 2p−1 + 2q−1 , comme p divise 2p−1 − 1, p divise 2q−1 + 1, donc 22(q−1) − 1. Dès lors, ωp
divise p − 1 et 2(q − 1) mais ne divise pas q − 1. Si la plus grande puissance de 2 divisant ωp (resp ωq ) est 2vp
(resp 2vq ), le fait que ωp divise 2(q − 1) et pas q − 1 entraîne que vp > vq , car q − 1 est divisible par ωq donc
par 2vq et pas par 2vp . Le même raisonnement, en échangeant p et q, aboutit à vq > vp , ce qui est manifestement
incompatible. Il n’existe donc pas de couples de nombres premiers impairs vérifiant cette condition.
Solution de l’exercice 16 Si p = 2, 22 + 32 = 13 vérifie bien la relation demandée : ce n’est pas une puissance > 2
d’un entier. Si maintenant p est impair, 2p + 3p est divisible par 2 + 3 = 5, et n’est divisible par 25 que si p est
divisible par 5 donc, puisque par hypothèse p est premier, si p = 5. En effet, 3p = (5 − 2)p ≡ (−2)p + p.5(−2)p−1
(mod 25). C’est aussi une conséquence du théorème LTE. On en déduit que, hormis éventuellement pour p = 5,
le facteur 5 apparaît avec l’exposant 1, ce qui suffit à démontrer le résultat cherché. Pour p = 5, il apparaît bien
avec l’exposant 2, mais 35 + 25 = 275 n’est pas une puissance > 2 d’un entier, ce qui achève la démonstration.
Solution de l’exercice 17 C’est une conséquence presque immédiate de l’exercice 14. Soit 2k la plus grande puis-
k k
sance de 2 divisant n − 1 : posons n − 1 = 2k q, s = mn−1 + 1 = x2 + y 2 avec x = mq et y = 1. D’après l’exercice

15
14, tout diviseur de s est donc congru à 1 modulo 2k+1 . Or par définition de 2k , n n’est pas congru à 1 modulo
2k+1 . Donc n ne divise pas s.
Solution de l’exercice 18 Les nombres entiers 1 et 3 sont solutions. Montrons qu’il n’y en a pas d’autres. Il est
clair que n est impair. Ensuite, en considérant p le plus petit facteur premier de n et ω, l’ordre de 2 modulo
p, on voit que ω divise à la fois 2n et p − 1. Par définition de p, le PGCD de ces deux entiers vaut 2. Donc
22 ≡ 1 (mod p). Donc p = 3. Écrivons n = 3u, avec u > 2 et appliquons le théorème LTE (n est impair) :
2v3 (n) 6 v3 (2n + 1) = v3 (2 + 1) + v3 (n) = 1 + v3 (n). Donc v3 (n) = 1 et 3 ne divise pas u. Soit maintenant q le plus
petit diviseur premier de u. Alors q | 8u + 1. Donc, en notant ω 0 l’ordre de 8 modulo q, comme précédemment, ω 0
divise le PGCD de 2u et q − 1, qui vaut 2. Donc q divise 63, soit q = 7. Finalement, écrivons n = 21r, avec r > 1.
Alors 7 | 221r + 1 ≡ 2 (mod 7), ce qui est absurde.
Solution de l’exercice 19 On suppose n 0> 2 et0 que 3n − 2n = pk pour k > 1. Montrons d’abord que n0 est impair.
0 n0 n0 0 0 0
Si n = 2n , alors 3 −2 = (3 −2 )(3n +2n ). Il existe donc α > β > 0 tels que : 3n +2n = pα et 3n −2n = pβ .
n n
0
Alors 2n +1 = pβ (pα−β − 1). Donc p = 2, ce qui est absurde, ou β = 0 qui conduit à n = 2, exclu. Ainsi n est
impair.
Raisonnons par l’absurde et considérons q est un nombre premier divisant n avec0 q < n. Écrivons n = qr. Un
raisonnement direct montre que 3q − 2q est une puissance de p, disons 3q − 2q = pk avec k 0 < k. En appliquant
0 k−k0
LTE, on voit que vp (r) = k − k . Écrivons donc r = p u avec p ne divisant pas u. Alors :
k−k0 k−k0 k−k0 k−k0
pk = 3n − 2n = 3qp u
− 2qp u
= (3q )p u
− (2q )p u

0 k−k0 k−k0 0 0 k−k0 k−k0


= (pk + 2q )p u
− (2q )p u
> pk−k u · pk · 2q(p u−1)
= pk u · 2q(p u−1)
> pk ,

ce qui est absurde : n est donc premier.


Solution de l’exercice 20 On commence par examiner la condition « p divise 1 + q r ». Elle se récrit q r ≡ −1
(mod p) et implique donc, en particulier, q 2r ≡ 1 (mod p). Ainsi l’ordre de q modulo p est un diviseur de 2r.
Comme r est supposé premier, c’est donc un élément de l’ensemble {1, 2, r, 2r}. Si on suppose en outre que p 6= 2,
on a q r 6= 1 (mod p), et donc l’ordre de q modulo p est nécessairement 2 ou 2r. Dans le premier cas, en utilisant
que p est premier, on obtient q ≡ −1 (mod p), alors que dans le deuxième cas, on en déduit que 2r divise p − 1.
En permutant les nombres p, q et r, on obtient bien sûr des conséquences analogues des deux autres conditions
« q divise 1 + rp » et « r divise 1 + pq ».
On suppose maintenant que p, q et r sont tous les trois impairs, et pour commencer que l’on est dans le cas
où q ≡ −1 (mod p). Le nombre premier p ne peut donc pas diviser q − 1 (puisqu’il divise déjà q + 1 et qu’il ne
vaut pas 2). D’après les résultats du premier alinéa, la condition « q divise 1 + rp » implique donc que r ≡ −1
(mod q). En appliquant à nouveau le même argument, on trouve que p ≡ −1 (mod r). Or les trois congruences
précédentes ne sont pas compatibles. En effet, par exemple, elles impliquent q > p − 1, r > q − 1 et p > r − 1,
ce qui ne peut se produire, étant donné que p, q et r sont des nombres premiers impairs, que si p = q = r ; on a
alors manifestement q 6≡ −1 (mod p). On en déduit que, toujours dans le cas où p, q et r sont supposés impairs,
2r divise p − 1. En permutant circulairement les variables, on démontre de même que 2p divise q − 1 et 2q divise
r − 1. Ainsi 8pqr divise (p − 1)(q − 1)(r − 1), ce qui n’est pas possible étant donné que 8pqr > (p − 1)(q − 1)(r − 1).
Finalement, il n’y a pas de solution lorsque p, q et r sont tous les trois impairs.
On en vient à présent au cas où l’un de ces trois nombres est égal à 2. Quitte à permuter circulairement à
nouveau p, q et r, on peut supposer que c’est p. Les conditions de l’énoncé disent alors que q est impair, que
r2 ≡ −1 (mod q) et que 2q ≡ −1 (mod r). Selon ce qui a été fait dans le premier alinéa, cette dernière congruence
entraîne que r = 3 ou que 2q divise r − 1. Le premier cas conduit à 9 ≡ −1 (mod q), ce qui ne se produit que si
q = 5 puisque l’on a déjà écarté le cas q = 2. On vérifie par ailleurs que le triplet (2, 5, 3) est bien solution. Dans
le second cas, le produit 2q divise r − 1, mais aussi 2(r2 + 1) puisqu’on sait que r2 ≡ −1 (mod q). Ainsi 2q divise
2(r2 + 1) − 2(r + 1)(r − 1) = 4, ce qui ne peut arriver.
En conclusion, il y a exactement trois solutions qui sont les triplets (2, 5, 3), (5, 3, 2) et (3, 2, 5).
Solution de l’exercice 21 Soit q un diviseur premier de (pp − 1)/(p − 1) = Φp (p). D’après le Corollaire 2, on a
q = p ou q ≡ 1 (mod p). Le premier cas étant exclu car q divise pp − 1, le résultat demandé en découle.
Solution de l’exercice 22 Écrivons
bn − 1 Y Y
= Φd (b) = |Φd (b)|.
b−1
d|n,d6=1 d|n,d6=1

16
Ainsi, pour tout diviseur d | n, d 6= 1, |Φd (b)| est une puissance de p. D’après le Théorème 7, cela implique que
pour tous diviseurs d, d0 | n, d, d0 6= 1, d/d0 est de la forme pk avec k ∈ Z. On en déduit que n est une puissance
de p.
Solution de l’exercice 23 On commence par remarquer que
n n−1
 n−1  Y
22 + 22 + 1 = Φ3 22 = Φ3d (2).
d|2n−1

D’après le lemme 3, on a Φ3d (2) > 1. Il suffit de vérifier que si d et d0 sont deux diviseurs distincts de 2n−1 , alors
Φ3d (2) et Φ3d0 (2) sont premiers entre eux. Supposons par l’absurde que ce ne soit pas le cas et choisissons un
nombre premier p qui divise leur PGCD. D’après le Théorème 7, d/d0 est une puissance de p, donc p = 2. Or
n n−1
22 + 2 2 + 1 est impair, ce qui implique que 2 ne divise pas Φ3d (2).
n0 n0 −1
D’après ce qui précède, le plus petit entier n0 > 1 tel que 22 + 22 + 1 est divisible par au moins n0 + 1
nombres premiers différents est le plus petit entier n0 > 1 tel que Φ3·2n0 −1 (2) n’est pas un nombre premier. Comme
n0 −2 n0 −1
Φ3 (x) = 1 + x + x2 et Φ3·2n0 −1 (x) = 1 − x2 + x2 pour n0 > 2, on vérifie que Φ3 (2) = 7, Φ3·2 (2) = 3,
Φ3·22 (2) = 13, Φ3·23 (2) = 241 sont premiers, mais que Φ3·24 (2) = 65281 = 97 · 673 ne l’est pas, de sorte que n0 = 5.

Solution de l’exercice 24 On va montrer que 2p1 p2 ···pn + 1 a au moins 2n−1 diviseurs premiers distincts, ce qui
conclura. D’après la Proposition 2 (i), on a
Y
2p1 p2 ···+pn + 1 = Φ2d (2).
d|p1 ···pn

D’après le Théorème 7, si Φ2d (2) et Φ2d0 (2) ne sont pas premiers entre eux, alors d/d0 est une puissance d’un
nombre premier. De plus, d’après le Lemme 3 on a Φ2d (2) > 1 pour d > 1 et on vérifie que Φ2 (2) > 1. Il suffit
donc de trouver une collection de 2n−1 diviseurs de p1 · · · pn tels que le quotient de deux quelconques d’entre eux
n’est pas une puissance d’un nombre premier. Pour cela, il suffit de choisir les diviseurs de p1 · · · pn qui ont un
nombre pair de facteurs premiers : il y en a exactement 2n−1 .
Solution de l’exercice 25 D’après le Théorème 8 (iii), le problème revient à calculer le produit des éléments a ∈
Z/pZ tels que φd (a) = 0 dans Z/pZ, qui vaut, d’après les relations de Viète, (−1)φ(d) Φd (0) dans Z/pZ. Pour
d = 2, cette dernière quantité vaut −1. Pour d > 2, celle ci vaut 1 car φ(d) est pair et Φd (0) = 1 car les racines
de Φd , de module 1, peuvent être réparties en couples de racines conjuguées.
Solution de l’exercice 26 L’égalité est équivalente à 1 + x + · · · + x6 = (y − 1)(1 + y + y 2 + y 3 + y 4 ). Comme
1 + x + · · · + x6 = Φ7 (x), d’après le Corollaire 2, un diviseur premier de 1 + x + · · · + x6 est soit égal à 7, soit est
congru à 1 modulo 7. Ainsi, un diviseur de 1 + x + · · · + x6 est soit divisible par 7, soit congru à 1 modulo 7.
Ainsi, y ≡ 1 (mod 7) ou y ≡ 2 (mod 7). Dans le premier cas, 1 + y + y 2 + y 3 + y 4 ≡ 5 (mod 7), ce qui n’est
pas possible, alors que dans le second cas, on a 1 + y + y 2 + y 3 + y 4 ≡ 2 (mod 7), ce qui n’est pas possible non
plus. Il n’y a donc pas de solutions.
Solution de l’exercice 27 On remarque que n2 + n + 1 = Φ3 (n). Afin de factoriser cette expression, l’idée est de
considérer des entiers n de la forme n = k m avec m un entier fixé non divisible par 3 défini ultérieurement. En
effet, dans ce cas, d’après la Proposition 2 (v),
Y
n2 + n + 1 = Φ3 (k m ) = Φ3d (k).
d|m

Si pour tout diviseur d de m on a Φ3d (k) < n = k m/2 , c’est gagné. Or en vertu du Lemme 3, on a
Φ3d (k) < (k + 1)φ(3d) 6 (k + 1)φ(3m) = (k + 1)2φ(m) .
Choisissons pour m un entier tel que φ(m)/m < 1/10. Ceci est possible. En effet, si on note (pn )n>1 la suite
X 1
croissante des nombres premiers à partir de p1 = 5, il est connu que la somme est infinie. Ainsi, si on pose
pi
i>1
mk = p1 p2 · · · pk pour tout k > 1, alors
  X k   k
φ(mk ) 1 X 1
ln = ln 1 − 6− .
mk i=1
p p
i=1 i

17
Ainsi, ln(φ(mk )/mk ) → −∞ lorsque k → ∞, ce qui implique que φ(mk )/mk → 0 lorsque k → ∞.
Mais alors (k + 1)2φ(m) 6 (k + 1)m/5 . Comme ce dernier terme est strictement inférieur à k m/2 pour tout k
suffisamment grand, cela conclut.
Solution de l’exercice 28 On remarque que an = (10n − 1)/9. Le théorème de Zsigmondy s’applique et fournit
l’existence d’un nombre premier p divisant 10n − 1 mais aucun des nombres 10n−1 − 1, . . . , 101 − 1. En particulier,
p ne divise pas 9, donc p 6= 3 ce qui montre que p divise an .
Solution de l’exercice 29 On réécrit l’équation sous la forme 19a − 2a = pb . Comme 17 divise le terme de gauche,
on a p = 17. D’après le théorème de Zsigmondy, si a > 2, il existe un nombre premier divisant 19a − 2a mais pas
191 − 21 = 17, absurde. La seule solution est donc (a, b, p) = (1, 1, 17).
Solution de l’exercice 30 Tout d’abord, k doit être impair. En effet si k était pair, xk et y k seraient des carrés et
il est facile de vérifier 3|a2 + b2 =⇒ 3|a et 3|b (il suffit de vérifier toutes les congruences possibles mod 3 pour a
et b). Si (x, y, k) 6= (2, 1, 3) et k > 1, on peut appliquer le théorème de Zsigmondy, qui fournit un nombre premier
p divisant xk + y k mais pas x + y. Or x + y divise xk + y k , ce qui implique que xk + y k admet au moins deux
diviseurs premiers. De plus, si (x, y, k) = (2, 1, 3) alors n = 2, et si k = 1, on a simplement 3n = x1 + (3n − x)
avec 1 6 x 6 3n − 1 et 3 - x.
Les solutions sont donc (x, y, n, k) = (2, 1, 2, 3), (x, y, n, k) = (1, 2, 2, 3) et (x, y, n, k) = (x, 3n − x, n, 1) avec
n > 1, 1 6 x 6 3n − 1 et 3 - x.
Solution de l’exercice 31 Soient p1 , p2 , p3 , . . . des nombres premiers impairs distincts. Posons nk = p1 p2 · · · pk . En
particulier, ani − 1 divise anj − 1 pour i < j. D’après le théorème de Zsigmondy, il existe un nombre premier qk
tel que qk divise ank − 1 mais ne divise pas ani − 1 pour 1 6 i 6 k. En particulier, cela implique que les nombres
premiers qk ; k > 1 sont tous différents, et cela conclut.
Solution de l’exercice 32 Posons N = 2np1 p2 ···pn +1. On va montrer que N a au moins 2n facteurs premiers
Q distincts,
ce qui impliquera que N a au moins 22 > 4n diviseurs. Soit A ⊂ {1, 2, . . . , n} et posons NA = 2 i∈A pi + 1, avec
la convention N∅ = 3. Alors NA divise N . D’après le théorème de Zsigmondy (qu’on peut utiliser car l’exception
23 + 1 ne peutY arriver puisque pi > 5), il existe un nombre premier qA divisant NA et ne divisant pas 2j + 1
pour 1 6 j < pi si A 6= ∅. De plus, on voit que qA 6= qA0 si A 6= A0 . Comme il existe 2n sous-ensembles de
i∈A
{1, 2, . . . , n}, cela conclut.
Solution de l’exercice 33 Comme m = 1 convient pour tous entiers a, n > 0, on peut supposer m > 1. Comme
a = 1 convient pour tous entiers m, n > 0, on peut supposer a > 1.
Si (a, m) 6= (2, 3), le théorème de Zsigmondy implique qu’il existe un facteur premier de am + 1 qui ne divise
pas a + 1 et donc (a + 1)n .
Si (a, m) = (2, 3), on voit que seuls les entiers n > 2 sont solution.
Solution de l’exercice 34 Si les hypothèses du théorème de Zsigmondy sont remplies, il existe un facteur premier
de xr − 1 qui ne divise pas x − 1. Comme x − 1 divise xr − 1, ceci implique que xr − 1 admet au moins deux
facteurs premiers et ne peut donc pas être une puissance d’un nombre premier.
Il reste donc à traiter les deux cas suivants :
(i) (x, r) = (2, 6) (qui ne convient pas),
(ii) r = 2 et x + 1 est une puissance de 2. Dans ce cas (x − 1)(x + 1) = pn , ce qui donne aisément p = 2 et
x = 3.

Solution de l’exercice 35 Réécrivons l’équation sous la forme y p + 1p = px . Si y = 1, on voit que p = 2 et x = 1.


Si p = 2, il vient aisément que nécessairement x, y = 1. On suppose donc p impair de sorte que y + 1 divise y p + 1.
Si les hypothèses du théorème de Zsigmondy sont remplies, il existe un facteur premier de y p + 1 qui ne divise
pas y + 1, de sorte que y p + 1 ne peut pas être une puissance d’un nombre premier. Il reste donc à traiter le cas
(y, p) = (2, 3) qui donne la solution x = 2.
Solution de l’exercice 36 Les entiers 2p − 1 et 2q − 1 divisent 2pq − 1. D’après le théorème de Zsigmondy, 2pq − 1
a un facteur premier p1 qui ne divise ni 2p − 1, ni 2q − 1. De même, 2q − 1 admet un facteur premier p2 qui ne
divise pas 2p − 1, et 2p − 1 admet un facteur premier p3 . De plus, par construction, p1 , p2 , p3 sont distincts.
Solution de l’exercice 37 Comme a = 1 est solution, supposons maintenant a > 2. Il est clair qu’alors n > p, q, r >
1.

18
Si l’un des entiers p, q, r est égal à n, on a a = 2 et les deux autres sont égaux à 1. On trouve donc les solutions
(a, n, p, q, r) = (2, n, 1, 1, n), (2, n, 1, n, 1), (2, n, n, 1, 1). On suppose dans la suite que p, q, r < n.
Si les hypothèses du théorème de Zsigmondy sont remplies, alors an − 1 admet un diviseur premier qui ne
divise aucun des entiers ap − 1, aq − 1, ar − 1, et on ne peut donc pas avoir an − 1 = (ap − 1)(aq − 1)(ar − 1).
Sinon, on a soit :
(i) n = 2 et a = 2s − 1. Dans ce cas, comme on a supposé p, q, r < n, on a p = q = r = 1, et a2 − 1 = (a − 1)3 .
Ceci implique a = 3 et on trouve la solution (a, n, p, q, r) = (3, 2, 1, 1, 1).
(ii) a = 2 et n = 6. Dans ce cas, on trouve aisément les solutions (a, n, p, q, r) = (2, 6, 2, 2, 3), (2, 6, 2, 3, 2), (2, 6, 3, 2, 2).

Solution de l’exercice 38 En regardant modulo 3, on voit que x est pair. On écrit x = 2w, de sorte que

3y = 52w − z 2 = (5w − z)(5w + z).

De plus, PGCD(5w − z, 5w + z)=PGCD(z, 5w ) = 1. On a donc nécessairement 5w − z = 1 et 5w + z = 3a .


En additionnant les deux égalités, il vient 3a + 1 = 2 · 5w . Pour a = 2, on a w = 1 ce qui donne la solution
(x, y, z) = (2, 2, 4). Si a > 3, d’après le théorème de Zsigmondy, 3a + 1 a un facteur premier p qui ne divise pas
32 + 1, ce qui implique p 6= 2, 5. Il n’y a donc pas de solutions dans ce cas.
Solution de l’exercice 39 Il est clair que p > 2. Supposons par l’absurde que a = uv soit composé. Alors d’après le
théorème de Zsigmondy, pu − 1 a un facteur premier q qui ne divise pas p − 1. Or pu − 1 divise pa − 1 = 2n (p − 1).
On a donc q = 2. Or p − 1 est pair, absurde. Donc a est premier.
Si a = 2, on trouve que p = 2n − 1.
Si a > 2, de même, le théorème de Zsigmondy implique que 2n (p − 1) = pa − 1 admet un facteur premier r
qui ne divise pas p − 1. Ceci implique que r = 2, absurde car p − 1 est pair.
Les solutions sont donc a = 2 et n tel que 2n − 1 soit premier.
Solution de l’exercice 40 On vérifie d’abord que (b, n) = (2, 6) ne convient pas. Ensuite, par l’absurde, supposons
que n ne soit pas un nombre premier et choisissons un diviseur 1 < d < n de n. Écrivons bn − 1 = pk (b − 1) et
appliquons le théorème de Zsigmondy : il existe nombre premier q divisant bn − 1 mais ne divisant ni b − 1, ni
bd − 1 bn − 1
bd − 1, ce qui implique p = q. Ainsi p - bd − 1, et | = pk , ce qui est absurde.
b−1 b−1
Solution de l’exercice 41 Supposons que (m, n) 6= (1, 1) (qui convient clairement), et aussi que a > 1 (si a = 1
on a la solution (a, m, n) = (1, (n + 1)! − 1, n)). On a alors m > n, et on peut écrire l’équation sous la forme
équivalente suivante :
a2 − 1 a3 − 1 an+1 − 1 am+1 − 1
· ··· = ,
a−1 a−1 a−1 a−1
ou encore
(a2 − 1)(a3 − 1) · · · (an+1 − 1) = (am+1 − 1)(a − 1)n−1 .
Si les hypothèses du théorème de Zsigmondy sont remplies, il existe un facteur premier de am+1 − 1 qui ne divise
aucun des entiers a2 − 1, a3 − 1, . . . , an+1 − 1. Comme m + 1 > 2, il reste donc à traiter le cas (a, m + 1) = (2, 6),
autrement dit a = 2 et m = 5. Dans ce cas, 3 · 7 · 15 · · · (2n+1 − 1) = 63, qui ne fournit pas d’autre solution.
Solution de l’exercice 42 Raisonnons par l’absurde en supposant que ar , as , at appartiennent à une suite géomé-
trique de raison b > 2, avec r < s < t. Alors

(3r − 2r )bk = 3s − 2s , (3s − 2s )bl = 3t − 2t (12)

avec k, l > 1. D’après le théorème Zsigmondy, il existe un nombre premier p divisant 3t − 2t mais pas 3s − 2s .
D’après la deuxième égalité de (12), p divise b. D’après la première égalité de (12), p divise alors 3s − 2s , absurde.

Solution de l’exercice 43 Si x = 0, on vérifie que y = 1, z = 2 et que si y = 0, x = 3 et z = 3. On suppose donc


x, y > 1. La suite est proche de l’exercice 38. Modulo 3, on voit que x est impair. Écrivons donc x = 2w, de sorte
que 3y = z 2 − 22w = (z − 2w )(z + 2w ). Le PGCD de z − 2w et de z + 2w est égal au PGCD de z et de 2w , qui
vaut 1. Ainsi z − 2w = 1 et z + 2w = 3y . En soustrayant ces deux égalités, il vient 2w+1 = 3y − 1. Si y 6= 2, alors
y > 3 et le théorème de Zsigmondy assure l’existence d’un nombre premier p divisant 3y − 1 = 2w+1 mais pas
31 − 1 = 2, absurde. Si y = 2, on trouve la solution (x, y, z) = (4, 2, 5).

19
Solution de l’exercice 44 Comme 2009 = 49 · 41, x49 + y 49 divise x2009 + y 2009 . D’après le théorème de Zsigmondy,
il existe un nombre premier divisant x49 + y 49 mais pas x + y. Comme x + y divise x2009 + y 2009 , on en déduit
que x2009 + y 2009 admet au moins deux facteurs premiers, absurde.
Solution de l’exercice 45 Comme dans la solution précédente de l’exercice 19, on commence par montrer que
si n > 2 et 3n − 2n = pk pour k > 1, alors n est impair. Supposons par l’absurde n = ab composé. Alors
(3a )b − (2a )b = pk . Comme n est impair, on a b > 2 et on peut appliquer le théorème de Zsigmondy : il existe un
nombre premier q divisant 3n − 2n mais pas 3a − 2a . En considérant un diviseur premier de 3a − 2a , on voit que
3n − 2n admet deux diviseurs premiers distincts, absurde.
Solution de l’exercice 46 Par symétrie, supposons n > m. Si les hypothèses du théorème de Zsigmondy sont
remplies, il existe un facteur premier de bn − 1 qui ne divise pas bm − 1. Ainsi bm − 1 et bn − 1 ne peuvent pas
avoir les mêmes facteurs premiers.
Il reste donc à traiter les deux cas suivants :
(i) (b, n) = (2, 6). On vérifie que cela ne donne pas de solution pour m ;
(ii) n = 2 et b + 1 est une puissance de 2, ce qui était demandé.

Solution de l’exercice 47 Notons q le nombre de diviseurs premiers de c. Supposons d’abord q = 1, de sorte que
c est premier. Si k = 1, il n’y a rien à faire. Sinon, si c 6= 2, on peut appliquer le théorème de Zsigmondy avec
k k
ai − bi pour chaque diviseur i de ck , ce qui nous fournit même k + 1 diviseurs premiers différents de ac − bc .
k k k−1 k−1 k−1 k−1
Si c = 2, on écrit a2 − b2 = (a2 − b2 )(a2 + b2 ). On applique alors le théorème de Zsigmondy avec
k k
ai + bi pour chaque diviseur i de 2k−1 , ce qui nous fournit k diviseurs premiers différents de a2 − b2 .
Supposons maintenant q > 2. On appliquer alors le théorème de Zsigmondy avec ai − bi pour chaque diviseur
k k
i de ck autre que 2 et 6, ce qui nous donne au moins (k + 1)q − 2 > kq diviseurs premiers différents de ac − bc .
Solution de l’exercice 48 Il suffit de trouver une infinité de couples de nombres premiers distincts (p, q) tels que
p | 2q−1 − 1 et q | 2p−1 − 1.
Soit p un nombre premier tel que p ≡ 3 (mod 8). Posons n = (p−1)/2. Alors d’après le théorème de Zsigmondy,
il existe un nombre premier q > 2 tel que l’ordre de 2 modulo q vaut (p − 1)/2. Comme p est congru à 3 modulo
8, p n’est pas un carré modulo 8, ce qui implique par réciprocité quadratique que 2 n’est pas un carré modulo p.
Ceci implique que l’ordre de 2 modulo p est différent de (p − 1)/2, et donc p 6= q. De plus, par cosntruction, q
divise 2p−1 − 1.
Notons ωp et ωq les ordres respectifs de 2 modulo p et q. D’après le petit théorème de Fermat, (p − 1)/2 =
ωq | q − 1. Comme q − 1 est pair et que (p − 1)/2 est impair, on en déduit que p − 1 | q − 1. D’autre part, d’après
le petit théorème de Fermat, ωp | p − 1 | q − 1. Donc ωp | q − 1, ce qui implique 2q−1 ≡ 1 (mod p). Ceci conclut.
Solution de l’exercice 49 On prouve d’abord (i). La preuve qui suit est dûe à Roitman [8]. On commence par
établir que pour tous entiers a > 1, n > 2, on a

n
Φn (a) > a 4 . (13)

Pour cela, montrons d’abord que Φn (a) > aφ(n)/2 . Déjà, si n est une puissance de 2, écrivons n = 2m . Alors
m−1 m−1
Φn (a) = a2 + 1 > a2 = aφ(n) . Sinon, soit q un diviseur premier impair de n. On écrit n sous la forme
i−1
n = q N avec N premier avec q. Alors, en posant b = aq
i
et en utilisant le Lemme 3,
 i
Φ N aq ΦN (bq )
 q
b −1
φ(N )
φ(N )  q−1 φ(N )
q−2 q−2 φ(N )
Φn (a) =  = > > b (b − 1) > (b ) > b 2 = aφ(n)/2
ΦN aqi−1 ΦN (b) b+1

où on a utilisé le fait que bq − 1 > bq−2 (b2 − 1) pour la deuxième inégalité. Il suffit alors d’utiliser l’inégalité


φ(n) > n/2. Celle-ci provient
√ du fait que φ est multiplicative, combiné avec les inégalités φ(2m ) = 2m−1 > 2m /2
et φ(pm ) = (p − 1)pm−1 > pm lorsque p > 2 est premier.
Revenons maintenant à l’exercice. Soient a > 1 et n > 2 tels qu’ils n’existe pas de diviseur premier primitif p
de an − 1 tel que |an − 1|p > nN + 1 (on va voir qu’il n’existe qu’un nombre fini de tels couples (a, n)). Soit p un
diviseur premier primitif de an − 1 tel que |an − 1|p 6 nN + 1. Ceci impose p 6 nN + 1. De plus, comme Φn (a)
divise an − 1, on a
|Φn (a)|p 6 |an − 1|p 6 nN + 1. (14)

20
Ensuite, d’après le Lemme 7, p ≡ 1 (mod n). Il existe donc au plus N diviseurs premiers primitifs p de an − 1
vérifiant p 6 nN + 1. Soit q le plus grand diviseur premier de n. En reprenant les notations de la preuve du
théorème de Zsigmondy (comme b = 1, Ψn = Φn (a)), on a

Φn (a) 6 λ · Pn 6 q · Pn 6 n · (nN + 1)N ,



n
où on a utilisé (14) pour la dernière inégalité. Mais alors, par (13), on obtient a 4 6 n(nN + 1)N . Comme il n’y
a qu’un nombre fini de couples (a, n) avec a > 1 vérifiant cette inégalité, cela conclut.
Pour (ii), soit s un nombre premier divisant an − 1 et remarquons que [an − 1]p > |rn − 1|s . Ainsi, si on fixe
N > 1, d’après (i), pour tout entier n > 1 suffisamment grand, il existe un nombre premier s divisant an − 1 tel
[an − 1]p
que |an − 1|s > nN + 1, ce qui implique > N . Ceci conclut.
n

Références
[1] http ://math.stackexchange.com/questions/660585/.
[2] E. Artin, The orders of the linear groups, Comm. Pure Appl. Math., 8 (1955), pp. 355–365.
[3] A. Bang, Taltheoretiske Undersøegelser, Tidskrift for Math, 4(5) (1886), pp. 70–80,130–137.
[4] G. D. Birkhoff and H. S. Vandiver, On the integral divisors of an − bn , Ann. of Math. (2), 5 (1904),
pp. 173–180.
[5] L. E. Dickson, On the Cyclotomic Function, Amer. Math. Monthly, 12 (1905), pp. 86–89.
[6] E. Lucas, Théorie des Fonctions Numériques Simplement Périodiques. [Continued], Amer. J. Math., 1
(1878), pp. 197–240.
[7] J. H. Maclagan-Wedderburn, A theorem on finite algebras, Trans. Amer. Math. Soc., 6 (1905), pp. 349–
352.
[8] M. Roitman, On Zsigmondy primes, Proc. Amer. Math. Soc., 125 (1997), pp. 1913–1919.
[9] M. Teleuca, Zsigmondy’s theorem and its applications in contest problems, Internat. J. Math. Ed. Sci.
Tech., 44 (2013), pp. 443–451.
[10] K. Zsigmondy, Zur Theorie der Potenzreste, Monatsh. Math. Phys., 3 (1892), pp. 265–284.

21

 
  

!#"%$&('*),+.-/$021435"%67'98:"%67;<1435"%676=?>5>=?'

@BA<C DFEGIH.JKFELLMNOGIPQPSR

T ASC

UEVMWC CM*XZY\[7]^I_a`7bdc5]

MN

@fegKihFEjie?eQLlknmpo\[7qbrb

s 0%t7>5=vu7=?'9wx021y3za&4=?'

{ |~}Q}€‚ƒ…„‡†ˆ}‰ Š

Š ‹Œ€Žp„‘€‰’“yQ‰ˆ„‘ƒ‘‰ ”

• –˜—<™Q‚†š„›“4S‰œ“yQ’ž†š„‘“ygQƒ…ƒ‘‰‰š™Q€ Ÿ¢¡¢‰š™Q€£/¡I‰š™Q€¤ ¥

¦ §¨‚.’“44†š„‘I™g„›†ˆ} ©

” ªŒ“4S‰š„…ƒ‘‰ †p}†š«Q“I¬Q‰ {Š
­®¯ °i± ² ²5³//´žµ¶‡· :³ ®
®¸®Œ®¸®
®¸®
®
®¸®
®¸®Œ®
®¸®
®¸®
®¸®
®Œ®¸®
®¸®
®¸®
®
®¹¯ˆº
­®»º ¼ 5½²µ5½::³rI¾ /¿À Á²¶  ®
®¸®
®
®¸®Œ®¸®
®¸®
®
®¸®
®¸®Œ®
®¸®
®¸®
®¸®
®Œ®¸®
®¸®
®¸®
®
®¹¯ˆÂ
­®»Â à 5³rÄÅ:³ ®
®Œ®¸®
®¸®
®¸®
®
®¸®Œ®¸®
®¸®
®
®¸®
®¸®Œ®
®¸®
®¸®
®¸®
®Œ®¸®
®¸®
®¸®
®
®¹¯ˆ­
­® Æ :´ µ˜ÇŐ  5²µd:³¢Wµ¶¶Á² ‡µ¶5½:³ ®
®¸®
®
®¸®
®¸®Œ®
®¸®
®¸®
®¸®
®Œ®¸®
®¸®
®¸®
®
®¹¯ˆÈ
­®»­ ½²Wµ5²µr :³%´ µ¶‡· :³ ®
®¸®
®
®¸®Œ®¸®
®¸®
®
®¸®
®¸®Œ®
®¸®
®¸®
®¸®
®Œ®¸®
®¸®
®¸®
®
®¹¯šÉ

Ê ËdÌ<€’„‘’‰ {©

¥ ÍI“yƒ…™S†š„‘“yQ‰ Šy”

¯
 6  &(0%>531  '
 
 

Œ:³¶¢¸½:W¶   % V¾ Á: Œ:³¶r˜¿… Á²¶  ®
à µrÇŲ !
" ½²5²µ¶²µ¢5 5:³% :³F¿… Á²¶ ³$#&%')(*' Á: ¶:³r5²  :³¢+( µ25 ³2µ5½:² ³-, ²-./
#102,34.5167#102,5839#102.5
: ´ž¸7µ5½:³5 µ5lÁ:²¶5l½:W¶ O¿À Á²¶ ²  (+  a<¾ :³¶˜³ ³˜ 5lW ³˜¶ Œ¿Vµ5
² :³¢µ4² ³ ®
<²;   ³¶¨Á:²µ¶³lÇŲµ5Á²Á::³( µ¶5²µ5 lWµ¿À ³³5µ:³Œ¿… Á²¶ ³lÁ:  ÇÅ:³7  ²Å
´žµ¶‡· :³= +  ³5 Á²</³¶  ÄWÁˆ¶   ³F¿À²µ7 5²µ¢ V¾ ÇÅ4 ž³5½/I³¶rŝ¿À Á²¶ ³¢I¾ 
´žµ¶‡· ¸µ5½:²   ®

> }@?ag„›†š„‘“y {BA  DCE FG-H IJ K&L=M@NM@O'QP


R 2STI UG~¿… Á²¶ V#WM@XC MY [Z\'^]_a` bZJ c ZdZJeJJIgfhiF @J [jIdk,WM@XC F4 FJ 
Illm]nG oO#102,85d]nJ X` & HYH[2p V¾ jM@-,&H[ IQ#qP
r ` [Z
Js!tJpOCubZ ^ bHYHp V¾ ²³5²Œ· ¸½ÇĶ VM@v#qP
wqx sQHbZ y Jz_{ Á ±  X,}|~'$a+ . ³5³5 Á²N#102,85$6W,G ®€ Z½ÇÄŒ³¶a¿… Á²¶ ZN'
³$' ®
tz{ Á ±  ²¶²µK‚B6…ƒ „[I+ . ³5³5 Á²7 7 Œ·µ5#102‚kr5 ½² ¨O Œ·µ5 l³5:³/´³5²µ5³
µ5²²µ5³ ® " ¸Á:²¶5/¿À:† DW +  ½ÇÄ%·² ˜¿À Á²¶   £n‡ ³O' ®
 ³= + Á: ¶µµ5² ²vˆ V¾ ÇŲ /µ5½:Á:½:²=+ ~/³¶( ž³5˜W ³2Á²Ir¿À µ¶Œ ¸³5 ˜
(²µ¶ ²¶5
¸½²5²µ¶²µ‰#102‚kF 5 µl:Š ³%
³¶ 
 Áˆ¶   ½²µ¶ ®
‹ .Á: Á² ³¶ Œ : ¶5²¶ \<Ž :³/¿À Á²¶ ³¸³5 ¸W ³¸5 5:³
½ÇÄ:³ŒWµ
:³˜¿… µ¶ :³
 ½²·µ¶:³% ³r    ³%³¶ :³ ®

> }@?ag„›†š„‘“y*ŠA  k#<%YC(*'‘ UGq



GPn’^& oQ#102Ch5q6”“•#102,85b–,<|—C™˜FP
š r ^J
›#9bZ nM2 oŒ³¶µœ¶:Á²¶´M@-CZ UI-žŸZbž67#102C~5d]TeP fUP M@P8Zby ¡ ,&|gC¢ _Z

2s! =£@NMY Z-ž¤
n ¡ D.g|—ž¤ @MsJ n  /s[Zc F~ a odMUa kH Iv#qP
š r nJ
_#4bZ yM o˜ ¶œ :Á²¶´_MUqC*Z UInž¥Zb ¡ F .¦|/žŸ YMsJ y  cH§ UZ$ U— a oldMY 
H[ IQ#qP R [Z‰¨ ‰HI  
[b]-dp ›Zbp£ S©XJ []8H  FIK ¡ UZOªa–d«c|—C™]1Zb#10ª[5167#10«
5\ ¨IbZOª67«P
r OJG
g#ŒZ n UGl · œ :Á²¶ 9M@OCEZb FIOž¬ZbnJ_bZ ^f!p QJ Zce­
= L®
1Zb FI2­
= L®
MU$C¯š Z UIOžjP
wqx sQH yŽS ¿… Á²¶ V#B%y,)°( ,  <¾ :³¶7W ³³¶µœ :Á²¶´~›' ³¶µX' Áˆµj±\² <¾»  ²lW ³
I¾»5½:Á:½: ®4à ³%µ5½:Á²³½² ²=W +  #102'q5Q6)'T³c+ ²% ÁKO# :³¶%³5µœ :Á²¶´
c' ³¶µ-'T³ ®
‹  Œ<¾ :³5d¶œ :Á²¶´ŒaX' ³¶µQ'$S+ a\f' ³¶µv' ³ Áˆµ=4+ ³˜ :³d²Å Áˆ ³=4+ Z#10´²e5-6
²O6µ#10´±\²e5 ®
à µrÁ: ¶µ5+O # :³¶¢¶œ :Á²¶´
c' ³ ³¶µ-'$S+  c'^#¶ ³¶µ$' ®
Ž 7¿À Á²¶ <# :³¶¢¸· ¶œ :Á²¶  ‰'q³ ³¶µ-'q³¸…·  ‰' ¶ ³5µ-'T³$5 ®
S

¹ s! I´
[ " ³%Á:Œ<³¶=W+  µlWµ¿À ³%X#&%@ºµ(*v » :³¶¢œ :Á²¶´¦‘· µ5:³¶ ® ³¶µ¶œ :Á²¶´+
· ¶œ :Á²¶´e2 ¼ ³³%µ5½:Á²³5²µ% ˜½ ‰fº ³¶µ$»W¾ ®
º
> }@?ag„›†š„‘“y #•…A   # % C (¯ž FGt ­
l
GP r ~¿À Á²¶  µ5½:Á²µ5 —MU‰#)Z ©¨ v
= ]
G olv#k¶ ]n L®‰#k¶ %Yž¸( CuJ ^M@2Sq¡-H I
  FIX ¡ .—|<ž!]y# ¶  02.5T6µ,–1›, bZ n` F
[sa -M@QC oJ J ‰#102,5T67. ePf@P MUP
,ŒbZJ Tl` UJ N a odlMY a -M@O.¦H Iv#zP
wqx sQH [ŽSl¿À Á²¶  µ5½:Á²µ5 
X#h%Y,—(*° ,¸ Á: ³5½²µ5½:
Á:  Œ½Çij¶µr²Oˆl´ž ²µ5³
³v'T³ :³¶˜ ‡¿À Á²¶ ™# ¶  %[,~
°( , ®yà µ
Á: ¶µ5(+ ³¶1p# :³5˜Á: ³¶½²µ5½:lÁ:  l½ÇÄl³¶µ
'©*¶ ²O7ˆ ´ž ²µ5³r³$' ³ +   µ5³Q#k¶  %,g°(E ± , ®
> }@?ag„›†š„‘“y ¦¸A  k#<%YC(*'‘ UGq

[]©J & U^H[ I ¡NM@OC}P
r ^

›#9bZJ TM o
Á²µ5 ³³5¡IlbZ¡H8PW³¶¶µ¶Á²5² ²%Á²µ5 ³5³5z_Z UI —pIbZJJ b]H  FI‰ ¡ UZ
ª–d«v| []1ZDªV« pIbZ$#10ª[5V#10«
5¡IlbZ¡H8PD#10ª 5Œ#10«
5ozYP
r -
= g#VbZ nM2 o7 ½:Á²µ5 ³³5¡IlbZ¡H84P ³5¶µ¶Á²5² ²r½:Á²µ5 ³5³ 5z_Zb FI /pIbZJJ b]H  UI
¡ @Z$ª–d«c| []1ZDª Œ« ¨IdZQ#10ª[5 Œ#10«
5 oIbZoH8P8#10ª[5 Œ#10«
5ozP
r -
= g#ŒbZJ TM o    5 !Z UI <¨IdZ
J 1` 2p_bZJ yZJ2 T
I´ZdZ
a o‰Zb FI <ZJ TM@
I´Z š
Z
a o‰Zb FI GP
’©~M@2ST2 ©M@Ks!  IN G ¨d£ [Xp !Z I
oKs!Y ¡¡KM` FGTJG
GP
¹ s! I´
[ ° Á²µl²Š 5² ²=<+  a(²v²; ¶µ5l ³©œ Á²²Å 7µ5²5²µ/5 
Á:² ‡ ³5 ³˜ ‡¿… µ¶ g
¿… Á²¶  Á²µ5 ³5³5 Á: ³5²µ¶´ :³
½² 5½:³=S + ¿À Á²¶  ½:Á²µ5 ³5³5µ5²´²µ5³5  \³5²³
:³%½² 5½:³ ®

wqx IJb à µ5 ´²µ%d³¶#&%C(*' :³5¨d¿… Á²¶  ³¶¶µ¶Á²5² ²2   5 ¸³¶µnC4+   µ5³-#


:³5¢œ :Á²¶´
\C,³¶µ-' ®

A m 
 ( ž³5 ³/_Z # ³5 /³¶¶µ¶Á²5² ²
Á²µ5 ³³57³¶µvC +4²˜Á: ³¶½²µ5 ³/²Å µ5½:² ³cª–d«N|hC ®
n ª  «   µ5³N#10ª 5 ¸#10«
5<+ ²7³¶T« ¸ªZ  µ5³X#10«
5 #10ª[5 ® " Á+S³Œ5 ³
:³ŒÁˆ ³e+<  
#10ª[5v67ƒ #10«
5 ® ‹ d³5¡+a# :³¶¢œ :Á²¶´Œ‰C ³5µ-' ®
€  µ³5 
/ ‡l!²;  ¸¿À:†  ³r ¸Áˆ ³% !<# :³¶%³¶¶µ¶Á²5² ²˜½:Á²µ5 ³5³ 5
³¶µOC ®#"
wqx IJb à µ5 ´²µ7 ³¶n#  % Cu( ž :³¶· ¶œ :Á²¶  ³5¶µ¶Á²5² ²Á²µ5 ³5³5&‘· µ5:³¶ ® ½Ç
Á²µ5 ³³5e˜ ¼ ³¶µ‰C4I+   µ5³K# ¶ :³5/½² ² ²Œ³¶¶µ¶Á²5² ²ŒÁ²µ5 ³5³5/‘· µ5:³¶ ® ½:Á²µ5 ³5³5e/¼ ³¶µ
ž®
A m 
 ( ž³5 ³¸# ³ ˜l· ¶œ :Á²¶ .³¶¶µ¶Á²5² ²ŒÁ²µ5 ³5³5 ®  ²Kª–d«|4ž ˆ´:Á$ ª ”« ®
 ²1,–.j|gC 5² ³a-#102,5q6) ª ²1#102.5167« ® :  µ5³e+# ¶ 0ª[5q6) , ²q# ¶ 0«
5q6). ®€ µ=+ ³5
 
ª Œ«® + Áž¾ :³¶? Á2-#102,5  #102.[5 ® ‹ = + Á:  $\# Á: ³5²µ¶´F F³²³?:³Q½² 5½:³¨³¶¶µ¶Á²5:³= + Áž¾ :³¶
 Á7\$ , ). ®(°˜® ˆ ®  ® #k¶  0ª[%5  #k¶  0«
54+ Á:Œg ³5³¶µ5N#k¶  :³¶˜³¶¶µ¶Á²5² ²¸Á²µ5 ³5³5
³5µ$ž ®
€  ¶µ5
/ ‡l!²;  ¸²Š µ5

Áˆ ³% !/O# :³¶%³¶¶µ¶Á²5² ²d½:Á²µ5 ³5³ 5 ® "
wqx IJb " ½²5²µ¶²µr5 5:³% :³%¿… Á²¶ ³QO# ³¶¶µ¶Á²5² ²˜Á²µ5 ³5³5:³˜³¶µ-'$I+ 5²  :³d/( µ
5 O,<|j'-S +  v#10#102,5516), ®
A m 
Â
 ^,<|j'
¼ 4:³5i¿V Á² /˜´½²µ¶ ÄW²µ2˜ ‡Œ¿… Á²¶  ½ÇÄ/Wµ $#102,516),4 µ25  :³¶2·²~
Š G€
³ ¶   µ5 · ²  ®  ´ž7µ5 ´²µr
Áž¾ :³¶¢ ‡l³5²  ®
$# Š [ª -#10ª 5v6 ƒ ª
   Á 2³5 ¶ \7µ5 · ²  ® 4 ž³5 ³¨<¾  Çų¶5Fµ5½:² Œ5²  ®
k#10ª[5 )ª e+
2 X# + !#10#10ª[55 )#10ª 5+
 ~  µ5³ ³5 O:³¶r³5¶µ¶Á²5² ²/Á²µ5 ³5³5 y    W²d Á
ª Œ#10ª[5
 FÁ:
<:³¶r·³¶µ5 ®
©#10ª[5 ‘ª =+ !; † D+ #10#10ª[55 ‘#10ª 5+
¸ ›ª ‘#10ª[5
 .  µ5³ Q ‡  ²  ¿V :   Q²7 Á  ¸Á:\F:³¶
½² ² ²˜·³¶µ5 ®
: ¡+ =+ D+ \ªG+
³¶ (³¸5 ³
:³
Áˆ ³ ( . ·¶²Á: ¶µ Á²¶  SÁ:¨ ³5³¶µ5ll4 µ¸5 
<#10ª 5167ª
 ·² ® "

> @} ?ag„›†š„‘“y*”A  k#<%YC(*'‘ UGq



GPn’^~M2 ©J c#ŒbZ %Wµ5 oIZ¡H8PyWµ5z!Z 
z   UIK ¡ F k,<|jC ]©~ ›0´±$,5^|gC9P
tz   UIX ¡ F ,&|—C™]n~ X#10´±$,5167#102,85 ¡IlbZoH8P8#10´±-,8516±O#102,5ozYP
wqx sQHbZ kQŽ ˜¿… Á²¶ g~# ½ÇÄ%Wµn#102‚k516 ‚D2 4 µ 5 i²¶²µT$‚ Œ„¸<¾ :³¶ W ³¨Wµ5+Áˆµ
³ \²³5²· ˜˜½ÇĶ ~Á²4:³5 Ÿ 4 + Á:d4²¶µ/r ‡ŒÁ: ¶  Y¼ <¾ :³¶iW ³F³¶³¿V5 ®
 à µ˜Á: ¶µ5 + ‡¿À Á²¶ <# ½ÇÄ
WµO#102‚k516 ‚Dd ( µ%5 r²¶²µ$.‚ :³¶¢Wµ5 ®
1Q Ž 
¿À Á²¶ /# ½ÇÄ+ 4 µ25 %²¶²µ-‚ Wµ$#102‚k516  Œ·µ5¸/´³5²µ5³2µ5²²µ5³%
‚Z:³¶rWµ5 ®
wqx IJb  UZ I §bz " ½²5²µ¶²µl5 5:³7 :³¿… Á²¶ ³#µ% ' ( ' 5²  :³l+S( µ
5 ³$,–.j|g'
#1
0 ®,5q6µ#    ,_3 .3}#    ,›± .
²
#102,  ±~.  5q6…02,!3 .[5´#102,¦±~.5D3702,›±™.[5´#102,34.[5
A m 
  O# 
³5 ¶  ½²´²¶²   ®
à µd5 -,&|j' ® ‹  Á ± ³¶³³Q.6 „[y + :´:Á¸ ‡7µ5²²Š µ5Á: ¶   I´²K
#1
0 ®,85q6 #   , 
‹ %( µ-.6 W+  ·¶²dÁ:²¶5¸¿… ³K…· ² µ5² ‡ˆ† O, WµK±-,85v
#10´± ®,85167# !  Y±  ,3  3}#   Y±  ,›±  /6 # ! "  #, /67#1
0 ®,5
à µr³¶5+# :³5¢Wµ5 ®
à µ/5 O.&|&' ® Q5²W
OÁ ± ³5Q,<6W„[W + ‡²Å²Š  Á: ¶ ZÁ: ¸  µ5³
ˆg
#10´±-.  5q6).[#10´±-.[5k±~.[#102.516).0#10´±$.[5 ± #102.5516 „
€ \²~½:iQ# Á: $ÀÁ²/:´:Ád ‡/¿À Á²¶    ˜³¶µT' ¶ ® ‹ ¨³5Q# :³¶¨Wµ5+ Áž¾ :³¶
 ÁŒ\# :³¶% ‡¿À Á²¶    Œ³¶µ-' 5 %²¶²µ ®
% ½:Á²µ5 ² ²=+ ‡¸¿… Á²¶    /³¶³¶¿Ài :³iÁ: ¶ ³2µ5 · ²Š  + ²2:³¶i Ád·²
Œ³5 ¶  ® "

Æ
> @} ?ag„›†š„‘“y#ʅA  O#%nC ( ' UG\

GP A    „Œ U IlJpPN’^7M KJ ›# bZ  š
(½²µ¶  4½²µ¶ ¸¸4½²µ¶ $zNpIbZJJ 
z   UIK ¡ F k,<|jC ]©~ \,3”|jC
tz   UIX ¡ F ,&|—C™]n~ X#102,85q67#102,N3 c5
wqx IJb l’   
Jz  Oª  µ5½:² ®  Q#<%Y')( 9' 5²  ¸+( µ¢5 %µ5½:² D,4
#102,3}ª 5T6 ² 3  #102,85 ±Œ0#102,55 
Y¼ à µ5 ´²µrK# :³¶¢(½²µ¶  ®
·¼ à µQª6…² +  ²µr ÇŲ 
I¾ ¸5²  /¿À Á²¶  ®
A m 
 ‚  # 5²   ¿À Á²¶  ® 5 ³7Q+ I¾»µl:Š ³ V¾ ½:W¶  ¿… Á²¶ ²  ~²Œ( µ
5 dÁ:² ‡%·²  ³5²³e +    Œ#102,5 B¸² 4 µ¢5 O,<|j' ®
 ›%Y,j°( #102,5[±  ® :  µ5³=+ž( µ 5  µ5½:² , +Y„ G02,85  ® " ¢ ³e+I¾»µlŠ:³ V¾ ½:W¶ 
¿À Á²¶ ²  
 
G02,39ª 5q6 ² ±Œ0G02,855 

à µr³¶5›0 G02,39ª 55 Q6   ±Œ0 G02,55 @ + ²% Á
0 G02,!3 ª[55  6  ² ±Œ0 02,!39ª 55  6…0 02,55 
°  l5 ¸:³¶/( ž³¶¶ ¿W +  a´²G02,¦3 ª 5Q6 G02,85<+ Á ® ˆ ®  ® #102,!3 ª[5O6¸#102,85(+ Á:7? ³5³¶µ5
\# :³¶ ª V4½²µ¶  ®
  €  (²¢WµrÇŲ 
Á ± ³¶µv#&%,g°(   "!$#  , %  3  ® "
 
& ' &4= wx3=a&4' ;g"%67'W=?35>'

€ lŠ
 
³¶( ž³5¸W ³d¸ ± ½: µ ² :³d  ¸µ5½:³¶ 5³%½²½²µÅ ( µ%µ5½:³5 µ5:³r½:W¶ ³

¿À Á²¶ ²  :³ À  µ5³\<¾  d² Çų¶5 ( µ :³ ½:W¶ ³  ½²·µ¶:³\ ³5:Á: #²žµ5½ Wµ
e¼ =+ Ž
ÇŲ  ®Ž° ²(²  ¸4²Q½²²µQ² :³Iµ¶Á²(:³<½²½²µÅ ® <:³S Œ·µ5²ŌÇŲµ5Á²Á::³
<³¶´²r²   ¶µ5²µ5 d V¾ ¶ 5½ ®

Š - =+ e+
°i± ²µ5Á ± ²µ¨:³?³5 ¶ ³ Wµ¶¶Á²  ²µ5:³ ³¶ :³ Ž¿… Á²¶ ³¨Á: ³¶5:³ *) :³ 4  ,+  :³ ®
(
+
° ² ‡ (²µ¶ ²˜²Å µ5½ ± ²²µ/l½:W¶  ¿À Á²¶ ²   4²/7³5¸¿Vµ5Œ½:
‹ =+
:³r³5 ¶ ³ ®  Wµ¶¶Á² ²µ W¸³ˆ´ µr³ˆ¾  < ²  ®®®

( " Š $#102,85+Wµ%ÇŲ \#10„@5+#10´²e5+[#10´±\²e5+#10


Y5+
½²5²µ¶²µ%:³¢´  ²µ5³2Wµ¶¶Á²  ²µ5:³r4 µ
²5Á ®
 5 ³ai³¶[#10„@5S #10´²e5<F4²y;²¶µ52½²5²µ¶½+  (²y;²¶µ5œÁ²²ÅF FÁ: ³¶½²µ5²µ
Á:  
 Wµ!²Š ¶µ5 ®

( ‹ ¶²µ :³¸µ5 µ¶½²5½:³:³³5 ¶ ³Œ½²´²¶²  :³›<œ :Á²¶´5½<+ ³¶µ¶œ :Á²¶´5½<+ · œ :Á²¶´5½+
Wµ¶5½+    5 W+ ³¶ž+ (½²µ¶ Á²5½ ®®®
( ‹ Å  5²µ% :³%³¶  ½²¶µ¶:³%½²´²¶²  :³d¸ V¾ ½:W¶  ¿À Á²¶ ²   ®

­
Ž ¾ ½: ½²½²µ  ½² ½²²µ\³5 )³ ² Á: ¶µ5:³ ³¶µ :³ ³ ¶ ³ ½²´²
¶²  :³ O V¾ ½:W¶  ¿À Á²¶ ²  .( µ 5²µ O ·µ5pZÁˆ5³³5 ¶ ³ %² ³5 e+
vˆ : Qˆ
µ ²²µ ¸³¶ :³%´½²µ¶ ÄWÁˆ¶ ³ ® ¶5²¶  5 5Ç¿… ³ l/W ³r · ²µ% :³%µ5½:Á²µ5 :³ D
wqx IJb  A pL®J Jz " ½²5²µ¶²µd :³r¿À Á²¶ ³v#™%@'B( ' +
5²  :³d ( µ%5 ³dµ5½:² ³
,k–.&
#102,¦± #102.[5516…²$±~,›±™.
A m 
  K#  ½²´²¶²   ³5 ¶  ® :  µ5³=+I².Á ± ³¶³5³\.<6 „ ²‰,™6 ªX3)#10„@5+( V¾ ½:W¶ 
¿À Á²¶ ²  O³ˆ¾ ½:Á²µ¶j#10ª 5<6¢±$ª›3 ²_±µ#10„@5+i4 µ 5 \µ5½:² ‰ª ®F°˜® ˆ ®  ® # :³5  ‡Z¿À µ¶ 
,j°(E±-,3  + !  :³¶¢
Á: ³55
µ5½:²   ®
% ½:Á²µ5 ² ²=ž+ ³5 ²  |j' ²q#&%,—°(E±-,T3  ®à µ?5 ³aµ5½:² ³y,–.  c#102,-±N#102.5516
±$,›±~.\3  ® " Á+aO# :³¶¢
³ ¶   µ5 · ²Š  Œ³¶<²%³² ² ² d³5  6  ®
 W ² ²=+  < 7¸¸³5 ¶  S:³¶O#<%Y,j°(E±-,_3  ® "
wqx IJb " ½²5²µ¶²µF5 5:³¨ :³¨¿À Á²¶ ³©
ˆ 4²5rÁ: ³¶5+ Á ® ˆ ®  ® :³?¿À Á²¶ ³-#/¶ %@')( '
( µS :³²  :³Q Çų¶5¨Œµ5½:² / ª 5² ˆ+ ( µS5 ³k,–.j|j' :´:Á©, 6)ƒ .š+ l
 ¶  6 ª ®
A m 
 ¼ ¶¶´² ²=4+  4²%µ5½²´ µd<¾  <³ˆ¾»žd:³2¿… Á²¶ ³˜*) :³ ®à µ5 ´ ³¶V  ®
 g#  ¿À Á²¶  p ˆ (²5 Á: ³¶52+ ½² hˆ ª ®ià µ~5 ³,k–.…|¸' :´:Á<,¬6 ƒ .+
   Ág#102,5n±V#102.[5K6¥ªG02,&±}.[5 ® ‹ =S +  4²µ5²µ5²µl Á:²¶5 µ5² ‡¶  µ5:³¶5 ´µ
!²;  ³¶,/6B. ®à µ/³¶5y + ² Á ± ³5³5³v.¦6B„[y+  ² ½:˜W+ 4 µd5 rµ5½:² ,k+ O—
#102,8516 ªU,_39#10„@5+ Á:¸S ³5³¶µ5Œ\# :³¶r*)  ®
% ½:Á²µ5 ² ²=¢+  ¸:³¶~¿À Á² Z ´½²µ¶ ÄW²µ <¾  ¿À Á²¶ 9*) Z:³¶ ·² ³5 ¶  
µ5 · ²Š   ® "
wqx IJb " ½²5²µ¶²µr5 5:³% :³¢¿… Á²¶ ³O#&%@' (*' 5²  :³r+ ( µ%5 ³-,–.j|g'
#102,N34.5q6),39#102.5
A m 
   IJsN Il›sJ 2iF=M@< ‹  Á ± ³¶³5³ K.h6‘~ „  ´²X#102,5v6 ,›3 #10„@d5 ( µ
5 \,V|}' ®
% ½:Á²µ5 ² ²=+ 5 5r¿… Á²¶ ~d ‡¸¿À µ¶ Q#&%,—°( ,c 3 ®+ ! d:³¶¨˜Á: ³¶5˜µ5½:²  ˜:³¶
Á² ‡µ5² ²d³5 ¶   µ5 · ²Š   ®
 R  x  s\sJ 2i =M@FŽS/ ²Œ·µ5¸/Á ± ¸˜ V¾ ½:W¶ ~¿… Á²¶ ²  Œ:³¶¢³¶  ½²¶µ¶
² , ²¦.2 +  ÁZ   ²Œ·µ5.Zµ5 5Z  V¾ ²; ¶µ5.³³¶ ®¢Ã µ ³552+ 4 µ~5 ³j,k–. |¥'-+
,›3 #102.5O6¸.X3 #102,85 ®<° ?Á: Xˆg#102.5q±}#102,5Q6….± ,k(+ ²¸  ¶µ5³¶?!# :³¶\ˆ
(²57Á: ³¶5 ® Ž ¾ ÇŲµ5Á²Á:7µ5½:Á:½:²/(²µ¶ ²˜I¾»*) µ¶ ²µ¸XZ # :³¶/*) y+ Œ4²5½² _ˆ
² ® QŽ lµ5½:Á²µ5 
:³¶Oˆ  ´ˆ  ½:‡5 ® "
wqx IJb " ½²5²µ¶²µr5 5:³% :³¢¿… Á²¶ ³O#&% ¤ (*' 5²  :³r+ ( µ%5 ³-,–.j| ¤
#102,34.5167#102,5839#102.5
A m 
 ¼ 4:³5FÁ² ‡µ¢r :³i¿À Á²¶ ³2 ½ˆµ5:³¢³5 ¢:³2³5 ¶ ³¢ µ5 · ²Š   ®a€  ´žŒµ5 ´²µ2
Á:¸³ % :³%³² :³ ®  O# /¿À Á²¶  ´½²µ¶ Äyd :³%Á: ¶ ³r µ5 · ²Š   ®
 R J oJIbs_2  hMU\L® pJ UIbZ-H IJ J U§ IZ
P

È
à µ c,h6”.—6W„[+ 4 Q´²˜ ½:‡5² ² N#10„@5$6W„ +
®yà µ
Á: ¶µ5 (  Œ(²¸½²5²µ¶²µ
#10´²e5X· ¡+
…Á:7a:³¶˜ µ¶ y³ V¾ . ´ 5 5:³d :³d¿À Á²¶ ³˜ ½ˆµ5:³¸Á: ´²W² ® b¼
à ž³5 ³d Á K#10´²e516 ª®
 ³g  µ5³
#10 Y5q6µ#10´²3›²e5q67#10´²e53N#10´²e5T6 ª " !; ©#10 Y5167#10 3›²e5q67#10 Y53N#10´²e5T6
 ® ¨ ²  


ª ‹ † +
® F2¿V :  ½²½²µ  ³¢( µ 5  n,<|
¤ ~ \#102,O3™²e5y67#102,85@3/ª+
rµ5½:Á²µ¶µ5²Á:
³ ³% )Á² 5½
Á:  vˆ#102‚k5167ª@‚q+
( µ¢5 r²¶²µ $ ® Q‚ V„
  I2 o=P
" N#10„@5^6W„[W+ O½:˜+ ( µd5 ˜µ¶ ² y,ky+  ¦#102,5D3Œ#10´±-,5^6W#10„@5^6B„[+4Á:
Q ³5³5µ5ŒKZ # :³¶%Wµ5 ®4à µ˜³¶5W+ ‡µ5² ‡¶ h#102‚k5T6Bª@p‚ :³5r´ž ‡· 
( µr5 d²¶²µ
‚®
ˆ ½²5²µ5Á:²¶5µ5² ‡¶  ˆ 5 ³/ :³˜µ¶ ² ³ ®Ià µ
Á:² ‡ y+  a³¶ ) ¸
¼ g7µ5:³¶5Œ ³¸<¾ \
µ5²µ5²µ\? + Wµ\  ´²   µ5½:Á²µ¶µ5²Á:  ½:‡5 …· ³5µ‚k¼a+ V¾ ½:W¶ #¿À Á²¶ ²  
Á: cˆ#102‚8,85T6µ‚#102,F 5 ( µ%5 d²¶²µQ‚  \„ ²r5 dµ¶ ² k, ® " ³dÁ::³dÁ: ¶ ³=+
( µ¢5 ³r²¶²µ5³ – ˆ´:Á
   V„ W–  g
ª j67#10 5167#
 
 6 # 








I¾ !& # —6 ª ®


° ¸Q ³³¶µ5¸\O # :³5¢·² ½ˆµ5
³¶µ ¤ ®
   

"

 +.-/$0¢1y35"%67'98:"%67;S1y35"%676=?>>5=?'9'y$Œ& 'W$&'y$Œ&

° ²µ¶:³r½:W¶ ³2¿… Á²¶ ²  :³d( µ¶5²r³¶µ%:³¢¿… Á²¶ ³%½ÇÄ:³d³¶µ¢ :³%²¶²µ5³ …  X·


¼ Ž
:³2µ¶ ² ³ ® ¾ ÇŲµ5Á²Á:ŒÁ² :³5³¶³%  ¶µ5½
Á:  ²rÁ:˜ 4/¸Á: ¶µ5
( ´ž ¶ ²µ kœ

\µ + ¢¿À  ² (³¨ ‡
µ5½:³5 ¶  ‘¶ ³¶ \( ž³5³¶· r% ‡¸µ5½:Á²µ¶µ5²Á: ®¼ W(²¨½² Ç e¼
; ^ˆ
 ²iµ¶µ¶´²µ¨2 V¾ 4 ž³%Ål³5 ¶ ³¨I¾ ²¶µ5%½² ² ² ˜´  ²µ5³ ²¶ ²µ5:³ Á:% 5 Š =+
I¾»r :³%³ ¶ ³%( ž³5³¶· :³ ®
" † +
¿V : Z½²½²µ  I5 5Á: ¶µ5 ( ž³5½: ÅZ³5 ¶ ³
½²´²¶²  :³  ²¶µ5Á: ³¶  \;
+
 ½²µ5½:lÁ:  77¿… µ¶¶ .³5 ½² ²µ5 I´ µ5lÁ:  7µ¶  \ ¶µ5l³¶( ž³¶¶ 
 _ˆ
( µ¢ ³d²µd³¢ ‡lµ5½:³5 ¶  / V¾ ½:W¶  ®

‹ =+ h#<% (
 Wµ¶¶Á² ²µ W  4²¢µ5²µ5²µd<¾ ˜¿… Á²¶  £ £ :³5r*) 
³¶<²r³5² ² ²
¡+
5³  4 µ?5 ¨²¶²µ ž  q‚q+ ‰#102‚Q3h²e5F±#102‚k5y67#102‚k5F±›#102‚\±—²e5 ‹ =+ ® (² ž \½:?¿À Á² ² ²
· =+ b¼
Wµ˜²Å µ5½:Á²µ¶µ5²Á::³ ‘ V¾ 
( µr :³˜²¶²µ5³˜4 ž³5¶ ¿…³  V¾»¶µ5
( µr :³d½²¶ ¿…³ %I¾»µ :³ lŠ
=+ $‚q+ _#102‚k516)‚T0#10´²e5k±~#10„@5583 #10„@5 Ž
‡7µ5² ‡¶  Á² :³³¶³ ( µ¢5 %²¶²µ    ® Qlµ5½:Á²µ5 
½²% ½:‡5 ®
Ž ^#&% ( ¦#<% ( +Gˆ
<%µ5½:³¶ iµ5:³¶5%´µW( µ¨ :³ ¿… Á²¶ ³ Ÿ £  Ÿ Ÿ Œ:³¨Á: ¶µ5:³2µ :³ lŠ
O#10„@5 O#10´²e5
³5µ 2² ®

wqx IJbM` bHI bZ^HIbH Zb }’   Jz " ½²5²µ¶²µ 5 5:³ :³¿À Á²¶ ³ ¦# % Ÿ ( Ÿ
5²  :³r
+ ( µ¢5 ³$,k–.¦| Ÿ
#10#102‚k583V#10 <5516)‚›3
! "

A m 
  O# 
³5 ¶  ½²´²¶²   ®

É
€
  µ5²µ5 5 \I¾»·( µ5  /# œ
:³¶ ¶:Á²¶´ ® #I² ³¶ ‹ =+ ~–‚ ³5 ²Å ²¶²µ5³
W¶µ5² ³d5² ³r ! K#10 <5q6”#102‚k5+ Q‚›3 6B#10#102‚k5839#10 <55T6”#10#102‚k5839#102‚k55T67‚›3}‚
  µ5³ ®
" j‚<6
" !
¾ ! ®
‹
 ¶ ³ ‡ (²5  ™#
®¢Ã µ 5  ²¶²µ /‚ ²+ V#10#102‚k5^3…#102‚k55 6 ®‚¬6
 % 9
02‚±~²e5 3}02‚K34²e5T67#10#102‚±~²e5 3h#102‚K34²e55 Ž œ ® ¨¾  :Á²¶´5½/ Q#
Á: ¢  µ5³ -ˆK#102‚k5 3h#102‚k516
#102‚j±V²e539#102‚3 ²e5 Ž =+
® Slµ5²µ5
Á² :³5³5³ (²µ¶ ²d  µ5³%I¾»*) µ¶ ²µd O:³¶r*)  ® \#
=+ =+
 % ½:Á²µ5 ² ² Wµ¶ :³S¿À Á²¶ ³?* ) :³ Ž :³¶Q¿À Á² Fi´½²µ¶ ÄW²µg¨ ‡d³5² F³5 ¶ 

 µ5 · ² :³¶%Á:²  
½ÇĸWµ v#102‚k516)‚
Z4 µ¢5  ® O‚ "
wqx IJb    s 
z " ½²5²µ¶²µ\ :³l¿À Á²¶ ³›#¸% Ÿ ( Ÿ ³¶¶µ¶Á²5² ²~Á²µ5 ³5³5:³
5²  :³rK#1
0 Y51 6 ²=+ 4 µr5 ³%²¶²µ5³ ~–O‚ µ5²²µ5³d²¶µ5
²Åh


! #10 j‚k5y67#10 <5´#102‚k5


!

A m 
 N# ³ ¶  ½²´²¶²   ®gà ³5# :³5Œ³¶¶µ¶Á²5² ²7Á²µ5 ³5³5Q +  —$„ ¸#10„@5 
#1 0´²e5  #1
0 Y5q6 + I¾ !<#10„@5167 „ ²O#10´²e5q6² ®
€ ( ž³5n#10 Y516 3  + !  | Ÿ ® :  µ5³1#10 Y5167#1
0 Y5´#10 Y5q6 [3  ® " ¾ !_#10 Y5  3  ® Ã µ
³55Q#10´²
„@5q67#1
0 Y5´#10 Y5 µ²
„q3    + I¾ !g#10 Y5  13   ²^#10´² Y5 µ² 13  ® ‹ F³¶¡+ #10´² Y5 
     

² n3  ®  ³=+ I¾»¶µ5˜Wµ¶=+  N#10 Y5  T3   Á‰#10´² Y5q6µ#10 Y5´#10 Y5 B0 T3  5J0 n3  5 ®€ 
  




² ½:˜!0 Q3  5J0 O3   5 B² O3  + Á:¸SÁ: ¢¿V Á² ² ²cˆ  6 „ ²Q#10 Y516 ®

  
    
 
 

à µ5 ´ ³%5²W˜Wµ%µ5½:Á²µ¶µ5²Á:Œ + 4 µr5 $‚™| Ÿ ‡ #_0 c37²e5q6 \3 ² ®


 €  ´²r/´ µr¸Á:¸µ5½:³¶ r:³5¢´µI( µ$‚<6² ®
 

F $‚™| Ÿk‡ ÄŽ ®a€  ³¶( ž³5¸\#


0 \3 ²e5q  6 c3 ² ® :  µ5³\


# ! ³  3 % 6µ#1
0 Y5´#
0 3)²e5n6 ³  3
 

" r ³=+ ³5c# :³¶i³5¶ µ¶Á²5² ²¢Á²µ5 ³5³5+ ²  d:³¶Fœ :Á²¶´+ ²i Ád :³¨ ·µ5:³$#1
0 ©3
Y5b–d#1
0 3 Y5b– ®–d#1
0 ³ 3 Y25 ³5 r²ŗlˆ ²Å~³¶¶Á²5³e+ Á² ‡ ³5³5½:³¢³dÁ:²% µ5µ5+ ²rWµ


¶²²TˆK“ 3 F– •– ³  3 U˜ ®žÃ µ ³¶5+Y#1


0 3 5q 6 k3 I4 µa5  T|~“ F– F– p– 13 U˜@+
  

Á:¸Iµ5 ´Œ² Wµ¶¶Á² ²µ˜X# ! ³  3 ² %  6 ³  3


² ²r Á ± ²Š ´Œ ‡lµ5½:Á²µ¶µ5²Á: ®
         
 

Ž F!²;  %µ¶ž ² Á: Œ·W5 µ5r4²µ¶ ²   µ5³?2½:µ5¢^#102‚k516 ~‚ ( µa5 a²¶²µ


<
‚$Œ„ ®
% ½:Á²µ5 ² ²=+ V¾ ²¶5½:³¶¢·² 
³ ¶   µ5 · ²Š   ® "
 ³=+ V¾ ²³5²Œ·  :³²¶²µ5³\W¶µ5² ³\4 ž³5³d:Š ~ µ5 µ¶½²5½ ¿À  ² }…·   ³
  ²¶¶µ5 ³rÁ²¼c

‹Œ€Ž“Q€ „‘}†ˆ}.{

 "!$#&%
('(*)
) +&'(,-, Ÿ #,/.0123)4!65798:!;1('7=<>5(<>.0>)>?

‹ =+Á:  ¸ ³˜   ³¢ ¸´ µ=+Á:²¶5¸µ5 µ¶½²5½


4²% ³O;²¶µ5¸·² ¶  ®®®
wqx IJb ¹  Us! ¡
z  ²$#<% Ÿ ( Ÿ ³¶µœ:Á²¶´+² ¦% Ÿ ( Ÿ œ:Á²¶´d5²  :³i+
( µ¢5 $‚™| Ÿ Q +  Q#102‚k5 G02‚k5 ®à µ5 ´²µd\#g6  ®
A m 
 à µ% V¾»·³5µ5!³¶( ž³5 ³¢<¾  SÇŐ³55/ ²¶²µO\ ª 5² S\#10ª 5‰6 ƒ G0ª[5 ®
@
:  µ5³=+ˆ V¾ ²³5²Œ· nº 6”“=‚y#102‚k5‰6 ƒ G02‚k5d˜F:³5I?Wµ¶¶  
´  Ÿ +²S Á V¾ ²³5²· 
»¸6B“$G02‚k5b–‚™|—ºK˜Œ:³¶2 I³³¶(/Wµ¶¶/  ´/ Ÿ ®S° ²%²³5²Œ· ‰»v  ²% Á/
 ³%4²¶r½² ½² ²= +  5½ 0«J5y+ :´:ÁX«v|—º ®  5 ³%<¾»  µ5³ G0«
5 Œ#10«
5 ®
à 
 
 5
³ 
 
 
 # :
 ¶
³

 ³¶µ¶œ :Á²¶´I
+  ¨Çų¶5lp²¶²µ l5² #10 =5v6 0«J5¸#10«
5 ® ‹ =+<².Wµ
¶Á² ²µ=I + . —6 ƒ « ®(à ³  :³¶/¶œ :Á²¶´I+ .   µ5³0
56 ƒ 0«J5v6¸#10 =5+II¾ ! !| ºB²
0
 5 Œ#10 =516 G0«
5W+ Á:¸SÁ: ¶µ5:r ‡7 5½Œ G0«
5 ®
" Á+ 4 µ¢5 r²¶²µQªG+  #10ª[516 G0ª[5 ® "
wqx IJb 1Gl FIdZ OI´  Jz  N#)% Ÿ ( Ÿ · ¶œ :Á²¶´ ®Sà µ5 ´²µl<¾  FÇŐ³55¶µ5 ³
²¶²µ5³%W¶µ5² ³Qªa–d«•– /5² ³r‰ª Œ«  ¸²O#10ª 539#10 =51 6 #10«
5 ®
A m 
  \Z # · œ :Á²¶ Z Ÿ ³¶µ Ÿ ® :  µ5³=W+  aÇų¶5K‚9| Ÿ 5² a_#10„@5 µ#102‚k5\…· ³ ³˜ ¡+
#10„@5837¸² WµrÇŲ ¸<¾»µ%W ³%I¾»5½:Á:½:/WµO#D¼ ®
° ² ‡ ³d4²µ¶ ²/I¾»*) µ¶ ²µ˜
V¾ ²³²Œ·   6¸“=‚4| Ÿ 1#10„@5 7#102‚k5d ˜ :³¶rŒWµ¶¶
  ´¸ Ÿk‡ ¡ + Wµ%Á: ³½:²=W+   ²%  ³2(²¶%½² ½² ²=W+ ˜ V¾  ´žŒ 5²µO« ®à µ
³55 +  _„ Œ˜« ²Q#10„@5 Œ#10«
5 ®
  ‚< 6 #10«J5Y±N#10„@5 ® à ³-\# :³¶Q· ¶œ :Á²¶´ +  ÇŐ³55i Á2²¶²µ i5² ^#10 =5q6)‚ ®
: ³¶¡+  _#10 =5G34#10„@5q6 #10«J5 ® " / ³$#10 =5q67#10«
5G3)0#10«J5k±™#10„@55  #10«J5  #10„@5+ Á:˜
 ³³¶µ5¸ ‰
|  ® <Ž 
Áˆµ Á²l²Š µ5Œ SK˜« Á: d  µ5³vˆ«  ®
: ³¶¡+ ² 4 ž³$ª!6 „[+  <´²Oª Œ «  ¸²O#10ª[5839#10 =5q6 #10«
5 ® "

0 ;Q"%6d1y36˜$731 
> }@?ag„›†š„‘“y ¥”A  Q# % )( ' FGX

[]X µM@bZbp£G< FG/Il UaBMk` 2a oIL• 2pZ<M@
pU£ [ FIbZK< U2bZXM@O'OP A  ª¦| GP
r OJG
g#ŒZ ^M2 o7Á: ¶/jª&Z   #102,85167#10ª 5
P
A #ŒbZJ ©la a [_gª¦H  FIK ¡ ª›| ]n~M2 ©J c#ŒbZ dÁ: ¶Z UI% GP
" ³iÁ:%³¶=+ ³¶ V¾ Wµ¶ %%Á: ¶5½/³³?µ5½:Á²³5²µi ¢ + Á:²  ÇÁ²W:³5  Á² 
5² ²d³¶( ž³5½:¸³¶µ¢ V¾ ²³²Œ· Œ¸½ÇĶ  ³¶µ¢ :² S  µ5½:³5 ¢ V¾ ½:W¶  ¿À Á²¶ ²   ®
 ¢Áˆµ Á²5½²µ¶³¶ \µ¶r2 ‡/Á: ¶5½˜:³¶  ½:%³ ‡/µ5 µ¶½²5½%³¶´ž5K…· 
 ³d  ²¶¶µ5 ³rÁ²¼c

‹Œ€Ž“Q€ „‘}†ˆ} Š

 + 5( 8 ) # ('(


) 8=!6% <<,>)  8 
 # 
)1('(
) #  8

)6(') >) ª 8>':1"8 135(>.0>) 8>' !;3%    813'7  02, 5  3'!
)6+>%#"
+>%>8 ª  
) # $ #02,  5q67#10ª[5 ? 

  %

wqx IJb'$& U £Y I¡ )(´z " ½²5²µ¶²µ/5 5:³/ :³d¿À Á²¶ ³\#4%'W(¯' Á: ¶:³
²/5²  :³
+ D,
#102,85q67# ,  3 ²
 ( µ%5 ¢µ5½:²

A m 


  O# \#

³5 ¶  ½²´²¶²   ®W¼ <:³¶%Á² ‡µr O:³¶¢Wµ5 ®

*
  ›%,j°( ,T3 
 ®¼ <:³¶¢¿V Á² ¸¸´½²µ¶ ÄW²µr ( µ%5  + Oª¦|±  –   +  g
ªG0ª[5 ² ·¯¼
+
²% ( µ¢5  Oª¦|  –  +  —
² 0ª 5Vª ·º ¼

2 O,h|±  –  ®y° ³¶½²µ5 ³d ‡³¶5¦02, 5¢½ÇÄ
WµO,Q6 ,O²O, ³  6v0, 5F( µ
 
5 n‚™| Ÿ ® " ¾»µlŠ:³i V¾ ½:W¶ ¿… Á²¶ ²   + \Œ ÁQ#02, ³  5q67#_02, 5a( µ¨5 n‚™| Ÿ +
  

Á ® ˆ ®  ® ‡l³¶50#02, 552:³¶%Á: ³¶5 ®


 

" ¾»¶µ5lWµ¶=+II¾»µlŠ:³¦· ¯ ¼+( ‡ ³¶5—02, ˜5 :³¶¸Á²µ5 ³5³5²˜•¶œ µ5½:Wµ  ® ‹  Á: ´ ²µ¶


 Á
´²µ5³¢
Á:²µ¶¸ 5
n¡ + WµrÁ: ¶5½Œ + ´½²µ¶ ÄW G0 ©5n6 ® " ¾ ! }6  ®


ŽS Á: ¶5½X#p ³5³¶µ5Œ Á  #02, 5©6W# ² ® ‹ =4+ Á:  Œ ‡³¶5›0#_0, 55
 %
:³5Á: ³¶5g +    Ág#102,5X6Ÿ# !  %YS + Á:\¢ ³5³¶µ5\¦# :³¶l²  ÇV!²;  ~Á: ³¶5 ³¶µ
  


±  –  ®
  
¸ Œ5²W_, |  – ®y€ pµ5 ´ Á:  \Á² :³³¶³Œ›#102,85\6 # !  %@<+ Á:

²¶µŒ\#O:³¶%Á: ³¶5
³¶µ±  – ®

Ž lWµ¶5½
\Z#  ³5³5µ5
  µ5³r<¾ ²   ¸:³¶%Á: ³¶5³¶µ-' 5 r²¶²µ ®
S
% ½:Á²µ5 ² ²=+  (:³5¨¿V Á² /r´½²µ¶ ÄW²µ%˜ :³¨¿À Á²¶ ³¢Á: ³¶5:³¢³5 ¢·² :³2³5 
¶ ³r µ5 · ²Š   ® "
Ž ¾ ÇŲµ5Á²Á:Oµ5½:Á:½:²   ¶µ5ZÁ:  ²  ¸:³¶~( ž³5³¶· OI¾ ¶ ³5²µ ‡ Á: ¶5½pŽÄ 
µ5½:³ µ5i:³a½:W¶ ³S¿À Á²¶ ²  :³ ® " ³S ‡rµ¶š+  :³¶Q³ ´²a¶µl:Š ³g¶ iFÁ: W ¶µ5
‡Fµ5 µ¶½²5½i³¶´ž5š+ ž(²µ¶ ²=ˆ+ ³5 ³ ± (  ± :Š ³5 ?Á: ¶5½‰…· Œa   5 e¼+ I¾ ½²5²µ5
ˆ V¾ ²³5²Œ· ¸:³iµ5½:² ³2:³¨µ5½:³5 5³F ·5²³¢³¶µ¨ :³Fµ¶ ² ³2²=+  Á:Á::³5³5 µ5² ²=+ %¿Àµ5d 

² :´:Á¸ /Wµžµ ± Œµ5½:Á:½:² ®

‹Œ€Ž“Q€ „‘}†ˆ} •

 >)  ¤ '   


 
8 >. 5( , 8 % # ('(
))>5 8  8
",>) 8 -, #&) 8 ? =? ? ,?  % <>5; 8
5 # 5'. '7  , 3) 8
3'7 -,
  >% 
' 8>8 #&)6 

% # ('(
))>5 8&? 5798 !6% < >' 8 <>.0>)6 : :% <>5: 8
 5 # 5'. '7  , 3) 813'7  , % # ('(
))>5 8 1
5 # 5'. '7  , 
3)*8
3'7 -,< >% 
' 8>8 #&)  , % # ('(
))>5 8&?

 I FL®
 €   5 ª Q ‡7Wµ¶¶
²¶Š²µ5 µ5½:² ª ® :  µ5³! ª" Vª # ª" 3 ² ®
{ V¾» ¸ ± ½: µlŠ² i:³I²µ¶ :³=+š :³¶(¿À Á² ¨I¾ ²Œ½:µ5    %%$ ' & 6    ¶ %%$ (& 6
p
²®
 K, .µ5½:² F4 ž³¶¶ ¿ ®<à µ5 7²¶²µN‚  „[Q + p( ž³5, 6 $   ( ) ) *& ²K,,+ 6 $  ()  *) & ³  ®
: ³¶¡+ , ²$, + ³ %µ5:³¶(:Á²¶´² ²˜ :³%´ž ²µ5³rµ5 Á ± ½::³/‰,hˆj²
„ ¶ ~µlŠ:³%Wµr½Ç¿Vr²  

Wµ%ÇÅÁ
:Š ³ ®

 

¼ <:³¶%Á² ‡µ˜<¾»  µ5³$, –,,+ | ¤ S + / ‡³¶502, 25 :³¶%Á²µ5 ³³5²r!02,,+ 25 :³5r½:Á²µ5 ³
³ 5 ® " / ³= + I¾»µl:Š ³dÁ² :³5³¶³=+  I´² ³ , 6 O, ² $ ³ , + 6), ®
   

, :³5d½²¶ ¿(+ OÁ: ³5¶µ¶˜ :³/³¶5:³! 02, % d5 ²0,+ d5 Á:   % Á² :³5³5³d³¸ ³5³ Á²½::³‰ˆ
 
 
 .
0´±$,5 ® <Ž :³d³¶5:³K0´±-, F5 ²X0´±-, + F5 Á: ´²²/  µ5³r4 µ-,
 
 
"
¯.-
ŽSO³¶¶µ5½²ž =+
: ³¶l5²W\Á² ‡µ5 ® à µ  ³¶¶µ5²µ  ¶µ5~µ5 ( ž³ g V¾ ÇŲµ5Á²Á: ³¶´  :³¶
Á: 5 µ¶W· ²¢ V¾ ½:W¶  ¿À Á²¶ ²  Œ:³5%Á: Œ³ ³¢ /  I¾    J [   M@ 1  [i &
wqx IJb " ½²5²µ¶²µi5 5:³? :³a¿À Á²¶ ³T#&%')( '^+WÁ: ¶:³c·‘µ5:³¶ ®    5 :³b¼+ ²?5²  :³
+( µ%5 ³-,–.g|j'
#102,34.5167#102,5839#102.5
A m 
€ O ´ µ5½:² ²
W+ ³¶k.# :³¶r7³5 ¶   µ5 · ²Š  l  µ5³=+ ( µr5 ˜µ¶ ²
 +  !#10 Y516 #10´²e5 ®
aG~ # :³¶i³¶4 ž³½:%Á: ¶ ®  n\, \µ5½:² ¡+ ²Q0, a5 r³¶5%rµ¶ ² ³iWÁ: ´²µ¶
  

´²µ5³_, ®y€ #  Áj#_0, 56 , #10´²e


5 ( µl5 _‚qg+ ² Á   ³ % #_02, 56¬,G#10´²e5 ®q€ µ=+


³j#n:³¶Á: ¶ ²9,k+a # $ ³ #02, 5!6#102,85 ® " ¾ !Œ#102,856 ,#10´²e5 ®¨°  
   

 ³³¶µ5¸\O # :³5¢ ½ˆµ5


³¶µ-' ®   %
?8# :³¶i³54 ž³5½:r   5  + WµFÇŲ dÁ²µ5 ³³5_‘· dÁˆ ³^# ½:Á²µ5 ³5³ 5/³5%¶µ5˜
¿V:†  W  že¼ ®  O,  µ5½:² ®  ²0, 2 5 Œ³55ŒÁ²µ5 ³5³5
µ¶ ² ³˜²_02, + 25 
³55\½:Á²µ5 ³5³ 5~ µ¶ ² ³=a + 2Á: ´²µ¶²5 5:³l :³7²Å ´²µ5³X, ® :  µ5³=Q+ 4 µ75 
 

²¶²µ$$ ‚ V„[+[, 9, 9, + ® ‹ =+ ³5K# :³¶%Á²µ5 ³5³ 5W+    Á
 


, #10´²e5q67#_0, 5 Œ#102,85 V# !, + %Q6), + #10´²e5
  

‹ .¿V³75²µ5#‚ ´²µ5³_3
7I+  ¨´²N,G#10´²e5  #102,5 ¸,G#10´²e5Q+ I¾ !}#102,85\6¥,#10´²e5 ® ‹ =+kˆ
 ´ˆD+O #  O²; ¶µ5¸ ½ˆµ5
³¶µ-' ®
% :
½ ²
Á 
 
 5
µ ² ²=¢ + ³\ :³ ²ÅnÁˆ ³=F+  ˜:³¶\¿V Á²   ´½²µ¶ ÄW²µ  5 5:³~ :³\¿À Á²¶ ³
½ˆµ5:³d³5 r·² :³%³5 ¶ ³d µ5 · ²Š   ® "
¹ s! I´
[bZ SŽ ¸:³¶¶  ¢ V¾ Çų¶5²Á:¢%³5 ¶ ³ 2 V¾ ½:W¶ ¿À Á²¶ ²  %µ5½:Á:½:²5
? ³5²µ²ŒW ³
Á: ¶:³=g + . .   5 :³eS+ :³¶
 )Á²  ®Qà µ7Á:²ÅZ¨²pµ²
²5² Wµ¶ ½Q + p(²
µ5 ´²µl<+ ³ ³
µ5½:³5²µ¶´\I¾»  ²¶¶µ5 V¾»ŽÅ   Á ±  Å8<+  ¨Çų¶5
 I:Á²¶´² ²˜I¾»¶µ5:³r³5 ¶ ³d¸Á:²  :³dÁ² :³5³¶³ ®
F² ²5²D + ‡7½²µ5Á ± ŒÁ² :³³¶³2<¾ :³¶2W ³¢ 5½:
Å\µ¶ ² ³e+ ³¢³ˆ¾» 5\ˆ


5 d³5 ³²³5²· 
²³5
/ V¾ ²³5²· Œ
½ÇĶ  :³2¿À Á²¶ ³%¶ ³5½::³ ®
 à µ5½:Á²³ ³¸²ÄO7Á:µ5½:³5 /:³5c\ ˆ Á: W ¶µ5lWµ¿À5² ²
Áˆµ=(+ Á:  7 O ´²µ¶µ +
/ ·µ5²³5:³˜½:W¶ ³%¿… Á²¶ ²  :³d³ˆ¾ \µ!²Š ² ®

wqx IJb  A M@  



z " ½²5²µ¶²µ¸5 5:³/ :³d¿À Á²¶ ³\#4%'(¯f' 5²  :³¸4+ 4 µ/5 
µ5½:² , ²¢5 %µ¶ ² ¦
 #102,5y± #1 0 @5  
^0,› ± @5 
A m 
 ‹ ´² ²=a+  ˆ g+ Áž¾ :³¶Œ  + I¾ \½:W¶  ¿À Á²¶ ²  ~<¾  2³ˆ¾»ž ®  ³·4 DQ+ 
Á ± 4 5 ³rW ³ ®®®
 # ~³5 ¶  ½²´²¶²   ®  ²!ªa–d « ²Å µ¶ ² ³=a+ ²_‚  „  ²¶²µ ®?à µ
v|}“e„ –=²– ®–‚y˜@4+ O( ž³5Nª T6ªK3  ¶  ®4¼ a:³¶˜Á² ‡µ¸75 ³d :³cª i³5 /µ¶ ² ³
²˜
ª ³  ±™ª  6  ¶  4 µ T|h“e„ –=²– –‚—±V²®˜ ®à µr³¶5+ I¾»µl:Š ³r V¾ ½² 5½
¶µ¶‡ž ‡µ5›
  

 2 


 #10ª[5 ± #10«
5   
¶   #_0ª o5 ± #0ª 5  
 ¶  0ª ±™ª 5  6
0«^±™ª 5 
³    ³ 
 

  ‚

¯ž¯
‹  ¿À³¸5²µ5
O _‚
X3
 +( Z².½:
!#10ª[5O6 #10«
5 ®IÃ µ
³55+( ‡\¿… Á²¶  # :³¶
´²µ5³
Á: ³55Œ³5µ ¤ ®
  ‰|g' 5² I\#10 @516 d4 µ¢5 ¢µ¶ ²  ®  -,h|j' ® " ¾»µlŠ:³¢ ‡µ5 µ¶½²5½  +
Çų¶5d/³¶50  5¨˜µ¶ ² ³%(Á: ´²µ¶/´²µ5³^, ²F5²  /+( µ25 ©‚~| Ÿ +(  
 ,¦±      ) ® :  µ5³\

 

 #102,5 ±  6  #102,5 ± #_0  5  


^0,›±  5  

²
„    

‹  ¿À³ r5²µ5K.‚ ´²µ5³v3! +  ² ½:˜X#102,5T6 W+ ²r ÁKO# :³5rÁ: ³¶5³¶µ

% ½:Á²µ5 ² ²=+  (:³5¨¿V Á² /r´½²µ¶ ÄW²µ%˜ :³¨¿À Á²¶ ³¢Á: ³¶5:³¢³5 ¢·² :³2³5 
¶ ³r µ5 · ²Š   ® "
Ž :³
 I½²µ5²5:³
 ¶ ³Œ7 V¾  \¶µ5 5:³-œ ³5<¾ \ˆ µ5½:³5²
³ ¸W ³¸5 ¶œ µ5³¸
<
Ž  µ5 µ¶½²5½l³¶´ 5(+ Œ ³r7½²  ¶µ5²µ5 ³¸W ³dÁ²

½²(²5:³/ :³d:³˜:³/¶µ5:³ ® S
( µ¢¸W ³¢¶µ5    µ5µ% V¾ ÇŐ( ž³5½ +  Œ² :³¢ ²³¢¶ :³X
‹Œ€Ž“Q€ „‘}†ˆ} ¦

 #/% N( '  
)1('(
)$  8
 3) ')6 >%
+ #&55( , ' ?
 #   >.0>) .0
) 
) 8
3% #&5(
%>8 #10 5  8
 3)4')6 >%
+ #&55(1 #  8


'7 3)
#  1 8
(% '1
   '  1('(
) , 8
3% #10 5?
'  8
 
)6(') 

  #   ('7+-18
(% '1  >.0>) .0
) 
)*8
3% #&5(
%>8 #  8
 
)6(')  8
3% ?
3)
 8
 ' 
1
  #   8
 
)(')  8
3% 1 ' 
1  ('7+ , 8
3% #10 F5 #&5(
%>8 #  81 8
(% '1 >.0>)6=.0
) 
)
'
'
8
3% ?

!#"%67'W=?35>5'9=g1 w  1
"¸u7=?'
Ž
<:³%µ5²µ5:³d º µ5:³¶5²r5 œ¶ µ5³r´ž ‡· :³=+³d  (²r²´³²µ˜I¾»¶µ5:³r  š ²³
( µr¶ ²µ%
½:W¶  ¿… Á²¶ ²   ®

     "!$#%'&(')+*, 


" ²ÅO (:³
Á ± ² ²5³7l´žµ¶‡· :³Œ³š¾ 4µ5² Xˆ !
 ³ I .(²  ²µ
³5µ¸ :³¸µ5½:² ³ ^œ
,k–.– 4I5²µ¶´²²˜³% V¾ +
½:W¶  ¿… Á²¶ ²   y  ³¶µ¢ :³2¿À Á²¶ ³r²  :³V ² :³ ® !;
- ˆ  ³r²Å ÇŲ :³ \
wqx IJb " ½²5²µ¶²µr :³2¿À Á²¶ ³Q#<%Y')(*' ´½²µ¶ Äyd :³%²Å~Á: ¶ ³˜³¶´ž5:³\
Y¼ à µr5 ³%µ5½:² ³-,k–.G+a#102,_34.[539#102,›±h.5q6 #102,85´#102.[5
·¼  #102,5167„
 ³ %
A m 
  vO # ³5 ¶ O½²´²¶²  l µ5 · ²Š   ® ‹  ( ž³Qª›6µ,¦± .Gy+  ½:˜Y¢¼ +
( µ¢5 Oª–~ . µ5½:² ³\
#10ªv3 ®.[5D3V#10ª 516 #10ª\34.5´#102.5
  Å ³$~ ª ²¢¿V³5 ³¢5²µ5\. ´²µ5³v3! + ‡Á: ¶  ·¢¼ Á: ˜  µ5³Qˆ#10ª[5q6µ„ ®à µ˜³¶5+
# :³¶¢ ‡l¿… Á²¶     ®
¯ˆº
% =+
½:Á²µ5 ² ²  <:³¶%Á² ‡µr/ ‡7¿À Á²¶    Œ:³¶%·² ³5 ¶   µ5 · ²  ® Š "
wqx IJb I=  ¡
bz p v|  „ –=²  ® " ½²5²µ¶²µ/ :³%¿À Á²¶ ³‰# %F'”( f' Á: ¶:³
²
„l5²  :³d+ ( µ%5 %µ5½:² 8,4
#102,5 ± #10 ,53}# !  , % 6), 
A m 
  Kp # l³5 ¶ Z½²´²¶²   Oµ5 · ²Š   ®k€ O( ž³5 h%a,™°( #102,85q±}#10 ,5 ®4¼ ?:³¶¸Á² ‡µ
 \:³¶¢Á: ¶Œ²j„[ + ²% 0„@5T6 „ ® " ˜ ³=+ ( µF5 -,&|j'$+ G02,85 ± 0,85q6), ®à µ
³55 + 4 µ%5 d²¶²µ$$‚ µ²
G0 ,5q±  !  ,5 % 6  , 
®®
®
 !  ¶
± c 0 ,85 6   ¶  , 
, %  

‹  ³5 % ²Œ·µ5Nˆl ²Œ·µ5Á::³%½² 5½:³=+ ²¢³5´_6ƒ ²+ I´²X


G02,85y± c0 ,85q6),  @²T  3  3 3   ¶  —6),  ²-²O± ± ´   


€ ¿VF  µ5³F5²µ5Q‚ ´²µ5³-3


 ®Ã ³5+  |  „ –=²  ²2  ³ % G0ª[5T6 G0„@5n6 „[+ ~²
½:d/( µ%5 %µ5½:² 8, + G02,5T6   +Á ® ˆ ®  ® #102,85 ± #10,8516   ®
‹  µ5 Á:½:dÁ:  ŒÁ² :³5³¶³=+   ¶
 ·¶²d+ 4 µ¢5 d²¶ ¶²
µO $‚ B²
#102,5 ± #N 0 ,516 ²$, ±   Y²n3  3 ®3   ¶  —6),  0²O²-±±  5 
  


‹ =+²~¿À³ %5²µ5‰Z‚ ´²µ5³v3


 +  <´²Q#102,85 ± #10„@516   ¶ 
   + ( µ%5 $,&|j' ®
°˜® ˆ ®  ® # :³5r/ ‡7¿À µ¶ c,g(° 
 3  !  :³¶¢¸Á: ³¶5
µ5½:²   ®
  ¶
¼ a<¾ :³¶¸W ³
 )Á² l´½²µ¶ ÄW²µŒ
  (+ µ5½:Á²µ5 ² ²=<+ 5 57¿À Á²¶ pÁ:²¶57¿À µ¶ :³¶
·² ¸³5 ¶   µ5 · ²Š   ® "

   &   !+  ( 


> }@?ag„›†š„‘“y…A  Q# %-C ( C FGKJ
 GP r bZ F o<MUbZ 5½²µ5½::³~M@¦#‘bZ X¨ Z U o&M@

= Z¦0#6 5   ]‰U]nH  FIg ¡ F ^,B| C ]-# 02,85N6 ,]-#  02,85N6 #102,85jJ nH  FIj ¡ -‚ ²®]
# ³  02,5q67#0#  02,55=P

° ²µ¶:³
½:W¶ ³˜¿À Á²¶ ²  :³/¿À 
Wµ ¶µ5 :³˜5½²µ5½::³ ® ¾ ½::³5/  µ5³/Wµ¿À ³ Ž =+
&, +
( µ˜ OÄŽ (Á: ³5½²µ5²µ¸ ‡³¶5 4 !  j02, 5J+ h, ¸ 6 # 02,5 Ž
® ¾ ²³²Œ· 75 ³˜ :³  :³¶ Q,
‰l` I
t o ^, T# Ž


(² ½ r \Wµ ® ¾ ½²¶d:³?µ5 µ¶½²5½:³¨rÁ:²¶5%³¶52(²µ¶ ² Wµ¿… ³¨I¾ ²½:µ5
:³%µ5 µ¶½²5½:³r ®K#
+ g0, 5+
% (²  ³d W( µ˜ V¾ ½²¶  W O³¶( ž³5
Wµ/ÇŲ l µ5½:³¶ /³¶´  y =+
 ³d  ²¶¶µ5 ³rÁ² q
‹Œ€Ž“Q€ „‘}†ˆ} ”

 0"! 5 
. $#  ª–d« %# 
. $# '&   
‚™| Ÿ – ³ 6 (ª ! ³  3}«)!

'7 3) 8
3'7  , !65(  81? 
'(>) ,1 !65(  8&? ) 8

!!18 !;3%  


) #&'7 !   
?

¯ˆÂ
#    <C O6 Uª C¸3~« %# % #>') 8   1  #&5(
%>8 '5 $# ' 8
 *,1%#
  ‚™| Ÿ  ! 6   3    ?
'5 < # ('(
) #,/.01 ,1 ,/' 8
(') 1  8

) 
. $#     
   C&O6 ª@C 3¦« , 5(
 #&5(
%>8 '56$# ' 8
 =,1%#
) 8
 #&)6  8
8
 #&)  8 !65(  8 1  >55( 8 !;3%
  

‚™|  !  6¸0  ‚›3  5  ?


' 5 < # ('(
) #,/.013)% # >')

. $#   1 
!65(  8  >55( 8    !;3%   Ÿ

¹ s! I´
[ ° ²¶5Zµ5 µ¶½²5½.³ ½²½²µ ³pÅ ³55:³ ™0"! 5 
/´½²µ¶ ÄW² Oµ5² ‡¶ 9
µ5½:Á²µ¶µ5²Á:/ ‡7¿À µ¶ 

! ³ 67ª  ! ³ ¶  34ª  ! ³ ¶  3 ®39ª !


 



!—ª  –lª  – =–lª ³5 %:³rÁ: ³¶5:³ ®


" ³aÁ:FÁˆ ³=+   ¶µ5¢²Wµ¶¶Á² ²µ?i³¶ V¾ ½:W¶ NC c67ª  C ¶  3ª  C ¶ @3 3_ª


  ²  ³5 ¶ ³S³5¶Á²5:³=:+  5½::³  – e– Ž–   µ5³< Çų¶5 :³<Á: ³55:³   – =–  5²  :³



 + ( µ%5 %²¶²µQ$‚ V„[+
 


! 6   





qw x IJb  IbH Z


›’     
z Œ ²nª–d«Q|—'T³ ‡ ®žÃ µ5 ´²µi<¾  ÇŐ³5522F¿À Á²¶ 
#<%Y'q³Œ(*'T³n5²  
+( µ¢5 %µ5½:² D, V„[+
#10#102,85583}ª #102,5167«0ª‰39«
5,
A m 
  \Z # l³5 ¶ Z½²´²¶²   ® Ž ¾ ½:W¶ O¿… Á²¶ ²  l( µ¶¸³¶µ/:³/5½²µ5½::³=4+  Q(²
;²¶µ5‰œ Á²²Å. Á: ³¶½²µ5²µŒ ‡ ³¶5/02, /5 ½ÇĝWµ\,  6 , Zµ5½:² F ½ ²=I+ ( µ
5 
²¶²µ$$ ‚ V„[+[, ³  67#02, 5 ® :  µ5³-, ³ 3}ªU, ³  ± «0ª‰3V«J5, 6 „ ®


° ¾ :³¶a2µ5² ‡¶ Fµ5½:Á²µ¶µ5²Á:¢ ½ˆµ5%I¾ µ5µ5 /²q/ˆ Á:  )Á²²5³ Á: ³¶5³ ® Ž ¾ ½:W¶ 


    

C  3Œª@C ±}«0ªX3 «J5Q6„   ²


²Å ³5 ¶ ³¸³¶¶Á²5:³¸?³5 \« ²N±X0ªX3Œ«
5 ®(¼ gÇų¶5
 ÁŒ:³%Á: ³¶5:³  –  5²  :³% + 4 µ¢5 r²¶²µO#‚ V„[+[, 6  « c3  0´±K²e5 0ª\39«
5 ®
  6 ƒ . „   µ5³ea+ ³5/pª ²›~« Wµ¶¶²²gˆ&'T³ ‡ ?+ *  «    0´±\²e5 80ª\39«
5  ²
  


 Á\( µK* ‚ ³5 )³ ²lžµ  i³5²µ74 ž³³¶· I¾»ˆ´ µN, ‘„[g+ ² Á: ¶µ Á²¶ #ˆ´:Á
  

V¾ ± (  ± :Š ³5
\O # :³¶O7ˆ ´ž ²µ5³¢( ž³¶¶´:³ ® " Á  6 „ ® 

 ³2  µ5³= + 4 µ©‚/6 7„ ²©‚/6²+  4´²iµ5:³54:Á²¶´² ²-,,O6),j6  ²n,  67#102,516  «®+


I¾ !&#102,5167«b, ®W° ² ‡ ³5³¶µ5/ V¾ Á²5½ ®
% ½:Á²µ5 ² ²=+  4:³¶?¿V Á² r%´½²µ¶ ÄW²µFr ‡/¿À Á²¶ ¦#&%,j(° «b~ , :³5 ·²˜³5 ¶ 
 µ5 · ²Š   ® "
wqx IJb " ½²5²µ¶²µ 5 5:³ :³¿À Á²¶ ³—#¥%  „ –=² (  „ –=²
 5²  :³~F+ ( µ\5 ~µ5½:²
,&|  „ –=² o+
#1
0 ®,¦±™#102,855q6),
A m 
  N# \³ ¶  ½²´²¶²   ®gà ³5g# :³¶7½ÇÄ~³¶µ „ –=² oa+   5<¾»  µ5³=S+ ( µ
5 v, |  „ –=² oI + O ®,—±4#102,85Q|  „ –=²  ®Ià ž³ ³/%,&°( ®,—±}#102,5 ® SŽ  ¿… Á²¶   :³¶/ Á
½ÇÄŒ²Q7 ˆ ´  ²µ5³r³¶µ! „ –=²  ® " / ³=+ ( µ%5 $,&|  „ –=² ¡y–  #10 G02,55q6 , ®
à µF5 i²¶²µ©# ‚ V„[+  5  ‡‰(‚ V²Š  r5½²µ5½:d  ®Ã µn,h|  „ –=² o+   G0 G02,55©6
*02,5n± #10 G02,855$6 *G02,85n±™, ®4à µ¸Œµ5½:Á²µ¶µ5²Á:³³˜ )Á² 5½² µ5 ´l  µ5³/Œ( µ


5 ³$,<|  „ –=² a  ²$‚~| Ÿ +  02,5q6 ‚ G02,5y± 02‚—±V²e5,j6 ‚T0 G02,5y±™,5D3 , ®




¯²Æ
X,  |  „ –=² 
4 ž³5 ³/<¾  QÇų¶5 a5² g c02,  5N6µƒ ,  ® c02,  51±™,  7„~  µ5³=+4I¾»µlŠ:³
=+
Á² :³³¶³ a  ² ½: $
³ ‰02,,e025q,,e±™5,,6   3
 „ ®4+gà Á: µ¸³¶¢5Á: +y4¶ µ5:µd5\ c ,™p| : ³¶„ –=² ·(o +4µ¶ Z½:  ®
  % 
€ !;
O ·¶²/ ‡  ² 7Á: ¶µ Á²¶ .³¶ 
02,5q6), + ˆ #120 ,8516),
 Á ® ®  ® ®
% ½:Á²µ5 ² ²=+  ´½²µ¶ ÄWd¿À Á² ² ²¢d V¾ ²¶5½¸:³¶i·²~d³5 ¶   µ5 · ²  ®Š
"
wqx IJbl’   )(´z à µ5 ´²µd<¾  <<¾ Çų¶5¸W ³%¸¿… Á²¶  h#&% Ÿ ‡ ( kŸ ‡ +
5²  Œ ( µ
5 d²¶²µ$$‚ V„[+
#10#102‚k5516)‚›3 ²
9

A m 
 à µ7 V¾»·³¶µ5/S³¶4 ž³ ³Œ# ³5 
 ¿… Á²¶ .´½²µ¶ Äy :³Á: ¶ ³l V¾ ½² Á:½ ®
€  ( ž³5!67#^# ®
€  Á:  ²Á:
Wµ%µ5²µ5²µrK#O:³¶¢œ¶:Á²¶´!²  I²=+³¶Dª–d«c| Ÿk‡ 5² ³d\#10ª[5q6
#10«
52  µ5³$ª\3 ²
X67#10#10ª[55167#10#10«
55167«13 ²
F+WI¾ !/ª!6 « ®


"   ³=I+ ( µŒ5 Œ²¶²µX‚ ¸„[+Q  —#102‚k5¦6 ƒ ‚#³³Œ ¡+Sp½²´²¶² F4 
ÄÅ 

9
9

#B…· ¡S+ ·(  I¾» Á:Á: µ58+Q ‡ ½ÇĶ  ~½˜4 ŒÄÅQ¾Z<¾»Wµl<¾» Á ± ¶µ5~³¶´  ®®® ¼
Á: µc7 ˆ V¾»·³¶µ55½\‚/6)‚›37²
®
à µ  | Ÿk‡  +   5 6$“  80  5b–‚4| Ÿ ~˜ V¾ µ¶·5Œ  Wµ ™‘· Á²Q Á+  ~½:³5ž
‡

9

‚(V²Š  d5½²µ5½:˜  ¼ ® :  µ5³=+ I ¾»µl:Š ³2 V¾ ½:W¶  ¿… Á²¶ ²  +   6”“  3}²
•‚1–a‚™| Ÿ ˜ ®
 

à µr³¶5+ Ÿ ‡ :³¶¢ ‡lµ5½²  ³¶œ 5Œ:³%²³5²Œ· :³  –   – =–  


®

9

‹  Wµ¶¶Á² ²µ=+g( µ  |7“@²– F– –=²


@˜@+a iÇų¶5\:³²¶²µ5³_‚B|µ“@²– F– ®–=²
@˜ ²


Œd„ 5² ³a-#10  516  N02‚k5 ® ‹ a V¾ ¶œ :Á²¶´5½2- #  2F³  6 /„ ²q#10  516)‚1 – ³5 

9 
9

6…r² ²n#10  5q6µ#10#102‚k5516)‚‰3 ²


® " ³i %³5:Á: \Áˆ ³=+ ~
  µ5³  6 #102‚k5 ®° ² ‡¸(²µ¶ ²
I¾»*) µ¶ ²µF% :³¨²¶²µ5³2Wµ¶5²W-ˆN“@²– F– p–=²
@Œ ˜ (²´²^²; ¶µ5¢µ5½²Wµ¶¶³F² Wµ5:³¨²Å

9

ˆ ²ŝ³¶œ 5:³F% ‡/¿À µ¶ ‰“=‚1–d#102‚k5d˜ ®[€ µ=+ Á::Á²4:³¶iÁ² ‡µ5² ²24 ž³5³5· %³5K²
Œ:³¶

9
Œ
9
Wµ ® "
¹ s! I´
[ DŽS!;²  µ³5 ² ²(²o;²¶µ5¶ ³5½³¶ V¾ .µ5² ‡ Á:<²
~Wµ
<¾ ( µ¶5
² W²¶²µ Œ„¸Wµ ®à µiÁ: ¶µ5+ ‡¸¿… Á²¶ g#<%Y,g°( ,O3 \  ¶µ5d+( µ 5 i²¶²µ

9

V„[+  SÇų¶5¸:³%³5 ¶ ³vˆ7 V¾ ½:W¶  ¿… Á²¶ ²  K#10#102‚k5516 ‚›3 ®


 
 

   (    
> }@?ag„›†š„‘“y#©…A  k#h%UC¤( '‘ UnJG
[]©J nZJ ªj|/C ^’^™M2 ^J vª bZJ T F.( %ÄÅ
MUQ#9¨IbZJ
[c#10ª[516 ª8P


Ž ¾ ½²¶:³r4 5³%ÄÅ:³r:³r¿À Á²¶ ³d³5 ¶ ³d:³¶rWµ¿À ³% Á ± ² ´²µ5³r ‡µ5½:³5 ¶ 
 µ5 · ²Š   ® - ˆ  ³d ½:‡5² ²˜ ÇŲ 

wqx IJb l’    z " ½²5²µ¶²µ/ :³r¿À Á²¶ ³‰#™%['T³ ‡ ( 'T³ ‡ ´½²µ¶ Äy/ :³/²Å Á:  
¶ ³r³¶´ 5:³K


Y¼ 4 µ¢5 ³rµ5½:² ³-,–.+#102,G#102.5516).[#102,5+


·¼  #102,5167„
 ³ %
A m 

¯ˆ­
 ‹ D+
 Á ± ²µ5Á ± 7.(² < Z¶µ5 ´\ ³5³´5 ‡ ¿À Á²¶ 
p ™,4(°
 :³¶¸³5 ¶ p
Š € Ž 
µ5 · ²  ®  ´ž\µ5 ´²µ
lÁž¾ :³¶¸ ‡ ³5²  ® Q ¿À µ¶ 7 V¾ ½:W¶ .¿… Á²¶ ²  ²
7 ‡

³ ¶  ²´³½:¸ ‡³5³/µ5½²´ µd/ V¾  ´ l³š¾ 5½²µ5:³5³5²µ l:³%ÇŐµ5:³³¶ ³%  ( ® c,G#102,85
 Q# 
³5 ¶  ½²´²¶²   ®
 °  ²†: ³¢Wµ2µ5 ´²µ¢v# :³¶F· œ:Á²¶´ ® ‹   I²=+( µn,—6²+ (´²-#10#102.5516
.[#10´²e5
4 µl5 _. ¥„[+a²  ³¦#10´²e5 „ ®q€  ² ½:~³¶©ª–d« „ ´½²µ¶ ÄW²


#10ª[5q6”#10«
5+W  µ5³Oª #10´²e5T6B#10#10ª 55n6µ#10#10«
55q6µ«J#10´²e5 ® ‹ d Á\ª6B«®+Á:
Q ³5³¶µ5
K#
:³5¢œ:Á²¶´ ®
" ¾»¶µ5lWµ¶=+(( µ¸5 ‰.$W„[+I p¦# •#    46¸.+4Á:l¨ ³5³¶µ5lXZ.   ²¸
5½:Á:½:˜WµQ# ²%<¾»³5k# :³¶%³¶µœ¶:Á²¶´ ®
 ‹ ¶ ³g5²Wg V¾ ²³²Œ·  *:³S4 5³<ÄÅ:³S©# ®Ž° ²¶5 ½:¨´²SW¶µ5²  ² ²
Á ±  ʼn,g6).¸³S V¾ ½:W¶ Œ¿… Á²¶ ²  +š³5<¾»  µ5³e:+ 4 µS5 D, Œ„[ˆ+ $#102,#102,5516


,#102,5 : ³¶¡+( µ%5 $,Œ„[+  N,G#102,85©| ®


 ³=+ (²aµ5²µ5²µ¨½² ² ²ic²\| ® ‹   I²= + ³5$
# :³¶a· ¶œ :Á²¶´+  5
 

% V¾ ˜5½:Á:½:%X%² Wµ^# ® :  µ5³e+ I¾»µl:Š ³2Á² :³5³¶³=+ ~K#10´²e5q6µ#10´² #10 =55q6


#10´²e5 ®
 

°  X#10´²e 5 V„[+    µ5³ O6W²+ ²r ÁK#10´²e5167#10 =5q6…² ®


¼ 4˜µ5:³¶5˜ ³¢<¾  ˆ µ5 ´²µ%X¸² :³¶2² ¿À2 /³5² (( FÄÅ/‰#y+ ³5<¾»  µ5³e+ ( µ
5 Q# , Œ„[W+  ²´µ\:´ µd½:Á::³5³µ5² ²v,G#102,85T6 ² ®Wà µ˜Á:² ‡ y+  ´žµ5²µ5²µdI¾   ³
µl:Š ³rÁ:²r²³5²Œ·   ®
 €  ´  Á:  ²Á:²µ7Wµµ5 ´²µ :³¶7³¶· \Wµµ5 7²W ³5³gˆ V¾ ´²µ5³5 ®
 ²$,–.j| ®a€  l ÁK#102,516 , ²O#102.516). ® O³r  µ5³v#102,a.5167#102,G#102.5516).[#102,516


.F,k+ Á:¸Q ³5³5µ5¸c,.¦| ® " ¾»¶µ5


Wµ¶=+ ³5!²\| !+  ³5³¶¡+ ( µ-,<|

  

²Q6 # , ² j, 67# , ² #102,85 6),# , ²










Á:¸S ³5³¶µ5Œ\# !  %v6   + ²r<¾»³¶   | ®



 
€ ~:³5i5²Wr²  :³¶µ5/I¾»¶5²µ5/ ¶µ5¸ · œ :Á²¶ ¿ ® ‹   (²=+ Á: ³¶½²µ5 ³2~½²´²
¶² a( ˜ÄÅXª46¥ ƒ ² ®  ª ¸l²   µ5³=4+ ³5 v:³¶/³¶· 7Wµ/µ5 =(+ O²Z½:/+


( µF5 F²¶²µ-‚ Œ„[ +  N#N0ª 5T6 ª ® O³2  µ5³   ³ % #_0ª 516B3
 + Á:d4Á: ¶µ5Ç


1
# 2
0 
, ©
5 6”„ ® ‹ =4+ ³¶ „  ª ²+ ³ :³¶˜³¶· 
Wµ˜W ³5³Nˆ V¾ ´²µ5³5+
  
˜ 
³


µ5½:² q«‰6

   %
:³¶¸³5³¶g ( ˜ÄÅ+I:´:Á«¸²+(Á:?:³¶d( ž³5³¶· 7I¾»µlŠ:³¸Á:l V¾ 


´²d/´  µ ®
" Á²¸:³¶¢ 
³5² I( ¢ÄÅ+²¢ ‡Á: Á² ³5  ² ½:Á:   ® "


  
 # (&" * (,  &"!   '&  (  *,'&(, 
€ lœ ˆ =+
 V¾».½ Z 5 5 ¿… µ¶¶  ³¶µ  ‡ ¿À Á²¶ *:³5 ·4  Zµ5²µ5 ¨²\Á:²µ¶:³ <ˆ +
!;
I¾ ²¶µ5\²  :³7(²´²7 ² ²µl:³½²µ5Á ± :³( ž³5³¶· :³´²µ5³ ‡ ³5 ¶  I¿À Á²¶ ³
<ˆ =+
½ÇÄ:³d²   7´ž ²µ5³r³¶µ¢ :³%²¶²µ5³ Á: ¶5½ y4  ,+  :³ ®®® +
" =+œ g Kœ
³7Á:²µ¶²µÁˆ ³ ³55² ² < ¶³7 · ²µ7<¾  4  ,+      F<¾»  ²
<¾   Œ·µ5
ÄS¸µ Á²:³ ®

qw x IJb   s )( bz " ½²5²µ¶²µ75 ³ :³


(  ,+  :³ ˆ ) Á²²5³µ5½:² ³5² ³Œ
Á:  
›0„@5q6 „ ²=+( µ%5 $,&|j'-+



!,  3 ² %v6…0 02,855  3 ²


 

¯ˆÈ
A m 
 à ³5  0„@5$6„[+ 0´²e5$6¥²+ 0 Y5$6 0 Y5$6 + Ž
4 Z½:¸ y³ 
 ²    (²5Á ® ¾ ½:l:³¶
c0ª 5
  µ5³?2Á: ³¶½²µ5²µ? ‡/³55 qª O6 „ =+
Q½ÇÄ2Wµ 1‚ Œ„[+ª ³ 6 ª  3<²
/² 4 µa5  ²¶²µ $   ®
 
h0ª 5
 µ5½:Á²µ¶µ5²Á:~³³l )Á² 5½\  ¶µ5 <¾»  µ5³Œ ‡ ³¶5
:
 5
³ 
 ¶
³ ¶
 ¶
µ 
 ²
Á 5
 ²
  
 ²
 
 l


²
Á 5
µ
³³5~²
/( µ¢5 r²¶²µ $ Q‚ V„[+ 0ª 5q67ª ®


+ 02,5v6 ›02,5T± , +



 
° ² ‡~²¶µ l l4  ,  +   (½ÇÄ Wµ <  ²¸lÄ5½ 
=+ ˆ 02,85q6), + $,&|j'

µ Á²:³d³¶¶Á²5:³ ²%:³5% Á¸ ¸(  ,  +  / ®4°˜® ®  ®  4 µ¢5  ®
Ž
Slµ5½:Á²µ5 
:³5¢ ½:‡5 ® "

  '&  & * #''&(,')+* 


 ˆÁ:  I¾ ½:W¶ ³ ¿… Á²¶ ²  :³ µ5² ² :³ ´  ²µ5³ :³~¿À Á²¶ ³ Á: :³ r²
F e+
¿V³ 5²µ¶´²µ ³¶Œ ½² ²~:³  Œ·µ5:³   4 /#102,85b–d#102.[5b– n€ œlˆ
 ® *O½ Z³¶žW ½
l;
<¾  W½²2³5 ´²FÇŐ¶µ ² ² ²2¶ r%Á ± ²µ5Á ± ²µ¢:³i³5  ½²¶µ¶:³i²\Á::³¨ I½²µ5²5:³F´žµ¶‡· :³ =+
¶5 Wµ¿À ³ Xˆ ›· +
:³Á²µ5½:²µ7²pÁ ± ³¶³³l:³
´ž ²µ5³7 ½:W5:³ ‘Wµ7ÇŲ  Qµ5² ‡ Á:²µ N.
Wµ O#102.5¼ ®
o; $œ
¼ ?(² ²¶µ5 ½² ² ²Wµ¿… ³ Á²²ÅZI¾ :³5³ˆ ²µ³5½²Wµ5²µŒ :³¸5²µ¶ :³
( µ¶Œ³5µ ‰,
¸Á:²Å I4 µ¶5²r³¶µ ® $.
- ˆ  ³%5 d
³¶5/ ÇŲ  !
wqx IJb  IbH Z
›’    z O ² –  |—' ® µ5 ´²µr5 5:³d :³2¿À Á²¶ ³Q#h%Y' ³ (*'
5²  :³r + ( µ¢5 ³%µ5½:² ³$,–.V„[+
#102,5´#102.516).  # , 3},
F# . 
A m 
  O# 
³5 ¶  ½²´²¶²   ®
 à µQ,g6).!6 „[ +  I´²Q#10„@5T6 „ ®
1S Ž d ²·µ5
˜Á ± ¸˜ V¾ ½:W¶ ~¿… Á²¶ ²  
½²%³¶  ½²¶µ¶/²g, ²^.G+ Á:² <
µ5 5Œ % V¾ ²; ¶µ5
½² ² ² ®yà µ˜³¶5 + ( µ%5 ³¢µ5½:² ³$,k–. Œ„[+  j
.  # , 34,
# . /6),  # . !34.
# , 
=,  ±™,
\# . /6
.  ±h.
\# , 
 ²Á: µ5

F 6 ƒ    µ5³= + ( µ$,–.g|j' ³ ‡ “@²®˜@y+  g


# !   % 6 # !  %
,  ±~,
.  ±~.

Á:˜4  ¶µ5¸d ‡
¿… Á²¶ j,j°(  0  5 :³5FÁ: ³¶5¸³¶µ^'q³ ‡ “@²®˜ ¼ (Çų¶5d Á˜~µ5½:²
˜5² S+ ( µ%5 %µ5½:² 8,h|g'T³ ‡ “   ˜@¶ + 
/

#102,8516  ,  ±
,

" ³rÁ::³%Á: ¶ ³=+ V¾ ½:W¶  ¿À Á²¶ ²  + ( µ$,–.g|j' ³ ‡ “  –=²®˜@y+ Á: Qˆg
 !  ,  ±
,
   .  ±
.
—6 .  J,  ±~,
3 ,
J.  ±~.
—6 =.  ,  ±~,
.


¯šÉ
 k. j½ 8¾ N,
 Å ³ Œ²gÁ: ³5½²µ5 ³?Á:²¶5i½² 5½2Á:  F¨µ5² ‡¶ ²¶µ5F²Å (  ,+  :³ Œ² ®
Ž
QÁ: Wµ³  :³FÁ:  )Á²²5³r O,  -ˆ    .  ±
.
6 b. 
Á: r  µ5³  ! % 4 µ25 
.g|j'q³ ‡ “  –=²®˜ Qˆ _½
 ®° ³¶½²µ Œ ´ˆ Á:²¶5˜½² 5½/Á:  /˜½² 5½¸ %(  ,+  :³ q¾G+
¸Á:  )Á²²˜ ‰.
  O6 „ ®
—#102,56 „
à µ³55 O4 µ5  …, |B'T³ “  –=²®˜ =+ !,µ6 .V6 ²
 ® O³ ?² Á ± ³¶³³ ~³
,™6./6 + 0#10´²e55 N6 # 
\³/ V¾ ½:W¶ O¿À Á²¶ ²  l¶‡  ( g´²¸µ5:³54:Á²¶´² ² !%
²g0#10 Y55  6  3
#10´²e5X6 „ " }#10´²e5X6Ÿ#  j6Ÿ„[+ #

! % ® ¾  ! !% S² +
:³¶ ‡ ¿À Á²¶     Q

µ5½:Á²µ5 ² ²d:³5¢·² ³5 ¶   µ5 · ²  ®

¢ 6    µ5³=y+ ( µO,<6B.GW+ V¾ ½:W¶  ¿À Á²¶ ²  7Á: ‰ˆ# !  % 6  ,¶  0#102,55  ®
: ³¶¡+ ( µ¢5 ³%µ5½:² ³-,k–. V„[+
#102,85´#102.[516 ² .  , ¶  0#102,855  3 ² ,  . ¶  0#102.[55 

ˆ
Á ® ®  ®  
6   +²d Á+yÁ:  Á² :³³¶³=+ QÇŐ³¶5ŒŒÁ: ³¶5
5²  Œ+4 µd5 ³
, V „[+#102,8 5q 6 ,  ® Ž ¾ ½:W¶  ¿À Á²¶ ²  Œ³ˆ¾ ½:Á²µ¶˜  µ5³\
µ5½:²  

, .  6  , .  
( µ~5 ³ µ5½:² ³g,–. *„ ® S Ž  !²;  Z½²µ5Á ± . Á² :³5³¶³~  ¶µ5Z<¾»  µ5³ 46 „ 
6 ¶  ®
$
% ½:Á²µ5 ² ²=+ ‡Œ¿… Á²¶    ¸²¢ ‡
¿À Á²¶ j,g(° !  %  ³5 2·² :³¢³5 ¶ ³%
µ5 · Š²  ® "

¯
@
 =a&4;g35;g=?'

: ¶5²¶ \aŽS:³rÇŲµ5Á²Á::³%¸³5 %W ³%Á² ‡ ³5³5½:³%Wµr µ5µ5


¸ )Á² 5½ŒÁ²µ5 ³³5 ®®®

ËrÌ<€Ž’„‘’p{ ªŒ€ ™<Ì  ‚†š«S.‚†š„‘’“y€™g  W• 

Š
" ½²5²µ¶²µd5 5:³¢ :³¢¿À Á²¶ ³ O#&%')(*' 5²  :³r+4 µr5 ¢µ5½:² D,
# !¡, q34, % 9, B0#102,55 n3V#102,5

ËrÌ<€Ž’„‘’OŠ 

" -#<% £ ( £ ·4 µ¶½::³2²F5²  :³i+4 µi5 ³F²¶²µ5³©‚1– 


½²5²µ¶²µ25 5:³F :³¨¿… Á²¶ ³

#102‚3  5D39#10  ~± ‚k516 #10  5´#102‚k5


ËrÌ<€Ž’„‘’ • –˜—<™S‚†š„‘“4#¬g IQ‰ˆ 

" ½²5²µ¶²µ¢5 5:³F :³i¿À Á²¶ ³ $#/%@')( ' Á: ¶:³r²F5²  :³2+( µ25 ³Fµ5½:² ³n,k–./
# ,_34 . 6 1# 02,5839 #102.5



ËrÌ<€Ž’„‘’ ¦ >  ‚g€ ‰ Q€“  “y‰š„›†š„‘“y  {©(W© 

" Xª 9# %  „=² Q( '


½²5²µ¶²µl :³µ5½:² ³ .4 µ :³5² ³ 2ÇŐ³¶5 ¿À Á²¶  Á: ¶ ²
K
´½²µ¶ Äyr :³%Á: ¶ ³d³¶´ž5:³
¼j#10„@516 „ O#10´²e5q6…²
  ² ®

 4 µ¢5 ³ O,–.g|  „=²  ‰, V.G+ # ³  6¸0´²$±™ª[5´#102,5D3}ª #102.5 ®
aˆ´:Á     ! %
ËrÌ<€Ž’„‘’O” {©W©  

° ³¶¶µ¶µ5¸/¿À Á²¶ <#&% ¤ ³ ‡ ( ¤ ³ ‡ 5²  Œ+( µ%5 ³-,k–.¦| ¤ ³ ‡

#102,#102.[5516 #102. ,5

ËrÌ<€Ž’„‘’ Ê  > 
‚g€ ‰ Q€“ “y‰š„›†š„‘“y {©W©I{ 

" Q#<% £ ( £ 5²  :³d+( µ%5 ³%²¶²µ5³ ~–‚V



½²5²µ¶²µd :³2¿À Á²¶ ³

#10 3}#10#102‚k555q6±O#10#10 3)²e55q±~‚


! !

!
ËrÌ<€Ž’„‘’.¥ ²€‚ {©y©W© 

" ½²5²µ¶²µd :³2¿À Á²¶ ³ Q#<%Y')( ' 5²  :³d+( µ%5 ³¢µ5½:² ³O,–.+
#10#102,5834.5167# ! ,  ±~. % 3  . #102,85

¯ *
ËrÌ<€Ž’„‘’ 

  Q#<%Y')( '9Á: ¶²%5²  


+4 µ¢5 ³rµ5½:² ³-,–.~
#102,34.[5´#102,¦±~.516…0#102,55 
à µ5 ´²µd\# :³¶¢²¶² ²/  Œ  \# 
³š¾» ¸W ³ ®


ËrÌ<€Ž’„‘’ © ‹
€“ ¨“W‰ˆ} 
{©W©yÊ 

  Q#<%Y')( ' + D, +    1# 02,5  B²


²
5 ²  Œ ( µ¢5 %µ5½:²

# , 3 ² 39#102,5167# ,3 ² 93 # ,3


²









à µ5 ´²µd \# :³¶¢4½²µ¶  ®

ËrÌ<€Ž’„‘’p{ !
ǀ ƒ‘‚S¬gZ{©W©W© 

Q#<% ‡ ( ‡
  Ÿ Ÿ 5²  ¸
¼ vª O« —#10ªF«
5q6µ#10ª 5´#10«
5
 4 µ¢5 ³r²¶²µ5³ \² ˜µ5²²µ5³d²¶µ5
²Å
¼ – +a#10 N3 Y5167#10 85839#10 Y5
 4 µ¢5 ³r Œ·µ5:³rµ5²²µ5³ 
\#10 Y5q6 +#10 Y5q6 O1# 0´² Y5T6² ®
   
à µ5 ´²µd
  ²
99
99


ËrÌ<€Ž’„‘’p{W{ ªŒ“W€}.{©W©y© 

" ½²5²µ¶²µd5 5:³¢ :³¢¿À Á²¶ ³ O#&%')(*' 5²  :³r+4 µr5 ¢µ5½:² D, |h“@±K²–=²®˜¦
# ,›,_3± ² 39# O3²$}™± ,, 6 ,







ËrÌ<€Ž’„‘’p{Š   {©(4Š

Q#<% k‡ ( ‡
  Ÿ K
Ÿ ´½²µ¶ Äyr :³%Á: ¶ ³d³¶´ 5:³
¼ ~–‚ V„[+#10 34‚k5 ± #10 <5 ± #102‚k5©|~“e„ –=²®˜
 4 µ¢5 ³r²¶²µ5³ $
¼j#10 Y516 „[+a#10 Y5 Œ„ Q#10 Y5T6
! !

  ² 999
" c#10´² Y5
½²5²µ¶²µ
9 ®


ËrÌ<€Ž’„‘’p{•  “4™S€ S“4„¨¬g‰S„‘ƒ…ƒ‘‰~{©W©yÊ 

à µ5 ´²µ<¾  i<¾ ÇŐ³¶5 Á² ¿À Á²¶  9#7%' ( ' +


5²   Q4 µ5 µ5½:² n,k+S  
#10#102,85516),-±9²
99 ®


ËrÌ<€Ž’„‘’p{¦  ™Q€Ž—<™g„‘Z{©y©W© 

" O#&%')(*' 5²  :³r


½²5²µ¶²µd5 5:³¢ :³¢¿À Á²¶ ³

 V¾ ²³5²Œ·     


–,&|j' ‡  ³5 ¢Ä

 4 µ¢5 rµ5½:² D,k+#102,¦±V²-± #102,55T67#102,5 ±~,¦±V²
ËrÌ<€Ž’„‘’p{”  > 
‚g€ ‰ g€Ž“ “W‰š„›†š„‘“y   {©W©I¥ 

‹ Ő³55ÇVV S²Å~¿… Á²¶ ³ O#&%' ( ' ¦%')( ' 5²  :³%+( µ¢5 %µ5½:² D,4
9²

#10G02,855T6 ,  ² G0#102,55n6),

ËrÌ<€Ž’„‘’p{Ê  > 
‚g€ ‰ g€Ž“ “W‰š„›†š„‘“y   {©W©4”

‹ Ő³55ÇVV </¿À Á²¶  Á: ¶  X#&%Y' ( ' ´½²µ¶ Äy˜ :³¢¶µ5 ³%Á: ¶ ³r³5´ 5:³ 

º -
¼  <ÇŐ³55/ µ5½:²  V „l5² S+4 µ%5 rµ5½:² D,k+8± Œ#102,85

¼j#10´²e516²
¼ 8,467ƒ „   µ5³=+
# [,3 , ²  6µ#102,583 # , ² 



 

ËrÌ<€Ž’„‘’p{¥    

Š 4Š
" O#&%')(*' 5²  :³r+4 µr5 ³¢µ5½:² ³-,–.G–[–$\
½²5²µ¶²µd5 5:³¢ :³¢¿À Á²¶ ³

0#102,5839#10F55J0#102.[5D3V#1055167#102,.N±539#102,(834.F5

ËrÌ<€Ž’„‘’p{    {©W©4Š

" O#&%')(*' 5²  :³r+4 µr5 ³¢µ5½:² ³-,–.~


½²5²µ¶²µd5 5:³¢ :³¢¿À Á²¶ ³

# !,  39#102.5 %Q6 .‰370#102,855 

ËrÌ<€Ž’„‘’p{©   {©W©y© 

" O#&%')(*' 5²  :³r+4 µr5 ³¢µ5½:² ³-,–.~


½²5²µ¶²µd5 5:³¢ :³¢¿À Á²¶ ³

1# 02,¦± #102.[5516µ#10#102.[55834,#102.[583V#102,5 ±V²

ËrÌ<€Ž’„‘’OŠ   {©(¥y¥ 

" ½²5²µ¶²µd5 5:³¢ :³¢¿À Á²¶ ³ O#&% Ÿ ( Ÿ 5²  :³r+4 µr5 %²¶²µO‚$Œ„j
#102‚›3)²e5 V#10#102‚k55

ËrÌ<€Ž’„‘’OŠ({   {©W©yÊ 

" O#&% Ÿ ( Ÿ 5²  :³r+4 µr5 ³%²¶²µ5³ ~–‚$V„—


½²5²µ¶²µd5 5:³¢ :³¢¿À Á²¶ ³

1# 0 3V#102‚k55167#10#10 <55839#102‚k5
! !

ËrÌ<€Ž’„‘’OŠWŠ ! †ˆ‚ƒ…„‘ {©W©y© 

‚ "²
½ 5
 ²
 ¶
µ 
 
 
 ²
 ¸
µ 5
 5:³/ :³˜¿À Á²¶ ³¸³¶¶µ¶Á²5² ²¸   5 :³ X#4%'W(E'f5²  :³/+y( µ
5 ³%µ5½:² ³ $,–.h
#102,39#102.55167#102,5834.
  à µ5 ´²µ?+š4 µg5 ?²¶²µy#‚ µ²+  <¾ Çų¶5iW ³g¨¿À Á²¶ 7³¶¶µ¶Á²5² ²?   5 
#<%Y' ( ' 5²  :³%+( µ¢5 ³¢µ5½:² ³$,k–.&
#102,N39#102.55q67#102,85834. 

º¯

ËrÌ<€Ž’„‘’OŠ• ÍI™g„‘‰ˆ‰ˆZ{©W©W© 

" ½²5²µ¶²µd5 5:³¢ :³¢¿À Á²¶ ³ O#&%' ‡ (*' 5²  :³%+4 µ¢5 ³%µ5½:² ³$,    T
² #10´±-,5839# ² )6 ,
, ,


ËrÌ<€Ž’„‘’OŠ¦  ~„‘†šQ‚ {©W©y© 

Q#<%, „ –=² 8( ' Á: ¶Œ5²  


!

 
¼j#10„@5167#10´²e5q6 „
¼ 4 µ¢5 ³O,–.g|  „ –=² ¡–y   #102,5D39#102.516 #   ³  ®
à µ5 ´²µd\#102,516 „ 4 µ¢5 $,&|  „ –=²  ®


ËrÌ<€Ž’„‘’OŠW” \™S†ˆ€„‘’ «S‹d“4ƒ‘“yQZ{©W©(¥ 

<#&% ( £ 5²  Œ+( µ%5 ³%²¶²µ5³O,k–.&


à µ5 ´²µd<¾  I<¾ ÇŐ³55¸W ³%˜¿À Á²¶  £

#102,39#102.55167#102,5 ±~.


ËrÌ<€Ž’„‘’OŠÊ 
I€Ž‚„…QZ{©y©(¥ 

" O#&% ¤ ³ ‡ ( ¤ ³ ‡ 5²  :³r+4 µ¢5 %µ¶ ² k,V„/


½²5²µ¶²µd5 5:³¢ :³¢¿À Á²¶ ³

#102,N3 ²e5q67#102,8583 ² ² # !,  %Q6¸0#102,55 

ËrÌ<€Ž’„‘’OŠ4¥

" ©#&%')(*' 5²  :³i   


½²5²µ¶²µi5 5:³¨ :³?¿… Á²¶ ³  
6²%²=+4 µ 5 ³¨µ5½:² ³
,k–.& 
#102,_34.5167#102,85839#102.[583 ®,a.
!
ËrÌ<€Ž’„‘’OŠ  ǀ ƒ‘‚S¬gZ{©W©(¥ 

" ½²5²µ¶²µd5 ³¢ :³¢(  ,+  :³  + D,


5² ³% 4 µr5 %µ5½:²

02,¦±V² Y5 0
®,85n6² [02,g±V²e5 02,85

 
 


ËrÌ<€Ž’„‘’OŠ© ‹
€Ž“ “W‰š}   

{ © (¥ 

c# %U'T³ ‡ (
  +
Œ¿À Á²¶  5 ²  7 y( µr5 ˜µ5½:² k,$ „[+#102,5^6W# !% ®yà µ5 ´²µ
/%,¨² b 3! @( ' 

<¾  <Çų¶5//¿À Á²¶  5²  Œ 
 ,_3 
67#102,5

 

( µ¢5 %µ5½:² D, V „ ®

ºžº
ËrÌ<€Ž’„‘’ •   ÍgÍ#{©I¥¦ 

v#~%  „ –=² k(' K#10´²e5©6 ² c#102,5  „4 µ%5 ˜µ5½:² , ® " ¸ ³=+W4 µd5 ³

  95²  7 Œ²
$, – ,  Œ„
µ5½:² ³  \, 43 ,  B²+ #02, 34,  5Œ#_02,  5839#02,  5 ®
 l5² ³%      
+ D,k+ #102,5 ®, ®
‚  à µ5 ´²µd ( µ¢5 %µ5½:²    
‹ + D, + O#102,5B²– ®, ²¢( µ5  
  ³5V SÇÅ Á²d ( µ¢5 %µ5½:²    


ËrÌ<€Ž’„‘’ •I{ ‹
€Ž“ “W‰š}   {© W© 

 ²  g%<(
Z/¿À Á²¶  D+aªg|Z²Œ| “@²®˜75² S  6¸² ®à µ5 ´²µd<¾  SÇų¶5
Œ²¢
³5² ˜¿À Á²¶  <#&%
<(  5²  Œ
#10F5D39#10  Q3}ª[516 G0F5
( µ¢5  ›| Z²%½²5²µ¶²µ˜Á:²¶5/¿À Á²¶ <# ®
ËrÌ<€Ž’„‘’ y 
• Š \p€ „›’‚ 
‚†š«S.‚†š„‘’‚ƒ “y4†š«gƒ   

Q‚ …²
  " \# %['”(¯'f5²  :³˜+y4 µ˜5 ³
 7 O²¶²µ ® ½²5²µ¶²µ¸5 5:³d :³d¿À Á²¶ ³
µ5½:² ³$,–.~
#02,34. 5167#102,85D3 0#102.[55
 


ËrÌ<€Ž’„‘’ •W• \“4™Qp‚Q„‘Z{©W©W© 

" c#<%Y')( ' 5²  :³r+( µ¢5 %µ5½:² D,4


½²5²µ¶²µd5 5:³¢ :³¢¿À Á²¶ ³¢   5 :³

#10#10#102,555 ± #10#102,855D3 #102,8516  , 3


 


ËrÌ<€Ž’„‘’ •¦ ‹
€Ž“ “W‰š}   {©y©y”

h#&% Ÿk‡ ( Ÿ ‡
 ¶µ5²µd<¾  SÇŐ³55¸
²%
³² /¿… Á²¶  +
5²  
 ( µ%5 ³r²¶²µ5³
~–‚$Œ„j
#10 39#102‚k5516 ‚›39#10 3 Y5
! ! 9

d²  
:³¶¢ ‡l´ž ²µ%    #10  5 
ËrÌ<€Ž’„‘’ •y” ªŒ€™SÌ   ‚†š«Qp‚†š„›’“W€ ™Q 


µ5 ´²µ/5 5:³r :³r¿À Á²¶ ³ v#™%U'B(' œ
³¶¶µ¶Á²5² ²¸Á²µ5 ³5³5:³/²d· :Á²¶´:³d5²  :³d +
( µ¢5 %µ5½:² D,4
#102,85839# ¶  02,5q6 ®,

ËrÌ<€Ž’„‘’ •WÊ ‹
€Ž“ “W‰š}   Š   


 K# %' ( 'B² ™%' (u'
µ5 ´²µŒ5 5:³Œ :³¸Wµ5:³Œ¿… Á²¶ ³ +
5²  :³Œ I( µ¸5 ³
µ5½:² ³$,–.~
#102,_3G02.55T6 ,G#102.5 ±~. #102,8583G02,85
ËrÌ<€Ž’„‘’ ( • ¥   {©W©  

€ [Š
~Á: ³5 ²5µ /5 5:³¢ :³% Áˆ¶ ³ O#&% Ÿ ‡ ( Ÿk‡ 5²  :³¢+4 µ¢5 ³¢²¶²µ5³Y–$V„g
# !  #105 %Q6U0#10l55 
ºžÂ
" ½²5²µ¶²µd ‡7 ³%(²¶5/´  ²µr4 ž³5³5· ¸ \#10´² Y5 ®
99

ËrÌ<€Ž’„‘’ •(

" ‰#<%Y' *
½²5²µ¶²µd5 5:³¢ :³¢¿À Á²¶ ³%Á: ¶:³ ( ' 5²  :³d+( µ%5 ³¢µ5½:² ³O,–.h
0#102,53}#102.[55´# ,_34 . 6 #102,85´#102.[5



!
• © ljš€Ž‚ƒr{©W©4”
ËrÌ<€Ž’„‘’ W

  |—' "


® ½²5²µ¶²µr :³2¿À Á²¶ ³ Q#<%Y'T³ ‡ ( 'T³ ‡ 5²  :³d+( µ%5 ³¢µ5½:² ³O,V„j
,  # , ² 39#102,8516 ,_3, ²




ËrÌ<€Ž’„‘’ ¦ 
\™S†ˆ€„‘’ «S‹˜“yƒ‘“4Sp{©y©¦ 

 ² Qª–d« " Oˆ


˜²Å µ5½:² ³ ® ½²5²µ¶²µr :³2¿À Á²¶ ³ l²Å ´ µ¶‡· :³ Q#&%'  ( ' 5²  :³% +
( µ¢5 ³%µ5½:² ³ $,–.G– /

#102,k–.[5T6 ªF#102,k–U5D3}«J#102.G–U5

ËrÌ<€Ž’„‘’ ¦<{  ™Q€Ž—<™g„‘Z{©y©WÊ 

à µ5 ´²µd<¾  I<¾ ÇŐ³55¸W ³%˜¿À Á²¶  <#&%')( ' 5²  Œ+( µ¢5 ³¢µ5½:² ³O,–.V„—
#102,_34.5Œ#102,5J0´²©34. #102,855
ËrÌ<€Ž’„‘’ ¦4Š   {©(¥”

" ©ˆ
½²5²µ¶²µF5 5:³i :³ ¿À Á²¶ ³¨(  ,+ ‡ :³ Œ²Å ´ µ¶‡· :³  ´½²µ¶ ÄW²F :³iÁ:  
K
¶ ³r³¶´ 5:³
¼ $‚ Œ„ +
  <ÇŐ³55/ ²¶²µ $ 5² < 4 µ¢5 ³rµ5½:² ³ -,–.G– + 0 ,– [. 5q6 ›02,–.5
$ W     $
¼ Qªa–d«•– =+ 0«q3 ®–lª 53 0 T3}ª–d«
5D3 0ª‰39«•– =5T6 „
 
 4 µ¢5 ³ /µ5½:² ³ W    
¼ 0´²–l„@5T6…²

 

ËrÌ<€Ž’„‘’ ¦y•   {©(yÊ 

" O#&%' ³ ( ' ³ I´½²µ¶ ÄW²˜ :³%Á: ¶ ³d³¶´ 5:³K


½²5²µ¶²µd5 5:³¢ :³¢¿À Á²¶ ³
¼j#10
Y516 „

¼ 4 µ¢5 O,<|  „ – ¨+G#102,5v6 ƒ „
¼ 4 µ¢5 ³rµ5½:² ³-,–. V„[+#102,#102.[55´#102.5167#102,34.[5

ËrÌ<€Ž’„‘’ ¦W¦   {©W©y• 

‹ <#<% ‡ ( Ÿ ‡ I´½²µ¶ ÄW¸ :³%Á: ¶ ³˜³¶´ž5:³ 


Ő³55ÇVV </¿À Á²¶  Ÿ
¼j#10´²e516
¼ 4 µ¢5 d²¶²µO‚$Œ„[+#10#102‚k55167#102‚k5834‚
¼j#102‚k
5 Œ#102‚3 ²e5
ËrÌ<€Ž’„‘’ ¦4”   {©W©W¦ 

" T#<% 
±¦²–b3!  ( 
±¦² b3! 
½²5²µ¶²µ? :³g¿À Á²¶ ³ ´½²µ¶ ÄW²¨ :³?Á: ¶ ³ ³¶´ž5:³-

 4 µ¢5 ³%µ5½:² ³ $,–. µ±\²+#102,_3}#102.5834,G#102.55q6).\39#102,583}. #102,85


 ‡¿… Á²¶  /,g°(  ±V² l„  -' ³ ‡

:³¶%³¶¶µ¶Á²5² ²˜Á²µ5 ³5³5
³¶µ ²r³¶µ

ºÆ

"r>5$
1435"%67'

ËrÌ<€Ž’„‘’p{

µ%©,W°( , 3µ, Ž
 " ® SO¿À Á²¶   :³¶Á: ¶O²³¶¶µ¶Á²5² ²~Á²µ5 ³5³5 ³¶µ ² '
02'n5v6B' + œ \' O' 1€ ¶
®gà µ/³¶5  :³¶%Œ· :Á²¶ O œ
³¶µ ®   5   ³· :Á²¶  µ5½:Á²µ5  ®
: =+ ¶ -'
 µ5³   :³5%½² ² ²/³¶¶µ¶Á²5² ²˜Á²µ5 ³5³5Œ³¶µ ®
T# : =+ O
  r³5 ¶  ½²´²¶²   ®  µ5³ V¾ ½² Á:½r²´² 4 µ?5 ¨µ5½:²    [,– ‰#10 G02,855

, 0#102,55 ‹
  ¶ ˆ Š
®  Á: 4 ž³ Wµ   Oµ5 5 ³5µ ‡ µ5² ²µ5 ½² 5½  ¢´² +  #102,85
 ¶ 02,5 ‹ ¶ ˆ +
 ®  Á: ( ž³rWµ   7Á ± Œ³¶µ¢ ‡ ³5:Á: ¸½² 5½  <´²    ® ¶ 02,5 #102,5
‹ K#102,8516 ¶ 02,5
r Á   ®
=+ ¶
% ½:Á²µ5 ² ²  <:³¶2¿V Á² 
/´½²µ¶ ÄW²µd   :³¶¢·² ³5 ¶   µ5 · ²  ® Š
ËrÌ<€Ž’„‘’OŠ 

  Q#

³5 ¶  ²½ ´²¶²   ®
à µ  Q‚—6 6 „[+
    ½:r K#10„@5©|h“e„ –=²®˜ ®8€  Á: ³¶[Š²µ5
 ÁŒ²Å Áˆ ³K
Y¼¢k#10„@5q6 „ ®
à µ$‚<67„[+ V¾ ½:W¶ ~¿À Á²¶ ²  ¸ ³%  #10  5q67„l4 µ¢5   ®W°˜® ˆ ®  ® # :³¶2 ‡
¿À Á²¶    
¡ + µ5½:Á²µ5 ² ²=y+ :³¶¢·² 
³5 ¶  ®
·¢ ¼ k#10„@5q6² ®
 à µ  6µ„[W + V¾ ½:W¶  ¿… Á²¶ ²  7 ³r _#102‚k583V#10´±$‚k5T6 #102‚k5F4 µd5 O‚ ®
°˜® ˆ ®  ® # :³5%Wµ5 ®€  (²r Á¸µ5:³¶¶µ5²µ5¸ V¾ ½²¶7Å~²¶²µ5³%( ž³¶¶ ¿À³ ®
 à µQ‚<6   +  ·¶²˜Á:²¶5/¿À ³$#1
0  5q6 [0#10  55 $±V²+ 4 µ¢5 r²¶²µ  ®
à µF V¾»·³¶µ5_³¶( ž³5 ³F  #10´²e5   ®[€ µ5 ´¸  µ5³iWµiµ5½:Á²µ¶µ5²Á:/³¶µ VŒ „ 
 #_0 5   ³  ®y° ²¶5¸½² 5½
:³5%´µ/( µ g67„ ®   V „  ²¶²µ¢ÄŽ ®€  ³¶( ž³5


  #
0 5   ³  ® :  µ5³
   

 

 ³  
 # ! %  6  -0¡#_0 55 ±V²   #N0 5 ±V²
  
 
‡\²µ¶²Š µ5l½² 5½7µ5 ˆ´²WŒŒ V¾ ½² 5½l¶µ¶‡ž ‡µ5 ® : ³¶¡+ # ! ³  %   ³ ±)² 
³ …· I¾»µl:Š ³r V¾ ± 4  ± :Š ³
¸µ5½:Á²µ¶µ5²Á:e¼W+ Á:ŒS Á ± ²Š ´
‡l½²  ³¶ ¶µ¶  ® 
 


 ³\Á:² ‡ZÁ: ¶µ5: /# ³5  ·( µ¶½: ® ‹   Á/#10´²e5<|‘“@±\² l „ =²®˜ ®2¼ ¢ Z Á ¶µ5 ³
³ ³Áˆ ³Ol ˆ ²´³²µK
¢ ¼ k#10´²e5q6W² ®
 à µ\‚96 ²( + V¾ ½:W¶ Z¿À Á²¶ ²   ³ˆ¾ ½:Á²µ¶K#10  3”²e5c6 #10  5n±9#10  ± ²e5I+ ( µ¸5 
²¶²µ  ®  ¸µ5½:Á²µ¶µ5²Á:³³r )Á² 5½Á: ˜  µ5³Qˆ#102‚k5q6 Z ‚ ( µ%5 d²¶²µO.‚ Á:

Á: ¶µ5:v7 ˆ  ´ˆ \O# ³5 ¢·( µ¶½: ®
¢ ¼ k#10´²e5q6±\² ®
 à µX‚ 6 ²S + V¾ ½:W¶ .¿À Á²¶ ²  ~³ˆ¾ ½:Á²µ¶N#10  3²e513 #10  5137#10  ±7²e5X6 „[<+ ( µ
5 ˜²¶²µ  ® S Ž \³¶5¦0#10  55  ´½²µ¶ ÄW ÁŒŒµ5² ‡¶ O
µ5½:Á²µ¶µ5²Á: ½ˆµ5I¾ µ5µ5
ˆ Á:  )Á²²5³Á: ³¶5³ ®k€ .´ž ~ Ál¶ ³5²µ
‡ µ5 µ¶½²5½ ­® ¨Ž ¾ ½:W¶ ™C  3 ®C 3B²!6 „
~
  ²±\l ² Á:  ³5 ¶ Z · I+ ²/ Á(+ 4 µ/5   µ„[(+ .g#10  5$6 0   3  5J0´±\²e5
!  ²  ³5 ˜²Å Á: ³¶5:³ ® < Ž :³dÁ: ¶ ³c#10„@5n6…² ²Q#10´²e5T6 ±\
² Á: ³5²cˆ  67„
²  6¥²I – ²/ Á_#10  5O6¥0´±\²e5  ( µd5   B„ ®yà µ
Áˆ³57Wµ¶5½(+ Z²O½:/
#10  516¸0´±\²e5 4 µ¢5 r²¶²µ  ®
% ½:Á²µ5 ² ²=W+  ´½²µ¶ ÄW
³³r )Á² 5½Œ¸Á:²¶5/¿… Á²¶  :³¶¢·² 
³5 ¶  ®
% ¼ #10´²e5q6 „ ®
ºž­
 à µ $‚<6²+
 V¾ ½:W¶  ¿À Á²¶ ²  
³ˆ¾ ½:Á²µ¶%Á:²¶5˜¿À ³  $#10 3 ²e5q6…±O#10 ±4²e5
 i( µ¢5 
²¶²µ  ®  " #10„@5K6 ² N#10´²e5\6 „[+
\² &·
S  ½:l  µ5³Œ¿À Á² ² ²Wµµ5½:Á²µ¶µ5²Á: …² Wµ¶5½ e¼
+
 ( µ%5 %²¶²µ 

#10   5T6²– #10   3 ²e5T6 „ – #10   3 Y5T6±\² ² #10   3 Y5T6 „


% ½:Á²µ5 ² ²=W+  ´½²µ¶ ÄW
³³r )Á² 5½Œ¸Á:²¶5/¿… Á²¶  :³¶¢·² 
³5 ¶  ®
 W ² ²=+  < l Á¸¶µ5 ³%³ ¶ ³ ®
ËrÌ<€Ž’„‘’ •

X#
  y€ =+
 ³5 ¶ p½²´²¶²   ® p4²
5  ³¶5µ5²µ5²µ<¾»  µ5³ I( µ¸5 5
g,j°( 1# 02,5a3
Á: ³55/µ5½:²    ‡
¿À Á²¶  Š  :³¶¢½² ² ²%³5 ¶  ~µ5 · ²  ®W° ² ‡ ³
+ + + v#10„@516 „
 
 ²žµ5½
/ ·(²µ¶5½ ²%(²µ¶ ²%I¾ 4 ž³5²µd Wµ%ÇŲ     ®
‰.—6„[+
à µ e+ y, +G#  6  #102,5 +
4 a´²¸  µ5³ W( µ/5 ˜µ5½:² ! %  ®(à µ¸³¶5 44 µ/5 ³˜µ5½:² ³
,k–.G+ ³  6µ# ³  6  ³ + ˆ 1# 02,_34.5q67#102,85839#102.[5
    
  !  %   Á ® ®  ® ®
€  Y+
 µ5:Á: W l V¾ ½:W¶   ° Á ±    ³ 7 :³³ ¶ ³7Á: ¶:³\³5 7 :³
¿À Á²¶ ³¢ ½ˆµ5:³ ®
€ Š
 ²~½:2¿V Á² ² ²d/ :³¢³5 ¶ ³¢/ ¶µ5/µ5 · ² Œ³5 ¢ :³F¿À Á²¶ ³%* ) :³ ®

ËrÌ<€Ž’„‘’ ¦

nª g#
  7 µ5½:² ( µ¨ :²  WÇų¶5¢%¿… Á²¶  ~³¶³¿V³  :³iÁ: ¶ ³2¢ V¾ ½² Á:½ ®
‹
³5³ˆ  ³r¸½²5²µ¶²µd² :³%´ž ²µ5³% \ 1
# 02,5v
 à µ  ² Q,g6 „ -.!6²+ v#  6)ª
 I´² !%
 à µ  ² Q,g6 „ -.!6  +
   ½:˜  µ5³ !  % Q# 6 ª 
 Q,g6 $.!6²+ !  %  X# Q6 ªN±™ªF
Q,g6  $.!6 + X#  Q6 ªF-± ª
 à µ ²   ·¶²
 à µ  ²  <´² !  %
€
    Ár:´ µ
 ª  ± ª 67ª+ ˆ ± ª0ªT±j²e5 ªN±  Q6 „[+
Á ® ®  ® !  %   ²Á: µ5  ® -ª¦|&“e„ –  –=²®˜
Fª_67„ ®
Ž ¾ ½:W¶ O¿À Á²¶ ²   ³ˆ¾ ½:Á²µ¶K# !  ³  % 6 #102,8%5 ( µ/5  ³c,–.™|  „ –=² ¨ 5² ³¸N, µ. ®
à µN,96 „[+S  ² ½:7#10 U5X6 „ 4 µ5   |  „ –  ®Sà µN,V6 +  ² ½:
  µ5³›#10U5\6¥„ ( µŒ5  ™|#  –  ®g° ³¶½²µ5 ³7 ‡ ³¶ 5/0, /  <

)  5 ½Çĝ WµK,6¥„ ²


, ³  6   ³ ( µ-‚$V„ ®   

¼ 2:³¶7¿À Á² ~ ´½²µ¶ ÄW²µ  µ5³5¦, ½:Á²µ¶ „ –, 2  ²_. ½:Á²µ¶  ,k–=² ¡¨–   µ5³  ³  ½:Á²µ¶
 „ –, ³   ‹ =+<Wµµ5½:Á²µ¶µ5²Á:a+ pµ5 ´\³5i³¶n# :³¶
   ³¶µ  „ –, F   µ5³²   V¾ :³¶


³³¶³¶µ  „ –, ³   ®@€ µ=ž+ Wµ µ5½:Á²µ¶µ5²Á:d½² ² ²=+    ¶µ5%³ ³  )Á² 5½r2 ‡/³¶5K02, 5
7
 

:³5rÁ²µ5 ³5³5²d•œ µ5½:ŒWµX² ® ‹  ŒÁ: ´²µ¶7 Á´²µ5³%


5 n

<´½²µ¶ ÄW V 6 ³  +

Á ® ˆ ®  ® ´²µ5³\² ®
à µ~³¶5+n9 # :³¶    ³5µ  „ –=²  ® ‹ =¨+ Wµ Á: ¶5½¢+ ²    Á    ³¶µ  „ –=² o2+ Á: 
Á: ¶µ5:d\#10´²e5q6² ®
à µrÁ: ³½:²Qª6  „ <¾ :³¶¢W ³¢Œ³5 ¶   µ5 · ²Š   ®
Fª_6…² ®
Ž ¾ ½:W¶ .¿… Á²¶ ²   ³š¾ ½:Á²µ¶‰# !  ³  %6 #102.r5 4 µ/5 ³‰,k–.™|  „ –=² i 5² ³¸_, µ. ®
€  µ5 ´/!²;  
<¾»  µ5³Q#102,516…¸² 4 µ¢5 O,<|  „ =² oy+ Á:
<Á: ¶µ5:d\#10„@5q6 „ ®
à µr³55+ª_6…¸ ² <¾ :³¶¢W ³%¸³ ¶   µ5 · ²Š   ®
Fª_6  ®
Ž ¾ ½:W¶  n¿À Á²¶ ²  O³ˆ¾ ½:Á²µ¶—# !  ³  % 6   
³    ( µ5 ³¦,k–.¸|  „ –=²
 5² ³\
, ”. ®4¼ a<¾ OW ³\ˆ Á ± ²µ5Á ± ²µŒ·²O  Z( µ/¶µ5 ´²µ¸7¿… Á²¶ ZÁ: ´²W· &…· ³¶µ¶5 
³¶
  ½ œ7 ˆ ¶µ5½¸ V¾ ÇŲµ5Á²Á: Æ ¼c V¾ ²¶5½¸¿V%¶µl:Š ³¢·² V¾» <µ5 ®


ºžÈ
 =+
W ² ²  ‡³² ¸³5 ¶  :³¶ $ª!6  ®
ËrÌ<€Ž’„‘’O” 

: ¡+ +
µ¶ µ¶  (³¶ ) id¶µ5 ´²µFd³5² %¿… Á²¶  W´½²µ¶ ÄWd V¾ ½:W¶ \¿À Á²¶ ²   ²2 
‰ˆ ‰ˆ
( µ¶µ¢³ˆ¾»¶5²µ5 ŒÁ:˜dÁ:² ‡˜³5 iW ³F·²  )Á² 
µ5½ˆ ³²µ ® d˜²  Y Ž
¾ ½:W¶ 

¿À Á²¶ ²   <¾ ½²W ³¶µ :³l³¶  ¦ˆ =+
µ5½ ± ²²µ a ³5 ¶  \³5\W ³7´µ ²
‹ Š
Å~ ²Å ® ³5³ˆ  ³r Á¸¸µ5²µ5
¸µ5 · ² ¸ ³r³5 W ±  ®
#
  *~³5 ¶  ½²´²¶²   ®i°    ³¶ :³lÁ: ³5² ³  ‡ Wµ¶¶ Á: µ5³ a  ´  =+
Q# U€ =+
Á ± ²µ5Á ± ²µ¢² :³iµ5 µ¶½²5½:³2 ® µ  µ5²µ5r ½:‡5² ²% v# œ ‹
:³¶i :Á²¶´ ® 
=+ kª–d«\| ³ ‡ c#10ª 5^6W#10«
5+
 I² W³5 ¤ ²
  y 
 

5
µ ³   
6”#10ªF#10ª[55^6”#10ª #10«
55n6   ²/ Á Kª›6W«
· \#10ª[5c6 ƒ „@¼ 


‘³ ®
‹ ³5³ˆ  ³%5²W˜¸½²5²µ¶²µ˜Á:²µ¶:³r´ž ²µ5³¢Wµ¶¶Á² Š²µ5:³N
 à µ-,g6 .!6W²+  _#10#10´²e55q67#10´²e5 ® O³=+ ~´²%d´ µ%‰# :³5Fœ¶:Á²¶´+ Á
#10´²e516…² ®€  ² ½:r ½:‡5² ²˜+ ( µ%5 $.g| ¤ ³ ‡ –  
#10#102.[5516 . ² ·Â ¼
 ³Œ  µ5³X# :³¶¸· œ :Á²¶´/II¾»µl:Š ³ŒÁ² :³5³¶³e( +  ³¶ ) 
( µ
Á:² ‡ lµ5 ´²µ<¾ ²  :³¶
³5µœ :Á²¶´ ®€ µ=W + ³¶D.—| ¤ ³ ‡ + ‡µ5² ‡¶ }·  %¼ Á: cˆja.›6B# #   W+ Á:S  ¶µ57K.
  ²r·²  5½:Á:½:dWµQ# ®

 à µv,<6W#  yW +  Q´²c# =#  c#102.5 ~6    6 ²c6#10´²e5K…· I¾»µl:Š ³·  ¼¼ ® " Á+
³\O

# :³¶¢¶œ :Á²¶´+  7( µ¢5 $.g| ¤ ³ ‡ +
# . ² #102.[5T6W²


·Æ ¼
 à µ¢5 ³^,–.j| ¤ ³ ‡  + ³5c#  :³¶F· œ :Á²¶´+  IÇų¶5©| ¤ ³ ‡ 5² IQ.6 # ! 
% ®Ã µ
³55 + I¾»µl:Š ³X·  ¼+  #102.5167# !o# !
% %Q6 ® ‹ d Á!
#102,a.5167# ,# ² 6 #102,5167#102,85´#102.[5

 
·­ ¼
Ž X· ¼ K· ¼ !;
<:³¢µ5² ‡¶ ³  F² ­ F³ ¢5 r/ ² ¸·²  ³džµ5½ˆ· :³rd V¾ ½:W¶  ¿À Á²¶ 
!; vˆ =+Fˆ 8+ ^ˆ
²  r¶‡  ®¼ µ5:³¶5dW ²  ¸µ5 ´²µF<¾ ²  :³F³5  ¸²  :³¨²Å ½:´ž ²5:³
Á:²¶5
²µ¶ ²µ5 ® Š
€ =+ j% ³ ‡ ( ³ ‡
µ W³¶  ¤ ¤ +
:³¶d¸¿À Á²¶  5²  7 4 µd5 ³ O,k–./| ³ ‡ + G0 G02.55-6
¤    ²
02,a.5-6 G02,5 G02.5+<  µ5³˜4 µ˜5 ³v,–.&| ¤ ³ ‡ –4 Z 02,,G02.55$6 G02,85 G0G02.55Q6
 
+yÁ:
 ³³¶µ5¸~:³¶¢´½²µ¶ ÄW¸·² V¾ ½:W¶  ¿À Á²¶ ²  ¸¶‡  ®
 W ² ²=+  (/ ³¢µ5:³¶5/ ³r<¾ ˆ7Á: ³¶¶µ¶µ5¸˜¿À Á²¶ /#&% ¤ ³ ‡ ( ¤ ³ ‡ ´½²µ¶ Äy
·  g¼ ²$· ­ ¼ ®@€ µ=ž+ \· ­ ¼ +  ½:? ½:‡5² ²2F³5  – •– ³5  :³? Œ·µ5:³ µ5²²µ5³
‘· Á²¡2+ pÁˆµ Á²l²Š µ5Zµ5²²µ <¾ :³¶ W ³ ¿…  ² ¡%+ ³  ¶µˆ´   p4 µ V¾»ˆ´²µ=+r² 
  


³  5³5<¾  ?¿VÁ: ³¶¶µ¶µ5¿À Á²¶  ³¶µ


:³Œµ¶ ² ³=< + :³½:Á: 4 ž³5¶ ³
² ¿À Á²5²µ5³µ5²²µ5³  ³³²µ5 ³ µ5 ·W· ² ² ¶ :³b¼i + ²³¶  – =– ³5 \:³ ²¶²µ5³
…· ½²´²¶²  ² ²/½²¶ ¿À³b¼
+   µ5³% (¿Vr



#0    ) 516µ#0  5   b#0 5  )


 

  

·È ¼

ºÉ
Ž c0 5 Q· ˆ
¾ ½:%:³¶   µ5³ 2Á: ³¶½²µ5²µ ‡/³55  g:³? ·µ5:³?µ5²²µ5³¨ µ5 ½:³ …Á ® ®  ® 6 +
 
 6 + ¼ =+ µ²!

 ²5Á ® 2²%¸( ž³5²µ ( µ¢5 r²¶²µ  

#0  ¶  516  ² #0  15 6 ² 


 ¶
  


à ³=+žI¾ ½²5²µ52 ‡/Á: ³¶¶µ¶Á²¶ -#gˆ ¤ ³ ‡ 5  ²¶²µ ®  ¨µ¶ ² ,V„¸³š¾ ½:Á²µ¶aF¿V†: 
³5 ³/ ‡\¿À µ¶ , 6 %    +I !Z :³ ³5 Œ:³¸²¶²µ5³² Á: :³=< + ³5² ² ²

 ·µ57ÄgI¾ ²¶µ5²Å ½²˜ O ³ ®D€  ½ÇÄŒ  µ5³‰#102,5\L= · È ¼ ®4¼ g<¾ :³¶/W ³/ )Á² 


˜´½²µ¶ ÄW²µ%+³¢Á::³¢Á: ¶ ³=+ :³2µ5² ‡¶ ³K·  i


¼ ²\· ­ ¨¼ ³ ¢´½²µ¶ ÄW½::³=+ ²¢ Á/‰# :³¶
·² ¸³5 ¶   µ5 · ²Š   ®

ËrÌ<€Ž’„‘’ Ê

Ž ½
¾ ½:¸:³¶¢ %Á²µ5½:²µ  T¾
 ˜ ‡l³¶  ½²¶µ¶/³2 V¾ ½:W¶  ¿À Á²¶ ²   ®   $#
¸³5 ¶ 
G€ X# Š \#
½²´²¶²   ®   5
‡  V  ² 
5½²µ5½:Œ ®à µd5 ³%²¶²µ5³ ~–‚q+
      µ5³

# !o#  0 <539#  02‚k5 %c6±O#  !o#  0 <583 ² %O±~‚


! !

ŽS ²Œ·µ5  Á ± ½²l³¶  ½²¶µ¶² ²K‚q+IÁ:² F µ5 5  V¾ ;²¶µ5½² ² ²=+
I¾ !
#  !¡#  0 <5D37² %n34‚<67#  !¡#  02‚k583 ² %^3
! "

Á ®ˆ ® ®
±™‚<6 #  ! #  0 <583 ² % ± #  ! #  02‚k5837² %
!

€ µ=+ˆI¾»µlŠ:³I V¾ ½:W¶ 
¿À Á²¶ ²   ¶‡ T#  ! #  0 <5D3 ² % 67# ! ±O# 
0 Y5y± % 67#10´±  ±
<5+ !  67#0
Y5 ® " Á+4 µr5 ³%²¶²µ5³ ~–‚q+  g
!

±~‚<67#10´±  ± <5y± #10´±  ±~‚k5


à µ©‚<6±  ² g6± ±  – \½:2%( µ¨5 i²¶²µ +U#10 5167#10„@5a± ®  5 ³
<¾ ² Wµ¶¶Á² ²µ=+#10#10 855167#10#10„@5y± 516µ#10„@5y±V0#10„@5y± 5T6
   
  ®
 

% ½:Á²µ5 ² ²=4+ ³5 # % &°( ‰± O ! ¦| £ :³¶dÁ: ³¶5I+   µ5³=W+ ( µ˜5 ³d²¶²µ5³


~–‚q+  7I¾ dWµ¶$#10 W3 #10#102‚k555167#10 W3~‚k5q6 -± ±—O‚ ²=+ I¾»¶µ5/Wµ¶‰±O#10#10 W3
  


²e551±~‚<6± ±9²$±™‚ ®à µr³55+ ‡l³5² 


³5 ¶   µ5 · ²Š  
:³¶O#&% g°( ²-± ®
! !  !
 

ËrÌ<€Ž’„‘’.¥

Q#
  
³5 ¶  ½²´²¶²   ®
 à µ Q.6),  + Q# #102,85834,  6µ#10„@5D3 ,  #102,85
 I´² ! % 
 Ã µ Q.6±Q#102,5+
  ·¶² c#10„@5q67# #102,85834, ± 0#102,55 
! % 
‹ =+ +
7 ¶ WgÁ::³Q²Å
µ5² ‡¶ ³ š Œ½:g ˆ4 µQ5 <µ5½:² @, +  #102,5 ! #102,5 ±~,G % 6
„ ‹ K#102,5167„ <#102,516 , 
® r Á    ®
 ³a¶5²¶ D+ ( µa1#<¾ :³¶QW ³S¿À µ5Á:½² ²a ‡2¿À Á²¶   2  ‡¢¿… Á²¶ lÁˆµ¶µ5½:+
?Á:²4²Œ³5 ¸·².:³
³5 ¶ ³
Zµ5 · ²Š   ® ‹  l4 µ¶µ
Á: $ÀÁ²²µˆ´:Á V¾  
V¾»¶µ5Œ
Á::³%²Å ¿… Á²¶ ³d³5²   :³%´ž ²µ5³%c, ®  ¶µ5 ³r¸Á:² ‡l<¾ :³¶¢W ³% ¸Áˆ ³ ®
4 ž³5 ³¸ Ál<¾  ?ÇŐ³¶5Nªh|™' ‡ 5² ?#10ª[5$6„~…· ³/ lÁˆ ³¸Á: ¶µµ5I + .µ
#10ª[516 ªF/ 4 µ¢5 rµ5½:² ª ¼ ®
 à µQ,j6µª + V¾ ½:W¶  ¿À Á²¶ ²  
¶‡ ³ˆ¾ ½:Á²µ¶K#102.5q67# ! ªF-±~. % ( µ%5 %µ5½:²
. ®8€ µ=4+ ³5k.}6 ƒ      µ5³c.  6 ƒ ! ª  ±~. %  ²/ Ál O 
ˆ´ µK#102.[5-6…# ! ª  ±~. % 6¸„ ® : ³¶¡+
#102.516 l„ ( µ%5 $.h6 ƒ    ®
º9@
=+ n,—6    + (³¶ ) ¢dÁ ± ³¶µ^.&6 ƒ „75² Iv#     3~.&6 ƒ    ²      ±/.&6 ƒ    ®
 ³ ( µ
à µ¸Á::³r´ž ²µ5³=W + V¾ ½:W¶  ¿À Á²¶ ²  ¶‡ 7Á: 
  µ5³dµ5:Á²5² ²Kˆ›#    ~6W„ ®
‹ r ÁK# :³¶¢ ‡7¿À Á²¶     ®
 W ² ²=+  < l²Å ³5 ¶ ³r<³5 r ‡7¿… Á²¶    
²$,j(° ,  ®
ËrÌ<€Ž’„‘’ 

Q#
  +
/¿… Á²¶  ³¶³¿V³% :³rÁ: ¶ ³r¸ V¾ ½² Á:½ W¶µ5Œ¸ ‡¿À Á²¶     ®
Q,g6).+ Q#10 ®,85´#10„@5q6…0#102,55 
 à µ  <´²
$,&|j' /( µ¢5  ®
K#
à ³5 + X#10„@5v6 ƒ „
<¾ :³¶%W ³¢ ‡¿À Á²¶     W ‡7µ5² ‡¶  µ5½:Á:½:²5Œ²¶µ ®
! ‰ª¦|j' c#10ª[5q6 „  œ
à µ% V¾»·³5µ5 ³¶4 ž³ ³%<¾  IÇų¶5 lŠ 5² < ®   µ5³rI¾»µ :³% ‡7µ5² ‡Ž
¶  µ5½:Á:½:²5 y   
+ _„N67#10ª 5´#10„@516 #  + <#  67„  ! !  % % I¾  ! !  % ® /µ5½:Á²µ¶µ5²Á:Œ ½:‡5
Á: vˆ!#  ) 6 „ Q‚ V„
!  % ( µ¢5 r²¶²µ $ ®
¶  ®
  ³  ) 67„ Q#
³ "  + K#10„@5T6
 %  ² O:³¶rÁ: ¶ 4 Á ³ #  ) v67„ "   % !  % ®y° ¶µ ÁÇ

ËrÌ<€Ž’„‘’ ©

n# N·
  \2¿… Á²¶ ´½²µ¶ Äy¨ :³¨Á: ¶ ³i2 V¾ ½² Á:½ ‘ 5 ³¨<¾  ÇŐ³¶5%·² ¢5²  :³
\
¿À Á²¶ ³  ‡¿À Á²¶  —,j°(   !  | ¨±K²–=² 
Q:³¶%¸Á: ³¶5 ® e¼
€ 4 ž³  ›%Y,—(° # ,3 ± #102, 5 " lŠ
! % +
® ¾»µ :³% V¾ ½:W¶  ¿À Á²¶ ²   W( µ%5 %µ5½:²  ,
´² X
 [,!3
² 6µ# ,_3  ² ± # ,_3
² 67# [,3 ² ± #102,8516 G02,85








 

à µ2³¶5+ ‡/¿… Á²¶  l:³5  V(½²µ¶  ® ‹  d:³¶i Ád½² ² ²‰²ÇV(½²µ¶ X·‘³5
²v6
  ¼ ® : ³¶¡+ ( µ%5 %µ5½: ² D,   G02,!3 ²e5q6 G02,8¢5 Á ® ˆ ®  ® # !¡,_3 ²n3  %$± #102,3 ²e5q6
# ! ,_3 % ± #102,5+  ²Á: µ5

# ,!3 ²T3 ² ± # ,_3 ² 6 #102,3 ²e5y± #102,85


 

 

° ² ‡³¶ž ÄW
/ ‡¿… Á²¶ —%,j°( #102,_3 ²e51±™#102,F 5 :³¶  V(½²µ¶  ® ‹ d Á
²  ¸:³¶
³³¶qDz V4½²µ¶  ®à µd³¶5 + ( µ¢5 %µ5½:² D,  02,!3 ²e5q6 k02,25 Á ® ˆ ®  ®
#102,N3 Y5y± #102,_37²e5q67#102,_37²e5y± #102,5 ·É ¼
  ª¦|j' ÄŽ ® " v· É ¼+ 7½:?¨ ‡dW¶5½-#10ª3_‚k5®±K#10ª3_‚$±!²e( 5 :³¶Q½²4²5
c‚ ®  5 ³ ¸Á:²¶5¸´ž ²µrÁ:  ® :  µ5³= + 4 µr5 %²¶²µ  µ²
#10ª‰3  5q67#10ª[583   #10ª‰3 51± #10ªc3 ±V²e5q6  T3V#10ª 5
 


4 V¾  ¿À25²µ5  ´²µ5³O3
  +  Á: ³¶5¸+ ³5‰# :³¶i·( µ¶½:+ Áž¾ :³¶¢ Á/ O6 „ ®
O³r  µ5³O#10ª‰3 ²e5y± #10ª 516 „[W + ²%Á:¸4 µ¢5 rµ5½:² ª ®à µd³55+O# :³¶\Dz V4½²µ¶  ®
ËrÌ<€Ž’„‘’p{ 

‰# j·
  .
¿À Á²¶ O´½²µ¶ Äy¸ :³/Á: ¶ ³¸7 V¾ ½² Á:½ ‘5²  ¿À Á²¶ ZÇŐ³¶5·² ®
! e¼
à µrÇŲ   V¾ ²¶5½ ®
 Ã µ Nª&6 ² N«N6 + ¼
I ‡ Á: ¶ p /Á:  Xˆ/#10 Y5‰6¥#10 Y5´#10 Y5 k€ =+ 46

 ® µ I( µ  ²
\6 + 5 K#10 Y5q6 #10 Y5 X#10 Y5c6 ƒ „[+ \#10 Y5q6

  ‡lÁ: ¶  7 ¢     ®y°      ² ½:d
 ®

º *
g6 \6 + ¼
 à µ     ‡lÁ: ¶   FÁ: ˜  µ5³ Oˆ!#10 516
 ®
 à µ  }6 ²  —6 + ¼ ¦#10 Y5\6 ‰3 #10 Y5 : =+ 96
I ‡ Á: ¶   /    ®  µ5³ I( µ   ²
K6 + ¼
 ‡ Á: ¶   2Á:  ‰ˆ!#10 5n6B#10 Y5D3 X6B#10 Y53

 ®  ³r4 µ ² 
 ‡/6 X6 F+
¼ Q#10´² Y5©6B#10 53V#10 Y5T6 #10 Y53  + Oª6
  
Á: ¶   2 7Á:²¶5¸¿À ³ !
  ® ³d ( µ  ²
«Q6 + ¼ ‰ˆ!#10´² Y5^6 #10 Y5 € X#10 Y5©6 +

W ‡ Á: ¶   ¢Á:  !   ®  ² ½:/  µ5³d   ³r
#10 Y516 Q#10 516
 ²

®
à µr½²5²µ¶²µv#10´² Y52 IÁ: ´²˜¸³:´ µ%² :³¶2µ5²²µ=+²%² $3 K6
„„ N6
²² ™² ®¼ <³¶ ) r Á
/¶µ5 ´²µv#10´²²e5¢²O#10´² Y5 ®

99
99
99

 à µvª_6  ²O«^6 + ‡Á: ¶  2 ¼ Á: vˆ!#10´² Y5q6² ®


  

 à µ g6² ² \ 6 + ‡ Á: ¶  2¼  Œ  µ5³v#10´² Y5q67#10´² Y5y± #10
Y516…² ®




 Ã µ g6²
² ² \6 + ‡ Á: ¶  2¼  Œ  µ5³v#10´²²e5q67#10´² Y5y± #1
0 Y516…²² ®
   
 

€ l²½: -#1
0 „„ Y5T67#1
0 Y5´#10
5´#10´²²e5´#10´² Y5©6 „„ ž+ ³?©#10´² Y5©67#10
„„ Y5Y±
  

#1
0 Y516…² ®

99

99

ËrÌ<€Ž’„‘’p{W{

K#
  .³5 ¶ .½²´²¶²   ®   c, |V“@±K² =²®˜ Ž
® S\¿À µ¶  V¾ ½:W¶ Z¿À Á²¶ ²  
( ³5³5 ~ˆ I:Á²¶²µ :³ ²Å Á ± ² ²5³  ´žµ¶‡· :³ ³¶´ž5³ ˆ O(²~W ³  ²µ ™·†
e¼c

² ¸Á ± ž³5

µ5
€
  ( ž³5 q6  ³¶  +
   µ5³ ² |h“@±\²–=²®˜ -,j6  ³¶
Ž

® ¾ ½:W¶  ¿… Á²¶ ²  Œ³ˆ¾ ½:Á²µ¶r  µ5³ \
#10 l5839# D3y± ² 6 $3²-±




( µ¢5  |h“@±\² =²®˜ ®


€
  ( ž³5 q6  ¶³  +  µ5³ |h“@±\²–=²®˜ ²-,j6
 ¶ ³
® Ž ¾ ½:W¶  ¿… Á²¶ ²  Œ³ˆ¾ ½:Á²µ¶r  µ5³\
# O3²$± 39#10 516 ²T± 3




( µ¢5  |h“@±\² =²®˜


®
‹  ³5  ²Œ·µ5 /ˆ  ²·µ5 :³ ²Å µ5² ‡¶ ³\ ·5²:³=+  ¢´²=+a4 µ5  |
“@±K² =²®˜¦
%  6 
1
# 0
 l5 D 9
3 # y± 93 #10 O3 6 #10583
²$±  D3 ² ²$±



 

‡˜²µ¶Š²µ5¢½² 5½¢½² ·5²%²l¶ ³ V¾ ½:W¶ l¿À Á²¶ ²  2¶‡  ®ž¼ ´²T#10l516
 ¶ 

 ± 
®
% ½:Á²µ5 ² ²=W+  <<¾ :³5%W ³d )Á² ¸´½²µ¶ ÄW²µ/¸ ‡l¿… Á²¶ <,—°(  ¶   ±  :³¶%·²
Œ³5 ¶   µ5 · ²Š   ® 
ËrÌ<€Ž’„‘’p{Š

  _# † Š
 ³5 ¶  ²½ ´²¶²   ® °  ² : ³lWµl½²5²µ¶²µ :³µ5² ²µ5:³l´  ²µ5³7
#102‚k5
¨( µ% ´ µ% 4²  Ž ¼ +
 ²Å ® S Á: ¶   i 
 4 µ¢5 ³d²¶²µ5³ ~–‚ Œ„[+
$
#10 3}‚k5Œ#10 <5839#102‚k5
! ! ·¼ @

‹  Wµ¶¶Á² ²µ=+4( µ 6µ‚~6 ²+W Q´²v#1


0 Y%5  #10´²e5 ® ‹ / ÁX#10´²e5^6”„<·…I¾»µlŠ:³˜¼¼ ®
à µ 6 ²$‚h6²+ <´²v#10 Y5T67#1
0 Y5D3Œ#10´²e5D3 T6 J+ ! -|~“e„=²®˜ ®4à ³5K#10 Y5Œ„[+
Áž¾ :³5% Á
\#10 Y5T6² ®
 


ŽS µ5² ‡¶ µ· ¼


4²µ¶ ²  µ5³l µ5²µl :³ Œ·µ5:³!#10 ®‚k
5 Wµµ5½:Á²µ¶µ5²Á:hg²  I²=+
( µ25 %²¶²µ$‚#V„[+  _#10 [02‚!3 ²e5 5 Œ#10 ®‚k5G3 #10 Y5T67#10 ®‚k53 ² ®  ˜µ5½:Á²µ¶µ5²Á:
³³
@  

 )Á² 5½ŒÁ: ˜  µ5³Oˆ!#10 ®‚k 5 V‚q+ 4 µ¢5 r²¶²µO$‚ V„ ®
   


 -
=+
° ²¶5r½² 5½dµ5 ˆ´²W%I¾ ½² 5½:³F³5Á:Á::³5³¶´:³ Áž¾ :³¶¨Á²4r V¾ \´ž¸( ´ µiÇÅ  5²µ
/#10 Y5n6
‡7Á: ¶  999  v‚™|&“e„ –=²– – U˜
®à µ¢ V¾»·³5µ5 ³¶( ž³5 ³2<¾  (Çų¶5 Œ5²
#10 ®‚k5 ‚ : \‚ !; † ˆ

   ®  µ5³  ²
‡ µ5½:Á²µ¶µ5²Á:µ5½:Á:½:²5 Á: 7  ‡~ ² ¿V : 
#10 [02‚_3 55 9‚N3
   ( µ25 F²¶²µ  5² ( Q‚N3 h| “=‚1–‚X39²– •– U˜ ‹
 =+
®  Wµ¶¶Á² ²µ
#10 Y5
 
999   <:³¶¢ ‡Á: ¶µ Á²¶  ³5  ± 5½: ®
" +
Á 4 µ¢5  $‚™|~e“ „ –=²– p– U˜Y–
 S  
#10 ®‚k516)‚
 ·* ¼
 ³g³5  :³g5²W ² :³¶µ5FFÁˆ Á² ²µy#10´² Y5 ® " ¾»µlŠ:³-· ¼<²^· * ¼+Ž X² X6
#10 K² Y5 Œ#1
0 K² Y5=3#10´² Y5  #10´² Y5 ® " ¾ !!#10´² Y5  
 +ŽÁ ® ˆ ®  ® #10´² Y5 „

9 @
9

‘· ³\#10´² Y% 5 :³¶%²¶²µb¼ ®

9
9
9
9
9
9 9

" ¾»¶µ5OWµ¶=r+ I¾»µl:Š ³ · \¼ ² · * \¼ 5 œ µ5³=¢+ 9 ³5³5‰#10´² Y5  #10´² „@5O3#1
0 Y5~6

9

#10´² „@5n67#10 „@5n6 „ ®  W ² ²c#10´² Y5T6 „


@
9
9

9  9 9
9 9 

¹ s! I´
[ € 4²  5²µi<¾ ¢5²  ¢¿… Á²¶ g\# Çų¶5 ®¼ ³¶ )  WµiÇŲ %rÁ: ³¶½²µ5²µ
#<%Y‚<°(  + !  g ½:³¶ž¸ ‡7Wµ¶¶Œ²¶²Š µ5 ®
 

ËrÌ<€Ž’„‘’p{•

  Q# 8€

³5 ¶  ½²´²¶²   ®  4 ž³ ¦%Y,—°(*,G-±V² Ž
99 ® ¨¾ ½:Œ:³¶r
µ5²µ5²µdI¾ 

(²p ³µ :³ :³Œ( 5³ŒÄÅ:³ ¦# Ž
²  ® <:³( 5³ŒÄÅ:³7 Z³5 7 :³7²Åp³5 ¶ ³
µ5½:²  :³d³¶¶Á²5:³%¸ V¾ ½:W¶  —,G-±~,›±V² N6 „[+ Oª O«
/ V¾  ´ž7 5²µ \² ®
j#10ª[5—6 j#10«J5/6 : ›ªµ6 G0ª 5<6 #10#10ª[55—6 #10 5+ G0 85/6

99
à ž³5 ³ ²  ®  µ5³  i²~  µ5³  
#10#10 855-6…#10ª 5-6 k+ " !; +
 
  yÁ:7gµ5 ´l .:³¶¸³5³¶gO4 ˜ÄÅ7  ® 7 ²  :³¶˜
( %ÄŸ  ®
" e+a#10 516 ª&67ƒ «-67#10 Y5
/ ³   F Á   ® 6 ƒ ‹ ¡+ ›“ – U˜‰6W“eªa–d«e˜
˜³¶     
€  4 ž³5 5²W k02,5›6 G0G02,855›6 !,-±Œ² %  ±”² ®¨¼ 2<¾ :³¶W ³ )Á²  
´½²µ¶ ÄW²µ%/ ‡Œ¿… Á²¶     ²  4 5³2ÄÅ:³¢³¶¶Á²5³ ® F² ²5²D+5 ¢( 2ÄÅ/

99
99

:³5%½² ² ²r 4 ¢ÄŸ  ®a€   5


 ÁKªa–d«•– ®– :³¢4 5³¢ÄÅ:³%  ®
Ž 
!²;  Œµ³5 ² ²/ŒÁ² :³5³¶³d4²µ¶ ²rŒµ5 ´²µdX# d
· ¶œ :Á²¶  


<
V¾ ²³²Œ· “eªa–d«•– ®– [  ˜ ³¶µ%  V!²;  g…· Á ® ˆ ®  ® “•#10ª[5b–d#10«
5b–d#10 =5b–d#10 F5d˜N6W“eª–d«•– ®– [˜®¼ ®D€ µ=+
²
7³5 dW ³/:³d4 5³rÄÅ:³/ Gy +  Á7Á:Œ7³5 dW ³/:³˜4 5³dÄÅ:³˜N.#    ³ ®yà µ
   

³55y + Z 


:´ µK#10 =5$6 ²\#10 5$6 ®  ³¸  µ5³ G0 =5O6 #10#10
55-6 y+ Á:lgÁ: ¶µ5:
 

 ˜
³5 ¢W ³r 4 ¢ÄŸ  ®
 W ² ²=+  <<¾ Çų¶5/W ³%¸¿… Á²¶ <#&%' ( ' 5²  :³r\# -#j6  ®
ËrÌ<€Ž’„‘’p{¦

€ Š
 Á: ³¶5¢µ² ²FF V¾ ²¶5½r:³¶a%³5 ¶  µ5 · ²  ®Ã µ5 ´ ³¨¢Áž¾ :³¶
‡l³²  ®  r Á K# 
³5 ¶  ½²´²¶²   ®
 €  Á:  ²Á:.Wµ µ5 ´²µ O V¾ ²³5²Œ·   6u“=,4±B#102,5b–, | '^˜ *:³¶ Ä ®%à µ
& : =+ 6ƒ ²
V¾»·³5µ5 g³¶( ž³5 ³l  ³5 7Ä ®  µ5³ Q ¢ÇŐ³55  Ä5½~\µ5½:² ³  ( µ
c, g| '
:³² ³¢ <Çų¶5
c, ± #_02, 5q6
5² Q  
® à + j· lŠ
µrÁ ±  Á² ¸Á::³¢µ5½:² ³     …I¾»µ :³
e¼‰
V¾ ½:W¶  ¿À Á²¶ ²  

#10  ±V²e5 6 #02, 5 ±Œ²$± #_02, 5 6 #_02, 5y±9²$±™, 6±\²-±
 ±Œ²  ±V²  ±V²  ±V²
° ² ‡ ²¶µŒ/ V¾ ²³5²· “   
–,&|j' ‡ ˜Œ:³¶%Ä ®4° ¶µ Á²¶  ®

 € Z(²
 ÁÁ: ³¶½²µ5²µ\ª~| ' 5²   ª_±™#10ª[5  ³5 / lŽÅ :³/ Œ·µ5:³  
½:Á²µ¶  ®  ¢  µ5³-«-6 ªQ± ²1±/#10ª 5 ®[€  X« ±/#10«
516 [0ªv±/#10ª[55 ® ŽQ
ŽÅ 5½

 µ5³ 

¯
  ª_± #10ª[5
 ²¶µ
  µ5³F vªv±&#10ª 5167„[+²2 Á·‘5 œ µ5³2( µ2Áˆ³5d˜ŽÅ 5½e¼
+
 ( µ%5 ¢µ5½:² D,k+#102,85q6),
®

ËrÌ<€Ž’„‘’p{”

g Q#
à µ¸ V¾»·³¶µ5 (³¶( ž³5 ³/l75²  :³d¿… Á²¶ ³ŒÇŐ³¶5² ®(à ³5 p:³5d ¶:Á²¶´ œ +
 (:³¶F¿V Á² ˜I¾ ² ½:µ5¸ v# !; œ +
:³¶2²  ÇV ² ¸ ¶:Á²¶´ ®Ã µ%³¶5  :³F Œ·µ5:³ O#10„@5+[#10´²e5
i²
#10´±K²e5 ™ˆ D€ =+ 1,k+
r³ 
²Å \²ÅZ³¶¶Á²5³ ® µ (( µ¸5 ¸µ5½:² (².Á: Œ·WŒ :³¸²Å µ5² ‡¶ ³ =+
 
# ! , % 67#10G0#102,8555T60#102,55 
" Á+?Á ±  Á²#:³ Œ·µ5:³¦#10„@5+q#10´²e75 ²!#10´±\²e75 :³¶ ½² Oˆ ³5  Áˆµ¶µ5½ ®¨°   V¾ ½:W¶ 
,…6*,G <¾»  ²  ²Å#³5 ¶ ³=F+ Áž¾ :³¶~ Á <¾»*  ³ ²Å#Wµ¶‰#10„@5+©#10´²e5²
#10´±K²eF5 ³5 r½²Å ®y° ¶µ Á²¶  ®
ËrÌ<€Ž’„‘’p{Ê

O N# + lŠ ¼+
à µQ V¾»·³¶µ5 ˆ³54 ž³5 ³S<¾  Çų¶5?¨5²  a¿À Á²¶  ®:à ³5 šI¾»µ :³S : ‡F¿À Á²¶ 
# +
:³¶¢·( µ¶½:   (²%Á: ³¶½²µ5²µd ‡7·4 µ¶
³54½²µ¶²µ5
  5½:  ® \#G–
 
 W„ ± „
 ~5²     ²
³¶µ/ :² .( ž³5²µ~:³¸Á: ¶ ³
Wµ
‡~³55 ®I¼
Çų¶57 ÁlOµ5½:² 5² ? yª #10ª[5 ± ‹ =+ # +
   ®  4³5 :³¶¸Á: ¶ < O4²¸³¶( ž³5²µ
‰ª&6 ƒ „ ®
" =+ ¼+
7 ³ I7 I Z½:
 j#10 Y5Q6 # d²n3

!   % 6¸#10´²e5y3B0#10´²e55  6 "
® ¾  !   ®
€ — 
   ± ^3 # ² 
Œ# aªc3 ª ²  6µ#10ª 583 # ª ²
 





 


ª 

à µr³¶5 +# !%



µ± ®  ³r  µ5³X
 Œ# ª ² 3}ª  67# ª ² 370#10ª 55  µ


 ± -370  ±®5 


ˆ
5q6¸0  ±®5  ±  ± v6  ±v± ²  ±v± Q± ² V„
Á ® ® ®
G0 


€ µ G02,85n6 !,¦±O±  %  ±c± v±  :³¶%


¿… Á²¶  Á²µ5 ³5³ 57³¶µ ©3  –b3
 ®G€ 
( ž³5   µ5³¦ˆO  W ³\½²W ³5³5²µ +¨Á: ˜ ³5³¶µ5 K3    ²\ Á 
0  5G0
Y5n6  ±  v± ^3 K6k0®5 ®  
¼ F¿Vl Á~ K0 5 „ ®  ³eQ + ³ 
  05!6 +g ¢:³¶l4 ž³5³5·  ~Á ± ³¶µ
¶‡ ² ² X| „ – Œ³5 µ¶5¸ k0 ®5V„[+Á:¸S!Š²
‡lÁ: ¶µ Á²¶ O½:³¶µ5½: ®
¹ s! I´
[bZ Y¼¢ < V¾  ´²O;²¶µ5/ ³dÁ: Á²µ5²=+ S³¶ ) ¢WµdÇŲ 
¸Á ± ³¶µ c6  ®
·¼ ‹ =a + ³¶F ³eQ+  ´²l½²´5²µ7 µ5:Á: µ5³!ˆ ‡  ¶  \·( µ¶\³¶(½²µ¶²µ5a+  (²
  µ5³¢Á ± ³¶µ  Á:  ¸½²% ˜ ³2(²¶¢•œ µ%\# I³5 ¢˜ ‡¿… µ¶ 
²¶²µ ®
 !gZ ‚ :³¶2


ËrÌ<€Ž’„‘’p{¥

Q#
  · e¼ O,–.G–[–$‰
/¿… Á²¶  µ5½:²   …³5 ³¢µ5½:³5²µ¶´ŒI¾ ÇŐ³¶5²Á: 25²  
/( µ%5 ³¢µ5½:² ³

0#102,5839#10 F55J0#102.[5D3V#1055167#102,.N±539#102,(834.F5 · ¯.- ¼


Y¼ à µ©,g6 Œ²q.6 T6 „[ž+  ´²  #102,5´#10„@5q 6 #10„@5+ ( µ 5 ¨µ5½:² , ® ‹ Wµ¶¶Á² ²µ=+
 0#10„@55 &6 #10„@5 ® ‹   Á+q#10„@56 Z „ V#10„@5!6  ® $#10„@5!6  a
  – ‡ µ5² ‡¶  µ5½:Á:½:²5
Á: vˆ!#102,516
 + ( µ%5 %µ5½:² 8, ®


žº
% =+
½:Á²µ5 ² ²  ‡¿À Á²¶  <#&%a02,j°(  52:³¶¢·² ³5 ¶   µ5 · Š²  ®
€  ³¶( ž³5/5²W˜\#10„@5q6 „ ®
·¼ à µ N6 q6 „ – ‡lµ5² ‡¶  · ¯.- ¢ ¼ ³ˆ¾ ½:Á²µ¶K
#102,5´#102.5167#102,a.5 · ¯ž¯ ¼
( µ/5 ³/µ5½:² ³‰,–. ® ‹ ZWµ¶¶Á² ²µ=I + ( µ/5 ¸µ5½:² 1,k(+ .¦#102,  5Q6Ÿ0#102,55  ®<°  5 
µ5½:² S4 ž³¶¶ ¿a   Q:³¶¢ Áˆµ¶µ5½ + Á:² ‡  ³³¶µ5¸\O# :³5¢4 ž³¶¶´¸    
³¶µ-'T³ ®
Áe¼GŽ<? ²Œ·µ5i µ5 5 a ‡2µ5² ‡¶ ¦· ¯.- ( ¼ :³¶I´žµ¶‡QWµ< ‡F¶µ³¿À µ¶¶ ¦02,k–.–[–$l5T°(
0–,–.G–=± 5 ®y¼ < d ÁŒ²/²; ¶µ5

!²;  
  ²Œ·µ57¸Á ±  ® ‹ r ÁW+ 4 µ¢5 ³rµ5½:² ³
,k–.–$– c
0#102,5839#10 U55J0#102.5D39#10 l5516…0#10 U539#102,55J0#102.5D39#10´± l55
 ²Á: µ5
0#102,5839#10 F55J0#10 5y± #10´± 55167„ · ¯ˆº ¼
( ž³5 ³d + ( µ%5 ³rµ5½:² ³$,– aW+  c#102,583V#10 F5q67„ ® ‹  Wµ¶¶Á² ²µ=y+ 4 µ$,/6 + 
´²v#102,516 „
% ½:Á²µ5 ² ²=+ ‡¿À Á²¶    :³¶¢·² ³5 ¶   µ5 · ²Š   ®


€  ³54 ž³5 5²Wj# <¾ :³¶lW ³l ‡ ¿À Á²¶     ® " ¾»µl:Š ³Á:~2µ5½:Á:½:a+ 
Çų¶5˜ Áv,– !|j'$4 + ( µi :³² ³-#102,5a3™#10 U5v6 ƒ „ ® O³F  µ5³e+ ‡Œµ5² ‡¶ ™· ¯ˆº ¨¼ µ5 ´˜
#10 l5167#10´± l5+ ( µ¢5 %µ5½:² J+Á:¸S ³5³5µ5K# :³¶¢Wµ5 ®
 5 ³%<¾»  µ5³=+ I¾»µl:Š ³r·¼+  # V „ ³¶µO' ®
a¼ " \· ¯ž¯ ¼+  ´²T#102,516µ#102,5´#10´²e5+ 4 µ?5 aµ5½:² [, ® ‹ =ž+ Á:  $ # <¾ :³5gW ³a ‡r¿À Á²¶ 
   + Áž¾ :³¶F Á/v#10´²e5T6² ® " ¾»¶µ5/Wµ¶=+ 4 µ^.!6 X6 q6²+ ‡Œµ5² ‡¶ ~· ¯.- i¼ Á: $ˆ
[0#102,85J3_#10´²e55T67#102,T±²e5J3_#102, 3¦²e5+ ( µS5 gµ5½:² U, ® ‹ a³¶ #102, 3¦²e5q 6 #102,85J3 ±\#102,n±²e5 ®
€ µ=+G#10„@5T6B„ ²c#10´²e5T6 ² ¼ S:³¶d  µ5³r¿À Á² ¸´½²µ¶ ÄW²µ˜Wµrµ5½:Á²µ¶µ5²Á:7K#102,85T6µ,  + ( µ
5 d²¶²µ$, 9„ ® ‹ r Á + Wµ%Wµ¶5½+  #102,8516),+ ( µ¢5 r²¶²µQ, ®
3

 ² k– 7²Å ²¶²µ5³ey + :´:Á ¦67ƒ „


" ¾»µl:Š ³¦· ¯ž¯ ¼<+     µ5³ _6‘#10 5c6‘#10 Y5´#  6 
#  I+ I¾ ! #  6  <+ Á:
   

          

 ³³¶µ5¸\#102,8516),   + ( µ¢5 %µ¶ ² k, ®


e¢ ¼  ²O,k– :³¢µ5½:² ³ ®à µO,j6). ²X6 J+ ‡7µ5² ‡¶ }· ¯.- F¼ Á: vˆg
0#102,539#10 U55  67# ! ,  ±   % 3V#1
0 ,5
à µ$,—6)~ . ² N6± J+²r² ¶ ³r ‡7Wµ¶5½Œ\#y+ ‡7µ5² ‡¶ }· ¯.- 2¼ Á: Qˆg
0#102,539#10 U55  67# ! ,  3   %
: ³¶¡+
# !,  3   %Q67# !¡,  ±   %©39#1
0 ,5 · ¯ˆÂ ¼
( µ¢5 ³%µ5½:² ³$,–  ®
 ²$ª– /|j'T³ ® ‹  Á ± ³¶³5³$,– !|—'T# ³ 5² ³¢v,$6 ªQ3 ² YO6 +  NªQ3 g6
,  3   ²Oª6),  ±   ® ‹ =+ I¾»µl:Š ³K· ¯ˆÂ 2¼ ²%Áe¼+  I´²˜  µ5³O#10ªc3 G5y± #10ª[5167#1
0 ,85 V„ ®
€  ² ½:r\O# :³5%Á²µ5 ³5³5Œ³¶µ$'q³ ®

žÂ
¿ ¸  ¼ \,Œ|}'T³ k€ ¦0 5 ›0 + 5
® p³ Œ<¾  iÇŐ³¶5²Å.³¶5:³   ¸²  ¸µ¶ ² ³Œ( ž³¶¶ ¿À³

 ³/5²  :³¸ 4( µ/5  + v‚ µ„[+ B, + +
 I .       (²     %      %   ³ a¼6 + g ³ + 6,
®
$‚ Œ„[+ \# lŠ
à µ%5  $ ³5 O:³¶%Á²µ5 ³5³5
²rI¾»µ :³% :³%µ5½:³¶ 5³%    

  67#_0  5Œ#102,85V# !  + % 6  + 
   

‹  ¿V³/5²µ5Np‚ ´²µ5³K3! +  g´²\#102,5-6W,  ® : ³¶¡+G#102,85-6”,  4 µ˜5 c, |~' ³ ® ‹ 


Wµ¢Wµ¶5½›#102,516),  4 µ¢5 O,<|j' ®
% ½:Á²µ5 ² ²=+ ‡¿À Á²¶ <#&%,g°( ,  :³¶¢·² ³5 ¶   µ5 · Š²  ®
 W ² ²=+ :³%³ ¶ ³r µ5 · ²Š  
³ Ka#<%Y,j(° ,+a#<%Y,j°(*  „ ²O#&%,j°(  ®
ËrÌ<€Ž’„‘’p{ 

 Q# 
³5 ¶  ½²´²¶²   ®
 à µQ,g6µ„[ +  <´²Q#10#102.55q6).\3µ0#10„@55 + ( µ%5 %µ5½:² . ®à ³K# $#O:³¶r*) +
²  Œ:³¶¢· œ :Á²¶´ ®G€  ² ½:¢¿V Á² ² ²dK# :³¶%½² ² ²r· ¶œ :Á²¶´ ®
" Œ ³=+ Œ ²Œ·µ5l¸Á ± lŒ V¾ ½:W¶  ¿… Á²¶ ²  l½²d´žµ¶‡/Wµr ‡ ¶µ³
¿À µ¶¶ g,j°(¢±-,k+  4 ¢²j²; ¶µ5/˜!²;  ˜~ ²·µ5¸˜µ5 5 ®à µ¢³¶5 + 4 µ25 2µ5½:²
, I+ p~0#102,55 !6¬0#10´±-,55  ® ¨Ž ¾ œ :Á²¶´5½ #  ³5³¶µ5   µ5³Œ!#10´±-,85Q6 ±O#102,85I+ ²Œ Á
\# :³¶%Wµ5 ®
 5 ³%<¾»  µ5³Q#10„@5T6 „[+ ²r Á
\#10#102.[5516 .G+ ( µ%5 %µ5½:² 8. ® " ¸ ³O# !,  %Q6
0#102,855 + ( µi5 Fµ5½:² G, ®°  r5 Fµ5½:² y( ž³¶¶ ¿S:³¶F~Áˆµ¶µ5½+ ‡Œµ5² ‡¶ \µ5½:Á:½:²5/ ³
 ³³¶µ5 + ²¶µ57¶µ5+ \# :³¶r4 ž³¶¶´¸³5µ-' ³ ®
¨ ²$ªa–d«c|—'1³ ®à ³\# :³¶¢³¶µ¶œ :Á²¶´+  IÇų¶5/ Á¸:³2µ5½:² ³^,–\ . 5² ³¢cª6),
²O«$6 #102.5 ® :  µ5³O#10«
5167#10#102.5516). ²=+#10ª[5q67# ! ,  % 6…0#102,855  ® :  µ5³

#10ªc39«
5q67# ! ,  39#102.5 % 6).\370#102,55  6 #10ª[5D39#10«
5
€  ² ½:7 ‡ µ5:³¶¶µ¶Á²¶  ¦#Vˆ/' ³ ´½²µ¶ ÄW\ V¾ ½:W¶   ° Á ± )…· ÁÇ¿ ®<Æ ¼g+ ²Œ ‡
½²µ5Á ± r³¶´¢³? ‡˜Wµ¶¶ ° µ5³¨³ˆ¾» 5%²5  (  ( µ?µ5 ´²µ¨$# :³¶? ½ˆµ5
³5µ ¤ ³ ®G€ µ=W + ³5X.# :³5r( ž³¶¶´Œ³¶µQ' ³ W+  ² ½:/y+ 4 µd5 ³rµ5½:² ³ – )„[4+ 
#10 3 5T67#10 5839#10 [5V#10 5W + ²% Á
\O# :³5%Á²µ5 ³5³5Œ³¶µ$'q³ ®
 

Ž ~½²µ5Á ± \³¶´ Æ ³ˆ¾» 5   µ5³Œ²Z5 ¸4 


4 µ/µ5 ´²µŒ!# :³¶/ ½ˆµ5
    
S
³5µ-'T³ ® ‹ = + ³5\O# :³¶¢Wµ5+ Áž¾ :³¶d Á
\# :³¶r ½ˆµ5
³¶µ-' ®
% ½:Á²µ5 ² ²=(+  g:³¶˜¿V Á² lŒ´½²µ¶ ÄW²µ
Œ ‡ ³5² ¿À Á²¶ O ½ˆµ5a³5 ¸³5 ¶ 
/ V¾ ½:W¶  ¿À Á²¶ ²  
¶‡ Œ:³¶¢ V¾ ²¶5½ ®

ËrÌ<€Ž’„‘’p{©

  -#j6”“•#10ª[5b–aª›|—'^˜ v# a€
 V¾ ²³5²Œ· :³%:³¢Wµ ®  4 ž³5 O6µ#10„@5 ®
 à µ Q,g6).!6 „[+ Q#10´±
5T6 #10
53 ^±V²+
 I´² Á:
S ³5³¶µ5¸ 6 ƒ „ ®
 à µ Q,g67#102.5+ ´ ² 
 < $6 #102,85D34,^±V² ®
" +
Á 4 µ¢5  $,&| -#y+
   

#102,516 T3)²$ ±™, · ¯²Æ ¼


° ² ‡Œ³¶ž ÄWdv# :³¶F½²5²µ¶½:/³¶µ -# ®¼ µ5:³55vˆŒ½²5²µ5r ž³FÁ: W³5³Á::³-ˆ‰'n5 
²¶²µ ® " ¾»µl:Š ³
l ²·µ5\lÁ ±  V¾ ½:W¶ .¿À Á²¶ ²  S
+ Á:² ‡ ³5²µ¸¿V Á²  ³¶?5 


µ5½:² Q½²% ‡l (½²µ5²Á:Œ


²Å~½² ½² ²5³d $# ®W° ¾ :³¶%Á:
/ V¾  ´ 7µ5 ´²µr5²W ®
 à µ‰.j6…„[y +  a´²#102,g± =5q±}#102,5-6¸#10
5 3 b,—±)²y+ 4 µ/5 c, |~' ®aà ³7 
 

 ²·µ5drµ5 5rrÁ:²¶5%µ5² ‡¶  :³¶¨%ÇŐµ5:³³¶  *) d²!, ²i Á: ³¶5N‘· ³5

ÂÆ
g6W
ƒ „@¼+4 Z² + K, c'-+
 ½:/ y  µ5³5 .½:Á²µ¶ g ‡W¶5½
 #102,j±
5q±4#102,85
¢µ5² ²  ³5³¶
5 5:³i :³ ´ž ²µ5³iµ5½:²  :³¨( ž³5³¶· :³ ®° ²‡ 
 ³i4²µ¶ ²FI¾»*) µ¶ ²µF%5 iµ5½:² y³ˆ¾ ½:Á²µ¶FÁ:  
‡l I½²µ5²Á:Œ¸²Å ½² ½² ²5³r  ® -#
  $,<|j' \ª–d«v| -#
®Q¼ <Çų¶5¸ Á  c,g6 ªN± « : =+ lŠ X· ¼v
5 ² ³r ®  µ5³ I¾»µ :³ ¯²Æ

#102,5167#10ªN± «
5E6 #10«
5D34ª «q39#10ª[5 ±V²
6 T37²- ±™ªU 39ªF«13 T3)²$ ± «b ±Œ²
6 -± ² 0ªN± «J5 
6 -± ² , 
‹  Wµ¶¶Á² ²µ=+³¶8,&| -#y+ ² Á: Wµr :³%²Å Ç Őµ5:³³¶ ³%\#102,5i¶µ5 ´½::³e+ <´²
$6² ®  W ² ²=+ ( µ%5 $,&|j'$<+  !#102,8516²-±  ,G ®


% ½:Á²µ5 ² ²=?+  ´½²µ¶ ÄW~³³l )Á² 5½  ‡ ¿À Á²¶ },)(° ²\±  ,  :³¶l·² 
³ ¶   µ5 · ²Š   ®

ËrÌ<€Ž’„‘’OŠ 

  Q#

³5 ¶  ½²´²¶²   ®
à µ5  K‚µ| + —#102‚—3W²e5  #10#102‚k55 ¸„ ²7 Á+<³5#102‚—3W²e5¸:³¶Œ²¶²µ
Ÿ ¨  
#102‚g37²e5 ² \# %[‚~°(E#102‚¦3µ²e5q± ² ® :  µ5³c#  :³¶d
¿À Á²¶   Ÿ ³ Ÿ +g5²  
-‚™| ‡ 
 ®yà ž³5 ³
+
 ( µ%5  Ÿ

#  02‚k53)²Q67#102‚›3)²e5Œ#10#102‚k5516µ#0#  02‚—±Œ²e5D3 ²e5q67#  0#  02‚—±V²e55k3 ²


 ²Á: µ5—# 02‚&3¸²e 5 Ÿ#  0#  02‚k55Ga– 4 µ5 _‚¸| Ÿ +%Á ® ˆ ®  ® #  :³¶ ³5³¶F ³5 ¶  

µ5 · ²Š   ® ‹  Wµ¶¶Á² ²µ=+# 02‚›37²e 5 µ¸² ²r ÁK#102‚3 Y5  ®
 ¸µ5½:Á²µ¶µ5²Á:7³³d ) Á² 5½Œ  ¶µ57  µ5³dŒ4 µ%5 ³˜²¶²µ5³Q‚1–  )„[+W  #102‚›3
 5   ® ‹  Wµ¶¶Á² ²µ=W+ 4 µ-‚<6 „[+  I´²Q#10  5   ( µ¢5 r²¶²µ  V„ ®
‹ ³¶¡<+  ·¶²_#10  3”²e5 ¸#10#10  55 ¸#10  5S+ Á: ¨µ5 ´# :³¶
³¶¶µ¶Á²5² ²
Á²µ5 ³³5l³¶µ Ÿ ® S Ž \µ5²²Š µ5l½² 5½Á: Œ  µ5³‰ˆ  3B² B#10  5   y+ Á:lg²¶µ
‘· ³\#10  2 5 :³5%²¶²µb¢¼ \#10  516  ®
% ½:Á²µ5 ² ²=+ V¾ ²¶5½:³¶¢·² 
³ ¶   µ5 · ²Š   ®

ËrÌ<€Ž’„‘’OŠ({
Ž O#
S¿… Á²¶    ¸:³¶%Á² ‡µ5² ²r¸³5 ¶  ®   +
˜³ ¶  ½²´²¶²   W¶µ5¸
‡¿À Á²¶     ®
 à µ 6)‚<6 [„ + Q#10#10„@55T6 #10#10„@55D39#10„@5+ <#10„@5167„
 <´² I¾ ! ®
 à µ 6 [„ + v#10#102‚k55 6µ#102‚k5+
 I´²d  µ5³ O‚ V„
4 µ%5 %²¶²µ $ ®
$#
° ² ‡ (²µ¶ ²ŒI¾»* ) µ¶ ²µ7 V¾ ²³5²·   N#µ·
 5 5:³
:³Œ:³ŒWµ ‘ ®  ® V¾ ²³5²· 
“•#10ª[5b–aª¦| ®˜ ¼ ‰#
Ÿ ˜:³¶F¿À µ¶ ½˜:³2( 5³FÄÅ:³¢ =+ ²  I²  ‡7µ5² ‡¶  µ5½:Á:½:²5¸  ¶µ5¸
5 l½² ½² ²l -# ‹ =+ =+ ª ¦#
:³¶p( ÄÅ ®  <µ5½:Á²µ5 ² ² a5 Œ( ÄÅ . ´½²µ¶ ÄW
ª67#10ª[5b– Kˆ -#
²dWµ¶¶²˜ Á ®
~–‚| -#y+ 1# 0 ¬3)‚k5_6 3 ‚ ‹

 à µ5 ³ Q V¾ ½:W¶  ¿À Á²¶ ²   ³ˆ¾ ½:Á²µ¶   µ5³ ! ® 
¡+ Ÿ3 ‚ ¦# ‹ =+ …‚ =+ lŠ

³5  #:³¶³5³5 p( ŒÄÅ  ®  Q³¶¨ ³
 *  µ5³ SI¾»µ :³Œ V¾ ½:W¶ 

¿À Á²¶ ²  

67#10 <516µ#10 ±~‚34‚k5167#10 ±~‚›39#102‚k5516µ#10#10 ¥±™‚k55839#102‚k5167#10 ±~‚k5834‚


! ! ! ! !

I¾ !&#10 ¥±™‚k516


! ±~‚q+ Á:
I  ¶µ5 ±~.‚ :³5r³5³¶I ( ¢ÄÅ
\# ®
ž­
=+ $#
à ³gµ5½:Á²³5½² ²  7´²?¨µ5 ´²µ?  :³¶g³5 ³²³5²· i l´i Ÿ ³¶· 
·
Wµ% ¶  ²2³5 ³¶¶µ Á²¶  …~   ²%<¾ ~µ5:³¶5¸:´:Á¸:³2²¶²µ5³%4 ž³¶¶ ¿À³2 \ ³ ® µ b¼ a€ =+
Š &
5² ³³ ³²³5²Œ· :³l \µ5 µ¶½²5½ Wµ¶¶Á²  ²µ5 QÁ:³5  :³7²³5²· :³7\ ‡~¿À µ¶ 
ª 6”“eªU‚1–[‚™| @˜ + <ª
Ÿ Ÿ < ! \:³¶¢ ²¶²µ ®
:
 ²¶5 ³Á:²¶5 * ) µ¶¶  ( µŒ    ² ®Q¼ ¨Çų¶5  Á ²¶²µ 5² ¨ _ª)6¥ƒ „ +
( µŒ5 7²¶²µ _‚ „[+
 <   ·µ5 j#102‚k5 ªG+
/³5 Œ Œ ¶ ~ S²5  ¶ \ O:³¶ ¦ª
 4 
Äŝ ®<à µ  ¦# ™| “e„ –=²– –lª¦±7²®˜@+
S p( ž³5  ¦#10 Y5‰6 ‚ ª ‹
®  Wµ¶¶Á² ²µ g   =+
‚ N6…„ ‰‚Œ| Ž N‚ \ª ‰‚46 ª v3 [+ /|

®   Ÿ ® S ´³5 .²Á² ²  Wµ ³ˆ¾ ½:Á²µ¶  I:´:Á  Ÿ ²
|&“e„ –=²– –lªN±V²®˜ G€ K

  ®   
 
 

5
µ ³
#102‚k516µ#10ª $3 @5q67#10 $39#10ª 5516µ#10#10 Y5583V#10ª Y5167#10 @583}ª \6…02‚ 3 5Fª
    

Á:d4 ³5³¶µ5rQ# :³¶¨²¶²Š µ5² ²%½²5²µ¶½:˜Wµi³5:³¨´  ²µ5³F³¶µ$“@²– •–lª$±h²®˜@+Á ® ˆ ®  ® Wµ


:³rÁ ±  Å~¿À5³%4 µ¢ :³%²¶²µ5³O‚  – =–‚ ¶  ®



% ½:Á²µ5 ² ²=ˆ+ 5 5  ½: I¾ ²¶²µ ª V¢„ ²SI¾ ²¶²µ5³<( ž³¶¶ ¿À³< ¸ ³‚  – •–‚  ¶ 
(²µ¶ ²~ Á: ³¶¶µ¶µ5  ³ ¶  *µ5 · ²Š  O²*( ž³g#10„@5—6*„[+^#1 0 @5j6 ‚ ª ( µ


—|Œ“e„ –=²– ®–lª!± ²®˜Y< – ²‰#102‚k5Q6 0‚ 3 Y5 ª ! ²  ³5 ¸µ5:³¶(:Á²¶´² ²


l ¶²Œ²/ 
µ5:³55¸³% ‡l´³¶  ²Á² ²ŒcZ ‚ WµQª ®
  

‹ * I²=+ ³ Á::³Á: ¶ ³=¨+ 4 µ\5 ³²¶²µ5³ ~–‚  „[¨+ #( ž³5 :³\´³5 ³²
²~—‚ Wµ›ª7T‚ 6*ª _3  ² 6 ª¦3  ® :  µ5³¦#10 3

Á² ²:³ µ5:³¶(:Á²¶´:³ 
#102‚k55-6¸#N0 Gªc3 ^3702‚ 3 Y5Fª 5O6 02‚ 3 -3 N34‚  5Fª ® ‹ =4+ I¾»¶µ5lWµ¶‰#10#10 <553 #102‚k5-6
  !

#_0l02‚  3 Y5 ª 5©3 0‚ 3 Y5 ª 6E02‚  3 Y5Fª›3 0‚ 3 Y5 ª ®F°  r  ¶µ5 /# :³¶\·²#
    !

³ ¶   µ5 · Š²  ®


   

Ž :³
³5 ¶ ³.µ5 · ²Š   ³5 Œ Á ‡ ¿À Á²¶ .  ²
5 5:³
:³/¿À Á²¶ ³ŒZ (
<
Á² :³³¶³ ®
à µN²; ¶µ5 Á:  ²=S+  ?µ5:³55~ˆ µ5 ´²µŒ V¾»*) µ¶¶  ¿V5 ²pÁ: µ5³Œ µ5 5 ®S¼ :³¶ŒÁ² ‡µ
%5 F²³5²Œ· ˜% ‡
¿À µ¶ Oª Ÿ :³¶F³¶· %Wµ2 ¶ ~²F³5 ³¶¶µ Á²¶  ® % ½:Á²µ5 ² ²=+
³  Ÿ   ´
²%³5· ¸Wµr ¶  ²%³5 ³5¶µ Á²¶ 
F W6”“e„F˜@y +   W67„ Ÿ ®
F D +  (²%Á: ³¶½²µ5²µQ\ª / ³¢(²¶r½² ½² ²d   Q ®à ³5 :³¶%³¶· 
Wµ¢ ¶ D +  Kª Ÿ   ®  ^‚~|  ®  ©‚<6 ª T3 [+ ‡´³¶  ²Á² ²¸O‚ Wµ©ª ®
à ³5 :³¶%³¶· 
Wµ%³5 ³¶¶µ Á²¶  ²%c. ‚ ²$ª lWµ¶¶²²cˆ +  ² ½:r


!|  ® Q Ž l 5½K\ª ( ž³5Œ  µ5³d X67„[W+ Á ® ˆ ®  ® c‚ ³5 %  ¶ \ª ®à µ


³55Vª Ÿ ®
‹ d  µ5³=+ …6 ª Ÿ <+ Á:  Œ Á:½ ®
ËrÌ<€Ž’„‘’OŠWŠ

‚   N#³ ¶ p½²´²¶²   ® 1€ .µ(²  <¾ l¿… Á²¶  ³¶¶µ¶Á²5² ²7   5 

:³5¢ :Á²¶´ ®
 à µ Q,g6).!6 „[+ Q#10#10„@55T67#10„@5F²r ÁK#10„@5q67„ ®
 I´²
 à µ Q,g6 „[+
    µ5³ O#10#102.[5516 .G+( µ%5 %µ5½:² 8. ®
 ²c,–. :³˜µ5½:² ³=+4²cªg6#102.5 ® " ¾»µlŠ:³˜ ‡µ5² ‡¶ Zµ5½:Á:½:²5+I Z Á_#10ª 5-6”. ®
à µŒ³¶5!#102,›3Œ.5Q6‘#102,3)#10ª 55c6 #102,5y3 ª<6 #102,5y3)#102.[5 ®D€ pÁ: ³¶5!# :³¶¸
¿À Á²¶     5 I´½²µ¶ ÄW¸ V¾ ½:W¶   ° Á ± ®€  ³%<¾»  µ5³Q# :³¶¢ ½ˆµ5 ®
% ½:Á²µ5 ² ²=I+  :³¶/¿V Á² l´½²µ¶ ÄW²µŒl :³Œ³5² :³
³5 ¶ ³¸ ½ˆµ5:³Zµ5 · ²Š  
³ $,j°( , ²$,j°(E±-, ®
   n$ ‚ µ¢²  ²¶²µ ®Ã µF V¾»·³¶µ5K³54 ž³5 ³i<¾  yÇŐ³¶5%¢¿À Á²¶ ¦~# ´½²µ¶ ÄW
:³rÁ: ¶ ³r¸ V¾ ½² Á:½ ®(°  Œ Y¼+aO # :³¶r  µ5³¢¶œ :Á²¶´
²% I´²v#10„@5q6 „ ²
#10#102.[5516 .  · ¯ˆ­ ¼
žÈ
D.
( µ¢5 %µ5½:² ®
D‚ : Q#10#10´±\²e5516²v67#10#10´²e55+
F Z:³¶%Wµ ®  µ5³ œ K#
Á:
<Á: ¶µ5:r V¾  ¶:Á²¶´5½
 ®

7 Ž g. '-+
:³¶Wµ ® < µ5³ —.
½:Á²µ¶ ¢   ·µ5 pµ5² 5 5:³  :³´  ²µ5³µ5½:²  :³
Ž ™· ¼ $# q#
( ž³5³¶· :³ ® Sµ5² ‡¶  ¯ˆ­ ¨  ¶µ5/<¾»  µ5³ œ + c#
:³¶2³¶µ ¶:Á²¶´ ²2 Á/ œ :³5F³¶µ ¶:Á²¶´
³³¶ ®
^,–.
 ² " lŠ =+ Q‚
:³Fµ5½:² ³ ® ¾»µ :³2Á:dd V¾  ´²¢d´ µ ²2³5 =+
:³¶FWµ  4Çų¶5
G« O« 6µ#102.[5 : =+ K#10#10«
55167« 67#102.5 =+ œ +
 Á%¢µ5½:² F5² W  ®  µ5³    a² žWµ¨ :Á²¶´5½
#10«
516). 

®à µd³¶5

#102,5839#102.5167#102,583V« 67#102,_3V#10«J55T6 #102,_34.5




°    Y¼+Q  ² ½:7¦# :³¶Œ ½ˆµ5Q + Á ® ˆ ®  ® <¾  iÇų¶5 µ5½:²  5² ¨+<( µ
5 rµ5½:² 8,k+  Q#102,8516 , ®
ŽS µ5² ‡¶ Œ· ¯ˆ­ ¼4+ ( µQ.j6¥7² ²˜( µv.j6 (+ Á: 
  µ5³‰ˆ \6¥7² ² K6 ® " Á
6 ²-‚<6²W+ Á:¸S:³5¢ ‡lÁ: ¶µ Á²¶ OÁ ± ²µ5Á ± ½: ®



ËrÌ<€Ž’„‘’OŠ•

Q#
  
³5 ¶  ²½ ´²¶²   ®
‹
  ( ž³ $,j6±$ª lŠ
² rµ :³% ¶ Áˆ¶  Wµ OªG+  ·¶²%4 µr5 %µ5½:² ª&6 ƒ „j
#10ª[5 ±™ª # ±\ª ² 6 ª 


· ¯ˆÈ ¼
 ‹  4 ž³ O,h6  +W O² ½:/+y( µr5 cª™6” ƒ „[+G# !  %O±  #10´±Oª 5©6    +W²/ Á
ª # !  % 5y±™#10´±$ª[516  ®à µ˜³¶5
±$ª # ± ª² ± #10ª[516± ª²


· ¯šÉ ¼
à µŒ³ ³¶¶µ Á²¶ Z ²Œ·µ5ˆ  ²·µ5 —· ¯ˆÈ ˜ ¼ ²!· ¯šÉ ¼<+ Z².½:
(+ ( µ¸5 ¸µ5½:²
ª&6 ƒ „—
#10ª[516 ² ª  3 ª² 6 ª 3 ª ²



% ½:Á²µ5 ² ²=W+  a:³¶¢¿V Á² Œ´½²µ¶ ÄW²µ˜


‡¿… Á²¶ &,<°(   ³  :³¶d·² ³5 ¶ O
µ5 · ²Š   ® 

ËrÌ<€Ž’„‘’OŠ¦
Q#   †

³5 ¶  ½²´²¶²   ®4°  ² : ³dWµr² :³%:³5³ ³¢ ½²µ¶:³ K
Q,g6 „
 Ã µ -.!6²+
 ² v#  6)„„
 I´² Q )
6
!% ®
Q,g6²
 à µ -.!6 „[+

² v# ! %
Q6 „
 I´² ®
Q,g6 „
 Ã µ -.!6 +
 ² v#
  I´² !  % ®
Q,g6
 à µ $.!6 „[+
 ² v#
 I´² 6 „
!% ®
Q,g6²
 Ã µ -.!6 +

²  v#
 I´² 6 „
!% ®
Q,g6
 Ã µ $.!6²+
 ² v#
 I´² 6 „
! % ®
) Q#
F  ®¼ 4³5²Œ· r·² ¢ :³i Œ·µ5:³2% ‡¸¿… µ¶   ³5 ²iÁ: 5 µ¶W· :³%²F ³5 
Nˆ =+
  Œ³¶µ˜Á::³% ·µ5:³ ® Q V¾  µ¶µ¶´ µ5 ´²µ/Á:
²µ¶²µd(  W ‡ Á: Á² ³¶  Œ³²µl ³
<¾ Œ:³¶¶  ŒÁ: ¶5½l³5
V¾ ²³5²· :³d Œ·µ5:³/
‡¿… µ¶   !
:³5%²³5
³! „ –=²  ®
) &ª–‚ | Ÿ

 ‚ „
 µ5½:Á²µ¶µ5²Á: ³ˆ¾ ( ž³5 ®   + ›ªW|
 .n²¶²µ\ÄŽ ® 4 ž³5 ³  ( µ5 
“e„ –=²– ®– ˜@+ y
c# ) \6W„ 8€
Z
 
 
  !  % ®  (
 Q‚ 6W„&· lŠ ¼¼
²˜ 5²µ˜ŒÁ::Á²?:³¶d´µg4 µ …I¾»µ :³˜ ®
‰,–. 6“ ) –aªg|™“e„ –=²– ®– a˜˜@+
 
¼ <:³¶¢¿V Á² 
Œ´½²µ¶ ÄW²µd¸  µ5³5 ½:Á²µ¶´²˜ V¾ ²³5²·      

ÂÉ
 ³
d Œ·µ5  ½:Á²µ¶¢ 4 V¾ ²³5²·     ³ ”6 “ )   –ªg|h“e„ –=²– •– ³  ˜˜ ® " d¼¨²¢I¾»µlŠ:³
Š + \# ! )   % 6 „l4 µr5  )   |  ³  ®
  
V¾ ± (  ± :³5
/µ5½:Á²µ¶µ5²Á: y  ² ½:r  
°  ¸µ5½²´D+ ‡7¿… Á²¶ &# :³5%  
³5µ6”“  ) –aª–‚™| Ÿ ²$„ Œª  ˜ ®€ µ=+) :³¶


²³¸³  „ –=² g²O#O:³5%Á: ¶³¶µ  


„ =² oy+  ÁK# :³¶¢ ‡7¿À Á²¶     ®


% ½:Á²µ5 ² ²=+ ‡¿À Á²¶    :³¶¢¶µ¶´‡ ² ²d³5 ¶   µ5 · ²Š   ®
ËrÌ<€Ž’„‘’OŠW”


à µ% V¾»·³5µ5 ³¶( ž³5 ³%<¾  <ÇŐ³55//¿… Á²¶  ´½²µ¶ Äyr :³rÁ: ¶ ³d/ V¾ ½² Á:½ ®
 à µ Q,g6 „[+ X  I´²
#10#102.5516±-.‰39#10„@5 · ¼
¯
@
ŽQl¿À Á²¶ &# -#.:³¶d Á7*) Œ4²5±\² ® ‹  Wµ¶¶Á² ²µ=+(Áž¾ :³¶d
· œ¶:Á²¶   £ ³¶µ
£ ®  ³r  µ5³Q# :³5r²  ÇV!²;  ¸¸· ¶œ :Á²¶   £ ³5µ £ ®
à µn.6 „ – ‡/µ5² ‡¶ &· ¯ a ¼ Á: ^ˆ‰#10#10„@55q67#10„@5a²F³¶k·‘Wµ œ¶:Á²¶´5½e¼q#10„@5q6 „ ®
à µrÁ: ³5½:²=y + I¾»µl:Š ³X· ¯ ¼+  #10#102.[5516±$. ( µ%5 r²¶²µO. ®

@

@

 ²©,–
. ²Å ²¶²µ5³ ®à ³5Q~# :³¶ r· œ :Á²¶ D+  yÇų¶5¢²¶²µ©7ª 5² WO#10ª[5q6
. ®¼ I´²d  µ5³O#102.5167#10#10ª 55T6±$ª ® : ³¶q
#102,34.5167#102,_39#10ª[55q67#102,85 ±™ª67#102,5839#102.5
€  µ5:Á: W /  \ˆ V¾ ½:W¶ Z ° Á ± ®®®€ Z³/<¾»  µ5³cp# :³¶˜ ½ˆµ57²/ Á4+ 4 µ˜5 
²¶²µc,I – Og#102,85-6W,G#10´²e5 ® " ¾»¶µ57Wµ¶v,y0#10´²e55 K6…#10#102,55O6¥±-, ® " ¾ !}0#10´²e55 N6¥±\²+
Á:\i:³¶Œ( ž³5³¶· ( µN#10´²e
5 ²¶²µ ®?° ¾ :³¶Œ ‡ Á: ¶µ Á²¶  Á ± ²µ5Á ± ½: ® SŽ  Á: Á² ³¶  ²
½:Á:   ®

ËrÌ<€Ž’„‘’OŠÊ


¼ 4:³¶iÁ² ‡µFd V¾ ²¶5½/:³¶ir³5 ¶ ~µ5 · ²  ®à µ5 ´ ³¢rÁž¾ :³¶i ‡
³5²  ®  
# " Š
³5 ¶  ½²´²¶²   ® l ‡~µ5² ²µ5Á: ¶ p²
Wµ¸lµ5½:Á²µ¶µ5²Á:³ ³
 )Á² 5½ +
+ $,&| ³ ‡
 ½:˜ ( µ¢5  $‚™| + #102,_34‚k5167#102,85834‚
¤ ²%5  Ÿ <   ®
Xª–d«›| ‡ €
 ² 39« 6  3 «^3 ª
Ÿ ®   !   % !% + ®<à µŒ³¶5 <²Z¶ ³Œ ‡~²Å ²  Š
D+ K
Á: ¶  W I´²  

# ª« 39«   6 =# ª« 39«   6 •# ª« 39«   6 e# ª«   39«  3 «J# ª« 




·¯* ¼
" ¾»¶µ5
Wµ¶

# ª 93 «   6µ# ª   39«  3 ª 6µ# ª   3h«  3 ª!6 e# ª #  3h«  3 ª…· º - ¼




«  « 


«  «
" !· ¯ * ¼¢²X· º - ¼+  ½:% ½:‡5² ²/\«J# !  %v6 ªG+Á ® ˆ ®  ® # !  %Q6  ®
° :Á²(½²%´µ(² ³2/³5 ²¢ :³¢²¶²µ5³%³5¶µ¶Á²5² ²¢( ž³¶¶ ¿…³^ª\²-«®+Á:² ‡l²¶µ¸
#102,8516), ( µ¢5 $,h| ¤ ³ ‡ ®
ËrÌ<€Ž’„‘’OŠ4¥

  Q#

³5 ¶  ½²´²¶²   ®
Q,g6).!6 „[+ Q#10„@5T6 „ ®
 à µ  I´²
ŽS~ 5  ½:\³
V¾ ½² Á:½´5¦ˆ\¿Àµ5Wµ¶µ55²  W¶5½ ®   Á
.g|j' ®Sà µ%5 %µ5½:² , 6 ƒ „[+I¾»µlŠ:³r V¾ ½:W¶  ¿… Á²¶ ²  +y  g
#102,_3}.[5 ± #102.5 6 #102,85 3 ®.
, ,
Â9@
‹ p¿V³5²µ5›, ´²µ5³X„[Q+  ¨´²       ³  ¶   6²Q3 ®. ®?° ² ‡ ³¶ž ÄW   ‡
¿À Á²¶ ›#~:³¶ ½²µ¶´ž· r².G+ž²¨$# + 02.[5q6…²3 ®. ®° 5ÇV5²~O#10„@5q6 „[+  ²½:
+( µ%5 -.G+#102.[516).  34. ®
% ½:Á²µ5 ² ²=+  :³¶?¿V Á² %2´½²µ¶ ÄW²µF2 ‡˜¿À Á²¶ !,j(*
° ,a3j,:³¶?·²\%³5 ¶ 
 µ5 · ²Š   ®

ËrÌ<€Ž’„‘’OŠ 
· ¼ ‹
   ¢³5 ¶ ½²´²¶²  d¶µ5¢2 2(  ,+  2 …:³5g·²¢³ ¶  ® 
µ5²Wd³¶Á:Á::³³¶´² ²d4 µ -, K²+ + +
:³¢´ž ²µ5³   ²   ·¶² 
Œ0 Y5q6 Œ0 5T6 0 Y5T6

0´² Y5T6 „
  
®
6¥“ F– – F–=² U˜ \ª}6…ƒ „ \ª | _ª +q NHGIbI


   
 ®   ~²  ® 4 ž³5 ³¸ ³5 /7µ Á²
Á:  ÇÅ
  ®
¡+
¸ <( µŒ5 7²¶²µ   „[+
Q    ª | ›+ Nª 6
 Q  µ5³ ƒ ² +

 g²Œ V¾ ½:W¶ p¿À Á²¶ ²  
ª ‹ =+
  ¶µ5Œ ~:³¶r³5³¶I¸µ Á²Œ  ®   I²  V¾ ½:W¶  ¿… Á²¶ ²  ½Çij5³r
½² 5½7²¶µ5l²Å (  , v'$+  ‹
 +  :³d´ž ‡· 7³¶µd5  g²  :³¶¸³5³¶Q´  ‡· 7³¶µ ®  µ5½²4½²
=+ + V„[+ ®ª
Á:/µ³ ² ² W  µ5 ´³¶S 4 µ¢5 %²¶²µ    /µ5½:² :³¶¢/µ Á²

+ ‹
 4Á:g¿… µ¶¸lÄ5½µ Á²:³Œ³¶¶Á²5:³
  ® 
 Á  :³¶¸ 7(  ,  +  l ®
° ¶µ Á²¶  ®
74¾  %ÇŐ³¶5   Ÿ„[+
?( µ :² ®ª7| ›+ !ª !„ ª
   µ5³ :³¶l µ5½:² %²   Z²  Á
„  W² " =+ lŠ +
   ® ¸ ³ WI¾»µ :³˜ V¾ ½:W¶  ¿À Á²¶ ²   4  ½:/  :³¶/³5³¶<Œµ Á²
 ‰ˆ
‹ =+
  ®  µ5½²4½²%Á:rµ³5 ² ²   ·¶²¢dÁ: ¶µ Á²¶  ³¶ ‡µ5 Œ ‡
µ5½:Á:½:²5 ®
à µ2³¶5+ :³iµ Á²:³2 fWµ¶¶²²¢5 5:³^ˆ “e„F¸ ˜ ²¨5 2½² ½² ²2  :³5¨
®€  ² ½:/<¾  aÇų¶5
 µ5½:²  6B ƒ „ ²d:³˜²¶²µ5³  „[y+ ²  – k– e– $²

µ Á²7
5² ³r+( µ%5 %µ5½:² 8,}


 02,5T6  , 802,¦± Y5
802,›±  502,¦± Y
5 02,g±V² Y
5



Ž ¾ ½:W¶  ¿… Á²¶ ²  ³ˆ¾ ½:Á²µ¶r  µ5³K


  ³
³  ³ ³ ,  02,›±Œ²e5
02,g± Y5  02,›±  5 02,¦± Y5 02,¦±Œ² Y5


6 ²  02,¦±V²e5, D02,¦± Y5
02,¦±  5 02,¦± Y
5 02,¦±V² Y
5




‹  ²¶ Äyr ¸Á:  )Á²²˜ W=+  I´² 3  3  3 - 3 6  + Á ® ˆ ®  ® 6 „ –  6


— 6 ‰6 6² ® " ¾ ! ›02,5q6  02,g± Y5J02,¦±  5J02,¦± Y5J02,j±9² Y5 ®
% ½:Á²µ5 ² ²=+ \´½²µ¶ ÄW%¿V Á² ² ²2%5 i(  ,+  d% ‡¸¿… µ¶ dÁ² :³5³¶³¨:³5¨·²



Œ³5 ¶   µ5 · ²Š   ®

ËrÌ<€Ž’„‘’OŠ©

€ \µ(²  /+4 µF5 2µ5½:² , V„[+  K,c3   ›·‘³ ,›±      V„@¼ ®  
, V„ ®€ \( ž³5Q.6  ³   + Á ® ˆ ®  ® ,g6 .$3  .  ±Vd² ³5, µ˜² ²©,j6).‰±  .  ±V/² ³¶G, µ² ®€ 
 6).\3  .  ±V² ®
¶ ¶ 
(²¢ 5²µr
 % ‡¿… Á²¶  <% q°( 
 # 83  ±V²  ½ÇÄŒ³¶µ!¨² b3
  ® :  µ5³%I¾»µl:Š ³dÁ² :³5³¶³\
FD , µ²W+  g  ,_3   67# J.\3  .  ±V²/67#102,85




F „ 9, µ²+  g


 ,_3  67#
.‰3  .  ±9² j6µ# ² 67#102,5



, 

 *
‹ +
r Á  ‡¿À Á²¶  ³5SÁ: ³¶¶µ¶5¸µ5½²(   µ5 · ²  ® Š
ËrÌ<€Ž’„‘’ • 

‚   Q#
/¿À Á²¶  ´½²µ¶ Äyd :³%Á: ¶ ³d/ V¾ ½² Á:½ ®
 à µ  Q, 6 ,  67„[+ Q#10„@5q6 „
 I´² ®
 € +
 µ5 ´O¿V Á² ² ² Wµ µ5½:Á²µ¶µ5²Á:p ¢( µ 5  ²¶²µ <‚  ²+%² ( µ 5 ³
,  – ,  – e–, |  „ =– ² 


g5² ³%  c, 34,  3 34, µ²+ g
    


#02,  34,  3 ®34, 5Œ#_02,  5839#02,  5D3 39#_0, 5







‹  Wµ¶¶Á² ²µ=+ ( µ¢5 $,&| „ – ,  + I´²




#102‚8,5 9‚#102,85 · º¯ ¼


a,     µ5³O²v67#10´²e5167#10´²±›,O3j,8 5 Œ#102,5U3/#10´²k± ,85 Œ#102, 5 ® " ¾ !g #102,5µ²  ®, ®
F

„ 9,    µ5³% (Çų¶5/ ²¶²µ$$‚ µ˜² 5² < 9,  Á ® ˆ ®  ® V‚8,µ² ® " 

· º¯ ¢¼ ²% Áˆ ³¢µ5 ½:Á:½:²=y+  ½:rc‚#102,5 Œ#102‚8,85   ®‚8, I¾ !&#102,5  ®, ®
 

: ³¶¡+ ( µ¢5 $,h|  „ –=² oy+  #102,85  ®, ®


  €  ´žŒµ5 ´²µd˜ ‡7Á: ³¶5 7d(²o²; ¶µ5¸µ5² ‡ Á:½:¸Wµ2
¶µ5/ ³¢(²¶5 ®
à µ Á:² ‡ + 7Á: ³5[²Š µ52 ‡¢¿À Á²¶   # ½ÇÄ¢³¶µ  „ –=²   Wµq#102,516 /„ ³¶ „ 9,   + ²1#102,8516²
³5  9
 €  , ´½²µµ¶ ÄW² /® ¿V Á² ² ²d\# ³¶³¿V¢ :³rÁ: ¶ ³d µ5 · Š²  ®
   |  „ –  ® :  µ5³=S +  iÇų¶5  ²¶²µN‚ Ÿ² 5² F   _±   ®    ² 

  ‚¦± ® :  µ5³   3    + Á ® ˆ ®  ® ! ‰±  %  3  #µ² ®à ž³5 ³-,O6  3  ®€  l Á





·²/,,c|  „ –=² o + ³v#102,,=516²   ,,+ Á:¸S ³5³¶µ5¸ ‡lÁ: Á² ³¶  ®
      

ËrÌ<€Ž’„‘’ •I{

}#
4 ž³5 ³l<¾ 5²  \¿À Á²¶  #ÇŐ³55 ® " ¾»µlŠ:³7 V¾ ½:W¶  ¿À Á²¶ ²  +a4 µ75 
|y+
 †
W² µ5² ‡ ˆ Wµ   Q3}ª– K
 I´²

#10  Q3}ª[5D39# !   v3 qªc39ª %v6 G0  ‰3}ª 5 · ºžº ¼


ŽSl!;² 
³¶·³5¶¶¶  Á: ˜  µ5³Oˆg
# !  v3 qªc3}ª %T39# ! Q3   ª\3 qªc39ª %v6  !  c3 qªc39ª %
Á ® ˆ ®  ® +³5  6…
² ² µ6ƒ ²!
# !   v3 qªc39ª % 39#10 U5q6  !   v3 qª‰3}ª % · ºžÂ ¼
" !· ºžº ¼+k· ºžÂ 2¼ ²r/ V¾ ½:W¶  ¿… Á²¶ ²  y+  ½:d  µ5³d
#10 F516 ² !  !   v3 qª\34ª % ± ‰ 0  v3}ª[583 G0 U5 %
Á:¸Iµ5 ´
V¾ Á²5½ ®
¼ ?³¶ ) ¸5²W7´½²µ¶ ÄW²µŒ7 ‡\¿… Á²¶  #
½ÇÄWµ¸ ‡¿À µ¶Œ µ5½:Á:½:²5 :³¶
Š +
·² ¸³5 ¶   µ5 · ²  yÁ:¸</( ž³5/W ³%Œ )Á² 5½ ®

ËrÌ<€Ž’„‘’ •yŠ

Q#
  
³5 ¶  ½²´²¶²   ®
 à µ Q,g6).!6 „[+  I´² Q#10„@5T6 „ ®
"-
Æ
 à µQ,g6 „[+  +
½:˜ ( µ¢5 %µ5½:² D.h
#N0. 5T6…0#102.[55
  · ºÆ ¼
 -ª V„7~µ5½:² ®¼ (ÇŐ³55d~µ5½:² . 5² Ivª6). +²¢ Á+4 µ25 ¢µ5½:² , I¾»µlŠ:³K· ºÆ ¼
 —


#102,3}ª[5167#102,58370#102.55 67#102,5839#10ª[5  · ºž­ ¼


‹ pWµ¶¶Á² ²µ=+Q4 µX,96±$ªS+  ½:l# :³¶
Wµ5 ®  ²N, |V' ²Xª4|V'q³ ®
:  µ5³-#102,±/ª[5q6±O#10´±-,\3 ª 516±X0#10´±-,53 #10ª[55167#102,5D±&#10ª[5 ®° 5ÇV5² · ºž­ ¼+ 
² ½:˜+( µ¢5 ³%µ5½:² ³$, ²OªG +  #102,_39ª 5167#102,85839#10ª[5 ®
° ¾ :³¶¨²Á: µ5% V¾ ½:W¶  ° Á ± ®®®®€ ³i<¾»  µ5³=+ 4 µ 5 ¨µ5½:² , ²¨5  µ¶ ²
[+  #102, Y516 #102,8F 5 ²=+ ² Wµ¶¶Á² ²µv#1 0 @516 Y#10´²e5 ®
 ³l V¾ ½:W¶  ¿À Á²¶ ²    ~ ³ I¾ ¿À µ¶¶ # V¾ ½:W¶ # ° Á ± Y¨ + ²
Áž¾ :³5³¶%~ V¾ #´  µ5½:³5 µ5 ~µ5 · ²Š   ®i¼ %³š¾»ž  Á I¾ ÇŐ  5²µ ‡ µ5½:³5²Á: ~ ‡
³³Á:
³¢ V¾ ½:W¶  ¶‡  ®
 ² _| ¤ ²O,<|j' ® €    ÁK#_0 0 $3},5 516…0#1 0 @5839#102,55 ®€ µ\  

#N0d 0 $3},5 516µ#     , ¶ 6     # =, ¶ 



 










²\
0#1 0 Y583V#102,55 6    0#1 0 @55 0#102,855 ¶ 6     0#10´²e55 0#102,855 ¶











I¾ !


 #
, ¶ /6  0#10´²e55 0#102,855 ¶



 
 
 
 

° ³¶½²µj, ÄŽ+F :³ ²Å*4  ,+  :³~·…² @¼Á² :³5³¶³~Á: $ÀÁ²² ³5µ 5 ³ :³ µŽ
¶ ² ³e+<²Œ Á  ³Œ³5 7½²ÅZ² .4²Œ²¶ ÄW²µŒ :³Á:  )Á²²5³ ® ‹ pWµ¶¶Á² ²µ=+<( µ
 6 ‚—± l²%( µ  6)‚—±9²+ <´²X
# ! ,  % 6…0#102,855  0#10´²e55 ¶  ² #102,5167#102,85J0#10´²e55 ¶ 
 · ºžÈ ¼ 

( µl5 , ®aà µ!, 6*²? +  ² ½: j#10´²e5!6 Z„ V#10´²e5!6*² Da+ ³¶^‚ :³¶lWµ=+
#10´²e516…±\² ®
Fk#10´²e516 „[ + · ºžÈ ¼+  I´²Q#102,516  „ 4 µ¢5 %µ5½:² , ®
% ½:Á²µ5 ² ²=+ ‡¿À Á²¶    :³¶¢·² ¸³5 ¶  ®
˜n#10´²e5‰6 ²< + /·  /¼ ²¸³55 
µ5½:² ¨4 ž³¶¶ ¿¢:³¶¸pÁˆµ¶µ5½S+  ?´²X# „ ³¶µ\'q³ ®
 ²d  µ5³$,k–. :³rµ5½:² ³¢5² ³%‰„ 9, 9. ® :  µ5³-,G 9.F + ²r³5q
„  # !¡.  ±~,  %O67# !¡.  %$± # !¡,  %Q6¸0#102.[5 ± #102,855J0#102.[539#102,55
‹ /  µ5³‰#102. 5 µ#102,54+ Á:a ³³¶µ5_.# :³¶˜Á²µ5 ³5³5l³¶µv'T³ ®I°  7²  7:³¶˜Wµ5+
²  /:³¶¢ Á/Á²µ5 ³5³5¸³¶µ^' ®€ µ= +  7½ œlˆ ´ <¾ ˜³ ¶     5 ¸d V¾ ½:W¶  
° Á ± Œ:³5S ½ˆµ5 ®ž°  ¨Á² #10´²e516²š+ Áž¾ :³¶g ÁiT # :³5S V¾ ²¶5½F¡ˆ+ µ5½:Á²µ5 ² ²=+
:³5¢·² 
³5 ¶  ®
 G‚ :³¶FWµ¢²F³5#10´²e5q6…±\²+  ( ž³5 6±O# ® Q Ž 
¿À Á²¶  ´½²µ¶ ÄW/  µ5³F V¾ ½:W¶ 
¿À Á²¶ ²  d¶‡ d²G0´²e5T6² ® < Ž dÁˆ ³¨µ5½:Á:½:²F ³id :³¶¨ V¾ ²¶5½d²2 Á
\# :³¶r½ÇĸWµv#102,516±-, 4 µ¢5 rµ5½:² D, ®

Ư
% =+ Š
½:Á²µ5 ² ² WÁ:²¶5
²µ¶ ²µ5
:³5¢·² 
³5 ¶   µ5 · ²  ® Š
 =+ +
 W ² ²  :³?³5 ¶ ³¨³5 ? ‡r¿À Á²¶     V¾ ²¶5½r² ž³a ¢Áˆ ³? ! =+ ‚ :³¶aWµ =+
‡¿À Á²¶  —,—°(E±-,
®

ËrÌ<€Ž’„‘’ •W•

 Q# 
³5 ¶  ½²´²¶²   ®
€
  Á:  ²Á:¸Wµ¢µ5²µ5²µd<¾»  µ5³ O# ! =+ D#
:³¶%Á²µ5 ³³5 ² I² ³¶ :³¶%½:Á²µ5 ³5³5
  µ5³/ ‡ ¿À Á²¶  # O# O#/± # O#›3 #
 "   :³¶/½² ² ²Œ½:Á²µ5 ³³5 I²¸ Á774² ²¶µ5 + ‰;
½²  7 ‡7¿… Á²¶  Xˆ  /,g°( ,!3 Š
7³5
Á:²¶5
²µ¶ ²µ5
:³5%³¶¶µ¶Á²5² ²˜Á²µ5 ³5³5 ®
vŽ ¾ ½:W¶ p¿À Á²¶ ²   4 µ¶7³5µŒ :³5½²µ5½::³7¦#y+I 2:³¶
W¶µ5² ¢Á: ³¶½²µ5²µ7 ‡
³55&02, 5½ÇÄ WµX,g6Ÿ,  µ5½:² F ½~²=+g4 µ75 l²¶²µ‚ ¥„[+ , ³  6¬#02, 5 ®
:  µ5³\   

, ³ ± ®, ³ 3 ®, ³  6  , 3
 

° ¾ :³¶Œ\µ5² ‡¶  \µ5½:Á²µ¶µ5²Á: ½ˆµ5\I¾ µ5µ5 +yˆ Á:  )Á²²5³Á: ³¶5³=S + ³ F


   

 p5²µ¶  Á: ³¶¦‘·  Y¼ ® d<¾ ˆ Á:² ‡ ¶²S + . V¾ ½²  ².( ž³=<+ 4 µ¸5 Œ²¶²µ


‚$Œ„[+ . 6), ±~‚ ®¼ <:³¶2¿V Á² ¸¸´½²µ¶ ÄW²µr<¾»  µ5³=+ ( µ¢5 r²¶²µQ$‚ V„j
 


. ³ ± ®. ³ 3 ®. ³  ±  . 6 „



 

Ž ¾ ½:W¶ OÁˆµ Á²5½²µ¶³¶¶l ³³5 Á²½:Œ:³¶OC ± ®C  3 ®C ±  6”„[4+  ²Á: µ5g02C±Œ²e5J02C  ±


¨
®C 3  5-6„ ®4¼ g \ ÁŒ¶µ5 ³/µ Á²:³/³¶¶Á²5:³¸a³5  6‘²+  6 # ² ~6 ¶ # ®


Ž ³55_0. i5 ½²Oˆ ´  ²µ5³2µ5½:²  :³e+  (Çų¶5/ Á˜¶µ5 ³F¿… Á²¶ ³rQ, ³2½²(²5:³
Q
c‚q +  5½::³QºK–l»¦– !+5²  :³r+ 4 µ¢5 %µ5½:² , ²%5 r²¶²µ$‚V



. 67ºN02,85D3 =»j02,5 ‚  3 !02,85  ‚  


Á ®ˆ ® ®

, 67ºN02,85D3
»j02,85 ‚  3 !02,85  ‚  !34‚
 · ºÉ ¼
r ²‰, B. :³/µ5½:² ³ ®(à ³5!. # :³5/Á²µ5 ³³5I+ .  Á(+ 4 µ/5 ¸²¶²µ‰‚ B„[+
6 # G02,5 Œ# G02.[5‰…· !&# ½:³¶ž¸ ‡N(‚ V² ¸5½²µ5½:ŒK#D¼ ® ‹ r Á+ I¾»µl:Š ³N· ºÉ ¼c
6 6

º _02,5y
± ºN02. 5  00»g02.5y±™»j02,855 ‚  !370 !02.5y± 02,55 " ‚  



 )
Á ® ˆ ®  ® 4 µ¢5 r²¶²µO$ ‚ V„[+[º  F» :´:ÁX» ³ 6±$» ®
à µ 2µ5½:Á²µ¶µ5²Á:r ½:‡5+  ´²?( µ?5 ¨²¶²µ V„[+» 6…0´±\²e5 » +» 6


0´±K²e5 »  ²‰» ³  6 0´±\²e5 »  ® " lÁ:l?µ5½:Á


:Š I+ .½:
4+ ( µ˜5 
²¶²µ ³ W „[+
  
 
 

º  »  µ± º ® SŽ ˜³¶5v
0 »  S5 :³¶a Á¨·( µ¶½: + Á:i<¾ :³5Q( ž³5³¶· ii4 µy»  6 „ ®
  


€  µ5 ´/!²;  


\»  67»  67„[+ ²% Á
‰» 6 „ ( µ¢5 r²¶²µQ$‚ V„ ®
 

" N» K6…„ I– O½:


N»j02,5$6»j02.5(+ ²˜ Á(+ ³5N»  6¸„[(+ O~³5³¶ !02,85$6


02.[5 ®° ² ‡l ³5³¶µ5˜d :³i¿… Á²¶ ³-B » ² ³5 ¢Á: ³¶5:³ ®  ²%  µ5³$%« ² r :³Fµ5½:² ³F5² ³
 + ( µ%5 ¢µ5½:² D,k+  O»j02,5167˜« ² 02,516 ®
¢ v,  µ5½:² ®yà µv‚~6”„[ +  Q´²Q,<6B,  6”º_02,53Œ« ®Wà µv‚™6 ²W+  ·¶²
  µ5³
#102,8516),  6 ,_3 ®
% ½:Á²µ5 ² ²=4+ ³5 Œ:³5r7Á: ³¶5l²v# %[,<°( ,3   µ5³/  ´½²µ¶ ÄW¿À Á² ² ²
\# :³¶r³5 ¶   µ5 · ²Š  Œ³¶<²%³5² ² ²d³¶ O6  ®
 W ² ²=+ ‡³² ¸³5 ¶  :³¶O#&%Y,j°( ,_37² ®

Ɓº
ËrÌ<€Ž’„‘’ •¦

‰#
  pl³5 ¶ Z½²´²¶²   ®<à ž³5 ³  02‚k5O6…#102‚k51± +(² ¥3 ›6  ® :  µ5³=+W( µ
$‚ B² X +
9 9
5 ³r²¶²µ5³ $
²  9  I´²

›0  3 02‚k55q6 ‚!3 0  5  ·º ¼   9@

à µ³¶5 ›0  3 0  3 02‚k556 0  37‚&3 0  55 ® O³=+?I¾»µlŠ:³&· º ¼+? *OI¾  Wµ¶


›0  3 0  3 02‚k55q6  3 02‚k5D3 0  5%²=+I¾»¶µ5
Wµ¶ ›0  34‚›3 0  55n6  3 ›0  34‚k5 ®
     9@

à µr³¶5
       

›0  34‚k516 02‚k583 0  5  ·º * ¼  

( µ¢5 ³r²¶²µ5³$$ ‚ µ¸² ²   ® 9

€  ² ½:%Wµ%µ5½:Á²µ¶µ5²Á:+ ( µ%5 r²¶²µ µ²+   0 5q6 0´²e5 ® ‹   I²=+


Á:²¶5\½² 5½\:³5Œ´µ ( µ 96² ® ‹ =Q + ³¶i²  \:³¶
´µ ( µŒ Á:²µ¶ ²¶²µ Ÿ² ÄŽ
     

  µ5³e + I¾»µl:Š ³X· º * 2¼ ²% V¾ ± (  ± :Š ³5


/µ5½:Á²µ¶µ5²Á:›
 

 0 _3 ²©3 Y5q6 0
5k3 0 5T6 0 Y5D3 ›0´²e53 0 8516 0 Y53µ0 3)²e5 0´²e5
 9      9     9  

" ¾»¶µ5¸Wµ¶=W+ I¾»µl:Š ³X· º * ¼+  j


0 3 ²T3 Y5T6 0 3 ²e5D3 0 Y5   9    9

‹  Á: Wµd :³%²Å ½² 5½:³=+  ·¶² 0 3)²e5n60 3 ²e5 ›0´²e5W+ Á:  


³5  ± 5½ ®    

à µd5 ³%²¶²µ5³O$ ‚ µ
² ²   W+     µ5³\ 9

 0  3 02‚k55q6…0  3 02‚k55 0´²e5T6¸0  34‚ 0´²e55 0´²e5©6  0´²e534‚T0 0´²e55 


      

²=y + I¾»µl:Š ³· º ¼+ 0  3 02‚k55^6B‚¦3 ›0  5©6B‚¦3  0´²e5 ®4¼ g² ½:Á:  7¦0 0´²e55 K6 ²W+ ²
 Á 0´²e5q6² ®
9@     

à µF³¶5ž+ ( µ 5 i²¶²µT$ ‚ µ²+   02‚k516)‚ Á ® ˆ ®  ® #102‚k516)‚v3 ® % ½:Á²µ5 ² ²=+




‡¿À Á²¶ <#&%a02‚&°(*‚!3 Y¢ 5 :³¶¢·² ³ ¶   µ5 · ²Š   ®


 9

   6² ®
9

à µd  ²µ5³X     #10  516² Q 3



 9
99

ËrÌ<€Ž’„‘’ •y”

‹ Á ± ²µ5Á ±  74²l µ5²µ5i¿À Á² ² ² i5 5:³a :³Q¿… Á²¶ ³a7 4 ©,j(° ,13 a+
+
Š €


!  :³¶˜lÁ: ³¶5lµ5½:²   (³5 ¸:³¸³5 ¶ ³¸ µ5 · ²  ® O´ž µ5 ´²µ
7Á:7³ 
:³r³5² :³ ®
 #n ³5 ¶  ® à µ5 µ5½:² +?  4 ž³5  Á&6*“=,|B' y#102,5&6 ,/3 ˜ ®
Ž¨¾ · œ:Á²¶ ¿Q:³5i Árdµ5 ´²µ2<¾  4ÇŐ³¶5rrµ5½:² l5² y6 ƒ  ®  µ5²²µFW ³
 

:³52˜µ5²µ5²µ%<¾ ²~5 ¢Áˆ ³=+~  ³¢ /Á::³¢²³5²Œ· :³%:³52 I:Á²¶´² ²r  ´+




² Á: ³5½²µ 67#10„@5 ®





¼ 5½²µ5:³5³5 ³V ³d/ ³%µlŠ:³Oˆ7 ‡³5¶µ¶Á²¶µ5



Á::³%²³5²· :³
Y¼ à µ/5 dµ5½:² a+4³¶, | 
7  µ5³v,!3 — | 
 ® ‹ O (²=4+ ³¶k, | l  µ5³v#102,85©6”,3
²O# ¶  02,_3 516), ®a€ µ=+  #102,_3 F53}# ¶  02,N3 F5q 6 [02,_3 F5+  Á\#102,_3 F5q6),_3 aW– Á:
   

S ³5³¶µ5¸c,!3 › | 
 ®
     

·¼ €  ´ 7½²  ¶µ5²µd¸³¶ a+ , ²-. ³5 r:³2µ5½:² ³=W + ˆ´:Á‰,<| + ²%³¶8, V~.   µ5³=+ ( µ
5 rµ5½:² +  aW +  _. |  ®


 °  ²:† ³
Wµ
½²¶²µ
Áˆ ³
!~, W$ . W,›3W0 !± + 5 ®S°  # :³¶¸³¶¶µ¶Á²5² ²
 

Á²µ5 ³³5 +  S´²Q#102.[5  #102,516),3 }.K3 "++ ²d Á‰. |  ®


 

 

ƁÂ
 €   µ5 ´   µ5³lWµµ5½:Á²µ¶µ5²Á: ³¶µ _‚ +
 a( µl5  ²¶²µ !‚ ²+
 a( µ5 µ5½:²
, | Y+ T,g302‚™±7²e5J0 ¦± +5 . ¸,g3 ‚T0 ›± + 5 X. | €

 S³¶  
  µ5³ 
 ®  ´²l ´ µ
c‚}6Ÿ² \‚ ‘² +
   

Á:²¶5 *) µ¶¶  :³¶¸´µl( µ ®    lZ²¶²µ¸ÄŽ (( µ¸ :² .³¶( ž³5
V¾»*) µ¶¶ ´µ ®  ² 1,&| .
2² Œlµ5½:² ´½²µ¶ Äy   1,T3‚T0 y± +5 9. 9,T3/02‚-3—²e5J0 y± + 5
: \
    
 µ5³
,3 v3702‚—±Œ²e5J0 N± + 5 9.\3 + 9,3 v34‚T0 N± + 5
      

€ µ=+šI¾»µlŠ:³QY¼+
,D3 ¦|  ® Ž¨¾ ± 4  ± :Š ³5i?µ5½:Á²µ¶µ5²Á: (²µ¶ ²Q³¶I¾»*) µ¶ ²µgy.a3 + | 

O³l  µ5³_. | 
 +g³¶ D+Q5 œ µ5³I¾»µl:Š ³Y¼a +  µ_.!3 +v|  ®?° ~¢ Á ± Š²´ ‡
  7


µ5½:Á²µ¶µ5²Á: ®
€ µ=+ˆÁ:   G± "+ Œ„[:+ 4 µS5 Sµ5½:² Y. 97,  ÇŐ³55?Œ²¶²µk‚ 5² 1,8302‚T±K²e5J0 G± "+5
.9,34‚T0 K± + 5+ ²r Á
 ½:˜³5<c. |   ®
   

 

Áe¼ €  µ5 ´Œ5²W¸


³¶ "+  "+  ³5 ˜:³%µ5½:² ³d4 µr :³5² ³d 
²  
³ 2  ´:³%  µ5³ 
 :³¶¢½² ² ²% ~´ ®  ²-, ²^, + :³2µ5½:² ³25² ³¢Q,& | /²
  

, + |  ® :  µ5³O#102,5k±&,g6 ²O#102, + 5k±h, + 6 + ® O³2³5\# :³5F· ¶œ :Á²¶´


²¢³¶¶µ¶Á²5² ²
Á²µ5 ³³54 + ²  :³5dÁ: ¶l³¶µO' ®1€  ²O½:˜
‡¿… Á²¶  —% ©°(¯#10 5q ± %:³¶/³5³¶
 

Á: ¶ ³¶µ‰' ® ‹ 7³¶  µ5²p5 5:³¸ :³


´ž ²µ5³¸5²µ¶ ½:‡µ5:³l²¶µ5 ² + …· ¡<
+ ¡+
Áž¾ :³5¨ % ± ½: µl²Š  ˜:³i´  ²µ5³i5²µ¶ ½:‡µ5:³ ®¼ y:³¶FÁ: 5²~³i dY¼ % ‡
µ5 µ¶½²5½ Æ ¼ ® ‹ 
 

Wµ¶¶Á² ²µ=W +  <ÇŐ³55/ µ5½:² D, + 5² SG02, + 516 + + ²r Á  n6 ƒ ®


a¼ €  4²¢5²W˜µ5 ´²µ% ¶µ5


Á: ¶œ :Á²¶µ5!³¶( ž³5 ³¢<¾  <Çų¶5¸:³¢µ5½:² ³ ²


"Q+ :´:Á +  ~5² ³d g6 ƒ ²  $6 ƒ ® " ¾»µl:Š ³/Áe¼W+  g³¶ ) /I¾ ½²¶²µ˜ :³˜Áˆ ³ +  7„


²$ „  +  ®a€ µK
    

 

2„  +  ~  µ5³= + I¾»µl:Š ³˜Y¼+ V¾ ²³5²Œ·    Á: ¶²˜:³d½² ½² ²5³/³5³¶Ižµ³r


 

V¾  ´²= + Á:¸<:³5¢4 ž³5³5· 


I¾»µl:Š ³%·¼ ®
 

r "+  µ„   µ5³=I+ I¾»µl:Š ³


Y¼4+ V¾ ²³5²Œ·  lÁ: ¶²Œ:³¸½² ½² ²5³Œ³5³¶a(²¶5³˜
V¾  ´²= + Á:¸<:³5Olˆ  ´ˆ ( ž³5³¶· 
I¾»µl:Š ³r·¼ ®
 

" ³% :³%²Å Áˆ ³=+  ·¶²r ‡Á: ¶µ Á²¶ O½:³¶µ5½:+ ²¢ ¸µ5½:³5 r² ½:Á:   ®
ËrÌ<€Ž’„‘’ •WÊ

X0#G– 5
   F :Á  ³5 ¶  ½²´²¶² ®
 à µ Q,g6 „ –W  ·¶² X
1# 002.[55n67«©±™ª@. ·Â - ¼
( µ¢5 %µ5½:² D.+ !/ª67#10„@5F²Q«-6 G0 „@5 ® ‹  Wµ¶¶Á² ²µN
#10«
5q67#10G0„@55n67« · ¯ ¼
 ‹  µ5² ‡ˆ† v, Wµ02,5W +  ·¶²=+ ( µ¢5 ³¢µ5½:² ³O,–.h
#10 G02,53 G02.55q6 G02,85´#102.[5q±~. #10 02,55D3 0 G02,55
Ž  ²Œ·µ5  Á ± ½²l³¶  ½²¶µ¶² , ²X.GS+ Á:² i µ5 5 V¾ :³¶Œ½² ² ² ® "
S Á+
( µ¢5 ³%µ5½:² ³$,–.h
G02,5´#102.51±™. #10 G02,855D 3 G0 02,55©6 G02.5´#102,5y±~,#10 G02.55D3 G0 G02.55 · žº ¼
‹  Wµ¶¶Á² ²µ=+ ( µ-,—6)„[+  <´²K
G0 02.[55©6 G0«
5y±~.[#10«
5839«J#102.[5 ± G02.[5 ª · ž ¼
€  µ5²4 µ¶5!·  - ¢¼ ²X· ž 2¼ ³K· žº ¼ ®à µr5 ³%µ5½:² ³$,–~.  ·¶²N
02,5´#102.5y±~.[«n±™,G#10«
5839«J#102,5 ±™ª G02,85T6 G02.5´#102,51±~,G«©±~.[#10«
5839«J#102.[5 ±™ª G02.5

ƞÆ
+
 ²Á: µ5 yI¾»µ :³ lŠ X· ¯ ¼G02,85´#102.[5k39«J#102,5 ±™ª G02,85q6 G02.5´#102,5k39«J#102.[5 ±~ª G02.5+WÁ ® ˆ ®  ®
0 G02,5y± «J5J0#102.5y±™ª[5q6…0 02.[5y± «J5J0#102,85y±™ª[5 · ÂÆ ¼
 D# :³¶FÁ: ³55 + ½² \ˆXª+   µ5³=+ I¾»µl:Š ³2 V¾ ½:W¶ ~¿… Á²¶ ²  /¶‡ +  ½:
+i4 µ 5 ³ µ5½:² ³›, ²¦# .   ªB6 ªG02,4±).[5^3 G02,5¢+ Á: ˜²¶µ ¿À Á² ² ²~
ª&6‘„ ² G02,5‰6 „ ®<à µŒ³¶5+  ²K# ³5 Œ5 5:³Œ²ÅO²¶² ²l  :³ ®  5 ³
+
µ5½:Á²µ5 ² ²=W + Á:² ‡l¿… µ¶%·²  Á:  Œ³5 ¶  ®
7Q#*<¾ :³¶ W ³ Á: ³¶5i +   µ5³=a+  dÇŐ³¶5~ µ5½:² O. \ 5² d—#_0. =5-±)ª…6 ƒ „ ®   ‹
V¾ ²³²Œ·  :³
µ5½:² ³\, 5² ³Œ#102,85‰6¥ª ® " ¾»µl:Š ³g· ÂÆ ¼S + ² Á ± ³¶³5³ N.h6‘.  <+     µ5³
02,5q67d « ( µ%5 $,&|  ®
‹ =+ 4 µF5 iµ5½:² , | + 5 œ µ5³2I¾»µl:Š ³\· ÂÆ ¼+  4´²  

¶¶  6   ¶¶  6 + ½²(²

r‰, ® " Á + 4 µ¢5 $, | ¬ 

G02,5n6 0#102,85y±™ª[5839« · ž­ ¼


€  Á: ³¶5˜rÁ:²¶5˜²µ¶²Š µ5rµ5² ‡¶  µ5:³¶5%´ž ‡· %( µT,&| f²i Ád4 µi5 ¨µ5½:² , ®
% ²´² ³r  µ5³v7ˆ V¾ ½:W¶  ¿… Á²¶ ²  ¸¶‡  ®yà µd5 ³¢µ5½:² ³$,k–.G+  g
#102,3 0#102.[5y± ª 5839«
5q6 ,G#102.5 ±~. #102,8583 0#102,5y±~ª[5D39«
‹  Wµ¶¶Á² ²µ=+ ( µ^.6 +  4´²-#102,\3 0#10 5D±<ª 53™«
516),#10 5D±<ª 3™«®+ Á:˜I ³5³¶µ5
\# :³¶d*)  ® ‹ r Á + I¾»µl:Š ³r :³5r*) 
½² ² ² ®
€  4 ž³5Z  µ5³/# %‰, °( j,h3WªG%+ :´:Á 6E ƒ „µ‘· ³5™# <¾ :³¶ W ³ Á: ³¶5e¼r+ ²
¦%a02,j°(  ,_39«
5 ® " / V¾ ½:W¶  ¿… Á²¶ ²  Œ¶‡ +  I´²r+ ( µ¢5 ³¢µ5½:² ³$,k–.&
02,!3  .\39«
583}ª!6), 0 j.\3}ª[5 ±~.80 j,_3}ª[5D3  ,39«-6 ª@,¦±™ªU.‰3  ,!39«
! !

Á:¸<½:´ Qˆ
 6 ª\3 

 6…±$ª
/«13}ª!67«
 6/²   µ5³©ª67„[+ I¾ !  6 7„ ²2³¶ 6 „ ®° ¶µ Á²¶  ® " Á+ 6ƒ ² ® " ³FÁ::³
Á: ¶ ³e +  <´²  6  ¶ +[ª6±  ¶  ²O«^6…±   ¶   ®
 W ² ²=<+ :³¸³5 ¶ ³Œ³5 ¸ :³
Á:  :³›0#G–  d5 5² ³
7³ K#  „[I+ ³5 K#}%G, °(
 
j,¦±  ¶  ¢5 ²›%Y,j°(  ¶ ,g±   ¶   F5 ( µ¢ Á:²µ¶ |—' ‡ “@²®˜ ®
ËrÌ<€Ž’„‘’ •(¥

 Q# ¸5²  Œ Áˆ¶  ®G€  4 ž³5\#10´²e516 ®


F< V¾  Á ± ³¶ 16…
²   µ5³=+ ( µ¢5 r²¶²µ  V„[+
#10#10 ®5516   · ÂžÈ ¼
‹  Wµ¶¶Á² ²µ#10#10´²e55N6¬#10 =5X6 J ®y€  ½:&· ÂžÈ Œ¼ Q+ ( µ5 ²¶²µ  ¥„[+
#10#10#10 555‰6 # !   % ® ‹ I¾»¶µ5\Wµ¶=S+ I¾»µl:Š ³j· ÂžÈ /¼ 5 ¶œ µ5³_#10#10#10 ®555\6  #10 5 ® : ³¶¡+
( µ¢5 r²¶²µ  V„j
# !   % 6  #10 ®5 · ÂÉ ¼
" ¾»µl:Š ³¨ V¾ ½:W¶  ¿… Á²¶ ²  + 4 µ 5 ³¨²¶²µ5³ Y$– V„[+@# ! ´
#10#10 ®55 % 67#10 5J0#10 55  ®
" ÁW+ ˆ´:Á· ÂžÈ ¼+  I´²Q# !    % 67#10 5J0#10 55  + ²rˆ´:Á›· ÂÉ 2¼  ½:d  µ5³d
 # !   %O67#10 5J0#10 l55 
Ɓ­
‹  W =+
 µ¶¶Á² ²µ a4 µ75 ³²¶²µ5³ $ ! % @–! %  ® <  ²Œ·µ5  „[+ =#  • 6 # e 0#10 l55  Ž
 J+
Á ± ¨½²S³¶  ½²¶µ¶ ² i² ² ? ²Œ·µ5i?µ5 5? V¾ :³¶S½² ² ² šÁ ® ®  ® ! %  =+ ˆ #   0#10 l55  6
! $% #  T0#1055 
®à µ  <´²d  µ5³ ! $% q6²+ I¾ ! ! $% O# e JO6 0#1055 + <# • $6…0#1055 
®
à µZ³¶5 ! $%  9#    6  0#10 ®55  =+
² dI¾»¶µ5 Wµ¶ ! $% ! $%  =+ #    6
#   0#10 l55  6
0#105´#10 l55 ®+ J0#10 55 O6…0#10®5´#10 l55  ‹ =+
 WI¾  !   ®  Á:  
5 r:³¶¢( ž³¶¶ ¿  I´² ´+ X
#10®5´#10 l516
#10)l5 ·Â ¼
9@

 7µ5½:Á²µ¶µ5²Á:l ½:‡5Á: Œ  µ5³\ˆ&44 µ/5 ³¸²¶²µ5³c‚  ²  – e–$ B„[+


 #N0  – =–$ 516 )  #0  5 #0 5 ® ‹  Wµ¶¶Á² ²µ= + ( µ%5 r²¶²µV„[+
 

 
   

 #_0 5q6 ² ¶  0#10 l55



 ·Â * ¼
: ³¶¡+D0#10l55 :³¶%´³5· /Wµ ¶  ( µ¢5 r²¶²µQ$‚  ®
µ„ 5² ?#10 <5-6 ²4+ Á:² ‡ ²¶µ  µ5³/ K6 ²(+ ²¸ Á
 
r4¾  ?Çų¶5O²¶²µ
6²‘· ³\O# :³¶¢¶œ :Á²¶´e¼ ®
!

r D+ K6     !  –  – =– | Ÿ ‡ ²  – =– ³5 ¸:³/ Œ·µ5:³
µ5²²µ5³
²Å~ˆ ²Å ³¶¶Á²5³ ®   ”~ „ O²¶²µ ® " ¾»µl:Š ³
Á:laµ5½:Á:½:I+ Z(²¸*) µ¶ ²µ

     

Á ±  Á² :³ <´³5v#10 5 ® € ~(²    < V¾ ÇŐ( ž³F <³i ‡Œ½:Á: ( ž³¶¶ ~²¿V Á²5²µ5³
µ5²²µ5³gn#10 l5 ® " ¾»µl:Š ³Q ‡2µ5² ‡¶ g·  * ¼Ž+ % Áš+ 4 µS5 Q²¶²µ $ ‚  +F02‚n±N²e5  9‚ ´+
 

Á ®ˆ ® ®
  ® ‹  ‚ X 3
!  +
 ¥²<+ Á ® ˆ ®  ®   ®

  ¶  .
 À
¿ 
 
 
³ 
 


 5
 ²
 
 
 5
µ  
´ ²
 5
µ ³ I
p
 ²
 .
 
 :
½ 

° ² ‡  ³5³¶µ5  Á ±  Á² :³ ¢Wµ l³
‡ ½:Á: 4 ž³¶¶  #10 l5¸:´:Á ÇÅ4 ž³ 

 
 
 Œ
 
 
 


 ³ žµ rÁ:² y:´:Á¢ :²  yWµ i³ ‡¸½:Á: ( ž³¶¶  + ²  Ár 2´³5


#10 l5+ Á::Á²S½²%´µS4 µ¢5 r²¶²µ V„ ®


€ .4²Œ  µ5³Á: ³¶½²µ5²µŒ ‡\¿À Á²¶  ™% Ÿk‡ ( Ÿk‡ ½ÇĝWµ G0 l5‰6  #10 5d( µ
5 
²¶²µ B„ ® :  µ5³ G0´²e5O6 ²+ G0 =5O6 7²=y + I¾»µl:Š ³!·  ¼4+ ( µ/5 ³¸²¶²µ5³ @$– ”„[,+ G0  l5O6
0 ®5 G0 l5‰…·  %  :³¶2   µ¶ ± ³5 /Œ ¶ Áˆ¶ ¿W+  ³2 /Áˆ ³¢:³¢²¶²µ5³=+  :³¶
9@

5  ² ²r ¶ Áˆ¶´e¼ ®


" / ³e+ I¾»µl:Š ³% V¾ ½:W¶  ¿À Á²¶ ²  + ( µ¢5 ³r²¶²µ5³ Y$– V„[+  I´²X
 0 G0 l55  G0 G0 55n6  !  0 ®5 %c67# !  #10 ®5 %v6 U0#10 l55  6   0 0 55 
²= + Á:   J0 0 55 ›6 ƒ „[+  ·¶² G0 G0 55n6 …· Á ® ˆ ®  ®  :³¶%/´ ¶ ¼ ®
% ½:³¶  ³- ³¶ # ´½²µ¶ ÄW2 :³?Á: ¶ ³ F V¾ ½² Á:½r  µ5³? l4²?  ³5³5 Á²²µa F  µ¶ ± ³¶ 
´ ¶ ¿  6  #y<+ F´½²µ¶ ÄW ½²´² ² :³Á: ¶ ³l\ V¾ ½² Á:½\³7² µ5²W:³
´ž ²µ5³gF½²W ³5³5²SW ³gÁ:²  :³Q^# ® : ³¶¡:+ ( µQ½²5²µ¶²µa ‡¢ ³Q(²¶5 ´ž ²µQ( ž³5³¶· 
¦#10´² Y5g +  i³¶ ) 7³5µ5:³5¶µ5²µ5~ÅZ  µ¶ ± ³¶ :³7´ ¶ ¿…³ Zi´½²µ¶ ÄW² G0´²e5N6²
‘·  F:³¶Á² ‡µl< + µ5½:Á²µ5 ² ²=a+ 5 ³Œ :³  µ¶ ± ³¶ :³7´ ¶ ¿À³ Z¨´½²µ¶ ÄW² G0´²e5X6¤²

99

´½²µ¶ ÄW²d½² ² ²r :³%Á: ¶ ³˜/ V¾ ½² Á:½e¼ ®


à µa75²   µ¶ ± ³¶  +  0´² Y5n6 
0 Y5J0 G0 Y55 * G0
5+ :´:Á G
0 Y5*+ G0 Y5g² G0
g 5 ²ÅNˆ
²Ål³¶¶Á²5³ ² ³ Ÿ ‡ “@²®˜ ®Ã µ 5²µ¨ V¾  (² F ‡˜³5³Á:ž+  Á ± ³¶¨ ÁG0 Y5n 6 ®

99    

à ³5 ~:³¶¢Œ´ ¶ D4 + Á:² ‡ ²¶µŒ G


0 Y5©6 ® ‹ =+ ³5ŒÁž¾ :³¶%   µ¶ ± ³¶ +
 

  G0 Y5!6 G0 Y5 G0 Y5¦6  g + Á:~25²µ5Á:²¶5 ´ž ²µ4 µ G0
5 ® ¼ F\µ5:³55\ ³<¾ ˆ
Á ± ³¶µ G0
5©6 ® ‹ ˜  µ5³ G0´² Y5©6!² „ ®
   

 ³³5 ³r5 /!²;   ‡Á: ³¶¶µ¶Á²¶ Z ™4 O G0 Y5$6
F4+ ²d( µ˜5  Oµ5²²µ
 
99

¶µ5F + + d²
F+ 7( ž³5G0 85q6 ®ž°  ¨5 a  µ¶ ± ³¶ 2:³¶g²¶²Š µ5² ²¨½²5²µ¶½
 

Wµ%³:³¢´  ²µ5³r³¶µ% :³¢ ·µ5:³%µ5²²µ5³=y + Á::³%Á ±  Å ½Çij5³5²%·²  ®


   

 W ² ²=+ ‡l´ž ²µ% 


X#10´² Y¢5 :³¶\!² „ ®

99

ƁÈ
ËrÌ<€Ž’„‘’ •(

Ž Š
S ¿À Á²¶ p  \:³¶
Á² ‡µ5² ²l³5 ¶  .µ5 · ²  ®   X# \³5 ¶ p½²´²¶²  
¶µ5Œ/ ‡7¿À Á²¶     ®
v#  &
 ³ˆ¾»  W ³ ¨ *4 ž³   µ5³  6 Ž  ® SZ¿À Á²¶   :³¶ Á: ¶ ²\´½²µ¶ ÄW
 ! %³  —6  0 G02,85T3 02.[55 q€
   4ˆ
®   ´

V¾ ÇŲµ5Á²Á: Æ <¾»  µ5³ Z:³5¿… Á²¶  *)  ® µ q€ =+
=+
³ ~
 dW ³%   ³d³ˆ¾» ²µ 4Áž¾ :³¶r Á<¾ ²  Œ:³¶rÁ: ³¶5 ®yà µ˜³55 O:³¶ +a#
Á: ³55 ®
=+
% ½:Á²µ5 ² ² :³l¿À Á²¶ ³ Á: ³¶5:³    :³³5 ·²#:³ ³5 ¶ ³\ µ5 
Š· ²  ®
Xˆ X#
 ¼ Iµ5:³¶5 7µ5 ´²µr Œ³ˆ¾» ¸W ³ ®
h# °   + &ª
<¾ :³¶ W ³\ ‡.¿À Á²¶ n   F ˜Çų¶5 h#10ª 5V6 ƒ „
5² ˜ } ®Fà µ~ V¾»·³¶µ5
, _#102,5^6„ 8€ )
³54 ž³5 ³˜<¾  gÇų¶5 µ5½:² Z5² g [Š ¦02, 5
®  Á: ³¶ ²µ5  µ5³˜ ‡\³¶5 r½ÇÄ
,/6¬, =+ !‚ „[+y, 6 ³ : !#_0, ³ 50¡#_02, 5D39#10ª[55!6
$#_0, 5a3 #10ª ³ 516µ#10ª 5c 67ƒ „
Wµ #² a( µl5  ²¶²µ     ®  µ5³ 


#02, 5 #10ª[5 8#_0, 516 „


 \#102, 5q6 „
$‚ V„[+ !#_0, ³ 5T6 „
  
®    µ5³ 7²2 Á  ®¼ (:³¶¢ Á
+
¿V Á² ¸¸µ5 ´²µ%Wµrµ5½:Á²µ¶µ5²Á:Œ 4 µ%5 d²¶²µ $ W  
  
®
€ =+ X‚
µ <( µŒ5 ²¶²µ „[+ N, ³ ±Vª™6  02, ±~ª[5
 S   ½² ² ²      ®Q¼ ¨:³5
¿V Á² I¾ ²
_0, 5
½:µ5/r ‡7³¶5

^ª+ +#10ª 516 „
¨Á: ´²µ¶/´²µ5³ ²2 Á/dWµ¢Á: ¶5½ ®° ¶µ ÁÇ
¶  ®

ËrÌ<€Ž’„‘’ •W©

Q#
  
³5 ¶  ½²´²¶²   ® :  µ5³=+( µ¢5 %µ5½:² D,Œ„j
,  # , ² 39#102,8516 ,_3, ²




²
² 1# 02,85839# ² 6  6 ²
,

 ,  3 ² ,_3 ²
" ¾ !™
! ²O±  % #102,516 ,y0´,! ²$± ,5
3)²
< |h“@±\²–=²®˜@+  ´²   ¶  
67d
³  „ ( µg5 gµ5½:² , V„[+ Á:i:³5gÁ² ‡µ5² ²¨·³¶µ5 ®
¼ I<¾ l Á¸W ³%
³5 ¶  4 µ¢¸5²  
´  ²µr ®
  ²+2  µ5³¦#102,85j6  ¶     ¶ ³   
® O³=¨+ 4 µ›, |  „ –   ¨+i * ·¶²j#102,85  „ ®
° ¶µ Á²¶  ®y¼ I<¾ 7W ³%    ³r¸  ³ ¶  ³rÁ:¸Áˆ ³ ®
  ¯±\²+¢  µ5³—#102,5<6  ¶     ¶ ³   
2+ ² Á:²¶5 ¿… ³j#102,$ 5  „ ( µ~5 ¦,  „ ®
° ¶µ Á²¶  ®  
¨D„ µ²+   µ5³$#102,5q6  ¶     ¶ ³   
+ ²O#102,85 Vl „ ( µF5 -,    ®° ¶µ Á²¶  ®
F | a±V²–l„ o+y  µ5³v#102,516  ¶      ¶³   
®
 
% ½:Á²µ5 ² ²=Ž+  <¾ :³¶QW ³g )Á² 2¨´½²µ¶ ÄW²µaF³¶ | d±!²–l„  ²y#&%,j°(  ¶     ¶ ³   

  µ5³QO # :³¶¢
³5 ¶   µ5 · ²Š   ® 
 W ² ²X
F | a±V²–l„ a   µ5³r I<¾ ~Á²³5 ¶  ®
F | a±V²–l„ a   µ5³v#/%@,g°(  ¶      ¶³   
:³¶¢ ‡ ³5² ¸³5 ¶  ®
ËrÌ<€Ž’„‘’ ¦ 

 Q# 
³5 ¶  ½²´²¶²   ®
 à µQ,g6).!6  +  I´²v#102,–,5T6…0ªc3V«J5´#102,k–,5 ®
ÆÉ
 < Y¼ @ª3\«N6ƒ ²+š  µ5³ #102,–,8516 „¢( µ<5 Sµ5½:² Y, ®•€ µ=+:( µD.!6 +: V¾ ½:W¶ 
¿À Á²¶ ²  
Á:  vˆj
#102,–.5q6 ª #102,–.5839«J#102.–.5q6)ª #102,–.5
" ¾ !—ª!6¸² <# :³¶¢ ‡l¿… Á²¶    +WIµ5½:Á²µ5 ² ²=+y:³¶¢·² 
³ ¶  ®
4 ž³5 ³Sqª6² ²<ˆ+ ? ³e+=7 # ?³5 <W ³I ‡F¿À Á²¶ 
   ® :  µ5³=+²4 µS5 ³Iµ5½:² ³
,k–.– a+#102,–.5T6B#102,– U5k39«J#102.– F5 ®Wà µO,/67.+  S´²/  µ5³X0´²^39«
5´#102,–U5©6µ#102,k–,5T6B„ ®
" ¾ !}«!6±\² ® Ž ¾ ½:W¶  ¿À Á²¶ ²  \³ˆ¾ ½:Á²µ¶l  µ5³N#102,k– U5K6 #102,–.513 #102.– U¸5 4 µ5 ³
µ5½:² ³$,–.G–  ®
‹  µ5²Wj,‘6 2+  ½:   µ5³ ¿V Á² ² ² F+ 4 µ~5 ³~µ5½:² ³j,k–.G+-#102,k–.[5h6
±Q#102.–,85 ®yà ž³5 ³G02,85O6¸#102,k–l„@5 ® :  µ5³=y+ ( µ˜5 ³/µ5½:² ³v,k–.G+8#102,–.5$6…#102,k–l„@5k3 #10„ –.5O6
02,5y± 02.[5 ®
% ½:Á²µ5 ² ²=y+ ³¶  :³¶%
¿… Á²¶  µ5½:²  Œ² Á: <´½²µ¶ ÄW G0„@5^67„[y+  ´½²µ¶ ÄW
¿V Á² ² ²˜/ ‡¿À Á²¶ <#&%02,k–.[5T°( G02,51± G02.¢ 5 :³¶¢¸³5 ¶   µ5 · ²Š   ®
·¼ aªq3—«$6² ® :  µ5³=+ V¾ ½:W¶ ¿… Á²¶ ²  ˜³ˆ¾ ½:Á²µ¶n#102,k–.[5q67ªF#102,k– U5Y3™0´²±ª[5´#102.– U5 ®
‹ ˜ Á_#102,–,85-6#102,– U54+ Á:a³¶ž ÄW_.# :³¶˜½²4²5l³\²Å²Š  ´žµ¶‡· 
…· ²% ÁŒ<¾  S³š¾»ž%² ¿À%I¾ ¸¿… Á²¶  I¾ Œ³5² ¸´žµ¶‡· e¼ ®
à ž³5 ³ k02,5$6¸#102,–.5 ® :  µ5³=y + 4 µ˜5 ³/µ5½:² ³v,–.+ 02,5O6ª k02,5 3B0´²v±4ª 5 k02.[5 ® " Á+
‡¿… Á²¶  9…· ²/ ÁN#D2 ¼ :³5˜Á: ³¶5l <ª¦6 ² ® " ³d ³5:Á:  Áˆ ³=W+ Z  µ5³‰«c6B„ ²=+
( µ¢5 ³%µ5½:² ³$,–.+#102,–.5q67#102,–,85 ®
% ½:Á²µ5 ² ²=+ :³¨¿À Á²¶ ³¢Á: ³¶5:³¢³5 2·² ³5 ¶ ³2  µ5³5QªO3™«$6²+ ²¢³¶(
 ³‰ª&6   ²   µ5³¸5 5¿… Á²¶  #V%y02,k–.[5v°( 02,5I+ ! p:³¶¸7¿À Á²¶ Zµ5½:²  µ¶·¶µµ5+
:³5¢·² 
³5 ¶  ®

ËrÌ<€Ž’„‘’ ¦<{

O N# : =+
à µQ V¾»·³¶µ5 ˆ³54 ž³5 ³S<¾  ÇŐ³¶5a¨5²  a¿À Á²¶  ®  µ5³ :( µ<5 ³<µ5½:² ³ ,–. V„[+
#102,‰3&.5G±/#102,5 V.0#102,855  V„
 v#
 ®¼ (² ½:Á:  / :³¶i³¶¶µ¶Á²5² ²%Á²µ5 ³5³5˜³5µ n' ³ ‡ ® ‹ 
=+ ~# O; +
Wµ¶¶Á² ²µ y ‡l¿À Á²¶  O
(² ²¶µ5²¶² ²/   4²r gÇŐ³¶5 Á
 µ5½:² )„
5² < K#_0 5‰6)ƒ „ ®
-. Œ„[+
à µd5       µ5³ Q#_0 43 .5 Œ#10 534.0#10 55  \

²

$ #10 5D3 .80#10 55  6B3



 ³ %
 ÁX# <¾ :³5%W ³¢•¶œ µ5½: ® ‹  Wµ¶¶Á² ²µ=W +  <ÇŐ³¶5/ µ5½:² ªŒ„5² <X#10ª 5V„ ®
à µŒ³55( + 4 µ/5 ¸µ5½:² q, WªI+ pg#102,5 W„ ®Ià µ
Z5² y, +I .Á ± ³¶‰.&6    
²
  µ5³K
# ,_3 #102² ,85  #102,85


· Æ"- ¼
 F  µ5³i ‡
³¶5K0, ?5 ½ÇÄrWµn,  6 lª ²=+ ( µ¨5 F²¶²µ©$‚ V„[+Y, ³  6), 3    ) ®
 7µ5½:Á²µ¶µ5²Á: ³³¸ )Á² 5½l  ¶µ5 7 ‡~³¶5j02, 5˜:³¶
  µ5³/·².½ÇÄ+I³¶¶µ¶Á²5² ²
  

Á²µ5 ³³5 + ²2 Á˜$, Œlª 4 µ25 2²¶²µ^$‚ V„ ®[€  ( ž³5˜  µ5³ ! 6µ#N02, 5 ® SŽ ³¶5



0 ! 25 :³5% ÁK7ˆ ´  ²µ5³r³¶¶µ¶Á²5² ²˜4 ž³5¶´:³%²=+ I¾»µl:Š ³X· Æ"- ¼+ ( µ%5 r²¶²µ$$‚ V„—
  

! ³  67#02, ³  5q67# [, 3 #_0², 5  #02, 5T


 


 
6 !
 


€  ² ½:¿V Á² ² ² h0 ! Œ5 ´²µ¶ ´²µ5³!3


 g+ ² ³7µ5½:Á²³5½² ² Q+ 4 µ75 
²¶²µ$$ ‚ V„[+ !  ! ® 



Æ@
 ³r  µ5³ $, ³  ±~, 6

6  )   )  ® ‹  ³ =+ <´²X


  )
, ³  ±™ª  !²     ²  !  



 ²Á: µ5›,
³  ¥ª_3  ®  ³l  µ5³=Q+ ³5g# :³57Á²µ5 ³5³ 5g+ 4 µ75 l²¶²µ_‚ ¥„[+
! ³  6¤#_0, ³  5   # =ªc3  y?+ Á: %²¶µ  0 ! 75 :³¶·4 µ¶½:+ ² ³\  ‡


  
Á: ¶µ Á²¶ O¶5² ®

ËrÌ<€Ž’„‘’ ¦4Š

+
   
³5 ¶  ½²´²¶²   y¶µ5Œ/ ¸(  ,+  ¸ ®
O«$6²T±<ª O6 „[+
 à µ ²  (´²  ›0´²T±&ªa–lª[5a3 0ª–=²n±<ª 53 0´²–l„@5n6 „74 µ25 2µ5½:²
ª " 0´²–l„@5©6²+ 
 
®     ½:d  µ5³d

›0´²$±™ª–lª 5q6…±\²-± 0ª–=²O±™ª[5


  · Ư ¼
 à µ -6…²$±™ªN± «®+ <´²=+( µ¢5 ³%µ5½:² ³$ª ²Q«N

0´²$±™ª–lª 53 0´²$± «•–d«


5D3 ›0ª‰3V«e–=²O±™ªX± «J5167„
  

‹ r Á+I¾»µlŠ:³X· Ư ¼v


0ª\39«•–=²$±™ªN± «
5q6 0ª–=²$±™ª[5D3 0«•–=²$± «J583
   · Ɓº ¼
à ž³5 ³q#&%,j°( 02,k–=²±\,85=3 ® :  µ5³$· Ɓº < ¼ ³š¾ ½:Á²µ¶g5 ?³¶ ² ²T#10ªD3!«
5167#10ª[5=3!#10«
5+
( µa5 ³gµ5½:² ³1Œ ª ²1« ®Y€ lµ5:Á: W ? V¾ ½:W¶   ° Á ± Y+ ²a ‡d½ÇĶ ^\#  ³5³5µ5F<¾ ²  


:³5/Á: ¶ ®1€ .³


<¾»  µ5³K# :³¶¸ ½ˆµ5 ®k€ µ=+D#10´²e5v6 ‰3 0´²–l„@5‰6 ~ Á#102,5Q6 ®,
( µ¢5 %µ5½:² D, ®W°˜® ˆ ®  ®


›02,–=²O±h,85q6 ®,g±
 · Ɓ ¼
( µ¢5 %µ5½:² D, ®
à µ5 ³µ5½:² ³ª–d « 5² ³ gª!3µ«46 ƒ „[a+ ²( µ 6 ª!3B«•+ ,76  ².V6  g+ ‡
Á: ¶  F ¼ Á: Oˆ ›0ªa–d«
5q6…0ªv3 «
5 02,–.i5 ²%  N,X3™.!6² ® " ¾» µl³ :Š  ³K· Ɓ ¼+   ³ ½: 
  

0ªa–d«
5q6¸0ª‰3V«J5 ª\39ª « ± 6…0ª\39«
5 ¶  0ª_± «
5
  µ5³r
  


‹ =S+ Á:   :³¶7Á: ¶g+ Á:²¶5\µ5² ‡¶ pµ5:³55 ´µ !²;   ³¶Tª3 «!6¥„ ® : ³¶¡I+ ( µ
5 ³%µ5½:² ³$, ²-. +   02,k–.[5T6…02,3}.[5 ¶ 02,¦± ®.5 ®


% ½:Á²µ5 ² ²=+  <:³¶2¿V Á² 


/´½²µ¶ ÄW²µd¸Á:/(  ,+  
:³5¢·² 
³5 ¶  ®
 

¹ s! I´
[ @Ž ¾ ± 4  ± :Š ³r³5²    ‡ ²  % ‡/¿… Á²¶ \µ5:Á ± ²µ5Á ± ½:/ T²; ¶µ5r (  ,+  r%³5²µ¶
²~¿Vr<¾  ˆ  ³5³¶µ5²µ¢ ‡Á: ¶5½ ® ‹  Œµd³5<—²; ¶µ5Œ <·  ® 

ËrÌ<€Ž’„‘’ ¦y•

Q#
  
³5 ¶  ½²´²¶²   ®
1, B„[+ g#102,›3 Y5v6¸#102,#10 Y55´#10 Y5Q6¸„ " ~#
 Ã µ
5 /µ5½:²  I .

® ¾ ! :³¶/²¶² ²
! b3

  
³¶µ ®
-.7|  „ – 
 à µ 5 µ5½:² -, Ÿ„ =+q#102,G#102.55´#102.56„
 ?²5 µ5½:²  Z  OI¾ ~Wµ¶ Z³¶
©#102,g3 .[5N6 „[+ ˆ ,g3). +
²l³5² ² ²l³5 aÁ ® ®  ® =+1#120 ,G#102.55´#102.5K6 „
 a²I¾»¶µ5 Wµ¶ ³5i²
1#102,G#102.55©6W„ ˆ ,G#102.5
³² ² ²¸³¶ Á ® ® ® c, \± .
% ®y¼ g ²O½:Á:  l :³˜Á: ¶ ³  ²

Æ*
,   ³5 r½:´ž ²5:³ ® ‹ +
r Á y  6 c±™. à µ˜³¶5+( µ%5 $.j|  „ – ¨+y  
6

#102.51 6   ¶  ®  
 W ² ²=+ ‡³² ¸³5 ¶  ( ž³5³¶· ¸ µ5 · Š² :³¶¢ ‡¿… Á²¶  ½ÇÄ
Wµ\

#102.516 ‰±~ . ³¶8.g|  „ –  ² #102.516 „ ³¶8. 


¼ <Œµ5:³¶5 ³r<¾ ˆ´½²µ¶ ÄW²µ/Á:²¶5
¿À Á²¶  :³¶d·² ³5 ¶ O µ5 · ²Š   ® Ž<:³
Á: ¶ ³r¼2²¢¼2³5 rÁ² ‡µ5² ²d´½²µ¶ ÄW½::³ ®  ²O,– . V „ ²Å~µ5½:² ³ ®
2D,_3}. ²-,G#102.5    µ5³% :³%²Å  ²·µ5:³d¸ V¾ ½² 5½³5  ± 5½:Œ³ r½²Å/ˆ
„®
a,O3j. ¸²n,G#102. 5  ®  n. 
²i Á% :³i²Å ²Œ·µ5:³F% V¾ ½² 5½r³ i½²Å
ˆ\„ ®  T. +ž³2  µ5³ V¾ ½² 5½Q,G#102.5  Œ³ˆ¾ ½:Á²µ¶T,  ¶   + Á:%y O,O3j.  +
² Á: ¶µ Á²¶ Oˆ´:Á
 ¶µ5Œ¶µ5 ± 4  ± :Š ³5 ®4° ¸Áˆ ³¢¸(² % Á¸W ³r³5/µ5 µ5 ®
(Y,D3c .  ¢²D,#102.[5  r  µ5³D.  ® : ³¶U#102.516  ¶  ² #102,G#102.5516  ¶ 
   6  ¶   ®
à µr³¶5X#102,G#102.55´#102.[516
 ¶   ¶      6   ¶  ¶ 67#102,_3}.[5 ®  
¨8,X3  .  l²^,G#102.5    µ5³-,  ¶  + Á:¸I²¶µ‰,_3™.  + ² Á: ¶µ Á²¶ 
ˆ´:Á¸ ¶µ5Œ¶µ5 ± (  ± :Š ³5 ®
 W ² ²=S+ ³Œ5 ³¸ :³
Áˆ ³=(+ ‡ Á: ¶ p/ ¼ :³¶¸·².´½²µ¶ ÄW½:<+ ²K# :³5¸ ³5 ¶ 
 µ5 · ²Š   ®

ËrÌ<€Ž’„‘’ ¦W¦

€ ¡+
 F  (²~Wµ~ÇŲ OÁ: ³¶¶µ¶µ5O 5²   ¿… Á²¶  Wµ~µ5½:Á²µ¶µ5²Á: ¢ + ‡.¿V:† 

³5´ 5 ®  4 ž³ ™#10´²e5~6 —‚ O€
®    9²¶²µ ÄŽ ®  ³54 ž³5ZO :³  ·µ5:³
#10´²e5b– •–d#102‚—±V²e5 =+ X#10´²e5 Œ#102‚j±V²e5
2 r½²5½¸Á: ³¶¶µ¶5³ Wˆ´:Á   ®
€ G02‚k5 |~“@²– •–‚/±V²®˜ K#10 5 V‚ ®€  ( ž³5

 ½:³¶ž¸Wµ  F ¸ ³ržµ ²¶²µ  Œ5² Q  
q#102‚k5y6)‚-3 02‚k5 Ž q#_0 5 G02‚k5 G02‚Q±¦²e5+

  µ5³  ® QdÁ: ¶ \FÁ²µ5 ³³ Á:2³¶µQ :³  a ³³¶µ5i<¾»  µ5³  
²% ÁŒ \#102‚k5 Œ#102‚—±Œ²e5
 ®
" =+
¾»¶µ5 Wµ¶ I³5l ‡\¿… Á²¶  # _“@²– F– ®–‚y˜²

:³¶
³5¶µ¶Á²5² ²Á²µ5 ³³5 ³5µ  
#10´²e5v6 + j#102‚k5 B‚
I Z  ®(à µŒ³¶5  G0#102‚k55v6 G“ U–=² ”‚ Q#10 5 #102‚k5d˜6…‚q+
     Z²  
&#10#102‚k55 6 G0#102‚k553}#102‚k5167#102‚k583}‚
I¾  !  ®
: ¶µ5¸ ½² ±  
 ¸Á: ³¶¶µ¶Á²¶ 4
€   5~6 ³   ‘· ˜ Œ·µ5¸I¾ µb¼ ®¼ (:³¶F¿V Á² ˜d´½²µ¶ ÄW²µ% p:³¶Fµ¶µ¶ ² S²F
  6 h±V² ®Wà µ˜5 d²¶²µv‚ W²W+  4 ž³Œ  µ5³v#102‚k5n6  ‚ j3  oW+ !  ? ½:³¶žŒ ‡lWµ¶¶
² ¶²Š µ5 ®


 Ã ³5
~6²– !ˆg²
„ ¶  µl:Š ³e +  #10´²e5q6 l²Q# :³¶%³5¶µ¶Á²5² ²/Á² µ5 ³5³5 ®
 à µ 5 ³ ²¶²µ5³ ~–‚ u²2 + 5² ³ ™#102‚k5~6 ™+  ‚ 43  3¥²¢+ Á ® ˆ ®  ®


‚Q±      9‚©3    ®à µ ³¶5T‚Q±  B0 v±²e5 9‚©3  ® " ¾ ! 43‚#   43   43‚©3j² ®
‹ ¢ÄW ² ²=W+  <´²Q#10 <516 3}‚q+ Á ® ˆ ®  ® #10#102‚k55y67#102‚k5834‚ ®

!

ËrÌ<€Ž’„‘’ ¦4”

  -# a€ ~( ž³5  6 F± ²–b3


  ®à µ¢5 ^,& | ^+²~Á ± ³¶³5³
/³5 ¶  ½²´²¶²   ®
.!6), + K
 I´²
#102,_39#102,85834,G#102,85516),_39#102,85834,G#102,85 · ÆžÆ ¼
²% Á\,_39#102,85834,G#102,85F:³¶% ( ¢ÄŸK# ®
° ² ‡  ³d´5N ˆ ½²¶²µd :³d( 5³rÄÅ:³˜X# ® " ¾»µlŠ:³d¼+y V¾ ½:W¶    
6 ²7  ²
\ ³i²ŝ³ ¶ ³i³¶µ  “e„F˜@+W ³¨ ³iI¾ d³5µiÁ ±  Á²~:³i5²µ¶´ž  :³ ±h² l„ ²T'T³ ‡ ®

­ -
2 =+ K#
:³¶dŒ
Á::³˜³5 ¶ ³ yÁž¾ :³¶r 4 %ÄÅŒ ®Wà µ v,<67.›6 +  S´²/  µ5³
# 3! % 6 3 + ²¨ Á 3 O# F€ =+
:³¶¨³5³5 (  ÄÅ% ® µ ž <¾ :³¶ W ³i )Á² 
 ´½²µ¶ ÄW²µŒ ² -3 !;
³5 Œ³5¶Á²5³Œ²
5 ³Œ :³
²Å.³¸  ² :³
5²µ¶´ž  :³
a ±Œ² l„    j' ³ ‡ +
Á:¸<Á: ¶µ5:˜ ¶µ5¸µ5²µ5¸µ5½:Á:½:²5 ®
+
à µ
³¶5 (( µ/5  ‰,9| “e„F˜@+
 < p   
6ƒ ² ˆ
®S°˜® ®  ® l³5² ?( ˜ÄÅl4 ž³5³5· 7 #
c„ =+ lŠ ›· ¼+
:³5 ®  ³
  µ5³ II¾»µ :³ ÆžÆ I4 µ¸5  ‰,V| ^+
  I p~ Á _,›3)#102,5 3Œ,G#102,85Q6 „[+ ˆ IÁ ® ®  ®
#102,8516± ³ 
 ®
% ½:Á²µ5 ² ²=+W³¶# :³¶%½ÇĸWµv#102,516±  ³  6…±\²T3  ³   ( µ%5 $,<|^+  µ5³=+
( µ%5 Q,h| ^W +  #102,5$| ²=+W³¶S
 ³O,}6µƒ „[+4  µ5³   
6¸±  ³   +WÁ:ŒQ ³5³¶µ5

‡¿À Á²¶ —,—°(   
:³¶%³5¶µ¶Á²5² ²dÁ²µ5 ³³5
³5µ ±V² l„ ²r³5µ-' ³ ‡ ®
 ZÁˆ Á² ³³¸  )Á² 5½l  ¶µ5 ²Ä.7 ‡ Á: ¶ .d¼ :³¶¸³¶³¿V5+I²¸ Á#
:³5¢·² 
³5 ¶   µ5 · ²Š   ®

 8  &(=?6;g=?'
 ¯  ® :  ³5³¶ ‡ + MU2S1Z\s!  2i s!  J bZ+ ‹  ³5:³ ®
º SÃQ® F µ¶³ :5²D+ A eH[Is Y 2iU+ - ·(²µ¶ ®

 
 SÃQ® F µ¶³ :5²D+ h£Y s Y 2iU+ - ·(²µ¶ ®
 
Æ : ® ‹ ² ¡+  I@ts š ZJmL=2U£›ZJ I´  o¡£Z4
  + µ¶²µ ®

­¯
­žº
Polynômes à une variable

Igor Kortchemski

Table des matières

1 Opérations sur les polynômes 1

2 Division euclidienne et racines 3


2.1 Division euclidienne de polynômes . . . . . . . . . . . . . . . . . . . . . . . . . . 3
2.2 Racines et factorisation de polynômes . . . . . . . . . . . . . . . . . . . . . . . . 4
2.3 Racines multiples et polynôme dérivé . . . . . . . . . . . . . . . . . . . . . . . . 6
2.4 Exercices d’application . . . . . . . . . . . . . . . . . . . . . . . . . . . . . . . . . 8
2.5 Interpolation . . . . . . . . . . . . . . . . . . . . . . . . . . . . . . . . . . . . . . . 8
2.6 Cas des polynômes à petit degré . . . . . . . . . . . . . . . . . . . . . . . . . . . 9

3 Polynômes symétriques élémentaires 10


3.1 Relations de Viète . . . . . . . . . . . . . . . . . . . . . . . . . . . . . . . . . . . . 10
3.2 Relations de Newton . . . . . . . . . . . . . . . . . . . . . . . . . . . . . . . . . . 12

4 Polynômes à coefficients entiers 14

5 Polynômes à coefficients réels 14

6 Polynômes à coefficients complexes 15


6.1 Nombres complexes . . . . . . . . . . . . . . . . . . . . . . . . . . . . . . . . . . 15
6.2 Théorème de d’Alembert-Gauss . . . . . . . . . . . . . . . . . . . . . . . . . . . . 16

7 Arithmétique de K[X] 16
7.1 Théorème de Bézout dans K[X] . . . . . . . . . . . . . . . . . . . . . . . . . . . . 16
7.2 Polynômes irréductibles de K[X] . . . . . . . . . . . . . . . . . . . . . . . . . . . 17
7.3 Polynômes irréductibles à coefficients entiers ou rationnels . . . . . . . . . . . . 18

8 Exercices supplémentaires 20

9 Quelques motivations 22

10 Distinction entre polynôme et fonction polynomiale 23

1
11 Éléments de réponse aux exercices 24

1 Opérations sur les polynômes


On commence par définir la notion de polynôme et voir quelques propriétés.
Définition 1. Une fonction P de R dans R est appelée polynôme à coefficient réels (abrégé en
polynôme dans ce qui suit) s’il existe un entier n > 0 et des nombres réels a0 , . . . , an tels que
pour tout x ∈ R :
P(x) = a0 + a1 x + · · · + an xn .
On verra plus tard (Corollaire 9) qu’un polynôme à coefficients réels s’écrit de manière unique
sous cette forme. Si an 6= 0, on dit que le degré de P, noté deg P, vaut n. Dans ce cas, an est
appelé le coefficient dominant de P. On décrète que le degré du polynôme nul est −∞. Si
le coefficient dominant de P vaut 1, on dit que ce polynôme est unitaire. On note R[X] l’en-
semble des polynômes à coefficients réels. De même, on note Q[X] l’ensemble des polynômes
à coefficients rationnels et Z[X] l’ensemble des polynômes à coefficients entiers.
Dans ce qui suit, nous ne ferons pas de distinction entre polynôme et fonction polynomiale
associée. Il faudrait la faire en toute rigueur, mais plutôt que de rendre l’exposition abstraite,
nous préférons insister sur les idées sous-jacentes. Voir l’appendice situé à la fin du cours pour
plus de détails.
On notera indifféremment P(x) ou P(X) ou encore P.

Exemple 2. La fonction P(x) = 2 − 2x + πx2 est un polynôme de degré 2 de coefficient
dominant π. La fonction Q(x) = |x| n’est pas un polynôme (pourquoi ?).
Remarque 3. Par convention, le degré du polynôme nul est −∞. Ainsi, les polynômes de
degré zéro sont exactement les fonctions constantes non nulles.
Proposition 4. Soient P, Q deux polynômes. Alors P + Q et P × Q sont également deux poly-
nômes.
Démonstration. Pour P + Q il suffit d’utiliser le fait que αxi + βxi = (α + β)xi pour un nombre
réel x, et pour P(x) × Q(x), il suffit de développer le produit.
Exemple 5. Pour tout réel a et tout entier positif n, P(x) = (x − a)n est un polynôme de degré
n.
Proposition 6. Soient P, Q deux polynômes. Alors deg(P+Q) 6 max(deg P, deg Q) et deg(P×
Q) = deg P + deg Q (avec la convention −∞ + α = −∞ pour que cet énoncé soit valable si
l’un des deux polynômes est nul).
Démonstration. On vérifie aisément que deg(P + Q) = deg P si deg P > deg Q, que deg(P +
Q) = deg Q si deg Q > deg P et que si deg P = deg Q, alors deg(P + Q) 6 deg P. Il peut
cependant ne pas y avoir égalité (prendre par exemple P(x) = x2 et Q(x) = −x2 ).
La deuxième partie de la proposition découle du fait que si an est le coefficient dominant
de P et bm est le coefficient dominant de Q, alors an bm est le coefficient dominant de PQ.

2
Exemple 7. Soit E un ensemble fini et f : E → N une application. Alors
X
P(x) = xf(α)
α∈E

est un polynôme à coefficients entiers. Si kn désigne le nombre nombre d’éléments α ∈ E tels


que f(α) = n, alors le coefficient devant xn est égal à kn . Le polynôme P est appelé fonction
génératrice associée à f. Ce genre de polynômes apparaissent fréquemment en combinatoire,
où il arrive qu’on ne connaisse pas de formule explicite pour kn , bien que le polynôme P
se calcule aisément (voir exercice 18). L’intérêt d’introduire cette fonction génératrice est que
la connaissance du polynôme P nous permet alors d’accéder à certaines informations (par
exemple des formules de récurrence ou un comportement asymptotique).
Le résultat crucial suivant permet de montrer l’unicité de l’écriture d’un polynôme :
Théorème 8. Soit P(x) = a0 +a1 x+· · ·+an xn un polynôme à coefficients réels tel que P(x) = 0
pour tout x ∈ R. Alors a0 = a1 = · · · = an = 0.
Démonstration. Raisonnons par l’absurde, est supposons que P(x) = ak xk + ak+1 xk+1 + · · · +
an xn avec k > 0 et ak 6= 0. Comme pour tout x ∈ R∗ on a P(x) = 0, en divisant par xk on en
déduit que ak + xak+1 + · · · + an xn−k = 0 pour tout x ∈ R∗ . En faisant tendre x vers 0, on en
déduit que ak = 0, ce qui est absurde.
Corollaire 9. Soient a0 , a1 , . . . , an et b0 , b1 , . . . , bm des nombres réels tels que an 6= 0 et bm 6= 0.
On suppose que pour tout nombre réel x :

a0 + a1 x + a2 x2 + · · · an xn = b0 + b1 x + · · · + bm xm .

Alors m = n et ai = bi pour tout 0 6 i 6 m.


Démonstration. Soit P(x) = a0 + a1 x + a2 x2 + · · · an xn − (b0 + b1 x + · · · + bm xm ), qui est un
polynôme à coefficients réels tel que P(x) = 0 pour tout x ∈ R. Le résultat découle alors du
théorème précédent.

2 Division euclidienne et racines


Dans cette partie, notre but est d’expliquer en quoi la connaissance des racines d’un po-
lynôme P, c’est-à-dire des éléments x tels que P(x) = 0, donne des informations sur P. On
commence par montrer qu’il existe une notion de division euclidienne de polynômes très si-
milaire à celle des entiers.

2.1 Division euclidienne de polynômes


Ici, et dans tout ce qui suit, K désigne Q ou R.
Théorème 10. Soient P, U ∈ K[X] avec deg U > 1. Alors il existe un unique couple de poly-
nômes Q, R ∈ K[X] tels que :

P = QU + R et deg(R) 6 deg(U) − 1.

3
Démonstration. Pour l’existence, on adapte en quelque sorte l’algorithme de division eucli-
dienne pour les nombres entiers aux polynômes. Plus précisément, on pose P0 = P et Q0 = 0.
On commence à l’étape 0 et voici ce qu’on fait à l’étape k : notons dk le degré de Pk et ck son
coefficient dominant. Notons également n le degré de U et un son coefficient dominant. Si
deg(Pk ) 6 deg(U) − 1, on arrête l’algorithme en prenant Q = Qk et R = Pk . Sinon, on pose :
ck dk −n ck dk −n
Pk+1 = Pk − X U et Qk+1 = Qk + X .
un un
On passe ensuite à l’étape k + 1. L’algorithme se termine bien car le degré de Pk est au plus
deg P − k, et les polynômes Q et R donnés par l’algorithme vérifient les conditions requises.
Pour l’unicité, supposons par l’absurde qu’il existe deux tels couples Q, R et Q 0 , R 0 . Alors
QU + R = Q 0 U + R 0 . En particulier, Q 6= Q 0 , car sinon on a aussi R = R 0 . Cela implique
également :
U(Q − Q 0 ) = R 0 − R.
Or, d’après la proposition 6, le degré du terme de gauche et supérieur ou égal à celui de U
et celui de droite est inférieur ou égal à deg(U) − 1, ce qui est contradictoire et conclut la
démonstration.
Exemple 11. La division euclidienne de X5 − 3X3 + 2X + 1 par X3 + 3X2 − 2X − 1 est :

X5 − 3X3 + 2X + 1 = (X2 − 3X + 8)(X3 + 3X2 − 2X − 1) + −29X2 + 15X + 9 .




Remarque 12. La division euclidienne telle quelle est fausse pour des polynômes à coefficients
entiers. Par exemple, il n’existe pas de Q ∈ Z[X] tel que 3x2 + 1 = Q(x)(2x + 1) (comparer les
coefficients dominants). En revanche, en reproduisant la démonstration précédente, si P, U ∈
Z[X] et que le coefficient dominant de U est 1, alors si deg U > 1, il existe il existe un unique
couple de polynômes Q, R ∈ K[X] tels que :

P = QU + R et deg(R) 6 deg(U) − 1.

En effet, dans la preuve précédente, il a fallu diviser par « un ». Or, lorsqu’on divise par des
éléments de Z, on ne reste pas dans Z. Ceci explique un peu d’ailleurs pourquoi la théorie des
polynômes à plusieurs variables est plus compliquée que celle des polynômes à une variable.
En effet, on peut par exemple voir les polynômes réels à deux variables comme les polynômes
en y à coefficients dans R[X]. Mais, de même que dans Z, tous les éléments de R[X] ne sont
pas inversibles.
Définition 13. Soient P, Q ∈ K[X] avec P non nul. On dit que P divise Q s’il existe R ∈ K[X] tel
que Q = PR.
Ainsi, P divise Q si le reste de la division euclidienne de Q par P vaut 0.
Exemple 14. Trouvons le reste de la division euclidienne de A(x) = x2013 + 2013 par B(x) =
x − 1. Par division euclidienne, on écrit A(x) = Q(x)B(x) + R(x) avec R(x) un polynôme de
degré au plus 0. Ainsi R est un polynôme constant qu’on notera c. Autrement dit, A(x) =
Q(x)B(x) + c et il nous reste à trouver la valeur de c. Prenons x = 1 : A(1) = Q(1)B(1) + c. Or
B(1) = 1. On en déduit que c = A(1) = 2014.
Exercice 1 Trouver le reste de la division euclidienne de x100 − 2x51 + 1 par x2 − 1.

4
2.2 Racines et factorisation de polynômes
Nous voyons ici que la connaissance des racines d’un polynôme permet de le factoriser.
Rappelons que K désigne R ou Q.

Définition 15. Un élément x ∈ K est appelé racine d’un polynôme P ∈ K[X] si P(x) = 0.

Exemple 16. Le polynôme réel X2 − 1 a deux racines réelles, qui sont 1 et −1. Le polynôme
X2 +1 n’a pas de racine réelle. Le polynôme réel X2 −2 a deux racines
√ réelles, mais le polynôme
2
à coefficients rationnels X − 2 n’a pas de racines rationnelles car 2 est irrationnel. Si a ∈ K,
le polynôme (X − 1)2012 est de degré 2012 mais n’a qu’une seule racine qui est 1.

Le théorème suivant est très important et doit être connu.

Théorème 17. Soient P ∈ K[X] et a ∈ K. Les deux propositions suivantes sont équivalentes :
1. a est racine de P, autrement dit P(a) = 0.
2. Il existe un polynôme Q ∈ K[X] tel que :

P(x) = Q(x)(x − a).

Démonstration. Il est clair que le deuxième point implique le premier. Quant à la réciproque,
le point clé est d’utiliser la division euclidienne. En effet, supposons que P(a) = 0. Écrivons
alors la division euclidienne de P par X − a sous la forme P(x) = Q(x)(x − a) + R(x) avec R
un polynôme de degré au plus 1 − 1 = 0. Ainsi, R est un nombre réel, noté c. Bref, P(x) =
Q(x)(x − a) + c. Évaluons cette quantité en x = a : 0 = P(a) = Q(a)(a − a) + c. Donc c = 0,
ce qu’on voulait démontrer.

Théorème 18. Soit n > 0 un entier. Un polynôme de K[X] de degré n a au plus n racines
différentes dans K.

Démonstration. On raisonne par récurrence sur n. Pour n = 0, c’est vrai car par définition un
polynôme de degré 0 est une constante non nulle. Soit n > 1 et supposons le résultat acquis
pour tous les polynômes de K[X] degré n − 1. Soit alors P ∈ K[X] de degré n. Si P n’a pas de
racines dans K, il n’y a rien à faire. Sinon, soit a ∈ K une racine de P. D’après le théorème
précédent, on peut écrire P(X) = (X − a)R(X) avec R ∈ K[X] un polynôme de degré n − 1, qui
par hypothèse de récurrence a au plus n − 1 racines différentes. On en déduit que P a au plus
n racines différentes.

Remarque 19. Il existe des polynômes qui n’ont pas de racines réelles, par exemple P(x) =
x4 + 1. En revanche, un polynôme P à coefficients réels et de degré impair a au moins une
racine réelle. En effet, soit c son coeffictient dominant. Alors P(x) → +∞ quand x → ∞ et
P(x) → −∞ quand x → −∞ lorsque c > 0 et P(x) → −∞ quand x → ∞ et P(x) → +∞ quand
x → −∞ lorsque c < 0. Le polynôme P doit donc forcément couper quelque part l’axe des
abcsisses (en termes rigoureux, c’est une conséquence du théorème des valeurs intermédiaires
appliqué à la fonction continue P).

Ce théorème important implique quelques corollaires donnant une information concer-


nant le polynôme sachant quelque chose sur ses racines.

5
Corollaire 20. Soit n > 0 un entier. Soit P ∈ K[X] un polynôme de degré au plus n ayant
au moins n + 1 racines. Alors P est le polynôme nul. En particulier, un polynôme ayant une
infinité de racines est forcément le polynôme nul.

Démonstration. Il suffit de dire que si P n’est pas le polynôme nul, alors d’après le théorème
précédent il a au plus deg(P) 6 n racines.

Corollaire 21. Soit P(x) = a0 + a1 x + · · · + an xn ∈ K[X] un polynôme de degré n. On suppose


qu’il a n racines différentes r1 , . . . , rn ∈ K. Alors :

P(x) = an (x − r1 ) · · · (x − rn ).

Démonstration. Soit Q(X) = P(x)−an (x−r1 ) · · · (x−rn ) ∈ K[X]. Ce polynôme admet au moins
n racines différentes (r1 , . . . , rn ), et on voit qu’il est de degré au plus n − 1 (le terme an xn se
simplifie dans la soustraction). D’après le corollaire précédent, Q est le polynôme nul.

Corollaire 22. Un polynôme de degré n ayant n + 1 racines est nul. Ainsi, un polynôme ayant
une infinité de racines est forcément le polynôme nul.

Exercice 1 En utilisant le corollaire précédent, retrouver le fait que Q(x) = |x| n’est pas un
polynôme.

Exemple 23. Soit P un polynôme de degré 2013 vérifiant P(k) = k pour k = 1, 2, . . . , 2013 et
P(0) = 1. Trouvons P(−1).
Le polynôme P(x) − x est de degré 2013 et admet 2013 racines qui sont k = 1, 2, . . . , 2013.
On a donc forcément
P(x) − x = c · (x − 1)(x − 2) · · · (x − 2013)
avec c un nombre réel. En évaluant en x = 0, il vient 1 = P(0) = −c · 2013!, de sorte que
c = −1/2013!. D’où
2014!
P(−1) = −1 + = 2013.
2013!
Pour conclure cette partie, prouvons la propriété utile suivante en identifiant les coeffi-
cients.

Proposition 24. Soit P un polynôme tel que P(x)2 soit un polynôme en x2 (c’est-à-dire qu’il
existe un polynôme R tel que P(x)2 = R(x2 ). Alors il en est de même de P(x) ou de P(x)/x
(c’est-à-dire qu’il existe un polynôme Q tel que soit P(x) = Q(x2 ), soit P(x) = xQ(x2 )).

Démonstration. Écrivons P(x) = an xn + an−1 xn−1 + · · · + a1 x + a0 avec an 6= 0. Comme


P(x)2 = R(x2 ), le coefficient devant x2n−1 dans P(x)2 , à savoir 2an an−1 , est nul. On en déduit
que an−1 = 0. De même, le coefficient devant x2n−3 dans P(x)2 , à savoir 2an an−3 , est nul. On
en déduit que an−3 = 0. De même, on obtient que an−2k−1 = 0 pour n − 2k − 1 > 0. Le résultat
en découle.
Pour illustrer cette propriété, on pourra chercher l’exercice suivant.
Exercice 2 Trouver tous les polynômes P ∈ R[X] tels que 16P(x2 ) = P(2x)2 .

6
2.3 Racines multiples et polynôme dérivé
Doit-on dire que le polynôme P(x) = (x − 1)n a une seule racine, ou bien n racines qui sont
les mêmes ? Pour ne pas faire de confusion, nous traitons le cas des racines multiples.
Définition 25. Soient P ∈ K[X], α ∈ K et un entier m ∈ N∗ . On dit que α est racine de multi-
plicité m de P s’il existe Q ∈ K[X] tel que P(x) = (x − α)m Q(x) et Q(α) 6= 0.
Il se trouve qu’on dispose d’un critère assez pratique permettant de reconnaître une racine
multiple.
Définition 26. Soit P = a0 + a1 x + · · · + an xn ∈ K[X]. On définit le polynôme dérivé P 0 par
P 0 (x) = a1 + 2a2 x + · · · + nan xn−1 .
La proposition suivante, réminiscente des propriétés de l’opérateur de dérivation sur les
fonctions réelles dérivables, est fondamentale. On laisse sa démonstration au lecteur.
Proposition 27. Pour P, Q ∈ K[X], on a :

(PQ) 0 = PQ 0 + P 0 Q.

Proposition 28. Soient a ∈ K et n > 1 un entier. Soit P(x) = (x − a)n . La dérivée de P est
P 0 (x) = n(x − a)n−1 .
Démonstration. Prouvons cela par récurrence sur n. Pour n = 1, le polynôme dérivé de x − a
est bien 1. Supposons le résultat acquis au rang n, montrons-le au rang n + 1. Soit Q(x) =
(x − a)n+1 . En écrivant (x − a)n+1 = (x − a)(x − a)n , on obtient

Q 0 (x) = (x − a)n + (x − a) ((x − a)n ) 0 .

Donc par hypothèse de récurrence, Q 0 (x) = (x − a)n + (x − a) · (n − 1)(x − a)n = n(x − a)n .
Ceci conclut la récurrence et la preuve de la proposition.
Théorème 29. Soit P ∈ K[X] et α ∈ K tel que P(α) = 0. Alors α est une racine multiple de P si,
et seulement si, P 0 (α) = 0.
Démonstration. Dans le sens direct, écrivons P(x) = (x − α)m Q(x) avec m > 2 et Q ∈ K[X].
En dérivant cette expression, il vient P 0 (x) = m(x − α)m−1 Q(x) + (x − α)m Q 0 (x). En prenant
x = α, on conclut que P 0 (α) = 0.
Pour la réciproque, supposons que P 0 (α) = 0 et raisonnons par l’absurde en supposant
que α soit une racine non multiple de P. Alors P s’écrit P(x) = (x − α)Q(x) avec Q(α) 6= 0 (si
Q(α) = 0, d’après le théorème 17, on pourrait écrire P(x) = (x − α)2 R(X)). En dérivant cette
expression, il vient P 0 (x) = Q(x) + (x − α)Q 0 (x). En prenant x = α, il vient P 0 (α) = Q(α) 6= 0,
ce qui est absurde.
Exemple 30. Soit n > 1 un entier et montrons que (X + 1)2 divise P(X) = X4n+2 + 2X2n+1 + 1.
D’après le théorème 17, il suffit que −1 est racine double de P. Ceci découle aisément du fait
que P(−1) = 0 et P 0 (−1) = 0.
Remarque 31. Si P 0 (α) = 0, cela n’implique pas que α soit racine multiple (ou racine tout
court !) de P. Il faut en effet s’assurer que P(α) = 0 pour utiliser le corollaire précédent. Par
exemple, si P(x) = x2 − 2, on a P 0 (x) = 2x, mais 0, bien que racine de P 0 , n’est pas racine de P.

7
Remarque 32. Soient P ∈ K[X] et a ∈ K. Pour un entier k > 1, notons P(k) le polynôme P
dérivé k fois. Soit n > 1 un entier. Plus généralement, on peut démontrer par récurrence sur
n l’équivalence

P(a) = 0, P 0 (a) = 0, . . . , P(n) (a) = 0 ⇐⇒ (x − a)n divise P.

2.4 Exercices d’application


Exercice 1 Trouver les réels a, b tels que (x − 1)2 divise ax4 + bx3 + 1.
Exercice 2 Trouver tous les polynômes P ∈ R[X] tels que pour tous réels x, P(2x) = P 0 (x)P 00 (x).
Exercice 3 Soit P(x) = a0 + a1 x + · · · + an xn ∈ R[X] qui possède n racines réelles différentes.
Montrer que pour tout x réel, P(x)P 00 (x) 6 P 0 (x)2 . En déduire que pour 1 6 k 6 n − 1,
ak−1 ak+1 6 a2k .
Exercice 4 (Oral ENS 2009) Soit P ∈ R[X] de degré n > 1. On suppose que toutes les racines
de P sont réelles. Montrer que (n − 1) (P 0 (x))2 > nP(x)P 00 (x) et déterminer les cas d’égalité.

2.5 Interpolation
Étant donnés un nombre fini de points du plan, existe-t-il un polynôme tel que que sa
courbe représentative passe par ces points ? Trouver un tel polynôme, c’est résoudre un pro-
blème d’interpolation.
Théorème 33. Soient a1 , . . . an et b1 , . . . , bn des nombres réels (avec les ai deux à deux dis-
tincts). Alors il existe un unique polynôme P de degré n − 1 tel que pour tout i, P(ai ) = bi .
Démonstration. Montrons d’abord l’unicité en considérant P, Q deux polynômes vérifiant les
conditions de l’énoncé du théorème. Alors le polynôme P − Q est de degré au plus n − 1, qui
admet au moins n racines différentes, à savoir a1 , . . . , an . Il est donc nécessairement nul.
Quant à l’existence, pour 1 6 i 6 n, introduisons les polynômes suivants, appelés poly-
nômes d’interpolation de Lagrange :

Y
n
x − aj
Li (x) = .
j=1,j6=i
a i − aj

L’intérêt est que pour tout j différent de i, Li (aj ) = 0, alors que Li (ai ) = 1. On en déduit
aisément que le polynôme :
Xn
P(x) = bi Li (x)
i=1

convient.
Ainsi, un polynôme de degré n est complètement déterminé par les images de n + 1 points
distincts.

Exercice 1 Soient a1 , . . . an et b1 , . . . , bn des éléments de K (avec les ai deux à deux distincts).


Trouver tous les polynômes P ∈ K[X] tels que P(ai ) = bi .

8
Exercice 2 Trouver tous les polynômes à coefficients complexes P tels que pour tout rationnel
q, P(q) est rationnel.
Exercice 3 On définit les polynômes de Hermite comme suit : H0 = 1 et pour n > 1, Hn (x) =
1
Qn−1
n! k=0 (X − k).
1. Vérifier que pour tout k ∈ Z, Hn (k) ∈ Z.
2. Trouver tous les polynômes P ∈ C[X] tels que pour tout k ∈ N, on a P(k) ∈ Z.
3. (i) Calculer, pour des entiers j 6 k la somme :

Xk   
k−i k i
(−1) .
i=j
i j

Indication. On pourra écrire Xk = (X + 1 − 1)k .


(ii) Soit (uj ) une suite de nombres réels. Montrer que les deux conditions suivantes
sont équivalentes :
1. Il existe P ∈ R[X] tel que, pour tout j ∈ N, on a uj = P(j).
Xi 
i−j i
2. Il existe un entier positif n tel que pour tout entier i > n+1, on a (−1) uj =
j=0
j
0.

2.6 Cas des polynômes à petit degré


Nous maintenant quelques applications des résultats précédents, parfois sous la forme
d’exercice corrigé.

Proposition 34. Soient b, c deux nombres réels. On souhaite connaître le nombre de réels x
tels que x2 + bc + c = 0. Soit ∆ = b2 − 4c, appelé le discriminant. Alors :
1. Si ∆ < 0, il n’y a pas de solution.
2. Si ∆ = 0, il y a une seule solution qui est − b2 .
3. Si ∆ > 0, il y a exactement deux solutions, qui sont :
√ √
−b + b2 − 4c −b − b2 − 4c
et .
2 2

Démonstration. L’idée est de se ramener au cas b = 0 en écrivant x2 + bx + c sous la forme


suivante, dite forme canonique :

b 2 b2
x2 + bx + c = (x + ) +c− .
2 4
L’intérêt réside dans le fait que x n’intervient qu’une fois dans la nouvelle expression. Cette
forme rend très souvent de précieux services et est à retenir. Ainsi, x2 + bx + c = 0 si, et
2 2
seulement si, (x + b2 )2 = b4 − c. Ainsi, un carré étant positif, si b4 − c = ∆/4 < 0, il n’y a pas

9
b2
de solution, d’où le premier point. D’un autre côté, si ∆ > 0, alors (x + b2 )2 = 4
− c si, et
seulement si : r r
b b2 b b2
x+ = −c ou x+ =− − c.
2 4 2 4
On en déduit les points 2. et 3.

Exemple 35. Le polynôme P(x) = x2 + x + 1 a un discriminant égal à −3, et n’a donc pas de
racine réelle.

Exercice 1 Soient a, b, c ∈ R, avec a 6= 0, et considérons le graphe de la fonction P(x) =


ax2 + bx + c. Montrer qu’en faisant une homothétie et une translation, on peut obtenir le
graphe de la fonction Q(x) = x2 .

Remarque 36. Il s’ensuit qu’étant donné un polynôme de degré 2, on peut aisément dire s’il a
des racines réelles, et le cas échéant donner leur expression. Ceci est tout à fait remarquable :
on peut montrer qu’il existe des polynômes de degré 5 dont les racines réelles ne s’expriment
pas en utilisant des racines carrées, cubiques, etc. Cependant, si P(x) est un polynôme de
degré 3 et si on trouve une racine évidente a (par exemple a = 1, 2, −1, −2, ...), alors on peut
effectuer la division euclidienne de P par x − a. On en déduit qu’il existe Q, un polynôme
de degré 2, tel que P(x) = Q(x)(x − a). Mais Q est de degré 2, et ce qui précède s’applique.
La moralité de ceci est que si on trouve une racine évidente d’un polynôme de degré 3, alors
on arrivera à connaître toutes ses racines. à titre d’illustration, on pourra chercher l’exercice
suivant.

Exercice 2 Trouver tous les nombres réels x, y, z vérifiant :




 (x + 1)yz = 12
(y + 1)zx = 4


(z + 1)xy = 4.

3 Polynômes symétriques élémentaires


Dans cette partie, nous nous intéressons aux liens unissant les coefficients d’un polynôme
à ses racines.

3.1 Relations de Viète


Proposition 37 (Relations de Viète). Soit P(x) = ax2 + bx + c un polynôme réel de degré 2
(avec a 6= 0) ayant z1 et z2 comme racines réelles. Alors z1 z2 = ac et z1 + z2 = − ab .

Démonstration. D’après le corollaire 21, on a P(x) = a(x − z1 )(x − z2 ). En développant le terme


de droite et en identifiant les coefficients, on trouve les égalités annoncées.

10
Ces relations sont utiles car elles expriment les coefficients du polynôme en fonction des
racines. à ce titre, on cherchera l’exercice suivant.
Exercice 1 Trouvez toutes les valeurs du paramètre a pour que l’équation
ax2 − (a + 3)x + 2 = 0
admette deux racines réelles de signes opposés.

Définition 38. Soit n > 1 un entier. Soient z1 , . . . , zn ∈ K. Pour 1 6 k 6 n, la k-ième fonction


symétrique élémentaire est par définition
X
σk (z1 , . . . , zn ) = z i1 · · · z ik ,
16i1 <···<ik 6n

Lorsque les éléments z1 , . . . , zn sont sous-entendus, pour simplifier on notera parfois σk au


lieu de σk (z1 , . . . , zn ).
Ainsi, par exemple, pour n = 3, on a σ1 = z1 + z2 + z3 , σ2 = z1 z2 + z1 z3 + z2 z3 et σ3 = z1 z2 z3 .
Proposition 39 (Relations de Viète dans le cas général). Soit P(x) = an xn + an−1 xn−1 + · · · +
a1 x+a0 ∈ K[X] avec an 6= 0. On suppose que P adment n racines z1 , . . . , zn ∈ K comptées avec
multiplicité (c’est-à-dire que si z est racine d’ordre k, z apparaît k fois dans la liste z1 , . . . , zn ).
Alors, pour tout entier 1 6 k 6 n,
an−k
σk (z1 , . . . , zk ) = (−1)k .
an
Autrement dit,
X
n
an−1 X an−2 Yn
a0
zi = − , zi zj = − ,..., zi = (−1)n .
i=1
an 16i<j6n an i=1
an

Démonstration. D’après le Corollaire 21, on a P(x) = an (x − z1 ) · · · (x − zn ). Or, dans le dé-


veloppement de an (x − z1 ) · · · (x − zn ), on voit aisément que le coefficient devant xn−k vaut
(−1)k an σk (z1 , . . . , zk ). Or ce coefficient vaut aussi an−k . Le résultat en découle.
Exemple 40. Cherchons tous les nombres réels x, y, z tels que
x + y + z = 17, xy + yz + xz = 94, xyz = 168.
D’après les relations de Viète, x, y, z sont racines de P(x) = x3 − 17x2 + 94x − 168 = 0.
Cherchons des racines “évidentes” de P. Il est naturel de d’abord chercher des racines entières,
qui sont forcément des diviseurs de 168 (pour tester si par exemple 2 est racine, on effectue la
division euclidienne de P pa x − 2 et on regarde si le reste est nul). On remarque que x = 4 est
racine, et sans difficulté on trouve que x = 6 et x = 7 sont racines du polynôme P(x)/(x − 4).
Ainsi, les solutions (x, y, z) sont les six permutations possibles de (4, 6, 7).
Remarque 41. Les fonctions σ1 (z1 , . . . , zn ), . . . , σn (z1 , . . . , zn ) sont appelés fonctions symétriques
élémentaires des zi . Symétriques, parce qu’une permutation des zi laisse les σk invariants. Élé-
mentaires, parce qu’on peut montrer que toute expression symétrique en n variables peut
s’exprimer polynomialement à l’aide de ces fonctions symétriques élémentaires. Plus précisé-
ment, si P(z1 , . . . , zn ) est un polynôme à n variables (on laisse le lecteur imaginer ce que c’est)
tel que pour toute permutation σ de {1, . . . , n} on ait P(z1 , . . . , zn ) = P(zσ(1) , . . . , zσ(n) ), alors il
existe un polynôme à n variables R tel que P(z1 , . . . , zn ) = R(σ1 (z1 , . . . , zn ), . . . , σn (z1 , . . . , zn )).

11
Exemple 42. En notant σ1 = x1 + x2 + x3 , σ2 = x1 x2 + x1 x3 + x2 x3 et σ3 = x1 x2 x3 , on a :
x31 + x32 + x33 = σ31 − 3σ1 σ2 + 3σ3 .
Bref, lorsqu’on a affaire à des quantités symétriques, il peut être parfois judicieux de faire
intervenir les fonctions symétriques élémentaires associées.

Exercice 2 Soit P ∈ R[X] non nul. Montrer que les sommes des racines complexes de P, P 0 , . . . , P(n−1)
(où P(n−1) désigne le polynôme P dérivé n − 1 fois) forment une suite arithmétique.
Exercice 3 Trouver tous les réels x, y vérifiant x5 + y5 = 33 et x + y = 3.

3.2 Relations de Newton


Nous allons voir que des sommes symétriques particulières (sommes des puissances k-
ièmes) peuvent s’exprimer assez simplement grâce aux fonctions symétriques élémentaires.
Théorème 43 (Relations de Newton). Soit n > 1, et notons σ1 , . . . , σn les fonctions symétriques
élémentaires de (z1 , z2 , . . . , zn ). Notons Sk = σk1 + · · · + σkn pour k > 0. Alors, pour tout entier
k > 1,
Xk−1
(−1)r σr Sk−r + (−1)k kσk = 0, (1)
r=0

avec la convention σ0 = 1 et σr = 0 quand r > n.


Ainsi, à titre d’illustration, pour r = 1, 2, . . . , n :
S1 − σ1 = 0
S2 − σ1 S1 + 2σ2 = 0
S3 − σ1 S2 + σ2 S1 − 3σ3 = 0
..
.
Sn − σ1 Sn−1 + · · · + (−1)n−1 σn−1 S1 + (−1)n nσn = 0
et pour r > n :
Sr − σ1 Sr−1 + σ2 Sr−2 + · · · + (−1)n σn Sr−n = 0.
Remarque 44. Si z1 , . . . , zn sont les racines du polynôme P(x) = an xn + an−1 xn−1 + · · · + a1 x +
a0 , alors pour tout entier k > 0 :
an Sk+n + an−1 Sk+n−1 + an−2 Sk+n−2 + · · · + a0 Sk = 0.
En effet, il suffit d’écrire que zk1 P(z1 ) + · · · zkn P(zn ) = 0. Les formules de Newton sont donc très
facilement établies lorsque k > n.
Par ailleurs, en réécrivant le polynôme P sous la forme P(x) = xn +b1 xn−1 +· · ·+bn−1 x+bn
(attention aux indices !), les relations de Viète donnent bj = (−1)j σj , de sorte que les relations
de Newton s’écrivent aussi, pour tout entier k > 1,

X
k−1
br Sk−r + kbk = 0.
r=0

12
Preuve des relations de Newton, qui peut être sautée en première lecture. Nous avons déjà traité le
cas k > n plus haut et pouvons donc supposer que k < n. La preuve qui suit est due à Doron
Zeilberger. Considérons A = A(n, k) l’ensemble des couples (A, j(l) ), où :
(i) A est un sous-ensemble de {1, 2, . . . , n},
(ii) j appartient à {1, 2, . . . , n},
(iii) |A| + l = k, où |A| est le nombre d’éléments de A,
(iv) l > 0, et si l = 0 alors j ∈ A.
On définit ensuite le poids w(A, j(l) ) de (A, j(l) ) par la formule
Y
!
w(A, j(l) ) = (−1)|A| za zlj .
a∈A

Par exemple, w({1, 3, 5}, 2(3) ) = (−1)3 z1 z3 z5 · z32 = −z1 z32 z3 z5 . On voit aisément que la somme
des poids de tous les éléments de A est égale au terme de gauche de (1).
Prouvons maintenant que cette somme est nulle. à cet effet, considérons l’application T :
A → A définie par : 
(A\{j}, j(l+1) ), j ∈ A,
T (A, j(l) ) =
(A ∪ {j}, j (l−1)
), j 6∈ A.
Cette application vérifie w(T (A, j(l) )) = −w(A, j(l) ) et est une involution (i.e. T composé avec
elle-même donne l’identité). On peut donc assembler tous les poids par paires de sorte que
chaque paire contienne un poids et son opposé. La somme de tous les poids est donc bien
nulle, ce qui conclut la preuve.
Mentionnons qu’il est également possible de procéder par récurrence sur n − k pour prou-
ver les relations de Newton.
Exemple 45. Soient x, y, z des nombres réels tels que x + y + z = 1, x2 + y2 + z2 = 3 et
x3 + y3 + z3 = 7. Trouvons la valeur de x5 + y5 + z5 .
À cet effet, notons Sk et σk respectivement les sommes des puissances k-ièmes et la k-ième
fonction symétrique élémentaire de x, y, z. Les relations de Newton donnent
S1 − σ1 = 0, S2 − σ1 S1 + 2σ2 = 0, S3 − σ1 S2 + σ2 S1 − 3σ3 = 0.
On en tire que σ1 = 1, σ2 = −1 et σ3 = 1. D’après les relations de Viète, x, y, z sont donc les
solutions de t3 − t2 − t − 1 = 0. Ainsi, d’après la Remarque 44, nous avons
Sk+3 = Sk+2 + Sk+1 + Sk
pour k > 0. Il s’ensuit que S4 = 1 + 3 + 7 = 11 puis que S5 = 3 + 7 + 11 = 21.
Exercice 1 Soient x et y deux nombres non nuls tels que x2 + xy + y2 = 0 (x et y sont des
nombres complexes, mais ce n’est pas trop grave). Trouver la valeur de
 2013  2013
x y
+ .
x+y x+y

Exercice 2 Trouver tous les nombres réels x, y, z tels que


x + y + z = 3, x2 + y2 + z2 = 3, x3 + y3 + z3 = 3.

13
4 Polynômes à coefficients entiers
Nous présentons maintenant quelques spécificités des polynômes à coefficients entiers :
? Nous avons déjà vu que si P, Q ∈ Z[X] et que deg Q > 1, on peut toujours effectuer la
division euclidienne de P par Q à condition que le coefficient dominant de Q soit égal à
1.
? Une propriété extrêmement utile est que si P ∈ Z[X], alors pour tous entiers a 6= b, a − b
divise P(a) − P(b). Ceci est une simple conséquence du fait que a − b divise an − bn
pour n > 1.
? Si P(x) = an xn + an−1 xn−1 + · · · + a1 x + a0 ∈ Z[X], et si p/q est une racine rationnelle
de P sous forme irréductible, alors p divise a0 et q divise an . Ce simple fait permet de
restreindre la recherche des racines rationnelles d’un polynôme à coefficients entiers.
? Un polynôme P ∈ Q[X] vérifie P(k) ∈ Z pour tout k ∈ Z si et seulement si il existe des
entiers a0 , a1 , . . . an tels que
     
x x x
P(x) = a0 + a1 + · · · + an ,
0 1 n

où on note kx = x(x−1)···(x−k+1) x
 
k!
si k 6
= 0 et 0
= 1. Cette propriété découle de l’Exercice
3 (2).
Exercice 3 Soient a, b, c des entiers différents. Montrer qu’il n’existe pas de polynôme P à
coefficients entiers tel que P(a) = b, P(b) = c et P(c) = a.
La factorisation par a − b du polynôme P(a) − P(b) peut être utile dans d’autres cas éga-
lement, voir par exemple l’exercice suivant.
Exercice 4 Soient P, Q ∈ R[X] deux polynômes unitaires tels que P(P(x)) = Q(Q(x) pour tout
réel x. Prouver que P(x) = Q(x).

5 Polynômes à coefficients réels


Nous considérons dans cette partie des polynômes à coefficients réels et voyons comment
des résultats de la théorie de l’analyse réelle s’appliquent dans notre cas. Nous utiliserons la
propriété suivante :
Proposition 46. Soit P ∈ R[x] un polynôme, et a, α, b, β des réels tels que a < b, P(a) = α et
P(b) = β. Alors pour tout réel γ compris entre α et β, il existe un réel c tel que a 6 c 6 b et
P(c) = γ.
En particulier, si a < b sont tels que P(a) < 0 et P(b) > 0, alors il existe au moins une
racine réelle de P dans l’intervalle ]a, b[.
La proposition précédent est un cas particulier du théorème des valeurs intermédiaires,
valable plus généralement pour toute fonction continue à valeurs réelles.
On en déduit le résultat intéressant suivant :
Proposition 47. Soit P ∈ R[X] un polynôme de degré impair. Alors P admet au moins une
racine réelle.

14
Démonstration. Écrivons P(x) = an xn + · · · + a1 x + a0 avec an 6= 0 et n > 1. Supposons en
premier lieu que an > 0 et rouvons tout d’abord que P(x) → ∞, lorsque x → ∞, c’est-à-dire
que pour tout M > 0, il existe x0 tel que pour tout x > x0 on ait P(x) > M. à cet effet, écrivons
 
n an−1 1 a0 1
P(x) = an x 1 + · + ··· + · .
an x an xn
La somme à l’intérieur de la parenthèse converge vers 1 lorsque x → ∞, et le terme an xn
diverge vers +∞ lorque x → ∞. Le résultat en découle. Lorsque x → ∞, on prouve de même
que P(x) → −∞. On en déduit qu’il existe a < 0 < b tels que P(a) < 0 et P(b) > 0. Le résultat
en découle d’aprs la proposition 46.
Si an < 0, le raisonnement est similaire, mais cette fois-ci P(x) → −∞ quand x → ∞ et
P(x) → ∞ lorsque x → −∞.
Exercice 5 Trouver tous les polynômes P à coefficients réels tels que pour tout réel x > 0 on
ait :  
P(x)P 1 6 1.

x

6 Polynômes à coefficients complexes


6.1 Nombres complexes
Nous avons vu qu’il existait des polynômes de R[X] qui ne possédaient pas de racines
réelles. Un des intérêts de l’introduction des nombres complexes (et c’est dans cette optique
qu’ils ont été introduits au XVI-ième siècle) est de pallier cette difficulté via le théorème de
d’Alembert-Gauss (énoncé par d’Alembert et démontré par Gauss).
Définition 48. Notons C l’ensemble des couples de nombres réels (a, b) munis :
1. de l’addition suivante : (a, b) + (c, d) = (a + c, b + d),
2. des multiplications suivantes : (a, b)×(c, d) = (ac−bd, ad+bc) et pour λ réel, λ(a, b) =
(λa, λb).
Nous voyons l’ensemble des nombres réels plongés dans l’ensemble des nombres complexes :
à chaque réel a, on peut associer le nombre complexe (a, 0). Notons enfin i le nombre com-
plexe (0, 1). Ainsi, nous pouvons représenter chaque nombre complexe (a, b) sous la forme
(a, b) = a(1, 0) + b(0, 1) = a + ib.
Remarque 49. Avec les règles de multiplication précédentes, on voit que i2 = −1, et que
(a + bi)(c + di) = ac − bd + (ad + bc)i. Ainsi, tout se passe comme si i était un « nombre » tel
que i2 = −1 dans toutes les manipulations. En particulier, i est racine du polynôme X2 + 1 = 0.
Remarque 50. Tout élément non nul de C possédant un inverse, les résultats des sections
précédentes sont aussi valables pour K = C.
Exercice 1 Pour quels entiers n > 1 le polynôme 1 + x2 + x2 + · · · + x2n−2 est-il divisible par
le polynôme 1 + x + x2 + · · · + xn−1 ?

15
6.2 Théorème de d’Alembert-Gauss
Nous admettons le théorème (qu’on appelle aussi théorème fondamental de l’algèbre) sui-
vant :

Théorème 51. Tout polynôme non constant de C[X] possède au moins une racine. On dit que
C est algébriquement clos.

Par une récurrence sur le degré, on en déduit :

Corollaire 52. Soit P ∈ C[X]. Alors P peut s’écrire sous la forme :

P(x) = c(x − α1 )m1 · · · (x − αk )mk ,

où c, α1 , . . . , αk sont des nombres complexes et m1 , . . . , mk sont des entiers strictement positifs.

Nous définissons finalement la conjugaison complexe, qui sera utile lorsque nous vou-
drons déterminer les polynômes irréductibles de R[X].

Définition 53. Soit z = a + bi ∈ C. On définit son conjugué z par z = a − bi.

Proposition 54. Pour tous w, z ∈ C, on a wz = w · z.

Démonstration. Exercice.

7 Arithmétique de K[X]
De même que dans le cas des nombres entiers, la division euclidienne entre polynômes
permet de démontrer le théorème de Bézout, et par voie de conséquence de définir la notion
de PGCD et d’avoir accès au lemme de Gauss. Les démonstrations étant similaires au cas des
entiers, nous ne les reproduisons pas. Dans tout ce qui suit, K = Q, R ou C.

7.1 Théorème de Bézout dans K[X]


Définition 55. Soient P, Q ∈ K[X]. Lorsque P est non nul, on rappelle que P divise Q s’il existe
R ∈ K[X] tel que Q = PR. On dit que P et Q sont premiers entre eux s’ils n’ont comme diviseurs
communs (dans K[X]) que les constantes non nulles. Nous utilisons aussi ces définitions dans
le cas de Z[X].

Remarque 56. La définition précédente laisse penser que la notion de primalité entre deux
polynômes dépend de l’ensemble choisi pour ses coefficients : ainsi, a priori, rien n’empêche
que deux polynômes à coefficients entiers soient premiers entre eux lorsqu’ils sont vus comme
éléments de Q[X], mais qu’ils ne le soient plus lorsqu’on les voit comme éléments de C[X].

Théorème 57 (Bézout). Soient P, Q ∈ K[X]. Alors P et Q sont premiers entre eux si, et seule-
ment si, il existe U, V ∈ K[X] tels que PU + QV = 1.

Exercice 1 Soit x ∈ R. Les énoncés suivants sont-ils vrais ou faux ?


a. Si x7 et x12 sont rationnels, alors x est rationnel.

16
b. Si x9 et x12 sont rationnels, alors x est rationnel.

Corollaire 58. Si P, Q ∈ Q[X] sont premiers entre eux, alors, vus comme éléments de R[X], ils
sont premiers entre eux.

Démonstration. D’après le théorème de Bézout, il existe U, V ∈ Q[X] tels que PU + QV = 1. A


fortiori, U, V ∈ R[X], donc, d’après la réciproque du théorème de Bézout, P et Q sont premiers
entre eux vus comme éléments de R[X].
Du théorème de Bézout on déduit le théorème de Gauss.

Théorème 59. Si P, Q, R ∈ K[X] sont tels que P soit premier avec Q et P divise QR, alors P
divise R.

7.2 Polynômes irréductibles de K[X]


Les polynômes irréductibles jouent le rôle des nombres premiers : ce sont en quelque sorte
les briques de base lorsqu’on souhaite factoriser des polynômes.

Définition 60. Un polynôme P ∈ K[X] est dit irréductible dans K[X] si P n’est pas constant et
si ses seuls diviseurs dans K[X] sont les constantes et les polynômes proportionnels à P non
nuls, ou, de manière équivalente, s’il n’existe pas Q, R ∈ K[X] avec deg Q > 1 et deg R > 1.

On en déduit l’équivalent du théorème de factorisation en nombre premiers.

Théorème 61. Tout polynôme de K[X] se décompose de manière unique, à l’ordre des facteurs
près, sous la forme :
P = cP1k1 P2k2 · · · Pkαk ,
où c ∈ K∗ , ki ∈ N∗ et les Pi sont des polynômes distincts unitaires et irréductibles dans K[X].

À titre d’exercice nous laissons la preuve de ce théorème (très proche de son équivalent
pour les nombres entiers).
Le théorème précédent nous invite à chercher les polynômes irréductibles de C[X], R[X], Q[X].
Nous commeno̧ns par une proposition générale.

Proposition 62. Un polynôme P ∈ K[X] de degré 2 ou 3 est irréductible si, et seulement si, il
n’a pas de racine.

Démonstration. Exercice.

Proposition 63. Les polynômes irréductibles de C[X] sont les polynômes de premier degré.

Démonstration. Il est clair que ces polynômes sont bien irréductibles. Réciproquement, si P ∈
C[X] de degré au moins 2 est irréductible, d’après le théorème de d’Alembert-Gauss, il peut
s’écrire P(x) = (x − α)Q(x), se qui contredit son irréductibilité.
Passons maintenant à l’étude des polynômes à coefficients réels.

17
Proposition 64. Tout polynôme P ∈ R[X] se décompose sous la forme :

Y
r Y
s
P(x) = c (x − αi ) (x2 + ai x + bi ).
i=1 i=1

En conséquence, les polynômes irréductibles de R[X] sont les polynômes de premier degré et
ceux du second degré à discriminant négatif.

Démonstration. D’après le corollaire 52, on peut écrire P(x) = c(x − α1 ) · · · (x − αn ), où les


αi sont complexes. En utilisant la proposition 54, on voit que si α est racine de P, alors α
est également racine de P. En effet, si P(x) = a0 + a1 x + · · · + an xn avec les ai réels, on
a 0 = P(α) = a0 + a1 x + · · · + an xn = a0 + a1 x + · · · + an xn = a0 + a1 x + · · · + an xn .
Dans l’expression donnant P sous forme factorisée, on regroupe alors par pairs les racines
complexes (non réelles) avec leurs conjugués. En remarquant que pour un nombre complexe
z, (x − z)(x − (z)) = x2 + ax + b, avec a, b ∈ R tels que a2 − 4b 6 0, on conclut.
Le raisonnement précédent montre qu’un polynôme irréductible de R[X] est un polynôme
de premier degré ou du second degré de discriminant négatif. Réciproquement, de tels poly-
nômes sont irréductibles en vertu de la proposition 62.

Exercice 1 Soient P, Q ∈ R[X] deux polynômes non nuls tels que pour tout réel x, P(x2 + x +
1) = P(x)Q(x). Montrer que P est de degré pair. Peut-on trouver de tels polynômes ?
Exercice 2 Soit P un polynôme à coefficients réels tel que P(x) > 0 pour tout réel x. Montrer
qu’il existe deux polynômes Q, R ∈ R[X] tels que P = Q2 + R2 .

7.3 Polynômes irréductibles à coefficients entiers ou rationnels


Dans le cas de Q[X], il n’y a pas de caractérisation satisfaisante des polynômes irréduc-
tibles (essentiellement parce que des propriétés arithmétiques de Z rentrent en jeu). On peut
toutefois donner quelques méthodes de recherche de racines et des critères d’irréductibilité.

Proposition 65. Soit P(x) ∈ Q[X] et cherchons ses racines rationnelles. Quitte à multiplier P par
le ppcm des dénominateurs de ses coefficients, on peut supposer que P(x) = an xn + · · · + a0 ∈
Z[X]. Soit p/q une racine rationnelle de P. Alors p divise a0 et q divise an .

Démonstration. Il suffit d’écrire P(p/q) = 0, de réduire au même dénominateur et d’utiliser le


lemme de Gauss pour les entiers.
Venons-on à l’irréductibilité.

Remarque 66. En vertu de la remarque 62, on peut en pratique vérifier si un polynôme de


degré 2 ou 3 à coefficients entiers ou rationnels est irréductible.

Exemple 67. Le polynôme x3 + x2 − 2x − 1 est irréductible dans Q[X] puisqu’il est sans racine
dans Q.

On commence par introduire le contenu d’un polynôme afin de montrer que les irréduc-
tibles de Z[X] sont irréductibles dans Q[X], ce qui n’est pas évident a priori.

18
Définition 68. Soit P ∈ Z[X] non nul. On appelle contenu de P et on note c(P) le pgcd de ses
coefficients (au signe près).
Exemple 69. Par exemple, c(−6x6 + 3x5 + 27x − 90) = 3.
Lemme 70. Pour P, Q ∈ Z[X] non nuls, c(PQ) = c(P)c(Q) au signe près.
Démonstration. Montrons d’abord le résultat lorsque c(P) = c(Q) = 1. Raisonnons par l’ab-
surde que c(PQ)P
P 6= 1 en considérant P un nombre premier p divisant c(PQ). Écrivons P(x) =
i i i
a
i i x , Q(x) = b
i i x , P(x)Q(x) = i ci x . Comme c(P) = c(Q) = 1, il existe i0 , j0 ∈ N tels
que :

∀i < i0 , p|ai mais p 6 |ai0


∀j < j0 , p|bj mais p 6 |bj0 .

Par hypothèse, on a :
X X
p|ci0 +j0 = ai bi = ai0 bj0 + ai bj .
i+j=i0 +j0 i+j=i0 +j0
i<i0 ou j<j0

Mais alors p divise ai0 bj0 , ce qui est absurde.


Dans le cas général, notons P 0 = P/c(P), Q 0 = Q/c(Q) de sorte que c(P 0 ) = c(Q 0 ) = 1.
Ainsi, c(P 0 Q 0 ) = 1. Or c(P 0 Q 0 ) = c(PQ)/c(P)c(Q), d’où le résultat.
On en déduit également le résultat suivant.
Proposition 71. Soit P ∈ Z[X]. Alors P est irréductible dans Z[X] si, et seulement, si P est
irréductible dans Q[X] et c(P) = 1.
Démonstration. Si P est irréductible dans Q[X] et c(P) = 1, il est clair qu’il l’est dans Z[X]. Ré-
ciproquement, supposons P irréductible dans Z[X] (ce qui implique c(P) = 1) et par l’absurde
supposons qu’il n’est pas irréductible dans Q[X]. Écrivons alors P = QR avec Q, R ∈ Q[X]
unitaires de degré au moins 1. Écrivons Q(x) = ab Q 0 (x) avec Q 0 ∈ Z[X], c(q 0 ) = 1 et a, b
entiers premiers entre eux. De même, écrivons R(x) = dc R 0 (x) avec R 0 ∈ Z[X], c(R 0 ) = 1
et c, d entiers premiers entre eux. Alors bdP(x) = acQ 0 (x)R 0 (x). Comme c(P) = 1, il vient
bd = c(bdP(x)) = c(acQ 0 R 0 ) = ac (au signe près). Ainsi, P = QR = bd ac
Q 0 R 0 = Q 0 R 0 (au signe
0 0
près) avec Q , R ∈ Z[X]. Ceci contredit l’irréductibilité de P dans Z[X].
De manière un peu similaire, on démontre la proposition suivante, parfois utile.
Proposition 72. Soit P, Q ∈ Q[X] unitaires tels que R = PQ ∈ Z[X]. Alors P et Q sont à
coefficients entiers.
Démonstration. Notons u (resp. v) le ppcm des dénominateurs des coefficients de P (resp. Q).
Alors uvR = uvPQ = (uP)(vQ). Donc c(uvR) = c(uP)c(vQ) d’après le lemme précédent. Or,
comme P et Q sont unitaires, c(uP) = c(vQ) = 1 et c(uvR) > uv. On en déduit que u = v = 1
et donc que P, Q ∈ Z[X].
Exercice 1 Soit P(x) = ax3 + bx2 + cx + d ∈ Z[X] avec ad impair et bc pair. On suppose que P a
toutes ses racines réelles. Montrer qu’au moins une racine de P est un nombre réel irrationnel.

19
Remarque 73. Ainsi, l’étude de l’irréductibilité d’un polynôme à coefficients entiers sur Q[X]
se réduit à l’étude de l’irréductibilité de Z[X], qui est a priori plus facile.
Voici un exemple (important) d’application de ceci.
Théorème 74. Soit P(x) = an xn + · · · + a1 x + a0 un polynôme de Z[X]. On suppose qu’il existe
un nombre premier p tel que :
1. p divise a0 , a1 , . . . , an−1 ,
2. p ne divise pas an ,
3. p2 ne divise pas a0 .
Alors P est irréductible dans Q[X].
Démonstration. D’après la proposition précédente, il suffit de montrer que P est irréductible
dans Z[X]. Supposons donc par l’absurde que P(x) = Q(x)R(x) avec Q, R deux polynômes
non constants de Z[X] avec Q(x) = qk xk + · · · q0 et R(x) = rl xl + · · · + r0 . Alors a0 = q0 r0 . Par
suite, d’après le point 3., p divise q0 ou r0 , mais pas les deux à la fois. Sans perte de généralité,
supposons que p|q0 et que p 6 |r0 . D’autre part, p ne divise pas qk car sinon il diviserait an , ce
qui est exclu. Soit donc i0 le plus petit indice i (1 6 i 6 k) tel que p ne divise par qi . Alors :

ai0 = qi0 r0 + qi0 −1 r1 + · · · + qo ri0 .

Comme i0 6 k < n, p divise ai0 et donc p divise qi0 r0 , et donc p divise r0 , ce qui est absurde.

Exemple 75. Soit p un nombre premier et P(x) = xp−1 + · · · + x + 1 ∈ Z[X]. En appliquant le


critère d’Eisenstein au polynôme Q(x) = P(x + 1), on voit que P est irréductible dans Q[X].
Exercice 2 (IMO 93, exercice 1) Soit n > 2 un entier. Montrer que le polynôme f(x) = xn +
5xn−1 + 3 est irréductible sur Z[X].

8 Exercices supplémentaires
Les exercices qui suivent sont plus ou moins rangés par difficulté croissante.
Exercice 3
Soit P un polynôme de degré 4 tel que P(0) = P(1) = 1, P(2) = 4, P(3) = 9 et P(4) = 16.
Calculer P(−2).
Exercice 4
1−α 1−β 1−γ
α, β, γ étant les trois racines de x3 − x − 1, calculer : 1+α
+ 1+β
+ 1+γ

Exercice 5p √ p √ p √ p √
Soit a = 4 + 5 − a, b = 4 + 5 + b, c = 4 − 5 − c et d = 4 − 5 + d. Calculer
abcd.
Exercice 6 (Canada 1970)
Soit P un polynôme à coefficients entiers. On suppose qu’il existe des entiers deux à deux
distincts a, b, c, d tels que P(a) = P(b) = P(c) = P(d) = 5. Montrer qu’il n’existe pas d’entier
k tel que P(k) = 8.

20
Exercice 7
Trouver les polynômes P tels que pour tout réel x :

(x − 16)P(2x) = 16(x − 1)P(x)

Exercice 8
Soit x ∈ R tel que x2 − x ∈ Z et il existe un entier positif n > 3 tel que xn − x ∈ Z. Montrer que
x est entier.
Exercice 9
Soit P et Q des polynômes unitaires de degré 2014, tels que pour tout réel x, P(x) 6= Q(x).
Montrer qu’il existe un réel x tel que P(x − 1) = Q(x + 1).
Exercice 10 (Benelux 2010)
Trouver tous les polynômes P ∈ R[X] tels que pour tous réels a, b, c on ait :

P(a + b − 2c) + P(b + c − 2a) + P(c + a − 2b) = 3P(a − b) + 3P(b − c) + 3P(c − a).

Exercice 11
Alcina et Bajazet jouent au jeu suivant : on écrit x4 +∗x3 +∗x2 +∗x+1 au tableau. Alcina choisit
une étoile et la remplace par un réel, puis c’est à Bajazet, et ainsi de suite jusqu’à épuisement
des étoiles. Alcina gagne si le polynôme P obtenu n’a pas de racine réelle. Sinon, c’est Bajazet.
Montrer que ce dernier a une stratégie gagnante.
Exercice 12 Q
Soit ai ∈ Z, i = 1, . . . , n des entiers deux à deux distincts, et soit P := ni=1 (X−ai )−1. Montrer
que P est irréductible dans Z[X].
Exercice 13 (IMO 1988, exercice 1) P
Montrer que l’ensemble des réels x vérifiant 70
k=1
k
x−k
> 5
4
est réunion d’intervalles dont la
somme des longueurs vaut 1988.
Exercice 14
Trouver tous les polynômes autre que les polynômes constants, à coefficients dans R et tels
que, pour tout z ∈ C,
P(z2 ) = P(z)P(z − 1).

Exercice 15
Soit P, Q ∈ R[X] deux polynômes réels non constants ayant les mêmes racines, et tels que P − 1
et Q − 1 aussi. Montrer que P = Q.
Exercice 16
On se donne 2n réels distincts a1 , a2 , . . ., an , b1 , . . ., bn . On ramplit une table n × n en mettant
ai +bj dans la case (i, j). On suppose qu’il existe une constante c telle que le produit de chaque
ligne est c. Montrer qu’il existe une constante d telle que le produit de chaque colonne soit d.

21
Exercice 17 (IMO 2004, exercice 2)
Trouver tous les polynômes P à coefficients réels qui vérifient, pour tous a, b, c réels tels que
ab + bc + ca = 0 :

P(a − b) + P(b − c) + P(c − a) = 2P(a + b + c).

Exercice 18 Soit n > 1 un entier. On note Sn l’ensemble des permutations de l’ensemble


{1, 2, . . . , n}. Pour σ ∈ Sn , on note aussi cyc(σ) le nombre de cycles de σ. Soit Pn (x) le poly-
nôme suivant : X
Pn (x) = xcyc(σ) .
σ∈Sn

Montrer que Pn a toutes ses racines réelles et que ce sont des entiers négatifs.
Exercice 19 (Test de sélection Chine 2008)
Soient m, n des entiers strictement positifs et P ∈ Z[X] un polynôme de degré n tel que tous
ses coefficients soient impairs. On suppose que (x − 1)m divise P. Montrer que si m > 2k (avec
k > 2 entier), alors n > 2k+1 − 1.
Exercice 20
Si P est un polynôme à coefficients entiers, on note w(P) le nombre de ses coefficients impairs.
Soit Qi = (1 + x)i . Montrer que pour tout suite d’entiers 0 6 i1 < i2 < · · · < in , w(Qi1 + · · · +
Qin ) > w(Qi1 ).
Exercice 21 (IMO 2002, exercice 3)
Trouver les couples (n, m) tels que n > 2, m > 2, et il existe une infinité d’entiers naturels k
tels que kn + k2 − 1 divise km + k − 1.

9 Quelques motivations
Pourquoi étudie-t-on les polynômes ? Voici quelques éléments de réponse donnés sans
démonstration.
Théorème 76. Soit f une fonction réelle infiniment dérivable (si vous ne savez pas ce que ça
veut dire, imaginez qu’elle est très gentille). Soit x0 ∈ R. Alors pour tout entier n, pour tout
 > 0, il existe η > 0 et des réels a0 , . . . , an tels que pour tout x ∈ [x0 − η, x0 + η] :

|f(x − x0 ) − a0 − a1 (x − x0 ) − · · · − an (x − x0 )n | 6 |(x − x0 )n |.

Ainsi, au voisinage de tout point, la fonction « ressemble » à un polynôme.


Théorème 77. Soit f : [0, 1] → R une fonction continue. Alors il existe une suite de polynômes
P1 (x), P2 (x), · · · telle que pour tout  > 0, il existe N > 0 tel que pour tout n > N :

pour tout x ∈ [0, 1] |f(x) − Pn (x)| 6 .

Ainsi, toute fonction continue sur [0, 1] peut être approchée sur tout [0, 1] par des polynômes.
Signalons finalement que l’étude de l’ensemble des zéros communs de plusieurs poly-
nômes à n variables, appelé variété algébrique, est centrale en géométrie algébrique.

22
10 Distinction entre polynôme et fonction polynomiale
Ici, nous expliquons pourquoi il est nécessaire de faire cette distinction en commençant
par définir d’une autre manière un polynôme. Ici, K = Q, R, C ou bien Z/pZ muni des lois
d’addition et de multiplication usuelles.
Définition 78. Un polynôme à coefficients dans K est une suite infinie d’éléments de K nulle
à partir d’un certain rang.
Exemple 79. Par exemple, (0, 1, 2, 3, 0, 0, . . . , 0, . . .) est un polynôme, de même que (0, 0, . . . , 0, . . .).
Par contre, (1, 1, . . . , 1, . . .) n’en est pas un.
Définition 80. Soit P = (un )n et Q = (vn )n deux polynômes. On définit le polynôme P + Q
par la suite wn = un + vnP (qui est bien nulle à partir d’un certain rang) et le polynôme P × Q
par la suite (zn ), où zn = i+j=n un vn (vérifier que (zn ) est nulle à partir d’un certain rang).
On identifie les éléments de K avec les polynômes constants via l’application qui à un élément
λ ∈ K associe le polynôme (λ, 0, 0, . . . , 0, . . .). Remarquons que ceci est cohérent avec la notion
de multiplication intuitive d’un polynôme par un élément de K : si (un ) est un polynôme et
λ ∈ K, alors le polynôme λ × (un ) est le polynôme (λun ).
Nous introduisons maintenant l’indéterminée X.
Définition 81. Notons X le polynôme (0, 1, 0, 0, . . .).
P
Proposition 82. Tout polynôme P s’exprime sous la forme P = n n
i=0 an X . On note indiffé-
remment P ou P(X) pour rappeler qu’on note X l’indéterminée (on pourrait très bien la noter
Y !).
Démonstration. Si P = (a0 , a1 , a2 , . . .), notons N un entier tel que i > N implique ai = 0. Alors
P(X) = a0 + a1 X + · · · + aN xN . Ceci est une conséquence immédiate de la définition de X et de
la multiplication entre polynômes.
Voici maintenant le lien entre polynôme et fonction polynomiale associée. Rappelons que,
pour l’instant, un polynôme est juste une suite de nombres qui est nulle à partir d’un certain
rang et n’est pas vu comme une application.
Proposition 83. Soit P(X) = a0 +a1 X+· · ·+an Xn ∈ K[X] un polynôme. On note P e l’application
définie par P(x)
e = a0 + a1 x + · · · + an xn pour x ∈ K, qu’on appelle application polynomiale
associée à P. L’application P 7→ P e est injective si K est infini. Si K est fini, cette application
n’est pas nécessairement injective.
Démonstration. Plaçons nous d’abord dans le cas où K est infini. Soient P, Q ∈ K[X] tels que
P e Écrivons P(X) = P ai Xi et Q(X) = P bi Xi . Alors le polynôme P(x) − Q(x), au sens
e = Q.
i i
des sections précédentes, a une infinité de racines, donc est nul. Donc ai = bi pour tout i.
Par contre, dans le cas où K est fini, le raisonnement précédent ne s’applique pas. Exhibons
d’ailleurs un contre-exemple. Considérons K = Z/pZ et P(X) = Xp − X. D’après le petit
théorème de Fermat, pour tout x ∈ K, on a P(x) = 0. Ainsi, P n’est pas le polynôme nul, mais
les deux fonctions polynomiales associées sont les mêmes.
En d’autres termes, lorsque K est infini (par exemple K = Q, R, C, ce qui explique que
nous n’avons pas perdu de généralité dans les premières sections), nous pouvons parler sans
distinction de polynôme ou de fonction polynomiale associée. En revanche, dans les autres
cas, il faut faire très attention !

23
11 Éléments de réponse aux exercices
Solution de l’exercice 1 On cherche le reste sous la forme R(X) = aX + b. On a R(1) = P(1),
R(−1) = P(−1), ce qui permet de calculer R(X) = −2X + 2.
Solution de l’exercice 2 Si Q(x) était un polynôme, alors Q(x) − x serait un polynôme avec une
infinité de racines, donc serait de degré nul, c’est absurde.
Solution de l’exercice 3 Comme P(x)2 = 16P(x2 /4) est un polynôme en x2 , on peut appliquer
la proposition 24. Dans le premier cas, s’il existe un polynôme Q tel que P(x)2 = Q(x2 ), on
obtient 16Q(x4 ) = 16P(x2 ) = P(2x)4 = Q(4x2 )2 , et donc 16Q(x2 ) = Q(4x)2 . Dans le deuxième
cas, s’il existe un polynôme Q tel que P(x)2 = xQ(x2 ), on obtient similairement que 4Q(x2 ) =
Q(4x)2 .
On peut donc réappliquer la proposition 24 à Q, et de même on obtien que pour tout entier
k
k > 0, il existe un entier 0 6 i 6 2k et un polynôme Rk tel que P(x) = xi Rk (x2 .
En choisissant k tel que 2k > deg P, il s’ensuit que Rk est forcément constant et donc que
P(x) = c · xi . En réinjectant dans l’équation de départ, on obtient P(x) = 16(x/4)i pour un
certain entier i > 0 (et toutes ces solutions conviennent bien, réciproquement).
Solution de l’exercice 4 1 doit être racine double de P. Cela nous donne deux équations : P(1) =
0 et P 0 (1) = 0, qui permettent de trouver a = 3 et b = −4.
Solution de l’exercice 5 On note n le degré de P. En passant l’équation aux degrés, on obtient
n = (n−1)+(n−2) = 2n−3, donc n = 3. On peut facilement calculer le coefficient dominant,
on laisse le soin au lecteur de terminer les calculs.
0 00 02
Solution de l’exercice 6 On remarque que la dérivée de PP est P P−P qui est du même signe
P 1 P −1
2
00 02 P 0 (x) P 0 (x) 0
que P − P . Or on voit facilement que P(x) = x−αi
donc ( P(x) ) = (x−αi )2
< 0 d’où
le résultat. Pour obtenir l’inégalité sur les coefficients on procède de la manière suivante.
Pour k = 1, l’inégalité provient de P(0)P 00 (0) 6 P 0 (0)2 . Ensuite on applique l’inégalité aux
2
polynômes P(k−1) : P(k−1) P(k+1) 6 P(k) d’où ak−1 (k − 1)! × ak+1 (k + 1)! 6 a2k × k!2 or
k!2 k
(k−1)!(k+1)!
= k+1 6 1 d’où le résultat.
Solution de l’exercice 7 Notons α1 , . . . , αn les n racines de P. On écrit :

P 00 (x)P(x) − P 0 (x)2

P 0 (x)
0 X
n
−1
= = .
P(x)2 P(x) i=1
(x − αi )2

Ainsi,

n(P 0 (x)2 − P(x)P 00 (x)) − P 0 (x)2


(n − 1)P 0 (x)2 − nP(x)P 00 (x) = P(x)2 ·
P(x)2
 !2 
Xn
n X n
1
= P(x)2  2
− ,
i=1
(x − αi)
i=1
(x − α i)

qui est positif d’après l’inégalité de Cauchy–Scwharz. Le cas d’égalité s’obtient lorsque tous
les αi sont égaux, i.e. lorsque P(x) est de la forme P(x) = c(X − a)n .

24
Solution de l’exercice 8 On a déjà résolu le problème lorsque le degré de P est au plus n − 1
grâce aux interpolateurs de Lagrange. Si le degré de P est supérieur ou égal à n, notons L le
polynôme interpolateur associé aux ai et bi . Le polynôme P − L s’annule en a1 , . . . , an . On a
donc
Xn Yn
X − aj
P(X) = c1 bi + c2 (X − a1 )(X − a2 ) · · · (X − an ).
i=1 j=1,j6=i
ai − aj

Solution de l’exercice 9 Un polynôme à coefficients rationnels est clairement solution. Récipro-


quement, si P est un polynôme de degré n vérifiant cette propriété, alors en interpolant en
n + 1 points rationnels, on remarque que P est à coefficients rationnels.
Solution de l’exercice 10
1. Soit i > n. Alors
1 Y
n−1  
i
(i − k) = ∈ Z.
n! k=0 n
On traite similairement le cas i < 0.
2. On remarque que Hn (n) = 1 et Hn (i) = 0 pour des entiers 0 6 i 6 n − 1. Si P ∈ C[X] est
tel que P(k) ∈ Z pour tout k ∈ N, notons n le degré de P et soit

X
n
Q(X) = P(X) − P(i)Hi (X).
i=0

Le polynôme Q est de degré n et possède n + 1 racines 0, 1, . . . , n. On en déduit que


Q est nul. Les polynômes cherches sont donc des combinaisons linéaires entières des
polynômes de Hermite.
3. (i) On a

X X k   X
k   i  
!
k k i
Xk = (X + 1 − 1)k = (X + 1)i (−1)k−i = Xj (−1)k−i
i=0
i i=0
i j=0
j
X X
k k   !
k i
= (−1)k−i Xj .
j=0 i=j
i j

Ainsi, en identifiant les coefficients, la somme cherchée est nulle pour 0 6 j 6 k − 1


et vaut 1 pour j = k.
(ii) Pour prouver que 1. implique 2., si P est de degré n, on peut écrire

X
n
P(X) = P(i)Hi (X).
i=0

On a alors pour tout entier j > 0.

Xn  
j
P(j) = P(k)
k=0
k

25
et
Xi  Xi  Xi X n   
i−j i i−j i i−j i j
(−1) uj = (−1) P(j) = (−1) P(k).
j=0
j j=0
j j=0 k=0
j k

D’après ce qui précède, cette somme est nulle pour i > n + 1.


Pour montrer que 2. implique 1., on voit que le polynôme

X
n
P(X) = ui Hi (X)
i=0

convient en utilisant un raisonnement similaire.

Solution de l’exercice 11 On met P sous forme canonique : P = a(x − b)2 + c. On translate de b


selon l’axe des abscisses, de −c selon l’axe des ordonnées, et on applique une homothétie de
rapport √1a .
Solution de l’exercice 12 Soit (x, y, z) une solution. Visiblement, aucun de ces nombres n’est nul.
En retranchant la troisième équation à la deuxième équation, on en déduit que zx = xy,
puis, en simplifiant par x (qui est non nul), on obtient que z = y. En retranchant la troisième
équation à la première équation, on obtient : y2 − xy = 8, ou encore xy = y2 − 8. La deuxième
équation se réécrit y2 x + xy = 4. Il vient donc :

y(y2 − 8) + y2 − 8 = 4,

ou encore y3 + y2 − 8y − 12 = 0. On remarque que y = 3 est une solution. En effectuant la


division euclidienne de y3 + y2 − 8y + 12 par y − 3, on trouve :

y3 + y2 − 8y − 12 = (y − 3)(y2 + 4y + 4) = (y − 3)(y + 2)2 .

On en déduit que y = z = 3 ou y = z = −2. Dans le premier cas, x = 13 et dans le deuxième


cas, x = 2. Réciproquement, les triplets (2, −2, −2) et ( 13 , 3, 3) sont solution et ce sont donc les
seules.
Solution de l’exercice 13 Supposons que ax2 − (a + 3)x + 2 = 0 admette deux racines de signe
opposé, notées z1 , z2 . Alors d’après les relations de Viète, z1 z2 = 2/a. Or z1 et z2 sont de signe
opposés si, et seulement si, z1 z2 < 0. On en déduit que a < 0. Réciproquement, si a<0, alors le
discriminant de l’équation vaut a2 − 2a + 9. Pour montrer qu’il est positif, utilisons la forme
canonique en écrivant a2 − 2a + 9 = (a − 1)2 + 8 > 0. Ainsi, lorsque a < 0, il y a deux solutions
réelles notées z1 , z2 . D’après les relations de Viète, z1 z2 = 2/a < 0, de sorte que z1 et z2 sont de
signe opposés.
Remarquons que dans la preuve de la réciproque, il a d’abord fallu montrer que le poly-
nôme avait deux racines réelles avant d’utiliser les relations de Viète.
P
Solution de l’exercice 14 On pose P = ak Xk , et on appelle n le degré de P. La somme des
a (n−1)(n−2)...(n−k)
racines de P(k) vaut n−1 an n(n−1)...(n−k+1)
= an−1a(n−k)
nn
. La suite est donc arithmétique, de raison
−an−1
nan
.
Solution de l’exercice 15 Indication : introduire σ1 = x + y et σ2 = xy, puis écrire les équations
correspondantes pour σ1 et σ2 , puis les résoudre.

26
Solution de l’exercice 16 On a clairement x/(x + y) + y/(x + y) = 1 et
x y xy
· = 2 = 1.
x+y x+y x + 2xy + y2

Ainsi, x/(x + y) et y/(x + y) sont les racines de t2 − t + 1 = 0, de sorte que les sommes
Sk = (x/(x + y))k + (y/(x + y))k vérifient S0 = 2, S1 = 1 et

Sk+2 = Sk+1 − Sk

pour k > 0. On en déduit que la suite (Sk ) est période de période 6, ses valeurs étant succes-
sivement 2, 1, −1, −2, −1, 1, 2, 1, −1, . . .. On en déduit que S2013 = −2.
Solution de l’exercice 17 On écrit les relations de Newton :

S1 − σ1 = 0, S2 − σ1 S1 + 2σ2 = 0, S3 − σ1 S2 + σ2 S1 − 3σ3 = 0.

Ainsi, σ1 = 3, σ2 = 3, σ3 = 1. On en tire que x, y, z sont racines de t3 − 3t2 + 3t − 1 = 0. Or


t3 − 3t2 + 3t − 1 = (t − 1)3 . Donc x = y = z = 1.
Solution de l’exercice 18 Si un tel P existait, on aurait a − b|P(a) − P(b), i.e a − b|b − c et de
même b − c|c − a et c − a|a − b donc |a − b| 6 |b − c| 6 |c − a| 6 |a − b|, donc on doit avoir
égalité partout, ce qui est impossible car, si par exemple a > b > c, alors |a − c| > |a − b| et on
vérifie de même les autres cas...
Solution de l’exercice 19 Écrivons

P(P(x)) − Q(Q(x)) = (Q(P(x)) − Q(Q(x))) + S(P(x)),

où S(x) = P(x) − Q(x). Suppsons que S 6= 0. Soient k le degré de S et n le degré de Q. On voit


aisément que le degré de Q(P(x)) − Q(Q(x)) est n2 − n + k et que le degré de R(P(x)) est kn.
Si k > 1, on a kn < n2 − n + k, et donc le degré de P(P(x)) − Q(Q(x)) est nn − n + k,
absurde.
Si k = 0, S est constant. Écrivons S = c. On obtient alors que Q(Q(x) + c) = Q(Q(x) − c).
Ainsi, Q(z + c) = Q(z) − c pour une infinité de réels z, et donc Q(x + c) = Q(x) − c. Donc
Q(kc) = Q(0) − kc pour tout entier k, et donc Q(x) = Q(0) − x. Ceci contredit le fait que Q est
unitaire.
Solution de l’exercice 20 Écrivons P(x) = xn Q(x) pour un certain entier n > 0 et Q(x) un po-
lynôme tel que Q(0) 6= 0. Alors Q vérifie la propriété de l’énoncé. Si Q n’est pas constant, en
faisant tendre x vers l’infini, on voit que forcément Q(0) = 0, absurde. Donc Q est constant
est P est de la forme P(x) = cxn avec c ∈ R et n > 0. Réciproquement, on vérifie que les
polynômes de la forme P(x) = cxn avec |c| 6 1 et n > 0 conviennent.
Solution de l’exercice 21 Soit P(x) = 1 + x + · · · + xn−1 = (1 − xn )/(1 − x). Ainsi, P(x) divise
P(x2 ) si et seulement si il existe un polynôme Q(x) tel que

1 − x2n 1 − xn
= Q(x) ,
1 − x2 1−x
ou encore
(1 + xn ) = Q(x)(1 + x).

27
Ainsi, P(X) divise P(X2 ) si et seulement si −1 est racine de 1+Xn , autrement dit si et seulement
si n est impair.
Solution de l’exercice 22 La première assertion est vraie (utiliser le théroème de Bézout pour les
nombres entiers avec 7 et 12). La seconde assertion est fausse (prendre x = 21/3 ).
Solution de l’exercice 23 Supposons par l’absurde que P admette une racine réelle, α. Alors
α2 + α + 1 est une autre racine du polynôme, strictement supérieure à la précédente. On
construit ainsi une infinité de racines distinctes, contradiction. Donc toutes les racines de P
sont complexes, donc P est de degré pair.
Solution de l’exercice 24 On écrit P comme produit de polynômes irréductibles dans R. P est
le produit de polynômes de degrés 2 de discriminant négatifs et de polynômes de la forme
(x − a)2k (en effet si la multiplicité d’une racine était impaire, au voisinage de cette racine on
pourrait rendre P négatif). Pour exprimer la partie complexe comme somme de carrés, on la
sépare en deux termes conjugués l’un de l’autre (en séparant les termes (X − z) des (X − z̄)).
Cela termine, car (P − iQ)(P + iQ) = P2 + Q2 .
Solution de l’exercice 25 On démarre par diviser P par son contenu, ce qui ne modifie pas les
hypothèses (car ce contenu est impair). Supposons par l’absurde que P a toutes ces racines
rationnelles. Comme c(P) = 1, ces racines sont entières. Comme d est impair, ces trois ra-
cines sont impaires, et les relations coefficients racines montrent que b et c sont impairs, c’est
absurde.
Solution de l’exercice 26 Supposons qu’il existe deux polynômes g et h, à coefficients entiers,
tels que f = gh. Comme f(0) = 3, on peut supposer, sans perte de généralité que |g(0)| = 3 et
on écrit : g(x) = xk +ak−1 xk−1 +...+a0 (a0 = ±3). On s’inspire maintenant de la démonstration
du critère d’Eisenstein : soit j le plus petit indice tel que aj ne soit pas divisible par 3. On pose
h(x) = xp + bp−1 xp−1 + ... + b0 et f(x) = xn + cn−1 xn−1 + ... + c0 , il apparaît que le coefficient
cj = aj b0 + aj−1 c1 + ... n’est pas divisible par 3 car b0 a0 = 3 et a0 = ±3. Compte tenu de
l’expression de f,j > n − 1, donc k > n − 1 donc p 6 1 donc le polynôme h s’écrit ±x ± 1, ce
qui est absurde car f(±1) = 0.
Solution de l’exercice 27
Dans ce genre d’exercice, on cherche un polynôme "proche" de P ayant (presque) autant de
racines que son degré, pour l’avoir sous forme factorisée. Avec un soupçon d’observation, on
se rend compte que P(1) − 12 = P(2) − 22 = P(3) − 32 = P(4) − 42 = 0. P(X) − X2 est de degré
au plus 4, donc il existe un réel c tel que P(X) − X2 = c(X − 1)(X − 2)(X − 3)(X − 4). Pour X = 0,
on trouve c = 241 .
Solution de l’exercice 28 Une méthode sûre même si, en l’occurrence, on peut trouver plus ra-
pide, est de chercher l’équation ayant pour racines 1−α , 1−β , 1−γ et de calculer la somme des
1+α 1+β 1+γ
racines de cette dernière équation à partir de ses coefficients. Si x est racine de x3 − x − 1, de
quelle équation est racine y = 1+x 1−x
? On remarque que x = 1−y 1+y
(la fonction est involutive),
 3  
1−y 1−y
donc 1+y − 1+y − 1 = 0, soit : (1 − y)3 − (1 − y)(1 + y)2 − (1 + y)3 = 0. L’équation en y
s’écrit donc : −y3 + y2 − 7y − 1, la somme de ses racines vaut 1.
Autre méthode : on aurait aussi pu tout mettre au même dénominateur, développer en
haut et en bas, et tout exprimer en fonction des polynômes symétriques élémentaires.

28
Solution de l’exercice 29
Puisque nous sommes chez les polynômes, débarassons-nous de ces vilaines racines. a, c, −b, −d
sont racines de X4 − 8X2 + X + 11. Donc abcd = bd × (−a) × (−c) = 11 et on a fini... eh
non ! pour être sûr d’avoir les quatre racines du polynôme (dont le produit des opposés fait
le terme constant) sans avoir de problème de multiplicité, il serait bien de pouvoir montrer
que
√ ces quatre √ racines sont distinctes. a, c sont positives, −b, −d sont
√ négatives. Si a = c,
5 − a = − 5 − a = 0. Donc a = 5, et en partant de l’énoncé, a = 4 = 2, contradiction.
Donc a 6= c, b 6= d de même et on a bien le résultat voulu.
Solution de l’exercice 30 On écrit P sous la forme P(X) = Q(X)(X − a)(X − b)(X − c)(X − d) + 5,
et on suppose par l’absurde que P(k) = 8. Alors Q(k)(k − a)(k − b)(k − c)(k − d) = 13. Or
(k − a), (k − b), (k − c) et (k − d) sont des entiers distincts, et comme 3 est premier, il ne peut
pas être écrit comme produit de 4 entiers distincts, contradiction.
Solution de l’exercice 31
Avec x = 1, on a P(2) = 0. Puis x = 2 donne P(4) = 0. De même, P(8) = P(16) = 0. On peut
ainsi écrire P(X) = (X − 2)(X − 4)(X − 8)(X − 16)Q(X). En réinjectant dans l’équation, on trouve
Q(x) = Q(2x) pour tout x (sauf x = 2, 4, 8, 16), donc Q(X) − Q(2X) est le polynôme nul.
Si r 6= 0 est racine de Q, 2n r l’est aussi pour tout n ∈ N, donc Q = 0 ou r = 0. Donc Q(X) =
aXk , et il est facile de voir que k = 0. Donc Q est constant.
On en déduit que P = c(X − 2)(X − 4)(X − 8)(X − 16) avec c un réel. Réciproquement, ce genre
de polynôme est bien solution.
Solution de l’exercice 32 On note a = x2 − x. Si a = 0, x = 0 ou x = 1. Si, a 6 −1, l’équation
x2 − x n’a pas de racines réelles, donc a > 1.
Montrons par récurrence que, pour tout ` ∈ N, ` > 3, in existe deux entiers positifs A` et
B` tels A` > 2 et
x` = A` x + B` .
Pour ` = 3 on a x3 = (a + 1)x + a. Supposons le résultat vrai pour `. Ainsi x` = A` x + B` . D’où
x`+1 = (A` + B` )x + aA` . A`+1 := A` + B` et B`+1 = aA` conviennent. Cela finit la récurrence.
Ainsi, pour ` = n, on a xn = An x + Bn . D’autre par, d’après l’énoncé, xn = x + b pour un
b entier. Donc x = b−B n
An −1
. On peut diviser car An > 2. Ainsi x ∈ Q.
2
Comme x vérifie x − x − a qui est à coefficients entiers, le dénominateur de x divise le
coefficient de tête. Ainsi x est entier.
Solution de l’exercice 33
Reformulons classiquement et légèrement l’énoncé : on veut une racine réelle au polynôme
R(X) = P(X − 1) − Q(X + 1). A quoi peut-il ressembler ? Il est clairement de degré au plus 2013
(le coefficient du terme de degré 2014 étant nul). S’il est de degré 2013, donc impair, alors il
aura bien une racine impaire. Regardons de plus près, en posant P(X) = X2014 +aX2013 +S(X) et
Q(X) = X2014 +bX2013 +T (X), où S et T sont de degré au plus 2012. Soit de plus c le coefficient de
degré 2013 de R. On obtient c = −2014−2014+a−b = a−b−4028 (on regarde, dans (X−1)2014 ,
le terme de degré 2013, et de même pour (X+1)2014 ). Se posent deux questions : comment gérer
ce a − b ? Et à quoi sert la condition P(x) 6= Q(x) pour tout x réel ? Heureusement, elles sont
liées !
P − Q n’a pas de racine réelle, donc n’est pas de degré impair. Or P − Q est de degré au plus
2013, et le coefficient du terme de degré 2013 est a − b. Donc a − b = 0.
Ainsi c 6= 0, et R est bien de degré 2013, ce que l’on voulait.

29
Solution de l’exercice 34 En injectant a = b = c = 0, on trouve P(0) = 0. En prenant b = c = 0,
on obtient P(2a) = 3P(a) + P(−a), et ce pour tout a. On suppose P de degré n. En examinant
les coefficients dominants, on obtient 2n = (−1)n + 3, donc n vaut 1 ou 2, et P est de la forme
aX2 + bX. On vérifie réciproquement que ces polynômes conviennent.
Solution de l’exercice 35
En +∞ et −∞, le polynôme tend vers +∞ car de degré pair. Pour avoir une racine réelle, il
suffit que le polynôme soit négatif a un moment, le pauvre sera bien obligé de passer par zéro.
Si Alcina met un coefficient c devant x3 , Bajazet met un nombre négatif c 0 de valeur absolue
assez grande devant x2 de sorte que x4 + cx3 + c 0 x2 + 1 soit strictement négatif en 1 et −1. Alors
si Alcina met un coefficient positif (ou négatif) devant x, P(−1) sera encore strictement négatif
(ou P(1)), donc Bajazet gagne. De même si Alcina joue d’abord sur x.
Si Alcina met un coefficient c devant x2 , c’est un petit peu plus compliqué puisqu’on ne peut
plus rendre le polynôme négatif en un réel et son opposé. On va alors se focaliser sur la
taille des monômes en fonction de celle de x (pour lequel on choisit une valeur > 1. Posons
D = 104 + c102 + 1, D 0 = 54 + c52 + 1. Bajazet place b devant x3 , et Alcina place a devant x. On
a P(10) + P(−10) = 2D (les termes impairs s’annulent, les pairs s’ajoutent). Si P est toujours
positif, avec b = −D, on trouve a > 99D. Donc P(−5) = D 0 + 125D − 5 × 99D < −2D 0 car
D > D 0 . Or P(5) + P(−5) = 2D 0 donc P(5) < 0 et Bajazet gagne avec ce choix de b.
Solution de l’exercice 36 Supposons par l’absurde qu’il existe Q, R ∈ Z[X] non constants tels
que P = QR. Alors P(ai ) = Q(ai )R(ai ) = −1 donc Q(ai ) = ±1 et R(ai ) = −Q(ai ) soit
(Q + R)(ai ) = 0. Comme Q et R sont de degrés strictement inférieurs à n, on obtient Q + R = 0
donc P = −Q2 , donc P est négatif sur les réels. Ceci est absurde car P tend vers +∞ en +∞.
Q
Solution de l’exercice 37 L’inégalité est équivalente, après multiplication par Q(x)2 = ( 70 k=1 (x−
k))2 et passage du second membre dans le premier, à P(x) > 0 où P(X) = Q(X)R(X) avec
P Q
R(X) = 4 70 k=1 k 16i6=k670 (X − i) − 5Q(X).
On remarque R(1) < 0, R(2) > 0, R(3) < 0, ..., R(70) > 0 et R(+∞) = −∞ donc R a pour
racines les ri tels que 1 < r1 < 2 < r2 < · · · < 70 < r70 , et P a pour racines les ri et les entiers
compris entre 1 et 70 inclus. Or P est négatif en ±∞ donc P est positif sur la réunion des
P
intervalles [k, rk ] pour 1 6 k 6 70, dont la longueur totale vaut 70 i=1 (ri − i). Par les relations
69
P 70 9
P70
coefficients racines, on déduit du coefficient de X dans R que i=1 ri = 5 i=1 i donc la
P
longueur totale des intervalles vaut 54 70 i=1 i = 1988.

Solution de l’exercice 38 Supposons par l’absurde que P n’est pas constant. Si z0 est une racine,
alors z20 , z40 , . . . sont aussi des racines. Si on a des racines |z0 | < 1 ou |z0 | > 1 alors P aurait une
infinité de racines. Donc toute les racines sont de module 1.
L’équation s’écrit également P((x + 1)2 ) = P(x + 1)P(x). Donc si z0 est une racine , alors
(z + 1)2 est une racine aussi. Comme toute racine est de module 1, on a |zO + 1| = 1. Pour
chercher les nombres complexes qui vérifient |z| = |z +√ 1| = 1 on pose z = a + ib. On trouve
a2 + b 2 = √ 1 = a2 + b2 + 2a + 1. Alors a = −1 2
et b = ± 23 . Donc P(x) = (x − )a (x − 1 )b pour
 = −12
+ i 23 . Comme P ∈ R[x], a = b, donc P = (x2 + x + 1)a . Réciproquement tout polynôme
de ce type convient.
Solution de l’exercice 39 Soit p0 et p1 le nombre de racines distinctes respectivement de P et
P − 1. Soit n le degré de P, qu’on peut supposer par symétrie supérieur ou égal à celui de Q.
Comme P et P − 1 sont premiers entre eux, ils n’ont aucune racine commune. On remarque

30
que P − Q = (P − 1) − (Q − 1), donc ce polynôme admet comme racines à la fois les racines
de P et celles de P − 1, donc p0 + p1 racines. Il suffit de montrer que ce nombre est strictement
supérieur à n pour conclure que P − Q est le polynôme nul, soit P = Q.
Pour cela, on étudie les racines multiples en considérant le polynôme dérivé P 0 = (P − 1) 0 .
Soit D0 = gcd(P, P 0 ) et D1 = gcd(P − 1, P 0 ). On sait que deg(Di ) = n − pi . Par ailleurs, D0 et
D1 sont premiers entre eux car P et P − 1 le sont, et divisent P 0 donc D0 D1 divise P 0 d’où en
considérant les degrés : n − p1 + n − p2 > n − 1 soit p0 + p1 > n, .
Solution de l’exercice 40 On pose P(x) − Πn
j=1 (x + bj ) − c. D’après l’hypothèse a1 , . . . , an sont
racines de P. Comme P est unitaire, P = Πni=1 (x−ai ). Soit j un indice de colonne. Le produit de
n n
Πi=1 (bj + ai ) vaut (−1) P(−bj ) = −c. Comme c ne dépend pas de j, les produits des colonnes
sont tous égaux.
Solution de l’exercice 41 On remarque tout d’abord, en prenant b = c = 0, que P est pair, et
ne contient donc que des termes de degré pair. En évaluant en zéro, on trouve que le terme
constant doit être nul. On essaie ensuite a = 6x, b = 3x et c = −2x. Cela donne P(3x)+P(5x)+
P(−8x) = 2P(7x). On note n le degré de P. En comparant les coefficients dominants, on trouve
3n + 5n + (−8)n = 2 · 7n . C’est impossible pour n > 5. P est donc de la forme aX4 + bX2 . On
vérifie réciproquement que ces polynômes conviennent.
Solution de l’exercice 42 Notons cn (k) le nombre de permutations de longueur n. Pour résoudre
l’exercice, nous établissons une relation de récurrence sur les cn (k). Nous allons, pour cela,
dénombrer les permutations σ ∈ Kn tel que cyc(σ) = k en les comptant séparément selon la
valeur de σ(n). Si σ(n) = n, on remarque que se donner une telle permutation revient sim-
plement à se donner une permutation de {1, . . . , n − 1} ayant (k − 1) cycles (puisque n est tout
seul dans son cycle). Il y a donc cn−1 (k − 1) permutations qui relèvent de ce cas.
Examinons maintenant le cas où σ(n) est un entier m fixé strictement inférieur à n. L’entier
n apparaît alors dans un cycle de σ qui est de longueur au moins 2 (puisqu’il contient au moins
n et m) et on peut construire une permutation τ de {1, . . . , n−1} simplement en retirant n de ce
cycle et en laissant les autres cycles inchangés. Par construction, il est évident que τ a encore
k cycles. Par ailleurs, on peut reconstruire σ à partir de τ et l’entier m comme suit : on regarde
le cycle de τ qui contient m et, dans ce cycle, on insère l’entier n juste avant m. On déduit de
cela qu’il y a cn−1 (k) permutations à k cycles telles σ(n) est égal à un entier m < n fixé.
En mettant ensemble les deux raisonnements précédents, on aboutit à cn (k) = cn−1 (k −
1) + (n − 1)cn−1 (k). En tenant compte du fait que cn−1 (0) = cn−1 (n) = 0 trivialement, et en
sommant l’égalité précédente pour k variant de 1 à n, il vient :

X
n X
n−1 X
n−1
k k+1
Pn (x) = cn (k)x = cn−1 (k)x + (n − 1) · cn−1 (k)xk = (x + n − 1) · Pn−1 (x).
k=1 k=1 k=1

Solution de l’exercice 43 L’hypothèse sur les coefficients impairs nous donne envie de réduire
modulo 2 : si on note P le réduit de P modulo 2, on a P = Xn + ... + X + 1 et, dans Z/2Z[X],
k
(X − 1)2k divise P si m > 2k , i.e (X + 1)2 |Xn + ... + X + 1.
Pk k
Or, (X + 1)2 = 2i=0 2i Xi et pour i 6= 0, 2k on a d’après la formule de Legendre :
k 

2k  X k
2k i 2k − i i 2k − i
v2 = b j c − b jc − b j c > 1 − b kc − b j c = 1
i j=0
2 2 2 2 2

31
k k k k
donc dans Z/pZ, (X+1)2 = X2 +1 divise P. On pose donc P = (X2 +1)Q : comme P 6= X2 +1,
k
on a n > m. Mais alors, pour faire apparaître le coefficient X2 −1 en développant le produit,
k
il faut que Q ait un coefficient non nul devant X2 −1 , soit deg(Q) > 2k − 1, d’où deg(P) =
deg(P) > 2k+1 − 1.
Solution de l’exercice 44 On remarque w(P ± Q) 6 w(P) + w(Q) (inégalité triangulaire), et en ce
qui concerne la multiplication, si deg(P) < k alors w((1+Xk )P) = w(P)+w(PXk ) = 2w(P) (∗).
En calculant w(Qi ) pour des i petits, on remarque l’égalité polynômiale dans Z/2Z suivante,
k k
qui est démontrée dans l’exercice précédent : (1 + X)2 ≡ 1 + X2 [2]. Cela nous permet de
k 0
simplifier le calcul de w, car si i = 2k +i 0 avec i 0 < 2k , w(Qi ) = w((1+X2 )(1+X)i ) = 2w(Qi 0 ).
Cela va nous permettre de procéder par récurrence sur in .
On suppose le résultat acquis pour in < N et on considère le cas in = N. Soit k tel que 2k 6
N < 2k+1 . Si i1 > 2k , posons il = 2k + il0 , alors d’après le calcul précédent, w(Qi1 + · · · + Qin ) =
2w(Qi10 + · · · + Qin0 ), w(Qin ) = 2w(Qin0 ) et par hypothèse de récurrence, w(Qi10 + · · · + Qin0 ) >
w(Qi10 ) donc en multipliant par 2, on obtient w(Qi1 + · · · + Qin ) > w(Qi1 ).
k k
P
P Si i1 < 2 soitPr tel que ir < 2 P6 ir+1 , alors w(Qi1 + · · · + Qin ) = w( 16l6r Qil +
r<l6n Qil ) + w( r<l6n Qil ) > w( 16l6r Qil ) > w(Qi1 ) en appliquant successivement la
0 0

remarque (∗), l’inégalité triangulaire, et l’hypothèse de récurrence, ce qui conclut.


Solution de l’exercice 45 Si (n, m) est un tel couple, soit Pn (X) = Xn +X2 −1 et Pm (X) = Xm +X−
1, alors le reste R dans la division euclidienne de Pm par Pn est de degré strictement inférieur
à n, et s’il est non nul, pour une infinité d’entiers k, Pn (k) divise R(k) et |Pn (k)| 6 |R(k)|, ce qui
est impossible car Pn domine R d’où R = 0 et Pn divise Pm . En travaillant modulo Xn + X2 − 1,
Xn
on a 1+X ≡ 1 − X (1 + X est inversible car −1 n’est pas racine de Xn + X2 − 1) et Xm + X − 1 ≡ 0
donc X ≡ 1 − X, d’où Xm+1 + Xm − Xn ≡ 0 d’où Pk (X) = Xk+1 + Xk − 1 est divisible par Pn
m

où k = m − n. Si k + 1 = n alors Pn divise Pk − Pn = Xn−1 − X2 donc n = 3, et m = 5 qui


convient. Sinon, k > n, et pour x ∈]0; 1[ xk+1 < xn et xk < x2 car n > 2, d’où Pk (x) < Pn (x), ce
qui absurde car Pn (0) = −1 et Pn (1) = 1 donc Pn a une racine sur ]0; 1[ que Pk ne partage pas.
Ainsi, seul le couple (3, 5) convient.

32
 


!#"%$'&)(+*+$,-+$(./$0/,%*-!!+*2134++!576)$8576)$'")9!#19:,),;=</>(?$+9:6%81,)")A@
";4B6DC'19:E &F*G>H>/9,)")$0I4+J)*EK4+/576)$L"%$2,%!M:$6 ,;4>!9:,;%9:6/,->$(+,;4+/9:,)ON
PL//$Q?9:,-R4119EK&;4:,)*$"ST$GCD$EK&)!$Q"S4&)&)/!1U4+/9:,%V19:(?(+/*W$GR")$X")!M$(?$GCD$(Y@
1!1$X"ST$,->(+/,%$E $,-Z>/(?*Z$, :*,)*(+[")$X")!\F*(?$,->$Z19:E &F*G>H>/9,)R576)[9:,-X//$6
")$&;(]/$.E 9:,)")$4W^$G2(+$(+9:6)&^*2&;(]>J%_E $N[`a?>$,-+/9:,SW;E bE $.T/Scd+9:6DM:$,-
&)/6%+/$6)(+fegh9:,)f&^9:+?/i)!$]")$8&)(?9:6%M$(26),)$./,)*-!!>*4WD1$.(+$(+9:6)&^$E $,-2$Y26),)$
/"%$8/E &F9:(Yj,-+$#";,)]/(+$1jJ%$(+1jJ%$k")$.3'?9:/6%+/9:,lN!NHN

m 9:6%(a3k&%/6)&;4(?UWn1$f/,)*-!!>*Z,S4&)&;(+/?+$,-]&;4a";,)o/$B&)(+9:(>E E $o"lp$,D@
Q
+$/:,)$EK$,- 6rqScD1*$4NoqV$d1jJ)9:HCseg4!/1f$Y ")$d+$d&)341$( $,t19:E &)!*E $,-")$
1$f&)(+9:(>E EK$NDuv9:6%f,)9:6)f!/E !>$(+9:,%X"%9:,)1.w&)(?*+$,->$(]")$fEK*G>J)9n")$f")$#(+*x@
+9:!6%>!9:,y"Sp!,)*:/H>*8576)X+9,7k+&^*1!z;576)$EK$,7/*i)(?/576)$N{PL1T$Y'&^9:6)(+576)9
,)9:6)eg4/+9,)/$|1jJ)9:HC}")$~,)$~&)'EK$,->/9,),)$(")$6nC6%+(+$cn&^$"Sp&)&)(?9D1jJ%$
e€9:,)";4EK$,7>/$8
@B‚2+//!>4+/9:,K")$.OT9:6%+/S4,;!c7+/576)$ ƒg*G>6)"%$"%$.e€9,)1+/9:,)W)")*(+!M*$NHN!N„
@BqV$(+/?9:,),)$EK$,-]&;(](+*16)(?(+$,%1$N
PRUWl?!$ki;::$,)*1$+>4/(+$'$?#z),;/$EK$,-8?+$…'!/E !+*W;/$+6=†?$G8(+$?+$>9:6%0")$
EKbEK$8?6%‡ >E E $,-]M4?>$4N

S$Y]*Mn/"%$,-]576)$81$8576)S?6)!a,)$#(+$EK&%31$(>&;f/$a/HMn(+$o")$8(+*Ge€*(+$,)1$]+6)(f!$
+6=†?$Gfƒ€&;(ˆ$GCD$EK&)!$0‰ Š W/‰p‹ Š W/‰TŒ Š „GWEK/D†T$?&F_(+$X576)$o1jJ;416),c8>(+96%M:$(>v"%$BŽp!,7+*(?bUN

mL/$(+(+$o9(+,)?…>$/,SN
ƒŽ#1>9i)(+$‹‘‘ „
‹
 

     
 

#! "%$'&)(+*-,/.'$102(3045$6,7&)*-8).',7$14:9
-;2<>= ?A@CBEDEF +G-HJIHLK

MGNIPOQGNKSRKTIUH
2KVXWYGNITZ\[
mQ9:6%(]+9:6)f&^9:/,-+ ")$.OT$?&;1$4W;9:,d~ ;UM:$1*:/H>*0?
$2+$6)/$EK$,-v?

]_^5^5`ba\cedgfha = iF
j lkmHJnhHJo k n 6p o
9:!$,- #/$f/9:,)6)$6)(?]")$]1 4>*2"ST6), >(+/,):!$N
mL(+9:6%M$(2576)$
nqRro R oqRrk R ksRrn_t#u [ ˆƒg`v6%>(?/1jJ)$ mQ9:!9::,)$U„ wv
xy{z}|~b€y{‚
k R n R o
„f#,~
u k R u n R u o
nqRro oqRrk kNRn„ƒ n†k Rro'‡R nqRro'n ‡ oˆRrkC‡R oqo RrkC‡ kNRn'‡2R … kNRn‰‡
… … … … …
t kŠ…R kNuR… n†RrnqRro'‡ o'‡R n†R … u o†RrkC‡ R oqR … u kŠRn‰‡
ƒ u [ ksRrnqRo
ƒ u
]_^5^5`ba\cedgfha = ir‹
‘‘‘eŒT’A’‰G/“  ‘‘‘ R ‘‘‘eŒT’h’‰Gs“  ‘‘‘ O ‘‘‘”ŒT’h’‰G/“ •R ŒT’A’‰G–“  ‘‘‘ ŒŽR HLG7‘‘‘eLŒT’hHL’‰GsG/“   Œ7’h’‰Gs“  ‘‘‘ [
-„amL(+9:6DM:$(2576)$W[&^9:6)(a+9:6)]&F9/,-> 2")$.OT$?&;1$4W[9:,d 

i[„fmL(+9:6%M$(2576)$W;&^9:6)(a+9:6)]&F9/,-> ŒŽHLG71H—GswHL’ G–˜wHLGl™AH


)")$.OT$?&;1$4W[9:,d 

Œ
 
 !
#"%$ &"%$ '$(
)
™ ‘ + ‘ * +0/21 ‘ + 1 ‘
, ‘‘ TŒ’L’ G“ ‘‘ O . - ->™ ‘‘ ŒT’J’ G“ R ŒT’6’1G “ ‘‘ [ 43 65
*+ ˆƒ k4&)(+_ o9:,%19:6)(?]:*,)*(> 879: „

xy{z}|C~\€y{‚
‘‘‘ ŒT’h’‰G/“  ‘‘‘ ƒ ‘‘‘ = ŒT’h’‰G7“ R ŒT’h’‰Gs“ ?>R = ŒT’h’‰G/“  ŒT’{’‰Gs“ @> ‘‘‘
-„f#,dd

‘<u=; ‘ ‘ ‘ u ; ’ ‘ ‘
O ; ‘‘>Œ7’h’‰G7“ R Œ7’h’‰Gs“  ‘‘ R ‘‘eŒ7’h’‰G7“  ŒT’h’‰G–“  ‘‘ >
u ‘‘ ŒTGs ‘‘ O = ‘‘ ŒTG/R ŒTG– ’ ‘‘ R ‘‘ ŒTG/ Œ7Gs ‘‘ >
‘‘‘”‘eŒT’h’h’‰’‰G/““  ‘‘‘ ‘ R ‘‘‘eu; ŒT’h’‰G–“ ‘e ’h‘‘‘ ’'“ O ‘‘‘”Œ7’h’h’‰G7’‰“ “ R ‘ Œ7’h’‰Gs“ ‘e’h‘‘‘’‰“ R ’‘‘‘eŒ7’h’h’‰G7“’‰“  ‘ ŒT’h’‰G–“  ‘‘‘ [
3
0$.E bE $

’
PRv/,)?R

A 5
6p
]$EK(+576)$4N o$/ ?/:,)Hz;$#576)$4W^"),)]6),d&;(>4//_/9:(>E EK$4W-3?9:EKE $8")$
/9,):6)$6)(+f")$.")$6DCd1 4>*]19:,)+*16%+!e€2,)$.")*&)+?$&;42/+9:E E $8")$
/9,):6)$6)(+f")$v"%3:9,;/$N

™ ‘ +‘ ‘
i[„B3kp&)(?_v-„GW%/lMn/$,-

*+ ‘ ‘ ŒT’L’ G“ ‘‘ O ‘‘ Œ7’h’‰G7“ R 7Œh’ ’‰Gs“  ‘‘‘ R ‘‘‘ Œ7’A’‰G7“  ’ ŒT’h’‰G–“  ‘‘‘
,
R ‘‘‘ Œ7’h’‰Gs“ ˜5R Œ7’h’‰G–“ ™ ‘‘‘ R ‘‘‘ Œ7’h’‰Gs“ ˜ ŒT’h’‰Gl“ ™ ‘‘‘
‘‘‘ ŒT’h’‰G/“  ŒT’h’‰Gs“  ‘‘‘ R ‘‘‘ ŒT’{’‰Gs“ ˜ Œ7’h’‰G–“ ™ ‘‘‘ O ‘‘‘ ŒT’h’‰G7“  Œ7’h’‰Gs“ 5R Œ7’h’‰Gs“ ˜ ŒT’A’‰Gl“ ™ ’ ‘‘‘
!,)+S576)$

’ ’ R ‘‘‘ Œ7’h’ ’‰G7“  ŒT’h’‰G–“  ŒT’h’ ’‰G–“ ˜:R ŒT’h’‰Gl“ ™ ‘‘‘


O ‘‘‘ Œ7’h’‰G7“ R ’ Œ7’h’‰Gs“ ˜ ‘‘‘ R ’ ‘‘‘ Œ7’h’‰Gs“ 5R Œ7’h’‰G–“ ™ ‘‘‘
R ‘‘‘eŒ7’h’‰G7“ R Œ7’h’‰G–“ ™ ‘‘‘ R ‘‘‘”Œ7’h’‰Gs“ 5R Œ7’h’‰Gs“ ˜ ‘‘‘
3 ™ C
* + ‘‘ ŒTG ‘‘ O * +0/21 ‘‘ ŒTG R ŒTG ‘‘ [
kT9 y
+ + 1

, 
‘ ’L’ “ ‘ . - ->™ ‘ ’J’ “ ’6’1“ ‘
2 D & E Š, 4 4:( 9GF0 0HT8IJHT8_4FLKMHNF$‰02$PORQ
5o$?>$.*Mn!")$,)1$M4',)9:6%2?$(YMD!(2w*Gji%//(o5n6%$/576)$v(?*?6)!>4>fe€9:(Y]6%>!/$N
? $  6    

#"%$ &"%$) R" 
) 
-;2<>= ?A@CBED ‹
k  Rn  u k>n
–ekmkCHJnNnhH O … kNRn'‡  2k nh[

@ mQ9:6%(]+9:6)](+*$!
B ;9:,d~
0$
UM:$1*:/H>*-ƒ€j„B?V$2+$6)/$EK$,-v?
 gH
ƒ
 R  u H
 ƒ [
@BmQ9:6D2+9:6%](?*$ ;9:,d~ )M$1*-4/!+*#+V$G2+$6%/$E $,-
+ ;
]_j ^5^5`ba\ce; dgfha = i 
9:!$,- lkmHJn
0")$6DCd(+*$/],)9,d,76)!N
k R k ™™ R k  R n R n ™™ R n  [
3
0*+$(?EK!,)$(f!$8EK!,)/E6%E ")$
n n n k k k
xy{z}|~b€y{‚
k  R k ™ R k  R n  R n ™ R n   k  R n R  k ™ R n ™™ R  k  R n 
#,~

n  n ™ n  k  k ™ k  ƒ  n  R kR  n ™ k  n k 
UM:$1*:/H>*#?V$2+$6)/$EK$,72? 2k ƒ nhu [ u u
]_^5^5`ba\cedgfha = i 
mL(+9:6%M$(2576)$.&F9:6%(]+9:6)](+*$!  2H  "!
™  R   R  R  ™ O #;  [ A 77 
 ˆƒ ]6)+?/$ „GN

xy{z}|~b€y{‚
#,~ ™ $R   R  $R   ™ O ™   R    ™
O  u# % R #   u#%
u [ u
ƒ #; 
-j ;2<>= ?A@CBE&D ' 'i )< ( dm`ba f +< *2D -d , c /; ./021 c 4; 3 d 6? 58769
lkg‰Hw[ [ [ HJ;k :gHJn6‰Hw[ [ [ HJn :
 ? 

< k  R[ [ [ARrk :> = < n  RQ[ [ [{Rrn :> =  … kg n6R[ [ [{R;k :en :e‡  H
9:!$,- '"%$2(+*$/N)`0!9:(+8

 M:$1*:/H>*#?V$2+$6)/$EK$,72?V/$aM$1G>$6%(+
U … kg‰Hw[ [ [ HJk;:”‡
o$G … n6‰Hw[ [ [ HJn:”‡o?9:,-
19:!/,)*/(?$N

?A@B D| C B
:  
 !
#"%$ &"%$ '$(
)
  : + : + + 
#, &F9:?$
:
 +
/G ƒ * +  k * +  n –I ƒ * +  k n [  $

*  
:, + + , + 1 : + + ,
* 
+ 
+G I ƒ  k n R k n  k n u .- -): k k n n
+ 1   * +   * 0
+ 2
/ 1
+ 1 + 1
’ *,+ /21 < + 1 , 1 + ’ , + 1 + 1 ’
`0!9:(+

ƒ .- -): k + n 1 Rrk 1 n +  ’ u k k n n =


ƒ .- * + /21 -): … k n ’ k n ‡
[
3kT9 Cd319,)1!6)+!9:,SN

A]$EK(+576)$4N
5o$?>$./,%*-!!+*0&^$6%2T/,-+$(+&%(+*+$(2:*9EK*G>(+!576)$E $,-UN)mZ/h9,)Y@ ,)9:6)f";4,)
j ’ “  … kg1j HJk”hHJk”˜1‡ ’ “  … n61HLn1hHLn1˜‰‡
OT$?&;1$$6)1!/")!$,~6)?6)$ŽN

        
a$G o?9:,-2,)9:,d,n6%/W[4/9:(?aOT/,)*-!!>*v")$


’ “ 

’ “ 

’ “ 

’ “ 
… ’ “  H ’ “  ‡ ƒ … ’ “  ’ “  ‡ 
5a6)1jJ-c-@ 1jJ o(?…k>(+")6)Hf+!EK&)!$E $,-f576)$ 

UM:$1*-/H>*#?+

]_^ ^:`ba\cedfAa = i


$
’ “  ’ “ 
?9:,-v19:/!,)*U4/(+$N

‰H hHw[ [ [ H -: gH


… R :R[ [ [AR -:”‡ ;  R ;  R[ [ [{R -; :    [
mL(+9:6DM:$(2576)$W;&^9:6)(a+9:6) )9:,~


xy{z}|C~\€y{‚
3 5
kp&)(?_ a6)1jJ-c-@ 1jJ o(+…  j  : +  
… R :R[ [ [AR -:”‡ ;  R ;  R[ [ [{R -; :   * +  ; +

 [, 
ƒ
]_j ^ ^:`ba\cedfAa = i
lk m J
H h
n J
H A
o 
H 
m 
H 
kN# RnqRro†RNR ƒ  –k  Rrn  Rro  R j  R  ƒ [
9:!$,- "%$](+*$/f+$/f576)$
$G
6)$!/$#$?23M4!$6)(fEK=CD/EK/$2")$ ƒg‚ $&%
` „ 7'29 ;"!
xy{z}|C~\€y{‚
? $  6    

#"%$ &"%$) R" 
) '
… ksRrnqRoqRC‡  O j …  < R  R  R  ‡ < k   Rrn  Ro  R  =
3kp&)(+_65a46)1jJ-c7@ 1jJ a4(+… 

ƒ ; k ; Rrn ; Rro ; R =


3 C … 
A ‡  O  =
kT9 

 …  ’ ‡ O g; [ ! ’
<

# O T’ O ;"!  [
1Npw%NT"SN

;!
5
0NwDNp"SN
 k ƒ n ƒ o ƒ 
qˆ'M4!$6)(
;!
*Gj,-v4?>$!,->$8&^9:6)(
kŠRrn:RroˆR ƒ u  H
1Npw%NT"SN k ƒ n ƒ o ƒ  ƒ ! [
  
1
 R   R  %   R R   2R H  H [
 

77: v 77'
„amL(+9:6DM:$(2576)$W[&^9:6)(a+9:6)](+*$/ %9,~


ˆƒgPZY>9:,%/$ „
; 2 m
k J
H h
n J
H o
… ksRrn‰‡ … nqRo6‡ … oˆRrkC‡  … ksRrn o6‡ … nqRro k‡ … oqRrk n‰‡•[
‹:„amL(+9:6DM:$(2576)$+
; ; ' p
0?9:,-v!$]/9,):6)$6)(+f")$v1 >*]"Sp6%,d+(+34,):/$4W%9:,d 

’ ’ ’ : +
j   –kg1HJk”hHw[ [ [ HJ;k : *+ k g[
Œ-„ 9:H
u
6%,$,7+/$(W;$
+ 1
'"%$](+*$/f+$/f576)$

 *+ :  +  ,  ƒ
* +0/21 k k   k [
mL(+9:6%M$(2576)$
’ u , 
j  : kg‰HJk”wH6[ [ [  HLk : : gH  ƒ+ kg2Rk”†RQ[ [ [ARrk;:g[
 +   H *+  + k   …  ‡ H
„ 9:!$,- 6),d$,->!$(W )$G
:
mL(+9:6%M$(

k +    [,  ’ k
+ 576)$  ; *+
 ’k ’ ;
0$G 576)$ *+

 k
’ ;
, ,

ƒ kp&)(?_2`06)Y>(+/!$ ’ ; 77 Œ:„

 j
2H  H " g[ 
R  R   R  R  [
„ 9:!$,-

mL(+9:6%M$(2576)$
    
: j –kmHJnhHJo … 6p
 k>nqRrn1oˆRo'kC‡ O ksRrnqRo6‡  t … k>n†Rrn1oˆRo'kC‡•[
…
„ 9:!$,- 0/$]/9:,)6)$6)(?f")$21 4>*]"ST6), >(?3,)/$N
m (+9:6%M$(2576)$
L
9  
 !
#"%$ &"%$ '$(
)

' j lkmHJnhHJo 'p


 ksRrn ’ o†R  n†Rro ’ 7sk7: R  o†Rrk ’ n–O  kNR  nqR  oA[
„ 9/$,- #!$f/9,):6)$6)(+]")$21 >*]"Sp6%,>(?3,%:/$4N
mL(+9:6DM:$(2576)$
ƒO#HcDE &)/")$0"%6mQ1Hz;576)$k4+3=>/576)$ „

9 „ j 9/$,- 2H H    [ 


'";,%
 R   R      gH
mL(+9:6DM:$(5n6%$#
ŠR’  –R’ $R’  $L")*G>$(+E /,)$(Q!$1"lp*-/H>*4N

7 „ j 9/$,- lkmHJnhHJo   +$/f576)$ T k>n1o ƒ [ 


k R n;  n ; R o;  o R k;  O [
’ ; ’ ; ’
mL(+9:6DM:$(2576)$ ˆƒO#YI ‹‘:‘:‘:„

: ;+ :; +
j lkg‰HJk”wH6[ [ [ HLk : g[   ƒ * + k • ƒ * + k  [
,  , 
‘-„ 9:/$,7 [#,d&F9:?$ $G

* • k    ‰[
: 
mL(+9:6DM:$(2576)$
,    ’ k
’
j –kg1HJk”hHJk”˜ gH < k  Rk  Rk ˜ =  < k ™ R k ™ Rk ™˜ = [
 7 kg‰ HJk”wHJk”˜ lkM‰HLk>6Hw[ [ [ HJ;k : g6H p u
: „Y-„ 9/$,- )+$/f576)$
L
j
m (+9:6DM:$(2576)$ v?9:,-2/$f!9:,):6%$6)(?2"%$]1 +*2"ST6),d+(+/,):!$N

< k  Rrk  RQ[ [ [ARk :> =  …  ‡ < k ™ Rrk ™ R[ [ [ARk ™:= [ + 1
i[„ 9/$,- '6),$,->/$(W;$G )+$/f576)$ 

— H
e H ’ ; –k HLk HJk 
'p 7 9' v 7 99
mL(+9:6DM:$(2576)$W;&^9:6)(a+9:6) "%$6DCw'")$6DCd")/Y>!,)1+W)!$f,)9:Eki)(+$
+9,72!$f/9,):6)$6)(+]")$21 >*v"lp6), >(?3,%:/$4NSƒ oJ)/,)$ „ 5
j lkmHJnhHJoA˜ H ˜  -k  Rrn  ƒ < o  R  = ˜ [
o R   [j
‹:„ 9:/$,7 >$!f576)$

mL(+9:6DM:$(2576)$
k n ; ˆƒ /,%-&^9:6)(2‹‘:‘‘-„

Œ-„ j 9:/$,7 2H  H " +$/]576)$


R  ; R ; ƒ u [
mL(+9:6DM:$(2576)$
 77 $R ; /R  
 ’ ; R %  ’ ; R   ’ ;[
;
ƒg&%(+9:&^9:+*#YI ‹„

j 2H  H "  H #   #/$ R  lR  2[


 #   … „$ R NR ”‡•[
„ 9:/$,7 %>$/]576)$
mL(+9:6DM:$(2576)$ {ƒ€,)")$‹‘:‘ „
 

  
 S P

!#"%$'&)(+*-,/.'$102( 8 45((H74
-H & &)9 9&„0
5o$?>$o!,)*:/H>*L$YR6),%$B*GMD!")$,%1$a19:EKE $(?1//$a4#,k::,)$X&)/6%Q"Sp(?:$,-R!9:(+?576)$
Lp
OT9:, :
M $,)"}1jJ%$(.3 &)!6)8:(+,)")$'&;4(?>!$'")$',)9:.&)(?9D"%6)!+Wl$G&F$6

&)/6D ]576)$./$819:,->(+/(?$
w~i^9:, E|4(+1jJ)*4W

-j ;2<>= ?A@CBED 
lkg+OQk”–O [ [ [mO k;: –nw OQnJ–O [ [ [mOn:

9:!$,- '$G ")$6DC +6%!>$]1(+9/++,->$v")$
(+*$/N
  ")$ H Hw[ [ [ H
 H)/+9:E E $
+ +
`0!9:(+W)&;(?EKl+9:6%+$]/$f&F$(+E6%>4>!9:,)
: u
 ƒ * +
 k n $?vEK=CD/EK/$2/9:(?+576)$ ƒ K$GvE /,%/EK/$2/9:(?+576)$
;
 … %‡ ƒ ,  X&^9:6)(a+9:6%  [
50p$?Y@ w=@ ")/’ (?$ '
 $?2EK=CD/EK/$2/9(++576)$.!$]+6)H>$2?9:,-2(>,)*$]";4,)]/$8E bE $
9:(+"%(+$W;$G2EK!,)/EK[/9:(?+576ST$!/$f?9:,-2(>,)*$]";4,)]OT9:(+")(?$819:,-+(>!(+$N

|
?A@B DC B 
5o9:E EK$8!l,S cd'5n6lp6),,)9:Ei%(+$#z;,)Xƒ
 „a")$.&F$(+E6%>4>!9:,)a&^9:?+/i%/$W;/^c~$,

j H u H6[ [ [ H 
'6),)$&F9:6%(f3576)$//$ $YvEK=CD/EK!$'ƒg(?$?&SN;EK!,)/EK!$„GN
 #  
! … %‡t  … 
‡•[


K")$6DCd/,%")/1$W 1 6),%$&^+ $(?E6%>4>!9:,~")$


9:!$,-

j $ n" On … n  ‡ $]+6%&)&F9+9:,)

H
576)$ %`0/9(+ 1( o$?v1(?9:/?>,->$4N
;
&% ! 

|9C
 $ …
C‡ ƒ  … %‡ ' $ … %‡ …  ‡
9:H [3'&^$(+Ek6%j4+/9:,~576)V19 ,%1/"%$8&;(?+9:6%vUM:$1 >46%eR&^9:6)(

)(  ƒ # … k
+ 1
o$
1
1
+ ƒ k ‡ < n "  n  # = [

+ 8ƒ€9:,d
 

+ *1jJ;,%1 :$!$aM4!$6)(?f$, X$G D„xN


2k t k ’ –n  t n "’  H ’*( H
`0!9:(+8
@X+ $ [4/9:(? ;1$.5n6%ˆ+?6)(+$576)$ 
k,Sp$?2&;

7
‘ 6 
 "(  "     

+ /$4N1 # 1  +
2k ƒ k n ƒ n" H ƒ C[
E =CD/EK
K
@X+ 9:6 ;9,  (
  $
j  … ‡ ƒ 
`0!,)+ŽW%(?$E &)31$( y&;( ,)$8egHf&;]")!EK!,76)$(a/+9:E E $N
 … C
‡ ƒ H
 $ … ƒ ‡
 

ƒ H
 /9(+W%$,d6%>!//+,-a1$(?*+6%!j=]&F9:6%( $G >$V576)$
$
 ƒ u HHw[ [ H
9:, +$(>4EK_,)$.w'6),)$.&F$(+E6Dj4+/9:, +$/!$8576)$
; ; ; %>96%2$,d,)$8eg/+,-;
 mH
&;]")!EK!,76)$(a/+9:E E $N[#, (+*&F_G>$/9:(?a1$G++$9:&^*(+4>!9:,~&^9:6)(
; ;
’ ƒ  m[

†?6)+576Spw 4++$/,%")(+$ )$vUM:$1")$]E|†?9(>4+/9:,)o+6)11$?+HM:$0")$]+9EKE $
19(+(+$+&^9:,)";4,7+$Nl#,d$,d")*"%6)!]576)$/$8EK=CD/E6%E $?v4++$!,7]&^9:6)(  
;
#, (>!+9:,),%$"%$E bE $#&^9:6)(]/$8E /,%/E6)EdN

A]$EK(+576)$4N
`
:
")$v!9:6)(+"%$6)(?f&)(+_v";4,)]3")*EK9:,%?>(+4>!9:,SW)1$8>J)*9:(+_EK$.&^$6%]p*1(+!(+$.")$
+ +
, k n
egh9:, &)/6)]EK,)/i)/$v+9:6%2/ke€9:(+E $'[qˆ'+9:E EK$ *+ $YvEK=CD/EK!$

/9(++576)$8!$]")$6DCd+6)H>$2?9:,-2(>,)*$]";4,)]/$8E bE $89:(+")(?$ ƒgE bE $8+V1$,lp$?


&;f+9:6=†?9:6)(?f1(?9:/?>,-UW;96 +9:6=†?9:6%(+]")*1(+9:!++,-x„GW;$G2EK!,)/EK[/9:(?+576ST$!/$
+9,72(+,):*$]";,)]Žp9:(?")(+$819:,->(+/(?$8OT6),)$.")$.Op6%+(+$N

]_^ ^:`ba\cedfAa = iF 


˜  ˜
3     R 
  lZ gH W [

“ u
0*G>$(?EK!,)$(f!$8EK!,)/Ek6)E ")$ +6%(

xy{z}|C~\€y{‚
j  VZ gH  W < 
 ˜ 2H  ˜ =    H 

u 3
9:H N[qV$]+6%!>$ $G ?9:,-vE 9:,)94>9:,)$
; ; 
 … ‡  
 ˜ 
R   ˜  ƒ 
  „R    ƒ [
")$.E bE $8+$,%N 09:,)14W;"Sp&)(+_]OT/,)*-/H>*v")6(?*9:(?")9:,),%$E $,-

3
kp6%+(+$.&;(YUW  ; =  > ƒ [ 3 ; 09:,)14W;/$8E /,)!E6)E ; [
1jJ)$(?1jJ)*$?

]_^ ^:`ba\cedfAa = ir‹ ; ;


– m
k J
H h
n J
H o  H
k ˜ Rn ˜ Ro ˜  k  nqRrn  oqRo  k  ek>n‰oA[
mL(+9:6DM:$(2576)$W;&^9:6)(a+9:6)](+*$! )9:, 

xy{z}|C~\€y{‚
   p   7k  n  oA[
k n o sk>n k>o n1oA[
m 4(2YcnEK*G>(+!$8")$]( :/$W;9:, &^$6%2+6%&)&F9+$(2576)$

`0!9:(+
 $G
3kp&)(?_2OT/,)*-/H>*2")6d(+*9(+")9:,%,)$E $,-kƒg")$6DC~e€9:!j„0
    
?     (6<
 6 :

k ˜ Rn ˜ Rro ˜ ƒ  k  kŠ Rrn  n: Rro  o 


k… k>n‰nq‡MRrkNnR oq… k>Ro'‡Mo oqk R … n‰o'‡Mn
ƒ k>n‰oˆRk>o'nqRn1o'k
ƒ ek>n1o{[
2 !#"%$'&)(+*-,/.'$102( )9
)9  F 9
-j ;2<>= ?A@CBED 
lkg+OQk”–O [ [ [mO ;k : –nw OQnJ–O [ [ [mOn:

9 !$,-
: '$G ")$6DC +6%!>$]1(+9/++,->$v")$
(+*$/N
kgRrk”:R [ [ [{Rk : nwRrnJ:R [ [ [hRrn: O kg nwRrk”1nJ: R[ [ [ARk;:en: [
j kg2Rk>:R[ [ [ARr;kn6:  n6n1 Rr nJ†[  [ [RQ [ [ n[{: Rrn :  kg n62Rk”JnJ†RQ[ [ [{Rrk;: n:
`0!9:(+8
OW%&;(f19,7+(+$4W !9:(+8

    [
?A@B D| C B
3kp&)(+_vŽp!,)*:/H>*2")6(?*9:(?")9:,),%$E $,-UW;?V/$]?6)!+$]+9:,-2(>4,):*$
kg nwRrk”JnJ†R[ [ [ARk;:en: ƒ kg n6 Rk”JnJ†RQ[ [ [{Rrk;: n:
";,)]!$8EKbEK$89:(?")(+$4W;9:,d~

kg nwRrk”JnJ†R[ [ [AR;k :en : kg nJ†Rk>1nJ˜:R[ [ [hRrk;:en6


[kg[ [ nwRrk”JnJ†R[ [ [ARk;:en:  kg n:–Rrk”Jn62RQ[ [ [{Rrk;: n:m
 … kg n62Rk”JnJ†RQ[ [ [{Rrk;: n:e‡  … kgRrk”†RQ[ [ [ARk:e‡ … n6RrnJ:R[ [ [ARn:>‡
$,d+9:E EK,-UW)!FMn/$,7

"ST9C!$8(+*+6)Hj4N
#,&)(?9D1_")$.")$.E bE $8";,)]!$81U]9C /$]+6)H>$],)$.?9:,-v&)](>,)*$
";,)]!$8EKbEK$89:(?")(+$4N

]_j ^5^5`ba\cedgfha = i


9:!$,- lkmHJnhHJo gk[ n o  
mL(+9:6%M$(2576)$
nqRro R oqRrk R ksRrn u [
xy{z}|~b€€ y{ ‚ ~ my 
?A@B @B B!
—p -k  n  {o [
m(]YcDE *+(+!$8")$2( :!$W)9:, &F$6%2+6)&%&F9:?$(2576)$
‹ 6 
 "(  "     

`0!9:(+8
Š
k r
R ; n O ksRr; o O n:Rr; o [
3
k R n R o N5  … kNRnqRro'‡  R R
kp&)(?_2OT/,)*-/H>*2")$ oJ)$i-cn+J%$M^Wl!FMn/$,7v!9:(+8

n†Ro  oˆRk kNRrn ; n†Rr; o kŠRr; o kNR; n 



#(8
nqR; o R ksRr; o R kNR; n   … … n†Ro6‡R … oqRkC‡ R … ksRrn‰‡—‡
1Npw%NT"SN
n†R; o R kNRr; k o R kNRr; n n u … kNo Rnq Rro'‡ [
R
nqRo oqRk kŠRrn ; R … N
k R :
n r
R '
o ‡ …
[
u ksRrnqRo6‡ ƒ u
PR]z;,;!$E $,-.

 B | e€   B  ~ my  B !


p
m4(2YcnEK*G>(+!$8")$]( :/$W;9:, &^$6%2+6%&)&F9+$(2576)$ 7 k  n  oA[
`0!9:(+8
q
n r
R ; o

s
k r
R ; o

N
k 
R ; n [
3
k R n R o  k R n R o
kp&)(?_2OT/,)*-/H>*2")6d(+*9(+")9:,%,)$E $,-kƒg")$6DC~e€9:!j„0

n†Ro k oˆRk n kNRrn o oˆ Rrk k kNRn n n:Rro o


$
n†Rro R oqRrk R kNRn kŠRrn R nqRo R ksRro
 k n R o  k R o R n R k R o R n
$,d+9EKEK,-UWD/lMn/$,-

R
u n†Rro oqRrk kNRkNn R o oˆnqRRk k kNoqRRro n kNRn ksRrn n:Rro nqRo
ƒ goq[ Rrk R kŠRn R nqRro
3 C
kT9 d319,)1!6)+!9:,SN
ƒ
]_^ ^:`ba\cedfAa = i  
j lkmHJnhHJoAH gk H ˜ TkCn†Rrn1n oq˜ Rro NR>k oƒ ˜ [
9:!$,- [UM$1
 ˜
mL(+9:6DM:$(05n6%$  R R
n†RroqR oˆR–Rrk sRrkNRn kNRrn†Rro ; ; R  [ RƒgmZ(?9:&^9:+*
#YI 877 ‘-„

xy{z}|C~\€y{‚ k˜ R n˜ R o˜ R ˜
nƒ qRƒ oqRn†RrmH o† Rƒ  oqRoqsRRrskmRrH k ƒ –NRrRkŠkNRRnAHn  ƒ  kskNRrRr nqn†RR o o{[
mQ9+9:,) 


p 7k n o m[
$
m4(2YcnEK*G>(+!$8")$]( :/$W;9:, &^$6%2+6%&)&F9+$(2576)$
    
?     (6<
 6 Œ

`0!9:(+8 k :" n :  o :   :  V”H ;  ;  ;  ; [


&^9:6)( )$G

k  Rn  Rro  R 


 k>n:Rrn1oˆRro NRCk ƒ [
3
0$.&)/6%W;"Sp&)(?_]OT/,)*-!!>*v")6d(+*9(+")9:,%,)$E $,-

N5
  < k ˜ Rn ˜ Rro ˜ R ˜ = R  R  R  [
qLp!,)*-4/!+*0")$ oJ)$i-cD?J)$M19,)")6)H2/9:(?]w~
 ;
; ; ; ; ; 
k ˜ Rn ˜ Rro ˜ R 
˜  ; <  k  Rrn  Rro  R  = … kŠRrn:RroqR‡
qˆE bE $8/,%*-!!+*0")9:,%,)$6)+?Z

k… Rrn Rro R


 kNRrn†RoqRC‡ ƒ „$ R ; NR  –R 
 (8
#
$

3 
09:,)1

;
… R
 Š
 R s
 R  ‡ R
;  ;  ; ;  R R 
;"! ƒ ; [
  
 1  
 
j
  [ [ [ -:MH … C     [ [ [   … :
:  ‰H ww+ Hw[ [ [ H +  :e‡ CJH  'Hw[ [ [ H :e‡ [
„ 9:!$,- %$ ")$](+*$!N

 ‡  O *+ …  ‡  [
#,19:,)+!")_(?$.6),)$.&F$(+E6Dj4+/9:, a")$
: + +
*+ … 87' 
mL(+9:6%M$(2576)$
, ’ , ’ VƒO#YI „

‹ „ 9:H
: j … k  ‡   !
6),)$.+6)H>$8"ST$,-+/$(?v,)4>6)(?$!f,)9:,d,n6%/]$]")$6DCw'")$6nC
")/Y>!,)1+N;mL(+96%M:$(v576)$4W^&^9:6)(a+9:6%
: k : ;
7' 9
 ,   ,  ; [
*   * {ƒO#YI „

j –kg1HJk”hk Hw [ [ [ HJ;kk :" g[ k  k 


 R  R [ [ [R : m R :  kg R)k”R [ [ [ R)k;:M[ 5
Œ-„ 9:!$,-

mL(+9:6%M$(Z576)$.
k” k>˜ ;k : kg [ƒ oJ)/,%$ 79 v 79  „

„ j –kmHJnhHJo k :   n V:  [ o : k :m Rrn : m Ro : m


9:!$,- '$

nqRro R oqRrk R ksRrn [



u
mL(+9:6%M$(2576)$

 j
1H hHw[ [ [ H -: g[


 [ [ [ :   …  w[ [ [ -:”‡           [
„ 9:!$,-
mL(+9:6%M$(2576)$

„ : j 9:!$,-
2H  H "  >$!f576)$ #  ƒ [
;
6 
 "(  "     

˜ R ˜ R ˜   [
mL(+9:6DM:$(v576)$K

&^9:+*#YI 8779
…
„
M‡ … R e‡ … R ”‡ … R ‡ … R ‡ … R$g‡
R QƒgmZ(?94@

; ; ; ; ; ;
 

 )) 
   P 

 … —‡sR
… ‰‡2R [ [ [ R  … -:”‡
qLp!")*$ e€9:,%";E $,-j!$ $Y 576)$4W8/9:(?+576ST6),)$s$GCD&)(?$?+/9, ")$s3re€9(+E $
f")$GMD!$,-#&%/6)2:(>4,)")$|ƒg(+$+&SN^&)!6)]&F$G>!+$„f/9:(?+576)$")$6nC"%$

 … —‡CR  … 1‡gRE[ [ [—R


M4(+/i)/$f+9:,-2(+$,)")6)$0&)!6)]&)(+9n1jJ)$W;>9:6Dv$,&%(+*?$(YM4,70/$f19:,->(+/,-+$2*M:$,D@

 … -:”‡  ƒ  ƒ [ [ [ ƒ -:g[
>6)$/!$Z576)F&^9:(Y>$,-X+6%(oOT$,%+$Eki)/$]")$21$BM4(+/i)!$W:/9:(?
a4++$/,-f?9:,dE|=CD!E6)E ƒg(+$+&SN;?9:, EK!,)/Ek6)E „L576;,)"

-;2<>= ?A@CBED ' 9 ] 9&9 7


j ‰H hHw[ [ [ H -:  e[
 ?
9:!$,- '"),) ;8,~
  R :R [ [ [AR -: :   h[ [ [ -:gH

UM:$1*:/H>*#?V$2+$6)/$EK$,72?   ƒ  ƒ [ [ [ ƒ -:g[
?A@B D| C B
u 9:6%v4//9,)a$\F$1G>6)$(v6%,)$.+6)H>$."ST9:&F*(>4+/9:,%f5n6%V3!++$(>!$8EK$Ei)(+$8"%$
v

/k ƒ + R :R [ [ [AR -:


-6)1jJ%$!,7M4(?3,-ƒ€319:,->(>4/,->$U„o$v6):E $,-+$(>'!$8EK$Ei)(+$8"%$")(?9:!+$
‰H hHw+ [ [ [ H -:g[
j
#,,)94>$
 +

+
/E 9c:$,),)$(+H>J)E *+/576)$#"%$
1
OQkUO A H
— H
+ k + [
 >96)]/$
S ,)$.?9:,-v&)]*-6nCd/9(+W%&)6)/?576)$ ;!

 t t $ ƒ kmH 
1 1 + 1
$ ƒ R km[
$GCD/Y>$ K>$/f576)$

’
#,(?$E &)31$8!9:(+ &;( ;$G &;(
5
7k
o$?>$9:&F*(>4+/9:,~//?+$8!,)1jJ;,)*8/$8E $Ei)(?$8")$.-6)1jJ%$ ƒ€$]")9:,%1/M4/$6%(
+ 1 + 1 + 1 1 + + 1
$ $ ƒ k …  R k‡ ƒ + R … kC‡ … k ‡  H
")$ n„GW)j4,)")/f576)$/$0EK$Ei)(+$.")$.")(?9:!+$86)EK$,7+$

’ ’Žk ’
PL,$G\l$GUW;9:,
/$v46%>(?$f>$(+E $f(+$?>,-0!$fE bE $N
#(W;&;(f1$?>$.9:&^*(>=>/9,SW)/$0,)9:Ei)(?$.")$ *-6DC w |6):E $,->$."ST6),)$.6),%!>*4N


: 6 
  <       <  "   
+ (+*&F*G>$(]!,)")*z),)/E $,-UN 5o9:EKE $#!$8+$6% >$?2"Sp(+(?b
 , ,)$.&F9:6%(+(>'")9,)1.3'
#  
$Y2"SUM9:/(a+9:6)]/$
*-6DCFW;$2576)$.";,%21$.1U2OT/,)*-!!>*2$Y2134/(+$EK$,-
6),)$.*-/H>*4W%9:,d,)9>(?$8(+*+6)Hj4N;PRv"),)f>9:6%2!$26%+(+$21W;Žp!,)*:/H>*
$Y2?>(?/1G>$N

+ 1
A]$EK(+576)$N

+


1 R [
u
-„f#, &F96)(+(+!]46)+?{1jJ%9:/?/(f")$.(+$EK&%31$( $ &;4( ;I/2!9:(+
+$.&^9:+$(>H]6),d&)(+9:i%/_E $#")$.19:,-M:$(?:$,%1$k576)ŽW;EKbEK$0T/V&^$6%2b+(+$.(+*+9:!6
/1OWD19:,)"%6)/(+!]w'")$v19:E &)/!1U4+/9:,)X/,76%+/!$N)uv*U4,)EK9/,)W%1$G++$
>(+,)Ye€9:(+EK4>!9:, (?$?+$+(+_2!,->*(?$?>,->$&^9:6)(f(+*")6)!(+$./$#,%9:Ei)(?$"%$.M4(?3i%/$N
i[„ao!$, $,-+$,)")6lW;/$8E bE $#&)(+!,)1!&F$0&F$6%2b>(?$86%>!/!+*#UM$1"S46%>(?$]EK9c$,),)$N

]_^ ^:`ba\cedfAa = iF


‰H hHw[ [ [ H -: gH
… R :R[ [ [AR -:”‡ ;  R ;  R[ [ [{R -; :    [
mL(+9:6DM:$(2576)$W;&^9:6)(a+9:6) )9:,~


xy{z}|C~\€y{‚ : + : +  :  
 * +  
  +  *+ + 
  : 
+ ƒ : + [
+
3
kp&)(?_ˆ` 
$
 ;
,  ;
 + ; 
: +
,

, ,


+   [
: ,
*+ *+
,  ,  ;
`0!,)+

]_^ ^:`ba\cedfAa = ir‹
j 2H   H „V   ”H  #  ƒ  [ …
R  R  R $R ŠR  ; u #/R$ lR  ‡[
9:!$,- ;UM:$1
mL(+9:6DM:$(
$ ˆƒŽI9:+19:6 ‹4‘:‘:‘-„

xy{z}|C~\€y{‚
O O g[
 … 2H  H e‡ ƒ  R   R   R „R /R  u … #/R$ lR  ‡[
m4(2YcnEK*G>(+!$W)9,d&^$6Dv?6)&)&^9:+$(v576)$

’
mQ9+9:,)

 … 2H  H e‡  … 2H  gH  ”‡ ƒ   R    R MR  u … #/$R   lR  ‡ u  >R >  


#,dd

’   ƒ …  ”‡ R <   =  ’   =  u <   ’    = 


ƒ<  <  ’   ’ =   … Š = R  <  ’ / „R R   ’ R = R ;  ”’ ‡ u ’ >
ƒ  ’  ’ u ; u
&'
 O _O …  ŠR …  u  g[ 
–k  ƒ  2… kmH HJnhH–nHJ”n‰‡ ‡ ƒ ’  k  2RrH [ ’  ks R gH  n km HJen ‡ ” k>n   –kCn  ƒ [  ƒ g[
 (
# ")9:,)1
PR04/,)? KUM:$1*-!!>*0++
mQ9:?9:,) $ )`0!9:(+ $G
PRv9, 
ƒ R R u n’ ;
ƒ n;™ n; u ˜ ’ n 
ƒ n ; ™ < u n ’  ”n Rr˜ n R  ; =


ƒ  n ; ™ … n ’ ; ‡ < u n R2n ”n R [ u n†R ; =


309:,)1  … 2H  H e‡   … kmHJnhHJn‰‡  H
UM$1*-!!+*#+?
ƒ ;  ƒ HJn ƒ H #  ƒ H
 ƒ  ƒ  ƒ [
;UM:$1*:/H>*#?+
1Npw%NT"SN
; ;
   
1 ;
j –kmHJnhHJo{H     7 k>n1o  ƒ [
k Rrn Rro R Rk>nqRr; k>oqRk sRrn1oˆRrnsRro   g[
„ 9:!$,- '>$/f576)$
m (+9:6%M$(2576)$
L
ƒŽ‚2(++ 7: ‹:„
;
j –kmHJnhHJo … g[ ˜  ˜ ˜ ˜  
kNRnqRro'‡ k Rn Ro R u ekCn1oA[
‹:„ 9:!$,-
mL(+9:6%M$(2576)$
:
Œ-„ j –kg1HJk”hHw[ [ [ HJk;:„V  ”H
9:!$,- *

;UM:$1 k ƒ [
 k  … k  ‡ O ’  : ‡ [
: …  , : ;

,  ; ’
mL(+9:6%M$(2576)$
;
j l m
k J
H h
n H -n k ƒ  g[
k  Rrn  R k ;  R k   [
„ 9:!$,-2")$6nC (+*$/ [UM:$1
mL(+9:6%M$(2576)$ {ƒg`v6%>(?/1jJ)$‹‘‘:‘-„

: 
  V ”H
„amQ96)( : ƒ
)9,&^9:+$
 :
R  ;  : ƒ R  ;  [ 
$

mL(+9:6%M$(2576)$.3'+6)H>$ … :e‡ ;  ; … :e‡


a$Yv1(+9:!++,->$W;$G25n6%$/+6)H>$  f"%*1(?9:/?>,-+$N

: j  
„ 9:!$,-
kg R k”lk g‰HJRXk”w[H6[ [ [1[ R[ HLk : k;V : m  eR [ k;:  [ A gj j
6%,$,7+/$(W;$
mL(+9:6%M$(a576)$k
k”:Rrk”˜ k”˜†Rk”™ ;k :lRrkM kMRrk” lƒŽ‚ 7:7 „

„' j –kg1HJk”hHw[ [ [ HJ;k :"  Tkg k”h[ [ [ ;k : ƒ [


9:!$,- >$/f576)$
;
9 6 
  <       <  "   
+ :
, … u Rk ‡ [5 v
mL(+9:6DM:$(2576)$  +  : {ƒ oJ)/,%$ 8797 877 ‘-„

9 j  :  1H hHw[ [ [ H -:  eH;$Glkg‰HJk”wHw[ [ [ HJk;: V [


TK …  ‡
„ 9/$,- 6),d$,->!$(W ";4,)

#, &F9:?$
ƒ  [
 * ;  30*+$(?EK!,)$(f/&%/6)f:(+,)")$.19:,)?>,->$ o>$//$
,

* k  …  ‡  K … ‡  k   [
: :
576)$
,  ’ , 
7(
„ 5o9:E E $,-]")9:H?@9,~(+*&)(?+/(
 
i 9:6)/$f";,) iF94</>$f")$8egh9,w'E /,)!EK!+$(
^
/$8,%9:Ei)(?$8")$.&;/(?$f")$i^9:6)/$f576)ˆ&)&;4(?>!$,),%$,-vw6%,)$.EKbEK$8i^9</+$

‘-„ j 2H H
 " +$/]576)$  
9:/$,7 R ŠR ƒ [
mL(+9:6DM:$(2576)$

R
; 

; 

R R
;  ;
 M[
j   ; ; ;
‰H wHw[ [ [ H R !  h[ [ [ ƒ [
: „ 9:/$,7
u
6),d$,->!$(W;$
+ -: 1 +$!f5n6%$ -:
30*G>$(?EK!,)$(L!$]EK!,)/E6%E "%$ * + /21


.- -):     R ˜ ˜R [ '
+  + 1  1 ^ƒ 3kp&)(?_ A29:6%E|,%; /$ 77

„

j lkmHJnhHJoAH
‹:„ 9:/$,7
 +$/f576)$ 7kŠRn†RoˆR ƒ [
k>n‰o†Rrn1o NRro CksR>k>nsO ; R ;  ! ; k>n1o m[ 7
  u  u
mL(+9:6DM:$(2576)$ {ƒgmZ(?9:&^9:+*#YI Œ-„

j  gp

1H hHw[ [ [ H -:  e[
Œ-„ 9:! 6),&^9:HcD, :EK$0")$.")$:(?* w'19n$‡ 1!$,->2&^9:+H>!e€W

W   Z R W    Z RQ[ [ [{R W   - :   Z  W  … ‡ Z  [


    ˜  ;
$ ";,)

mL(+9:6DM:$(2576)$
;
j lkg‰HJk”wH6[ [ [ HLk : V   H -kMRk>:R[ [ [{: Rr;k : ƒ [
 : ; C˜  kg k”h; [ [ [ ;k : • …  ’ ‡gR *  k ;  [ ;
„ 9:/$,7 )M$1

mL(+9:6DM:$(2576)$
; ,
 j GTHJIHJK/ H

„ 9:/$,7 576;4+(+$.&^9:/,->f?6)(]6),)$+&)J%_(+$")$.(>Uc9:, W%>$/]576)$

GNI  GsK  G
 I K  I
 K
ƒ u ˜ [
mL(+9:6DM:$(2576)$./$#+*>(+_"%(+$ /GNI K
7' 
$?2(?*:6)!/$(NVƒgmL(+9:&^9:+*#YI „
7

: „ j lkg‰HJk”wHw[ [ [ HJk;: kg k”h [ [ [ k;: … Tkgkg Rrk>k” †RQ[[ [ [ [[{Rrk;k;:e: ‡ t [


9/$,- +$/f576)$

… g
k 2
 
R ”
k :
 
R [ [ A
[ r
R ;k :e; ‡ ’ … ’ kg—‡ … ’ k”’1‡•[ [ [ … k;:>‡ O  : ;  
mL(+9:6%M$(#576)$K ; NƒŽmL(+9=@

&F9+*#YI 7 79 „
; ’ ; ’ ; ’
‹‘ 6 
 <       <  "   
 

     

qV$&%(+/,%1/&^$#&)(?*1*")$,70+$.:*,)*(+/!+$8";,%2!$81U]"ST6),)$.e€9:,)1G>!9:,~19:,-M$GCD$N

   ‚m€ ~b€y{‚ !
 H;")*Gz;,)!$?6)(]6),d/,->$(?M4/!$  "%$ 2WV$?2")!+$ y{‚-C B  B./9(++576)$4W
 VXW H Z\H)$]>96) 2H_V  H;9:,d~
‚ ,)$8e€9:,%1>!9:,
v
&F96)(a>9:6D
 ; …  „R …  ‡DM‡ O   … ‡2R …  ‡  … M‡ [
A2$E|4(+576)$N
; ’ ; ’
5 $/'+/,)!z;$8576)$/$8"%9:E|4/,)$

-„(o
ƒ  … 2Hg‡ V      … ‡  $Yv19:,-M:$xC%$4N
  y{‚  C B  $Yv19:,-M:$xC%$4NlPR]>96%21$.576)V+6)H
’
i[„]#,d"%!]576)$ $? .!9:(+?5n6%$
p";&D>$.>,%v")H‡ 16)H>*f"%6E 9:E $,-]576)$.OT9:, (+$,-M$(+?$k!$f/,%*-!!+*N

7K 
1„]#, &)(+9:6DM:$/9:(?f576)$.3e€9:,)1G>/9, $?v19:,-M$GCD$k?6)( +V$G2+$6%/$E $,-2+
/$2&^$,-+$2")$]19(+")$v")$ .+9:,-21(?9:/?>,-+$2$, e€9:,)1+/9:, ")$.!$6)(?
$GCn>(?*E !+*N
m(]?6)!+$W;+ $?2")*(?!M4i)!$8+6)(
 $ /9:(?8

K 
$?v19,7M$GCD$k?V$]?$6)!$E $,-2+ $Y21(+9/++,->$.+6%( N
PRv"),)]1$.1U4W $?v6D@ ")$?+6)2")$8>96%>$v?$fj,%:$,->$N

"[„aV$,d")*19:6)!$85n6%$?

$?2")$6DC e€9:/f"%*(+HM4i)/$0+6)(
 $ $  !  [
qˆ'e€9,)1+/9:, $Yv19:,-M:$xC%$+6)(
 ?V$2+$6)/$EK$,-v? '+6)(

k
$„f‚v,%$.e€9,)1+/9:, ?+(+!1>$EK$,-]19:,-M:$xCD$k19,7+/,76)$")E $]?9:,dE|=CD!E6)E $,d6),
&F9/,-],)9:, /,-+*(+!$6)(]w
 ƒ€$,d$\F$W;+ 
4++$!,7]?9:,dE|=CD!E6)E $, |576)V,ST$?

… k H  … k ‡ ‡  … kNR H  … kŠR ‡ ‡
&;]6),)$.i^9:(+,%$"%$ /9(+W%9:, &F$6%]>(+96%M:$(v6),%$19:(+"%$"lp$xCD+(+*EKH>*
7K
’ ’
a$ f576)V,lp$?v&;]6D@ ")$?+6)v")$ „GN
 
-;2<>= ?A@CBED' 'i <)( dm`ba f <+*2D•DCi6DCi 7
 ?

‹
‹:‹ 6 
  ("  6


j    [
9:!$,- 6),$,->/$(W;$G
:
6),%$.e€9,)1+/9:, 19,7M$GCD$k?6)(]OT/,->$(?M4!/$

`0!9:(+W)&^9:6)(a; >96)](+*$/ •1H mwHw[ [ [ H :„V
ƒ H  +$/f576)$ *
‰H wH6[ [ [ H -: V  H ,  ;
;$f+9:6)


;9:,d 
: :
 *    O *    …  ‡ [
,  , 

  ƒ  ƒ [ [ [ ƒ -:[
P 2+
R $YvY>(?/1+$E $,-219:,-M$GCD$W^OT*-4/!+*01H@"%$+?6)],Sp !/$6~576)$
&^9:6)(

?A@B | C  B  
mQ96)(

: 
ƒ $?v*Mn!")$,-UN[qV$.1U4
ƒ u $?23")*Gz;,)H>/9,d"%$/19:,-M$GCD!+*N

*   
; : 
#, (>!+9:,),%$$,)+6%!>$.&;4(f(+*16)(+(?$,)1$?6)(

*    R -:  -:  …
W;$,(?$EK(+576;,-2576)$

:   ‡D/R :   -:  


 , ƒ :  ,  :
  ƒ
; ’
 
 , ’   ,  ’ :   ƒ ; [
  * 
UM:$1  ƒ * 
- :  
- $

; ;
A]$EK(+576)$4N

q $8>J)*9:(+_EK$ : ,ST$Y#!9:(+o5n6lp6),%$19:,)+*576)$,)1$")$.319:,)1MnH>*.")$.3
[
V
e€9:,)1G>!9:, +6%( 
‚m€ ~b€y{‚ 
j 
9:!$,-
:
N

+ u 6),$,->/$(W kg‰Hw[ [ [ HJk;: •1Hw[ [ [ H :H


'$G p")$](+*$/]?+(+!1>$EK$,-v&^9:?!>He€

,  ƒ ; [
UM:$1 *+

   …
‡ ƒ

 k + [
*+
: + +
#, ")*z;,%!23ke€9:,%1>!9:,

+  …
‡  y B m‚ ‚ B Yy @  @B ,  l k
?6)( &;(

 [
q $.,)9:Ei)(?$
V o$?v3 ")$],)9:Ei)(?$ &F9:,%")*(?*$&;(
/$

PCD$E &)/$N

ƒ [ [ [ ƒ : ƒ  ; !
3#,%2!$81U]9C •
: + +
 … ‡ ƒ ;  k
* +  y  B ‚m‚ B >@ € ~  ~\€6| B sk [
$Y0/ .")$
; ,
‹Œ
 +
 … ‡ƒ +:   y B ‚m‚ B  > @  y{‚m€6| B –k [
’;
$?23 8")$


k 

,
 + : +
+
 …u ‡ ƒ


$?v3  y B ‚m‚ B 6|   @ ~b€6| B")$lk [
, 
#,E 9:,->(?$8")$.&)/6)f576)$3ke€9,)1+/9:, +$.&)(+9/9:,)$8&;(f19,7+/,76)H>*8$, g
 H;$G
: + +
… ”‡ ƒ  k $Yv/  y B ‚m‚ B  y  ~ @ €6| B.")$–k [

576)$ 
: + 

, 



 -;2<>= ?A@CBED ' 'i )< ( dm`ba f < -D f BE=D. DCi i D  * = ? * ?AD 769 : +

?

j lkg‰Hw[ [ HJk;: H


9:!$,- •1Hw[ [ [ H :" 
),)9:, >9:6%2*-6DCFW;$G
, ƒ ; H
*+  UM:$1

 ! …  
… ‡—‡ l[
>9:6%]zDC%*N[`0!9:(+8

“
3e€9:,)1G>!9:, o$?v?>(?/1G>$E $,-]1(+9:!+>4,7+$.+6)(

lkg‰Hw[ [ HJ;k :" gH t H  …


‡ O  … ‡ H
1Npw%NT"SNV

kg ƒ k> ƒ [ [ [ ƒ k:[


mQ9:6%(]+9:6) )$ )9, 

UM:$1*:/H>*#?+

?A@B D| C B z}€ @B +   B y{€ !


j k 
 
+
”[ k
 >96)]/$
S +9,72*-6DCFW;/l$?213!(a576S4/9:(?f3e€9:,)1G>/9, $?v19:,)Yj,-+$

: + +
+6)( [#,?6)&)&^9:+$EK/,->$,;,-f576)$/$ ,)$.?9:,-v&)f>9:6)]*-6DCFN

/‹ 7
8#,  …  … + ‡—‡ ƒ ;  * +   k+ ƒ 
… ‡
PRUW;&)6)!+576)$ 
k ƒ , lk gH
&^9:6)(a>96%

 … ‡ [
ƒ
)9:,~
; +   … k ‡gk
 : + + + 

 $
… ‡ ƒ ,  + : + +
 k
m(2!/!$6)(?W ")9:,)14W;19:E EK$81H@ ")$?+6)

+:   …k ‡ , : +
+ + 

  $
… ‡ ,  : + * +   … k + ‡  + : k +  [
ƒ +  ƒ , ƒ ,  

, 


: + 
3 lp
kp&)(+_v/(+_:/$8")$.Ž :&)!>OWn9:, $,d")*")6)!


   …  … ‡ ‡ ƒ   k  +  

,
‹ 6 
  ("  6

: +

#6 $,)19:(+$    … ‡
ƒ , : k + [
 +  

`0!,)+ŽW%$,&^9:>4,7  … e‡
ƒ + ,  k H)9:,d&)(+9:!9:,):$  &;4(219:,->!,n6%!>*8$,DN
 

j 9:H [
qˆe€9:,)1+/9:, 
:; + + $: ?v+ !9:+ (+ f Y>(?/1+$E $,-219:,-M$GCD$k?6)(  e[ +
309:,%1  * +  k ’ “ * +  k ƒgOT/,%*-!!+*v$?2?+(+/1G>$.1U4(2!$k ,)$.+9:,-2&;
,  , 
k 
>96)]*-6nClN„
: + + : + +
 … ‡ ƒ *+  *+
,  k ƒ  … ; ‡ [
, 
3kT9Cy
j H l[

t [ t [
9:!$,- ";,%


 V  [ [ [  H
 1U.)?
+
j;  ƒ
 9:H
[m{9:6)(f>9:6D

%9:,&^9:?$ Tn ƒ k + [
: + + ; : + +
 
;
*+  n *+  n

3kp&)(?_v-„GW)9, 
: + + , : + + ,
  

*+  k *+  k

,  , 
50Npw%Np"lN 

 … ‡  … ‡ 
  [ ’“
9:6 $,%19:(?$ o1U4(2/ke€9:,)1G>/9, $YvY>(?/1+$E $,-

t[ t g[  V  [ [ [   H
1(?9:/?>,-+$?6)(

uj  1.;+ +
ƒ9:H

[m{9:6)(f>9:6D   %9:,&^9:?$ Tn ƒ k + [ 
; : + + ; : + +
*+  n *+  n

3kp&)(?_v-„GW)9, 
: + +  , : + + ,
  

*+  k *+  k

,  , 
50Npw%Np"lN 

 … ‡  … ‡ 
 ”[ ’“
9:6 $,%19:(?$ o1U4(2/ke€9:,)1G>/9, $?2?+(+!1>$EK$,-

")*1(+9/++,->$.+6%(
 !,;/$EK$,-UWn3e€9:,)1G>/9,
  $Y2?>(?/1G>$E $,-]1(+9:!+>4,7+$?6)(  :W;+6)(  eH;$
$!/$#$?219,7+/,76)$.$, %N 
‹ 
m(]19:,)+*576)$,-UW^$!/$#$?v?+(+/1G>$EK$,7]1(+9:!++,->$.+6)( l[
A2$E|4(+576)$N
 ! +
T k [ p
m 9:6%(
Q

kg ƒ  {kg k” [ [ [ k;: [ : + + 


Sc '6),d,)9:Ei)(?$8z;,)S")$
 [mQ(f?cnE *>(?/$.")$2( :/$W)9,&^$6%2?6)&)&^9:+$(2576)$

: +  + 
 …  …
‡ ‡ ƒ  * +   k ƒ ;  k  * +   kgk  
  

`0!9:(+8
;
: +  +  ,
3 C  …  …
‡—‡  … kg ‡R  * + 
,
k
kT9 
ƒ + ;  kg  +
 t kgk  O [ 3 - t kgk  O [
#(W;&^9:6)(a>96% —H
;9:,d
,
09:,)1 

: +  k+ : + ; k + : + 
*+  •
  R *+ 
kg  ƒ : ,  + kg+   ,  ƒ ; O *+  [ ;
, 
PR

 … • ‡ O  * +   k  O g[
`0!,)+L
; , 
kg 
  … 
‡ ‡ ƒ  … kg—‡
…

 
… ‡ ƒ  {kM k>  [ [ [ ;k :  [
#,$,")*")6)Hfeg1!/$E $,-f576)$

5
0NwDNp"SN   
t !


7kM ƒ
 {kg k” [ [ [ ;k :  [ : + + 
@BmQ9:6%(

… 
… ‡ ‡  … kg—‡R  * +   k H
#,?6)&)&^9:+$.1$G++$e€9:!f576)$   

5
o9:E EK$81H@"%$+?6)W;9,~
+
 ƒ + ; ,  kM 
k  — [  k
UM:$1
kg ; &^9:6)(a>96%  09,)1 3  t kg  O ; [

  … ‡ ƒ
 {kg k” [ [ [ k;:  [
PRv9,d?6)!2!$8(>!+9:,),%$E $,-]&)(+*1*")$,-v&F9:6%(219:,)1/6)(?$'
  



  … ‡ ƒ  {kg k” [ [ [ ;k :  
 
… ‡ ƒ
 {kg k” [ [ [ k:  [
/,)/$EK$,7#
$G
lH
     

V !
{kg k” [ [ [ ;k :  t  … ‡ t  {kg k” [ [ [ ;k :  [
 
PRv19:E EK$4W)3ke€9:,%1>!9:, $?2?+(+!1>$EK$,-v1(+9:!++,->$.+6)( V9:,'")9:,)1
&F96)(a>9:6D      

 T= i 6<  , DCi c D
mQ9:6%( • ƒ [ [ [ ƒ : ƒ  [
;
‹ : 6 
  ("  6


3#
 ,%21$.1U.
{kM  k>  [ [ [  k;:  t  … ‡ t  … ”‡ t  … ‡ t  … u ‡ t  {kg  k”  [ [ [  k:  [
’;
k +
#6 $,)19:(+$ 
+:
;
 +

{ kM  [ [  k;:  O + :  O +  k O  ; * +  k O
+ :  :


 ,  O  {kg [ [ k;: 
 
 ,
,   + ,
`ˆM:$1*:/H>*#?V$2+$6)/$EK$,72?S>9:6)]!$ k +9:,-2*-46DCFN

]_^ ^:`ba\cedfAa = iF


j  kmHJnhHJo     7 kŠRn†Rro ƒ [
9:!$,-f>(?9:/f(?*$/ #?+(+!1>$EK$,-v&^9:?!>He€o>$/f576)$

3
0*EK9:,-+(+$(]576)$ ksR k;  R qn R n;  R oqR o;   ; ;  [ ;
xy{z}|C~\€y{‚ < 
 = 
 “       kNRnqRro
q e€9:,)1+/9:,
ˆ $?219,7M$GCD$k"%9:,)1W;"l&%(+_]Žp!,)*-4/!+*0")$ $,)+$,SW
9:,d~ 
; W ksR k ;  R n:R ;n  R oqR o;  Z  R  ; W k ; R  n; R o; Z 


ƒ  ; R  ; W k ; R  n;  R o; Z 


; R … k>n‰; o'‡  
 "Sp&)(?_]`

  R  
; kŠRr n:Rro   "l&%(+_]`

ƒ >;  R  


    ƒ ;  ;
C kNR
k;  R n:R ;n  R oqR o;  ;  [

"ST9

]_^ ^:`ba\cedfAa = ir‹


j G-HLIHJK
9:!$,- !$2,):!$f"ST6), >(?3,)/$N


GR 
 IR 
KO     GR  IR   KO  [
mL(+9:6DM:$(2576)$

u u
$G
‹ '
xy{z}|~b€y{‚ 

-W gH Z 

GR 
I R 
K O  
  GR IRrK  
     H
qˆ'e€9,)1+/9:, $?219:,)1UUM$k+6%( V")9:,)14W;"Sp&)(+_]OT/,)*-/H>*v")$ $,)?$,}

  ƒ  ƒ u
UM:$1*:/H>*#?+ˆ/$0+(+/,):!$#$Yv*576)//4>*(>ŽN

qˆ'e€9,)1+/9:,
 
,lp$?v,)S19,)1UUM:$,)S19:,-M:$GCD$+6)( -W H  Z\[
[#,d,%$&^$6Dv"%9:,)1
5
—p TG  I  K7[
19:,%1/6%(+$/,%+S";4,)]/$81U42*,)*(>ŽN o$&^$,)"),-k

j G O  [
m(]YcDE *+(+!$8")$2( :!$W)9:, &F$6%2+6)&%&F9:?$(2576)$

u 
@ 

W gH  ZH
  GrR   IR  KO    Gu RIRrK     H
mL6)/?5n6%$ $?219:,)1UUM$?6)( ;9,~

 ƒ ƒ u
j G  !  #I  K7[
UM:$1*:/H>*#?+ˆ/$0+(+/,):!$#$Yv*576)//4>*(>ŽN

u u 
@  ;!9:(+

TW gH  ZH
  I R  KO    IRrK   u  G 
G [
PRv"%9:,)1W;&%6)/?5n6%$ $?219:,)1UUM$?6)( %9:,~

u u  ƒ u
 GR   I R   KO  GR 
 u ’ G u  ƒ u u

 uG R u 
G R
3
09:,)1

ƒ ’ u  
 u  G u u  ; 
ƒ ’ u u ’ u; 
R
H u
G t VZ u H  Z 
 G  [
  GR   IR u  u K O  H u  ƒ u;
OT/,)*-!!>*2*>,-2?>(?/1G>$.1U( ˆ$G]")9:,)1

u
/,)/$EK$,7# [UM:$1*-!!>*0++V!$#>(?3,)/$#$?
*576)!34+*(>4ON

]_^5^5`ba\cedgfha = i  
… N
k R
lk HLnAHLo "! k  R u n  R eo  O [ †
n R e
 6
o ‡
u
mL(+9:6%M$(2576)$W[&^9:6)(a>96)

xy{z}|~b€y{‚ !
‹ 9 6 
  ("  6


kNR u †n R eo O k  R u n  R eo  H … kŠ R u n† R eo'‡  O [


3kp&)(?_2OT/,)*-/H>*2$,->(?$./$]E 9c:$,%,)$v(+H>J)E *+/576)$#$G25n6)")(>=>/576)$'

k R u n R eo !
;1Npw%NT"SN

 !  
1
~ !
B3 
lk HLn  !>k  Rn  O  … Nk Rn‰‡  [
„ 0*G>$(+E /,)$(f3'&)/6%f&F$G>!+$819:,%?j4,7+$ >$/!$#576)$K
&^9:6)(a>96)
: +
‹:„ j ‰H hH6[ [ [ H -:„V +  
9/$,- *+

+$/]576)$
ƒ [
:  , ;
,  ’  ’ [
mL(+9:6DM:$(2576)$ *+ + 

j lkmHJn gH k ;  : V   n : ;
 : [
Œ-„ 9/$,- )$ #N
= R > R R
mL(+9:6DM:$(2576)$
n ; k u {ƒ€`06%+(+/1jJ%$k‹4‘:‘:‘-„

   –k HLnAHLo gH
;

k kR  n1o R n  R n  o‰k R o  R o  k>n   R  [ 43


„amL(+96%M:$(v576)$4W;&F9:6%(]+9:6)f(?*$! $G ;9:,d~
{ƒ k4&)(+_0#YI ‹‘:‘ „

 j
 lkmHJnhHJ o TW gH Z[  ;
„
k R ; n R o R … kC‡ … n‰‡ … o'‡ O [
9/$,- #";4,)

j 79 n†Rro†R ; oqRrksR ; kNRrn†R ; ; ’ ; ’ ; ’ ;


mL(+9:6DM:$(o5n6%$k
ƒŽ‚ ` ‘:„

: j „
 9! 6[ j H  V W : R  + W}H V   [
6),$,->/$( 9:/$,7  )$G

j lkg‰HJk”w; Hw[ [ [ HJ;k :    * +; k [



9:!$,- '";4,)

.>$/]576)$
ƒ
:
k +
R   ,
 ;
mL(+9:6DM:$(2576)$ *+
, k ;
+  
 ; R 





UM:$1*-/H>*#?V$]?$6)!$E $,-2+ 2kg ƒ k> ƒ [ [ [ ƒ k : ƒ  ; [


' j  t  OQkmHJnhHJo{H  H 7O C[ 
„ 9/$,-

… kNRnqRroqRNRA‡ k; R n; R o; R ; R ;  O u  R    [  
! ’ 
mL(+9:6DM:$(ˆ576)$f

ƒŽ‚ j 7'' ` „GN


‹ 7

9  
„amQ96)(
; R

$,-+/$(azDC%*4W[")*+$(+E /,%$(f3&%/6)]&^$>H>$#M4!$6)(f")$./+9:E EK$
 R [ [ [{R  ::
u [
/9:(?+576)$ ‰H wH6[ [ [ H -:
+9:,-2")$2(+*$!a>$!]5n6%$
;  R ;  R[ [ [AR -; : ƒ
ƒŽm{9:/9:,)$ 77  „

7 j –kmHJn  gH  H 
„ 9:!$,- )$G R ; ƒ [
+$/f576)$
k R n  k>nh[ ;    ;
;
  
mL(+9:6%M$(2576)$

j ‰H hH6[ [ [ H -: [
‘-„ 9/$,- 
; R R ; †; R R [ [ [>R -:l; R

mL(+9:6%M$(576)$

79
R …  h[ [ [ -:”‡ 
ƒŽmL(+9&F9:?*

#YI „ ; ; ; ;
: „ j ‰H h™ H ˜w+H m™  ™ + ™   ˜ m™ ƒ [
9/$,- >$!]5n6%$

* + ˜    * + H * + +[ ; 8779
,  ,  ,  ;
mL(+9:6%M$(2576)$ ˆƒg(+, „

j 2H  H  <  ˜ g[ ˜ ˜   <  <  < 
R  R  = R  =  R  =  R # = [
‹:„ 9/$,-
mL(+9:6%M$(2576)$
 $
Œ-„j  H 
‰H wHw[ [ [ H  -:  -: R :R 2[ [R [hR - : :ƒR[ [ [[AR -:
9/$,- %$ '+$!f5n6%$

mL(+9:6%M$(Z576)$. ; R  R  [ [ [ R -:     ;  [
j H H ; ’  ; ’
„ 9/$,- 
 ; ’  V  [  ’ ;
!$],)/$f"lp6), >(?3,%:/$4W%$

 : = > :  R :   : = > R  : = >  : = >   :   [



mL(+9:6%M$(2576)$
       

u   u u u u u

 j
 u
3

„ 9! 6),$,->/$(N 0*G>$(+E /,)$(f/$8E /,)!E6)E ")$

†R ˜†RQ[ [ [{ R 


R [ [ h
[ R :
R -: R ˜:R[ [ [{ R  -R :  R[ [ [AR  R  :R [ [ [ [AR -:m 77'
/9:(?+576)$ ‰H wH6[ [ [ H -:    >$!f576)$ : ƒ ˆƒ
{6)(+576)!$ „
;
Œ:‘ 6 
  ("  6

 

          
  S 
  X  S

! 9GF KMH7. _&! 9GFLF  ( 045$Ž8)9GF


30*z),)!+/9:,SN
j lkg‰HJk”wHw[ [ [ HJk;: 6[
–ow1HJo1hHw[ [ [ HJo : gp
9:!$,- ")$](?*$!
qV$]19n$‡ 1!$,-> ")6&^9:HcD, :EK$
:
… ‡ ƒ … „Rkg—‡ … „Rrk”1‡•[ [ [ … „Rk;:>‡ ƒ * o  : 
, +
 y{‚  ~\€y{‚   ~ @ €6| B  z  + B ‚m~  € @B –k [


mo  –k
+9:,-v&)&^$!*]/$    .")$
`0!,)+L $Y0/+9:E E $8")$]&)(?9D")6%!>f"%$
1jJ%9:/?/ ~w /ke€9:/N

o ƒ : +
#,'")9:,%1 

oh ƒ * + ; k + 1


o‰ ƒ ., - * + /21 -): k k




[[[
o : ƒ kg k>h[ [ [ ;k :[
L
U g
k 1
 L
H >
k h
 6
H [ [ [ L
H k
:
M p M p
q p!,7+b(?$")$ 1$'e€9:,)1G>!9:,)$?576)$~OT9:, ")*EK9,7+(+$ 576)$~+9:6%'&^9:!cn, EK$K?cnEK*x@

$, oh‰HLo‰wH6[ [ [ HLo :C[


>(?/576)$a$, 0&F$6%Lp$xC%&%(+/E $(2ƒ€")$oegh9:,'6),)!576)$„19:EKE $o6%,'&F9:Hcn, :E $

Œ
Œ-‹ 6 
  
   (6<
    
 %$)  " $(< $)  
y{‚  ~b€y{‚   ~ @ €6| B  z  B ‚ 

~  € @B  y B ‚m‚ B
m4(?e€9:!Wn/^$?]&%/6)B19:E EK9n")$v")$#19,)+!")*(?$(a!$
")*Gz;,)!$]&;(.
 …
  

g‡  o H


 ƒ  ; o  ƒ


’    ƒ H u Hw[ [ [ H  [
;&F96)(

 ;

-;2<>= ?{@CBED ' 'i2<D( dm`ba f < "* D D3 f = i 7


j lkg‰HJk”wHw[ [ [ HJk;:    [ 
 ? 

)V  H u Hw[ [ [ H H
9:!$,-

’ ;   m     O   H
mQ96)(f>9:6% %9:,dd
;
UM:$1*-/H>*#?V$]?$6)!$E $,-2+ 2kg ƒ k> ƒ [ [ [ ƒ k:[
?A@B | C B [
 !  kMRk> 
m4(2(?*16)(+(?$,)1$?6)(
@BmQ96)(
 ƒ
;8,
u    ƒ kg k”–O u 
"l&%(+_2/$81jJ)&)!+(+$ N
j u j 

 k $ HLk $ Hw[ [ [ HJk $:m g[


@ 9:H z%CD*N 6)&%&F9:?9:,)]576)$/$#>J%*9:(?_E $8+9:HfMD(+V&^9:6)(a>96)

j lkg‰HJk”wHw[ [ [ HJ;k :  j
(+*$!

j
9:!$,-
+ + 7kg‰HJk”wHw[ [ [ HJ;k : m 1
DN T//f?9:,-]>9:6)]*-6DCFW;9:,d134/(+$EK$,-fOT*-4/!+*N

7 o $H $ k H
!,)9:,SW)9,d&^$6Dv?6)&)&^9:+$(v576)$ a,)$.?9:,-v&)f>9:6)]*-6DCFN

o $m ƒ  $m ƒ o $: ƒ  $: ƒ g [


#, ,)9>$ !$aM4!$6)(?f19:(?(+$?&F9,)";,-06DC ;$29:, &^9:+$

+
`0!9:(+W)&^9:6)(  + ƒ gH Hw[ [ [ H H + 
%9:,~
o ƒ o $ Rk;:” o $ m H ;  ƒ   ’   +$ R   k;:  +$ m [
+
;$]"%9:,)1
<  m    = GRrI-k :RrK7k : H
G ƒ <…  … g‡  … =   $  m  ’ $    ƒ …  …  M‡  …  $ ‡  …
 
m4(2?6)!+$' [UM:$1K

I ƒ  ’ /R ’ ‡ ; /R ‡   $ m  ’ $ R  ’ ‡ ‡  $ C  $   u …  g‡  $ m  $


K ƒ < ’ =  ; $ C  $ ; <  $ m = [ ’ ’ ; ’ ; ’ ’
’ ;…  ’ < 
 $ m $  C$    $ mt  $ $ $ ‡  H  $ C<   $ $ mt $ =  $ m = H $ C $  $ m $ [
#(W)"l&%(+_2OTJ-cn&F9+J)_?$.")$.(+*16)(+(?$,)1$W^9:,d~

$
    mG     …t $ ‡    HLI   $ m $  H  lK   t  <  $ m  =  [
"%9:,)1
m4(2?6)!+$'
• <  t m ’       = t#u   … $  ‡   R $ mt  ’ $ k:   < $ m =  k :
;$G
PRv/,)?
  ’  t ’ <  $ $ u m  ;k : =   ’ 
ƒO g’ [   ’  
  R"(      
G %$    Œ:Œ

3kT9C!$8(+*+6)Hj4N
A2$E|4(+576)$N
PL,deg+ HUWD1$2/,%*-!!+*a(?$?+$,-2M43i)!$aE bE $8+VŽp9,~,)$.+6)&)&^9:?$&)!6)]576)$
/$ –k
?9:,-0&^9:+H>He€N

-j ;2<>= ?A@CBEDEF' 6i2<)( dm`\a f <  *2D


dmc  d-, ? a i 7
2kg‰HJk”wHw[ [ [ HJk;:" H
 ?
 ;/9(+f9:,d 

  …  1‡  [ [ [  …   ‡   [ [ [  …  :e‡  H
UM:$1*:/H>*#?V$2+$6)/$EK$,72? 2kM ƒ k” ƒ [ [ [ ƒ k;:[
 j?A2@BkgD| ‰CHJk”B wHw[ [ [ HJ;k :" H 
)$2"Sp&)(?_2/$f!,)*:/H>*o")$uv$ f+9:,SW;&^9:6)( O t H
m + +
 m   +  … J‡ …  ˜1‡  …  ™‰‡ ˜ [ [ [ … m  ‡ 
9:, 
 ;
     ,  ƒ   +        
O +   +


m   ,    O    [
3 C    O  H    
kT9
    )1Npw%NT"SN
  
`{M$1*-4/!+*#+V$G2+$6%/$E $,-2+V!lc '*:/H>*#";4,)]/$f!,)*:/H>*o")$uv$ f+9:,SN 
]_j ^5^5`ba\cedgfha = iF
lkmHJnhHJo{H  g[
 ƒ k>nqRk>oqRrk = N R> ˜ n1oˆ Rr= n N> R [ o   ƒ k>n1oˆRrk>nNRk>o sRrn1o m[
9:!$,- ;8,d&^9:+$
$
mL(+9:6%M$(2576)$ {ƒ€`0/!$EK:,)$U„

xy{z}|~b€y{‚ ! ˜  ˜
…  1‡  …  ˜1‡  H
`{M$1/$2,%9j4+/9:,%a")6d1jJ;&)H>(+$4W;OT/,)*-!!>*2")*+/(?*$.T*1(+H
9:6d$,)19:(+$
 
;5n6%V$?26),%$"%$2/,)*-/H>*o")$I1x@ qˆ6)(?/,SN

]_j ^5^5`ba\cedgfha = ir‹


9 !$,-
: lkmHJnhHJo{H  g[
mL(+9:6%M$(2576)$

k>; n R kC; o R k ;  R n‰; o R n;  R o ;   O  k; R n; R o; R ;  [
  

xy{z}|~b€y{‚
Π6 
 
   (6<
    
 %$)  " $(< $)  
… C
k 1
n o 
C ‡ 
k>n1o  … o sRn NRrn1oqRrk sRrk>oˆRrk>n‰‡ O  … n1o NRrk>o sRrk>nNRkCn1o'‡ 
qLp!,)*:/H>*0"%*+!(+*$$?2*576)HM4/$,7+$KƒgE6)H>!&)/!1U4+/9:, &;( „aw~

 o sRrnNRn1oˆRrk NR kCo†Rrk>n  n‰o ŠRkCo NRk>n sRrk>n1o 


hk>n‰o  O

9:68$,)19:(?$]
 
1Npw%NT"SN™  –O  ˜ H ! 
%5n6%{$?26),)$.")$v!,)*:/H>*B")$uv$ f>9:,SN

2 ! 9GF $'&)(+*-,/.‰$‰02(GF H 7*&)9GF


H

S!f!1l"lp*G>6)")!$(f!$a!,)*-4/!+*B")6~cD&^$ … ‰H wHw[ [ [ H -:”‡  g H C 


Mp …
)9 $?]6),

j  ‰H
wHw[ [ [ H -:”‡ gp *
&^9:!cn, :E $#J)9EK9:_,)$8YcDE *+(+!5n6%$N
  
 … ‰H wHw[ [ [ H -:e‡ ƒ  <   H "  H6[ [ [ H " : = H C 
9:H o6),&^9:HcD, :EK$4N
#, ,)9+$(+ * 29 ")*1(?!

 H u Hw[ [ [ H  [ 




OT$,%+$Eki)/$'")$+9:6%+$./$&^$(?E6%>4>!9:,)#")$ Fc ~")9:,%1 >$(+E $

*   R R  ;
";,%21$?>$.+9:E E $N
m4(2$xCD$E &)/$'
ƒ u u u

*    ŠR #    –R   $R   –R    [





ƒ R
* 

#  ƒ # g[

!
3
ƒ
… ‰H wHw[ [ [ H :”‡   y @B ƒ … 1H wH6[ [ [ H :”‡
0*Gz;,)!+/9:,lN
#, ")!]576)$.3'?6)!+$ ./+6)H>$

    [[[  :      [[[  :
/9(++576)$'

2R :R[ [ [AR R †RQ[ [ [AR


-„ '$G

R †RQ[ [ [AR :  ƒ R :R[ [ [AR : [


i[„

1„

-;2 <>= ?{@CBEDEF F ' )= B a i dgfha = i 0 6i2)< ( dM`\a f < * D


, a ?{;2D d * 7 9
j H
 +



‰H hHw[ [ [ H -:
 $G +9:,-2")$6DC +6%!>$]")$.(+*$/]&^9:+H>!e€o9:6,76)!W)$G25n6%$ EK†?9:(?$

!9:(+a&^9:6)(f+9:6)f(+*$! '?+(+!1>$EK$,-v&^9:?!>He€WD9:,dd

*   [ [ [ :   *   [ [ [ :  H   



?       (6<
  " "     Π
UM:$1*:/H>*#?V$2+$6)/$EK$,72?  ƒ  ƒ [ [ [ ƒ -:[
ƒ 9:6
  B  B z    y{‚ ~ @  ~b€y{‚ !


3
j   H w
H [ [ [ H  H
#,)2/$f19:,%")!+/9:,)o")6d+J)*9(+_E $W;9,d4&)&)6)!$?6)(]/$8!$E EK$#?6)!M4,-‰ 1e N Š 
 EK†?9:(+$

/9(+W)w1jJ)576)$&^$(?E6%>4>!9:,
     
"%$
u %9:, &^$6%

*  <   H } Hw[ [ [ H " : = … 1H 6Hw[ [ [ H :”‡•[


+?9D1/$(26%,d(?*$ +$V576)$ 
*  ;
 ƒ ;
$
 ƒ

 &^9:,)")*(+*$4N^#,~
      [ [ [ :    ƒ *         [ [ [ :    
V+6%‡v!9:(+f"ST6%+//!+$(aOT`
*
 

 *      [ [ [ :     

ƒ *

  [[[
        :     [
A2$E|4(+576)$N
m(]$xC%$EK&%/$ * 
   *   ™ Wp9646)+? * ˜
  *     g[ [ [


  


]_^5^5`ba\cedgfha = i
j lkmHJnhHJo{H  g[
9 !$,-
:

 < k ˜ Rrn ˜ Rro ˜ R ˜ =  k  n1o†RXn k>o†RXo  kCn5RXn‰o k –Rk>n o –Rk>o nm[
mL(+9:6%M$(2576)$

uxy{z}|~b€y{‚      
* k ˜ * k>n o 
  

qLp!,)*-4/!+*0")*+!(+*$.?$(?**1(+H. ƒ€";,)]/$8E $Eki)(+$

-k ˜ :

… nhHJo{H ‡ H
Šk>n  o  … … k HLn'‡
")$.-6)1jJ%$ &)&;(+4</ e€9/W%5n6%V$?2!$8,)9:Ei)(?$.&F$(+E6%>4>!9:,)a"%$

o{H ‡
 /9(+f576Spw ")(?9:!+$
 K&)&;4(>4<! e€9/WD")$6nCd&F$(+E6%>4>!9:,)f")$ o$G
")$6nCd&F$(+E6%>4>!9:,)a"%$ o"%9:,),)$,7v1$8+$(+E $„xN
ƒ … H gHgH”‡ ƒ  H H H  H
; ; u u
PL,19:,)+!")*(+,-2/f+6%!>$]"Sp$xCD&F9:+,-> B$ ;9:,
19:,%?j=>$.576)$ EK†?9:(?$ H
; ;
)$2")9:,%1Žp!,)*:/H>*2,Sp$?v576Sp6%,)$19:,)?*576)$,%1$

")/(?$1G>$.")$.OT/,)*-!!>*2")$I6)!(+J)$"SN

]_^5^5`ba\cedgfha = i
Π: 6 
 
   (6<
    
 %$)  " $(< $)  
j lkmHJnhHJo [
9:!$,-
mL(+9:6DM:$( 576)$}
k ˜ Rrn ˜; Rrk>n1o R n ˜ Ro ˜; RkCn1o R o ˜ Rrk ˜; Rrk>n1o O k>;n1o [ j 2ƒg‚ `
77'„

xy{z}|C~\€y{‚
?A@B €  @B  ~ my  B !
 , */!EK!,)$0!$]")*,)9:E /,;4+$6)(?f$29:, E6)H>!&)/!$v&;4(2‹DN[qLp!,)*-4/!+*0")*+!(+*$
#

* < k ˜ Rn ˜ Rrk>n1o = < n ˜ Ro ˜ Rrk>n1o = k>n1o


$Yv/9:(?f*576)HM4/$,-+$.w 



O u < k ˜ Rn ˜ Rrk>n1o = < n ˜ Ro ˜ Rrk>n1o = < o ˜ Rk ˜ Rrk>n1o =
* < k Jn‰o†R ek ™ n ™ oˆ$ R ek n  o  Rrk ˜ n ˜ o ˜ =
9:6 $,%19:(?$W[$,d")*M$!9:&)&;,-



O * < k ˜ n ˜ o ˜ R u k  n ˜ R ek ™ n ™ oqRk n1oˆR u k Jn  o  =
* < kn˜ k Ln  o  = 
 H
1Npw%NT"SN
u ’ u ;5n6%V$?26),%$19:,)?*576)$,%1$")!(+$1>$.")$


OT/,%*-!!+*v")$I6%/(+J%$U"d1U( BEK†?9:(+$ … HgH”‡ …  H u H u ‡ [
 B | e€   B  ~ m y  B !   ! ˜ ˜
… k n‰‡ < k n =  7k Rrn kCn … ksRrn‰‡•H 2k ƒ nh[
’ < k  ’ HJn  … kmHJn‰‡
#,d '")9,)1 [UM:$1*:/H>*#?+
ƒO#,d6)(+!a&)646)+?{6D>/!/?$(oŽp!,)*:/H>*0"%6(?*9:(?")9:,),)$EK$,-UW;$,d+!:,;/,-

7k Rrn k n†Rn  k
˜ ˜  =
576)$./$2?6)!+$ $G a+9:,-29:(+"%9:,),)*$2";,)]/$8E bE $8+$,)N

˜k Rrn ˜; Rrk>n1o O kCn … ksRr; n‰‡2Rrk>n1o ƒ k>n‰o … kNRo nqRro'‡ [
PR2")9:,)1
V$,d")*19:6)!$8576)$K

3 0$LE bE $a ˜n Rro ˜; RkCn1o O k>n1o … kNRrk n†Ro6‡ o ˜ Rk ˜; Rrk>n1o O k>n1o … kNRn nqRro'‡ [
$G

˜k Rrn ˜; RkCn1o R n ˜ Ro ˜; Rrk>n1o R o ˜ Rrk ˜; Rk>n‰o O k>n1o oq… RrksRrnqRnqRk o6‡ ƒ kC;n1o H
PL,?9:E E|,-W%/FMD!$,-

UM:$1*-/H>*#?+ 2k ƒ n ƒ oA[
-;2<>= ?{@CBEDEFC‹ ' 'i )< ( dm`ba f < *2D 1 c ; , ;? 7
 V lkmHJnhHJo gH
 ?

k … k n‰‡ … k o'‡Rrn … n o'‡ … n k‡Ro … o kC‡ … o n‰‡  gH


mQ96)(f>9:6% 2Wˆ$ %9:,~

’ ’ ’ 2k ’ ƒ n ƒ o{[ ’ ’
  
UM:$1*-/H>*#?V$]?$6)!$E $,-2+
?       (6<
  " "     Π'
|
?A@B DC B
kmHJnhHJo{H
gH Tk n  o 
qLp!,)*-4/!+*0$?v?cnEK*G>(?/576)$.$,
@X+ 
  ;9:, &F$6%]>9:6†?9:6)(+f?6)&)&^9:+$%(v")579:6),)$ 1 
@X+ 
t gH
 )9:,d&^$6%f+9:6=†?9:6)(?]+6)&)&^9:?$(25n6%$ 
3#,)2/$f")$6nCd1UW;9:,dd t kUOQnsOQo{[ 
k … k n‰‡ … k o6‡ n … n o'‡ … k n‰‡ –o … o kC‡ … o n‰‡ g[
 
’ ’ ’ ’ ’ ’
 a$ 
qV$(?*?6)!>4]$,d")*19:6)!$&)(f+9:E E $N

]_^5^5`ba\cedgfha = i
j ˜ lk˜ mHJnhHJ˜ o g[  …
k Rn Rro R ek>n1o k>n ksRrn‰‡2Rrn1o … nqRro'‡Rro‰k … oˆRrkC‡ [
9:!$,- ;mL(?9:6%M:$(v576)$

xy{z}|~b€y{‚
k … k n‰‡ … k o6‡Rn j … n o6‡ … n kC‡Rro … o kC‡ … o n‰‡  [
qLp!,)*-4/!+*0")*+!(+*$.$?v*576)!M4!$,->$.w 

5 ’
0p$?vOT/,)*-!!>*2")$ ’ k ’ n oA[ ’
1jJ76)(2";,)]!$]1U  ƒ’ [ ’
qLp*-/H>*8'!/$6~?+
ƒ ƒ ;
A
*  k ˜ k  nqR k>n‰o  g [
2$E|4(+576)$N

j

u; ’ u; 
qLp!,)*-4/!+*0")$ 1jJ76)(2T*1(+Hv"%9:,)1

]_^5^5`ba\cedgfha = i 
  ˜ R  ˜     w[ ƒ 
`{M$1/$2,%9j4+/9:,%a1A@")$++6%W[&)(+96%M:$(v576)$.";,%2!$81U ~

xy{z}|~b€y{‚ ˜ 
  R ˜  
 kNRnqRro  ˜      ksRrnqRo  kCn†Rrn1oqRro‰k
qLp!,)*-4/!+* 2T*1(+!

   Rrk>n1o ˜  
9:6d$,)19:(+$ … ksRrnqRo6‡ R k>n1o  … kN RnqRro'‡ … kCn†Rrn1oq R o‰kC‡
5
k … k n‰‡ … k o6‡Rn … n o6‡ … n kC‡Rro … o kC‡ … o n‰‡  [
0NwDNp"SN[4&)(+_2"%*M:$/9:&%&F$EK$,7

5 ’
0p$?vOT/,)*-!!>*2")$ ’ k j ’ n oA[ ’
1jJ76)(2";,)]!$]1U  ƒ’ [ ’
qLp*-/H>*8'!/$6~?+
ƒ ƒ ;
  
1 :
 
„ 9:H j  … ‡ ƒ Rkg :m R[ [ [AR;k :m „R H Mp
)6%,&^9:!cn, EK$8w19n$‡ 1/$,-+
;
Π9 6 
  
   (6<
    
 %$)  " $(< $)  

(+*$!f$2&F9+!+!e€a96,76)!W;UM:$1
mL(+9:6DM:$(2576)$W;&^9:6)(a+9:6% )9:,   H
(+1!,)$](+*$!/$N

 … ‡  … „ R ‡ : [ {ƒ v9:,)(+/$
+
79 Œ-„

‹ „ 9/$,-
: j lkg‰HJk”wHw[ [ [ HJk;: gH   ;$
;
3?9:E EK$8")$]&)(+9n")6)H>f")$ sk
&)(?/ ~w /

  
e€9:!N)mL(?9:6%M:$(v576)$ 

 /:


  kM k”w[ [ [ k;:M[ VƒO#!cnE &)3"%$#")6m1!z;576)$?34+/576)$ 77 ‘-„GN

< k  Rk>nqRrn  = < n  Rrn1oˆRro  = < o l Rrk HLo‰nAkŠHLo{RH k  =  … k>nqRn‰o†Rro‰k‡ ˜ [


Œ-„amL(+96%M:$(v576)$4W;&F9:6%(]+9:6)f(?*$! ;9:,d 

ƒŽmL(?9:&F9+*#YI 77 ‘-„

„j lkmHJnhHJo H Tk>n1o ƒ [


9/$,- ;UM:$1
mL(+9:6DM:$(2576)$ ˜k … n†; Ro6‡ R n ˜ … o†; Rr; kC‡ R o ˜ … kN; Rrn'‡  u [ 877  {ƒO#YI „

 j „ lkmHJnhHJoAH k g[
9/$,-
n R o R   u[
mL(+9:6DM:$(2576)$ R
n†R 7 7 u oqR   o†R u –R ek sR u ksR en kNR u nqR  o 
ƒŽmL(?9:&F9+*#YI Œ-„

: j lk m J
H h
n J
H o [
nq Ro R o† Rrk R kN Rn O R R k  Rrn1o R n  Ro'k R o  Rrk>n 
„ 9/$,- ;mL(+9:6%M$(2576)$

kNk RRrnqn1Rro oA[ n Rro‰k o RkCn k; n; o; n:Rro oqRrk kNRn


$

' j 2H  H  g[


„ 9/$,-

mL(+9:6DM:$(576)$d … #/R  lR  ‡  … $R ; g‡  R … ŠR ; ”‡  R … sR ; ‡    [ Xƒ€(>,

77 : „

9 j lkmHJnhHJo … [  …
n†Rro  kC‡  R oqRrk  n‰‡   R … kŠRn  o'‡     [
„ 9/$,-

77'
mL(+9:6DM:$(L576)$8
… nqRo6‡ ’ Rrk … oqRrkC‡ ’ Rrn … kNRn'‡ ’ Ro  )ƒ :&^9:, „

7 j lkmHJnhHJo  ˜ gH ˜ TkN˜ Rrn†Rro ƒ  [   < ˜ ˜ ˜


k>nqRrn1oqRo'kUOQk Rrn Rro R k>n1oNOQk ; Rrn Rro O u k Rrn Ro = R  kCn1oA[
„ 9/$,- [UM$1 ;mL(+9:6%M$(2576)$

!
 

 S            
  S

`06 1jJ;&)H>(+$
j 
Wˆ,)9:6%UM:9:,%k")*†+w(+$,)19,7+(+*KOT/,)*-/H>*k+(+/,):6)//(?$$OT/,%*-=@
/H>*0"%$ 5a46)1jJ-c7@ 1jJ a4(+…~ƒ€")9:,-29:, '+!:,;!*#OT/,->$(+&)(?*>4>!9:, :*9:EK*G>(?/576)$„xN;#,
")9:,),%$!1S"%$6DC6D>(+$]/,)*-/H>*a1*/*i%(+$.

-;2<>= ?A@CBEDEF  ' 6i2<)( dm`\a f <+*2D ` * D>?;7


?

j  H  
9:!$,- "),) R ; ƒ [
>$/]576)$
j lkg‰Hw[ [ [ HJk;: –n6‰Hw[ [ [ HJn : 
9:!$,- '$
;
;
")$](?*$!N;`v/9:(?8
: + + : +   : + 
 *+ k n  O *+  k  *+  n  H
,  ,  ,
 

’ “  –k … k+ n  + HLk  Hw[ [ [ HJk : ‡


 M:$1*:/H>*#?+ˆ/$aM:$1G>$6)(+
U
19:!/,)*/(?$a$G25n6%$!$],)9:Eki)(+$
   a$
’ “  … n  HJn  Hw[ [ [ HJn : ‡
 
?9:,-v?9:!f+9:6)]&^9:+H>!e€o9:6 ,n6%/W;?9:!
 a+9:,-

>9:6%2,%*-4+!e€f9:6 ,76)!N

?A@B D| C B
: +   : +  
j ƒ  k
9:!$,- * +   
ƒ
 ,  
$G * + n   [
3 + + ,+ 7 +
k n O =k > R n
kp&)(+_vŽp$xCD$(+1/1$

@1jJ;&%!>(?$

W)&^9:6)(a+9:6% +W)9,~

   ;  ;  
P ,?9:EKEK,-]1$](+$/4>!9:,)a$G2$,dE6)!+/&)!3,-B&;(
L
  W)9:,d9:i%+/$,-]Žp!,)*-4/!+*

")$  :!")$(N

A2$E|4(+576)$N
Π7
‘ 6 
       G<
  "G
G $)  
j  ƒ  ƒ uH
 )9:,d(+$G>(+96%M:$.OT/,%*-!!+*0")$ a6)1jJ-c-@ N5 1jJ o(?…N j 
-j ;2<>= ?{@CBEDEF ' 'i <)( dm`ba f <*2D
a i = 3  a 7
 VXW R  W H ‰Hw[ [ [ H -:
1H6[ [ [ H -:
 ?
9 !$,-
:  )$G $ ")$2(+*$/N

: + +   : +   : +  
`0!9:(+8
;
* +  R$  O *+    R *+    
,    W

 M:$1*-/H>*#?+V!$fM:$1>$6)(+
U
’“  … g
k 1
 J
H ”
k ,
h
 w
H [ [
f$G [ J
H ;: ‡
k …
’ “  ‰HLn16Hw[ [ [ HJn:>‡
w
n ,
o+9,7
19//,%*U!(+$o")$.EKbEK$#?$,)N

?A@B | C B
j "  9:H '>$S576)$ R [ … ‡ [
;  +  ; ƒ +   +  + ’  m  ƒ +  + 
)`v/9:(?

 H  R  ; ƒ  + R    + ; +  mR   +   + +  m
O $R  R  R
mQ96)(f>9:6% ;9:,dd  
 

: + + :
m  :
  m =
PL,?9:E E|,-W%/FMD!$,-
+ + + + + +
*+  $ R   O *+ <     R   = R *+ <     
 R
,  ,  , 
  


: +   : + + 
#(W)"l&%(+_2OT/,%*-!!+*v")$  :/")$(W

* + <    R$  m = O * +   * +  R$   m 


: + + +

,  , :  + +
   

+   , : 
*+   * +  R  
ƒ , 
, 


: +   : + +  
3
0$.E bE $'
:
*+ <     $
+ +
R 
+
 m
  = O * +   *+  $ R  
,  ,  , 
  

: +  : +  : + + 
PRv/,)?R
: + +
* +  R$  O W * +    R * +     Z * +  R  
,    
   

: + ,+     : , +   : , +  
* +  R$  O *+   R *+   
3 C
kT9 y

: , + +  


: , +  
 

: + ,  


* + … R ‡ O *+ R *+ 
,  ,  , 
1Npw%NT"SN    N
A2$E|4(+576)$N
j  …  ‰H wH ˜‰‡  … C1H wH ˜J‡•H

#,?$&%31$8";,)]OT$?&;1$$6)1!/")!$, 6)?6)$ŽN  o$
: + + 
 …  H T‡ ƒ  *+  [ 
 
’ 
9:,d&F9:?$

 "  ,


5 … G-HJI ‡+O  … GTHJKT‡R … K/HJ$I„ ‡ H


qLp!,)*-4/!+*0")$I!, 9 ] n{+!:,)Hz;$#576)$
  
o$.5n6%{&^$(?EK$G]")$.&)(+96%M:$(v576)$


$?26),)$ € ~  ‚ B
;&^9:6)(a+9:6)]&^9:/,->

+
GN
T
G J
H 
I J
H /
K [
M:*(+Hz;$.OT/,)*-/H>*]>(?3,)6)3!(+$

  
1
 
j
2H  H " g[
„ 9:!$,-
mL(+9:6%M$(2576)$

 R 

 R
 R … R$ ‡  … „R ”‡   R …  R  ‡  … ŠR ‡   R …  R  ‡  … lR$g‡ 
 
 O
[
; j –kmHJnhHJo{H 
 ƒ … kNR ‡ R u … n u ‡ R … oqR ”‡  R  … nqR ‡  R u … o u ‡  R … sR ”‡ 

‹:„ 9:!$,- 3
K")$2(?*$/N v*>$(+E /,)$(f/$8E /,)!E6)E ")$


R  … oqR ; ‡  R u … ’ u ‡  R … ksR ”‡  R  … sR ; ‡  R u … ’ k u ‡  R … nqR e‡  [


; ’ ; ’
‹ 6 
  
    G<
  "G
G $) 
 

 S    S   

 , .‰$'98 .”"%(+*T,7.‰$102(


Q(?9:6%M$(o6%,K1B"lp*-/H>*f";4,)BŽp!,)*:/H>*o*G>6)"%/*$2")9:,%,)$v&)(?e€9:!L6),)$2/,)"%/1U=>/9,
+6)(d/$ cn&^$}"ST/,)*-!!>*~")$y(+*e€*(+$,)1$ w6D>/!/?$(Naq{s&)!6)&;(Y"%$!,)*:/H>*
&)(+*+$,7+*$ /1a+9:,-K")$ *:/H>*k+]$ +$6)/$EK$,-K?f/$M4(?3i%/$'?9:,- *-!$N
309:,)14W7+[&;(Z$GCD$EK&)!$]OT*-!!+*a.//$6 6DC$xCD+(+*EKH>*X")$]OT/,-+$(?M4!/$B"Sp*G>6)"%$'ƒg$G
w1$|+6=†?$GUW!Z&F$6%bG>(+$.†?6)")/1/$6DC ")$Keg!(+$>$,)")(+$|6),)$ 9:6}&)/6)?/$6)(+M4(+34i)/$
M:$(+]1$f$GCn>(?*E !>*WDEKbEK$v";,)a!$#1U4a"ST6), /,->$(?M4/!$v96%M:$(?UW%9:6~+FOT/,)*-/H>*
*+6)")/*$B$?6),)$X/,)*-/H>*RY>(?/1+$„xW=!-&;(+4</ˆ&^$6&)(+9:i)i)/$Z576)$X/$Z(+*+6)Hj4Q1jJ)$(+1jJ)*
&)(+9Mn/$,),)$.")$.1$v!,)*:/H>*o")$.(+*e€*(+$,)1$N
m( 19:,->(+$4Wf/0s"%* †+w *+*y?/:,)/* 576ST6),)$ e€9:,%1>!9:, 4‡ ,)$ 4?>$!,- +$ i^9:(+,%$
6DC$GCn>(?*E !>*R")$fŽp!,7+$(YM4//$X"ST*+6)")$N:PLW:&)!6)R:*,)*(+/$EK$,7W576ST6),)$fe€9:,)1G>!9:,

j
19:,-M$GCD$'ƒ€(+$?&SN:19:,)1UUM$„Z4++$/,-L?9:, E|=CD!E6)E ƒg(?$+&lNpE /,)!E6)E|„Q6nC'i^9:(+,)$L")$
OT/,->$(?M4!/$]"ST*>6%")$N l1$3,ST$Y]&;a$xCD&)/9:Hji%/$W7!F(?$?+$#>9:6†?9:6)(+B&F9++!i)/$v")$
&)(+9n1*")$( &;4(~(+*16)(+(?$,)1$ ƒ€+6)( /$,)9:Ei)(?$ ")$ M4(+34i)/$j„GWB9:6 w 6),)$ *+6)")$ ")$
e€9:,)1G>/9,)ƒg")*(+HM:*$4W;M4(+3=>/9,)NHN!N„GN

2
,T&)*79 9& 0 ) 9 N ,/45$', ‰. 9GF
j
V$Y2+9:6=†?9:6%(+]&F9++!i)/$8"%$&%(+9n1*"%$(2w'6),1jJ),):$EK$,72")$8M4(?3i)!$f&^9:6)(8
@ /E &)!!z;$(oOT$GCD&)(?$?+/9,*G>6)")!*$4N
`a++$,->!9:,SW;/^eg6%v!9:(+o+$,)!(f19:E &%+$8")$/19:,-+(>!,7+$8*M$,->6)$/!$&^9:(?>,-

Œ
  
   6 R    $(
) 

j
+6%(2!$aM4(?3i)!$a"%$"%*&;(YUN
@ !EK&)!!z;$(o/19:,->(+/,-+$N

309:,%,)9:,)]")$6DCd$GCD$E &)!$]/E &F9(?j4,7+#


j kmHJnhHJo 6p
 ksRrn S o
„ V!$aM4(?3i)!$a?9:,-2/$]!9:,):6)$6)(+ #"%$]1 +*2"ST6),d+(+/,):!$N

:n Rro k
qˆ>(+!&)/$019:,->(+/,-+$
o†Rrk n &^$6DvbG>(?$,%$6%+(>!/+*$8&;(f!$81jJ;,):$EK$,-

R 
   k
   ƒ n:kNRrRo n ’ k o   n ƒ „R$u  [
 ƒƒ o†Rrk ’ n  o ƒ /R u 
")$8M4(+/i)!$ 1Npw%NT"SN

uv94>9:,)f576Sp!9:(+ 2H H 
’ ƒ u
'+9:,-2+$6)/$EK$,-v?6)&)&^9:+*vY>(?/1+$E $,-2&F9+!+!e€N

j –ksRrnqRo ƒ k>n1o{H -kmHJnhJH o   k ƒ


„ V/19:,-+(>!,7+$.$? [UM:$1 %N


50T$?v+96%M:$,7v3&^9:(Y>$.9:6%M$(?+$61jJ),):$EK$,72")$8M4(?3i)!$8 n ƒ [


H H &^$6%M$,-vb>(?$.19:,)?/")*(+*v19:EKE $8/$],):!$f"ST6),
oƒ 


>(?3,%:/$0/:6lW)1Npw%NT"SN H H VZ H  W%$ R R ƒ [


uv94>9:,)f576Sp!9:(+ 

u
 

22 45$ F29 4 TH 8 E 54 (+9 4 8 &)9 F  ( 04:$‰9


@ j VŽp$xC%&%(+$?+!9:, $?v?cnE *+(+/576)$4W;9:, &F$6%29:(+"%9:,),)$(2!$aM4(?3i)!$N 5o$/
")9:,%,)$.")$]&^9:+?/i)!!>*f+6)&%&)/*EK$,7>/(?$f";4,)]3':$Y>!9:, ")$]1U4/16%/W%9:6
&^$(+E $W)&;(f$xCD$E &)/$4W;"Sp$,-MD!>$(2OT6%>!//+4>!9:, ")$.OT/,)*-/H>*2")6
(+*9:(+"%9:,),)$EK$,729:6 ")$.Žp!,)*-4/!+*0")N $ 5oJ)$i-cD?J)$M
j
@ VŽp$xC%&%(+$?+!9:, $?v1cn1//576)$4W;9:(+"%9:,),)$(f/$fM4(?3i)!$a,lp$?v&)/6)f&^9:?+/i%/$N
m4(219:,->(?$W;9:, &^$6%]>96=†?9:6)(?21jJ%9:/?/(a1$/!$8")$]M4(+/i)!$a576)V?$(>3'&)!6)
&^$>H>$8")$8+9:6%>$ƒg9:6 3'&)!6)f:(+,)")$4W;E|!a&)(?9:i;i)!$E $,-]&;]!EK&^9:+$(a/$

j
")$6DCdw3ke€9:!j„GN
@ VŽp$xC%&%(+$?+!9:, $?vJ)9:E 9::_,)$W%9:, &F$6%]>9:6=†?96)(+
:  k!$8&)(+9:i%/_E $
+
‚•y @ :   }z + € B @
+ ƒ [ 3
, ƒ
*+
, 
$,d/E &^9:>,-f6%,)$.19:,->(+/,-+$"%6dcn&^$ 9:6 #4,)v!$
; ;
?   B G   "%$    $     6
G )
&)(+$EK!$(f1W;9:, ?$E $2!9:(+o$,d+!+6;4+/9:,~"ST6%+//!+$(a/$2!,)*:/H>*Xcn&F$
$,)?$,SN
uv9+9:,)]576ST/,-M:$(++$EK$,-UW^9:, &F$6%2&;(?e€9/a(?$,)")(?$6%,)$./,)*-/H>*vJ)9:E 9::_,%$8$,
6%>!/!>,-f/19:,->(+/,-+$"%$.eg4h9:,k†?6)")!1!$6)?$N 5o$.576)V&)(+*&;(?$w6),)$.6%+/!/>=>/9,
")$.OT/,)*-!!>*2")$I6)!(+J)$"SN

DCB 5
^ ` D
j 2H  H [        …      
# ”‡ [
9:!$,-
 ;     
mL(+9:6%M$(2576)$
ƒŽmL(+9&F9:?*‚ ` j 77 7 „

xy{z}|~b€y{‚
<  R   =  „R <   R  =  NR <   R # =  
qLp!,)*-4/!+*0&)(?9:&^9:+*$$?v*576)HM4/$,-+$.wd

u u  … #ŠR u lR R  ‡ …  R  ŠR  ”‡

… M‡ … ” ‡  „ R …  ”‡ …  ‡  NR …  ‡ …  g‡    g[


1Npw%NT"SN

’ ’  2H   H   ’  ’ 2H  H" [ ’ ’
uv9+9:,)]576)$ '1U(
   g[
…  ‡ …  g‡    g[ 3  
qˆ"%$(+,%/_(?$./,)*-/H>*v*j4,72YcDE *+(+!5n6%$W;9:,d&F$6%2+6)&%&F9:?$(]576)$
;

`0!9:(+
 ”H ’ … ’ M‡ … e‡  …  ”‡ … g‡   H
0$&%/6)W)&)6)!+576)$3ke€9:,)1G>!9:, $Y01(+9:!++,->$

’ ’ ’ ’
+6)( )9:, )1(f1jJ;576)$.eg1G>$6%(
$Yv&^9:?!>He{9:6 ,n6%OW;$2576)$.1jJ;576)$.eg1G>$6%(2"%6E $Ei%(+$8")$.-6%1jJ)$.$?vEK†?9:(+*
&;(]6),~eg1G>$6%(2"%!\F*(+$,-v"),)]/$8E $Ei%(+$8")$.")(?9:!+$N
qLp!,)*-4/!+*0")*+!(+*$.")*19:6%/$.")/(?$1G>$E $,-2")$.1$v"%$6DC !,)*:/H>*N

 0

d = ?ADC? ' A? D  ^ 9:B a i2= ?ADC?;7 c ; d  , D f D>? B D*2D d = i 2.BE< f ? a  , D#9
j

 OT6), ")$2E $Eki)(+$2"lp6),%$!,)*-4/!+*0YcDE *+(+!5n6%$6%,)$.+9:E EK$8"%$.+$(?EK$
V
?cnE *>(?/576)$v$,!$aM4(+/i)!$WD9:,d&F$6%]>$,-+$(]")$8>(+M4!/!$(o?6)(]1jJ;16),")$
1$2+$(+E $]!,)")!Mn!")6)$!/$EK$,7N

DCB ^ 5 ` D
j lkmHJnhHJo gH >k n TkCn1o ƒ [ n1o
9:!$,- ;UM$1
R R ‰o k O [
mL(+9:6%M$(f576)$'
77: „
k Rrn Rrk>n n Ro Rn‰o o Rk R o'k ;
; SƒgmZ(?9:&^9:+*#YI

xy{z}|~b€y{‚
k †Rn ƒ … kNRrn‰‡ < k ™ k ˜ n†Rrk  n  k>n ˜ Rrn ™ =
#,~

’ ’

:  
   6 R    $(
) 
…ƒ ksRrn‰‡W …  k  n‰‡ < k ˜ n ˜ = < Rr˜ k  n  ˜ Z 
… kŠRn'‡Mk n ’ H … k ’ n'‡ k n = g[
kCn O ’
;1(
k>n  ’
o k Rn Rrk>n … ksRrn‰‡Mk n RkCnUƒ kCn … ksRr; n‰‡2R ; ƒ k>n … ksRr; nqRo6‡-ƒ
3
09:,%1|

kNRnqRro
k>n R n‰o R o‰k O o R k R n
#, &)(+9n1_")$"%$E bE $#&^9:6)(]/$]6%+(+$f+$(+E $N;`0!9:(+8

k Rrn RkCn n Ro Rrn1o o Rrk [ Rro‰k kNRnqRro kNRrn†Rro ksRrnqRo


k ƒ n ƒ o{[ ƒ ;
PR2OT*-!!+*#'/!$6~+?

  ,7& $PK 8).'9 4 , -9GE,! _459GFF29


uv$o†+EK!o9:6)i%//$(a576)$1$(Yj!,)$]!,)*-4/!+*B,)$.,)*1$+?!+$,-86%16),)$
19,),;!+>4,)1$v&;4(?>!16)!/_(+$W%$Gv&^$6DM:$,-vT9:i%+$,)!(2&)(f")$]EK,)!&)6)3=>/9,)
!:*i)(?/576)$f")/(?$1+$N

DCB ^
j lkmHJD nhHJo  …5 `
k n k n‰‡2Rrn  o … n o'‡ Ro  k … o kC‡•H
9 !$,-
: #/$f/9:,%:6)$6%(+f"ST6),+(+/,):!$N

879 ’ ’ ’
mL(+9:6DM:$(2576)$ ;$2")*+$(?EK!,)$(f!$f1U4
"ST*-4/!+*NSƒO#YI Œ-„

xy{z}|C~\€y{… ‚
j  kmHJnhHJo'‡ ƒ k  n … k n‰‡2Rrn  o … n o'‡Rro  k … o kC‡ [
’ ’ k  … kmHJ’ nhHJo'‡•[
9:H

… kmHJnhHJo'‡ [
#,~M:*(?!z;$8eg1/!$E $,-f576)$
T
$?2/,-M4(?3,->$8&)(f&F$(+E6Dj4+/9:,~1GcD1/!5n6%$"%$

 … k HLnAHLo6‡ ƒ k … n o6‡  … nqRro ƒ k‡6RŽn … k n‰‡ … k o'‡ … kŠRn o'‡  g [


^#, &F$6%2")9:,)1.?6)&)&^9:+$(v576)$

’ k ƒ ’ n ƒ oA[ ’ ’ ’
I!4/9:(?
3 C
kT9 d319,)1!6)+!9:,SW;UM:$1*-!!>*0++

  
1
~
j ‰H wHw[ [ [ -: * + /21 + 1   [
# % ’
„Y-„ 9:!$,- ")$v(?*$/]&)&)(?+$,;,-vw  

. - -):
6)$V$?2/$8E /,%/E6)E ")$ ; ;
+
BA V W Z\H —[
’;;
i[„ ]$&)(?$,)"%(+$/$8&)(?9:i)/_EK$.UM$1  ;&^9:6)(a>96%

2 m
k J
H h
n J
H o 

k>n1o  … kŠRn o'‡ … nqRo kC‡ … o†Rrk n‰‡•[
‹:„amL(+96%M:$(v576)$.+ 4/9:(?

’ ’ ’
    
 B   N$)   6 '
Œ-„ j –kmHJnhHJo
9:!$,- >$/]576)$ 7kNRnqRro ƒ kCn1oA[ 
mL(+9:6%M$(2576)$
r
R k  R ; Rn  R ; Rro  O u [ 5 779
{ƒ o9:(?*$ „
; ;  ; :  ;
„ j
1H hHw[ [ [ -:   
9:!$,- +$/f576)$ *+ 
 ; ƒ [
R
+
:  +
+ 
 
4 : 
 [
,
;
;
mL(+9:6%M$(2576)$
,
 j –kmHJnhHJo  … 7 kNRg[nqRro ƒ kCn1oA[
 kmHJnhHJo'‡ 
„ 9:!$,- >$/]576)$
mL(+9:6%M$(2576)$

: j 
„ 9 H
:
B3 u
6%,$,7+/$(N
7K
‰H hHw[ [ [ H -: gH
 „ 0*G>$(?EK!,)$(]/&)/6)f&^$+!+$19:,)Yj,->$
- +$/!$#576)$W[&^9:6)(a+9:6)](+*$/

: + ™
%9:,d!.

* +0/21 <  R  = OQK * +


+ 1 + 1
. - -): , [
i[„fmQ9:6%(21$?>$.19:,%?j4,7+$ -K/H 77 7
)")*G>$(+E /,)$(f/$f12"Sp*-!!>*4NVƒŽ8YI „

„' j –kmHJnhHJo   7 k>n‰o ƒ [ 


9:!$,- >$/]576)$

k R n;  n ; R o;  o R k;  O [
’ ; ’ ; ’ ;
mL(+9:6%M$(2576)$ {ƒO#YI ‹4‘:‘:‘-„
;
9 j –kmHJnhHJo g[
k R  n R   o gH [ 43
„ 9:!$,-
mL(+9:6%M$(2576)$W[&^9:6)(a>96%](+*$/ )9:,~

 k R  n‰o  n R  o'k  o R  k>n tQu


ˆƒ kp&)(+_##YI ‹‘:‘ „
9  
  6 R    $(
) 
 

 

 -; dM^5a f ?AD F
ŽO = %   R > R = 
„f#,~
  R >

"ST9 C+  ’ $ R   R  ; %   R ’   R$  ;  R  [
; ƒ %  ;R H  R  ƒ  R  R uH
;
qLp*-/H>*2,Spc:,-f/!$6K5n6%$#+
 ƒ R ;  D$Ga")9:,%1
 ;
1$G+>$/E &^9:+?/i)!/!+*2+?6)(+$.OT/,%*-!!+*0?+(+!1>$4N

… kNRr n‰‡ … nqRro'‡ … o†Rr kC‡    k>n  n1o  o'k ƒ  k>n1o


k  k  … n o'‡ ƒ  kNRrn  o  k nqRro
‹:„f#,~

3
/n  …  n†R’ k ’o …  nqRo k … so ’  oq’ Rrk n  oˆRrn k
0$.&)/6%
3
mk>n1o ksRrn ’ o'‡ n:Rro ’ kC‡ oqRrk n'‡ ’ ’
0$.EKbEK$ K$G

’ ’ ’
`0!,)+L
3 C
kT9 !$8(+*+6)Hj4N

  k +   …  ‡Mk + … k + ‡  2* + 1 … k + k 1 ‡  O * + 1  k + k 1 


Œ-„amQ96)( ZzDC%* 

ƒ ’ ; ’ ’ ƒ , ’ ’
  H ,
 * + :  k +  O * + /21  k + k 1  H C
PL,?9:EKEK,-]+6%( ;/^MD!$,-

, 
u
lkM  ƒ ’ k” ƒ [ [ [ ƒ k;: m ƒ –k;: …  ‡D 2H
;"ST9 3'19:,%1/6%+/9,SN

’ ’ ;
uv9+9:,)]576)$.&F96)( $ ;9:,'OT*-!!+*W

u
1$.576){+?6)(+$.576)$ ,)$.&F$6%2b+(+$.(+$EK&)/1*8&)(]6),)$8M4/$6)(f&)/6%f:(>,%")$N

7
 ‘ 6 
  J"($(
)P"  

j+
:
 +  :

+
k    [
„B3kp&)(?_50N N!W;9:,~ *+
,  : ’ +
 k
*+
, 
’ : 
*+  k 
: :
, ’  ƒ: ,  ; ’ + ; ,  ƒ : ’ ; H C ,   ’ k  ’ [
#(8 *+ *+ k ;"Sp9 * +  + 

*+  + k  R *+  + ;
3
kp6%+(+$.&;(Y.
, 
’ k :ƒ ’ ,  : k +
 k    *+ k
: +
 ƒ [
 +
#(W)19:E EK$81H@"%$+?6).
:  k +
*+
 k *+

UM$1
 
 ,
   …  , , ;
, : ’ k  : ’ +R  ƒ : ’+  ;  ‡ [
`0!,)+ *+ + 
: + 
*+ *+ k   *+ k H *+ k    [
PL,%z;,y
 ,+   : , +   : +  ,   ;"%9:,)1

: , +;
* + W k …  k ‡%Z k +  *+ k  
 k ƒ
*+
,:  + ’ ,  ’ , 
#(8

k +  : + : +     [
 k  + k + k   : ƒ
*+
,  ’
")9:,%1 

,  ’ ,  ’ ’ ;
   =      

„aqLT/,)*-/H>*v&)(+9:&^9:?*$.$Y0*5n6%!M4/$,->$.w~

 ’   R  ’ > R ’   ;576)V$Y2+(+HMD//$4NgH
uv94>9:,)f576)$OT*:/H>*#'!/$6~?+ ƒ  ƒ g[
:
… ksRrnqRo6‡    … k>nqRrn1oˆR o'kC‡    
„aqLT/,)*-/H>*v")$.-6)1jJ)$.$?v*576)!M4!$,->$.w 

W; … k ’ n'‡ R … n o6‡ R … o kC‡ Z g[


3 C u ’ ’ 2’ k ƒ n ƒ oA[
1Npw%NT"SN

kmHJnhHJo 
kT9 d/$8(+*+6)Hj4W;UM:$1*-/H>*#?+
uv94>9:,)f576)$319:,)1/6)?/9:, (+$?+$Mn(>4/$8&^9:6)( 576)$!19:,%5n6%$N

5
o9:E EK$ Nk HLnAHLo
 k n  oAH  n o  kmH  o k  nA[
#+9:,-2/$f/9:,)6)$6)(?]")$]1 4>*2"ST6), >(+/,):!$WD9:,d  6p
’ t ’ t ’ t  )$G

… k>nqRn1oqRro‰kC‡ … kNRrn†Rro'‡ ƒ o  … k n‰‡ R„k  … n o6‡  R„n  … o k‡  g[


`0!9:(+8

’ ’ ’ ’ ’ ’ ’

' ƒ Nk Rn oAH R ƒ nqRro NRkm H ƒ oˆsRRk  nh[
2H H –k ƒ ’ HJn ƒ ’ HJo ƒ ’ [ 
„f#,&^9:?$
`0!9:(+  " ' $ 
$  
u … „R g‡ „u R /R u  #u  ƒ <  „R u   = H
ƒ [
uv9+9:,)]576)$ [UM$1*-!!+*
+? 

3 kŠRn ’ oqR n† RnqRo o ’kŠR k ƒ oq Rr k R  nsO O u  u o  R$ ƒ u  nA[


`0!,)+L

 o†Rrk ’  n†R  ks’ Rrn ’  oNO u  ’ km[ 


0$.EKbEK$'
$

 ƒ  ƒ H k ƒ n ƒ oA[
PL,?9:EKEK,-UW%9:, 9:i%>!$,-]OT/,%*-!!+*0")*+/(?*$4N
qLp*-/H>*8Uc:,-v!/$6d?+ ^14NwDNp"SN

9 –k ƒ $R  HJn ƒ ŠR gHJo ƒ –R 2[


kmHJnhHJo g[
„ mQ9+9:,)
a

   R    R     … k n‰‡Mo R … n o6‡Mk R … o k‡n


`0!9:(+ [PR]9:,~

ŠR ’  –R ’ „kCR’ o  n1ƒ k n1’ o n ’o ’k


ƒ R  o R k k n o
n
 k>o n1k
R k n Rro  k ’ ’ ’ n oA[
u ; n o  ƒ n1o  n1k n1o ƒ
 k>o n1o u 
#(8 KUM:$1*:/H>*#?+

R o R  nh[
u; n k  k>o R n1k u; R n1o o k k  n o  [
3
0$.EKbEK$' #$G

PL,?9:EKEK,-UW%9:, 9:i%>!$,-
5
n o k ’ ’ ’
2k ƒ n ƒ o{H  ƒ  ƒ g[
o$.5n6%Q4++6%(+$.3'19,)1!6)+!9:,SN
uv9+9:,)]576)$.OT*-4/!+*#/!$6~+? )14NwDNp"SN

7 „ +6%&)&F9+9:,)]>96%2"S4iF9:(?" 5n6%$1jJ)576)$eg41>$6)(]")6dEK$Ei)(+$8"%$:6)1jJ)$+9:H
o
&F9+!+!e{9:6 ,76)ON

n R o; ƒ n  n; R n‰; o  ƒ n  Rk n;  [
’ ; ; ’ ;  ’ 
#,~
 
k R n;  n R o;  ƒ n k n OQn1k [
’ ; ’ ; ’ ;’
m(]?6)!+$ 
; 

n R o;   o R k;  Oo‰n   o R k;   k R n;  OQk>o  [
PL,&)(?9D1*";4,72")$.E bE $8&F96)(f/$v46%>(?$feg1+$6)(?W;!lMn/$,-

’ ; ’ ’ ’
$G

3 C W k R ;n  ;  n R o;   o R ; k;  Z  O … k>n‰o'‡ ;  ƒ [
’ ; ’ ; ’ ;
kT9
;
 ‹ 6 
  J"($(
)P"  

$29:,d'OT/,%*-!!+*0")*+/(?*$4N
j k R n; t  +k t –n [
’ ;
ŽW)&;(f$xC%$EK&%/$W /9(+ $G

3 mn R o;;  lo; R k; g[


’ ; ’ ;
#,%21$219:,%")!+/9:,)W%9:,/9(+8 $G
`0!,)+ŽW%6),d+$6)ˆ")$feg1G>$6)(+2$Yv,%*-4+!e WD$29:,dK4/9:(?a1//(?$E $,-f3
19,)1!6)+!9:,SN
+  + 
B3 5 j +  H • k  …  k ‡
‘-„ &%(+_ 0N NHW^&^9:6)(a+9:6%
• k +   ;9:,~

…  k
+
‡ ’ ’ ; ’
m4(2?6)!+$'
:  ’ k 
 
: ’
;
•
  ’ k ’ ;
  ; * …   k  ‡  1[

*
,  ’   ’ k ƒ
’; ;  ,  ’
$2")9,)1

p Tkg OQk”sOQk”˜w[
TkMRk> Sk”˜h[
: „Y-„omQ(]?cnEK*G>(?/$.")$]( :/$W)9:,d&^$6%2+6%&)&F9+$(]576)$

< k  Rk  Rrk ˜ =  < k ™ Rk ™ Rk ™˜ =


V+6D‡ v!9:(+o")$.&)(+9:6DM:$(2576)$
#(8
… kgRrk”:Rrk”˜‰‡ … kMRrk” u k”˜1‡ … k”5Rrk”˜ kM—‡ … k”˜†Rkg k”‰‡ [
$Y0*5n6%!M4/$,->$.w~

’ ’
m 6)/?576)$8>(+9/f")$2eg1G>$6)(+]+9,7213!(+$E $,-f&^9:?!>He€WD/S$,$?2")6dEKbEK$8")6
L
3 C
576;4+(+/_EK$4N kT9 d319,)1!6)+!9:,SN
’
i[„ ?A@B  €  @B  ~ my  B !   M[
qV$.1U
ƒ 
p
*>*.&)(?9:6%M*46-„xN[#,?6)&)&^9:+$")9:,%185n6%$
Tkg‰HJk”wHLk>˜
'p
m4(2YcnEK*G>(+!$8")$]( :/$W;/F+6%‡0/9(+a")$.&)(?9:6%M$(25n6%$ 2+9,72!$

5 j
/9,):6)$6)(+f")$v1 >*]"Sp6%,d+(+34,):/$4N

…  ‡ < k ™ Rk ™ R[ [ [{Rrk ™:;= t < k  Rk  RQ[ [ [ARrk :; = 
#(W)"l&%(+_2OT*,%9D1*8$ 0N NV

’ ; k  Rrk  Rrk ˜ R k  Rk  Rk ˜ R * : k ™  


ƒ u … k  Rrk  Rrk ˜ ‡ u  … k  R , k ™  Rk ˜ ‡  : 
O … ‡ R R * k™
™
3 C < k ™  Rrk ™ Rrk ™˜ = < k  Rrk  Rrk ˜ = H
’ ; 
 ,

kT9
u t )$2/$.:„o&F$(+E $]")$.19,)1!6)(+$4N

 B | e€   B  ~ my  B !  


[
j   ƒ 
m4(2(?*16)(+(?$,)1$?6)( [qV$.1U *>*.&)(?9:6%M*k6d-„xN
9:H zDC%*4W;&F96)(f/$5n6%$V9:,d+6)&%&F9:]/&%(+9:&)(?/*G>*#Mn(>!$N
 Œ

:  :   
j lkg‰HJk”wHw[ [ [ HJk;:     * k ™ * k [
9:!$,- >$/f576)$ 
  t  

5  ™ 
… ‡Mk :   u k :   *  k  R *  k  *  k  t g[
:
 
 , :
™
 , :
 
’ ; ’ ’ ,
0NwDNp"SN
, ,
k :   H
5
o9:,)?/")*(>,-21$G+>$/,%*-!!+*019EKE $#6),)$.!,)*576;4+/9:, ")6d+$19:,%"")$:(+*.$,
;")$.19D$G‡K1/$,72")9EK!,;,-f?+(+!1>$EK$,-v&^9:?!>He W%!Feg6%]"%9:,)1.576)$!$


")/?1(?/E /,;,-o+9:H]?>(?/1G>$E $,-2&^9:+H>!e N
:
     : ™ :    
#(8 
ƒ * ,  k  ’ … ’ ; ‡ * ,  k  ’ *  ,  k 
:
    : ™
  * k … ‡ * k [
`0!,)+ '/E &)/!5n6%$

+ 1  ,  
’ ;  ,  
O  t
t )O  H
lk HLk HJk  'p ; 
qLpJ-cn&F94>J)_+$k"%$(?*16)(+(?$,)1$k?+6)(?$k4/9:(?a576)$W[&^9:6)( ;!$

—p O  t
t )O R [
,)9:Ei%(+$ +9,72!$]/9:,%:6)$6%(+f")$21 >*v"lp6), >(?3,%:/$4N)m4(f?cnEK*G>(?/$
")$2( /$W)1$3'$Yv6)+?VMD(+S&^9:6)(
5
o$.5n6%Q4++6%(+$.3'19,)1!6)+!9:,SN
; ;
j
ƒ  … k HLnAHLo{HC‡ V    k ˜ Rrn  ˜ ƒ < o  R  =   ˜

 … kmHJnhHJoAHC‡ ƒ o k R  n [
‹:„ 9! $ 3ke€9:,%1>!9:,

 V ’  ; !
")*z),)/$.+6%( &;4(

… kmHJnhHJ< oAH˜ C‡ V ˜


 … kmHJ nhHJoAHC‡  <  ˜ kmH  ˜ nAH  oAH   = V
V$Y2eg41/!$#")$8M:9/(f576)$W[&^9:6)(a+9:6%


  k H  nhH  oAH   = g[


$ '+V$Gv?$6)!$E $,-2+
$
j   … kmHJnhHJoAHC‡ V

–k Rn ƒ [ o  Ro ˜   ƒ ˜ [
,)]&^$(Y>$.")$.:*,%*(>4/!+*W)9,d&^$6Dv"%9:,)1.+6)&%&F9:?$(2576)$

Rk n ; … k>oˆRrnC‡  < o  R  =  ƒ [


$ )`0!9:(+8
3 65 j ;
   
kp&)(+_ 0N NHW;/lMn!$,-

kCo†Rrn˜ O ˜ R k 
R o n 
R  < k  Rn  Rro  R  = ; [
o R  u u  [ ƒ u; ƒ ;
#(8

PR04/,)?R
k n k>oqRr; n ;
B3
Œ-„ kp&)(?_ 0N N!W;!lMn/$,-5 j
R
 ’ ; % ’ ;   ’ ;  
 R  O „
R  /
 R  R
’ ; ’  ; ’  ;
 R 
#(8
’ ; R ’  ; R ’  ; ƒ  ’ ; R  ; R  ;  ƒ ; [
2 6 
  J"($(
)P"  

3kT9Cd319,)1!6)+!9:,SN
qLp*-!!>*#!/$6~++

 ’ ; ƒ   ’  ; ƒ   ’  ; ; R ; R ; ƒ u
$

 ƒ  ƒ  ƒ  [
u
1Npw%NT"SN

„am{9:+9,) sk ƒ ; HJn ƒ ; HLo ƒ ; [


qˆ'19:,->(+/,-+$ #   #Š$ R  lR  2H
[p*1(+Hv6)+?
; R  ; R ; O [
PL/!$#19:,%")6)!]"%9:,)1w
3
UksRrnqRosO [
#    … R$/R ”‡ ;
;
k>nqRrn1oq R… o'kUO [     …
kp6%+(+$.&;(Y a$?2*5n6%!M4/$,->$w~

 (8
# kŠR5 n†j ; Rro'‡ ƒ kk>nqRrRrn n1oqRRo o'kUR O u k>k nq RrRn1n oˆ RRo'o kC ‡ n  Rro  Rrk 
; 
k R n  … Ro   k>nqRrn1oqRo'km[ 
PRUW;"Sp&)(?_ 0N N!W;9,~

 kCn†Rrn1oqRro‰kC‡•H C
1Npw%NT"SN
`0!,)+ )"ST9 d3'19:,%1/6%+/9,SN

 -; dg^:a f ; ?AD ‹ : + + : + +
*+  O *+  H
,  , 
„aqLT/,)*-/H>*v+9:6)J)!+*$.$Yv*576)!M4!$,->$.w  ;576)V")*19:6)!$

")!(+$1G>$EK$,7]"%$Žp!,)*:/H>*0"%6(?*9:(?")9:,),)$EK$,-UN

‹:„ j  [
9!
: k
; * 
, 
3
kp&)(?_2OT/,)*-/H>*2")6d(+*9(+")9:,%,)$E $,-UW;9, E /,%/EK/$]/9(++576)$

kg OQk”–O [ [ [mOQk;:g[
#(W)"),)]1$v19:,)")H>/9,)WD&)6)/?576)$!$f,)9Ei)(+$ lkM‰HLk>6Hw[ [ [ HJ;k : k +  
+9:,-2")$]$,7+/$(+
: k :
?+(+!1>$EK$,-]&F9:?!+!e€a")$6DCw'")$6nCd")/Y>/,%1>W)9:,eg1!/$E $,- Z&F96)(

—[
 ,   ,  ; [
>96% [PRv/9(+ *   *

s;k :  kg‰[
?A@B €  @B   ~ my ƒ  B !
Œ-„am{9:+9:,%

: k + : k + : +
qLp!,)*:/H>*0"%6(?*9:(?")9:,),)$EK$,-v19:,)")6%!]")!(+$1G>$EK$,7vw~
+  *+ + *+ k H C
*+
,  
k  , k ƒ , %"Sp9 /19:,)1/6)?/9:,lN

 B | e€   B  ~ my  B !


;
3kp&)(+_650N
: + k  j * + : k +  k ++   O * + : k +  * + : k + + 
NV

 ƒ + ,   :  +  k   k 
*+
,  ,  
, :
k+  * + k [
$2")9:,%1 * +
,  k  ,
@ y{€ €   B  ~ my  B ! +

3



kp&)(+_vjNT`kN 'N!W;&^9:6)(a+9:6% ˆV  Hw[ [ [ H ! k  Rrk   u k [  k +
+
 +
:
k +
+
 ; : + 
R * +  k  + u * +  k [
: +
PL,?9:EKEK,-UW%/FMn!$,- *+
: , +  k  
: +
*+ k  *+ k H
, :
k + 
:, +
*+ k [
PRv19:E EK$
, 
 ƒ , 
;9:,d~ *+
,  
k  , 
— p T k  n  oA[
k :  n :  o :  V  H n:Rr; o  oqR; k  kŠR; n [
„am4(2YcnEK*G>(+!$.")$]( /$W)9:,d&F$6%2+6)&%&F9:?$(]576)$
`0!9:(+ &^9:6)( ;$G
3 :
k R n: R o:  k: R n: R o:
kp&)(+_vŽp!,)*:/H>*2")6(?*9:(?")9:,),%$E $,-UW;9:,~

nqRrok : oqRk n : kNRrno : ks Rrnk : n†Rro n : oˆRrk o :


$
n†Rro R oqRk R kNRrn o†Rrk R kNRn R nqRo [
k : R n : R o :   k : Rn : R n : Rro : R o : Rk :
PL,?9:EKEK,-UW%/FMn!$,-

nqRro oqRk kNRrn u; NkŠ5 Rn nqRro oqRk 


k : Rrn :  ; <: k : m : Rrn :m = … ksRrn‰‡•H C k kN: : RRnn : :  ; < k : m Rn : m = [
3
0$.&)/6%W;"Sp&)(?_]OT/,)*-!!>*v")$ oJ)$i-cn+J%$M 

u n Ro  < n :m Rro :m = o Rrk  u < o :m Rrk :m =
;"ST9

3
0$.EKbEK$'
n†Ro u; k : n : o : oqRr k k : m u; Rn :m Rro :m
$

$v!,)+ŽW%$,d+9EKEK,-.
nqRo R oqRk R kŠRrn u+
[
 …  + ‡ —p
„ m4(2YcnEK*G>(+!$.")$]( /$W)9:,d&F$6%2+6)&%&F9:?$(]576)$3+6%!>$
a a$Yv1(+9:!++,->$N … ‡
`0!9:(+f/+6)H>$
: + + : + 
aOT$Y]*-!$E $,-UN)qZT/,%*-!!+*0")$ oJ)$i7cn?J)$M19:,%")6)!2w 
: N5
*+    *+ *+ 
+

,  ; ,  , 
: 6 
  J"($(
)P"  

+  : +  
+

:

+
 +  ,  [

1Npw%NT"SN
,  , 
: p
„amQ(]?cnEK*G>(?/$8")$v( :/$W)9:, &^$6%2?6)&)&^9:+$(v576)$ O UO [
`0!9:(+
… $g‡ ; … R ”‡ O … R ”‡ ; … R ‡ O … R ‡ ; … $
R R g‡ [
3

N5 ˜
 ˜ ;  ; ˜ $ R ˜ R ˜W
&)(+_]OT/,)*-/H>*2")$ oJ)$i-cn+J%$M^Wl9:,/9(+#
;  ; ;
R
… RM‡ … R ”‡ … R ”‡ … R ‡ … R ‡ … $R g‡ R  … R$g‡ ; … R ”‡ R
… ; R ”‡ ; … ; R ‡ R ; … R ‡ ; ; … R$g‡ Z ; ˜ ˜ ; ˜ ; ;
R$ R  l R „R$NR 
; ;
R$ŠR  k [
; ;
ƒ  R M‡ … R ”‡ … R ‡ …
mQ9+9:,)
 ƒ ; ; ;
˜ R$ ˜ R  ˜  k ˜ H  k $
`0!9:(+W)"Sp&)(?_]/$2!,)*:/H>*B$,->(?$E 9c:$,),)$

ƒ … R R / R ‡2 R …  R … #M ‡•”R‡  … ƒ R gH ”‡ ˜



 )$

… R M‡ … R ”‡ … R ‡ O W ; Z … r
R C
k ‡ ˜[
;
`0!,)+ ˜ ; R ;  ˜ R ;   ˜ ;  k ˜ ƒ ;
… R M‡ … R ”‡ … R ”‡ … R ‡ ˜ … R ‡ … R M‡ … Rr! kC‡ ˜ [
k ˜;   [ ;
; ;
V+6D‡ 2")9,)1.")$.&)(+9:6DM:$(]576)$; ;
… ! Rr˜ kC‡ ˜
;

-k  k ˜ ƒ ’ Rr; k  H

#(W)&%6)/?5n6%$ $Gv19:E EK$
˜ ; … ;
! r
R C
k ‡ !
[UM$1

 ; ;
&! k  r
R k
;  H
“ ! ; ’ ; 
Y>(+!1+$E $,-21(+9/>4,7+$.+6)(
; 
9:,d  … kC‡  … ‡ ƒ H )1$.576){+?6)(+$./19:,)1/6)?/9:,SN

-; dg^:a f ?AD  ;


k REn  R o  R    R k>n2REk>o REk REn1o5R n R o    < k  n  o    k>n‰k>o‰k Cn1o‰nCo  =


B3
„ kp&)(?_2` W%9:,'")!(+$1>$EK$,-

; ƒ  < k Jn 1o  =


ƒ ;; H
 
 '
kƒ nƒ oƒ ƒ [
UM:$1*:/H>*#?+

‹:„f#,
˜
… ksRrnqRo6‡ ƒ  k Rr˜ n Rr˜ o R˜ k>n1oˆR  < k  nqRrk  oqRn  kNRrn  oqRo  kŠRro  n =
˜ ˜ ˜ ;

k ˜ Rn ˜ Rro ˜ R ! k>n1oqR  < k  n  o  = 


kƒ Rn Ro R u! ”k>n‰oA[ !
ƒg"Sp&)(?_]` „

: 
Œ-„B3kp&)(?_2` W;9:,d")/(?$1G>$E $,-.
: :
 k … k‡ ƒ  k  … k‡
,  ; ’  ,  :  ,  : ; ’ :  :
O ; *  k  ;  … ’ k  ‡
*
…O     : , ‡ : , ;
’ ;
k Rn  R  R n  nqR  Rrk  R  
„f#,~

k ; k ƒ   u ; k  ”k
k R ”k  n ƒ ;k
’ u
UM:$1*-!!>*0++

  
u "Sp&)(+_v`
n  H

3 C k  Rrn  R  R k ™  ln
kT9
k; k    ;UM:$1*:/H>*#?+
ƒ ƒ ’ u ;k [
$G

 B3  u
: m   …  < R :m  = R
„ kp&)(?_2` W;&^9:6)( 

R ;  ‡ R ;
’ ; ;

t ; ƒ
C ; : m ’ ; :
"ST9  
t ;
3
kp6%>(?$.&;(?
:
  < : = R 
’;    ;   : t   ;  ’ R : ;  ; ƒ  R ;
  
"ST9 C ’ ;  t  R  H :; m t ; : H ;14Nw%NT"SN
: t :mJ[
 
$2")9:,%1  

 … kg‰HJk”wHw[ [ [ HJ;k :”‡ ƒ k>:kMRr k”˜ R k”˜5k”Rr ke™ R[ [ [{R k:k: Rrm kg R kgk R: k” [
;
: „f#,&^9:?$
9 6 
  J"($(
)P"  

qV$]/,%")/1$f+9,7219:,)+!")*(?*2EK9n")6)!9
[
5o9:E EK$0/S,S c 576Sp6%,,)9Ei)(+$8z;,%V")$8>$(?EK$W;9:, &^$6%]+9:6=†?9:6)(?f+6)&%&F9:?$(
7kg ƒ  … kg‰HJk”wHw[ [ [ HJk;:e‡•[
 ƒ [
576)$

j  lk> –k”˜hH 2k” ƒ k”˜


#, &F9:?$/9:(?

 ƒ u [ lO  R u [


9:H 2OT/,%")/1$#")6d&)/6)]:(+,)" ")$2"%$6DCd,)9Ei)(+$ 2$ )$Gv? 29:,
;
&^9:+$ %`0/9(+
… ‡ +
  lk   
#, 19:,)Y>(?6)!v!,)+S6%,)$.+6)H>$ a&;(f(?*16)(+(?$,)1$|

+ + $Y019:,)?+(+6)HUW)9,d,%9>$ a+ Žp!,)")/1$#+ ")6d&)/6%f:(>,%"d"%$2,)9:Ei)(?$
–k   wH     ƒ   R 2k    ƒ k   w[
  O  R u [
$ [UM:$1 +
3#,%21$219:,%")!+/9:,)0
 ƒ H     O R u H [
;
NkM  ƒ k;:   ƒ   … kg‰HJk”wHw[ [ [ HJk;:e‡ H  … ƒ R [
mL6)/?576)$ ;/^Mn/$,- )&^9:6)(f>9:6D
5o9:E EK$  
 ƒ   ƒ [ O  O R u ‡•[
;/S$xCD/?+$ k>$V5n6%$
; ; 
3kp9 C
 ’ [ ’ ’
`0!9:(+ 9:6
    ;
+ + +
 u … kg‰HJk”wHw[ [ [ HJ;k :e‡  k k + R + +k k + R+ [ [ [{R k k + 
PRv/,)? ;
{ ƒ „ ; ; ;

u u  u 
#,dK!9:(+0 ƒO‹:„

  k + k + [ [ [ k +   
u k k  [[[k   
ƒg"l&%(+_]` „

ƒ  u
[
 … kMJHJk”hHw[ [ [ HJ;k :e‡ ƒ H
 ƒ 
PR2&F9:6%(f5n6%$ )/Feg6%]576ST/Sc Hf*:/H>*#";4,)ƒ „o$GƒŽ‹:„GN


#(W)Žp*-!!>*v";,)ƒŽ‹:„o/E &)/!576)$#576)$.OT9:,d! >$(+E $f";,%v!$8EK$Ei)(+$

 … kg‰HJk”wHw[ [ [ HJ;k :”‡ [


")$.")(?9:!+$W)1$576)V$?2/,%19:E &;4+/i)!$#UM:$1Žp*-/H>*v";,)ƒ „GN
3
09:,%1

' + —H +  + 
u Rrk ƒ ; : R ; Rk +  … k ‡  : [ +  
„am{9:6)(f>9:6D ;"Sp&)(?_2` W)9,~

+ … u Rk ‡   : + k ƒ  : [
,  , 
PR2")9:,)1

UK …  ‡ ƒ  …  ‡•[
’ ; 1  [ 1 1 
9„f#,d

H
eH 
ƒ [
qV$]/,%")/1$f+9,7219:,)+!")*(?*2EK9n")6)!9
mQ96)(f>9:6)]$,7+/$(+ ;9:, &^9:+$
7
:
! 1  
1 ƒ [
, 
`0!9:(+W)&F96)(a>9:6D


3kp&)(+1 _v` 1 
m  1
;
:
k … ‡ : 1 : 
 ’ ƒ : m,   k  

*1  1 *1 *

, : 1 1 ,
ƒ  * ,  k 

*1
m  1 : 1  
:, 
 ,k:
 *
,
 …  ‡ 1 k 1  
ƒ ’ ; ,
  [ [ [ ƒ -:M[
3 C K …  ‡  …  ƒ ‡ H ƒ
UM:$1*:/H>*#?+
kT9
ƒ ’ ; ;19:E EK$84,),)9:,)1*N

*+ 
 + 
* + K :

7„f#,dM$6%vE /,)!EK!+$(
, + 1 , ƒ [ 
+9:6)]/19:,)")H>!9:,

j H
  
#,dM4eg/(?$#")6d/!+>4:$8")/?1(?$UN
T/S$xC%!?+$ K+$/f576)$
’ u + 1
eH/9:(?WD$,d$,)!$M4,-26),)$.iF96)/$8")$./

7K :  RrK :   K :  m K :    ƒ   [



iF94</>$8, Z&F9:6%(f3'EK$G++(+$8";,%2/iF94</>$8, ;/$8,)9:Eki)(+$8")$.&;4/(+$f")/E /,76)$

’ ’ ’ ’ ;
")$
 

 
m(]?6)!+$W;9:, E /,)!EK!+$(+!$8,)9:Ei)(?$8")$&;/(?$f$,?+6)(+,-0576)$.!$],)9:Ei%(+$
")$.iF96)/$f";,)2")$6nC iF94</>$f")!\F*(?$,->$v,%$19:,->!$,),)$,-v&)]")$.&)/6)]")$.6),%$
6),)H>*N%VT$,?6)!]576)$1jJ;576)$i^9</+$#19:,-+/$,%")(> 9:6 i^9:6)/$N


‘-„f#,d&F9:?$
ƒ

R ; 

R ; 

R ; 
ƒ ; R ; R ; R ; R #;  R  ;  R  ; R #; 
; ; ;
`0!9:(+

3kp&)(+_v` ; R ; R ;  CH C  ƒ … # ; ”‡


 ;9

30$.EKbEK$' 
R  ;  R  ;   H #;  ƒ  [
 ˜
  R  R    R  ˜ … R e‡ [
˜
;$G
#
; 
`0!,)+L
;
ƒ ;
: ‘ 6 
  J"($(
)P"  

I!W%+9:6=†?9:6%(+]"Sp&)(+_]`     „R/R  ƒ 
  ˜ MH ƒ  ƒ  ƒ  [
")9:,%1
ƒ !
[UM:$1*-!!>*v+?
;

lkmHJn gH kk  R’ kCk>n†nqRrRnn   ; H


: „aqV$EKE $N

mQ96)(f>9:6) ;9:,d~ ;UM$1*-!!+*#+? 5k ƒ nh[


?A @B | C B
k  kCn†Rrn   < k  k>nqRrn  =  k  Rrk>nqRn 
k R’ k>k nq RrRn n   ; k>nhH
*576)HM46%2w  
’ 5k ƒ nh[
1Npw%NT"SN
u
;576)ˆ$Y2Mn(+/$4N;PRvŽp*-!!>*0'//$6~+?
+ 1
 … 1H hHw[ [ [ H -:e‡ ƒ .- * + /21 -): + R  +˜R 1˜ R 1 H lk ƒ ˜
+ +
#, &F9:?$ ;$G &^9:6)(f>96% —[
`0!9:(+W mkg k”h[ [ [ k : ƒ [
)$W;"Sp&)(+_]/$8/$EKE $
k +
˜ r
R k 1
˜
 … ‰H wHw[ [ [ H -:>‡ ƒ ; . - * + /21 -): k  Rk k Rrk 1 + 1
+ + 1
1 k  k k Rk 
+ 1
+
ƒ . - * + /21 -): W … k Rr+ k ‡ 1 k + Rr’ k k Rk 1 Z
+ 1

; .- -): … k Rrk ‡


 * + /21
 *+ : +
ƒ  ’  ; ,  k
 … ‡ … kM k>h[ [ [ ;k :e‡  
 … ’ ; ‡ ƒg` „

ƒ  ’  ; … [ ‡
3
09:,%1  … ‰H hHw[ [ [ H -:e‡ ’ ; H
[UM:$1*:/H>*0?+  ƒ  ƒ [ [ [ ƒ -: ƒ [
;
 … kmHJnhHJo{H ‡ ƒ k>n‰oˆRn1o sRro CkNR>k>n  k>n‰o 
nƒ 1o … kNRC‡Rk   n:Rro ’ ; ; u  ! ! n1o 
‹:„am{9:+9,)

#, ,)9>$.576)$  … kmHJnhHJoAHC‡


o$YvYcnEK*G>(+!576)$N
’ u
! 2n:Rro  n1oNO g[
’ u  !
5a], ;+
;
;
:%
˜
  … kmHJnhHJoAHC‡ OQn1o … ksRC‡ O   ƒ [ 
u;
`0!9:(+W)"Sp&)(+_v` 'W)9:,~
;
5o$.5n6%Q4++6%(+$./$8(?*+6%!j=]";,)21$.1U4N
5a],
u ! 2nqRo  ’ ; u  ! n1o g[
;+

s
k
`0!9:(+W)"Sp&)(+_v` 'W)9:,'1$G+>$8e€9:!8

R   k R JH nhHJoAH kNR [
 N
 … kmHJnhHJoAHC‡ On‰o … kNRC‡—R 
:
n r
R o  n1o 
u  ’ ;u !  ƒ u u 
 … kmHJnhHJoAHC‡ O   kŠR HJnhHJo{H kNR 
#,H>*(?$81$.&)(?9D1*")*

 kNu R kNR u


ƒ   nhH u H u HJo   1U4( $?2?cnE *>(?/576)$

O  n:Rro H kNR H kŠR H nqRro 


 kNRu  n†Rru o ksRu  n†Ru o
ƒ  u H u H u H u 

O  ; H nqRo H kŠR H ; 
 nqRro u u ksR
ƒ   u H ; H ; H u 
O  ; H ; H ; H ; 
ƒ [
5 u
;
o$.5n6%Q4++6%(+$./$8(?*+6%!j=]";,)21$.1U4N

+ +
… ‡ƒ k
:
H -k
+
 &^9:6)(   H lk;: g[
Œ-„f#,d&F9:?$  *+ ;UM:$1
t ;$

j  ‰H hHw[ [ [ H -: g, [ 


 ƒ ‰[
-: 
j ƒ ; H  * + :
9:!$,- ^#,d&^9:?$
+ + 1
j 
 ;!Fc 1//(?$E $,-f*-/H>*4N
+  + 1

uH  … ‡ƒ ,
)9:,  k R u kk * + 2
/ 1  [
 ;      [
 : 
 VeH  ƒ

mQ9:6%( ;9:, ,)9+$ *

,
: ‹ 6 
  J"($(
)P"  
: 
  
 ƒ ;[
    
`0!9:(+W)"Sp&)(?_]` 

  ,
:
;    ƒ 
I!]/9(+#
   : + +
k  •  R k
+ 1 + 1
k 
: +
k  R k k ƒ  … ‡[
+ 1
*
, :
  * +  u * + 2
/ 1  
* +
u * + 2
/ 1

3 C *     
 ,
  
 … ‡ H
 ,
  ƒ  ƒ [ [ [ ƒ -:g[
;
kT9
 ,  
[UM:$1*-/H>*#?+
;
j
ƒ  … kg‰HJk”wHw[ [ [ HJ;k :e‡+V :    kgRrk”†RQ[ [ [{Rrk;: ƒ 
„ 9 !
:

:
   :
$ 3ke€9,)1+/9:, ")*Gz;,)/$.?6)( &;(.
;
 … kg‰HJk”wH6[ [ [ HLk:e‡ ƒ … ‡  k  R *   … kg‰HJk”wHw[ [ [ HJk;:”‡
’ ; :  , +  , 
C   … kg‰HJk”wHw[ [ [ HJ;k :e‡
9
ƒ k ;  + ,  k [
+
j lk
[/?9:,)f6), !/++:$813?+/576)$
7+ kMJHJk”
7kg t … t k”wH C … sk [
S+9:6)f/$ ,)$.?9:,-v&)]*-6nClW;!S$,d$GCD!?>$.")$6DCFW[&;(f$GCD$EK&)!$ 2+$/

 :kMJHJk”+ hHw[ [ [ HJ;k :”‡ ƒ kg2Rk>J‡g: GRr+ kg k”: JIH +


576)$ ;9 "%*+!:,)$.3E 9c:$,),)$(+H>J)E *+/576)$#")$

  …  ‡ + k R * +  … kg‰HJk”wHw[ [ [ HJk;:e‡  H
#,d

C G  + k sI
9
ƒ ,˜ $G
ƒ    ’ ; , ˜ , ˜ kg k> ƒg$]?
ƒ u
)9:,

/G ƒ –I ƒ … ‡
’
&^9:+$ $ ?„xN

 … HLkMRrk” Hw[ [ [ HJ;k :e‡ ;  … kg‰HJk”wHw[ [ [ HJ;k :e‡ ƒ I … … kg2Rk” ‡ kg k”1‡
`0!9:(+8
;
’ ’ ƒ Ig[ … ’ kM—‡ … k” ’ ’ ‡ ’
 
 … kg‰HJk”wHw[ [ [ HJ;k :”‡ O  … H Hw[ [ [ H ‡
#, $?v$,d&)!$/,~"),)]3?!>6)4>!9:, ")6d196)(+ jNHN!N
3
kg ƒ k> ƒ [ [ [ ƒ ;k : ƒ [
09:,%1 fUM:$1*:/H>*#?+
  …  ‡   ˜
#(W)!1
ƒ  ; H  … H   Hw[ [ [ H ‡ ƒ :  : ’ ; R   :; m ƒ  : [
%$2")9:,)1

PRv/,)?R  :; C˜  kg k>h; [ [ [ ;k : … ’ ‡mR *  k ;  H


,
UM:$1*-/H>*#?+ 2kg ƒ k” ƒ [ [ [ ƒ ;k : ƒ  ; [ ;

 j  „ 9:! s3'+&%J)_(?$")$.1$,->(?$ -Œ
$G2")$.(>Uc:9:,  [ j G7‰H—GswHLG–˜hHLG–™
9:!$,- 2")$
: Œ

:[
+0/21 G + G 1  O
&F9/,->f")$

G G
+ 1
3kp&)(+_v` 'W;9:,d~ * + /21
˜ ;
9:6d$,)19:(+$ +0/21 G + G 1 O ˜ * + /21 G + G 1  ! H [M$1*-4/!+*#+?  G
+ 1
G ƒ
19:,)Yj,->$4N
;
+ !
* + /21 G G  * + /21 = ŒTG  RrŒTG   * + /21 ŒTG + ŒTG 1 [
3
kp6%>(?$.&;(?+ 1 1 + 1
ƒ T
ΠG T
ΠG
u ’L’ “  ’w’J“ ƒ ; u ’ u ’J’ “  ’6’1“
> 
’
3
‘‘ * + ™ Œ7G + ‘‘  * + ™ Œ7G +  * + ™ Œ7G + 
0$.&)/6%

‘‘‘  ’J’%“ ‘‘‘ ƒ  ’J’%“   ’—’ “


, ™* + , + * + /21 , + 1
ƒ ,  ŒTG R u + Œ7’L’ G“  1 Œ7’w’JG “
  R * + /21 Œ7G Œ7G
ƒ u ’J’%“  ’w’L“
‘ ™ +‘
 
u’ * + /21 Œ7’—’ G“  Œ7’6’JG “ ƒ  ’ ‘‘‘‘ * + ,  Œ7’J’%G“ ‘‘‘‘ O  H
+ 1
m(]?6)!+$ pUM:$1*-/H>*#?+

* + Œ7G
™ + +
, ’—’ “ ƒ ’ “ _H + 1 Œ
;14NwDNp"SN $?2OT/?9:i;(Ycn1$,-+(+$.")$ G [
#,'")9:,%1  * + /21 G G  O  H
;$v!,)+ + /21 G + G 1 O ˜ <   = ˜ ƒ u ˜   H
;! lG H ! ; ; ! G G
+ + 1
Œ
ƒ 
e[ 5 /G/ GsJGs˜—G–™
 M:$1*:/H>*#?+
U $Y0Žp!+9:i)(?cn1$,->(+$8"%$ ;$G]/$2"%/?>,)1$ +9,7


>9:6D>$]*-/$]/9:(?+576)$ o$?>$.")$(+,)!_(+$.19:,)")H>/9,~+/,)!z;$8576)$
$Yv6%,d+*>(+_"%(+$.(+*:6)/!$(W)$2$,-+(>!,)$8")9:,)1.576)$ $Y0Žp!+9:i)(?cn1$,->(+$8"%$
+ + 1
G [ ;`0/,%+L + /21 G G O u ˜   H 
)M$1*-4/!+*#+? 2G7 G–JGs˜JG–™
v$Y26),d+*+(>_")(+$

5
7G7 G–JGs˜LGl™
(+*:6)/!$(N o9:E EK$8,%9:6)]+9:E E $f&;(]J-cn&F94>J)_+$";,)]/$81]"ST*-!!+*W%14p$?
")9:,)1.576)$ 0$?v6), >*+(>_")(+$.(?*:6)!/$(N

: 7k ƒ : … kMRrk”:R[ [ [hRr;k :”‡•[


k  * ; ’k  ƒ [
„f#,d&F9:?$

`0!9:(+ '$

 , ;
: 6 
  J"($(
)P"  

gk  k”w[ [ [ k;: … kg k” [ [ [ k:e‡ : k 


30$.&)!6). … kgRrk”†RQ[ [ [ARk:e; ‡ ’ … ’ kg—‡ … ’ k>’‰‡ [ [ [ … ;k :”‡Šƒ :  …, k  ‡ 

  H ;
’ ;’ ; ’ , ; ’ 

 
#(W)"l&%(+_2` 'W)&F96)(f>9:6% ;9:,d~
:
 
k ƒ * k  k ƒ *  +k    +k  [ 
+ +

; ’  ’ , ,
: ,




 / : 
 … k  ‡    k    k  [[[   k       
,  +
3
, ; ’
09:,%1 

: , , :
4 : 
 

 ƒ   kH -k
, 
)1U(]1jJ;45n6%$ &)&;4(>4<!f$GC%1G>$E $,-

e€9:!N
:
k
… : , O  :;  H
k‡
PRv/,)?R  ;19:EKE $#")*+!(+*W[UM$1*-!!+*

, ’
2k ƒ kg ƒ [ [ [ ƒ ;
k;: ƒ  R ; [

+?


-; dg^:a f ?AD

;
=  >
 +Z gH1R  W [
“
„aqˆe€9:,)1G>/9, $Y219:,)1UM:$+6)(
309:,%1W;"Sp&)(?_2OT/,)*-/H>*2")$$,)?$,SW[9:,~
k  Rn  O  sk Rrn  H k  Rrn  O u … kNRrn‰‡  H 5k ƒ hn [
u u   u   u  [
;1Npw%NT"SN [UM:$1*-!!>*v+?

qˆ'&)/6)f&^$+!+$819:,)Yj,-+$
ƒ u  ƒ 1jJ)$(?1jJ)*$k$?2")9,)1

: +

 Z H1R  W}[ *+
, ƒ ; H
“  ; ’ 
‹:„oqˆke€9:,%1>!9:, $?219:,-M:$GCD$+6)( )mL6)!+576)$
+ ; 
/$  + ;
&F$6%M:$,70+$(?Mn/(f")$ &F9:!") #";,)]Žp!,)*-4/!+*v"%$ $,)+$,SN[SMn/$,-
:
*+
,  ’
+ 
; + : +
[

;’ ,
;
:
: + 
5 j
: +
* +    ;
#(W;"Sp&)(?_ 0N N!W;9:,~
: + ,  ;
*+
,  ƒ
,  ’  ; :  ƒ  ’ H
")9:,)1 *+ + 

UM:$1*:/H>*#?+
;
 ƒ  ƒ ; [ [ [ ƒ -: ƒ  [
’ ;
;
j  ! :
Œ-„X@ 
…  ‡ [ :
= R k > : R  “R n R R R 
qˆ'e€9,)1+/9:, o$Yv19:,-M:$xC%$+6)( )`0!9:(+8
:

n k u ; u ; :



; ;  k n
ƒ u

R ;u: n k   :
R
k R n  H = R; k > R  R n  : : [
j nt k ! u n k u u ƒ u
#(8 ;")9:,)1
; ;
’ ; ƒ k :  [ n :
@ 
#,&^9:+$
= R >’ R ; R  :  n R k  m
n k u … kŠRn'‡ … ksRrn‰‡ u ;
 
`0!9:(+ T*1(+H
; n Rk  ;  ksRrn
  
  
[
u u
9:6d$,)19:(+$

5
…  ‡   H 
o$?>$")$(?,)/_(+$8/,%*-!!+*#$Yv6%,)$.19:,)?*576)$,)1$!EKE *"%!>$v")$.319:,-M:$GCDH>*")$

“ V g H  –kmHJn
3e€9:,)1G>!9:,  o+6%( ;1$576)ˆ4++6%(+$319,)1!6)+!9:,SN

 V   gH  –k nh[  
’;
uv9+9:,)]576)$.OT*-4/!+*#/!$6~+? ƒg$ 0576)$/19:,)576)$>„GW[9:6
$
’; ƒ
kNRnqRro ƒ 
„aqLp!,)*:/H>*#w&)(?9:6%M:$(]$?vJ)9:E 9::_,)$W%9:, &F$6%2")9:,)1.+6%&)&F9+$(2576)$
ƒ 4„GN

;  
’“ 
qˆ'e€9,)1+/9:, $?219:,-M$GCD$W^")9:,)1."Sp&)(?_2OT/,)*-!!>*2")$ $,%+$,
; 

k kR  n‰o R n  R n  o'k R o  R o  k>n  % k … k  R  n1o6‡2Rrn … n  ; R  o'kC‡Rro … o  R  k>n‰‡


$kƒ=„0

  
k < k  R  n‰o = Rn < n  R  o‰k = Ro < o  R  k>n = O ; R  [
V+6%‡2")9:,%1"%$&%(+9:6%M$(2576)$
:: 6 
  J"($(
)P"  

1Npw%NT"SN k ˜ Rn ˜ Ro ˜ R   k>n1oNO ; R  [


k ˜ Rn ˜ Ro ˜ ƒ … kŠRrn:Rro'‡ ˜  < k  n†Rrk  oqRkCn  Rrk>o  Rn  o†Rrn1o  R u kCn1o =
#(8

3
˜k Rn ˜ R o ˜ R   k>n1o  ’ < k  n†Rk  oqRrk>n  Rk>o  Rrn  oqRn‰o  = R  …  ‡gk>n1o
09:,%1|ƒ€"Sp&)(+_ƒ4„?„0

ƒ O ; ’  k>n1oqR  …  ‡Mk>n1o  ’ u
u
ƒ ;; R’  …  ! ’  ‡gk> n1kŠo Rn†’ Rro ˜
ƒg` „

O R  …   ‡      
’
ƒg` $G21U( „
;
R  
ƒ ;R ’
ƒ k; [ n o
!,;/$EK$,-UWn9:, R
k R  n‰o n  R  o‰k o  R  k>n  R W
    ;
k ƒ n ƒ oA[ 5k ƒ n ƒ o
$2Žp*-/H>*v,)$.&F$6%2UM:9:!(2!/$6~5n6%$?V$!/$.'/!$6~";,%2` 'W;1Npw%NT"SN;576)$&F9:6%(
[#(W;/l$?]eg1/!$#")$8M:*(+!z)$(]576)$W;? 0/9:(?f/^c  i%/$,
*:/H>*N

  … kC‡ ƒ n†RkoqR R oqRkNn R R kNRron†R R … kC‡ … n‰‡ … o'‡ H


; ’ ; ’ ; ’
„ZmQ9:?9:,)
C nhHJo
9
; 5 ; ;

 Rr G
+9,7219:,)+!")*(?*219:E E $0z%CD$N 0T$Y06%,)$.+9:E EK$8"%$.e€9,)1+/9:,)]=‡K,%$]$2")6
T T
G J
H I  g[ 5
’ “ I 
cn&F$ UM:$1 0T$?v")9:,%16%,)$.+9:E EK$8"%$.e€9,)1+/9:,)


ˆW gH Z\[
19,7M$GCD$NFV$,"%*196)/$.576)$./$8EK=CD/E6%E ")$ $Y0=+>$/,-26DC~$GCn>(?*E !>*
")$.OT/,-+$(?M4!/$v"Sp*G>6)"%$W;/1 [PL,(+/?9:,),;,-]")$.E bE $#&^9:6)(]/$]6%>(?$
;
EK=CD/E6)E $,d6),}ƒg9:6 &)!6)+!$6)(?j„a")$2J76)Hf>(?/&)!$> … kmHJnhHJo6‡ C kmHJnhHJosV  gH  [
M4(+/i)!$WD9:, $,")*")6%!]576)$./$8E $Ei%(+$.")$.-6)1jJ%$"%$Žp!,)*:/H>*v4++$/,-]?9:,
o9
#,~M:*(?!z;$8eg1/!$E $,-f576ST$,1jJ)16),")$.1$2+(+/&%/$+WDOT$GCD&)(+$+?/9:,d$Yv*-!$
;
w W)1$576)ˆ?+6)(?$.3'19:,%1/6%+/9,SN

: <   = $Yv19:,-M:$xC%$+6)(  H
“
„ mL6)!+576)$3ke€9:,)1G>!9:,
a ;9:,ddƒg!,)*-4/!+*0")$
$,%+$,;„.
:
*+ k
R+
+
    *+

k R +

: + 

, 
k;  ; ,  k; 
ƒ=„
'
:

  * + : k + R  * + : +  
ƒ ; ,  ; ,  k; 
 (8
#
 H
: +
…  C‡  H
 ; * +  k   ; ; * +  k ƒ  ; “
-„amL6)/?576)$ ;3ke€9:,)1G>!9:, o$Y019:,-M:$xC%$+6)( )$Gv9,~
: +

, ,
gH eH;$29:,d~

 
“ ;
i[„fmL6)/?5n6%$ ;/ke€9:,)1+/9:, $Yv19:,-M:$xC%$+6)(


  
 
: 

;  k ; + : k +
* + + 

 ,



,



ƒ
3
*+
:
k +


R +

0$Kƒ4„xW[-„o$2i[„xW)/FMD!$,-2/E EK*")3=>$E $,-0
   R 


,3 
 k; 
;
19:EKE $8,%,)9:,)1*N

0$.&)/6%W;";,%21jJ)16),%$")$v!,)*:/H>*B")$.19:,-M$GCD!+*W[/lc~*-!!>*0+V$
: +
+$6)/$EK$,72? 2kM ƒ k” ƒ [ [ [ ƒ ;k :g[ 5 , k ƒ ; H
o9:E &%>$x@ >$,76")$./19:,)"%!>!9:, *+

9:,d$,d")*")6%!]576)$OT*-!!+*#'/!$6~+V$G2+$6%/$E $,-2+ 2kg ƒ ”k  ƒ [ [ [ ƒ k;: ƒ  ; [


' … k HLnAHLo{HmHA‡
„aqV$.EK$Ei)(+$8"%$:6)1jJ)$$Yv6%,)$8e€9:,)1+/9:, 19:,-M:$GCD$$,d1jJ;16%,)$"%$
u
Tk HLnAHLo{HmH/V   H  [

M4(+/i)/$N%PZ!/$.4?>$/,-]+9:,dEK=CD/E6)E $,d6),d")$ 576)!,7+6)&)!$>

j 
>$/f576)$
H 
9:H
CH  V  gH H HgH  MH   [ ’
/$8,)9:Eki)(+$.")$8M4(+/i)!$a*-/$]w )$2")9:,%1  !$8,)9:Ei)(?$.")$.1$!/$
576)V+9:,-v*-4/$]w [UM:$1
u  … ‡ … R …  ‡  ‡   R   [
;
:p:HV")9:,%1L")$LE|C%!EK!+$(lOT$GCD&)(?$?+/9,
ƒ  ’ ’  
 …  ‡ ƒ   R …   ‡  R  …   ‡   R  

• R …   ‡  R  ’ …   ‡R  …’   ‡     
#(8

ƒ ’ u ’ ’  ’ 
:9 6 
  J"($(
)P"  

 … ‡  
ƒ u  R 
 … ‡ ’ ’        
…  ‡ ƒ u
’ ’ ’ u 
")9:,%1 a$Y2E|=CD!E|!$v?+ $?vEK=CD/EKOW

  g[  ƒ u R 
  [
1Npw%NT"SN
ƒ u 9:6
ƒ [PRv/9(+ 

! ’ + 
9 : ƒ R R[ [ [{R   ƒ Hw[ [ [ H ! ƒ  ": [
: ; + u;
„f#, &F9+$ $W;&F96)( 
+ :;  ;
:  ; ƒ ; : : ƒ [
;
`0!9:(+  
'$G *

+ 
  ƒ ;
* +



j  ƒ 2R : RQ+ [ [+ [{R :  [ +


 , :: , ;

 * + u +  + +
9:H

`0!9:(+8
 ":ƒ  

 "Sp&)(?_2`  &^9:,)")*(+*$
, , : +
  
ƒ , +
 :
: +    
I!W)"l&%(+_]`    + :  ƒ [
  +



,


 ;
  H ,
3
09:,%1
 : )M$1*-4/!+*#+?   ƒ  ƒ [ [ [ ƒ -: ƒ [

7 B3
„ kp&)(?_2`
;
 &^9:,)")*(+*$ 
 k  n
R  … k ‡  … n ‡  ƒ k>nh[

 
;



&!  R;   
“ 
‘-„Lqˆ8e€9:,)1+/9:, $?B19:,-M:$xCD$#+6)( N7PL,|$\F$W7$/!$f$Yo")$6DC

e€9:!a")*(?!M4i)!$8+6)( 
aW;$]&^9:6)(f+9:6% ;  
DW;9:,d   $ $ … ‡ ƒ … R’   ‡ ˜  gH
 ;1(

   [ +  + ;+ 
 ƒ H u H6[ [ [ H H ƒ  [5 H  g[
 d$Y01(+9:!+>4,7+$.+6)( 
mQ96)( )9:, &^9:+$  o9:E EK$  %9:,d
:
`0!,)+ŽW%"Sp&)(?_2OT/,)*-!!>*2")$ $,%+$, 
; :  ;
 ; * +  ;R ƒ  ; * +    ; R +  ;  ƒ …  h[ [ [ -; :>‡  R H
+ 
  ,  R
DUM:$1v*-4/!+*

, , 
; ; ;
;
:7

+?  ƒ [ [ [ ƒ -:M[
™* + +˜ + +˜
/G ƒ  H –G ƒ G [
’
: „f#,d&F9:?$ )$G

™* + , +
`0!9:(+ +G ƒ ;  G [
,  < ˜ ˜ ˜
3
kp&)(+_v` 'W;9:,d  ; G7 +  ˜ ™ =  ƒ  ˜ m™ ƒ ;  [
G;  ; +  ƒ u HH M[
™* + +
#,&)(?9:6%M$"%$E bE $8576)$ &F96)(

PR04/,)? G  ; [ 
3 ,
™* + +˜  * + ™ +  ˜
kp6%>(?$.&;(?W;"Sp&)(+_]OT/,)*-/H>*v$,->(?$E 9c:$,),)$2"ST9:(+"%(+$ '$ W;$]`   
; G ƒ ; 
;
; 
, ,™ + ™ + 
ƒ ; * + , ™  + ; * + ,  ™ +
  * +  …  ˜ m™1‡
 ƒ
*+
ƒg9:,d
;  ;  „

™* + + , , ;
3 /G
, [
09:,)1

PRv/19:,%1/6%+/9,d$?v?+6)(?*$N

j
‹ „X@ VŽp6%,d"%$],)9:Ei%(+$]$?v,n6%OW)&;4(f$GCD$E &)/$
:  ƒ "!
R  R ˜ ˜=  ˜ ˜H
qLp!,)*-4/!+*0p*1(+H

  u 
<    )576)ˆ$Y2134/(+$EK$,-aMn(>!$W;UM:$1*-!!>*

j  2ƒ H  H "ƒ gƒ H
+? DN

R   = <  R  = <   R # = O < u   $R   R   = <  R u   R   = <  R$  R u   =


@  )9,~
< 
O W  … $R   R  ‡ Z ˜ ˜ 
   R  R  
ƒg"l&%(+_]` „

ƒ   
' ‘ 6 
  J"($(
)P"  

O   ˜ R ˜R ˜ 
$,-+(+$
  
ƒ€"S4&)(+_ OT/,)*-!!>*

E 9c$,),%$2"Sp9(+")(?$Œ'$G#‹„
˜ R ˜ R ˜= H
 ƒ  ƒ [ ƒ 
< 
UM:$1*-/H>*#?+

&!    23
+ “  ’ : +
Œ-„Fq{ae€9:,%1>!9:, $Y{19:,-M:$xCD$N k4&)(+_{Žp!,)*:/H>*{")$ $,)?$,SW
: 
/^Mn/$,- ; *+   … ‡   ; ; *+  ƒ   ;  ƒ  [
3
, , ’ ; !
+  u
kp6%+(+$.&;(YUW)"Sp&)(?_2OT/,)*-!!>*2$,-+(+$EK9c:$,),)$v"ST9:(?")(+$ $
: + : ;
; ƒ ; * +  ; * +  

: , +
 ,
R †RQ[ [ [{R -: 
* + O H     O  H
"ST9C
,    ’ ;
)$G2")9:,)1

’
1$.576)ˆ?+6)(?$!$8(+*+6)Hj4N ;

G: ƒ  : = > :   R :    : = > R  : = >  : = >


„f#, &^9:+$
       

–I ƒ = u >  u  R   u  = > u R = > u = > u


       

5 j u u I [u u u u
$

3
H H
M 7
G
kp&)(?_ 0N NHW;9:,d  
= >   R  R <  = <  =
#(W)&%6)/?5n6%$ ?9:,-2/$]4,):/$f"ST6),+(+34,):/$4WD9,~
 

uTI ƒ [ u ’ mG7u   [ƒ u  ƒ ; ’ <  = R <  =


    
 

")9:,%1
ƒ ; : ;PR04/,)?R

G :   G7 H mG:   :   [
I!]/9(+W%"S4&)(+_2Žp!,)*:/H>*2$,->(?$!$fE 9c$,),%$2"Sp9(+")(?$

$G 

   ;1Npw%NT"SN

uv94>9:,)f576)$OT*:/H>*#'!/$6~?+ˆ/$#+(+/,):!$#$Yv*576)//4>*(>ŽN

2R † RQ[ [ [{R -:[


~ my ƒ
 „f#, &^9:+$
€
?A@B  @B    B !
'
D

3kp&)(+_650N
: + :
j W * +   Z W  …  %‡ Z * + 
+ *+
+

:

+ 
’ , ’
NV
,
   * + : +  
,
ƒ ; ,
•  * + + 
:
; 
 ,
ƒg"Sp&)(?_2OT/,)*-!!>*2$,-+(+$./$2E 9c:$,),)$2"ST9:(+"%(+$ $
u u
„

ƒ  [
UM:$1*:/H>*#?+  ƒ  ƒ [ [ [ ƒ -: ƒ  [
3
kp6%>(?$.&;(?  ;
: +
:
 t *+ …  ‡ ƒ …
+
 ‡
: +
 …  ‡ *+  O …   ‡


 *+ 
: +
’ ; , ƒ ’ ; ,

*+
,  ’ ’ ; ; ,


: +  … ‡
ƒg"Sp&)(?_2OT/,)*-!!>*2$,-+(+$./$2E 9c:$,),)$2"ST9:(+"%(+$ $ „
u
 t * +
 …  ‡ O ’ ;H ; /  ƒ  ƒ [ [ [ ƒ -: ƒ  [
’
")9:,)1 )UM:$1*-/H>*v++
  ;
+    
+ ,
:
PR2z),;/$EK$,-.

,  ’   …   ’ ‡ ƒ  …  ; ’ ‡ H  ƒ  ƒ [ [ [ ƒ
*+ VUM:$1K*-/H>*++

-: ƒ  [ ; ;
 ;
 B | e€   B ~ my  B ! +
 H + R  +[ < ™ H ™ Hw[ [ [ H ™: =
mQ9:6%(]+9:6% ;9:,~
 ƒ ’ )`v/,)?OW)!$f?6)!+$

    ’   ’
  H   Hw[ [ [ H  -: 
$
;
+9:,-2(>4,):*$v"),)]/$8E bE $#9:(?")(+$4N

3 ’ + ’ N5 ’
kp&)(+_vŽp!,)*:/H>*2")$ oJ)$i-cn+J)$GM^Wl9:,'")9:,)1
: + +

:
™ :
,  ’  ; ,  ,   ’ 
*+ +  * + * + +

3   !   
“ ’
0$.&)/6%W;&^9:6)( z%CD*$W[3ke€9:,%1>!9:, $?219:,-M$GCD$N
' ‹ 6 
  J"($(
)P"  
: + : +
 : +
 
,  ’ ƒ ,   … ‡  ,   ƒ    ƒ  ’ H
*+ + *+  * +  
3
09:,%1 QUM:$1

  ƒ  ƒ [ [ [ ƒ -:g[
*:/H>* ;
+?

: + 
3 kp6%+(+$.&;(YUW)"Sp&)(?_2OT/,)*-!!>*2$,-+(+$/$fE 9c:$,),)$2"ST9:(+"%(+$ $0‹DW;9:,d~

; *+  ; *+ +   ƒ  ;  H
™
: +
   ƒ  ƒ [ [ [ ƒ -:g[
[UM$1*-!!+*#+?
, : , 
,  ’  ;   ’ ƒ  …  ; ’ ‡ [
PR2")9:,)14W *+ + 

#,~M:*(?!z;$!9:(+f576)$.Žp*-/H>*0 !/$6~&^9:6)( ;   ƒ ;  ƒ [ [ [ ƒ -: ƒ  H
1$.576)ˆ?+6)(?$576)$.1$++$M4!$6)(f$?2i)!$, 6),dEK!,)/E6%EN ;
 -; dg^:a f ?AD 
 ƒ Hw[ [ [ H  [
+ +
H gH
„ mL6)!+576)$/$f19D$G‡ 1/$,->2+9:,-2&F9+!+!e€a96d,76)!W;!$f(>41/,%$]+9:,-]>96%>$
a

’
?+(+!1>$EK$,-],)*-=>!M$Nl#, /$],)9>$ )M$1 )&^9:6)(
: +
+ ƒ [   ;
, 
`0!9:(+ ;PRUW;"l&%(+_2` 'W)+V!$]+9:E E $f$2&)(?9D"%6)!+f,)9:, /,)"%$GCD*
+
;
+ + + + + +
+9,72&)(?/f?6)(f>9:6%+$]/$]19:Ei%/,;!+9:,%fw */*E $,-+2&)(+E l!$ 

k  ƒ * [ [ [   K :  =  [ [ [  > 
: +  
K : mm
+
Kƒ :    + ƒ€1U(]1jJ;576)$ &)&)(>4<! e€9:/

K :  mm  ,
$
K : ƒ N„

ƒ K :
: + + gH
N

`0!,)+ŽW%&^9:6)(f>96%
: + +
;!FMn/$,-

 … ‡ ƒ : R * +  k R  * + K : ƒ … „R ‡ : H ;"Sp&)(?_]3e€9:(?E6)!$#")6

Mp
i)!, :E $N
, ; ,  ;

‹:„(5o9:E E $8";,)]OT$GCD$(+1!1$.&)(?*1*")$,-W^9,&)(?9:6%M$576)$ 
: +  
&^9:6)(a>96%

O )O H   K :   k [
  +
; ,
' Œ

K K 
: +  : +    : +
k k  ƒ K K k
`0!,)+L   /: 
 
:
: :    +  + :
: :    +

$W;19:E EK$ : /K ƒ K H , ,
: :  )/FMD!$,-   /:    < :D = K kg k”h[ [ [ k g[
;:
,

Œ-„f#,~ < k  Rrk>nqRrn  n  Rrn1oˆRo  o  Ro'kŠRk 


= < = < =

k ˜ n ˜ R k  n  o  R k ™ n1oˆR k ˜ n  oqRk ™ n 
˜
*
ƒ 
u; u; u ; u 
… k>nqRn1oqRro‰kC‡
*  k ˜ n ˜ Rrk  n  o  R ek ˜ n  o
$

ƒ 
u; 
< k  Rrk>n:Rrn  = < n  Rrn1oˆRro  = < o  Rro‰kNRrk  = … k>nqRn‰oˆRo'kC‡ ˜
")9:,)1

*  k ™ n1oˆRk ™ n  ’ k no k˜no


ƒ 
u; ’ u ; ’ 
… MH u H”‡ … MH H ‡ …u Hu Hu ‡
#
…  H u H ‡ [ 3
 (W;/$f+6)H>$ B$ BE|†?9:(+$,7](?$?&F$1>HM:$EK$,7v/$]+6)H>$

™k ; n1oˆRrk ™ n  k  n  o ; ; k ˜ n  o  gH
$ 09:,)14W;"Sp&)(+_vŽp!,)*:/H>*2")$I6)/(?J)$U4"SW
*

u; ’ u; ’ 
;1$.5n6%ˆ+?6)(+$319:,)1!6)+!9:,SN

„ ?A@B €  @B  ~ my  B !


 ,19:EKE $,%1$8&;(f(?$,)"%(+$OT*576;4+/9:,~J)9EK9:_,)$8$,*//E /,),-a3
#
19:,-+(>!,7+$N[mL(+96%M:9:,%v576)$.OT9:,d~

˜k … nq; Rro'‡ R n ˜ … †o ; RrkC‡ R o ˜ … Nk ; Rn'‡  … k> n1o'‡  [


mQ9:6%(2?/E &)/Hz;(oŽp$xCD&)(+$++!9:,SW;9:, &^9:+$ 7k ƒu ; HJn ƒ ; HJo ƒ ; [
 R   R     … # ”‡  [
qLp$xC%&%(+$?+!9:,")$GMn/$,-
ŠR  sR R u
™ ˜
* < R ŠR   = *   =      R       >
#,*//E /,%$0/$f")*,%9:EK!,;4+$6)(?W)&^9:6)(]9:i%+$,)!(aOT/,%*-!!+*0*576)HM4/$,7+$'

u 



#(W;/$f+6)H>$ … MH gH ”‡ H … gH H 
”‡ … u H H ‡
B$ BE|†?9:(+$,7](?$?&F$1>HM:$EK$,7v/$

 H H H  H H H HH [
  ;  ; ;
+6)H>$
  ;    ;  ;$
3
2k ƒ n ƒ o{[
09:,)1./"%$(+,%/_(?$./,)*-/H>*v")*19:6)/$.")$.Žp!,)*-4/!+*v"%$kId6)/(?J)$U"lN
uv9+9:,)]576)$.OT*-4/!+*#/!$6~+?
' 6 
  J"($(
)P"  

 B | e€  B    B ! ~ my
7k ƒ ; HJn ƒ  ; HLo ƒ ; [  #  ƒ [
;  R   R   [
#, &F9:?$ ;`v/9:(?

PR
k R R
˜ … n†; Rro'‡ n ˜ … oq; RkC‡ o ˜ … kN; Rn'‡/ƒ NR  –R R$
5 j 
  
3
 … R$/R ”‡  [
kp&)(?_ 0N NV

NR  – R  R 
R   $R     R$NR  —
… … Š
 R e
 ‡ R … –
 R
 2
‡ R … R
 M
 ‡ ‡
3
09:,%1 

NR  – R R „ $R ŠR „$R   u [
R 
 … # ”‡  ƒ [
u R u˜  u H
PRUW;"Sp&)(?_]` 

kT93 C ˜ … R ˜
k nq; Rro'‡ n … oq; RkC‡ o … kŠ; Rrn‰‡ u ;UM$1*-!!+*#+? 5k ƒ n ƒ o ƒ [
;
 mG ƒ nqR u oqR   HLI ƒ oˆR u –R  k H
K ƒ sR u ksR enhH
ƒ kŠR u n:R eoA[
„f#, &F9+$

N5 j
;$G

RG I n R K o R
  … k”GRrn1I Rro‰K#R
)‡  … kŠRrn:RroˆR‡ 
3
 k
kp&)(?_2OT/,)*-/H>*2")$ 0N NV

")9:,%1 
k R n R o R   … ksRrnqRoqRC‡ 
n†R u oqR  o†R u –R ek NR u kNR en kNR u nqR eo … k>n†Rrk>oˆRk sRrn1oqRn NRro ‡
… k>nqRk>oqRrk NRn1oˆRrnNR o C‡ O  … ksRrnqRoqRC‡ 
V+6D‡ 2")9,)1.")$.&)(+9:6DM:$(]576)$ 

   lO   
…  1‡ O  ‰H ; !
9:6 $,%19:(?$K
1Npw%NT"SN )1$576)V")*19:6%/$./E EK*")3=>$E $,-B")$2/,%*-!!+*a")$
I1x@ qˆ6)(?/,SN

:
:n Rro … k n‰ ‡ … k o6‡ H oq Rrk … n o' ‡ … n kC‡ H
„Y-„f#,~
kN Rn
k;… o ’ kkC‡ Rr… o n1o n‰ƒ ‡ k ’ … k Rn1’ o6‡ n; ’ n Ro'k ƒ n ’ … n Rro‰’ kC‡ o; ’ o Rk>n ƒ
o3 ’ … o  Rrk>’ n‰‡
 … kmHJnhHJo'‡ ƒ k; R n; R o;  k n† RrRron1o R n o†RRro‰k k R o kŠ RrRrk>n n 
09:,%1 

… k n‰ ‡ … k ’ o6‡ R … n o6 ‡ … n kC‡ R … o kC ‡ … o n‰‡


ƒ k ’ … k Rrn1’ o6‡ n ’ … n Rro‰’ kC‡ o ’ … o RkC’ n‰‡
'
mL6)/?5n6%$
… o Ck  ‡ … o n‰‡  gH
$Y2?cnEK*G>(?/576)$W;9,&^$6%2?6)&)&^9:+$(2576)$ TkOQnsOQoA[
o ’ … o R kC’ n‰‡ 5o ƒ k o ƒ nh[
 … ksRrn‰‡
`0!9:(+8 ;UM$1*-!!+*#+? |9:6

… k ‰
n  ‡ … k '
o ‡ R … n '
o  ‡ … n 
k ‡ … n  C
k ‡  … k … o ‰
n 
‡ 
R '
o ‰
n ‡ 
3
0$Z&)/6%R
gH
k ’ … k Rn1’ o6‡ n ’ … n Ro'’ kC‡ ƒ k>n … k ’ Rn1o6‡ … n Rro‰kC‡ ’
k nh[
 … k HLƒ nAHLo6‡  gH
UM:$1*:/H>*#?+

5k ƒ n ƒ oA[
/,)/$EK$,7W )1$576)V$?2OT/,%*-!!+*v")*+!(+*$4W^M$1*-4/!+*
+?

(3 j  ƒ H ƒ kNRrnhH ƒ nqRroAH
 ƒ oqRrkmH ’;
i[„ kp&)(+_vŽp!,)*:/H>*2")$ 1jJ76)(v&^9:6)(
$ ;9:,d 
ŽO … g‡ … ”‡R  …  ”‡ …  ‡ R  …  ‡ …  M‡
;
… k ’ o'‡ … n ’o6‡ R … n ; k‡ ’ … o kC‡ ’ R … o ; n‰‡ … k ’ n'‡ ’
ƒ o  Rr’ kNk>n Rn ’ k  Rn1’ o n:Rro ’ n  Rro‰k ’ oqRrk ’
ƒ k ks RrRrn n1o ’ o†n R Rrn†o‰k Ro o ’ RrkŠk>R n  o†Rrk ’ n
"ST9 C
†
n r
R o R o†Rrk R kŠRrn ksRrnqRo{H ;UM$1*-!!+*#+?
ƒ  ƒ gH [1Npw%NT"SN k ƒ n ƒ oA[
'… #/R  lR  ‡ < … R M‡  … NR ”‡  R … /R ”‡  … –R ‡  R … –R ‡  … RM‡ 
„f#,~
=
™ ™ ˜˜ ˜ 
ƒ * 
NR u  R u  lR u  R ;   sR >   
  

… „$R g‡  … NR ”‡  … sR ‡  ƒ *  ™   R ™  lR ˜  ˜ R ˜   –R       


$



#,degHf")9:,%1"%/+&)(>4<!>(?$#/$f")*,)9EK!,;4>$6)(+W;&F96)(f9:i%+$,)!(fOT/,)*-/H>*
!
* < > ŠR ™  lR      ™    ˜  ˜ u ˜    =  
*576)HM4/$,-+$ 

…  H H ”‡ ’ ’ ’ … gH u H ”‡ … gH gH ”‡ [


ƒ „


 (W;3?6)!+$
# oEK†?9:(?$#/$2"%$6DC ?6)!+$ o$G

* < >  ™  ;   ˜  ˜ =  g[


3
09:,)14W["S4&)(+_2Žp!,)*:/H>*2")$I6)/(?J)$U4"SW;9:,dd

’ ’
VƒO‹:„
3 

kp6%>(?$.&;(?W;9:,dMD6576)$.OT/,)*-/H>*2")$ j
1jJ76)(2T*1(?!M4H.
': 6 
  J"($(
)P"  

* 
˜R #     g[
PL
, Ek6)u !+/&)!3u ,-B1jJ;’ 576)$.>$(?EK$8&)( # gH;/lMn!$,-
; ; 
™ lR      ˜    = u
 dƒŽŒ-„
’u
* <  
`0
!,) +ŽW%$,?9:EKEK,-ƒO‹„o$kƒŽŒ-„xW)9:, 9:i%+/$,7kƒ
ƒ  ƒ [
„GN
3kT9Cd319,)1!6)+!9:,SW;UM:$1*-!!>*0++
9
… n:Rro kC‡  < … oqRrkC‡  Rrn  =  < … kŠRn‰‡   Rro  =
„f#,dd

’ R … oqRk n‰‡  < … kŠRn‰‡  Rro  = < … nqRro'‡   Rrk  =


R … kŠR n ’ o'‡ < … nqRo6‡ Rk = < … oqRrkC‡ Rrn =
 ’ k  R k JnqRrk ™ n  R ek ™ n1oˆR k ˜ n ˜ R  k  n  o 
ƒ   
u u
*
u 
< … o†RrkC‡  Rrn  = < … kNRn'‡  Ro  = < … n:Rro'‡  Rrk  =
$

*  k  R k JnqR  k ™ n  R  k ™ n1oqR k ˜ n ˜ R k ˜ n  oqR  k  n  o 


ƒ   
u; u u   
H
#, */!EK!,)$0"%9:,)18>9:6%2!$f")*,)9:E /,;=>$6%(+W;$G]19:E &%+$G@ +$,76d")619D$G‡ 1/$,-

* < ek  R k 1n k ™ n  R ek ™ n1oˆR k ˜ n ˜ k ˜ n  o†$R ”k  n  o  =  g[
9:, 9:i%+/$,7]Žp!,)*:/H>*0*576)!M4!$,->$ ƒ „0

u ’u u ’ ;u

j
˜ 
#(W)"l&%(+_2OT/,%*-!!+*v")$ 1jJ76)(
* 
k R ; kCn1o k n  g[ 

u ; u ’
* < ”k ™ n1o ˜k n  kNR ”k  n  o  =   ”k>n1o{H
PL,Ek6)!+/&)!3,-B1jJ;576)$.>$(?EK$8&)( [/^Mn/$,-

’
ƒŽ‹:„


* < ek  R k 1n
V(+$?+$"%9:,)1w'&)(+9:6DM:$(2576)$
k ™ n  k ™ n1oqR k ˜ n ˜ ”k ˜ n  o =  g[
u ’u ’ u ’

… HgH”™ ‡ ˜   … MH H ‡ … gH u H ‡ H
* < ek  ! k n1o k n o = g[ ; ;
#(W)/+6)H>$ XE|†?9(+$8/$f+6)H>$ B$
")9:,%1
’ …  H ’ H ”‡ u ˆƒŽŒ:„ ;
3
  

0$.&)!6)W;/+6)H>$ oEK†?9:(?$#3'?6)!+$ … MH u H ”‡ H
;
''
* < ku 1n u k ™ n  =  g[
’ … HgH”‡
")9:,)1 ˆƒ „
  

* < k˜n˜ ˜ n  o =  [ … gH u H ‡ H
k
PRv$,%z;,SW;/'+6)H>$ XE|†?9(+$83'?6)!+$
")9:,)1
  

u ’u {ƒ „  ;

k ƒ n ƒ oA[
PL,?9:EKEK,-ƒO‹„GW/ƒŽŒ-„xW3ƒ „B$kƒ „GW)!lMn/$,7kƒ „GN[#6%e
qˆ19:,)1/6)?/9:, $,d")*19:6)!$W[UM:$1*-!!>*0++

7
oh ƒ ksRrnqRo{HJo1 ƒ k>nqRrn1oˆRo'kmH so1˜ ƒ k>n‰oA[
„o`ˆM:$1/$f,)9j=>/9,)f")6 1jJ;&)H>(?$W;9:,d&F9+$
)$G

oh o1lOQo ˜ eow o1†R o‰˜lOQow < o  u o‰ = O u < o ˜ eow o1 = R o‰˜6[
qV$]/,)*-!!>*a+96)J;H>*$]+9:,-v/9(+f*576)HM4/$,7+$2w~

o ˜ R o‰˜  ’ ”ow˜  o‰6H  u  ˜ R ’ ˜      6H ’


#(W;3&%(+$E /_(+$8$Gv/k>(?9:/?/_E $#?9:,-v$576)!M4!$,->$vw~

   R  ˜    wH
;1Npw%Np"lN
9:6d$,)19:(+$
ƒ ! j
;")9,729:,dMn6$, &%&)/!1U4+/9:,K5n6lp$//$.*GjH
*576)HM4/$,-+$46d1
3 m w
o  1
o   o1˜wH
")$.Žp!,)*-4/!+*v"%$
kp6%>(?$.&;(?W;1$//$0")6E /!/$6 T*1(+!
1jJn6%(N
;
1Npw%NT"SN … ksR n†Ro6‡  k; R n; R o;   H )576)ˆ$Yvi%/$, 19:,),76)$|ƒg1e N;1jJ)&)!+(+$.Œ%W

&)&)!/14>!9:, 3 T9Cd319:,)1!6)+!9:,SN
„xN k

-; dM^5a f ?AD 


„V#,8&F9+$ ƒ
 R  R 
 R …  R  ‡ … „R ”‡   R …   R   ‡  … ŠR ‡   R …   R  ‡  … sRM‡ 

$ 
[
 ƒ k ˜ H ƒ n ˜ H ƒ o ˜ [

PR
3
< R  =  … R e‡  W <  k  = ˜ R < n  = ˜ Z  W < o  =  R < k  =  Z 
kp&)(+_vŽp!,)*:/H>*2")$  :!")$(8

ƒ  k  o  Rn  k 
ƒ  … #g‡  R … ”‡  
PRv"%9:,)1 O 
 R …  R$  ‡  … R ”‡   R … #g‡  R … ”‡  ƒ  R  R  
#,&)(?9D1_")$.")$.E bE $8&F96)(f/$]6%>(?$]>$(+E $f$W;$,d+9:E EK,-UW)!lMn/$,-.
   
rO  R R [
 R$  R    R$  R    R$  R   ƒ
;
j –kmHJnhHJo{H  l[  ƒ ksRrnqRoqRm[
 "
‹ „ 9:!$,-
: K";,% {#,d&F9:?$
3
kp&)(+_vŽp!,)*:/H>*2")$I/, 49 ] 7B
'9 6 
  J"($(
)P"  

  ƒ  … ksR ‡  R u … n u ‡  R … oˆR ”‡  R  … nqR ‡  R u … o u ‡  R … sR ”‡ 


  … kNRnq; R ‡  R ’ … n:Rro ”‡  R … oqRsR ‡ ; ’
…  u … u  ’…   … !  …  … 
• ƒ 
o†R ‡ R u  u ‡ R kŠR e‡ R sR ‡ R u k u ‡ R nqR ”‡
  … oqRN; R ‡  R ’ … –Rrk >‡  R … kNRnqR ‡ ; ’
u u ’ !
>‡  R u …   ‡  R …  R u ‡ 
PRUW[w',)9:6DM:$U46}
 ƒ  R •   …   R
    R u    ’ u  [
9:6 $,%19:(?$K
3    u u H

;
k ƒ n ƒ o ƒ  ƒ g[
09:,%1/kM4/$6)(fEK!,)/EK!$v")$ s$? ;=+>$/,->$.&^9:6)(W)&;(f$xC%$EK&%/$W;&^9:6)(
;
-; dg^:a f ?AD 

ƒ W + Z: H


ƒ  ……  ‰‰HH hwHwHw[[ [[ [[ - -:e:e’ ‡‡  ; ;+ VV  ’ ;   ;   H


„f#, &F9+$  
ƒ … ‰H wHw[ [ [ -:>‡ V
U’ H ; ;  … ‰H 6 Hw[[ [ [ -:>‡ * + /21 + 1 [
  & 9:6)(f+9:6%
^
 &F96)(a>9:6D
PRUW;&^9:6)( ;9:, &^9:+$
ƒ : .- : -):
   u 
 [
mL6)/?576)$ $ +9,7]z;,%/ƒg"%$1(+"%/,;6nC (+$+&^$1+!e€ $G „xW)3e€9:,%1>!9:,


 +H   O 
  [
")E $]6%,E /,)!E6)E +6%( $2+6)(
PRUW;19:EKE $ )9:,
 1H hHw[ [ [ -:H 
TW Z
 (W)&)(f(>&%&F9:(Y2w 1jJ)16),%$")$fM4(+34i)/$
# ;3ke€9:,)1G>!9:, $?

 … ‰H hHw[ [ [ -:e‡ 


 ’ ;  ;… H hH6[ [  [ -:”‡
H  … H wHw[ [ [ -:”‡  [
4‡ ,)$4N[PZ!/$8=+>$/,-]")9:,%1?9:,dEK!,)/E6%E ?6)(  ˆ$,d6),)$.")$2$GCn+(+*E !+*

p  … 
‰H ’ hHw; [ [ [ -
 :”‡ 
 [ ;
")$L1$ˆ/,-+$(?M4!/$4N`v/,)?OW
m4(2YcnEK*G>(+!$8")$]( :/$W;9:, $,d")*")6)!2576)$
m4(2?6)!+$W  "%EK$G]6),E /,%/E6)E ?6)( $   ƒ  ƒ [ [PL,
 

  [
&;(Y>!16)!/$(WD/'(+*&^9:,)?$6DC 576)$?>!9:,)2-„a$]i;„f$?23'E bE $WD$2/S?6%‡2")$
")*G>$(?EK!,)$(
j 9:Hf")9:,%1 … ‰H hHw[ [ [ -:e‡+ V  [ j  V  gH Hw[ [ [ H
9:! /$8,%9:Ei)(?$8")$
  
+
*-6DC

… ‰H wHw[ [ [ -:>‡ ƒ *  ’ +  1 R * m + ; 1 R *


m + 1
w ƒg$2")9,)1W;!lc~$, *-6DC w@ „GN%`0/9(+#

Kƒ  , Rr  K , :    …  ,  ,  ‡
   , ,
 
<…  ‡ ’  = ’[
PR2")9:,)1
ƒ u; u  ’ ’
'7
j   … ‰H 6Hw[ [ [ -:>‡ 
 
  ƒ [ 
 … ‰H hHw[ [ [ -:e’‡ u  ’ u R
@  $Yv&)/(W UM:$1*-!!>*v+?

@ j  $? !EK&)/(W


; u
UM:$1y*-!!>*d+?
 ƒ u ; 9:6

 ƒ ’u ; [
j
j 7 k  … kmHJnhHJo'‡•[ l N
k 
R n o
‹:„  !$8EK$Ei)(+$8"%$"%(+9:H>$8$Y2,)*-=>!eQ9:6 ,76)OW%OT/,)*-/H>*v?>(?/1G>$8$YvM*(?!z;*$N
V

–oqRk n
/,%9:,SW)9:,'&;(f$xC%$EK&%/$
ƒ
;S$Y013!(o576Sp/9(+
3 ’
$
’
0?9:,-vY>(?/1+$E $,-2&F9+!+!e€N 09:,%1W;/$#+(+9:!+!_E $0eg1+$6)(]$?2/6)
 m
k J
H h
n J
H o
6 p
6)?+ˆ?+(+/1G>$EK$,7]&^9:?!>He N;#, $,d")*"%6)!2576Sp/9:(? #+9:,-2/$f/9:,%:6)$6%(+

kNRn o ƒ 2HJn†Rro k ƒ R  HL o†Rrk /n R ƒ  –R$


")$21 4>*]"ST6), >(?3,)/$WD1$.576)V19,)")6)H261jJ),):$EK$,72")$8M4(?3i)!$8

’ 2H  H  –k ’ ƒ HJn ƒ ’ HJo ƒ  [


… „u R ”‡ … ŠR u ‡ … – R$g‡ u  # [
1Npw%NT"SN '$

 … „R ”‡ … ŠR ‡ … –R$g‡ u   u D u > ƒ  # gH


qLp!,)*-4/!+*0p*1(+Hv/9:(?8

ƒ  ƒ gH k ƒ n ƒ oA[  
#(W;"Sp&)(?_]` 
UM:$1*:/H>*#?+ ;14Nw%NT"SN

Tk ƒ 2HLn ƒ  HJo ƒ [


  

  „R 2H  H  ŠR    O 


Œ „f#,&^9:?$
-
`0!9:(+ '+9,72!$2,):!$f"ST6), >(?3,)/$8!:6SWD$2OT/,)*-!!>*2T*1(+H

u
2k ƒ n ƒ o ƒ  [
1$.576)$OT9:,  "%* †+w'&)(+96%M:*|ƒg1Ge N;1jJ;&SN 4&)&)/!1U4+/9:,~‹:„GN
qV$(?*?6)!>4]$,d")*19:6)!$W[UM:$1*-/H>*#?+
+
 H k ƒ+ R ; + [
:  +
„amQ96)(f>9:6% ;9:, &F9:?$
+

 t k t H ƒ ’ k H * + k ƒ [
+
+ k ;
, 
`0!9:(+ )$
; :+  
; +
:  + … k ‡
;
+ ƒ ,  : ;  ’ + [
, 
+  k
`0!,)+L

 , 
#(W;"Sp&)(?_]`
+
k ƒ k *  +



    + k H  
k 
; ’ : ,
;UM:$1*-/H>*#?+V>96)]/$ ?9:,-v*-6DCFN
,
+
     
  
, … ; ’ k ‡   ,  k  [ [ [  , :   k 
3 09:,)1  +   4 :  
9 ‘ 6 
  J"($(
)P"  

4:   :+   k +:  
ƒ : ,  +
4:   +  k [
:+  
ƒ ,+
:  + … k ‡
+ ƒ ,  : ;  ’ + 4 :   H
, 
+  k
PRv/,)?

, 
UM:$1*-/H>*#?+S+9:6)]/$ kM ƒ k”  ƒ [ [ [ ƒ k;:   ƒ  R ; H
1Npw%NT"SN   ƒ  ƒ [ [ [ ƒ -:   ƒ [

7k 2HJn ƒ  HLo ƒ g[ ;
 2H  H ƒ
  

„ #, &F9+$
f

 … 2H  H ”‡   g[
`0!9:(+ '?9:,-v!$],%:/$2"lp6), >(?3,%:/$0/:6lW)$]Žp!,)*-4/!+*0p*1(+H.
  

#(W)Žp6%,d"%$v,):!$f")6 >(?3,)/$Wn&;(]$xC%$EK&%/$ 2H
;$Y0?6)&^*(+!$6)(f96*-Vw  [
3 
TW   W H 

  [
u
09:,%1W;19EKE $83ke€9:,%1>!9:, $?21(?9:/?>,->$.?6)(  %9:,

: + ™ + 1 
: „f#,dd
: +

* + R * + /21
ƒ  u + 1
*+
,  : , +
 *+  … s
k r
R ‰
n ‡   ”k>n
,  + 1 :
u * + /21
ƒg1( G„ƒ „

* +0/21 * 

ƒ  + 1 + ,  1 
 * + /21 <  R  =
 ƒŽ‹:„

+
˜ ƒ [ [  [ ƒ -: ƒ g[ 3 u
’ u   ƒ
#,d *-/H>*v";,)ƒO‹„f?+ˆ6dE 9:/,) " $
% ?9:,-2,n6%/N[mQ(]$GCD$E &)!$W

 R  H   ƒ h[
#,%v1$]19:,%")!+/9:,)Wˆƒ o„ $?26),%$*-/H>*v++
)14Nw%NT"SN
K ƒ ; H  +
!,;/$EK$,-UW
")$6DCd6%+(+$v?9:,-v*-6DCFN 
[UM:$1*-!!>*0++ˆ46E 9:/,%
’ u
")$
+9:,-2,76)/W;$2/$

' j lkmHJnhHJo  Tk>n1o ƒ [


„ 9/$,- +$/f576)$
;
9%

`0!9:(+W)OT6), ")$.1$v,)9Ei)(+$]$?2?6)&F*(+!$6)(]9:6 *:ˆw W)$26),6%+(+$.$Y


N 5o9:E E $8OT/,)*-/H>*vw'&)(+9:6DM:$(2$Y21cn1!/576)$W;9,&^$6%
7kO lo  [
/,%e€*(+!$6)(f9:6 *-Vw

Š; k>n1o ƒ H k n o
")9:,)1.?6)&)&^9:+$(v576)$ $
`0&)(?_2")*M$!9:&)&^$E $,-UW;$Gv19:E EK$ ;Žp!,)*:/H>*2")*?/(+*$.$?
;
*576)HM4/$,-+$w~ ksRrnqRoqRrk>nqRn‰o†Rro‰kO; lR o R k R n H
… k ‡  o;  R … n ‡  k;  R … o ‡  n;   [
’ ; –n ’ ; H ’ ; ’ ; +’ k  ; oA[ ’ ;
1Npw%NT"SN

Mk  … k>; o kC‡ … o ‡R … k>o ‡  … k>oqR kC‡  


#,~(+$E &)/1$ 2&)( n$a9:, E6)H>!&)/!$a>9:6%o&;( ;#, 9i%>!$,-a6),%$#/,)*-!!>*

; ’ ’ ; ’ ; k>o ; ’ bk o
*576)HM4/$,-+$ 
3#,)21$G++$"%$(+,%/_(?$./,)*-/H>*4WD1jJ)576)$.eg1G>$6%(2$?21//(?$E $,-f&^9:+H>HeQ96d,76)ŽN

1Npw%NT"SN kƒ oƒ nƒ [
qˆ19:,)1/6)?/9:, $,d")*19:6)!$W[UM:$1*-!!>*0++ $Gƒ
ƒ ; ƒ ; ƒ ;
9:6 „GW

9 ;
 t kOQnNOQo –k>n1o ƒ [ kO OQo –k>nsO [
„ qLp!,)*:/H>*#w&)(?9:6%M:$(]$?vJ)9:E 9::_,)$8$2YcDE *+(+!5n6%$W;9:, &^$6%2")9:,%1
a
/E &F9+$( 0$ )`v/9:(? 0$
3
k  n 
#,)21$]19:,)"%!>!9:,)W)/$8E $Ei%(+$8")$.-6%1jJ)$.$?

 … kmHJnhHJo'‡ ƒ k  R R n  R R o  R
;
o  ; ;

k˜ ˜ R ˜ n ˜ R ˜ o ˜
  


ƒ k k ˜R H  n ˜ H n R o  ˜ [ o R 
 #  ƒ ƒ H  t ƒ #_O ƒ O gH  O _O g[
 ,&^9:+$
#
`0!9:(+ ;$G

`0!,)+  … k HLnAHLo6; ‡ ƒ … 2H  H e; ‡ ƒ R  R NR   R –R  




–R  t H
5
o9:E EK$89:,d'1//(?$E $,-
 ;/F+6%‡v")$.&)(+9:6DM:$(

R  [ ;
576)$
R  /R rt
; R  R #
#(f1$1ˆ$?2*5n6%!M4/$,-0w
R   /R  u … R  ‡ … ŠR  ‡ t
#  # ;
1Npw%NT"SN
u … R  ‡ … /R  ‡ t … R  ‡’ … N R  ‡
# … R  ‡ … ŠR  ‡ 
 u  H %    #  H t ’
9:6d$,)19:(+$ ƒ „ #
5
> # … R  ‡ … Š; R  ‡ t <  ’ # =
o9:E EK$ )9:, )$kƒ „o$?2*5n6%!M4/$,->$w~

’
ƒO‹:„xN
9 ‹ 6 
  J"($(
)P"  

8 $ „R _O u  ƒ #u 
… R ‡ … ŠR  ‡ ƒ > #  < #ŠR   R  … „R$g‡ =
#(W ")9:,)1

> # 
O > # #/R   R #u  
… M ‡  R > #   R 
ƒO –$R   R    
#

…  R ‡
<  # =  <;    =   [
 ƒ
<  ’ ; = … -R ‡  H
3
’
kp6%+(+$.&;(Y

… -R ‡ …  ”‡ [ ’ ;gH


V$Y]eg1!/$#")$8M*(+Hz;$(]576)$ ;T*1(?!v6)?+

’
DmL6)!+576)$ 79:, $,|")*")6)Hoeg1//$EK$,-vƒO‹:„xWn$Ga/19:,)1!6D@
;
-k ƒ o ;  n ƒ oAH 
  … kmHJnhHJo'‡ ƒ u H
+!9:, ")*?/(?*$.$,d")*19:6)!$N
;
#, &^$6%',%9>$(k576)$ &^9:6)( $ {9:,  Q1$ 576)

& $(+E $"S=‡K(?EK$(576)$~/$|EK$Ei)(+$|")$~")(+9!>$|,)$ &^$6%bG>(+$~(?$E &)341* &;4(6),)$
^
M4/$6)(f&)!6)f&F$G>H>$N
 

 
 

!#"%$'&)(+*'-,./102323465798:8;$(+<8)4&='(+>@?BA3CED'F#GIHKJLHMAONOPQR#SUT'"306$'V)5XWZY[0\)5"3]302^&
<_"35E]%]E
-`acbdfegfhi02j"%06Y[8k\:06lm(on!?gF#GIpmJqHMrZ>s?BA3CED[F#GIHKJqHMAON@(oe;t["%06Y[V#5E"E
uvc<-ew-x9y[464z5EY(+bfew-hi02j"30zY[8k\:06l#([<-h{}|~m]306lm(B{A3F#?'N€F#?g‚J„ƒ+A@HK…Z>s?BA3CED[F#GIHKJqHMAON@(
† 46y=7‡5"E
}ˆmc<x‡8)"3Y']3‰235E0zY(ŠoDa‹+A@…sŒF#JKƒ=PB|Žy'06Tg5E"%2E
-ac<x‡8)"3Y']3‰235E0zY(B{M‘=F#ŒF#JKƒ=Pw|Žy[06Tg5E"’2“
”v1•–-x!-e;8)y'4—#S0M(+˜1AONš™vGIpmŒ!‹gHqF=:AONX:A€ŒF#J„ƒ+@ŒF#JqHqCEDoAONOPB,_464z06t[]%5E]
-›a1œd,ŽY'V)5E4L(w9…O™#žEG6A@ŒšŸMNk™vGI kHK?:‘UN@JL…%F#J^AM‘#HMAONOP1e:t["30zY[V)5E"
¡vc,¢/8)‰“mY[]3£=&Z¤BQ!¥¢8)y[]3]%5“#yB(!¦§HK?+?+HK?:‘Nk™vGID:JLHM™v?'NOPe;t["%06Y[V#5E"E

¡)u

You might also like